Download as pdf or txt
Download as pdf or txt
You are on page 1of 858

Bank Po Mains PDF Course 2024

Reasoning Day -1 (Eng)

Reasoning Aptitude

Directions (1-5): Study the following information 2) How V is related to the one who sits
carefully and answer the given questions. immediate left of S?
Eight persons – P, Q, R, S, T, U, V and W from a a) Nephew
three generations family are sitting around a b) Brother-in-law
concentric square table in such a way that four c) Son
persons are sitting in the middle of the sides of d) Daughter
the inner table facing away from the centre e) None of these
whereas four persons are sitting in the middle of
the sides of the outer table facing towards the 3) _____ sits immediate right of ______, who is
centre. the _____ of Q respectively.
Note: If the persons are facing each other, then a) U, P, Granddaughter
both are sitting at different tables whereas if the b) V, W, Wife
persons are sitting opposite to each other, then c) R, T, Sister-in-law
both are sitting at the same table. Only two d) Both A and B
married couples are there in the family, either e) None of these
both or none of the parents are alive.
P is facing the one who sits immediate right of 4) _____ male persons sit along with _____ at the
U’s sister-in-law. One person sits between U’s same table.
sister-in-law and V, who is the unmarried son of a) One, W
W. V is an immediate neighbour of his mother, b) Two, T
who is facing away from the centre. T is the c) Three, U
brother-in-law of Q and vice-versa. Q is the d) Two, S
father of S and faces W’s only brother. S is the e) None of these
only sister of P’s father. One person sits between
T and U’s daughter. R is not facing V’s sister. 5) Which of the following statement(s) is/are not
1) What is the position of R with respect to his true as per the given arrangement?
wife? I. T is the uncle of P
a) Immediate left II. Two persons sit between Q and T’s sister
b) Second to the right III. R is facing his brother
c) Facing each other a) Only III
d) Immediate right b) Only I and II
e) None of these c) Only I and III

Click Here For Bundle PDF Course | support@guidely.in Page 1 of 10


Bank Po Mains PDF Course 2024
Reasoning Day -1 (Eng)

d) All I, II, and III 6) Who among the following person watched the
e) Only II match immediately after and immediately before
the one whose jersey number is 18 in week 1
Directions (6-10): Study the following information and week 2 respectively?
carefully and answer the given questions. a) R and P
Seven persons – P, Q, R, S, T, U and V watched b) P and S
the cricket match for two weeks starting from c) S and V
Sunday to Saturday. Each person wears a jersey d) S and R
which has different number between 5 - 20. e) None of these
For week one:
At most two persons watched the cricket match 7) The number of persons watched the match
after S, whose jersey number is a perfect square. before V in week 2 is the same as _______ in
The number of persons watched the match after week 1.
S is one more than the number of persons a) The number of persons watched the match
watched the match before T. T’s jersey number between R and S
is twice the jersey number of R. R’s jersey b) The number of persons watched the match
number is a prime number. V watched the match before Q
three days before the one whose jersey number c) The number of persons watched the match
is 7. As many persons watched the match after U
between V and T as between V and the one d) Both A and C
whose jersey number is 12. Q’s jersey number is e) Both B and C
19. U’s jersey number is three less than the
jersey number of P, who watched the match two 8) _____ persons watched the match between
persons after Q. V’s jersey number is twice the _____ and ____ in week 2.
jersey number of the one who watched the match a) Three, Q and the one whose jersey number is
immediately before U. 12
For Week Two: b) One, R, Q
All the persons whose jersey numbers are odd c) Two, P, the one whose jersey number is 18
went to watch the match in ascending order of d) Both A and C
their jersey number on alternate days starting e) Both A and B
from Sunday. The remaining persons went to
watch the match in reverse alphabetical order 9) What is the sum of the jersey number of S and
from Monday one after another. Q?

Click Here For Bundle PDF Course | support@guidely.in Page 2 of 10


Bank Po Mains PDF Course 2024
Reasoning Day -1 (Eng)

a) 21 U sits third to the right of T and vice versa. T sits


b) 32 second from one of the extreme ends. The one
c) 28 who sits opposite to T sits fourth to the left of F.
d) 25 Neither G nor D sits adjacent to F. G sits third to
e) None of these the right of D, who is facing south. The number of
persons sitting between F and D is the same as
10) Which of the following statements is/are not the number of persons sitting to the left of R. A
true as per the given arrangement? sits opposite to the one who sits immediate right
I. T watched the match on Monday in week 1 of Q. E and R are facing in the opposite direction.
II. Only two persons watched the match between As many persons sit between Q and R as
S and U in week 2 between A and B, who sits immediate left of E.
III. R watched the immediately after V in week 2 Neither T nor V sits adjacent to Q. C sits
a) Only III opposite to S and sits to the left of A. P neither
b) Only II and III sits adjacent to T nor faces the same direction as
c) Only I and III A. Both V and C are facing the same direction as
d) All I, II, and III P, who faces the opposite direction as G.
e) Only I 11) Four of the following five are alike in a certain
way based on the given arrangement and thus
Directions (11-15): Study the following form a group. Which one of the following does
information carefully and answer the given not belong to the group?
questions. a) The one who sits immediate left of Q
Fourteen persons are sitting at two parallel rows b) C
containing seven persons each in such a way c) The one who sits second to the right of E
that there is an equal distance between adjacent d) V
persons. Row 2 is north of Row 1. In row 2 P, Q, e) The one who sits immediate right of B
R, S, T, U, and V are seated and some are
facing north whereas some are facing south. In 12) _____ persons are sitting to the right of
row 1 A, B, C, D, E, F, and G are seated and _____.
some are facing north whereas some are facing a) Three, G
south. Not more than two adjacent persons are b) Two, Q
facing in the same direction. Each person in row c) Four, E
1 sits exactly opposite to one person in row 2. d) Both A and C
e) Both B and C

Click Here For Bundle PDF Course | support@guidely.in Page 3 of 10


Bank Po Mains PDF Course 2024
Reasoning Day -1 (Eng)

13) _____ sits opposite to _____ who sits _____ to machine rearranges each letter step by step to
the right of E respectively. process the final output.
a) A, P, Fourth
b) Q, F, Third
c) P, D, Immediate
d) S, C, Second
e) Both A and C

14) What is the position of P with respect to the


one who sits opposite to D?
a) Second to the right
16) What comes in place of ____(A)___.
b) Fourth to the left
a) X8U
c) Immediate left
b) Z9C
d) Third to the right
c) Z8V
e) None of these
d) X9V
e) None of these
15) Which of the following statements is/are not
true as per the given arrangement?
17) What comes in place of ____(D)___
I. F sits immediate right of C
a) L7V
II. Only two persons sit between V and the one
b) J10U
who sits opposite to F
c) L5T
III. R sits immediate left of T
d) J5V
a) Only III
e) Either B or C
b) Only II and III
c) Only I and III
18) What comes in place of ____(C)___.
d) Only II
a) V4U
e) All I, II, and III
b) D9K
c) B6L
Directions (16-20): Study the following
d) C4F
information carefully and answer the given
e) None of these
questions.
Some letters and numbers are given in square
19) What comes in place of ____(B)___.
boxes. A number and letter rearrangement
a) J7T

Click Here For Bundle PDF Course | support@guidely.in Page 4 of 10


Bank Po Mains PDF Course 2024
Reasoning Day -1 (Eng)

b) G7J a) 59
c) F9K b) 61
d) F8J c) 64
e) None of these d) 48
e) None of these
20) What is the sum of all the numbers in step
III?
Click Here to Get the Detailed Video Solution for the above given Questions
Or Scan the QR Code to Get the Detailed Video Solutions

Answer Key with Explanation


Direction (1-5):  One person sits between U’s sister-in-law
1. Answer: B and V, who is the unmarried son of W.
2. Answer: C  V is an immediate neighbour of his
3. Answer: D mother, who is facing away from the
4. Answer: D centre.
5. Answer: B That means, in case (1) V sits immediate
left of his mother, in case (2) V sits
immediate right of his mother.
Based on the above given information we have:

We have:
 P is facing the one who sits immediate
right of U’s sister-in-law.

Click Here For Bundle PDF Course | support@guidely.in Page 5 of 10


Bank Po Mains PDF Course 2024
Reasoning Day -1 (Eng)

Again, we have:
 T is the brother-in-law of Q and vice-
versa.
 Q is the father of S and faces W’s only
brother.
 S is the only sister of P’s father. Case (1) is not valid as one person sits between
Since, only two married couples are there P and T, who is facing Q.
in the family, also either both or none of For Blood Relation:
the parents are alive, thus Q must be
grandfather of P.
That means, V must be the son of Q, now
as V is unmarried, thus V is not the father
of P.
As only either members are there in the
family, U must be sister-in-law of S.
Based on the above given information we have: Directions (6-10):
6. Answer: D
7. Answer: D
8. Answer: C
9. Answer: C
10. Answer: B

Again, we have:
 One person sits between T and U’s
daughter.
Since, only eight persons are there in the
family, thus P must only daughter of U.
That means, in case (2) T sits facing Q,
case (1) is not valid.
 R is not facing V’s sister.
That means, U is facing S, who is the
sister of V.
Based on the above given information we have We have:

the final arrangement:

Click Here For Bundle PDF Course | support@guidely.in Page 6 of 10


Bank Po Mains PDF Course 2024
Reasoning Day -1 (Eng)

 At most two persons watched the cricket  As many persons watched the match
match after S, whose jersey number is a between V and T as between V and the
perfect square. one whose jersey number is 12.
 The number of persons watched the That means, in case (1) V watched the
match after S is one more than the match on Monday, in case (2) V watched
number of persons watched the match the match on Wednesday, in case (3) V
before T. watched the match on Tuesday.
 T’s jersey number is twice the jersey Based on the above given information we have:
number of R.
 R’s jersey number is a prime number.
Since, only possible prime number jersey
are 7, 11, 13, 17, 19.
But jersey number of any person can’t be
more than 19, thus only possible jersey
number of R is 7 and jersey number of T
is 14.
That means, in case (1) S watched the
match on Friday, in case (2) S watched
Again, we have:
the match on Thursday.
 U’s jersey number is three less than the
Based on the above given information we have:
jersey number of P, who watched the
match two persons after Q.
 Q’s jersey number is 19.
That means, in case (2) Q watched the
match on Sunday, case (1) & case (1a)
are not valid.
 V’s jersey number is twice the jersey
number of the one who watched the
match immediately before U.
That means, S’s jersey number is 9.
Based on the above given information we have:
Again, we have:
 V watched the match three days before
the one whose jersey number is 7.

Click Here For Bundle PDF Course | support@guidely.in Page 7 of 10


Bank Po Mains PDF Course 2024
Reasoning Day -1 (Eng)

12. Answer: D
13. Answer: B
14. Answer: D
15. Answer: E

Case (1) & case (2a) are not valid as no day We have:
available to place Q.  U sits third to the right of T and vice
For Week two: versa.
All the persons whose jersey numbers are odd  T sits second from one of the extreme
went to watch the match in ascending order of ends.
their jersey number on alternate days starting  The one who sits opposite to T sits fourth
from Sunday. The remaining persons went to to the left of F.
watch the match in reverse alphabetical order Based on the above given information we have:
from Monday one after another.
Based on the above given information we have:

Again, we have:
 Neither G nor D sits adjacent to F.
 G sits third to the right of D, who is facing
south.
 The number of persons sitting between F
and D is the same as the number of
persons sitting to the left of R.
That means, in case (1) R sits immediate
left of U, in case (2) R sits second to the
Directions (11-15): right of U.
11. Answer: C( all the given persons are sitting Based on the above given information we have:
at the end of the row except option C)

Click Here For Bundle PDF Course | support@guidely.in Page 8 of 10


Bank Po Mains PDF Course 2024
Reasoning Day -1 (Eng)

Based on the above given information we have:

Again, we have:
 A sits opposite to the one who sits
immediate right of Q. Directions (16-20):
 As many persons sit between Q and R as 16. Answer: A
between A and B, who sits immediate left 17. Answer: C
of E. 18. Answer: D
 E and R are facing in the opposite 19. Answer: B
direction. 20. Answer: A
 Neither T nor V sits adjacent to Q. We have:
That means, in case (2) B sits immediate For step I:
right of D, case (1) is not valid. Letters are changed to their corresponding
Based on the above given information we have: reverse letter as per the alphabetical series,
whereas if the number is odd then, add ‘1’ else if
the number is even subtract ‘1’ from it.
For step II:
Letters in the diagonally opposite boxes are
interchanged and those letters are changed to
Case (1) is not valid as E and R not facing the
the next letter as per the English alphabetical
same direction.
series, whereas the numbers in the opposite
Again, we have:
boxes are interchanged, except in corner boxes.
 C sits opposite to S and sits to the left of
For step III:
A.
Letters at the corner boxes are changed to the
 P neither sits adjacent to T nor faces the
opposite letter as per the English alphabetical
same direction as A.
series, and numbers from diagonal boxes are
That means, in case (2) P sits immediate
interchanged.
left of R.
Also, Letters from the middle boxes are changed
 Both V and C are facing the same
to the third successive letter as per the English
direction as P, who faces the opposite
direction as G.

Click Here For Bundle PDF Course | support@guidely.in Page 9 of 10


Bank Po Mains PDF Course 2024
Reasoning Day -1 (Eng)

alphabetical series, whereas ‘2’ is added to the


number.
Based on the above given step by step rule we
have the final arrangement as follows:

Click Here For Bundle PDF Course | support@guidely.in Page 10 of 10


Bank Po Mains PDF Course 2024
Quantitative Aptitude Day -1 (Eng)

Quantitative Aptitude

1) Ratio of curved surface area to total surface Find the original discount given by the
area of a right circular cylinder is 3: 4 and the shopkeeper?
difference between its height and radius is 8 cm. a) ₹ 2160
If the height of a right circular cone is 3 cm more b) ₹ 1840
than that of the cylinder and the radius of the c) ₹ 2000
cone is 2 cm more than that of the cylinder, then d) ₹ 2250
find the ratio of the volume of the cylinder to that e) ₹ 2080
of the cone?
a) 37: 36 4) Two boats A and B are sailing in a river in
b) 16: 15 downstream direction and the speed of the
c)19: 18 stream is 2 m/s. Boat B is 102 m ahead of boat A
d) 25: 24 and ratio of speed in still water of boat A to boat
e) 31: 30 B is 6: 5. If boat A crosses boat B in 34 seconds,
then find the total downstream distance covered
2) Compound interest received on ₹ ‘x’ after 2 by boat A?
years at R% rate of interest compounded a) 640 m
annually, is ₹ 1680 more than that received on b) 510 m
₹ (x – 5000) after 1 year at R% rate of interest c) 680 m
compounded half-yearly. If the compound d) 560 m
interest received on ₹ (x – 3000) at 15% rate e) 600 m
after 2 years is ₹ 3870, then find the value of R?
a) 8% 5) When A and B work with their original
b) 12% efficiencies, they together can complete the work
c)10% in ‘16t’ hours. When A and B work with 80% and
d) 6% 160% of their respective efficiencies, they
e) 15% together can complete the work in ‘15t’ hours. If
A alone can complete the work in 12 hours and
3) A shopkeeper marked up an article by ₹ 4000 C alone can complete the work in 6 hours less
above the cost price and sold it at 20% discount than that of B, then in what time A and C
on its marked price. If he had given 25% together will complete the work?
discount on its marked price, his profit would a) 8.4 hours
have been ₹ 520 less than the original profit. b) 6.0 hours

Click Here For Bundle PDF Course | support@guidely.in Page 1 of 14


Bank Po Mains PDF Course 2024
Quantitative Aptitude Day -1 (Eng)

c)7.2 hours e) 9.6 hours


d) 4.8 hours

Direction (6-10): Study the following data carefully and answer the questions:
Data given below is related to number of boys and girls in a school, who play 5 different sports
Basketball, Football, Tennis, Cricket and Badminton.
The pie chart given below shows the percentage distribution of the number of boys, who play the given 5
sports.

Pie chart given below shows the percentage distribution of the number of girls, who play the given 5
sports.

Click Here For Bundle PDF Course | support@guidely.in Page 2 of 14


Bank Po Mains PDF Course 2024
Quantitative Aptitude Day -1 (Eng)

Note:
1: Difference between the number of boys, who play basketball and those, who play football, is 80.
2: 36% of the total number of students, who play football, are girls.
6) Total number of students, who play basketball, 8) Which of the following is/are true?
is approximately what per cent of the total A: Difference between number of boys and girls,
number of students, who play badminton? who play cricket, is 185.
a) 55.6% B: Total number of girls in the school, is 400.
b) 65.6% C: Ratio of boys to girls, who play badminton, is
c) 59.6% 8: 11.
d) 63.6% a) All are true
e) 57.6% b) Only A and B
c)Only B and C
7) Find the ratio of total number of boys, who d) None is true
play football, tennis and cricket to the total e) Only A and C
number of girls, who play basketball, football and
tennis? 9) What per cent of total number of students who
a) 25: 18 play badminton in the school?
b) 20: 9 a) 24.4%
c)60: 29 b) 16.4%
d) 3: 1 c)22.4%
e) 15: 8 d) 18.4%

Click Here For Bundle PDF Course | support@guidely.in Page 3 of 14


Bank Po Mains PDF Course 2024
Quantitative Aptitude Day -1 (Eng)

e) 20.4% a) 259
b) 229
10) Find the average of the total number of c) 239
students, who play basketball, those who play d) 219
football and those who play cricket? e) 249

Direction (11-15): Study the following data carefully and answer the questions:
Table given below shows the ratio of male to female employees in five different companies. Average
salary of each male and female employee is different in every company. Table also shows the ratio of
total salary of male to female employees.

Note:
1. The total employees in companies P and Q are 100 each while total male employees in companies R
and S are 80 and 75 respectively. Male employees in company T are 10 more than female employees.
2. Difference between the average salaries of male and female employees of companies P, Q, R, S, and
T is ₹ 500, ₹ 1600, ₹ 700, ₹ 400, and ₹ 1000 respectively.
11) What is the average salary of all the c) I for both
employees (male and female together) of d) I and III
company Q and S respectively? e) II for both
I: ₹ 2800
II: ₹ 3250 12) If 20 more employees joined the company T
III: ₹ 3850 whose average salary is ₹ 4200, then find the
a) I and II average salary of all the employees of company
b) II and III T.

Click Here For Bundle PDF Course | support@guidely.in Page 4 of 14


Bank Po Mains PDF Course 2024
Quantitative Aptitude Day -1 (Eng)

a) ₹ 4400 e) Only I and II


b) ₹ 4200
c) ₹ 4500 14) Find the ratio of the difference between the
d) ₹ 4100 total salary of male and female employees of
e) ₹ 4600 company P to the average of total salary of male
and female employees of company R.
13) If ‘x’ male employees and ‘x + 10’ female a) 6: 11
employees joined the company Q such that ratio b) 5: 13
of male to female employees becomes 17: 9, c) 4: 9
then which of the following can be determined. d) 7: 16
I: Average salary of male employees in the e) 5: 16
company after the joining of new employees.
II: Total number of employees in the company 15) If the average of the total salary of female
after the joining of new employees. employees of all the five companies together is
III: Ratio of newly joined male to female ₹ 355P, then the value of ‘P’ lies between:
employees in the company. a) 350 and 380
a) Only II b) 380 and 410
b) Only I c) 320 and 350
c) Only II and III d) 410 and 440
d) Only I and III e) 440 and 470

Direction (16-20): Study the following data carefully and answer the questions:
Some questions are assigned to a freelancer for creation in five different months, out of which he created
some questions while the remaining are yet to be created. Out of the total created questions, some are
easy, and the remaining are hard difficulty levels.
The line graph given below shows the difference between the number of created questions and the
number of questions that are yet to be created. It also shows the difference between the number of
easy-level and hard level questions created.

Click Here For Bundle PDF Course | support@guidely.in Page 5 of 14


Bank Po Mains PDF Course 2024
Quantitative Aptitude Day -1 (Eng)

Table given below shows the number of questions created as percent of total assigned questions to him.

Note:
1. x2 + y2 = 21250
2. In each month he created the majority of the assigned questions to him and also, he created easy-level
questions more than hard-level questions.
3. The observation took place somewhere in the mid of a month and questions that are yet to be created
in that month will be created later in that month only.

Click Here For Bundle PDF Course | support@guidely.in Page 6 of 14


Bank Po Mains PDF Course 2024
Quantitative Aptitude Day -1 (Eng)

16)If the freelancer is paid ₹ 40 per question for a) 300


each easy level and the average of total amount b) 350
received by him for creating easy level questions c)200
in all five months together is ‘80P’, then which of d) 250
the following can be a factor of ‘P’? e) 400
a) 15
b) 17 19) If the ratio of the total amount received by the
c)19 freelancer for creating easy level to hard level
d) 21 questions in Apr is 8: 5, then find the ratio of pay
e) 23 for each easy level to each hard level question to
the freelancer in that month.
17)Out of total questions yet to be created in a) 8: 15
Jan, number of easy and hard level questions b) 7: 17
are ____ and ____ respectively. If he is paid ₹ 30 c)4: 9
for easy level questions and ₹ 50 for hard level d) 5: 9
questions and will be paid ₹ 5600 for all the e) 6: 11
questions which are yet to be created in that
month, then which of the following value will fill 20)Total questions created by the freelancer in
the blanks in the same order? all five months together when the observation is
a)90 and 60 made is what percent of the total number of
b)95 and 55 questions assigned to him in all the five months
c)105 and 45 together?
d) 80 and 70 a) 60%
e) 100 and 50 b) 65%
c)70%
18) What is the difference between the total d) 75%
number of easy-level questions created and the e) 80%
total number of hard-level questions by the
freelancer in Jan, Mar, and May together?

Click Here For Bundle PDF Course | support@guidely.in Page 7 of 14


Bank Po Mains PDF Course 2024
Quantitative Aptitude Day -1 (Eng)

Click Here to Get the Detailed Video Solution for the above given Questions
Or Scan the QR Code to Get the Detailed Video Solutions

Answer Key with Explanation


1) Answer: B 529x – 1587000 – 400x + 1200000 = 1548000
Let the height and the radius of the cylinder are x = 15000
‘h’ cm and ‘r’ cm respectively. Since, the CI received on ₹ ’x’ after 2 years at
So, R% rate of interest compounded annually, is
₹ 1680 more than that received on ₹ (x – 5000)
after 1 year at R% rate of interest compounded
4h =3h + 3r
half-yearly.
h: r = 3: 1
So,
The height of the cylinder = = 12 cm
The radius of the cylinder = 12 – 8 = 4 cm
The height of the cone = 12 + 3 = 15 cm R = 8%
And the radius of the cone = 4 + 2 = 6 cm 3) Answer: E
Now, the volume of the cylinder = Πr2h = 192Π Let the CP of the article = ₹ ‘x’
cm3 So, the MP of the article = ₹ (x + 4000)
And the volume of the cone = (1/3)Πr2h = 180Π The SP of the article after 20% discount on its
cm3 marked price:
Required ratio = 192Π: 180Π = 16: 15 80% of (x + 4000) =
And the SP of the article after 25% discount on
2) Answer: A its marked price:
Since, the CI received on ₹ (x – 3000) at 15% 75% of (x + 4000) =
rate after 2 years is ₹ 3870. So,
So,

16x + 64000 – 15x – 60000 = 10400

Click Here For Bundle PDF Course | support@guidely.in Page 8 of 14


Bank Po Mains PDF Course 2024
Quantitative Aptitude Day -1 (Eng)

x = 6400 So,
Now, the MP of the article = 6400 + 4000 =
₹ 10400 And,
And the original discount given by the
shopkeeper = 20% of 10400 = ₹ 2080
By equation (1) * (1/5) – equation (2) * (1/8):

4) Answer: C
Let the speeds of boats A and B in still water are
‘6x’ m/s and ‘5x’ m/s respectively. t = 0.5
So, the downstream speed of boat A = (6x + 2) From equation (1):
m/s
And the downstream speed of boat B = (5x + 2) 8b = 24
m/s Now, the time, in which B alone can complete
Let the downstream distance covered by boat B the work = 24 hours
= ‘y’ m The time, in which C alone can complete the
So, the downstream distance covered by boat A work = 24 – 6 = 18 hours
= (102 + y) m Part of work complete by A and C together in 1
Now, hour =
So, the time, in which A and C together will
And, complete the work = 7.2 hours

From equations (1) and (2): Direction (6-10):

102 + 170x + 68 = 204x + 68 From the 1st pie chart:

x=3 2X + Y = 100 – (10 + 25 + 15)

From equation (2): 2X + Y = 50 -----------(1)


From the 2nd pie chart:

y = 578 m
So, the downstream distance covered by boat A 3X + 18 + Y = 78

= 102 + 578 = 680 m 3X + Y = 60 -----------(2)


From equations (1) and (2):

5) Answer: C X = 10, Y = 30

Let the time, in which B alone can complete the


work with his original efficiency = ‘8b’ hours

Click Here For Bundle PDF Course | support@guidely.in Page 9 of 14


Bank Po Mains PDF Course 2024
Quantitative Aptitude Day -1 (Eng)

Since, the difference between the number of Total number of boys, who play football, tennis,
boys, who play basketball and those, who play and cricket = 160 + 200 + 240 = 600
football, is 80. Total number of girls, who play basketball,
So, the number of boys, who play basketball = football, and tennis = 72 + 90 + 108 = 270
= 80 Required ratio = 600: 270 = 20: 9
The number of boys, who play football =
= 160 8) Answer: B
The number of boys, who play tennis = = From A:

200 Difference between number of boys and girls,

The number of boys, who play cricket = who play cricket = 240 – 45 = 195

= 240 So, A is not true.

And the number of boys, who play badminton = From B:

= 120 Total number of girls in the school = 72 + 90 +


108 + 45 + 135 = 450
Since, 36% of the total number of students, who
So, B is not true.
play football, are girls.
From C:
So, the number of girls, who play football =
Ratio of boys to girls, who play badminton = 120:
= 90
135 = 8: 9
The number of girls, who play basketball =
So, C is not true.
= 72
Hence, none is true.
The number of girls, who play tennis = =
108
9) Answer: E
The number of girls, who play cricket = =
Total number of students in the school = (80 +
45
160 + 200 + 240 + 120) + (72 + 90 + 108 + 45 +
And the number of girls, who play badminton =
135) = 1250
= 135
Total number of students, who play badminton =
6) Answer: C
120 + 135 = 255
Total number of students, who play basketball =
Required percentage = = 20.4%
80 + 72 = 152
Total number of students, who play badminton =
10) Answer: B
120 + 135 = 255
Total number of students, who play basketball =
Required average = = 59.6%
80 + 72 = 152

7) Answer: B

Click Here For Bundle PDF Course | support@guidely.in Page 10 of 14


Bank Po Mains PDF Course 2024
Quantitative Aptitude Day -1 (Eng)

Total number of students, who play football = Let total salary of male and female employees in
160 + 90 = 250 company S is 50s and 27s respectively.
Total number of students, who play cricket = 240 Let total salary of male and female employees in
+ 45 = 285 company T is t and t respectively.
Required average = = 229 Table given below shows the number of
employees (male, female, and total) in each
Direction (11-15): company and average salary of male and female
Total employees in company P = 100 employees.
Male employees in company P =
Female employees in company P = 100 – 60 =
40
Total employees in company Q = 100
Male employees in company Q =
Female employees in company Q = 100 – 75 =
25
Male employees in company R = 80
According to the question:
Female employees in company R =
Total employees in company R = 80 + 60 = 140
p = 10000
Male employees in company S = 75
Female employees in company S =
q = 20000
Total employees in company S = 75 + 45 = 120
Difference between male and female employees
in company T = 10 r = 56000

Male employees in company T =


Female employees in company T = 50 – 10 = 40 s = 6000
Total employees in company T = 50 + 40 = 90
Let total salary of male and female employees in t = 200000
company P is 15p and 8p respectively.
Let total salary of male and female employees in
company Q is 9q and 5q respectively.
Let total salary of male and female employees in
company R is 5r and 3r respectively.

Click Here For Bundle PDF Course | support@guidely.in Page 11 of 14


Bank Po Mains PDF Course 2024
Quantitative Aptitude Day -1 (Eng)

675 + 9x = 595 + 17x


8x = 80
x = 10
I: We cannot determine the average salary of
newly joined male employees as we are not
given the total salary of male employees.
II: Total number of employees in the company
after the joining of new employees = 100 + x + (x
11) Answer: D + 10) = 130
Total salary of all the employees of company Q = III: Ratio of newly joined male to female
180000 + 100000 = ₹ 280000 employees = x: (x + 10) = 10: 20 = 1: 2
Total employees in company Q = 100 Hence, only II and III can be determined.
Required average salary =
Total salary of all the employees of company S = 14) Answer: E
300000 + 162000 = ₹ 462000 Difference between total salary of male and
Total employees in company S = 120 female employees of company P = 150000 –
Required average salary = 80000 = ₹ 70000
Average of total salary of male and female
12) Answer: A employees of company R =
Total employees in company T = 90
Total salary of all 90 employees = 200000 + Required ratio = 70000: 224000
200000 = ₹ 400000 = 5: 16
New number of employees = 90 + 20 = 110
New total salary of all 110 employees = 400000 15) Answer: B
+ 20 * 4200 = ₹ 484000 Total salary of female employees of all the five
Required average = companies together = 80000 + 100000 + 168000
= ₹ 4400 + 162000 + 200000 = ₹ 710000
Average =
13) Answer: C 355P = 142000
Initial number of male and female employees in P = 400
company Q is 75 and 25 respectively. Hence, value of P lies between 380 and 410.
According to the question:
Direction (16-20):

Click Here For Bundle PDF Course | support@guidely.in Page 12 of 14


Bank Po Mains PDF Course 2024
Quantitative Aptitude Day -1 (Eng)

In the Apr, legends of the both the lines are 0.4x = 140
equal. Which means: x = 350
x + 75 = y + 125 Total created questions = x = 350
x – y = 50 Total yet to be created questions = x – 200 = 150
x = y + 50 ………. (1) Total assigned questions = 350 + 150 = 500
We are given: Total easy level created questions =
x2 + y2 = 21250
From equation (1): Total hard level created questions =
(y + 50)2 + y2 = 21250 Similarly, we can calculate for other months as
y2 + 2500 + 100y + y2 – 21250 = 0 well:
y2 + 50y – 9375 = 0
y = 75 and -125
We cannot take negative value of ‘y’.
Hence, y = 75 and x = 125
Find the table below that shows the simplified
data for the line graph given above:
16) Answer: E
Total easy level questions created by the
freelancer in all the five months together = 250 +
200 + 250 + 300 + 150 = ₹ 1150
Total amount received by him = 1150 * 40 =
₹ 46000
Average =
80P = 9200
P = 115
Since, he created majority of the assigned Factor of 115 = 1, 5, 23, and 115
questions which means created questions are Hence, 23 will be a factor of P.
more than yet to be created questions.
Let total number of created and yet to be created 17) Answer: B
questions in the Jan is ‘x’ and ‘x – 200’ Total questions which are yet to be created in
respectively. Jan = 150
According to the question: Let number of easy level and hard level
70% of [x + (x – 200)] = x questions which are yet to be created in Jan are
1.4x – 140 = x ‘x’ and ‘150 – x’ respectively.

Click Here For Bundle PDF Course | support@guidely.in Page 13 of 14


Bank Po Mains PDF Course 2024
Quantitative Aptitude Day -1 (Eng)

According to the question: 19) Answer: A


30x + 50(150 – x) = 5600 Let total amount received by the freelancer for
7500 – 20x = 5600 creating easy level and hard level questions in
20x = 1900 the month of Apr is 8x and 5x respectively.
x = 95 Total easy level questions created in Apr = 300
Number of easy level questions which are yet to Total hard level questions created in Apr = 100
be created in Jan = x = 95 Pay for each easy level questions in Apr =
Number of hard level questions which are yet to Pay for each hard level questions in Apr =
be created in Jan = 150 – x = 55
Hence, values that can fill the blanks are 95 and Required ratio =
55 respectively. = 8: 15

18) Answer: B 20) Answer: D


Total number of easy level questions created by Total questions created by the freelancer in all
the freelancer in Jan, Mar, and May together = the five months together when the observation is
250 + 250 + 150 = 650 made = 350 + 300 + 400 + 400 + 200 = 1650
Total number of hard level questions created by Total number of questions assigned to the
the freelancer in Jan, Mar, and May together = freelancer in all the five months together = 500 +
100 + 150 + 50 = 300 400 + 450 + 600 + 250 = 2200
Required difference = 650 – 300 Required percent =
= 350 = 75%

Click Here For Bundle PDF Course | support@guidely.in Page 14 of 14


Bank Po Mains PDF Course 2024
English Day - 1

English Language
Directions (1-7): The passage given below bank has never been tried earlier, though there
contains few words or phrases highlighted. Read are several private Asset Reconstruction
the given passage carefully and answer the Companies which are functioning sub-optimally.
questions that follow. There is blank that needs This is another major institutional reform and the
to be filled with the correct word from the options. Government must be given the credit for it.
India’s first-ever government-sponsored “Bad Whether it will work or not, only time can tell.
Bank” is now closer to reality. Following the While it has met with success in many countries,
Finance Minister’s announcement in the Union in China they have been struggling, and the coup
Budget earlier this year, a National Asset of Evergrande is likely to worsen the situation
Reconstruction Company (NARCL) has been set further. In any case, ARCs cannot be a
up to acquire fully provisioned stressed assets permanent mechanism, indeed it has a sunset
worth about Rs 2 lakh crore from various clause allowing it to be wound up within five to
commercial banks in different phases, about a eight years.
quarter of the total. Another entity called India Banking reform has been one major focus area
Debt Resolution Company Ltd (IDRCL) will then of the government which came up with a 4-R
sell these stressed assets in the market to pay strategy - recognition, resolution, recapitalization
back to the banks. Public Sector Banks (PSBs) and reforms. After recognition, quantification of
will have almost half the total shares of both (51 NPAs started in a planned manner and banks
and 49 percent respectively), in which the private slowly started recovering their dues. In 2018, just
sector will also be stakeholders. The initial two out of 21 public sector banks were profitable.
investment would be around Rs 6,000 crore. But in 2021, only two banks (Punjab & Sind Bank
NARCL will buy the ____________ assets at a and Central Bank) reported losses. As part of the
discount from the banks by paying 15 percent reforms, the government has merged many
cash; for the balance 85 percent, they will issue smaller PSBs with larger ones, and now their
security receipts like debentures, bonds, etc., to total number is stands at 12. There are several
the banks. problems, apart from finding potential buyers
Bad bank is not a new idea —they have been in from the stressed assets of banks. Valuation of
existence since the 1980s, but the East Asian the assets is one, which will determine the loss.
currency crisis and the 2008 subprime crisis In fact, there has been some criticism that the
made many countries to set up bad banks, government is actually guaranteeing the losses
including the US, Sweden, Finland, Belgium, of private enterprises and that the PSBs, being
Malaysia, South Korea, China or Indonesia, but shareholders of both NARCL and IDRCL, will in
in India the idea of government sponsored bad effect be buying their own stressed assets. So

Click Here For Bundle PDF Course | support@guidely.in Page 1 of 11


Bank Po Mains PDF Course 2024
English Day - 1

far, the Indian experience has preferred C. Infuse capital; save banks
resolution via IBC over the private ARCs, which D. Setting up ‘bad banks’ for the good
have a low capital requirement of only Rs 100 E. ARCs and the western banking reform
crore. influence
The regulatory environment for the ARCs which
has remained practically static ever since they 3) What impacted the banks’ lending capacity as
had emerged in the 2000s also needs an per the passage?
overhaul. Stressed assets will not disappear A. The RBI tightened controls on the banks
anytime soon, and we need to enable such B. The RBI forced the banks to clean their
institutions to help absorb the stressed assets. balance sheets
Economy cannot grow without an adequate C. The banks needed to write their NPAs off
supply of credit and creating a level playing field D. All of the above
for all. E. Not been discussed in the passage
1) What can be said about the state of the banks
between 2018 and 2021 as per the passage? 4) Pick the word that can replace the word ‘coup’
A. Due to the crony capitalism, banks were as used in the passage above
forced to lend even to projects that were unable A. Collapse
to materialize or generate enough income B. Insurrection
B. The government had to recapitalize them from C. Success
time to time with public funds to keep the banks D. Dispel
alive E. Fallacy
C. There was a complete shift in the profitability
of the banks as they could slowly recover 5) Why is the resolution framework facing
considerable amount of their debts criticism?
D. Their capital base was eroded making them A. The actual loss that is being generated by the
unable to lend afresh, and their profitability was NPAs is very difficult to calculate and that is one
seriously affected thing on which the entire process has been
E. Both (b) and (d) based
B. As the PSBs are shareholders in both the
2) Which of the following can be a suitable title entities being set up with the focus of cleaning
for the passage given above? the bad loans, it will indirectly mean that the
A. The future of the Banking Sector in India government is buying its own stressed assets
B. Banking and Scams: concomitants to each C. India prefers resolution viz the IBC because it
other has seen the same performing very well and it is

Click Here For Bundle PDF Course | support@guidely.in Page 2 of 11


Bank Po Mains PDF Course 2024
English Day - 1

always good to play safe in an area where one You are required to choose the appropriate word
has already been reeling under losses for each blank from the given options.
D. Both (a) and (b) 8) Science has been a powerful ____________for
E. All (a), (b) and (c) humanity’s progress. It has become
the____________ of human follies and also
6) A blank has been given in one of the lines theremedial tool that rescues us from
above. The blank would however require a word theirconsequences. Anti-microbial resistancehas
from the options that can meaningfully complete become a major _____________ to human
the sentence. Choose the best option as your survivaland well-being, as several multi-
answer drugresistant microbes have ____________
A. Intangible throughantibiotic misuse. The prospect of
B. Valuable findingpotent novel antibiotics ____________ dim
C. Distressed tillit was announced recently that
D. Fixed artificialintelligence (AI) helped to identify a
E. Both (b) and (d) newantibiotic (again) that is highly
effectiveagainst the pathogenic bacterium.
7) The author is most likely to agree with which I. Appeared
of the following points? II. Emerged
A. In India, Government-sponsored bad bank has III. Propellant
never been tried, though there are several asset IV. Threat
reconstruction companies functioning sub- V. Instrument
optimally A. V, IV, I, I, II.
B. Inherited as a legacy from the past, partly as a B. IV, III, I, I, II.
fallout of the economic meltdown of 2008 has C. III, V, IV, II, I.
forced the banks to compromise while clearing D. I, II, III, IV, V.
some projects for lending E. None of the combination is correct.
C. Since guarantees constitute contingent liability
for the government, there would be no immediate 9) One of the achievements of the NarendraModi
fiscal impact for the Centre government is the network of excellent highways
D. All (a), (b) and (c) across the country, which____________ matching
E. Only (b) and (c) driving skills, especially as car manufacturersare
developing more powerful engines and foreign
Directions (8-10): In the following questions, a car makersare bringing their best wares. That
passage has been given with multiple blanks. can make for a deadly combination.And drivers

Click Here For Bundle PDF Course | support@guidely.in Page 3 of 11


Bank Po Mains PDF Course 2024
English Day - 1

must _____________ that with great speedcomes andprocesses of the past. Thenew technological
great responsibility. The ______________ with revolutionin recent years is helpinga lot in the
strapping onseatbelts is of recent origin. While process by creatingnew types of classroomswith
those in the front seats do itwilly-nilly, backseat unrestricted access to all,_______________ the
passengers ______________ this safety measure. considerationslike region and distance.
Untilthe accident which claimed billionaire Cyrus I. Obsolete
Mistry in September 2022 on the Mumbai II. Lacks
Ahmedabad Expressway, the seatbeltdid not get III. Blurring
its due importance. Neither was it widely IV. Emphasis
knownthat airbags do not ___________ if the V. Reorientation
seatbelt is not strapped on. A. IV, II, III, V, I.
I. Compliance B. II, V, III, I, IV.
II. Deploy C. III, IV, V, III, I.
III. Require D. II, V, IV, I, III.
IV. Remember E. None of the combination is correct.
V. Ignore
A. III, IV, I, V, II. Direction (11-13): A sentence is divided into five
B. V, II, I, IV, III. parts. One of the parts does not belong to the
C. IV, III, I, II, V. given sentence. Remove the part that does not
D. II, III, V, IV, I. belong to the original statement and rearrange
E. None of the combination is correct. the remaining parts to make a sentence
complete.
10) Indian education system_____________ the 11) Ukraine of stalling negotiation talks (A)/ into
required flexibilityover the years, which is Ukrainian territories, Russian (B)/ President
verymuch required to take it tothe next level and Vladimir Putin accused (C)/ Putin can keep
make it truly global in approach andpracticality. pounding Ukraine for months (D)/ As Russia
In Budget 23,the government has focusedon the continued its offensive (E).
_______________ of educationand skills A. BDAE; Odd: C
according tothe aptitude of the youthand the B. CEDB; Odd: A
demands of thefuture.It is for the first time thatan C. ACDE; Odd: B
equal ______________ is beinggiven to both D. EBCA; Odd: D
education andskilling as part of the E. BDCA; Odd: E
neweducational policy, whichunburdens the 12) Following administration, the (A)/ when the
students fromthe ______________ regulations second dose of Covishield is given (B)/ genetic

Click Here For Bundle PDF Course | support@guidely.in Page 4 of 11


Bank Po Mains PDF Course 2024
English Day - 1

material of the part (C)/ of coronavirus is III. Despite having the largest English-
expressed which (D)/ stimulates an immune language readership in the world, children’s
response (E). literature in English from India has a meager
A. CEDB; Odd: A presence on the international stage.
B. BDAE; Odd: C A. Only II
C. ACDE; Odd: B B. Both I and II
D. EBCA; Odd: D C. Only III
E. BDCA; Odd: E D. Both II and III
E. All are correct
13) They should not come up with
recommendations (A)/ the qualifications for the 15)
job (B)/ there may be thousands of (C)/ have I. Had there been no tampering with the flowing
access to, but who do have (D)/ qualified people waters, our urban and rural settlements may not
who you do not (E). have suffered river pollution and potable water
A. CEDB; Odd: A crunch.
B. BDAE; Odd: C II. Indonesia and the Philippines lie along the
C. ACDE; Odd: B Pacific “Ring of Fire” — the seismic faults around
D. EBCA; Odd: D the Pacific Ocean which most of the world’s
E. BDCA; Odd: E earthquakes and volcanic eruptions occur.
III. Chile wants to extract more of the soft shining
Direction (14-17): In the given sentences I, II and metal called the lithium, meant to be essential for
III some may have errors in them. Choose the making batteries to run vehicles without
sentence(s) that have error/s in them. If there is petroleum.
no error in the given sentences choose ‘all are A. Only II
correct’. B. Both II and III
14) C. Only I
I. Today’s data shows that the UK and India are D. Both I and II
two of the world’s leading hubs for digital E. Only III
shopping companies with high levels of global
investment and unicorns. 16)
II.MS Dhoni is a fantastic leader and has I. The Chief Minister has been highlighting the
probably had the most success in terms of evils of drinking liquor along with dowry and child
silverware than any other captain in the world. marriage in his campaign.

Click Here For Bundle PDF Course | support@guidely.in Page 5 of 11


Bank Po Mains PDF Course 2024
English Day - 1

II. A 28 years old man, who was out on an 18) Vitriolic


interim bail, was shot dead allegedly by A.The media has launched a vitriolic campaign
unidentified assailants. against the politician.
III. A man has been arrested for harassing and B.Dipa’s vitriolic attitude could bring a smile on
stalking his female colleague by creating its fake everyone's face.
social media profiles and sending defamatory C.His vitriolic remarks really hurt our sentiments.
messages to her relatives and friends. D.The vitriolic behavior on part of the
A. Only I management was quite unexpected.
B. Only II A. Only A
C. Only III B. Only B
D. Both II and III C. Only C
E. All I, II and III D. Only D
E. All are correct
17)
I. As many23 drones are being used to locate 19) Philanthropic
illegal liquor manufacturing units and mafias. A. John is associated with a philanthropic
II. Thousands of homes and other buildings were organization.
damaged by the tremors that were felt as far B.SonuSood is known for his philanthropic work
away as Malaysia and Singapore. in our country.
III. The court asked the State governments to C. Raghu was a philanthropic person who
submit fresh status reports in three weeks and surrounded his house with barbed wires to keep
scheduled the next hearing after a month. out any visitor.
A. Only I D. Mandakini’s intentions were philanthropic
B. Both I and II rather than commercial.
C. Only III A. Only A
D. Both II and III B. Only B
E. All are correct C. Only C
D. Only D
Directions (18-20): Given below is a highlighted E. All are correct
word followed by 4 sentences. You need to
understand whether the sentences are using the 20) Negligent
word correctly or not. Mark the incorrect A. Piangshuk is quite punctual and negligent
sentence with the incorrect word usage as your when it comes to official works.
answer.

Click Here For Bundle PDF Course | support@guidely.in Page 6 of 11


Bank Po Mains PDF Course 2024
English Day - 1

B. I was utterly shocked to find out how negligent A. Only A


Arun had been during these years. B. Only B
C. Please do not be negligent in your work else C. Only C
you will suffer. D. Only D
D. Pulak seems quite negligent about his health. E. All are correct
Click Here to Get the Detailed Video Solution for the above given Questions
Or Scan the QR Code to Get the Detailed Video Solutions

Answer Key with Explanation


1) Answer: C recovery part of the matter. We can also find
On reading the third paragraph carefully you can reference of some other countries that have
easily eliminate the irrelevant options. Also, done similar things in the past during crises.
whenever there are years mentioned in the A complete idea about how the NARCL and
question, you can directly look for the data with IDRCL will help bolster the economy has been
those years in the given passage to quickly find given. So, the best option for a title that justifies
the answer. Refer to the lines below for this all of these points is (d).
question:
In 2018, just two out of 21 public sector banks 3) Answer: E
were profitable. But in 2021, only two banks The passage discusses the government-
(Punjab & Sind Bank and Central Bank) reported sponsored bad bank and the way in which it may
losses. turn out to be beneficial in bringing the banking
Therefore, the best option is (c). sector back on track. The author has not directly
discussed the reasons that have affected the
2) Answer: D lending capacity of the banks.
The whole passage has emphasized reforms in
the Banking sector with a special focus to the

Click Here For Bundle PDF Course | support@guidely.in Page 7 of 11


Bank Po Mains PDF Course 2024
English Day - 1

So, we do not have any specific reason as given banks. Valuation of the assets is one, which will
in the options been discussed directly in the determine the loss. In fact, there has been some
passage. criticism that the government is actually
Therefore, option (e) is the best answer. guaranteeing the losses of private enterprises
and that the PSBs, being shareholders of both
4) Answer: A NARCL and IDRCL, will in effect be buying their
In this question we can say that knowing the own stressed assets. So far, the Indian
meanings of the given options can be of a experience has preferred resolution via IBC over
secondary importance. This is not a question the private ARCs, which have a low capital
where just a plain vocabulary would help. requirement of only Rs 100 crore.
This requires a careful reading and proper Reading this part clearly will help us eliminate
comprehension of the entire idea that is being and reach the correct option.
discussed. There lies the key to the question. So, option (b) is correct.
The line where the word ‘coup’ has been used is 6) Answer: C
talking about the bad bank of China that could The NARCL will be buying the mentioned assets
not materialize the aim with which it was built. as per the line to sell them further in the market.
So, the word ‘collapse’ will be more appropriate It is a company that has been created to deal
than any other option. with the bad loans or the NPAs of the banks. So,
Let us understand the other words: the word that must be used as an adjective for
Coup means a revolt, a sudden, violent, and these assets cannot be positive or neutral.
illegal seizure of power from a government. Options (b), (d) and (a) get cancelled
Insurrection means a rebellion (similar to respectively.
coup) Distressed assets an investment in real
Dispel means dismiss or eliminate; make (a property that is priced below market value—
doubt, feeling, or belief) disappear. typically due to solvency or cash flow issues
Fallacy means the use of invalid or otherwise Therefore, option (c) is the best answer.
faulty reasoning
7) Answer: A
5) Answer: B The answer option can be inferred based on the
The following lines from the second-last reading from the following lines:
paragraph can be used as reference: India’s first-ever government-sponsored “Bad
There are several problems, apart from finding Bank” is now closer to reality.
potential buyers from the stressed assets of

Click Here For Bundle PDF Course | support@guidely.in Page 8 of 11


Bank Po Mains PDF Course 2024
English Day - 1

So far, the Indian experience has preferred 9) Answer: A


resolution via IBC over the private ARCs, which Here, the passage is about driver’s safety and
have a low capital requirement of only Rs 100 the first sentence is expressed in a positive note.
crore. So, we cannot use the word “ignore” and the
The regulatory environment for the ARCs which word “compliance” which is a noun, so “require”
has remained practically static ever since they will be placed here. The second sentence says
had emerged in the 2000s also needs an that drivers should go in speed with great
overhaul. Stressed assets will not disappear responsibility, they have to keep this in mind, for
anytime soon, and we need to enable such that, the word “remember” should be used. Here,
institutions to help absorb the stressed assets. the word “the” is present which will precede the
Economy cannot grow without an adequate noun usually so it will take “compliance” in the
supply of credit and creating a level playing field blank. The fourth statement says backseat
for all. passengers should also use this safety measure.
Therefore, option (a) is correct. From this, we can infer “ignore” should be placed
8) Answer: C here. The fifth blank will take “deploy” as the
Propellant – a fuel force or something that answer.
propels. Therefore, option A is the correct answer.
Here, the blank requires a word that describes
the word “science”, because “science” is helping 10) Answer: D
humans to get progress. So, the word The second sentence says something is still
“propellant” should be used. The second needed to move to the next level, so we can
statement indicates that it works as a remedial infer that flexibility is missing in the Indian
tool from overcoming the consequences, so the Education System, so the word “lacks” should be
word “instrument” should be used. The third used in the first blank. In 2023 Budget,
statement says anti-microbial resistance is government has focused to reshape this, so the
becoming something to human survival and well- word “reorientation” should be used. The third
being, here the word “threat” should be filled, as statement says that this is the first time
the other words are grammatically incorrect. government is considering both, so it has given
Because of the misuse, microbes have emerged. importance, for that the word “emphasis” should
Then, the remaining word “appeared” should be be used. Here, the fourth blank requires a word
filled in the fifth blank. that should describe the word “regulation” an
Therefore, option C is the correct answer. adjective is needed so the word “obsolete”

Click Here For Bundle PDF Course | support@guidely.in Page 9 of 11


Bank Po Mains PDF Course 2024
English Day - 1

should be used. The word “blurring” should be The correct arrangement:


used in the fifth blank. There may be thousands of (C)/ qualified people
Therefore, option D is the correct answer. who you do not (E)/ have access to, but who do
have (D)/ the qualifications for the job (B).
11) Answer: D Hence, option A is the correct answer.
E: introduces the theme that Russia continued
offensive 14) Answer: E
B: where: in Ukrainian territory All of the given sentences are correct.
C: President's steps
A: steps for negotiations. 15) Answer: B
D: doesn't fit anywhere in the sentence. ● In sentence II, ‘which’ with ‘where’.
The correct arrangement: ‘Where’ is used to indicate a location.
As Russia continued its offensive (E)/ into ● In sentence III, remove ‘the’ before
Ukrainian territories, Russian (B)/ President ‘lithium’. ‘The’ cannot be used before any
Vladimir Putin accused (C)/ Ukraine of stalling metal unless you particularize it.
negotiation talks (A). ● Eg: The iron of Jamshedpur.
Hence, option D is the correct answer. ● Sentence I is error free.

12) Answer: C 16) Answer: D


B is not fit anywhere. ● In sentence II, replace ’28 years old’ with
The rest of the parts are already arranged. ’28-year-old’. When age is used as an
The correct arrangement: adjective, we use year-old.
Following administration, the (A)/ genetic ● Eg: A 40-year-old man.
material of the part (C)/ of coronavirus is She is 6 years old.
expressed which (D)/ stimulates an immune ● In sentence III, replace ‘its’ with ‘her’ as
response (E). ‘female colleague’ takes the pronoun
Hence, option C is the correct answer. ‘her’.
● Sentence I is error free.
13) Answer: A
C introduces the sentence and E adds more 17) Answer: A
information about qualified people. Parts D and ● In sentence I, replace ‘as many’ with ‘as
B complete the sentence meaningfully. many as’.
Part A doesn't fit anywhere in the sentence.

Click Here For Bundle PDF Course | support@guidely.in Page 10 of 11


Bank Po Mains PDF Course 2024
English Day - 1

● ‘As many as’ is a phrase used to suggest direct contradiction of sense as given in
that a number or amount is surprisingly the context of the sentence. A
large. philanthropic person would not put up
● Sentence II and III are error free. barbed wires as a sign of hatred or
18) Answer: B keeping out visitors.
● Vitriolic means filled with bitter criticism or ● Therefore, in the given question the
malice. correct answer is option C.
● In all the sentences the word makes 20) Answer: A
sense except sentence B. Here, the word ● Negligent means failing to take proper
contradicts the sense of the given care over something.
sentence. ● In the very first question we can see that
● Therefore, in the given question the there is a clear contradiction of senses. A
correct answer is option B. person cannot be punctual as well as
19) Answer: C negligent both. Hence we can understand
● Philanthropic means (of a person or that it is incorrectly used here.
organization) seeking to promote the Therefore, in the given question the correct
welfare of others; generous and answer is option A.
benevolent. In sentence C there is a

Click Here For Bundle PDF Course | support@guidely.in Page 11 of 11


Bank Po Mains PDF Course 2024
Reasoning Day -2 (Eng)

Reasoning Aptitude

Directions (1-5): Study the following information d) May 2023


carefully and answer the given questions. e) September 2022
Nine persons from the same family - A, B, C, D,
E, F, G, H and I joined a gym in three different 2. Four of the following five are alike in a certain
months – February, May and September of three way based on the given arrangement and thus
different years 2021, 2022 and 2023. Only one form a group. Which one of the following does
person joined in each month and only three not belong to the group?
persons joined in each year. a) C
A is the sister in law of F. A joined in an even b) E
numbered year and in the month having an even c) B
number of days. Only two persons joined d) F
between A and B, who is the brother of A. I, who e) D
has only two children, is the mother in law of F. F
has no siblings. Nobody joined between B and I, 3. If the positions of C and A are interchanged in
who joined in the month having only 31 days. E the same way the positions of F and I are
is the granddaughter of B’s father but not the interchanged, then who among the following
child of F. The number of persons joined after I is person joined the gym two persons before I?
one less than the number of persons joined a) C
before A’s only Daughter. H, who is the mother b) G
of G, joined immediately before E. G is the c) F
brother in law of F’s Husband. Only two persons d) D
joined between H and E’s father. D is the father e) H
in law of G and joined three persons after his
daughter in law but not in an even numbered 4. Who among the following person joined on
year. C is the father in law of A. As many May 2023?
persons joined before D as after the paternal a) The one who joined immediately before G
grandfather of E. b) I
1. On which of the following month and year c) The one who joined two persons after F
does E join the gym? d) H
a) February 2023 e) Both b and c
b) May 2022
c) September 2021

Click Here For Bundle PDF Course | support@guidely.in Page 1 of 10


Bank Po Mains PDF Course 2024
Reasoning Day -2 (Eng)

5. If all the persons joined the gym in Only a few Cricket is Hockey
alphabetical order from February 2021, then how All Hockey is Golf
many persons remain in the same position? No Golf is Badminton
a) Three Conclusions:
b) One I. W≥S; Some Cricket is definitely not Soccer
c) Two II. I>Q; No Hockey is Badminton
d) Four a) Only conclusion I follows
e) None b) Only conclusion II follows
c) Either conclusion I or II follows
Directions (6-10): Study the following statements d) Neither conclusion I nor II follows
and then decide which of the given conclusions e) Both conclusions I and II follow
logically follows from the given statements
disregarding the commonly known facts. 8. Statements:
6.Statements: T ≤ R<E<W; T ≥ A ≤ F>D; M>N>W = Z
V ≥ E = D>L; O<V ≤ F = X; Q = O ≤ T>B Some stumps are balls
Only a few watches are ties Only a few balls are Helmet
Some ties are not socks All Helmets are shoes
All socks are rings Only Shoe is bat
Some rings are cap No shoe is a Glove
Few caps are purse Conclusions:
Conclusions: I. T<M; No bat is Ball
I. F>B; All Ties can never be Ring II. E>F; All shoes can never be stump
II.X>L; All rings can be tie a) Only conclusion I follows
a) Only conclusion I follows b) Only conclusion II follows
b) Only conclusion II follows c) Either conclusion I or II follows
c) Either conclusion I or II follows d) Neither conclusion I nor II follows
d) Neither conclusion I nor II follows e) Both conclusions I and II follow
e) Both conclusions I and II follow
9. Statements:
7. Statements: R<P ≥ F>M = S; B<E>S ≤ L; Z> B < K
Q<S = M ≤ A = L; B>W ≥ I ≥ L; F<G<B All Hero is Royal Enfield
No soccer is Tennis Some Hero is Bajaj
All Tennis is cricket Only a few Bajaj are Honda

Click Here For Bundle PDF Course | support@guidely.in Page 2 of 10


Bank Po Mains PDF Course 2024
Reasoning Day -2 (Eng)

No Honda is Suzuki sitting in T1 are facing towards the centre and


Some Suzuki is Yamaha the persons sitting in T2 are facing away from the
Conclusions: centre.
I. P>B; Some Royal Enfield is Bajaj Note: The consecutive alphabetically named
II. Z>L; All Yamaha being Honda is a possibility persons and complementary paired persons(as
a) Only conclusion I follows per the English alphabetical series) are not sitting
b) Only conclusion II follows adjacent to each other.
c) Either conclusion I or II follows K sits third to the right of R. One person sits
d) Neither conclusion I nor II follows between R and I, who is not an immediate
e) Both conclusions I and II follow neighbour of K. J sits second to the right of S.
Two persons sit between S and O. T sits to the
10. Statements: immediate right of P. The number of persons
B >J = P ≥ S = F; L<Q = R ≤ N<S; X ≤ N ≤ V sitting between R and P(when counted from the
Only a few burfi is Kulfi left of P) is one less than the number of persons
All Burfi is vada sitting between T and G(when counted from the
Only vada is Kheer right of T). H sits second to the left of Q. The
Few Kulfi is Laddoo number of persons sitting between Q and
No Laddoo is Halwa M(when counted from the right of Q) is one less
Conclusions: than the number of persons sitting between H
I. B>R; All Kulfi can be Halwa and N(when counted from the left of H). N is not
II. V≥Q; All vada can never be Laddoo an immediate neighbour of Q. L does not face
a) Only conclusion I follows the centre of the table.
b) Either conclusion I or II follows 11. How many persons sit between H and N?
c) Only conclusion II follows a) One
d) Neither conclusion I nor II follows b) Two
e) Both conclusions I and II follow c) Three
d) Four
Directions (11-15): Study the following e) Either a or d
information carefully and answer the given
questions. 12. What is the position of R with respect to T?
Fourteen persons from G to T are sitting around a) Second to the right
two circular tables such that seven persons sit in b) Second to the left
each table named T1 and T2, where the persons c) Third to the right

Click Here For Bundle PDF Course | support@guidely.in Page 3 of 10


Bank Po Mains PDF Course 2024
Reasoning Day -2 (Eng)

d) Immediate right Directions (16-20): Study the following


e) Third to the left information carefully and answer the given
questions.
13. Four of the following five are alike in a certain A certain number of persons are sitting in a linear
way based on the given arrangement and thus row facing north. The distance between the
form a group. Which one of the following does adjacent persons is ‘4’ in meters. The total length
not belong to the group? of the row is not more than 100m.
a) G S sits 16m to the right of V. The distance
b) T between S and M is 24m. Only one person sits
c) R between M and K. U sits 20m to the left of K. The
d) Q distance between S and U is 8m less than the
e) I distance between K and A. H sits fourth to the
left of A, who sits to the right of M. L sits 8m
14. If all the persons are sitting in the away from H and sits fourth from one of the
alphabetical order from G in a clockwise direction extreme ends of the row. Only three persons sit
in table T2, then how many persons are between L and D. The number of persons sitting
unchanged in their position (excluding G)? between D and H is one less than the number of
a) Two persons sitting between D and B, who sits
b) Four exactly in the middle of the row. Only seven
c) Three persons sit between B and G. P sits exactly
d) One between V and G.
e) None 16. In which of the following option, less than two
persons are sitting between the first person and
15. If the position of J and P are interchanged in second person?
the same way the position of O and K are a) GV
interchanged, then who among the following b) BA
person sits second to the left of O? c) HM
a) P d) KM
b) I e) SV
c) J
d) L 17. How many persons are sitting in the linear
e) N row?
a) 24

Click Here For Bundle PDF Course | support@guidely.in Page 4 of 10


Bank Po Mains PDF Course 2024
Reasoning Day -2 (Eng)

b) 21 I. X sits at the right end of the row


c) 19 II. X sits fourth to the left of U
d) 23 III. X is an immediate neighbour of A
e) 25 a) Only I and III
b) Only II
18. The number of persons sitting to the right of c) Only I and II
D is two more than the number of persons sitting d) Only II and III
to the left of ____ e) All I, II and III
a) The one who sits third to the right of G
b) S 20.The number of persons from the right of H to
c) The one who sits 12m to the left of V ___ is the same as the number of persons from
d) B the left of U to ____.
e) Both a and c a) K, G
b) M. P
19. If X sits second to the right of M, then which c) M, V
of the following statement(s) is/are true with d) G, P
respect to X? e) L, A
Click Here to Get the Detailed Video Solution for the above given Questions
Or Scan the QR Code to Get the Detailed Video Solutions

Answer Key with Explanation


Directions (1-5): 5. Answer: B
1. Answer: C Final arrangement:
2. Answer: D (All the persons joined the gym on
an odd numbered year except option d)
3. Answer: A
4. Answer: E

Click Here For Bundle PDF Course | support@guidely.in Page 5 of 10


Bank Po Mains PDF Course 2024
Reasoning Day -2 (Eng)

Again we have,
 Nobody joined between B and I, who
joined in the month having only 31 days.
 E is the granddaughter of B’s father but
not the child of F.
 The number of persons joined after I is
one less than the number of persons
joined before A’s only Daughter.
 G is the brother in law of F’s Husband.

We have,
 A is the sister in law of F.
 A joined in an even numbered year and in
the month having an even number of
days.
 Only two persons joined between A and
B, who is the brother of A.
 I, who has only two children, is the mother
in law of F. F has no siblings.
From the above conditions, we have four
possibilities,

Again we have,
 H, who is the mother of G, joined
immediately before E.
 Only two persons joined between H and
E’s father.

Click Here For Bundle PDF Course | support@guidely.in Page 6 of 10


Bank Po Mains PDF Course 2024
Reasoning Day -2 (Eng)

 D is the father in law of G and joined three


persons after his daughter in law but not
in an even numbered year.
We cannot place D in case 1. Hence it is
eliminated.

Directions (6-10):
6. Answer: B
I. F>B (F≥V>O≤T>B) -> False
All Ties can never be Ring -> False
II.X>L (X=F≥V≥E=D>L) -> True
All rings can be Tie -> True

Again we have,
 C is the father in law of A.
7. Answer: E
 As many persons joined before D as after
I. W≥S (W≥I≥L=A≥M=S) -> True
the paternal grandfather of E.
Some Cricket is definitely not Soccer -> True
We cannot place C in case 2 and case 4. Hence
II. I>Q (I≥L=A≥M=S>Q) -> True
it is eliminated. Thus Case 3 gives the final
No Hockey is Badminton -> True
arrangement.

8. Answer: A
I. T<M (M>N>W>E>R≥T) -> True
No bat is Ball -> True
II. E>F (F≥A≤T≤R<E) -> False

Click Here For Bundle PDF Course | support@guidely.in Page 7 of 10


Bank Po Mains PDF Course 2024
Reasoning Day -2 (Eng)

All shoes can never be stump -> True 13. Answer: D (All the persons are sitting in table
2 except option d)
14. Answer: A
15. Answer: C
Final arrangement:

9. Answer: D
I. P>B (P≥F>M=S<E>B) -> False
Some Royal Enfield is Bajaj -> True
II. Z>L (Z>B<E>S≤L) -> False
All Yamaha being Honda is a possibility -> False
We have,
 K sits third to the right of R.
 One person sits between R and I, who is
not an immediate neighbour of K.
 J sits second to the right of S.
From the above conditions, we have two
possibilities,
10. Answer: C
I. B>R (B>J=P≥S>N≥R) -> True
All Kulfi can be Halwa -> False
II. V≥Q (Q=R≤N≤V) -> True
All vada can never be Laddoo -> True

Again we have,
 Two persons sit between S and O.

Directions (11-15):  T sits to the immediate right of P.

11. Answer: E  The number of persons sitting between R

12. Answer: B and P(when counted from the left of P) is

Click Here For Bundle PDF Course | support@guidely.in Page 8 of 10


Bank Po Mains PDF Course 2024
Reasoning Day -2 (Eng)

one less than the number of persons  N is not an immediate neighbour of Q.


sitting between T and G(when counted  L does not face the centre of the table.
from the right of T). We cannot place N and M in case1a and 2a.
From the above conditions, we have two more Hence it is eliminated.
possibilities, L faces the centre of the table in case 1. Hence it
is eliminated. Thus, case 2 gives the final
arrangement.

Again we have,
 H sits second to the left of Q.
 The number of persons sitting between Q
and M(when counted from the right of Q)
Directions (16-20):
is one less than the number of persons
16. Answer: D
sitting between H and N(when counted
17. Answer: B
from the left of H).

Click Here For Bundle PDF Course | support@guidely.in Page 9 of 10


Bank Po Mains PDF Course 2024
Reasoning Day -2 (Eng)

18. Answer: E
19. Answer: A
20. Answer: C
Final arrangement:

We have, Again we have,


 S sits 16m to the right of V.  L sits 8m away from H and sits fourth
 The distance between S and M is 24m. from one of the extreme ends of the row.
From the above conditions, we have two  Only three persons sit between L and D.
possibilities,  The number of persons sitting between D
and H is one less than the number of
persons sitting between D and B, who sits
exactly in the middle of the row.
B doesn’t sit exactly in the middle of the row in
case 2. Hence it is eliminated.

Again we have,
 Only one person sits between M and K.
 U sits 20m to the left of K.
 The distance between S and U is 8m less
than the distance between K and A. Again we have,

 H sits fourth to the left of A, who sits to  Only seven persons sit between B and G.

the right of M.  P sits exactly between V and G.

From the above conditions, we have one more We cannot place P in case 1a. Hence it is

possibility for case 1. eliminated. Thus case 1 gives the final


arrangement.

Click Here For Bundle PDF Course | support@guidely.in Page 10 of 10


Bank Po Mains PDF Course 2024
Quantitative Aptitude Day -2 (Eng)

Quantitative Aptitude

Directions (01 - 05): Study the following information carefully and answer the questions given below.
The given pie chart shows the average quantity (litres) of petrol and diesel in five different filling stations
[P, Q, R, S, and T].

Note – The ratio of the quantity of petrol and diesel in S is 4:3. The Total quantity of petrol and diesel in T
is the same. The quantity of diesel is the same in Q and S. The quantity of petrol in R is 25% more than
the same in S. The quantity of petrol in P is 55% less than the same in R. The quantity of diesel in S is
120 litres.
1) The ratio of regular, premium 95 and premium petrol is Rs.102 and the price per litres of diesel
98 types of petrol in Q are 3:2:1. Total 75% of is Rs.94. Find the total revenue earned by P?
petrol sold in Q. The ratio of regular, premium 95 a) Rs.14578
and premium 98 types of petrol sold in Q is 5:4:2. b) Rs.15268
Find the percentage of premium 95 and premium c) Rs.14252
98 types petrol unsold in Q? d) Rs.15336
a) 12.5% e) Rs.15642
b) 19.5%
c) 18.18% 3) Total quantity of petrol sold in R is _______ %
d) 11.11% and total quantity of diesel sold in R is _______%.
e) 16.67% The total unsold quantity of fuel in R is _______.
Find which of the following satisfies the blank?
2) Out of the total quantity of petrol in P, 6.66% a) 75, 80, 120
are unsold and out of the total quantity of diesel b) 80, 75, 95
in P 7.69% are unsold. The price per litres of c) 60, 70, 105.5
d) More than one option is true

Click Here For Bundle PDF Course | support@guidely.in Page 1 of 11


Bank Po Mains PDF Course 2024
Quantitative Aptitude Day -2 (Eng)

e) None of these trucks and the rest for cars. Find the total
quantity of fuel sold to the car in S?
4) 80% of petrol and 80% of diesel are sold in S. a) 70.4
Out of the total quality of petrol sold in S, 75% is b) 74.4
sold for bikes and the rest for cars. Out of the c) 72.4
total quantity of diesel sold in S, 60% sold for d) 76.4
e) None of these

Directions (05 - 08): Study the following information carefully and answer the questions given below.
The line graph shows the number of boys, the number of girls and the total number of chocolates
distributed among all the students of the five different classes [A, B, C, D, and E].

Note – In classes D and B, each boy and girl got 2 chocolates, in class C, boys got 2 chocolates and girls
got 5 chocolates. In class A, boys got 3 chocolates and girls got 3 chocolates.
5) The average age of boys in class A is 5 years II. 5.9>average age>5.3
and the average age of boys in class B is 6 III. 5.1<average age <5.7
years. The average age of girls in class A is 6 a) I and III
years and the average age of girls in class B is 5 b) II and I
years. Find the possible range of average age of c) II and III
all the students of classes A and B? d) I and II
I. 6.5>average age>5.8 e) only II

Click Here For Bundle PDF Course | support@guidely.in Page 2 of 11


Bank Po Mains PDF Course 2024
Quantitative Aptitude Day -2 (Eng)

6) The boys of class E got (z-1) chocolates. If a) only II


each boy got 2 chocolates more and each girl b) All three
got 3 chocolates more then find the total number c) only I and II
of chocolates distributed in class E? d) only II and III
a) 254 e) None of these
b) 242
c) 282 8) The ratio of mango chocolate and orange
d) 222 chocolate is 4:5 out of the total chocolate
e) 262 distributed in A and the ratio of mango and
orange chocolate is 5:3 out of the total chocolate
7) The ratio of the number of boys in classes D distributed in B. If the price of each mango
and F is 2:3 and the ratio of the number of boys chocolate is Rs.1.5 and the price of each orange
and girls in class F is 3:4. The average number chocolate is Rs.2. Find the total amount spend in
of girls in classes F and G is 30. The average classes A and B?
number of students in class G is 32. Find the a) Rs.285
possible difference of total boys in class F and b) Rs.295
G? c) Rs.245
I.3z+4 d) Rs.265
II.2x+3 e) None of these
III.6y+1

Directions (09 - 12): Study the following information carefully and answer the questions given below.
The given bar graph shows the difference between the CI and SI in two years and the principle amount in
four different schemes [A, B, C, and D].

Click Here For Bundle PDF Course | support@guidely.in Page 3 of 11


Bank Po Mains PDF Course 2024
Quantitative Aptitude Day -2 (Eng)

9) A person has Rs. (X+12000). He divided the c) More than one option is true
amount into Rs. m and Rs. n. He invested Rs. m d) Rs.25000, Rs.20000, Rs.17917.5
in scheme D in SI and he invested Rs. n in e) Rs.12000, Rs.16000, Rs.11934
scheme C in CI for 2 years. The interest amount
in scheme C is 3570 more than he earned in 11) The difference between SI and CI is Rs.3200
scheme D. The ratio of m and n 3:5. Find the CI when Rs. (x + 2500) invested for 2 years in
when Rs. X invests at scheme D for 2 years? scheme B. Find the SI when (2x -5000) invests
a) Rs.5420 for 5 years in the same scheme?
b) Rs.5888 a) Rs.52000
c) Rs.5088 b) Rs.60000
d) Rs.6088 c) Rs.85000
e) Rs.7288 d) Rs.56200
e) None of these
10) Rs. ______ invest in scheme C at simple
interest for 2 years and Rs. _______ invest in 12) The ratio of SI when Rs.x+2000 and
scheme C at compound interest for 3 years. The Rs.x+5000 invest in scheme A for 3 and 4 years
sum of SI and CI is ________. is 3:5. Find the CI if Rs.2x+500 invest in scheme
Find which of the following is true? A for 2 years?
a) Rs.12000, Rs.16000, Rs.1540 a) Rs.4305
b) Rs.20000, Rs.22000, Rs.14852 b) Rs.4450

Click Here For Bundle PDF Course | support@guidely.in Page 4 of 11


Bank Po Mains PDF Course 2024
Quantitative Aptitude Day -2 (Eng)

c) Rs.4250 e) None of these


d) Rs.5420

Directions (13- 16): Study the following information carefully and answer the questions given below.
The given table shows the total number of bottles [large + small] manufactured, the difference of number
[large – small] size bottles manufactured, the percentage of default large size bottles manufactured, the
percentage of default small size manufactured and the number of default large size bottle manufactured
by five different factory [P, Q, R, S and T].

Tota bottle manufactured = large + small size bottle = perfect and default bottle manufactured
13) The total manufacturing cost of the large 14) The ratio of red and green bottles
bottle is Rs. 18 and the small bottle is Rs.12. The manufactured out of the total large bottles
selling price of the large bottle is Rs.25 and the manufactured in Q is 2:3 and the ratio of red and
small bottle is 20. If the bottle is the default, then green bottles manufactured out of the total small
it is sold at a 20% discount on the selling price. size bottles manufactured in Q is 4:3. Find the
Find the total profit earned by P? possible difference between the total red and
a) 21.32% total green manufactured in Q?
b) 45.35% I. E-6
c) 40.32% II. B/4
d) 75.32% III. D/2
e) 32.22% a) only I

Click Here For Bundle PDF Course | support@guidely.in Page 5 of 11


Bank Po Mains PDF Course 2024
Quantitative Aptitude Day -2 (Eng)

b) only II There are four tourist spots [A, B, C, and D].


c) only III Different numbers of people visit in June and
d) only I and II July. The ratio of tourists visiting A in June and
e) all true July is 6:7. The ratio of tourists visiting in July in
B and C is 6:7. The number of tourists visiting B
15) Find which of the following is true? in June was 25% less than the number of tourists
I. The sum of the perfect large bottles visiting B in July. In June, the number of tourists
manufactured in P and Q is 180. visiting C was 33.33% more than the number of
II. Difference between default bottles tourists visiting D in the same month. The
manufactured in R and S is 0. number of tourists visiting D in July is 150%
III. Total small bottles manufactured in S and more than the number of tourists visiting A in
Tare 190. June. The total number of tourists visiting A in
a) only I July and D in June is 3800. The total number of
b) only II and III tourists visiting B in June and D in July is 4800.
c) only II The number of tourists visiting C in July is 12.5%
d) only III less than the number of tourists visiting C in
e) None of these June.
17) The ratio of the number of tourists visiting E
16. Quantity: I Total large perfect bottle and D in June is 3:2 and the ratio of the number
manufactured in P and T is what percent of total of tourists visiting E and C in July is 5:4. The
small default bottle manufactured in R and S? Total number of tourists visiting in June and July
Quantity: II Total large perfect bottle in F is 20% more than E. Find the total number of
manufactured in Q and R is what percent of total tourists visit in F in July if The ratio of the number
small default bottle manufactured in S and T? of tourists visit F in June and July is 3:2?
A. Quantity: I < Quantity: II a) 3508
B. Quantity: I ≥ Quantity: II b) 3208
C. Quantity: II ≥ Quantity: I c) 3408
D. Quantity: I > Quantity: II d) 3068
E. Quantity I = Quantity II or relation can't be e) 3048
established
18) The ratio of male and female visitors in A in
Directions (17-20): Study the following June is 7:5 and the ratio of male and female
information carefully and answer the questions visitors in A in July is 4:3. The Ticket price for
given below. males is Rs.8 and the Ticket price for females is

Click Here For Bundle PDF Course | support@guidely.in Page 6 of 11


Bank Po Mains PDF Course 2024
Quantitative Aptitude Day -2 (Eng)

Rs.5. Find the total revenue earned by A in June c) 6820


and July together? d) 6640
a) Rs.14500 e) None of these
b) Rs.14570
c) Rs.14520 20) The average age of tourists in June in D is
d) Rs.17500 29 years and the average age of tourists in July
e) Rs.16520 in D is 21 years. Find the average age of all
tourists in D in June and July together?
19) The number of tourists increased by 25% a) 24.55 years
and 20% in B and C in August than previous b) 22.55 years
month. Find the total number of tourists in B and c) 25.65 years
C in August? d) 29.65 years
a) 6540 e) None of these
b) 6360
Click Here to Get the Detailed Video Solution for the above given Questions
Or Scan the QR Code to Get the Detailed Video Solutions

Answer Key with Explanation


Directions (01 - 05): or, The total quantity of petrol and diesel in T is =
The quantity of diesel in S and Q is 120 liters 140*2=280litres
The quantity of petrol in S is 120*4/3=160 litres The quantity of petrol and diesel in T is =
So, the average quantity of petrol and diesel in S [280/2]=140 liters
is = [120+160]/2=140 litres The quantity of petrol in R is = 160*125/100=200
So, total value of pie chart is = [140/20] liters
*100=700liters The total quantity of petrol and diesel in R is =
So, the average quantity of petrol and diesel in T [700*30/100]*2=420litres
is = 700*20/100=140litres

Click Here For Bundle PDF Course | support@guidely.in Page 7 of 11


Bank Po Mains PDF Course 2024
Quantitative Aptitude Day -2 (Eng)

So, The quantity of diesel in R is = 420- Some total quantity of fuel sold to S is
200=220litres = [160*80/100] * [25/100] +
Quantity of petrol in P is = 200*45/100=90 liters [120*80/100]*[40/100]
Total quantity of petrol and diesel in P is = = 32+38.4=70.4 liters
[700*12/100] *2=168 liters
So,The quantity of diesel in P is = 168-90=78 Directions (05 - 08):
liters So, in D, 6z*2+12y*2=28z
Total quantity of petrol and diesel in Q is = Or, 12z+24y=28z
[700*18/100] *2=252 liters In C, 15y*2 + 5z*5=26x+5
The quantity of petrol in Q is = 252-120=132 In A, 2x*3+10y*3=30z
liters In B, 5x*2+3x*2=40Y
1) Answer: C Or, 16x=40Y
So, the total premium 95 and premium 98 types Or, x=40y/16=2.5y
of petrol in Q is 132*3/6=66 liters Or, 6*2.5y+30y=30z
Total premium 95 and premium 98 types of Or, 45y=30z
petrol sold in Q is Or, z=1.5y
= [132*75/100] *6/11=54 liters So, 30y+25*1.5y=26*2.5y+5
So, required percentage = [66-54] Or, 2.5y=5, y=2
*100/66=18.18% So, z=1.5*2=3
So, x=2.5*2=5
2) Answer: D
6.66% = 1/15 and 7.69% =1/13
So, the total revenue earned by P
=[90*14/15]*102 + [78*12/13]*94
= 84*102+72*94=Rs.15336

3) Answer: B 5) Answer: C
Let's check option B, The required average age is

The total unsold quantity of petrol in R is =

= [200*20/100] + [220*75/100] [10*5+15*6+20*6+25*5]/[10+15+20+25]=385/70

= 95 =5.5
II. 5.9>average age>5.3 III. 5.1<average age

4) Answer: A <5.7

Click Here For Bundle PDF Course | support@guidely.in Page 8 of 11


Bank Po Mains PDF Course 2024
Quantitative Aptitude Day -2 (Eng)

6) Answer: D 9) Answer: C
Each girls got = [100-28*(3-1)]/22=44/22=2 Let m is 3i and n is 5i.
So, now total chocolate distributed is = [5i*[115/100]2-5i ] – [3i*12*2/100]=3570
28*4+22*5=222 Or, 1.6125i-0.72i=3570
Or, 0.8925i=3570
7) Answer: B Or, i=3570/0.8925=4000
Boys in F is = 18*3/2=27 So, X=5*4000+3*4000-12000=20000
Girls in F is = 27*4/3=36 So, required CI = 20000*[1+(12/100)]2-
Girls in G is = 30*2-36=24 20000=Rs.5088
Boys in G is = 32*2-24=40
So, difference = 40-27=13=3z+4=6y+1=2x+3 10) Answer: C
Let's check option D,
8) Answer: B So, SI = 25000*15*2/100=7500
Total amount of chocolate spent in A and B CI = 20000*[1+15/100]3-20000=30417.5-
= [90*4/9]*1.5 + [90*5/9]*2 +[80*5/8]*1.5 + 20000=10417.5
[80*3/8]*2 So, sum of interest = 10417.5+7500=17917.5
=60+100+75+60=295 Similarly, we can check others' values also.
We can see only D and E are true.
Directions (09 - 12):
The amount of investment in scheme A is 11) Answer: B
Rs.10000 So, we can say,
The difference between CI and SI is Rs.100 3200 = [x+2500]*[40]*[40]/ [100*100]
So, 100=10000[R/100]2 Or, [x+2500]=[3200*100*100]/[40*40]=20000
Or, R2=100 Or, x=20000-2500=17500
Or, R= +10, -10 so, R cannot be negative. So, required SI = [17500*2-
So, R=10%. 5000]*40*5/100=30000*40*5/100
So, similarly, we can calculate the interest rate of =60000
other schemes.
12) Answer: A
So,
[(x+2000)*3*10/100]:[(x+5000)*4*10/100]=3:5
Or, 4x+20000=5x+10000
Or, x=10000

Click Here For Bundle PDF Course | support@guidely.in Page 9 of 11


Bank Po Mains PDF Course 2024
Quantitative Aptitude Day -2 (Eng)

So, Required CI = [20500*110*110/(100*100)]- 13) Answer: C


20500=Rs.4305 Total cost = 18*140+100*12=Rs.3720
Total selling price = [100*25+40*(25*80/100)] +
Directions (13- 16): [80*20 + 20*(20*80/100)] = 3300+1920=5220
The total number of large size bottles So, profit percentage = [5220-
manufactured in P is = [40/28.57]*100=140 3720]*100/3720=40.32%
Similarly, the total number of large size bottles
manufactured in Q, R, S, and T is 100, 140, 90, 14) Answer: D
and 160 respectively. The total number of red colour bottles
So, [A+B]/2=140, [C+D]/2=100, [A+10+D]/2=140, manufactured is = 100*2/5 + 70*4/7
[C-10+B/2]/2=90 =40+40=80
So, A+B=280 The total number of green colour bottles
C+D=200, A+D=270, 2C-20+B=360, 2C+B=380 manufactured is = 170-80=90
So, A+B-A-D=280-270, B-D=10, B=D+10 So, the difference is 90-80=10=B/4=E-6
So, 2C+D+10=380 So, I and II true
Or, 2C+D=370
So, 2C+D-C-D=370-200=170 15) Answer: B
So, C=170, I. The sum of perfect large bottles manufactured
D=200-170=30 in P and Q is = 70+120=190.
B=30+10=40 II. The Difference of default bottles manufactured
A=280-40=240 in R and S is = 20+20-30-10=0.
So, E=[320-280]/2.5=16 III. Total small bottles manufactured in S and T is
Number of small size bottles manufactured in =120+70=190.
T=[240+40-160]=120
16) Answer: A
Quantity I
Required percentage =
[(100+140)/(20+10)]*100=800%
Quantity II
Required percentage
=[(70+120)/(10+10)]*100=950%
Quantity I<Quantity II

Click Here For Bundle PDF Course | support@guidely.in Page 10 of 11


Bank Po Mains PDF Course 2024
Quantitative Aptitude Day -2 (Eng)

Directions (17-20): In June, the number of tourists visiting D is


Let, The number of tourists visiting A in June and =6*400=2400.
July be 6x and 7x, respectively. 17) Answer: C
Let the number of tourists visiting B and C in July The total number of tourists visiting E in June
be 6y and 7y, respectively. and July is
The number of tourists visiting B in June is =2400*3/2 + 2800*5/4=3600+3500=7100
6y*75/100=4.5y. The total number of tourist visits in F is =
In July, the number of tourists visiting D is 7100*120/100=8520
6x*250/100 = 15x. The number of tourists visiting F in July is =
So, 4.5y+15x=4800 --(I) 8520*2/5=3408
The number of tourists visiting in June in C is
7y*100/87.5=8y. 18) Answer: D
In June, the number of tourists visiting D is = So, the total revenue earned by A in June and
8y*100/133.33 = 6y. July together
So, 6y+7x=3800---(II) = [1200*7/12]*8+[1200*5/12]*5 +[1400*4/7]*8 +
By solving (I) and (II), we get x = 200 and y = [1400*3/7]*5
400. = 5600+2500+6400+3000=17500
The number of tourists visiting A in June and
July is 6* 200 = 1200 and 7* 200 = 1400. 19) Answer: B
The number of tourists visiting in July in B and C So, the total number of tourists in B and C is
is 6* 400 = 2400 and 7* 400 = 2800, = 2400*125/100 + 2800*120/100= 6360
respectively.
The number of tourists visiting B in June is 20) Answer: A
4.5*400=1800. So, required average age =
The number of tourists visiting in July in D is [2400*29+3000*21]/5400
=15* 200 = 3000. =132600/5400=24.55 years
The number of tourist visits in June in C is 8 *
400 = 3200.

Click Here For Bundle PDF Course | support@guidely.in Page 11 of 11


Bank Po Mains PDF Course 2024
English Day - 2

English Language
Directions (01-05): In the following questions, A. Key actors are trying D. ahead of the
two columns (column I and column II) are given. to figure out their most LokSabha and
In column I, sentences A, B and C are present, advantageous Assembly elections
while column II has sentences D, E and F. You positioning that is scheduled for
are required to choose the option that can make next year.
the most meaningful and contextually correct B.The Congress E. is now plotting its
sentence with the help of the following options. projects itself as the next move to
1. secular national dominate State politics
COLUMN I COLUMN II alternative to the BJP on its own.
A.The first satellite D. billion dollars C. The BJP, having F. but has to engage
communication policy annually to procure benefited from the split in a balancing act that
was introduced in 1997, earth observation data in the Shiv Sena might necessitate
and imagery from sacrificing some of its
foreign sources. share of seats.
B.The policy framework E. requiring guidelines a) Only A-E.
envisaged will need for foreign direct b) Only C-D.
clear rules and investment (FDI) c) Both A-E and C-D.
regulations across the satellite d) Only B-F.
industry. e) All A, B and D.
C.Indian users including F. excluding to FDI
the security and and licensing, 3.
defence agencies government COLUMN I COLUMN II
spend nearly a procurement to A.The Prime Minister’s D. have been the result
sustain the new space personal involvement of sustained efforts
start-ups. in the enterprise, as over decades.
a) Only A-E. well as the significant
b) Only C-D. publicity
c) Both A-E and C-D. B. It is implicit that E. effort by government
d) Only B-F. there will be many departments to promote
e) All A, B and D. deaths between the the endeavor as India’s
animals exemplary commitment
2. to wildlife conservation.
COLUMN I COLUMN II

Click Here For Bundle PDF Course | support@guidely.in Page 1 of 12


Bank Po Mains PDF Course 2024
English Day - 2

C. The increase in the F. if one factors in both


lion population in Gir, the natural lifespan of 5.
Gujarat, as well as the cat as well as the COLUMN I COLUMN II
tiger numbers challenges. A. Pakistan’s former D. probe the
a) Only A-E. Prime Minister Imran allegations against him
b) Only C-D. Khan was on in the Al-Qadir Trust
c) Both A-E and C-D. Wednesday sent on an case.
d) Only B-F. eight-day remand to
e) All A, B and D. B. Attack on public E. anti-corruption
property is an act of watchdog while a
4. terrorism and enmity sessions court indicted
COLUMN I COLUMN II towards him in a separate graft
A. India’s status, D. programme were case.
security and ability to shrouded in a veil of C. The NAB lawyers F. the country Mr
influence the ambiguity and requested the court to Sharif said, adding that
international system opaqueness. grant a 14 day remand those taking the law
received arguably of Mr.Khan to into their own hands
B.The 1998 tests E. the greatest fillip- will get iron fist.
unleashed a fury of then, since a) Only C-E.
events and catapulted independence and b) Only C-D.
India unarguably the c) Both A-F and C-D.
strongest boost since d) Only B-D.
the end of the Cold e) All A, B and D.
War.
C. In the previous two F. into probably its Directions (6-12): Read the following passage
decades, the military worst confrontation with and answer the following questions based on the
aspects of India’s the United States. information provided in the passage.
nuclear policy and It’s now apparent that efforts to cut down GHG
a) Only A-E. emissions are not making an appreciable
b) Only C-D. difference. The latest warning has come from the
c) Both A-E and C-D. National Oceanic and Atmospheric
d) Only B-F. Administration in the US. Its observatory in
e) All A, B and D. Mauna Loa in Hawaii has reported that carbon

Click Here For Bundle PDF Course | support@guidely.in Page 2 of 12


Bank Po Mains PDF Course 2024
English Day - 2

dioxide levels in May have recorded a new high Tuesday, the US delegation asserted that
— 424 parts per million (PPM), up from 421 PPM bridging the gap was not the sole responsibility of
in May last year. The agency’s data shows that developed countries. It argued that the next
carbon concentration grew at about 1 PPM per round of climate action plans must involve
year till about the 1970s. But the rate of increase contributions from all sectors of the economy —
has spiralled since then. In the last decade, the a veiled criticism of India whose mitigation plans
annual increase touched 2.5 PPM. It has now focus heavily on renewable energy and
crossed the 3 PPM mark. The current carbon increasing the forest cover. India has remained
levels are 50 per cent higher than the pre- steadfast in aligning climate targets with its
industrial era. The writing has been on the wall developmental goals. On Tuesday, it coordinated
for some time now. The trouble is climate with other developing countries to demand that
negotiations have not achieved much since the the developed countries fulfil their pre-2020
high of Paris eight years ago. commitments under the Paris Pact’s
Climate diplomats from around the world are predecessor, the Kyoto Protocol.
currently meeting in Bonn to lay the ground for The arguments of the developing countries have
the UNFCCC’s COP 28 in Dubai at the end of received a boost from a recent study published in
this year. This year’s conference has added the journal Nature Sustainability. It reckons that
significance because climate scientists have the US has used up more than four times its fair
begun work on a critical mandate of the Paris allocation of the world’s carbon budget — the
Pact. Known as the Global Stocktake, the global carbon budget starting from 1960 is 1.8
exercise will determine the exact gap between trillion tonnes of CO2, according to the IPCC.
the pact’s goals and the climate actions taken so India, in contrast, has used less than a third. In
far. The review is slated to conclude at the COP the course correction after the Paris Pact, such
in Dubai, where delegates are likely to deliberate calculations of climate justice must find a place.
on the means to address the shortfall. The 6. Choose the word that is most SAME to the
exercise is not mandated to assess the efficacy given highlighted word SPIRALLED in the
of the measures taken by individual countries. passage.
But with collective global action proving a) Dwindle
inadequate, questions are being raised about b) Mushroom
Paris’s credo of voluntarism. That means that c) Scourge
familiar differences over apportioning d) Confute
responsibilities are likely to dog the stocktaking e) None of these.
exercise as well. The initial discussions at Bonn
indicate the shape of things to come. On

Click Here For Bundle PDF Course | support@guidely.in Page 3 of 12


Bank Po Mains PDF Course 2024
English Day - 2

7. Choose the word that is most OPPOSITE to a) Only I.


the given highlighted word SLATED in the b) Only II.
passage. c) Only III.
a) Felicitate d) Both I and II.
b) Wobble e) Both II and III.
c) Extol
d) Fortuitous 10. According to the given passage, which of the
e) None of these. following statement/statements is/are TRUE to
the given context?
8. What is the primary cause behind the recent I. US has used more than four times of the
discussion according to the passage? allocation to carbon budget.
I. To provide the review of the climatic events by II. The next round of talks should discuss about
climatic diplomats at Dubai. the renewable energy sector to mitigate the
II. To provide a boost to the countries which are emission.
exceeded the carbon dioxide levels more than III.The agency’s data shows that carbon
the previous year. concentration grew at about 1 PPM per year till
III. To provide ideas and steps to mitigate the about the 1970s.
carbon dioxide levels to reach lower level. a) Only I.
a) Only I. b) Only II.
b) Only II. c) Only III.
c) Both I and II. d) Both I and III.
d) Both I and III. e) Both I and II.
e) None of the above.
11. Which of the following option can better
9. According to the given passage, which of the convey the reason behind criticism faced by
following option can be inferred as the primary India?
objective of the upcoming discussion? I. India is the top most country to emit carbon
I. It will discuss why the Paris agreement cannot among the other developing countries.
be achieved yet. II. India is concentrating only on improving the
II. This year’s climate critical mandate will be forest cover and focusing on renewable energy.
discussed in the upcoming meeting which has III. India has withdrawn from the pact agreement
planned to be held in Dubai. as it knows reducing carbon emission is highly
III. The global effort will be discussed as the impossible.
reason for the shortfall. a) Only I.

Click Here For Bundle PDF Course | support@guidely.in Page 4 of 12


Bank Po Mains PDF Course 2024
English Day - 2

b) Only II. drugresistant microbes have ____________


c) Only III. throughantibiotic misuse. The prospect of
d) Both I and II. findingpotent novel antibiotics ____________ dim
e) Both II and III. tillit was announced recently that
artificialintelligence (AI) helped to identify a
12. Through which of the following, the article newantibiotic (again) that is highly
conveys that greenhouse gas emission target is effectiveagainst the pathogenic bacterium.
not achieved? I. Appeared
I. Climatic diplomats discussed about the II. Emerged
changes in carbon emission in the recent III. Propellant
meeting held at Bonn. IV. Threat
II. Paris Act has set a target every year for V. Instrument
analysing the climate change, but it is not a) V, IV, I, I, II.
achieved. b) IV, III, I, I, II.
III. An entity based in Hawaii reported that the c) III, V, IV, II, I.
carbon level is higher when comparing the d) I, II, III, IV, V.
previous year. e) None of the combination is correct.
a) Only I.
b) Only II. 14.One of the achievements of the
c) Only III. NarendraModi government is the network of
d) Both I and II. excellent highways across the country,
e) Both II and III. which____________ matching driving skills,
especially as car manufacturersare developing
Directions (13-17): In the following questions, a more powerful engines and foreign car
passage has been given with multiple blanks. makersare bringing their best wares. That can
You are required to choose the appropriate word make for a deadly combination.And drivers must
for each blank from the given options. _____________ that with great speedcomes great
13.Science has been a powerful ____________for responsibility. The ______________ with strapping
humanity’s progress. It has become onseatbelts is of recent origin. While those in the
the____________ of human follies and also front seats do itwilly-nilly, backseat passengers
theremedial tool that rescues us from ______________ this safety measure. Untilthe
theirconsequences. Anti-microbial resistancehas accident which claimed billionaire Cyrus Mistry in
become a major _____________ to human September 2022 on the Mumbai Ahmedabad
survivaland well-being, as several multi- Expressway, the seatbeltdid not get its due

Click Here For Bundle PDF Course | support@guidely.in Page 5 of 12


Bank Po Mains PDF Course 2024
English Day - 2

importance. Neither was it widely knownthat IV. Emphasis


airbags do not ___________ if the seatbelt is not V. Reorientation
strapped on. a) IV, II, III, V, I.
I. Compliance b) II, V, III, I, IV.
II. Deploy c) III, IV, V, III, I.
III. Require d) II, V, IV, I, III.
IV. Remember e) None of the combination is correct.
V. Ignore
a) III, IV, I, V, II. 16.Contaminated foodscan cause a__________
b) V, II, I, IV, III. outcome.From posing a risk to life and
c) IV, III, I, II, V. causingsevere permanent damage tothe body, it
d) II, III, V, IV, I. is of _____________importance to identify
e) None of the combination is correct. foodhazards and ensure 360-degree food safety.
Suchmeasures not only protectindividual health
15.Indian education system_____________ the but also_______________ to the overallwell-being
required flexibilityover the years, which is of communities.By being _______________ in
verymuch required to take it tothe next level and foodsafety practices, one can significantlyreduce
make it truly global in approach andpracticality. the _________________of foodborne illnessesand
In Budget 23,the government has focusedon the their associated economicand social burdens.
_______________ of educationand skills I. Fatal
according tothe aptitude of the youthand the II. Paramount
demands of thefuture.It is for the first time thatan III. Contribute
equal ______________ is beinggiven to both IV. Proactive
education andskilling as part of the V. Incidence
neweducational policy, whichunburdens the a) IV, I, III, V, II.
students fromthe ______________ regulations b) II, I, V, III, IV.
andprocesses of the past. Thenew technological c) I, IV, II, V, III.
revolutionin recent years is helpinga lot in the d) III, V, II, I, IV.
process by creatingnew types of classroomswith e) None of the combination is correct.
unrestricted access to all,_______________ the
considerationslike region and distance. 17.It has been known for nearly a decade now
I. Obsolete that to_____________ the cops, drug traffickers
II. Lacks are ______________ to tradingon marketplaces
III. Blurring on the darknet — an Internet platformthat uses

Click Here For Bundle PDF Course | support@guidely.in Page 6 of 12


Bank Po Mains PDF Course 2024
English Day - 2

secret alleys on The Onion Router (ToR), D. The inaugural trial flight carried 10 units of
provides________________ to the traffickers and whole blood samples from the GIMS and the
their customersand is, thus, difficult to detect and LHMC in visual line of sight.
____________. Theirmodus operandi involves E. All are related to one another.
clandestinely _____________ customerson social
media, dealing in cryptocurrency andusing 19.
courier services for delivery. A. The recommendation was made by the
I. Luring International Council on Monuments and Sites
II. Anonymity (ICOMOS), which is the advisory body to the
III. Resorting UNESCO World Heritage Centre, based on a file
IV. Evade moved by the Centre.
V. Penetrate B.Santiniketan, associated with Nobel laureate
a) V, III, II, IV, I. Rabindranath Tagore, has been recommended
b) IV, III, II, V, I. for inclusion on UNESCO’s World Heritage List.
c) IV, I, II, IV, III. C.If it is included in UNESCO’s Heritage List,
d) II, V, III, I, IV. India will have 41 sites across various states.
e) None of the combination is correct. D. The quaint town in West Bengal’s Birbhum
district is also home to the ViswaBharati
Directions (18-20): In the following questions, a University builtby Tagore.
set of sentences is given and is related to one E. All are related to oneanother.
another. Out of which, only one sentence will not
be relevant to the given context. You are 20.
required to choose the odd sentence and mark A. Many colleges and universities have opened
the same as your answer. their admission window owing to this.
18. B. In an official tweet on Wednesday, the CBSE
A.The Indian Council of Medical Research flagged the fake notice.
(ICMR) on Wednesday successfully conducted a C.The Central Board of Secondary Education
trial run of delivery of blood bags under its iDrone (CBSE) has flagged a fake circular that the
initiative. results of Class 10 and 12 will be declared on
B.The Central Government has allotted Rs 1500 Thursday.
crores for the iDrones initiative to be D. The hoax had gone viral on social media,
implemented across states. causing anxiety to students and parents.
C. More drone flights will be conducted to E. All are related to one another.
validate the quality of blood products transported.

Click Here For Bundle PDF Course | support@guidely.in Page 7 of 12


Bank Po Mains PDF Course 2024
English Day - 2

Click Here to Get the Detailed Video Solution for the above given Questions
Or Scan the QR Code to Get the Detailed Video Solutions

Answer Key with Explanation


1. Answer: B 2. Answer: D
Option A is incorrect, because statement A says Option A is incorrect, the word “their” is used in
policy was introduced whereas E says it requires statement A, whereas in statement E, the word
guidelines. If it does not have guidelines “it” is used referring a singular pronoun.
regarding the satellite policy, then what is the Option B is incorrect, the word “is” is used
policy all about? thus makes it incorrect. Also, instead of “are”, it indicates the noun
the word “across” is incorrectly used. “elections”(plural).
Option D is incorrect, “excluding” will not take The correct sentence is,
“to” after it. I.Key actors are trying to figure out their most
The correct sentence is, advantageous positioning ahead of the
I.The first satellite communication policy was LokSabha and Assembly elections that are
introduced in 1997, with guidelines for foreign scheduled for next year.
direct investment (FDI) in the satellite industry. II.The BJP, having benefited from the split in the
II.Indian users including the security and defence Shiv Sena, is now plotting its next move to
agencies spend nearly a billion dollars annually dominate State politics on its own.
to procure earth observation data and imagery III.The Congress projects itself as the secular
from foreign sources. national alternative to the BJP but has to engage
III.The policy framework envisaged will need in a balancing act that might necessitate
clear rules and regulations pertaining to FDI and sacrificing some of its share of seats.
licensing, government procurement to sustain Therefore, option D is the correct answer.
the new space start-ups.
Therefore, option B is the correct answer. 3. Answer: C

Click Here For Bundle PDF Course | support@guidely.in Page 8 of 12


Bank Po Mains PDF Course 2024
English Day - 2

Option D is incorrect, statement B has the word The correct sentence is,
“animals” which is denoting plural form, so I.The NAB lawyers requested the court to grant a
“among” should be used. 14 day remand of Mr.Khan to probe the
The correct sentence is, allegations against him in the Al-Qadir Trust
I.It is implicit that there will be many deaths case.
among the animals if one factors in both the II.Pakistan’s former Prime Minister Imran Khan
natural lifespan of the cat as well as the was on Wednesday sent on an eight-day
challenges. remand to the anti-corruption watchdog while a
II.The increase in the lion population in Gir, sessions court indicted him in a separate graft
Gujarat, as well as tiger numbers have been the case.
result of sustained efforts over decades. III.Attack on public property is an act of terrorism
III.The Prime Minister’s personal involvement in and enmity towards the country Mr Sharif said,
the enterprise, as well as the significant publicity adding that those taking the law into their own
effort by government departments to promote hands.
the endeavor as India’s exemplary commitment Therefore, option B is the correct answer.
to wildlife conservation.
Therefore, option C is the correct answer. 6. Answer: B
Spiralled –show a continuous and dramatic
4. Answer: E increase.
Here, all the given sentences can be matched Mushroom - increase, spread or develop rapidly.
with the respective sentences given in the Dwindle – diminish gradually.
option. Confute – prove to be wrong.
Therefore, option E is the correct answer. Scourge – cause great suffering to.
The word “mushroom” is the synonym of the
5. Answer: B given highlighted word “spiralled”. Therefore,
Option A is incorrect, the word “to” will take option B is the correct answer.
infinite verb after it, it will not take noun directly
after it. 7. Answer: D
Option C is incorrect,A-F is contextually Slated – planned or scheduled; criticized.
incorrect. Fortuitous – happening by chance.
Option D is incorrect, the sentence B is about Extol – praise enthusiastically.
terrorism and enmity but D states a different Wobble – walk unsteadily due to lack of balance.
context. Felicitate – praise.

Click Here For Bundle PDF Course | support@guidely.in Page 9 of 12


Bank Po Mains PDF Course 2024
English Day - 2

The word “fortuitous” is the antonym of the given 11. Answer: B


highlighted word “slated”. Therefore, option D is Only option B is correct, according to the given
the correct answer. passage India concentrates only on renewable
energy and focuses to improve the forest cover.
8. Answer: E This is the reason behind the criticism faced by
The passage states that the recent meeting is India.
about to lay a base for the UNFCCC’s COP 28 in Therefore, option B is the correct answer.
Dubai by the end of the year. It is about to
review what has been left out to bridge the gaps 12. Answer: C
between the pact and action. Here, the According to the given passage, the National
statement is not mentioned, hence option E is Oceanic Atmospheric Administration based in
preferred. US have an observatory in Hawaii. That has
Therefore, option E is the correct answer. reported that the carbon emission level is
increasing when compared to the previous
9. Answer: B month. Through this, we can infer that the
As per the given passage, only statement II will carbon emission steps is not appreciable.
be discussed in the upcoming meeting. Therefore, option C is the correct answer.
Statement I is incorrect, it is irrelevant to the
given question, statement III is incorrect, 13. Answer: C
because the global efforts will be discussed Propellant – a fuel force or something that
based on what makes the countries inadequate propels.
to provide good results in the carbon emission Here, the blank requires a word that describes
control. the word “science”, because “science” is helping
Therefore, option B is the correct answer. humans to get progress. So, the word
“propellant” should be used. The second
10. Answer: D statement indicates that it works as a remedial
According to the given passage, statements I tool from overcoming the consequences, so the
and III are discussed in the passage. word “instrument” should be used. The third
Statement II is incorrect, because the next round statement says anti-microbial resistance is
of discussion should involve all the sectors of the becoming something to human survival and well-
economy, not any particular sector. being, here the word “threat” should be filled, as
Therefore, option D is the correct answer. the other words are grammatically incorrect.
Because of the misuse, microbes have emerged.

Click Here For Bundle PDF Course | support@guidely.in Page 10 of 12


Bank Po Mains PDF Course 2024
English Day - 2

Then, the remaining word “appeared” should be be used. Here, the fourth blank requires a word
filled in the fifth blank. that should describe the word “regulation” an
Therefore, option C is the correct answer. adjective is needed so the word “obsolete”
should be used. The word “blurring” should be
14. Answer: A used in the fifth blank.
Here, the passage is about driver’s safety and Therefore, option D is the correct answer.
the first sentence is expressed in a positive note.
So, we cannot use the word “ignore” and the 16. Answer: E
word “compliance” which is a noun, so “require” Here, the blanks should be filled with the
will be placed here. The second sentence says answers in the sequence I, II, III, IV and V.
that drivers should go in speed with great Because, eating contaminated food will result to
responsibility, they have to keep this in mind, for deadliest consequence. For that, the word “fatal”
that, the word “remember” should be used. Here, would come. To avoid this, we need to give
the word “the” is present which will precede the importance to food safety, for that the word
noun usually so it will take “compliance” in the “paramount” should be used. If the importance is
blank. The fourth statement says backseat given to food, it will not only help individuals but
passengers should also use this safety measure. also communities, for that “contribute” would be
From this, we can infer “ignore” should be placed correct. The fifth blank requires a noun so
here. The fifth blank will take “deploy” as the “incidence” should be placed and “proactive”
answer. should be placed at fourth blank.
Therefore, option A is the correct answer. Therefore, option E is the correct answer.

15. Answer: D 17. Answer: B


The second sentence says something is still Luring – tempt.
needed to move to the next level, so we can Anonymity – the condition of being unknown.
infer that flexibility is missing in the Indian Resorting – The act of turning to a person or
Education System, so the word “lacks” should be thing for help or as a means of achieving
used in the first blank. In 2023 Budget, something.
government has focused to reshape this, so the Here, only the first and the fourth blank will have
word “reorientation” should be used. The third base form verbs, as the word “to” precede. The
statement says that this is the first time word “penetrate” should be placed in the fourth
government is considering both, so it has given blank as the word “detect” is used. The word
importance, for that the word “emphasis” should “anonymity” should be used in the third blank as

Click Here For Bundle PDF Course | support@guidely.in Page 11 of 12


Bank Po Mains PDF Course 2024
English Day - 2

the word “provides” require a noun to complete Here, except C, all the statements are conveying
the sentence. How the customers get involved the same one. Statement A talks about the
into it, they are “lured” by social media. recommendation to the respective entity for
Therefore, option B is the correct answer. adding the mentioned site in the list. Statement
B talks about the site which should be added in
18. Answer: B the list. Statement D talks about the further
Here, except B, all the statements talk about the details of the city. Statement C talks about India
trial flight that was executed by Indian Council of will have 41 sites, it is redundant with the other
Medical Research. Statement A talks about who sentences. Therefore, option C is the correct
has launched the trail one and why it was answer.
launched. Statement C talks about more drone
flights will be deployed to check the quality of the 20. Answer: A
blood the trial flight carried. Statement D talks Here, except A all the statements talk about the
about what it has carried during the trail run. fake notice regarding the results of CBSE board
But,statement B talks about the government has exams. But statement A discusses about the
planned to provide funds to implement this universities and colleges. It is redundant to the
scheme, this is irrelevant to the other sentences. given question.
Therefore, option B is the correct answer. Therefore, option A is the correct answer.
Observation
19. Answer: C 1. Very good content... File can be transferred.

Click Here For Bundle PDF Course | support@guidely.in Page 12 of 12


Bank Po Mains PDF Course 2024
Reasoning Day -3 (Eng)

Reasoning Aptitude

Directions (1-5): Study the following information c) The one who is immediately senior to Minu
carefully and answer the below questions. d) Pihu
Seven medical persons – Jenu, Sinu, Minu, Kittu, e) None of these
Pihu, Rinku, and Katty are working in an
organization with different positions viz.- HOD, 2) How many persons are senior to the one who
Chief Surgeon, Surgeon, Senior Doctor, Doctor, likes Orange?
Nurse, and Ward Assistant. The designation of a) Three
the persons is given in decreasing order such b) As many persons junior to Jenu
that HOD is the seniormost whereas ward c) No one
assistant is the juniormost. Each person likes d) Four
different fruits viz.- Mango, Apple, Orange, e) None of these
Papaya, Guava, Grapes, and Kiwi. All the
information is not necessary in the same order. 3) Who among the following person is Chief
Only two persons are junior to the one who likes Surgeon?
Kiwi. Three persons are designated between the a) Rinku
one who likes Kiwi and Pihu. Sinu is three b) Pihu
positions senior to the one who likes Mango. c) The one who likes Grapes
Kittu is not designated as Surgeon. Katty is d) The one who likes Apple
senior to Minu and the one who likes Papaya but e) None of these
not senior to Kittu. Minu likes neither Mango nor
papaya. Sinu likes neither Papaya nor Grapes. 4) Four of the following five are alike in a certain
The number of persons designated between way as per the given arrangement and thus form
Jenu and the one who likes Apple is one less a group. Find the one that doesn’t belong to that
than the number of persons senior to the one group?
who likes Guava. Rinku doesn’t like Mango. a) Doctor – Rinku
Atleast one person is designated between the b) Kittu – Grapes
one who likes Guava and the one who likes c) HOD – Orange
Grapes. . The one who likes Grapes is not d) Nurse – Jenu
designated as HOD. e) Katty – Guava
1) Who among the following person likes Apple?
a) The one who is designated as Surgeon 5) Which of the following statement is not true?
b) Jenu a) Minu is a Doctor

Click Here For Bundle PDF Course | support@guidely.in Page 1 of 10


Bank Po Mains PDF Course 2024
Reasoning Day -3 (Eng)

b) Nurse doesn’t like Orange B. The committee’s report gave the proposal the
c) The one who likes Kiwi is a Ward Assistant go-ahead
d) Pihu is the HOD C. The UGC had to shelve the plan due to the
e) All the above statements are true Covid-19 pandemic
a) A is the Cause and C is the Effect
Directions (6-10): Read the given passage b) B and C are Causes of independent Effects
carefully and answer the questions based on the c) B is the Cause and C is its Effect
same respectively d) A and C are Effects of independent Causes
From the 2022-23 academic session, a common e) C is a complete Effect and Cause statement in
entrance test is likely to be implemented across itself
central universities in India for admissions to
undergraduate and postgraduate courses, 7) Which of the given options can be an
marking a departure from the current assumption as per the given passage?
predominant pattern of screening based on class a) The unrealistic cutoffs for admission to
12 marks. premier institutions like Delhi University have
On November 26, the University Grants underlined the need for a CET
Commission (UGC) wrote to the vice-chancellors b) The students have faced difficulties in getting
of the 45 central universities that “after detailed proper coaching for getting good marks in
deliberations, it was resolved that the Common Boards
Entrance Test for UG and PG may be conducted c) At present, the CUCET papers consist of two
for Central Universities from the academic segments that includes an aptitude test followed
session 2022-23 through National Testing by domain knowledge test
Agency. The committee’s report gave the d) There are criticisms that are being faced by
proposal the go-ahead, but the UGC had to the new pattern of exams that is under
shelve the plan due to the Covid-19 pandemic. discussion
The latest push came on November 22, when the e) None of the above
UGC held a meeting with the vice-chancellors of
45 central universities, following which the letter 8) Find the best conclusion as per the contents
was sent. of the passage discussed above?
6) Statements: a) The students will find it difficult to get
A. There are contemplations that it will create admissions in colleges due to the COVID
socio-economic disparities among students pandemic

Click Here For Bundle PDF Course | support@guidely.in Page 2 of 10


Bank Po Mains PDF Course 2024
Reasoning Day -3 (Eng)

b) The Common Entrance Test will be the level based on various parameters instead of just the
playing field which will help students get their marks
deserving colleges II. A common exam or selection process should
c) The year 2022-23 will be very crucial for the be introduced that can help bridge the gap and
upcoming educational reforms make the process look achievable
d) The coaching classes will benefit from the a) Only I
reforms by the committee recommendations b) Only II
e) Both (a) and (c) c) Both I and II
d) Either I or II
9) Choose the option that weakens the premise e) Neither I nor II
on which the CET is being proposed
a) The students come from different educational Directions (11-15): Read the following
and social backgrounds so it would be difficult for information carefully and answer the questions
them to compete given below.
b) The proposed changes will give rise to the Eight person namely-G, H, I, J, K, L, M, and N
unhealthy competition and largely benefit the are sitting in the linear facing north. Distance
coaching culture between adjacent persons is a consecutive
c) The students will understand the need of integral multiple of ‘9m’ from the left end. The
logical and analytical skills and improve distance between adjacent persons is neither
themselves to compete for the seats less than 10m nor more than ‘80m’
d) The level playing field will eliminate the barrier G sits 45m left of J. The distance between M and
that prevent certain students to get a college of G is twice the distance between H and K. G does
their choice not sit to the left of M. The distance between J
e) Both (a) and (b) and N is thrice of the distance between H and K.
The number of persons sits to the left of H is the
10) Statement: The current pattern of screening same as to the right of I. The one who sits
based on class 12 marks is predominant which second to the right of N is sitting immediate right
creates a problem for some students to get an of L.
admission in their desired universities After the above arrangement, K walks 20m
Course of Action: towards the north then took a left turn and walks
I. The admission criteria should be made more 8m to reach K’. H walks 20m towards the south
transparent and evaluation should be done then took the left turn and walks 10m to reach H’.
G walks 10m towards south then took left turn

Click Here For Bundle PDF Course | support@guidely.in Page 3 of 10


Bank Po Mains PDF Course 2024
Reasoning Day -3 (Eng)

and walks 40m to reach G’. J walks 15m towards c) Second


the north then took the right and walks 20m to d) Sixth
reach J’. L walks 30m towards the east then took e) Fifth
the right turn and walks 25m to reach L’.
11) Who among the following person sits exactly 15) What is the direction of G’ with respect to L’
between G and the one who sits immediate right in the final arrangement?
of L before the movement? a) East
a) M b) North-west
b) The one who sits second to the right of M c) North-east
c) K d) South-east
d) The one who sits immediate left of L e) South-west
e) None of the above
Directions (16-20): Study the following
12) What is the direction and distance of K’ with information carefully and answer the questions
respect to H’ in the final arrangement? given below:
a) 20m, south An alphanumeric machine when given an input
b) 30m, north rearranges them by following a particular rule in
c) 35m, south each step. The following is an illustration of the
d) 40m, south input and steps rearrangement:
e) 40m, north Input: corona 54 pandemic 38 isolated 62
quarantine 82 containment 94
13) What is the distance between J and L before Step –I: 94 54 pandemic 38 isolated 62
the movement? quarantine 82 containment corona
a) 189 m Step-II: 94 pandemic 54 isolated 62 quarantine
b)117 m 82 containment corona 38
c) 99 m Step-III: 94 pandemic 82 54 62 quarantine
d) 162 m containment corona 38 isolated
e) None of the above Step-IV: 94 pandemic 82 quarantine 62
containment corona 38 isolated 54
14) What is the position of N from the extreme Step-V: 94 pandemic 82 quarantine 62 corona
right end? 38 isolated 54 containment
a) Fourth Step 5 is the last and final step of the
b) Third rearrangement. With the same rules followed in

Click Here For Bundle PDF Course | support@guidely.in Page 4 of 10


Bank Po Mains PDF Course 2024
Reasoning Day -3 (Eng)

the above arrangement, determine the steps for a) 62


the following input. b) 55
Input: Errors 93 Fancy 55 Mango 65 Next 62 c) Mango
Rose 81 d) Next
16) Which of the following element is second to e) Errors
the left of “Next” in step III of the given input?
a) 55 19) What is the sum of the numbers between the
b) Fancy elements “Mango” and “Rose” in step III of the
c) Mango given input?
d) 81 a) 213
e) None of the above b) 198
c) 207
17) In which of the following step, the elements d) 239
“Next Fancy 62 Rose 65” obtained in the same e) 202
order?
a) Step I 20) How many numbers are there between the
b) Step II elements “Next” and “Rose” in step IV of the
c) Step III given input?
d) Step IV a) Two
e) There is no such step b) Three
c) Four
18) Which of the following element is sixth from d) Five
the left in step III of the given input? e) One

Click Here For Bundle PDF Course | support@guidely.in Page 5 of 10


Bank Po Mains PDF Course 2024
Reasoning Day -3 (Eng)

Click Here to Get the Detailed Video Solution for the above given Questions
Or Scan the QR Code to Get the Detailed Video Solutions

Answer Key with Explanation


Directions (1-5):  Sinu is three positions senior to the one
1) Answer: A who likes Mango.
2) Answer: C That means, in case (1) Sinu is a
3) Answer: C Surgeon, in case (2) Sinu is a senior
4) Answer: A doctor.
5) Answer: C Based on the above given information we have:

We have: Again, we have:


 Only two persons are junior to the one  Kittu is not designated as Surgeon.
who likes Kiwi.  Katty is senior to Minu and the one who
 Three persons are designated between likes Papaya but not senior to Kittu.
the one who likes Kiwi and Pihu.  Minu likes neither Mango nor papaya.

Click Here For Bundle PDF Course | support@guidely.in Page 6 of 10


Bank Po Mains PDF Course 2024
Reasoning Day -3 (Eng)

 Sinu likes neither Papaya nor Grapes.


That means, in case (1) & case (2) Minu
likes Kiwi.
Based on the above given information we have:

Again, we have:
 Atleast one person is designated between
the one who likes Guava and the one who
likes Grapes.
 The one who likes Grapes is not
designated as HOD.
Again, we have: That means, in case (1a) the one who is
 The number of persons designated HOD likes Orange, and Kittu likes
between Jenu and the one who likes Grapes, case (1) is not valid.
Apple is one less than the number of Based on the above given information we have:
persons senior to the one who likes
Guava.
 Rinku doesn’t like Mango.
That means, in case (1) Sinu likes Guava,
in case (1a) Sinu likes Apple, case (2) is
not valid.
Based on the above given information we have:

Case (1a) is not valid as the one who likes


Guava is designated adjacent designation of
either the one who likes Apple or Grapes.

Click Here For Bundle PDF Course | support@guidely.in Page 7 of 10


Bank Po Mains PDF Course 2024
Reasoning Day -3 (Eng)

6) Answer: E author’s contention and hence should be the


If we look at the given sentences and read the best options.
passage, we find that anything directly related to
A has not been discussed. So, it doesn’t fall well 10) Answer: C
in the Cause-Effect relationship. Also, B looks The given problem can be lessened or solved by
like an information and is not related to A or C as applying any of the two given COAs. So, both I
a Cause or an Effect. So, options other than (e) and II will be the correct answer.
can all be ruled out. Hence, option (c) is correct.
Hence, (e) is the correct answer.
Directions (11-15):
7) Answer: A 11) Answer: D
Of all the given statements, statement (a) can be 12) Answer: E
counted as an assumption as it provides a base 13) Answer: B
for the whole idea about why a common 14) Answer: B
examination is being planned at all. 15) Answer: B
Therefore, the best option is (a). Final Arrangement

8) Answer: B
Statement (b) can serve as the best conclusion
as the other options are too specific and can be
eliminated for being able to present a clear
closure or result to the passage.

9) Answer: E
The passage discusses the reasons and the
updates about the proposed changes in the
admissions to the elite colleges by the UGC. By We have,

far all the positive aspects have only been  G sits 45m left of J.

discussed which help us understand that the  The distance between M and G is twice

proposed changes are beneficial. the distance between H and K.G does not

If we look at the options, statements (a) and (b) sit left of M.

present a critical picture by pointing out the From the above condition, there are two

problems that might be created due to the possibilities.

changes. So, these would surely weaken the

Click Here For Bundle PDF Course | support@guidely.in Page 8 of 10


Bank Po Mains PDF Course 2024
Reasoning Day -3 (Eng)

 H walks 20m towards the south then took


the left turn and walks 10m to reach H’.
 G walks 10m towards south then took left
turn and walks 40m to reach G’.
 J walks 15m towards the north then took
the right turn and walks 20m to reach J’.
 L walks 30m towards the east then took
the right turn and walks 25m to reach L’.
Based on the above-given information we have
the final arrangement as follows.

Again we have,
 The distance between J and N is thrice of
the distance between H and K.
 The number of persons sits to the left of H
is the same as to the right of I.
 The one who sits second to the right of N
is sitting immediate right of L.
From the above condition, case2a get
eliminated. case2 shows the final arrangement.
Directions (16-20):
16) Answer: A
17) Answer: D
18) Answer: A
19) Answer: B
55 + 62 + 81 = 198
20) Answer: E
For words: The words are arranged in
alphabetical order based on the last letter. For
Again we have, each step, one word is shifted to the left and
 K walks 20m towards the north then took right alternatively, starting with the word at the
a left turn and walks 8m to reach K’. right end in step I.

Click Here For Bundle PDF Course | support@guidely.in Page 9 of 10


Bank Po Mains PDF Course 2024
Reasoning Day -3 (Eng)

For Number: The numbers are arranged in Step-II: 93 Mango Errors Fancy 55 Next 62 81
descending order based on its digital sum. For Rose 65
each step, one number is shifted to the left and Step–III: 93 Mango 55 Fancy Next 62 81 Rose
right alternatively, starting with the number at the 65 Errors
left end in step I. Step- IV: 93 Mango 55 Next Fancy 62 Rose 65
Input: Errors 93 Fancy 55 Mango 65 Next 62 Errors 81
Rose 81 Step-V: 93 Mango 55 Next 62 Rose 65 Errors 81
Step- I: 93 Errors Fancy 55 Mango 65 Next 62 fancy
81 Rose

Click Here For Bundle PDF Course | support@guidely.in Page 10 of 10


Bank Po Mains PDF Course 2024
Quantitative Aptitude Day -3 (Eng)

Quantitative Aptitude

Directions (01 - 04): Study the following information carefully and answer the questions given below.
The given pie chart shows the percentage distribution of the number of items [charger + headphone]
sold in five different shops [A, B, C, D and E]. The given table shows the percentage of chargers sold out
of the number of headphones sold, The Ratio of the number of wireless chargers and wire chargers sold,
and the ratio of the number of wireless chargers and wireless headphones sold in each shop. The Total
number of items sold in all shops together is 6000.

1)The number of wire headphones sold in number of wire headphones sold in shop C is n.
shop A is z more than the number of wireless find which of the following is true?
chargers sold in shop D. Number of wireless I. n=[z+m]
headphones sold in shop B is m more than the II. 3n=[2z+3m]
number of wireless chargers sold in shop E. The III.5n=2*[z+7m]+23

Click Here For Bundle PDF Course | support@guidely.in Page 1 of 13


Bank Po Mains PDF Course 2024
Quantitative Aptitude Day -3 (Eng)

a) All true in Shop E. If the total of 456 headphones and


b) Only II and III true 657 chargers sold in shop F then find the
c) All false difference between the total no. of wired
d) Only II false headphone and the total number of wired
e) Only III false chargers sold in shop F?
a) 323
2) The average price of 40% of wire headphones b) 372
sold in shop C is Rs.455 and the average price c) 347
of the rest of the wire headphones is Rs.855. d) 368
The average price of 60% of wireless e) None of these
headphones is Rs.1050 and the average price of
the rest wireless headphones is Rs. 1250. Find 4) Find which of the following is true?
the total revenue earned by shop C by selling all I. Average number of wireless headphones sold
the headphones? in shops C and D is 449.
a) Rs.422350 II. Average number of wireless chargers sold in
b) Rs.402350 shops A and D is 315.
c) Rs.432350 III. Total number of wire headphones sold in all
d) Rs.442350 shops together is 1859.
e) Rs.412350 a) All true
b) All false
3) The no. of wireless headphones sold in Shop c) Only I false
F, is 20% more than the same in Shop E, and d) Only I true
the no. of wireless chargers sold in Shop F, is e) None of these
10% less than the no. of wireless chargers sold

Directions (05 - 08): Study the following information carefully and answer the questions given below.
The given line graph shows the ratio of boys and girls in class X and the ratio of girls and boys in class
XII in five different schools [A, B, C, D, and E].

Click Here For Bundle PDF Course | support@guidely.in Page 2 of 13


Bank Po Mains PDF Course 2024
Quantitative Aptitude Day -3 (Eng)

Note:
I. The number of girls in class XII of schools A, B, C, D and E is 56, 30, 60, 48 and 63 respectively.
II. The number of students [boys + girls] in class X of schools A and B is 94 and 38 more than the number
of students in class XIIof the same school.
III. The number of students in class X of schools C, D and E is 16,39, and 56 less than the number of
students in class XII of the same school.
5) The ratio of the total no. of students of class X e) None of these
take admission to class XI in stream science,
arts and commerce of school A is 4:5:1 and the 6) The ratio of the number of boys and girls in
ratio of same of school B is 3:2:3. Find the total class X of school F is 3:2 and the ratio of the
no. of students of class X take admission in number of boys and girls in class XII of school F
science stream of school A and B together is is 4:5. If the total number of boys in school F is 3
what per cent students of class X take admission more than the number of boys of school B and
in arts stream of school A and B together? number of girls in school F is 6 less than the
a) 92.17% number of girls in school D. Find the total
b) 92.5% number of students in school F?
c)93.80% a) 124
d) 95.64% b) 125

Click Here For Bundle PDF Course | support@guidely.in Page 3 of 13


Bank Po Mains PDF Course 2024
Quantitative Aptitude Day -3 (Eng)

c) 119 Directions (09-12): Study the following


d) 128 information carefully and answer the questions
e) 138 given below.
In four different cities [A, B, C, and D], different
7) numbers of government and private buses are
Quantity I: Find the difference between the running. The number of government buses
average number of boys in class XII in schools A running in city A is three less than the number of
and B together and the average number of girls government buses running in city C. The ratio of
in class X in schools B and D together? private and government buses running in city D
Quantity II: Find the difference between the is 2:3. The number of private buses running in
average number of boys in class X of schools A city A is five less than the number of government
and E together and the average number of girls buses running in city B. The number of private
in class XII of schools A and D together? buses running in city B is 40% more than the
a) Quantity: I Quantity: II number of government buses running in the
b) Quantity: I > Quantity: II same city. The ratio of private and government
c) Quantity: I < Quantity: II buses running in city C is 2:3. The number of
d) Quantity: I ≤ Quantity: II private buses running in city D is three less than
e) Quantity I = Quantity II the number of private buses running in city B.
The number of government buses running in city
8) Out of the total no. of students in class XII of D is 60% more than the number of government
school C, 40% of students take admitted to buses running in city A. The total number of
government colleges and the rest of the students government buses running in cities A and B
take admission to private colleges. The ratio of together is 55.
the no. of students take admission to private 9) The ratio of ac and non-ac buses out of the
colleges of schools C and D is 3:2. Find the total number of government buses running in city
percentage of students of class XII who take A is 3:2. The total number ac buses running in
admission to government college from school D city A is 24. Find the total number of private non-
out of the total no. of students in class XIIof ac buses is what per cent of the total number of
school D? government non-ac buses running in city A?
a) 52.35% a) 154.22%
b) 54.54% b) 159.33%
c) 44.44% c) 116.66%
d) 65.25% d) 120.33%
e) None of these e) None of these

Click Here For Bundle PDF Course | support@guidely.in Page 4 of 13


Bank Po Mains PDF Course 2024
Quantitative Aptitude Day -3 (Eng)

10) The no. of autos running in city B is 20% a) 14546


more than the no. of government buses running b) 14594
in the city, and the no. of taxis running in city B is c) 14654
20% more than the no. of private buses running d) 14824
in city B. If the total no. of autos running in the e) 14464
city B and D is 56, then find the difference
between the no. of autos running in D and the Directions (13 - 16): Study the following
number of taxis running in B? information carefully and answer the questions
a) 17 given below.
b) 19 Difference schemes provide either Simple
c) 16 interest or compound interest.
d) 13 Scheme A – Provide at 25% interest rate. If
e) None of these Rs.15000 invest for two years then the amount of
interest is Rs.8438.
11) The ratio of the no. of government buses Scheme B – Provide at 15% interest rate. If
running in cities A and F is 6:7, and the ratio of Rs.12320 invest for three years then the amount
the no. of private buses running in cities B and F of interest is Rs.5544.
is 7:8. The total no. of buses running in city G is Scheme C – Provide simple interest. If Rs.15000
20 more than the total no. of buses running in F. invest for two years then the amount of interest is
Find the difference between the no. of Rs.5400.
government buses running in cities G and B if Scheme D – Provide compound interest. If
the ratio of private and government bus running Rs.8000 invest for two years then the amount of
in G is 3:2? interest is Rs.3520.
a) 14 Scheme E – Provide at 10% interest rate. If
b) 19 Rs.13500 invest for two years then the amount of
c) 13 interest is Rs.2835.
d) 16 Scheme F – Provide at 16% interest rate. If
e) 18 Rs.12562 invest for three years then the amount
of interest is Rs.6029.76.
12) If x is equal to the total number of Scheme G – Provide at 30% interest rate. If
government buses running in all given cities Rs.20000 invest for two years then the amount of
together and y is equal to the total number of interest is Rs.13800.
private buses running in all given cities together. 13) A person has Rs.28000. some of the amount
Find the value of x*y? he invests in scheme A and some of the amount

Click Here For Bundle PDF Course | support@guidely.in Page 5 of 13


Bank Po Mains PDF Course 2024
Quantitative Aptitude Day -3 (Eng)

he invests in scheme B for 2 years. After 2 years, scheme E and Rs.18600 was invested in scheme
the interest amount is Rs.12600. Find the F for 2 years?
difference between the amounts invested in Quantity II: Find the difference between the
scheme A and scheme B? interest amount when Rs.10400 invests in
a) Rs.4500 scheme G and Rs.11600 invests in scheme B for
b) Rs.4000 2 years?
c) Rs.6400 a) Quantity: I Quantity: II
d) Rs.8400 b) Quantity: I > Quantity: II
e) None of these c) Quantity: I < Quantity: II
d) Quantity: I ≤ Quantity: II
14) Rs.12400 was invested in scheme C for t e) Quantity I = Quantity II
years and earned Rs.13392 as interest. IF Rs.
20400 was invested in scheme D for (t/2) years 16) Rs. _______ invest at scheme ______ for 3
find the interest amount? years and interest earned after 3 years is
a) Rs.14851 Rs.9913 (approx.) and Rs.15200 invest at
b) Rs.12546 scheme ____ for 2 years and after 2 years
c) Rs.14581 interest amount is Rs.4864. find which of the
d) Rs.15632 following value satisfy the blank?
e) None of these a) 10400, F, A
b) 10900, A, D
15) c) 10400, A, F
Quantity I: Find the difference between the d) 10400, A, G
interest amount when Rs.16400 was invested in e) 11400, B, D

Directions (17-20): Study the following information carefully and answer the questions given below.
The below funnel DI shows the recruitment process of an organisation and the posting of selected
students. In the Recruitment process, data was given about the number of students who applied, the
percentage of students who appeared, the number of students who qualified in round I, the percentage of
students who appeared in round II, and the number of students who were rejected in round II. In the post
about selected students, data is given about the percentage of students selected in different cities [A, B,
C, D, and E].

Click Here For Bundle PDF Course | support@guidely.in Page 6 of 13


Bank Po Mains PDF Course 2024
Quantitative Aptitude Day -3 (Eng)

Note- The total number of students selected is only five cities. All the percentages on the posting of
selected parts are calculated based on the total number of students selected in the second round. The
percentage of students who appeared is calculated on the number of students apply. The percentage of
students who appeared in round II is calculated on the number of students qualify round I. 10,000
students did not appear in the round I exam.
17) The ratio of male and female students posted e) 214.33%
in city A is 3:1 and the ratio of male and female
students posted in city B is 3:2. Find the total 18) Find which of the following is true?
number of males posted in city A and B together I. Difference between the number of students
is what percentage of total number of females who did not appear in Round I and the number of
posted in city A and B together? students who did not appear in Round II is 7000.
a) 214.52% II. Difference between the number of students
b) 215.37% rejected in the first round and the number of
c) 259.32% students rejected in secondround is 16400.
d) 218.18%

Click Here For Bundle PDF Course | support@guidely.in Page 7 of 13


Bank Po Mains PDF Course 2024
Quantitative Aptitude Day -3 (Eng)

III.Total number of students posted in cities D b) 1750


and E is 1450. c) 1680
III. d) 1950
a) Only I true e) None of these
b) Only II true
c) Only III true 20) The number of students rejected in round II
d) All false is ______% of the number of students posted in
e) None of these City E and the number of students posted in City
D is _____% of the number of students rejected
19) Out of the total number of students posted in in Round I.
city C, 60% are living in the company's hostel Find which of the following is satisfy the blank?
and the rest are living in the room rent. Out of the a) 625%, 4.32%
total number of students posted in city D, 25% b) 600, 4.32%
are living in company's hostel and the rest are c) 625%, 5.32%
living in the room rent. Find the total number of d) 550%, 5%
students living in room rent? e) None of these
a) 1800
Click Here to Get the Detailed Video Solution for the above given Questions
Or Scan the QR Code to Get the Detailed Video Solutions

Answer Key with Explanation


Directions (01 - 04): So, 160x=1320
The Number of items sold in shop A is Or, x=8.25
6000*22/100=1320 So, the number of headphones sold is
Let the number of headphones sold in A be =8.25*100=825
100x, The Number of chargers sold is =60*8.25=495
The Number of chargers sold is 60x.

Click Here For Bundle PDF Course | support@guidely.in Page 8 of 13


Bank Po Mains PDF Course 2024
Quantitative Aptitude Day -3 (Eng)

The Number of wire chargers sold is =


495*3/5=297 3) Answer: B
The Number of wireless chargers sold is =495- Number of wireless headphones sold in shop F
297=198 is 210*120/100=252
The Number of wireless headphones sold is Number of wireless chargers sold n shop F is =
=198*3/2=297 90*90/100=81
The Number of wire headphones sold is = 825- So, the required difference = [657-81] – [456-
297=528 252] =372
Similarly, we can calculate others' values also. 4) Answer: A
I. Average number of wireless headphones sold
in shops C and D is =[250+648]/2=449.
II. Average number of wireless chargers sold in
shops A and D is =[198+432]/2=315.
III. The Total number of wire headphones sold in
all shops together is
=[528+384+230+552+165]=1859.

1) Answer: C Directions (05 - 08):


So, z=528-432=96, m=216-90=126, n=230 The number of boys in class XII of school A is =
I. n=[z+m] 56*10/14=40
[z+m]=126+96=222, so, not true. The total number of students in class XII of
II. 3n=[2z+3m] school A is 56+40=96
2z+3m=2*96+3*126=570, 3n=690, so, it is not The Number of students in class X of school A is
true = 94+96=190
III. 5n=2*[z+7m]+23 The Number of boys in class X of school A is =
5n=1150 , 2*[z+7m]+23=2*[96+7*126]+23=1956 190*9/19=90
So, it is not true. The Number of girls in class X of school A is =
190-90=100
2) Answer: D Similarly, we calculate school B also.
The total revenue earned by shops C is The Number of boys in class XII of school C is
=[230*40/100]*455+[230*60/100]*855+[250*60/1 =60*10/15=40
00]*1050+[250*40/100]*1250 The Total number of students in class XII of
=92*455+138*855+150*1050+100*1250 school C is =60+40=100
=Rs.442350

Click Here For Bundle PDF Course | support@guidely.in Page 9 of 13


Bank Po Mains PDF Course 2024
Quantitative Aptitude Day -3 (Eng)

Total number of students in class X of school C By solving the above equation, we get x =13 and
is=100-16=84 y=6
The Number of boys in class X of school C is = So, the total number of students in school F is =
84*14/24=49 5*13+9*6=65+54=119
The Number of girls in class X of school C is =
84-49=35 7) Answer: C
Similarly, we can calculate the values of schools Quantity I
D, E also. So, difference is = [40+12]/2-[32+14]/2=26-23=4
Quantity II
So, difference = [90+42]/2- [56+48]/2=66-52=14
Quantity I < Quantity II

8) Answer: B
The Number of students of class XII take
admission to private colleges from C is =
[40+60]*60/100=60
The Number of students of class XII take

5) Answer: A admission to private colleges from D is

The total number of students take admission in 60*2/3=40

the science stream of schools A and B together The Number of students of class XII takes

is = [100+90] *4/10+ [48+32]*3/8=76 + 30 = 106 admission to government college from school D

The total number of students take admission in is =[40+48]-40=48

the arts stream of school B is = [48+32]*2/8 + So, required percentage = [48/88]*100=54.54%

[100+90] *5/10 =20 + 95 = 115


Required percentage = [106/115] *100=92.17% Directions (09-12):
Let's say the number of government and private

6) Answer: C buses running in C is 3x and 2x, respectively.

Let the number of boys and girls in class X of Let's, The Number of government and private

school F be 3x and 2x. buses running in D is between 3y and 2y.

The number of boys and girls in class XII of The number of government buses running in A is

school F is 4y and 5y. 3x-3.

So, 3x+4y=60+3=63 and 2x+5y=62-6=56 The number of government buses running in A is


3y*100/160=15y/8.

Click Here For Bundle PDF Course | support@guidely.in Page 10 of 13


Bank Po Mains PDF Course 2024
Quantitative Aptitude Day -3 (Eng)

So, 3x-3=15y/8 The number of private ac buses running in city A


Or, 8x-8=5y is = 24-18=6
Or, 8x-5y=8 The Number of private non-ac buses running in
The number of private buses running in B is = city A is = 20-6=14
[2y+3]. So, required percentage = [14/12]*100=116.66%
The number of government buses running B is =
[2y+3] *100/140 = [2y+3] *5/7. 10) Answer: C
So, [2y+3]*5/7+15y/8=55 The number of auto running in D is 56-
Or, 10y+15+105y/8=55*7 25*120/100=56-30=26
Or, 80y+120+105y=55*7*8=3080 The number of taxis running in B is =
Or, 185y=2960, y=2960/185=16 35*120/100=42
So, 8x=8+80=88 So, the required difference is = 42-26=16
So, x=88/8=11
The Number of government and private buses 11) Answer: C
running in C is 3* 11 = 33 and 2* 11 = 22. The number of buses running in city F is
Let, The Number of government and private =30*7/6+ 35*8/7=35+40=75
buses running in D is 3* 16 = 48 and 2* 16 = 32. The number of buses running in city G is =
The number of government buses running in A is 75+20=95
3x-3 = 3*11-3 = 30. So, the difference is 95*2/5-25=38-25=13
The number of government buses running in A is
3y*100/160=15y/8. 12) Answer: D
=15*16/8=30 So, x=30+25+33+48=136
The number of private buses running in B is = So, y=20+35+22+32=109
[2y+3] = 32 + 3 = 35. So, 136*109=14824
The number of government buses running B is =
[2y+3]. *5/7=35*5/7=25 Directions (13 - 16):
The number of private buses running in A is 25- So, in scheme A, 15000*25*2/100=7500
5=20. 15000*(1+25/100)2-15000=8438
9) Answer: C So, scheme A provides compound interest.
The number of government ac buses running in In scheme B, 12320*15*3/100=5544
city A is 30*3/5=18 So, scheme B provides simple interest.
Number of government non-ac buses running in Let scheme C provides r% interest
city A is 30-18=12 So, r=5400*100/ (15000*2) =18%

Click Here For Bundle PDF Course | support@guidely.in Page 11 of 13


Bank Po Mains PDF Course 2024
Quantitative Aptitude Day -3 (Eng)

Scheme C provides at 18% rate of interest. Scheme D provide compound interest and the
Let Scheme D provides at n% rate of interest. interest rate is 20%
So, 8000*(1+n/100)2=11520 Time is = 6/2=3 years
So, (1+n/100)2=[11520/8000]=1.44 So, interest amount is =20400*[1+20/100]3-
Or, [1+n/100) ^2= [1.2] ^2 20400= Rs.14851 (approx)
So, n=0.2*100=20%
Scheme E 13500*10*2/100=2700 15) Answer: C
So, scheme E provides compound interest. Quantity I
Scheme F, 12562*16*3/100=6029.76 Interest in scheme E is = 16400*[1+10/100]2-
So, scheme F provides simple interest. 16400 =19844-16400=3444
Scheme G, 20000*30*2/100=12000 Interest in scheme F is = 18600*16*2/100 =5952
20000*(1+30/100)2-20000=13800 So, difference = 5952-3444=2508
So, scheme G provides compound interest. Quantity II
13) Answer: B The difference of interest is
Let the person invest Rs. x at scheme A and Rs. = [10400*[1+30/100]2-10400]-11600*15*2/100
(28000-x) at scheme B. = 7176-3480=3696
So, interest earn from scheme A is Quantity I < Quantity II
x*(1+25/100)2-x=25x/16 – x =9x/16
Interest earns from scheme B is = [28000- 16) Answer: C
x]*15*2/100 Let's check options C,
= [28000-x]*3/10 Rs.10400 invest in scheme A,
So, 9x/16 + [28000-x]*3/10=12600 So, interest = 10400*[1+25/100]3-10400=9913
Or, 90x+28000*48-48x=126000*16 15200 invest in scheme F, so,
Or, 42x=672000, x=672000/42=16000 15200*16*2/100=4864
So, required difference is = [16000-(28000- So, option C is satisfied.
16000)] =4000
Directions (17-20):
14) Answer: A The number of students who appeared in the
Scheme C provides simple interest and an exam is = 50000-10000=40000
interest rate 18% So, R%= [40000/50000]*100=80%
So, 12400*t*18/100=13392 So, the number of students applying is 50000,
So, t=13392*100/ (18*12400) =6 The number of students who qualify for round I is
20000

Click Here For Bundle PDF Course | support@guidely.in Page 12 of 13


Bank Po Mains PDF Course 2024
Quantitative Aptitude Day -3 (Eng)

The Percentage of students who appeared in Required percentage =


round II is = R+10=90% [6912/3168]*100=218.18%
The Number of students who appeared in round
II exams is =20000*90/100=18000 18) Answer: B
The Number of students selected in the exam is I. Difference between the number of students
= 18000-3600=14400 who did not appear in round I and the number of
So, 14400 students posted in different cities. students who did not appear in round II is
The Number of students posted in city A is = =10000-2000=8000.
[14400*80*100/2] =5760 II. Difference between the number of students
So, M %+[( M/6)-1] %=( 100-40-6-20) % rejected in the first round and the number of
Or, 7M=34*6+6=210, M=210/7=30 students rejected in round II is =20000-
So, The Number of students posted in city B is = 3600=16400.
14400*30/100=4320 III.Total number of students posted in cities D
The Number of students posted in city C is = and E is 864+576=1440.
14400*20/100=2880 So, only II true
The Number of students posted in city D is =
14400*6/100=864 19) Answer: A
The Number of students posted in city E is = The total number of people living room rent is
14400-864-2880-4320-5760=576 = 2880*40/100+864*75/100=1800
17) Answer: D
The Total number of males posted in cities A 20) Answer: A
and B together is The number of students rejected in round II is
= [5760*3/4] + [4320*3/5] =4320+2592=6912 3600
The Total number of females posted in cities A So, percentage is = [3600/576]*100=625%
and B together is The number of students rejected in the round I is
= [5760+4320]-6912=3168 20000
So, percentage is = [864/20000]*100=4.32%

Click Here For Bundle PDF Course | support@guidely.in Page 13 of 13


Bank Po Mains PDF Course 2024
English Day - 3

English Language
Directions (1-6): In the following passage, some A. ceasefire
of the words have been made bold, each of B. allegation
which is preceded by a letter. Find the suitable C. reverence
word from the options given below that could D. blockade
replace the bold word so as to make the E. trouble
paragraph meaningful.
Killing a living animal is hard. At least, it should 2) Which of the following words will replace blank
be. There is a ___(A)___risk and respect for life (B)?
that a hunter should recognize when they choose A. poach
to harvest an animal. Indeed, many hunters do B. charge
learn this when they are taught how to hunt by an C. appear
elder. Unfortunately, not all hunters carry this D. associate
responsibility, and their actions can reflect poorly E. accuse
on the hunting community as a whole when they
purposelessly ____(B)____preserve wildlife. 3) Which of the following words will replace blank
These are the people that drive a (C)?
____(C)____retreat between those who choose to A. oust
hunt and those who choose not to. That respect B. withdraw
for wildlife is something each hunter must carry C. framework
and observe on their own. ____(D)____Hunting, D. revoke
on the other hand, is something that all hunters E. wedge
and nonhunters alike must do together.
As the top of the food chain, we all play a role in 4) Which of the following words will replace blank
____(E)____relating that wildlife is cared for and (D)?
protected to the best of our abilities. Humans are A. Negotiation
unique in that we’ve managed to control our B. Allegation
environment to a greater degree than any other C. Defiance
species could hope to. It is then our responsibility D. Conservation
to ensure we are managing wildlife in a way that E. Confrontation
is ____(F)____termination for its future, not
necessarily ours. 5) Which of the following words will replace blank
1) Which of the following words will replace blank (E)?
(A)? A. consulting

Click Here For Bundle PDF Course | support@guidely.in Page 1 of 11


Bank Po Mains PDF Course 2024
English Day - 3

B. ensuring
C. growing
D. fighting
E. willing

6) Which of the following words will replace blank


(F)?
A. aversion
B. irregular
C. combating
D. beneficial A.A-D, B-E
E. interrupting B.B-D, C-F
C.A-F, B-D
Directions (7-10): In the following questions two D.A-F, B-E
columns are given containing three E. None of these
Sentences/phrases each. In the first column,
sentences/phrases are A, B and C and in the 8)
second column the sentences/phrases are D, E
and F. A sentence/phrase from the first column
may or may not connect with another
sentence/phrase from the second column to
make a grammatically and contextually correct
sentence. Each question has five options, four of
which display the sequence(s) in which the
sentences/phrases can be joined to form a
grammatically and contextually correct sentence.
If none of the options given forms a correct
sentence after combination, mark (e), i.e. “None
A.A-F, B-D
of these” as your answer.
B.A-D, C-E
7)
C.A-E, C-F
D.A-D, C-F
E. None of these.

Click Here For Bundle PDF Course | support@guidely.in Page 2 of 11


Bank Po Mains PDF Course 2024
English Day - 3

9) Directions (11-17): Read the following passage


and answer the questions that follow. Some
words are given in bold to help you locate while
answering some of the questions.
Privatisation of the public sector, including banks,
has been part of the wish list of economic
reformers since 1991. This was at the core of the
‘Washington Consensus’. The private sector is
inherently more efficient. The ground realities of
democratic politics in India, however, kept
coming in the way of actual privatisation, though
A.C-E, B-D
progressive disinvestment of the shares of public
B.B-D, A-F
sector undertakings in the market has been
C.A-E, B-F
taking place over the years.The National
D.A-D, C-E
Democratic Alliance in the early 2000s did
E. None of these
undertake some privatisation but had to pause
because of political backlash. This was never in
10)
the United Progressive Alliance agenda. For six
years, Prime Minister Narendra Modi also did not
consider it important. But it is now being pursued
with vigour, has high priority with extremely
ambitious targets, (I) of the Thatcher era in the
United Kingdom.
India is right now going through its worst
economic crisis. The highest-ever contraction in
the economy took place last year, unemployment
has risen, incomes for growing numbers are

A. Only A-E falling, bank non-performing assets (NPAs) may

B.A-E, B-F and C-D be ballooning, and the fiscal deficit is rising. In

C. Only B-F these circumstances, it would be prudent to think

D.B-F and C-D through the pros and cons of the aggressive

E. None of these privatisation of public enterprises that is on


the nicety (II). There are three categories of

Click Here For Bundle PDF Course | support@guidely.in Page 3 of 11


Bank Po Mains PDF Course 2024
English Day - 3

public sector enterprises, with each needing its pursued on priority. Air India and the India
own analysis. Tourism Development Corporation (ITDC) hotels
There is the category of enterprises which have are good examples. But these need bold
been sick for a long time. Their technology, decisions. Air India should ideally be made debt
plants and machinery are obsolete. Their free and a new management should have
managerial and human resources have freedom permitted under the law in personnel
atrophied. Reviews have come to the same management to get investor interest. Once debt
conclusion: these are beyond redemption. They free; management control with a 26% stake may
should be closed, and assets sold. But this has be given. As valuation rises, the Government
been difficult with successive governments as could reduce its stake further and get more
the labour in these enterprises have had a money. If well handled, significant revenues
political constituency which has prevented would flow to the Government.
closure. With its political strength, the 11) Why the author opines that privatisation is a
Government should be able to close these in a part of economic reformers’ wish-list and core of
time-bound manner with a generous handshake the ‘Washington Consensus’?
for labour. After selling machinery as scrap, there (I) Progressive disinvestment in PSUs
would be valuable land left. These land values (II) An inherent efficiency
have become high. Prudent disposal of these (III) Ambitious targets on high priority
plots of lands in small amounts would yield large A. Only I
incomes in the coming years. All this would need B. Only II
the creation of dedicated efficient capacity as the C. Only III
task is huge and challenging. These enterprises D. Both I and II
may be taken away from their parent line E. Both II and III
Ministries and brought under one holding
company which should have the sole mandate of 12) What are the indicators supporting the fact
speedy liquidation and asset sale. that India is going through the worst economic
Then there are enterprises which have been crisis?
financially sick but can be turned around. Their (I) Highest ever economic contraction
difficulties can be traced to ministerial (II) Rise in unemployment and fiscal deficit
micromanagement especially in enterprises with (III) Declination income for growing number
a direct consumer interface. Where private (IV) Rapid increase in banks’ NPAs
management through privatisation or induction of A.I, II, IV
a strategic partner is the best way to restore B.II, III, IV
value of these enterprises, this should be C.I, III, IV

Click Here For Bundle PDF Course | support@guidely.in Page 4 of 11


Bank Po Mains PDF Course 2024
English Day - 3

D.I, II, and III E. Spangle


E. All I, II, III, and IV
16) Which of the following words can replace the
13) What should be the Government’s steps to word ‘NICETY’ highlighted in (II) to make the
manage the enterprises being sick for a long sentence grammatically correct and the
time? paragraph contextually meaningful?
A. Using political strength to close them with A. Aggrieve
proper labour compensation B. Anvil
B. Selling machinery and extracting valuable land C. Urbane
C. Utilizing high land values and disposing the D. Blighted
lands prudently E. Defray
D. Bringing the enterprises under one company
to enable speedy liquidation and asset sale 17) Which of the following words is similar in
E. All of the above meaning to “OBSOLETE” as highlighted in the
passage?
14) What ‘bold decisions’ are needed when the A. Discontinued
author talks about Air India and ITDC? B. Contemporary
(I) Make them debt free C. Modern
(II) Allowing revenues to flow to the government D. Updated
(III) Giving them permission to get investor E. Renewed
interest
A. Only I Directions (18-21): In the following questions, a
B. Only II sentence consists of two blanks. Choose the
C. Only III most suitable pair of words from the given
D. Both I and II alternatives to fill in the blanks. If none of the
E. Both II and III pairs is correct, choose ‘none of these’ as your
answer.
15) Which of the following can fill in the blank (I) 18) According to Campaign Report, rural areas
to make the sentence grammatically correct and are more affected by lighting than urban areas,
the paragraph contextually meaningful? which is __________ to the rural pattern of
A. Distend working in open ___________ spaces and lack of
B. Glib awareness among people.
C. Rejoinder A. prepared, agitated
D. Reminiscent B. attributed, vulnerable

Click Here For Bundle PDF Course | support@guidely.in Page 5 of 11


Bank Po Mains PDF Course 2024
English Day - 3

C. oriented, worsen Peninsula and ____________ the implementation


D. dedicated, vengeful of a tension reduction agreement.
E. None of these A. representation, militate
B. negotiation, fraught
19) The agreement called the Comprehensive C. prominence, blockade
Military Agreement (CMA), ___________ a series D. restoration, expedite
of tension-reducing measures, such as a E. None of these
___________ to hostile acts against each other.
A. envisages, halt 21) Rammohun Roy opposed the establishment
B. forecasts, incites of a legislative council for India as the British
C. expected, provoking Parliament could make a dispassionate
D. forewarns, obstacle ___________ of India’s needs and ____________.
E. None of these A. criticism, aspersions
B. defamation, equipment
20) The ministry said the latest ___________ is C. assessment, aspirations
expected to ease military tensions on the Korean D. hindrance, paramilitary
E. None of these
Click Here to Get the Detailed Video Solution for the above given Questions
Or Scan the QR Code to Get the Detailed Video Solutions

Answer Key with Explanation


1) Answer: C Thus, ‘reverence’ fits correctly in the blank as it
In the given passage, the blank is immediately means ‘deep respect for someone or something.’
followed by the conjunction ‘and’. ‘and’ connects Therefore, option C is the correct answer.
two similar ideas or words. So, we need a word
that is similar to the word ‘respect’. 2) Answer: A

Click Here For Bundle PDF Course | support@guidely.in Page 6 of 11


Bank Po Mains PDF Course 2024
English Day - 3

As the given sentence is talking about hunting as it means ‘resulting in good; favourable or
animals, the word ‘poach’ fits correctly in the advantageous.’
blank as it means ‘illegally hunt or catch (game Therefore, option D is the correct answer.
or fish) on land that is not one's own or in
contravention of official protection.’ 7) Answer: C
Therefore, option A is the correct answer. Part A ends with two dates, mentioning a
residential school system. Part F talks about
3) Answer: E residential schools that were funded by the
‘Drivea wedge’ is a phrase which means ‘to federal government, which pairs it correctly with
cause disagreement or anger between (people part A. Similarly, part B ends with the
who had been friendly before)’. Reconciliation Commission. Part D begins with
Therefore, option E is the correct answer. documenting the history and the lasting impact,
which pairs it correctly.
4) Answer: D From the given options, only A-F, B-D are
As the given passage is talking about respecting matched and make a meaningful sentence. The
and preserving wildlife, the word ‘conservation’ correct answer after matching will be:
fits correctly in the blank as it means (A-F): Canada’s residential school system was a
‘preservation, protection, or restoration of the telling example of how indigenous people were
natural environment and of wildlife’. treated. Between 1882 and 1996, an estimated
Therefore, option D is the correct answer. 150,000 children were sent to residential schools
that were funded by the federal government and
5) Answer: B run mostly by the Catholic Church.
As the given sentence is talking about the role (B-D): In 2008, Prime Minister Stephen Harper
played by us in making sure that the wildlife is issued a formal apology. The Government also
cared for and protected, the word ‘ensuring’ fits set up a Truth and Reconciliation Commission to
correctly in the blank as it means ‘make certain document the history and the lasting impact of
that (something) will occur or be the case.’ the school system on the indigenous
Therefore, option B is the correct answer. communities.
Therefore, option C is the correct answer choice.
6) Answer: D
As the given sentence is talking about managing 8) Answer: D
wildlife in a way that is useful for its future, the Part A ends with systemic discrimination and
word ‘beneficial’ fits correctly in the given blank violence and talks about the discovery of a

Click Here For Bundle PDF Course | support@guidely.in Page 7 of 11


Bank Po Mains PDF Course 2024
English Day - 3

hundred graves. Part D also mentions graves part D is mostly similar, it does not pair correctly
being discovered, which pairs it correctly. because of the presence of errors like ‘hopefully’
Similarly, part C talks about a saying by and ‘aspect’ and ‘short-hall’ which make no
Canada’s first Prime Minister, Sir John A. sense. Similarly, part B ends with ‘will depend’
Macdonald. Part F talks about the saying by the which needs to be followed by a preposition.
Prime Minister, which pairs it correctly. Part F begins with a preposition and connects
In (A-E) has is used instead of have both the sentences appropriately.
(A-E) After as many as 26 students tested From the given options, only A-E, B-F are
positive for Covid-19, the Basavanahalli High matched and make a meaningful sentence. The
School and College in Karnataka has suspended correct answer after matching will be:
offline classes. (A-E): While resumption will depend on the
From the given options, only A-D, C-F are behaviour of the coronavirus and the progress in
matched and make a meaningful sentence. The vaccination in India and other countries, industry
correct answer after matching will be: insiders are hopeful of an opening by the end of
(A-D): The recent discovery of hundreds of 2021 and expect the demand to be higher for
graves in Canada has put the spotlight back on short-haul international travel in the beginning.
its dark past, when indigenous people faced (B-F): We anticipate that international short-haul
systemic discrimination and violence. In June, is likely to see an uptick before long-haul, but
the Cowessess First Nation, an indigenous this will depend upon the destinations opening
organisation, found 751 unmarked graves at the up as well,” said Indiver Rastogi, group head –
site of a former residential school in global business travel, at Thomas Cook.
Saskatchewan. Therefore, option C is the correct answer choice.
(C-F): In 1883, Canada’s first Prime Minister, Sir
John A. Macdonald, defended in Parliament the 10) Answer: C
system of residential schools, saying indigenous In (A-E) Revival is used instead of revive
students should be kept away from their parents, In (C-D) its is used instead of their
“who are savages”. (A-E) India on Friday said it stood for revival of
Therefore, option D is the correct answer choice. democracy in Myanmar and argued that
Violence would not solve the prevailing situation
9) Answer: C in the country.
Part A ends with ‘are’ and part E begins with (B-F) State-owned oil marketing companies
‘hopeful’ which connects the building suspense (OMCs) may find it hard to comply with the
of the sentence and pairs correctly. Although

Click Here For Bundle PDF Course | support@guidely.in Page 8 of 11


Bank Po Mains PDF Course 2024
English Day - 3

government’s recent order to cut crude imports Read the third paragraph, you can draw the hint
from West Asia, especially Saudi Arabia. from,
(C-D) The ministry of road transport and With its political strength, the Government
highways exceeded its highway construction should be able to close these in a time-bound
target for the fiscal year 2021 despite the manner with a generous handshake for labour.
pandemic-led disruptions and nationwide After selling machinery as scrap, there would be
lockdown during the first quarter when valuable land left. These land values have
construction activity came to a standstill. become high. Prudent disposal of these plots of
lands in small amounts would yield large
11) Answer: C incomes in the coming years. All this would need
Read the lines of first paragraph, the hint can be the creation of dedicated efficient capacity as the
drawn from the lines, task is huge and challenging. These enterprises
Privatisation of the public sector, including may be taken away from their parent line
banks, has been part of the wish list of economic Ministries and brought under one holding
reformers since 1991. This was at the core of the company which should have the sole mandate of
‘Washington Consensus’. The private sector is speedy liquidation and asset sale.
inherently more efficient. Hence, option E is the right answer choice.
Hence, option C is the right answer choice.
14) Answer: D
12) Answer: E Refer to the fourth paragraph, the hint can be
Read the lines in the second paragraph; you can drawn from the lines, Air India and the India
draw the hint from the lines quoted below Tourism Development Corporation (ITDC) hotels
India is right now going through its worst are good examples. But these need bold
economic crisis. The highest-ever contraction in decisions. Air India should ideally be made debt
the economy took place last year, free and a new management should have
unemployment has risen, incomes for growing freedom permitted under the law in personnel
numbers are falling, bank non-performing assets management to get investor interest.
(NPAs) may be ballooning, and the fiscal deficit Hence, option D is the right answer choice.
is rising.
Hence, option E is the right answer choice. 15) Answer: D
Reminiscent: (a) tending to remind one of
13) Answer: E something.

Click Here For Bundle PDF Course | support@guidely.in Page 9 of 11


Bank Po Mains PDF Course 2024
English Day - 3

(b) absorbed in or suggesting absorption in According to Campaign Report, rural areas are
memories. more affected by lighting than urban areas,
(c) suggesting something by resemblance. which is attributed to rural patternsof working in
Here, the context of the blank is similar to the open vulnerable spaces and lack of awareness
meaning given in (a) (i.e. reminding of the among people.
Thatcher Era). Hence, option D is the right Attributed: regard something as being caused
answer choice. by.
Distend: swell from or as if from internal Vulnerable: exposed to the possibility of being
pressure attacked or harmed, either physically or
Glib: having only superficial plausibility emotionally.
Rejoinder: a quick reply to a question or remark 19) Answer: A
Spangle: adornment consisting of a small piece The agreement, called the Comprehensive
of shiny material Military Agreement (CMA), envisages a series of
tension-reducing measures, such as a halt to
16) Answer: B hostile acts against each other.
On the Anvil: to be in a state of discussion, Envisages: contemplate or conceive of as a
formation, or preparation possibility or a desirable future event.
Nicety: conformity with some standard of Halt: bring or come to an abrupt stop.
correctness or propriety
Aggrieve: infringe on the rights of 20) Answer: D
Urbane: showing a high degree of refinement The ministry said the latest restoration is
Blighted: affected by something that prevents expected to ease military tensions on the Korean
growth or prosperity Peninsula and expedite the implementation of a
Defray: bear the expenses of tension reduction agreement.
Hence, option B is the right answer choice. Restoration: the action of returning something to
a former owner, place, or condition.
17) Answer: A Expedite: make (an action or process) happen
Obsolete: no longer produced or used; out of sooner or be accomplished more quickly.
date.
Hence, option A is the right answer choice. 21) Answer: C
Rammohun Roy opposed the establishment of a
18) Answer: B legislative council for India as the British

Click Here For Bundle PDF Course | support@guidely.in Page 10 of 11


Bank Po Mains PDF Course 2024
English Day - 3

Parliament could make a dispassionate Aspirations: a hope or ambition of achieving


assessment of India’s needs and aspirations. something.
Assessment: the action of assessing someone
or something.

Click Here For Bundle PDF Course | support@guidely.in Page 11 of 11


Bank Po Mains PDF Course 2024
Reasoning Day -4 (Eng)

Reasoning Aptitude
Directions (1-5): Study the following information c) Only I
carefully and answer the given questions. d) Only I and III
Ten persons - A, B, C, D, E, F, G, I, O, and U are e) Only II
living on different floors of a twelve storey
building where the lowermost floor is numbered 2. How many persons live between the two
one and the floor immediately above it is vacant floors as per the given arrangement?
numbered two and so on. No two persons live on a) Three
the same floor. b) As many persons live between D and F
Note 1: If the persons whose name is a vowel, c) Four
then they are living on the even numbered floors, d) Five
while others are living on the odd numbered e) As many persons live between C and A
floors.
Note 2: There are two vacant floors. None of the 3. Who among the following person lives on the
adjacent floors are vacant. Neither the topmost third floor?
nor the lowermost floor is vacant a) B
Not more than three floors are above C. At least b) The one who lives three persons below D
five persons live between C and G. Only four c) D
floors are between G and A. The number of d) The one who lives immediately above E
floors between A and B is three more than the e) G
number of floors between E and I. E lives above
B. D lives three floors below I. The number of 4. In which of the following the sum of the floor
persons living above D is two more than the number is Maximum?
number of persons living below F. Not more than I. I and O
one person lives between O and U, who lives II. C and F
below O. III. E and U
1. If Z occupies one of the vacant floors, then a) Only III
how many persons live above O as per the given b) Only I and II
arrangement? c) Only II
I. As many persons live below C d) Only I and III
II. Six e) Only I
III. Seven
a) Only III
b) Only I and II

Click Here For Bundle PDF Course | support@guidely.in Page 1 of 9


Bank Po Mains PDF Course 2024
Reasoning Day -4 (Eng)

5. If B is related to D and O is related to G in a


certain way, then who among the following
person is related to A?
a) F
b) C
c) E
d) U
e) None of these

Direction (6-10): Study the following statements a) Only AP follows


and then decide which of the given conclusions b) Both BR and CQ follow
logically follows from the given statements c) Only CQ follows
disregarding the commonly known facts. d) Only BQ follows
6. e) Both AR and CP follow

8.

a) Only AR follows
b) Both BP and CR follow a) Only BQ follows

c) Only AQ follows b) Both AR and CP follow

d) Both AQ and CR follow c) Both BR and AP follow

e) Only BP follows d) All AQ, BR and CP follow


e) Only BR follows

7.
9.

Click Here For Bundle PDF Course | support@guidely.in Page 2 of 9


Bank Po Mains PDF Course 2024
Reasoning Day -4 (Eng)

Directions (11-15): Study the following


information carefully and answer the given
questions.
“A % B (37)” means “A is 22m north of B”
“A # B (18)” means “A is 26m east of B”
“A @ B (25)” means “A is 13m south of B”
“A & B (48)” means “A is 25m west of B”
If “A %# B” means “A is northeast of B”
Statement:
U%V (33); Q#R (8); S@Q (36); P@R (24); W#V
(9); Q&V (44); T&U (50); Y@W (31).
a) Both BP and CQ follow
11. If X&T (33) then what is the direction and
b) Only BR follows
shortest distance of X with respect to R?
c) Both AQ and BR follow
a) South, 18m
d) Only CP follows
b) North, 18m
e) All CR, AQ and BP follow
c) South, 16m
d) North, 16m
10.
e) None of these

12. What is the direction of T with respect to S?


a) T%S
b) T@&S
c) T%&S
d) S@&T
e) S%#T

13. Four of the following five pairs of points are


alike in a certain way based on the directions in
a) Both CP and AQ follow the given arrangement and thus form a group.
b) Only CP follows Which one of the following does not belong to
c) Only AP follows the group?
d) Both BR and AQ follow a) QP
e) None follows b) VS

Click Here For Bundle PDF Course | support@guidely.in Page 3 of 9


Bank Po Mains PDF Course 2024
Reasoning Day -4 (Eng)

c) UV is between 21 and 65 years. Not more than one


d) TR person is working in each Probationary officer
e) WS post. At least one person and not more than two
persons work in each Clerk post.
14. If Z%&S and the distance between Q and Z Note: If it is given that the person who is a
is 10m less than the distance between Q and V Probationary officer or Clerk works with person X
in the final arrangement, then what is the or the one whose age is Y, then it indicates that
shortest distance between Z and P? both persons work in the same bank but not
a) 12m necessarily at the same post. If it is given that X
b) 11m works with one whose age is “Y” years, then X’s
c) 9m age is not “Y” years.
d) 13m N, who works in UCO bank, neither works in the
e) 14m same bank nor in the same post as K. K works
with the one whose age is 28 years but not in
15. If K@#T and K@U, then what is the direction PNB bank. The age of M is a perfect square and
of K with respect to V? works with P in the same post but works neither
a) East in Indian bank nor in UCO bank. L is 31 years old
b) North and does not work in PNB bank. The difference
c) West between the ages of L and the one who works in
d) South PNB bank is 29 years. The sum of the ages of M
e) Either b or d and the one who works with M is 90 years. The
one who works in Indian Bank is 15 years
Directions (16-20): Study the following younger than P. The age of N is 31 years elder
information carefully and answer the given than O. O is not working in UCO bank. I is
questions. younger than J and the difference between the
Eight persons – I, J, K, L, M, N, O and P are ages of them is 6 years.
working in three different banks viz., Indian Bank, 16. Which of the following persons work in
UCO Bank, and Punjab National Bank (PNB) at Punjab National Bank (PNB) bank?
different posts – Probationary officer (PO) and I. The one whose age is 60 years
Clerk. They are all of different ages. The age of II. I
the person who is a Probationary officer is a III. The one whose age is 41 years
prime number and the age of the person who is a a) Only III
Clerk is a composite number. Each person’s age b) Only I and II

Click Here For Bundle PDF Course | support@guidely.in Page 4 of 9


Bank Po Mains PDF Course 2024
Reasoning Day -4 (Eng)

c) Only II 19. Which of the following statement(s) is/are


d) Only I and III true as per the given arrangement?
e) All I, II and III I. I is four years younger than L
17. How many persons are younger than the one II. The age of P is 60 years
who works in UCO bank as PO? III. Only two persons work in UCO Bank
a) Five a) Only III
b) Four b) Only I and II
c) Six c) Only II and III
d) Three d) Only I
e) None e) All I, II and III

18. What is the sum of the ages of the persons 20. Who among the following person is not
who work in Indian Bank? working as a Clerk?
a) 104 years a) P
b) 102 years b) J
c) 105 years c) K
d) 100 years d) L
e) 108 years e) Both b and d

Click Here to Get the Detailed Video Solution for the above given Questions
Or Scan the QR Code to Get the Detailed Video Solutions

Answer Key with Explanation


Directions (1-5): 3. Answer: B
1. Answer: D 4. Answer: D
2. Answer: B 5. Answer: E

Click Here For Bundle PDF Course | support@guidely.in Page 5 of 9


Bank Po Mains PDF Course 2024
Reasoning Day -4 (Eng)

Final arrangement: After applying the above conditions, case 1a


gets eliminated because more than three floors
between A and B.

 Not more than three floors are above C.


 At least five persons live between C and
G.
 Only four floors are between G and A.
After applying the above conditions, there are Again, we have
four possibilities.  The number of persons living above D is
two more than the number of persons
living below F.
 Not more than one person lives between
O and U, who lives below O.
After applying the above condition, case 1 and
case 2 are eliminated because the number of
persons living above D is two more than the
number of persons living below F is not satisfied,
hence case 2a shows the final arrangement.

Again, we have
 The number of floors between A and B is
three more than the number of floors
between E and I.
 E lives above B.
 D lives three floors below I.

Click Here For Bundle PDF Course | support@guidely.in Page 6 of 9


Bank Po Mains PDF Course 2024
Reasoning Day -4 (Eng)

10. Answer: B
6. Answer: D

Directions (11-15):
% - (37m-22m) – (-15m)
# - (18m-26m) – (+8m)
7. Answer: C
@ - (25m-13m) – (-12m)
& - (48m-25m) – (-23m)

8. Answer: E

9. Answer: E

Click Here For Bundle PDF Course | support@guidely.in Page 7 of 9


Bank Po Mains PDF Course 2024
Reasoning Day -4 (Eng)

11. Answer: B After applying above conditions, there are two


12. Answer: C possibilities.
13. Answer: C (First point is northeast of second
point except option c)
14. Answer: D
If Z%&S,
The distance between Z and Q is 11m and Z is
in west of Q. Again, we have
(PZ)2 = (ZR)2 + (RP)2  L is 31 years old and does not work in PNB
= (5)2 + (12)2 bank.
= 25 + 144  The difference between the ages of L and the
(PZ)2 = 169 one who works in PNB bank is 29 years.
PZ = 13m  The sum of the ages of M and the one who
15. Answer: E works with M is 90 years.
After applying the above conditions, the age of
Directions (16-20): the one who works in PNB is 60 years because
16. Answer: D the difference between the ages of L and the
17. Answer: C one who works in PNB bank is 29 years and also
18. Answer: A the only possible age of M is 49 years and the
19. Answer: C one who works with M is 41 years because the
20. Answer: E sum of the ages of M and the one who works
Final arrangement: with M is 90 years and M’s age is a perfect
square.

We have,
 N, who works in UCO bank, neither works in
the same bank nor same post as K.
 K works with the one whose age is 28 years
Again, we have
but not in PNB bank.
 The one who works in Indian Bank is 15
 The age of M is a perfect square and works
years younger than P.
with P in the same post but works neither in
 The age of N is 31 years elder than O.
Indian Bank nor in UCO bank.

Click Here For Bundle PDF Course | support@guidely.in Page 8 of 9


Bank Po Mains PDF Course 2024
Reasoning Day -4 (Eng)

 O is not working in UCO bank. P and also the age of N and O is 59 years and
 I is younger than J and the difference 28 years respectively because the age of N is 31
between the ages of them is 6 years. years elder than O and O is not working in UCO
After applying the above conditions, case 2 gets bank.
eliminated because the age of N is 31 years
elder than O, hence the possible age of N is 72
years but the age of each person is not more
than 65 years.
In case 1, the one who works in Indian bank is
45 years old because the one who works in
Indian Bank is 15 years younger than the age of

Click Here For Bundle PDF Course | support@guidely.in Page 9 of 9


Bank Po Mains PDF Course 2024
Quantitative Aptitude Day -4 (Eng)

Quantitative Aptitude

Directions (1-4): Study the following data carefully and answer the questions:
A shopkeeper sold different units of five different items (A, B, C, D, and E) on two different days Monday
and Tuesday.
The scatter graph given below shows the ratio of the number of units sold on Monday to the per unit cost
of each item and the ratio of number of units sold on Tuesday to the per unit cost of each item.

The data given below is related to the number of units of all 5 items sold on Tuesday.
The ratio of the number of sold units of item A to that of item D is 4: 7, and the ratio of the number of sold
units of item B to that of item E is 5: 4. The number of sold units of item C is 7 more than that of item D.
The number of sold units of item D is 38 less than that of item E, and the number of sold units of item A is
95 less than that of item B.
1) Find the total amount received by the A: The ratio of the number of sold units of item A
shopkeeper on Monday by selling the given units on Monday to that of item E on Monday is 16:17.
of items B, C, and D? B: The average per unit cost of items C, D, and E
a) ₹3218 is ₹10.
b) ₹3018 C: The total amount received by selling all the
c) ₹3518 given units of item E on Tuesday is ₹1934.
d) ₹3618 a) Only A and B
e) ₹3418 b) Only C
2) Which of the following is/are not true? c) None is true

Click Here For Bundle PDF Course | support@guidely.in Page 1 of 14


Bank Po Mains PDF Course 2024
Quantitative Aptitude Day -4 (Eng)

d) Only B and C
e) Only A

3) The total amount received by selling all the


given units of item D on Tuesday is what percent
more or less than that received by selling all the
given units of item A on Tuesday?
a)
b) Note:
c) 1: If the speed of bus B was 8 km/h more than its
d) original speed and the speed of bus D was 8
e) None of these km/h less than its original speed, the distance
covered by bus B would have been 8 km more
4) Find the ratio of the total units sold of items D than that covered by bus D in the same time
and E together on Monday to the total units sold duration as given in the table.
of items B and E together on Tuesday? 5) If bus A started from bus stand P at 12:03 PM,
a) 15: 26 then at what time (approx) it will reach bus stand
b) 3: 7 S?
c) 9: 16 a) 3:47 PM
d) 5: 9 b) 3:07 PM
e) 27: 40 c) 3:17 PM
d) 3:37 PM
Directions (5-8): Study the following data e) 3:27 PM
carefully and answer the questions:
Four buses A, B, C, and D, are operated from 6) Another bus E is chasing bus C, which is 165
bus stand P to bus stands Q, R, S, and T m ahead of bus E. If bus E crosses bus C in 22
respectively, and the speed of each bus is seconds, then find the speed of bus E?
different. a) 80 km/hr
The table given below shows the following data: b) 71 km/hr
c) 68 km/hr
d) 65 km/hr
e) 77 km/hr

Click Here For Bundle PDF Course | support@guidely.in Page 2 of 14


Bank Po Mains PDF Course 2024
Quantitative Aptitude Day -4 (Eng)

7) If the average speed of buses A and B is M, and also doubles that in April, which is 37.5% of
and the average speed of buses C and D is N, that in May.
then find the ratio of M to N? The average time spent studying in January was
a) 27: 26 0.6 hours less than that in February, which was
b) 135: 134 2.2 hours more than the average time spent
c) 15: 14 exercising in February. The difference between
d) 81: 80 the average time spent studying in January and
e) 33: 32 March is 1.2 hours, and the difference between
the average time spent studying in February and
8) After reaching bus stand T, bus D has to go to March is 1.8 hours. Average time spent studying
bus stand R from bus stand T. If the speed of in March is 48% of that in April, and the ratio of
bus D has decreased to 85% of its original speed average time spent studying in April to that in
due to some technical issue and it reaches bus May is 5:4.
stand R in 3.5 hours, then find the distance 9) Find the average time spent studying in
between bus stands R and T? January, February, and March together, if there
a) 204 km are 28 days in February.
b) 255 km a) hours
c) 238 km b) hours
d) 289 km c) hours
e) None of these d) hours
e) hours
Directions (9-12): Study the following data
carefully and answer the questions:
10) Find the ratio of the average time spent
The data given below is related to the average
exercising to that spent studying in the month, in
time per day spent on exercising and the
which the average time spent exercising is
average time per day spent on studying by a
maximum?
person in five different months: January,
a) 5: 8
February, March, April, and May.
b) 6: 5
The ratio of average time spent exercising in
c) 9: 25
January to that in March is 5:6, and the ratio of
d) 3: 2
average time spent exercising in February to that
e) 8: 3
in May is 1:3. The average time spent exercising
in March is 1 hour more than that in February

Click Here For Bundle PDF Course | support@guidely.in Page 3 of 14


Bank Po Mains PDF Course 2024
Quantitative Aptitude Day -4 (Eng)

11) Find the difference between the total time e) Only C


spent exercising in January and February
together and the total time spent studying in 14) Two series (I) and (II) are given below with
March and April together? (Consider 28 days in one wrong term in series (I) and one missing
Feb) term in series (II).
a) 55.3 hours (I): 2, 5, 9, 54, 648, 15552
b) 41.3 hours (II): 12, 15, 24, 39, ?, 87
c) 43.3 hours If the number, which will come in place of the
d) 52.3 hours wrong term in series (I), is P and the missing
e) 48.3 hours term in series (II) is Q, the find the value of
?
12) If the total time spent exercising in June is a) 7
10.8 hours less than that spent exercising in b) 9
March, then find the ratio of the average time c) 5
spent exercising in April to that in June? d) 8
a) 3: 5 e) 6
b) 1: 2
c) 9: 10 15) Three series (I), (II) and (III) are given below
d) 3: 4 with missing terms P, Q and R in series (I), (II)
e) 9: 7 and (III) respectively.
(I): 1400, 376, 120, 56, ?, 36
13) Two series (I) and (II) are given below with (II): 3780, 420, 60, 12, ?, 4
missing term P in series (I) and missing term Q (III): 103, 67, 42, 26, 17, ?
in series (II). Find that the value of (P * Q – R) is nearest to
(I): 8, 13, 22, 39, P, 137 the square of which of the following number?
(II): 139, 90, Q, 29, 13, 4 a) 12
Find that which of the following is/are true? b) 16
A: Ratio of P to Q is 4: 3. c) 13
B: d) 15
C: Sum of P and Q is a multiple of 24. e) 14
a) Only A and B
b) All are true 16) A series is given below with one wrong term
c) None is true in it.
d) Only A

Click Here For Bundle PDF Course | support@guidely.in Page 4 of 14


Bank Po Mains PDF Course 2024
Quantitative Aptitude Day -4 (Eng)

32, 8, 4, 4, 8, 64, 256, Q efficiency and B works with 200% of his


If P is the wrong term of the series and Q is the efficiency is days.
next term of the series, then find the relation Statement III: Time take by B and C together to
between P and Q? finish the work is 12.5 days while A alone can
a) P = Q1/2 + 64 finish the work in ‘3x’ days.
b) P = 2 * Q1/2 a) Statements I and II together are sufficient
c) P = Q1/2 + 16 b) Statements I and III together are sufficient
d) P = Q1/2 c) Statements II and III together are sufficient
e) P = Q1/2 – 64 d) Either (a) or (b)
e) Either (b) or (c)
17) Find the value of ‘N’.
Statement I: Average age of four persons A, B, 19) Find the value of MN (M > N, Where M and N
C, and D is 28.5 years, average age of A and D are natural numbers).
is ‘N’ years while average age of B and C is ‘M’ Statement I: 1 + M + N + MN = 391.
years. Statement II: M2 – N2 = 228 and M2 + N2 = 740.
Statement II: Total interest amount received Statement III: LCM and HCF of M and N are 176
when amount ‘M’ is invested for 5 years at 24% and 2 respectively.
SI and amount ‘N’ invested for 6 years 8 months a) Statement I alone is sufficient.
at 18% SI is ₹68.4. b) Statements I and II together are sufficient.
Statement III: M is 11 less than N. c) Statement II alone is sufficient.
a) Statements I and II together are sufficient d) Statement III alone is sufficient.
b) Statements I and III together are sufficient e) Either (a) or (c) or (d)
c) Statements II and III together are sufficient
d) Either (a) or (b) 20) Find the speed of a car whose speed is 20
e) Either (b) or (c) km/hr more than the speed of a train A.
Statement I: Train A can cross a pole in 14.4
18) Find the time taken by A and B together to seconds and can cross a platform of length 140
finish the work. meters in 21.6 seconds.
Statement I: Two persons A and C together can Statement II: Car can cover ‘D + 30’ km in
finish the work in ‘x’ days while C alone can hours while ‘D – 30’ km in hours.
finish the work in ‘x + 5’ days. Statement III: Car and a bus are initially 200 km
Statement II: Time taken by A and B together to apart and they started travelling towards each
finish the work when A works with 50% of his

Click Here For Bundle PDF Course | support@guidely.in Page 5 of 14


Bank Po Mains PDF Course 2024
Quantitative Aptitude Day -4 (Eng)

other simultaneously, then they will meet after c) Statement III alone is sufficient.
travelling for total 1 hour 20 min. d) Either (a) or (b)
a) Statement I alone is sufficient. e) Either (a) or (c)
b) Statement II alone is sufficient.
Click Here to Get the Detailed Video Solution for the above given Questions
Or Scan the QR Code to Get the Detailed Video Solutions

Answer Key with Explanation


Directions (1-4): 4y – 70 = 38
Let, the number of sold units of item A and D on y = 27
Tuesday are ‘4x’ and ‘7x’ respectively.
Also let, the number of sold units of item B and E
on Tuesday are ‘5y’ and ‘4y’ respectively.
So, the number of sold units of item C on
Tuesday = 7x + 7
Since, number of sold units of item D is 38 less
than that of item E.
So,
4y – 7x = 38 --------------(1)
1) Answer: B
Since, number of sold units of item A is 95 less
Total amount received by selling 75 units on item
than that of item B.
B on Mon = 15 * 75 = ₹1125
So,
Total amount received by selling 132 units on
5y – 4x = 95 --------------(2)
item C on Mon = 11 * 132 = ₹1452
By equation (1) * 5 – equation (2) * 4:
Total amount received by selling 63 units on item
20y – 35x – 20y + 16x = 190 – 380
D on Mon = 7 * 63 = ₹441
x = 10
Required sum = 1125 + 1452 + 441 = ₹3018
From equation (1):

Click Here For Bundle PDF Course | support@guidely.in Page 6 of 14


Bank Po Mains PDF Course 2024
Quantitative Aptitude Day -4 (Eng)

Directions (5-8):
2) Answer: C Time taken by bus A to go bus stand Q from bus
From A: stand P = 3.2 hours
Number of sold units of item A on Monday = 64 Time taken by bus B to go bus stand R from bus
Number of sold units of item E on Monday = 72 stand P = 2.5 hours
Required ratio = 64: 72 = 8: 9 Time taken by bus C to go bus stand S from bus
So, A is not true. stand P = 4.6 hours
From B: Time taken by bus D to go bus stand T from bus
Per units cost of item C = ₹11 stand P = 2.25 hours
Per units cost of item D = ₹7 Let the total distance covered by all the 4 buses
Per units cost of item E = ₹18 together = ‘100x’ km
Required average = So, the distance covered by bus A = 30% of
So, B is not true. 100x = ‘30x’ km
From C: The distance covered by bus B = 18.75% of
Total amount received by selling all the given 100x = ‘18.75x’ km
units of item E on Tuesday = 108 * 18 = ₹1944 The distance covered by bus C = 28.75% of
So, C is not true. 100x = ‘28.75x’ km
Hence, none is true. And the distance covered by bus D = 22.5% of
100x = ‘22.5x’ km
3) Answer: A Now, the speed of bus B =
Total amount received by selling 70 units of item The speed of bus D =
D on Tuesday = 70 * 7 = ₹490 Since, if the speed of bus B is 8 km/h more than
Total amount received by selling 40 units of item its original speed and the speed of bus D were 8
A on Tuesday = 40 * 8 = ₹320 km/h less than its original speed, the distance
Required percentage = covered by bus B would have been 8 km more
than that covered by bus D in the same time
4) Answer: D duration as given in the table.
Total units sold in items D and E together on So,
Monday = 63 + 72 = 135
Total units sold in items B and E together on
18.75x + 20 – 22.5x + 18 = 8
Tuesday = 135 + 108 = 243
x=8
Required ratio = 135: 243 = 5: 9

Click Here For Bundle PDF Course | support@guidely.in Page 7 of 14


Bank Po Mains PDF Course 2024
Quantitative Aptitude Day -4 (Eng)

8) Answer: C
Original speed of bus D = 80 km/h
New speed of bus D = 85% of 80 = 68 km/h
Time taken by bus D to reach bus stand R from
bus stand T = 3.5 hours
So, the distance between the bus stands R and
T = 68 * 3.5 = 238 km

Directions (9-12):
Let the average time spent exercising in January
and March are ‘5x’ hours and ‘6x’ hours
respectively.
5) Answer: B Also let the average time spent exercising in
Speed of bus A = 75 km/h February and May is ‘y’ hours and ‘3y’ hours
Distance between bus stands P and S = 230 km respectively.
The time, in which bus A will reach bus stand S So, the average time spent exercising in April =
= = 3 hours 4 minutes 6x/2 = ‘3x’ hours
Required time = 12:03 PM + 3 hours 4 minutes = Since, the average time spent exercising in
3:07 PM March is 1 hour more than that in February.
So,
6) Answer: E 6x – y = 1 -------------(1)
Speed of bus C = 50 km/h Since, the average time spent exercising in April
Let the speed of bus E = ‘x’ km/h is 37.5% of that in May.
So, So,

x = 77 km/hr

From equation (1):


7) Answer: A
The average of the speeds of buses A and B =
M= = 67.5 km/s x = 0.3, y = 0.8
Average time spent exercising in January = 5 *
The average of the speeds of buses C and D =
N= 0.3 = 1.5 hours
= 65 km/h
Required ratio = M: N = 67.5: 65 = 27: 26

Click Here For Bundle PDF Course | support@guidely.in Page 8 of 14


Bank Po Mains PDF Course 2024
Quantitative Aptitude Day -4 (Eng)

Average time spent exercising in February = 0.8


hours
Average time spent exercising in March = 6 * 0.3
= 1.8 hours
Average time spent exercising in April = 3 * 0.3 =
0.9 hours
Average time spent exercising in May = 3 * 0.8 =
2.4 hours
Now, 9) Answer: E
The average time spent studying in February = Total time spent studying in January = 31 * 2.4 =
0.8 + 2.2 = 3 hours 74.4 hours
The average time spent studying in January = 3 Total time spent studying in February = 28 * 3 =
– 0.6 = 2.4 hours 84 hours
Since, the difference between the average time Total time spent studying in March = 31 * 1.2 =
spent studying in January and that in March is 37.2 hours
1.2 hours Required average = hours
So, the average time spent studying in March
will be either 2.4 + 1.2 = 3.6 hours or 2.4 – 1.2 = 10) Answer: B
1.2 hours The month, in which the average time spent
Since, the difference between the average time exercising is maximum, is May.
spend on studying in February and that in March So, the average time spent exercising in May =
is 1.8 hours. 2.4 hours
So, the only possible average time spend on And the average time spent studying in May = 2
studying in March = 3 – 1.8 = 1.2 hours hours
The average time spend on studying in April = Required ratio = 2.4: 2 = 6: 5
= 2.5 hours
The average time spend on studying in May = 11) Answer: C
= 2 hours Total time spent exercising in January and
February together:
(31 * 1.5) + (28 * 0.8) = 46.5 + 22.4 = 68.9 hours
Total time spent studying in March and April
together:
(31 * 1.2) + (30 * 2.5) = 37.2 + 75 = 112.2 hours

Click Here For Bundle PDF Course | support@guidely.in Page 9 of 14


Bank Po Mains PDF Course 2024
Quantitative Aptitude Day -4 (Eng)

Required difference = 112.2 – 68.9 = 43.3 hours


8=7–1
12) Answer: A So, B is not true.
Total time spent exercising in March = 31 * 1.8 = From C:
55.8 hours Sum of P and Q = 72 + 54 = 126.
Total time spent exercising in June = 55.8 – 10.8 Since, 126 is not a multiple of 24.
= 45 hours So, C is not true.
The average time spent exercising in June = = Hence, only A is true.
1.5 hours
The average time spent exercising in April = 0.9 14) Answer: A
hours Logic in series (I):
Required ratio = 0.9: 1.5 = 3: 5 2 * 1.5 = 3(p)
3*3=9
13) Answer: D 9 * 6 = 54
Logic in series (I): 54 * 12 = 648
8 + 5 = 13 648 * 24 = 15552
13 + 9 = 22 So, the number, which will come in place of the
22 + 17 = 39 wrong term in series (I) = P = 3
39 + 33 = 72 Logic in series (II):
72 + 65 = 137 12 + 3 = 15
So, the missing term in series (I) = P = 72 15 + 9 = 24
Logic in series (II): 24 + 15 = 39
139 - 72 = 90 39 + 21 = 60
90 - 62 = 54 60 + 27 = 87
54 - 52 = 29 So, the missing term in series (II) = Q = 60
29 - 42 = 13 Now, the value of
13 - 32 = 4
So, the missing term in series (II) = Q = 54 15) Answer: A
From A: Logic in series (I):
Ratio of P to Q = 72: 54 = 4: 3 1400 – 1024 = 376
So, A is true. 376 – 256 = 120
From B: 120 – 64 = 56
56 – 16 = 40

Click Here For Bundle PDF Course | support@guidely.in Page 10 of 14


Bank Po Mains PDF Course 2024
Quantitative Aptitude Day -4 (Eng)

40 – 4 = 36 17) Answer: E
So, the missing term in series (I) = P = 40 From statements I and II together:
Logic in series (II): Average age of four persons A, B, C, and D =
3780 ÷ 9 = 420 28.5
420 ÷ 7 = 60 A + B + C + D = 28.5 * 4 = 114 ……… (1)
60 ÷ 5 = 12 Average age of A and D = N
12 ÷ 3 = 4 A + D = 2N ……. (2)
4÷1=4 Average age of B and C = M
So, the missing term in series (II) = Q = 4 B + C = 2M …….. (3)
Logic in series (III): From (1), and (2), and (3):
103 - 62 = 67 2N + 2M = 114
67 - 52 = 42 M + N = 57 …….. (4)
42 - 42 = 26 Total interest amount received =
26 - 32 = 17
17 - 22 = 13
So, the missing term in series (III) = R = 13
1.2M + 1.2N = 68.4
Now, the value of (P * Q – R) = 40 * 4 – 13 = 147
M + N = 57
The number 147 is nearest to the square of the
Since we are getting two same equations. So,
number 12.
we cannot determine the value of N.
Hence, Statements I and II together are not
16) Answer: D
sufficient.
Logic in the series:
From statements I and III together:
32 * 0.25 = 8
Average age of four persons A, B, C, and D =
8 * 0.5 = 4
28.5
4*1=4
A + B + C + D = 28.5 * 4 = 114 ……… (1)
4*2=8
Average age of A and D = N
8 * 4 = 32(64)
A + D = 2N ……. (2)
32 * 8 = 256
Average age of B and C = M
256 * 16 = 4096
B + C = 2M …….. (3)
So, the wrong term of the series = P = 64
From (1), and (2), and (3):
And the missing term of the series = Q = 4096
2N + 2M = 114
Relation between P and Q:
M + N = 57 …….. (4)
P = Q1/2
N – M = 11 …… (5)

Click Here For Bundle PDF Course | support@guidely.in Page 11 of 14


Bank Po Mains PDF Course 2024
Quantitative Aptitude Day -4 (Eng)

From (4) and (5): Time taken by A and C together to finish the
N = 34 and M = 23 work = ‘x’ days
Hence, Statements I and III together are According to the question:
sufficient.
From statements II and III together:
Total interest amount received =
4x + 5 = 3x + 15
x = 10
Time taken by B and C together to finish the
1.2M + 1.2N = 68.4 work = 12.5 days
M + N = 57 …… (6) According to the question:
N – M = 11 …… (5)
From (5) and (6):
N = 34 and M = 23
Hence, Statements II and III together are B = 75
sufficient. Time taken by A and B together to finish the
work =
18) Answer: B
From statements I and II together:
In statement I, we are given data about persons
A and C and in statement II we are given data Hence, statements I and III together are
about persons A and B. sufficient.
But in statement II, we are given data about A From statements II and III together:
and B but at different efficiencies. When A works with 50% of his efficiency, time
So, we cannot determine the time taken by A taken by him alone to finish the work = 2A
and B together to finish the work. When B works with 200% of his efficiency, time
Hence, statements I and II together are not taken by him alone to finish the work = B/2
sufficient. According to the question:
From statements I and III together:
Time taken by C alone to finish the work = ‘x + 5’
days
A = 3x ………. (3)
Time taken by A alone to finish the work = ‘3x’
From the above three equations, we cannot
days
determine the time taken by A and B together to

Click Here For Bundle PDF Course | support@guidely.in Page 12 of 14


Bank Po Mains PDF Course 2024
Quantitative Aptitude Day -4 (Eng)

finish the work as we have only three equations = 352


but have total four variables in it. Statement II alone is sufficient.
Hence, statements II and III together are not From statement III alone:
sufficient. Since HCF of two number M and N is 2.
Let M = 2a and N = 2b [‘a’ and ‘b’ are co-prime.]
19) Answer: E LCM of 2a and 2b = 2ab = 176
From statement I alone: ab = 88
1 + M + N + MN = 391 Required product = MN
We know that: (1 + M) (1 + N) = 1 + M + N + MN = 2a * 2b
(1 + M) (1 + N) = 391 = 23 * 17 = 1 * 391 = 4ab
Since, 391 have only four factors 1, 17, 23, and = 4 * 88
391. Also, we know that M and N are natural = 352
number which means we cannot take value (1 + Statement III alone is sufficient.
N) as 1.
So, 20) Answer: D
(1 + M) (1 + N) = 23 * 17 From statement I alone:
After comparing: Let the speed of train is ‘x’ m/s while the length
M = 22 and N = 16 of train is ‘y’ meters.
Required product = MN = 22 * 16 According to the question:
= 352 y = 14.4x ……… (1)
Statement I alone is sufficient. (y + 140) = 21.6x ……… (2)
From statement II alone: From (1) and (2):
M2 – N2 = 228 14.4x + 140 = 21.6x
M2 = N2 + 228 ……… (1) 7.2x = 140
M2 + N2 = 740 ……… (2) x = 175/9 m/s
From (1) and (2): Speed of train (in km/hour) = = 70
(N2 + 228) + N2 = 740 km/hour
2N2 = 512 Speed of car (in km/hour) = 70 + 20 = 90
N2 = 256 km/hour
N = 16 [Since N is a natural number.] Hence, statement I alone is sufficient.
M2 = 228 + 162 = 228 + 256 = 484 From statement II alone:
M = 22 Let the speed of car = ‘s’ km/hour
Required product = MN = 22 * 16 According to the question:

Click Here For Bundle PDF Course | support@guidely.in Page 13 of 14


Bank Po Mains PDF Course 2024
Quantitative Aptitude Day -4 (Eng)

From statement III alone:


Let the speed of bus = ‘z’ km/hour
Relative speed of car and bus when travelling
towards each other = (s + z) km/h
From (3) and (4): According to the question:

s = 90 km/hour (s + z) = 150
Speed of car = 90 km/hour We cannot determine the speed of car.
Speed of train = 90 – 20 = 70 km/hour Hence, statement III alone is not sufficient.
Hence, statement II alone is sufficient.

Click Here For Bundle PDF Course | support@guidely.in Page 14 of 14


Bank Po Mains PDF Course 2024
English Day - 4

English Language
Directions (1-6): A passage has been given A. abhorrent
below with some words highlighted in it followed B. gruesome
by six questions. Read the passage carefully and C. salubrious
answer the following questions. D. innocuous
The Iranian Foreign Ministry’s comment, that E. No replacement required
only “Salman Rushdie and his supporters” were
to be blamed for the enticing knife attack on the 2. What is the antonym of the highlight word
author in New York State last week, is yet “blasphemous”?
another reiteration of the clerical establishment’s A. Both C and E
well-known regressive position on the Rushdie B. Impious
affair. The 1989 fatwa against Rushdie, issued C. Sacrilegious
by Iran’s then Supreme Leader Ayatollah D. Profane
Khomeini, because he believed the author’s The E. Sacrosanct
Satanic Verses was blasphemous, has already
wreaked enough havoc across the world. It was 3. Which of the following idioms can be used in
a rare instance of a leader of a country issuing the below mentioned highlighted parts without
an extra-territorial death sentence against an changing the actual meaning of the sentence.
author who was living in another country by The Satanic Verses was blasphemous, has
impeding his pan-Islamist beliefs and clerical already wreaked enough havoc across the
authority. The book’s translators and publishers world.
were attacked. It drove Rushdie underground for I. Created chaos
years. Thirty-three years later, Rushdie was II. Dilly dally
attacked by a 24-year-old American citizen of III. Laid-waste
Lebanese descent, whose social media accounts A. Both I and III
are reportedly filled with pro-Khomeini and pro- B. Only I
Iran content. It is tedious that Iran could not even C. Both I and II
issue a statement condemning the attack and the D. Only III
attacker. (Worse, pro-state media in Iran E. All I, II and III
applauded “the courageous and duty-conscious
man who attacked the disciple and depraved 4. Which of the following words can replace the
Salman Rushdie in New York”.) highlighted word “impeding”?
1. Which of the following words can replace the A. evoking
highlighted word “enticing”? B. invoking

Click Here For Bundle PDF Course | support@guidely.in Page 1 of 9


Bank Po Mains PDF Course 2024
English Day - 4

C. provoking (B)/ The boards and banners, crowds outside


D. revoking Thackeray’s (C)/ bungalow in Bandra, a steady
E. No replacement required stream of visitors (D)/ The Sena workers’ roar
added to the mood of the moment (E).
5. Which of the following words can replace the A. ADBE; C
highlighted word “tedious”? B. BEDC; A
A. invigorating C. CDAB; E
B. conspicuous D. ABDE; C
C. beguiling E. DBAC; E
D. astonishing
E. No replacement required 8. approach, we are about to leave an (A)/ place
for ourselves with our irresponsible (B)/ two
6. Which of the following highlighted words is incidents in Maharashtra remind one of (C)/ just
contextually inapt? as we have made the world into a harmful (D)/
(Worse, pro-state media in Iran applauded “the uninhabitable world to our children (E).
courageous and duty-conscious man who A. DECB; A
attacked the disciple and depraved Salman B. AEDB; C
Rushdie in New York”.) C. BDEC; A
A. disciple D. DBAE; C
B. applauded E. CAED; B
C. depraved
D. Both A and C 9. daughter on social media is loved by fans (A)/
E. All are correct I wouldn't jump on the bandwagon (B)/ her 9-
year-old daughter, Samairra (C)/ and her content
Directions (7-11): The following questions below with her (D)/ Juhi has been a single parent to (E).
contain sentences that have been divided into A. AEBD; D
five parts each. One of these parts might be B. ECDA; B
unrelated with the other parts. After identifying C. ABEC; D
the odd part, you are required to arrange the D. DCEA; B
remaining parts into a meaningful sentence and E. No rearrangement required; A
mark the option containing the correct sequence 10. including India and Pakistan (A)/ be
as your answer. accounted for by eight countries, (B)/ the next 30
7. to the Sena Bhavan in Dadar, all evoked visual years will (C)/ unlike China, which compulsorily
(A)/ imagery at a time of grave crisis in the party

Click Here For Bundle PDF Course | support@guidely.in Page 2 of 9


Bank Po Mains PDF Course 2024
English Day - 4

enforced the (D)/ half the growth of population D. When the police arrived, the street was in total
during (E). chaos.
A. AEDB; C E. Exposure to the sun can accelerate the ageing
B. CDAB; E process.
C. ECBA; D
D. BEDA; C 13.
E. DEAC; B A. And judging by the repeated glances the
woman gave the sky, she too was cognizant of
11. In both cases, both the individuals (A)/ being the failing light.
political appointees at (B)/ least in present times, B. The new reporter was apprised of
the (C)/ decision-making process is mostly developments in this story.
skewed (D)/ who though have not split away C. Good reporters are keenly observant of
have (E). everything around them.
A. DECB; A D. I wasn't even conscious of what was
B. EDBC; A happening.
C. ABED; C E. Authors are famously ignorant about the
D. CDBE; A realities of publishing.
E. No rearrangement needed; E
14.
Directions (12-16): In the questions given below, A. Other companies are in an even worse
five sentences have been given which use words predicament than ourselves.
that have been highlighted. One of these Opposition leaders this week implored the
sentences uses a word that is unrelated to the president to break the deadlock.
rest. You are required to choose the same as C. The police have announced a breakthrough in
your answer. the murder case.
12. D. Kate found herself in a dilemma when her two
A. The modality of teaching is changing to best friends were fighting.
become more modern. E. The imbroglio seemed to be on the floor, on
B. New employees are taught the proper safety the other side of the table, near the wall.
procedures.
C. In business, the pragmatic approach to 15.
problems is often more successful than an A. Soldiers from many different countries have
idealistic one. been subsumed into the United Nations peace-
keeping force.

Click Here For Bundle PDF Course | support@guidely.in Page 3 of 9


Bank Po Mains PDF Course 2024
English Day - 4

B. She found him entertaining enough to C. Kunal is a very sweet, amiable and adamant
overlook his faults. boy.
C. Because the information was important, the D.Unemployed youths of this country are
government decided to classify several of the adamant about finding a suitable job.
documents as top-secret. A. Only B
D. Since I have never been married, I am unsure B. Only A
of what my wifely duties will entail. C. Only D
E. I can only imagine other uses you might have D. Only C
embraced, for personal financial enhancement. E. All are correct

16. 18. Approbation


A. His death sparked outrage throughout Europe A. With your approbation she can stay at my
and culminated in the King making public house.
penance. B. You need to build a powerful resume to gain
B. I pledge to repent and atone for this crime for approbation from the hiring company.
the rest of my life, she said. C. The clerks need prior approbation from the
C. To expiate for breaking his neighbor's window, manager on this issue.
John shoveled snow for three months. D. Without my prior approbation, there should not
D. They've offered no apologies and seem to be any absentees tomorrow.
have no thoughts of reparation. A. Only A
E. All the given sentences are correct B. Only B
C. Only C
Directions (17-21): Given below is a highlighted D. Only D
word followed by 4 sentences. You need to E. All are correct
understand whether the sentences are using the
word correctly or not. Mark the incorrect 19. Aspersion
sentence with the incorrect word usage as your A. Even though Sheila is angry at you, I will not
answer. cast an aspersion upon your character.
17. Adamant B. I was shocked when he dared to cast
A.The doctors at Apollo hospital had been aspersion on his wife.
absolutely adamant that Meena’s grandmother C. We decided to have an aspersion before
was not going to get better. arriving at a conclusion.
B. Raani is so adamant in her political beliefs that D. Kunal has every right to direct an aspersion at
no one can change her mind! his cheating wife.

Click Here For Bundle PDF Course | support@guidely.in Page 4 of 9


Bank Po Mains PDF Course 2024
English Day - 4

A. Only A D. Only D
B. Only B E. All are correct
C. Only C
D. Only D 21. Boisterous
E. All are correct A. I could see the noisy and boisterous children
fighting with one another.
20. Bustle B. Kavish is a boisterous young man and hence
A. 1The bustle in this city really irritates me a lot. is loved by all.
B. On weekends you can experience bustle at C. The boisterous welcome by my friends
the box office. cheered me up.
C. On the eve of any festival, people bustle about D. The boisterous dog started barking at me as
the store trying to purchase various items. soon as it saw me.
D. Since the lane was very bustling there were A. Only A
hardly any people in sight. B. Only B
A. Only A C. Only C
B. Only B D. Only D
C. Only C E. All are correct
Click Here to Get the Detailed Video Solution for the above given Questions
Or Scan the QR Code to Get the Detailed Video Solutions

Answer Key with Explanation


1. Answer: B The term enticing means “attractive or tempting;
Usage of the highlighted word “enticing” in this alluring”, we need a word having negative
context alters the entire meaning of the meaning.
sentence, how could an attack be enticing?? Gruesome is the most apt word to be used which
means “extremely disturbing or repellent”.

Click Here For Bundle PDF Course | support@guidely.in Page 5 of 9


Bank Po Mains PDF Course 2024
English Day - 4

Abhorrent- inspiring disgust and loathing; (someone or something) by obstructing them;


repugnant. hinder” which is totally out of the context.
Innocuous- not harmful or offensive. The context says that it rarely happens when a
Salubrious- beneficial to the health of body or leader of a country issues an extra-territorial
mind death sentence against an author who was living
Hence option B is the correct answer. in another country by begeting his pan-Islamist
beliefs and clerical authority.
2. Answer: E Revoking means officially canceling (a decree,
The term blasphemous means “sacrilegious decision, or promise).
against God or sacred things”, or we can Evoking means to bring or recall (a feeling,
describe someone who shows disrespect for memory, or image) to the conscious mind.
God or a religion as blasphemous. Provoking- giving rise to the specified reaction or
All the words except “Sacrosanct” are synonyms emotion.
of blasphemous. Hence option B is the correct answer.
The correct antonym is sacrosanct, which means
“(especially of a principle, place, or routine) 5. Answer: D
regarded as too important or valuable to be The context says that it is surprising that Iran
interfered with.” has not even issued a statement criticizing the
Hence option E is the correct answer. attack and the attacker. In this context usage of
the highlighted word “tedious” in this context
3. Answer: A alters the entire meaning of the sentence.
The idiom “wreaked havoc” means “to cause Instead “astonishing” will be used which means
very bad damage to (something)” and the idioms “causing a feeling of great surprise or wonder”
which have similar meaning are “Created chaos” Tedious- too long, slow, or dull; tiresome or
and “Laid-waste”. monotonous.
Dilly- dally means to delay without cause or Invigorating- making one feel strong, healthy,
reason and not to rush. and full of energy.
Hence option A is the correct answer. Conspicuous- clearly visible.
Beguiling- charming or enchanting, often in a
4. Answer: B deceptive way.
Usage of the highlighted word “impeding” in this Hence option D is the correct answer.
context alters the entire meaning of the
sentence. “Impeding” means “delay or prevent 6. Answer: A

Click Here For Bundle PDF Course | support@guidely.in Page 6 of 9


Bank Po Mains PDF Course 2024
English Day - 4

Usage of the highlighted word “disciple” in this A doesn’t continue the same in a coherent
context alters the entire meaning of the manner. This option can hence, be cancelled for
sentence. “Disciple” means “one who follows the the same reason.
opinions or teachings of another” which is totally So, D should start and B should follow most
out of the context. Instead we have to use logically. This hint takes us to option (d) as the
“apostate” which means “a person who correct answer.
renounces a religious or political belief or 9. Answer: B
principle”. EC is a link as E mentions Juhi to be a single
Applauded- show strong approval of (a person or parent and C tells us about her daughter. This
action); praise. link can be found in two options that is (b) and
Depraved- morally corrupt; wicked. (c). All other options can hence be eliminated.
Hence option D is the correct answer. If we go with the sequence given in (c), we
would find that the given combination of
7. Answer: C sentences does not make any sense. So, this
Either E or C can start the sentence as all the option can be ruled out.
other parts are quite incomplete to start with. The correct answer therefore, should be option
From the options we can figure out that, the only (b).
choice that can be found is to start the sentence
with C. This clearly cancels all the other options 10. Answer: C
out. The link is ‘… during……the next 30 years …’
The sequence makes perfect sense and also EC is a great link. The best is to check the
tells us that E should be the odd sentence as it options and mark option (c) as the answer.
doesn’t fit with the other given parts.
Therefore, option (c) is the correct answer. 11. Answer: E
The given parts are already in their correct
8. Answer: D sequence and there’s no need to bring them in
Either C or D can start the sentence. Once we sequence. Also, the last part ie; E doesn’t fit the
scan through all the parts given in the sentence, sentence and therefore, can be taken as an odd
we can find that the other parts are incomplete one out.
and cannot be used as an introduction. This Thus, the correct answer is option (e).
helps us cancel the second and the third options.
If we check the options, we can find that if we 12. Answer: D
start with C in the way as given in the fifth option,

Click Here For Bundle PDF Course | support@guidely.in Page 7 of 9


Bank Po Mains PDF Course 2024
English Day - 4

The words in (a), (b), (c) and (e) are all 15. Answer: B
synonyms that mean; A specific mode in which The word subsumed is used when we include or
something is expressed or is experienced, the absorb one thing into the other. To entail,
steps of doing things. The word ‘chaos’ however, classify and embrace are also similar things.
means disorder. This is the opposite of the given The word that is different from the rest is hence,
words in the other options. overlook which means to fail to notice or simply
Therefore, the best would be to go with option have a look from a height.
(d). Therefore, the best answer is option (b).

13. Answer: E 16. Answer: E


Being aware or knowing things is what is meant The given words are all synonyms to each other.
by being ‘cognizant’. ‘Apprised’ means being Penance means the act of inflicting punishment
informed about things. ‘Observant’ and on oneself or to punish someone. To atone and
‘Conscious’ are synonyms of ‘cognizant’ and expiate is to make amends for one’s mistakes.
‘apprised’. The word ‘ignorant’ however is the Reparation means to compensate for a
opposite of the given words as it means being wrongdoing.
unaware of things. Therefore, all the given sentences use similar
Therefore, the best answer is option (e). words and hence, the correct answer would be
option (e).
14. Answer: C
Predicament is a difficult, unpleasant, or 17. Answer: D
embarrassing situation. Adamant means refusing to be persuaded or to
A deadlock means to come to a point where no change one's mind.
progress can be made because of fundamental If we look at the sentences given here we can
disagreement. easily understand that in sentence C the context
A dilemma is a difficult situation or problem. does not quite match with the word adamant. We
An imbroglio an extremely confused, need a word which has a positive sense. So it is
complicated, or embarrassing situation. incorrectly used here.
A breakthrough is a sudden discovery that Therefore, the correct answer is option D.
solves a problem. This word is different from the
others as all the other words are synonyms of 18. Answer: E
each other. Approbation is a noun which actually
meansapproval or praise.

Click Here For Bundle PDF Course | support@guidely.in Page 8 of 9


Bank Po Mains PDF Course 2024
English Day - 4

In all the given sentences the word has been Bustle means moving in an energetic and busy
correctly used. manner. In sentence D, the word bustle makes
Therefore, the correct answer is option E. no sense at all. Thus it is incorrect here.
Therefore, in the given question the correct
19. Answer: C answer is option D.
Aspersion means an attack on the reputation or
integrity of someone or something. In sentence 21. Answer: B
C the word makes no sense and seems out of Boisterous means noisy, energetic, and cheerful.
the context. And “someone spirited, loud, and slightly out of
Therefore, in the given question the correct control”
answer is option C. In sentence B, the word boisterous makes no
sense and is clearly out of the context.
20. Answer: D Therefore, in the given question the correct
answer is option B.

Click Here For Bundle PDF Course | support@guidely.in Page 9 of 9


Bank Po Mains PDF Course 2024
Reasoning Day -5 (Eng)

Reasoning Aptitude
Directions (1-5): Study the following information e) Can’t be determined
carefully and answer the questions given below
Ten persons –P, Q, R, S, T, U, V, W, X and Y are 2. Who lives three floors above the one who is
living in a ten storey-building, where the lower- the niece of W?
most floor is numbered as one and the top-most a) The one who is the grandfather of R
floor is numbered as ten. They belong to the b) Q
same family with three generations and there is c) The one who is the mother of U
no single parent. Not more than two persons of d) The one who is the nephew of X
the same gender live on adjacent floors. e) S
P is the son-in-law of the one who is the mother
of S. W lives four floors below her sister-in-law V, 3. Which of the following statement(s) is/are not
who has no siblings. The one who lives on the false with respect to the final arrangement?
eighth floor is the son-in-law of T, who lives a) The one who is the daughter-in-law of T lives
immediately above his granddaughter. The one on an odd numbered floor
who lives on the third floor is the brother of S, b) Y and her mother live on adjacent floors.
who lives two floors below her only brother. R is c) Three persons live between T and son-in-law
the niece of W’s sister who is a spinster. Only of Q
three persons live between Q’s one of the d) U is not a daughter of P
daughters and X’s only daughter, whose e) All the given statement is true.
grandfather is T. The one who lives on the ninth
floor is the son of P, who is the brother-in-law of 4. Four of the following five are alike in a certain
Y’s father. Y lives on an even numbered floor way based on the given arrangement and thus
and lives below one of her paternal aunts. U is form a group. Which one of the following does
the grandson of W’s mother, who lives on a not belong to the group?
prime number floor above floor number 5. Both R a) The one who lives on floor number 7
and the one who is the maternal aunt of R are b) The one who is the daughter of T
living below R’s grandfather. c) The one who lives two floors above Y.
1. How is P related to the one who lives two d) The one who is the cousin of U.
floors below X? e) The one who lives three floors below P.
a) Sister-in-law
b) Brother-in-law 5. Which of the following statement is true with
c) Husband respect to X from the final arrangement?
d) Wife a) X is the daughter of T

Click Here For Bundle PDF Course | support@guidely.in Page 1 of 13


Bank Po Mains PDF Course 2024
Reasoning Day -5 (Eng)

b) Only two persons live above X Some random letters are picked from both
c) X lives immediately below his niece arrangements to form a word. If the letters form a
d) X is the brother-in-law of W meaningful word then the word is written as it is.
e) All the statement is false If the letters did not form a meaningful word, then
the letters are written in reverse alphabetical
Directions (6-10): Study the following information order from left to right.
carefully and answer the questions given below The first letter and the last letter of arrangement
A ten letter word (may or may not be meaningful) 1 are taken along with the letter which is fourth
with no repeated letters is arranged in three from the left end and fifth from the right end of
arrangements to form a series. arrangement 2 is taken to form the first word. E is
Arrangement 1: not the second letter of the first word. Next, the
O is placed fifth to the left of E. The number of letter which is fourth from the right end of both
letters placed to the left of O is one more than arrangements along with the letter which is
the number of letters placed to the right of N. M second from the left end in arrangement 2 is
is placed third to the right of U, where neither of taken to form the second word. Then, the letter
them is placed adjacent to N. As many letters which is fourth from left end and fifth from right
between T and H as between H and V, which is end in arrangement 1 is taken along with the
placed immediate right of A. A is placed four letter which is third from left end and second from
places away from C. More than two letters are right end of arrangement 2 is taken to form the
placed between H and A. third word.
Arrangement 2: 6. Which of the following letter is placed third to
If the word formed in arrangement 1 doesn’t give the right of E in arrangement 2?
a meaningful word, then all the vowels in the a) Q
word are replaced with their third preceding letter b) L
as per the English alphabetical series and all the c) K
consonants are replaced with their opposite letter d) S
as per the English alphabetical series. Else, If e) F
arrangement 1 forms the meaningful word, then
all the consonants in the word are replaced with 7. Which of the following letter is placed exactly
their third preceding letter as per the English between the letters which is second from left end
alphabetical series and all the vowels are and third from right end in arrangement 1?
replaced with their opposite letter as per the a) U
English alphabetical series. b) C
Arrangement 3: c) E

Click Here For Bundle PDF Course | support@guidely.in Page 2 of 13


Bank Po Mains PDF Course 2024
Reasoning Day -5 (Eng)

d) V archery, tennis, boxing and rowing in Olympics.


e) M Each country gets different rank from 1 to 6, such
that rank 1 is the highest rank whereas rank 6 is
8. In which of the following combination, the first the lowest rank. Each Country gets different
letter is exactly between the second and third ranks in all four sports. No two countries get the
letter in any of the three arrangements? same rank in each sport.
a) COU Japan gets the highest rank in tennis which is
b) STE three ranks higher than China in boxing. US rank
c) JEK in tennis is same as Malaysia’s rank in rowing,
d) EFV but neither of them ranks first nor last. China
e) MNT neither gets the second highest nor lowest rank
in any sports. In tennis, Turkey gets higher rank
9. How many letters are there in the English than Malaysia and lower rank than US. Japan
alphabetical series between the letters which is gets lowest rank in archery and second lowest
second from both ends in arrangement 1? rank in boxing. Sum of the rank of India in boxing
a) Four and tennis is equal to rowing. In boxing, Turkey
b) Five gets two ranks higher than Japan. India neither
c) Six gets the highest nor lowest rank. The one who
d) Seven gets first rank in boxing gets last rank in tennis.
e) More than seven In rowing, Japan gets higher rank than US and
lower rank than Malaysia. The rank difference
10. What is the third word (may or may not be between Turkey in boxing and rowing is equal to
meaningful) formed in arrangement 3? China in archery. In archery, US get three ranks
a) LOHK higher than India.
b) ULKH 11. How many countries get higher ranks than
c) KLOH China in boxing?
d) LHOK a) One
e) HULK b) Two
c) Three
Directions (11-15): Study the following d) Four
information carefully and answer the questions e) Five
given below
Six countries – US, China, India, Japan, Turkey 12. What is the rank of Turkey in Boxing and
and Malaysia participated in different sports – Archery and US in rowing respectively?

Click Here For Bundle PDF Course | support@guidely.in Page 3 of 13


Bank Po Mains PDF Course 2024
Reasoning Day -5 (Eng)

a) 4, 3, 2 A businessman shared his profit amount of


b) 5, 6, 3 Rs.60,000 with his nine employees - A, B, C, D,
c) 6, 1, 2 E, F, G, H and I are sitting around a triangular
d) 3, 2, 4 table in such a way that one person sits at each
e) Both a and d corner facing outside and two persons sit on
each side of the table facing inside the table.
13. What is the sum of the rank of Japan in Profit was shared in different percentage - 2%,
archery, India in boxing and Malaysia in tennis? 4%, 5%, 8%, 11%, 15%, 16%, 18% and 21%
a) 10 based on their performance.
b) 14 Note: The share percentage of the persons
c) 24 sitting at the corners is a prime number.
d) 34 Four persons sit between E and F, who is not
e) 18 adjacent to B. The one who gets 4% of profit sits
third to the right of F, whose share percentage is
14. Which country gets three ranks lower than a non-prime odd number. B sits third to the left of
US in tennis and two ranks higher than Turkey in I, who sits second to the right of the one whose
rowing respectively? share is Rs.1200. The sum of the share received
a) China, US by F and I is Rs.13800, where F and B don’t sit
b) Malaysia, US adjacent to each other. The difference between
c) China, Japan the share percentage of C and H is 2%. C sits to
d) Japan, Turkey the immediate left of the one whose share in
e) India, US profit is Rs.12600. The number of persons sitting
between G and D is same as the number of
15. The sum of the rank of which country in all persons sitting between E and B. The share
the four sports events is the lowest? difference between A and the one who sits
a) China and Turkey immediate right of the one whose share
b) US and Malaysia percentage is 15% is Rs.7800. The one whose
c) India and Japan percentage share is 16% and D are not sitting
d) India and Turkey together.
e) China and US 16. Who among the following person gets the
share of Rs.9600?
Directions (16-20): Study the following a) The one who sits second to the left of F
information carefully and answer the question b) I
given below. c) B

Click Here For Bundle PDF Course | support@guidely.in Page 4 of 13


Bank Po Mains PDF Course 2024
Reasoning Day -5 (Eng)

d) The one who sits immediate right of E d) B


e) Both d and c e) G

17. Which of the following statement(s) is/are 19. If I is related to 16% and H is related to 15%
not false with respect to the final arrangement? in a certain way, then who is related to 21%?
a) Three persons sit between C and the one a) B
whose share percentage is 5 b) F
b) F sits second to the left of the one whose c) A
share is 9600 d) I
c) Sum of the share of A and E is less than c) D
Rs.15000
d) Both a and b 20. Which of the following statement is true with
e) Both b and c respect to D?
a) The share amount of D is Rs.3000
18. Four of the following five are alike in a certain b) D sits immediate left of the one whose share is
way based on the given arrangement and thus Rs.9600
form a group. Which one of the following does c) D’s percentage share is a prime number
not belong to the group? d) D sits second to the left of the one who sits
a) F immediate left of B.
b) D e) D sits third to the right of the one whose share
c) E percentage is 8%

Click Here For Bundle PDF Course | support@guidely.in Page 5 of 13


Bank Po Mains PDF Course 2024
Reasoning Day -5 (Eng)

Click Here to Get the Detailed Video Solution for the above given Questions
Or Scan the QR Code to Get the Detailed Video Solutions

Answer Key with Explanation


Directions (1-5):
1. Answer: B
2. Answer: A
3. Answer: D
4. Answer: E
5. Answer: C
We have,
 P is the son-in-law of the one who is the
mother of S.
 W lives four floors below her sister-in-law
V, who has no siblings.
 The one who lives on the eighth floor is Again we have,

the son-in-law of T, who lives immediately  R is the niece of W’s sister who is a

above his granddaughter. spinster.

 The one who lives on the third floor is the  Only three persons live between Q’s one

brother of S, who lives two floors below of the daughters and X’s only daughter,

her only brother. whose grandfather is T.

From above conditions, we have two  The one who lives on the ninth floor is the

possibilities: son of P, who is the brother-in-law of Y’s


father.
 Y lives on an even numbered floor and
lives below one of her paternal aunts.

Click Here For Bundle PDF Course | support@guidely.in Page 6 of 13


Bank Po Mains PDF Course 2024
Reasoning Day -5 (Eng)

Directions (6-10):
6. Answer: C
7. Answer: B
8. Answer: D
9. Answer: D
Again we have,
10. Answer: E
 U is the grandson of W’s mother, who
Arrangement 1:
lives on a prime number floor above floor
We have,
number 5.
 O is placed fifth to the left of E.
 Both R and the one who is the maternal
 The number of letters placed to the left of
aunt of R are living below R’s grandfather.
O is one more than the number of letters
Note: Not more than two persons of the same
placed to the right of N.
gender live on adjacent floors.
While applying above conditions, case 1, and 3
gets eliminated

Again we have,
 M is placed third to the right of U, where
neither of them is placed adjacent to N.
 As many letters between T and H as
between H and V, which is placed
immediate right of A.
 A is placed four places away from C.
 More than two letters are placed between
H and A.

(Note - sign indicates female and + sign


indicates male)
Thus, case 1.a gives the final arrangement.

Click Here For Bundle PDF Course | support@guidely.in Page 7 of 13


Bank Po Mains PDF Course 2024
Reasoning Day -5 (Eng)

 The first letter and the last letter of


arrangement 1 are taken along with the
letter which is fourth from the left end and
fifth from the right end of arrangement 2 is
taken to form the first word. E is not the
Thus, case 2.a gets final arrangement. second letter of the first word.
Arrangement 2: Arrangement 1: AVOUCHMENT
 If the word formed in arrangement 1 Arrangement 2: ZSLFZEJVKQ
doesn’t give a meaningful word, then all New word: FATE
the vowels in the word are replaced with  Next, the letter which is fourth from right
their third preceding letter as per the end of both arrangements along with the
English alphabetical series and all the letter which is second from the left end in
consonants are replaced with their arrangement 2 is taken to form the
opposite letter as per the English second word.
alphabetical series. Arrangement 1: AVOUCHMENT
 If arrangement 1 forms the meaningful Arrangement 2: ZSLFZEJVKQ
word, then all the consonants in the word New word: SMJ
are replaced with their third preceding  Then, the letter which is fourth from the
letter as per the English alphabetical left end and fifth from the right end in
series and all the vowels are replaced arrangement 1 is taken along with the
with their opposite letter as per the letter which is third from the left end and
English alphabetical series. second from right end of arrangement 2 is
Since the word AVOUCHMENT form the taken to form the third word.
meaningful word, condition 2 is implemented. Arrangement 1: AVOUCHMENT
 Vowels: A,O,U,E -> Z,L,F,V Arrangement 2: ZSLFZEJVKQ
 Consonants: V,C,H,M,N,T -> S,Z,E,J,K,Q New word: HULK
Thus, the newly formed word is ZSLFZEJVKQ Thus the newly formed word is FATESMJHULK
Arrangement 3:
Some random letters are picked from both Directions (11-15):
arrangements to form a word. If the letters form 11. Answer: C
a meaningful word then the word is written as it 12. Answer: D
is. If the letters did not form a meaningful word, 13. Answer: B
then the letters are written in reverse 14. Answer: A
alphabetical order from left to right. 15. Answer: E

Click Here For Bundle PDF Course | support@guidely.in Page 8 of 13


Bank Po Mains PDF Course 2024
Reasoning Day -5 (Eng)

Final arrangement:

We have,
 Japan gets the highest rank in tennis
which is three ranks higher than China in
boxing.
 US rank in tennis is same as Malaysia’s
rank in rowing, but neither of them ranks
first nor last.
While applying above conditions, we have
four possibilities:

Again we have,
 Japan gets lowest rank in archery and
second lowest rank in boxing.

Click Here For Bundle PDF Course | support@guidely.in Page 9 of 13


Bank Po Mains PDF Course 2024
Reasoning Day -5 (Eng)

 In boxing, Turkey gets two ranks higher


than Japan.
 In tennis, Turkey gets higher rank than
Malaysia and lower rank than US.(While
applying this conditions, case 4 gets
eliminated)
 Sum of the rank of India in boxing and
tennis is equal to rowing.
 India neither gets the highest nor lowest
rank.
Since, only possible combination is (2, 3,
and 5), While applying this condition, case
2 gets eliminated.
 China neither gets the second highest nor
lowest rank in any sports.
Thus, case (3) and case (1a) are not valid.

Click Here For Bundle PDF Course | support@guidely.in Page 10 of 13


Bank Po Mains PDF Course 2024
Reasoning Day -5 (Eng)

Thus, case 1 gives the final arrangement.

Directions (16-20)
16. Answer: C
17. Answer: B
18. Answer: D
19. Answer: A
20. Answer: E
Again we have,
Final arrangement:
 The one who gets the first rank in boxing
gets the last rank in tennis.
 In rowing, Japan gets higher rank than US
and lower rank than Malaysia.
 The rank difference between Turkey in
boxing and rowing is equal to China in
archery.
Since, china doesn’t get rank 2 or 6 in any
We have,
event, thus only possible rank for china in
 Four persons sit between E and F, who is
Archery is 3, and in rowing is 1.
not adjacent to B.
 In archery, US gets three ranks higher
 The one who gets 4% of profit sits third to
than India
the right of F, whose share percentage is
non-prime odd number.

Click Here For Bundle PDF Course | support@guidely.in Page 11 of 13


Bank Po Mains PDF Course 2024
Reasoning Day -5 (Eng)

That means, F doesn’t sit at the corner as


the persons sitting at the corners share is
a prime percentage.
From above conditions, we have four
possibilities:
Again we have,
 The difference between the share
percentage of C and H is 2%.
 C sits to the immediate left of the one
whose share in profit is Rs.12600.
Since, 21% of 60,000 is 12600, thus(Case
1,2 gets eliminated)
 The number of persons sitting between G
and D is same as the number of persons
sitting between E and B.
Again we have,
 B sits third to the left of I, who sits second
to the right of the one whose share is
Rs.1200.
Since, 2% of 60,000 is 1200, and the one
whose percentage is prime sits at corner)
 The sum of share received by F and I is
Rs.13800, where F and B don’t sit
adjacent to each other.
Since, 23% of 60,000 is 13800, thus only Again we have,

such possible combinations are (15, 8).  The share difference between A and the

By the above condition, case3 gets one who sits immediate right of the one

eliminated. whose share percentage is 15% is


Rs.7800.
Since 7800 is 13% of 60,000, thus
difference percentage must be 13%, (4
and 4a eliminated)
 The one whose percentage share is 16%
and D are not sitting together.

Click Here For Bundle PDF Course | support@guidely.in Page 12 of 13


Bank Po Mains PDF Course 2024
Reasoning Day -5 (Eng)

Thus, case 4.i gives the final arrangement.

Click Here For Bundle PDF Course | support@guidely.in Page 13 of 13


Bank Po Mains PDF Course 2024
Quantitative Aptitude Day -5 (Eng)

Quantitative Aptitude

Directions (1-3): Study the following information e) Data in statement I and statement II is
carefully and answer the given questions. necessary to answer the question
1) Train A travels from Chennai to Mumbai and
train B travels from Mumbai to Chennai. Find the 2) Find the ratio of number shirts folded by a boy,
time taken by train A to travel distance of 25% by a woman and by a man in one hour.
more than the original distance. I. 10 boys can fold 2000 shirts in a day and each
I. The train B leaves Mumbai at 11.00 AM and boy is working 8 hours per day
train A leaves Chennai at 9.00 AM and they met II.4 women can fold 1200 shirts in a day and
at 3.00 PM. each woman is working 10 hours per day
II. The total distance travelled by train B is III.8 men can fold 3200 shirts in a day and each
480km,when two trains A and B met and the ratio man is working 8 hours a day
of time taken by train A to train B to travel the a) Data in statement III alone is sufficient to
total distance is 3:2. answer the question and data in statement II or
III. Speed of train A is 33.33% less than that of statement I is not sufficient to answer the
train B and they met in 4hrs,after the train B question
starts traveling, train A travels for ‘t’ hours before b) Data in statement II alone is sufficient to
the train B starts traveling. answer the question and data in statement I or
a) Data in statement III alone is sufficient to statement III is not sufficient to answer the
answer the question and data in statement II or question
statement I is not sufficient to answer the c) All the given three statements are necessary
question to answer the question
b) Data in statement II alone is sufficient to d) Data in statement I alone is sufficient to
answer the question and data in statement I or answer the question and data in statement II or
statement III is not sufficient to answer the statement III is not sufficient to answer the
question question
c) All the given three statements are necessary e) Data in statement I and statement II is
to answer the question necessary to answer the question
d) Data in statement I alone is sufficient to
answer the question and data in statement II or 3) Hari’s family has five members – his father,
statement III is not sufficient to answer the mother, wife Pooja and his son Sundhar. Find
question the average age of a family 4 years ago?

Click Here For Bundle PDF Course | support@guidely.in Page 1 of 14


Bank Po Mains PDF Course 2024
Quantitative Aptitude Day -5 (Eng)

I. Ratio between present ages of Hari and his statement III is not sufficient to answer the
mother is 7:13 and difference between the question
present age of his father and mother is 10 years. c) All the given three statements are necessary
II. Average age of the family before Sundhar was to answer the question
born is 39. d) Data in statement I alone is sufficient to
III. Ratio between present ages of Pooja and answer the question and data in statement II or
Hari’s mother is 6:13 and Hari is 4years elder to statement III is not sufficient to answer the
Pooja. question
a) Datain all the given three statements are not e) Data in statement I and statement II is
sufficient to answer the question necessary to answer the question
b) Data in statement II alone is sufficient to
answer the question and data in statement I or

Direction (4 - 8): Study the following data carefully and answer the questions:
There are two cities A and B in a state. A bird flies from city A to city B in the morning for its food and
comes back to city A in the evening on five different days of a week starting from Monday to Friday.
Note 1: Flight speed (without wind) of bird from city A to city B in the morning is different on all the five
days while its flight speed (without wind) from city B to city A in the evening is also different on all the five
days.
2: Direction of wind remains same and is from city B to city A. Speed of wind in the morning is 100 km/h
while the speed of wind in the evening is 80 km/h.
First pie chart given below shows the per cent distribution of flight speed (without wind) of bird from city A
to city B in the morning on all the five days and sum of the pie chart is 2000 km/h.
Second pie chart given below shows the per cent distribution of flight speed (without wind) of bird from
city B to city A in the evening on all the five days and sum of the pie chart is 1600 km/h.

Click Here For Bundle PDF Course | support@guidely.in Page 2 of 14


Bank Po Mains PDF Course 2024
Quantitative Aptitude Day -5 (Eng)

4) If the distance between cities A and B is 540 c) 45 minutes


km, then what is the difference between time d) 27 minutes
taken by the bird to reach at city B from city A on e) None of these
Monday in morning and time taken by the bird
reach at city A from city B on Friday in evening? 5) On Wednesday morning, the bird starts from
a) 36 minutes city A towards city B and at the same time
b) 18 minutes another bird with its flight speed (without wind)

Click Here For Bundle PDF Course | support@guidely.in Page 3 of 14


Bank Po Mains PDF Course 2024
Quantitative Aptitude Day -5 (Eng)

240 km/h starts from city B towards city A. If increased by 40 km/h and the bird also increases
difference between total distances travelled by its flight (without wind) speed while returning
both the bird before meeting is 180 km, then from B towards A. Now, time taken by bird to
what is the distance between cities A and B? reach A from B is decreased by 4.2 minutes than
a) 310 km the calculated time, then what is the increment in
b) 320 km the flight (without wind) speed of bird?
c) 330 km a) 80 km/h
d) 340 km b) 100 km/h
e) None of these c) 120 km/h
d) 140 km/h
6) If the bird starts flying from city A towards city e) None of these
B at 6: 00 AM in the morning on Friday and
search for food in the city. In the process of 8) If on Thursday, bird flies from city A to city B in
searching food it spends 7 hours 9 minutes there the morning but with its flight speed (without
and comes back to the city A at 3: 00 PM in the wind) 20% more than its original flight (without
evening on the same day, then how far city B is wind) speed and comes back to the original point
from city A? Assume there is no other loss of in the evening on the same day with speed along
time other than flight and searching. the wind as mention in the charts, then how
a) 300 km much less time the bird will take than the original
b) 280 km time in its flight if cities A and B are 160 km
c) 380 km apart?
d) 320 km a) 3.2 minutes
e) None of these b) 5.4 minutes
c) 4.8 minutes
7) If distance between cities A and B is 196 km d) 3.6 minutes
and on Tuesday evening the speed of wind is e) None of these

Directions [9 - 12]: Read the following information carefully and answer the questions based on it.
The table depicts certain missing values in terms of variables, regarding posts delivered in 6 cities A, B,
C, D, E and F. Posts delivered includes postcards delivered and couriers delivered. Certain couriers get
damaged during the transit. The two variables used, ‘x’ and ‘y’ will vary in values according to all the
questions and the given table also shows percentage distribution of couriers damaged out of total
couriers delivered to the respective cities.

Click Here For Bundle PDF Course | support@guidely.in Page 4 of 14


Bank Po Mains PDF Course 2024
Quantitative Aptitude Day -5 (Eng)

9) Total postcards and couriers delivered to city delivered to city A. Now find the average of
F are in what proportion with respect to those couriers delivered in good condition to cities C,
delivered to city E, if postcards delivered to city F D, E and F?
are 50% more than those delivered to city E? a) 268.5
a) 42:11 b) 278.5
b) 32:11 c) 260.5
c) 31:21 d) 270.5
d) 32:21 e) None of these
e) None of these
12) 20% of the average postcards delivered to
10) Find the value of (x + y), if 25% of the value cities C, D and E, is 26 less than 10% of the total
of total couriers delivered is 695? couriers delivered to cities C, D and E. How
a) 25 many postcards were delivered to city D?
b) 35 a) 240
c) 32 b) 250
d) 15 c) 265
e) None of these d) 260
e) None of these
11) Couriers delivered to city E, which are in
good condition, are 40.5% of 120% of couriers
Directions (13-16): Study the following information carefully and answer the questions given below.
The line graph shows the sum of total number of electric scooters and total number of non-electric
scooter sold in five cities[i.e. A, B, C, D and E] and the difference of total number of electric and non-
electric scooter sold in five cities is also given. The given table shows the ratio of 100cc and 120cc
electric scooter sold out of total number of electric scooter sold and percentage of 100cc non-electric
scooter sold out of total number of non-electric scooter sold in the given cities.

Click Here For Bundle PDF Course | support@guidely.in Page 5 of 14


Bank Po Mains PDF Course 2024
Quantitative Aptitude Day -5 (Eng)

13) Number of non-electric scooters sold in d) Quantity II < Quantity I <Quantity III
cities D and E is more than the number of e) Quantity I = Quantity II >Quantity III
electric scooters sold in the same city.
Quantity I: Find the total number of 100cc 14) Number of electric scooters sold in city A and
scooter sold in city D? city C is more than the number of non-electric
Quantity II: Find the difference between the total scooters sold in the same city. Find the
number of electric scooters and the total number difference between the total number of 100cc
of non-electric scooters sold in city D and E electric scooter sold in city A and C together and
together? the total number 120cc non-electric scooter sold
Quantity III: Find the total number of 120cc in city A and C together?
scooters sold in city E? a) 97
a) Quantity I < Quantity II < Quantity III b) 95
b) Quantity I < Quantity II >Quantity III c) 94
c) Quantity I > Quantity II <Quantity III d) 123

Click Here For Bundle PDF Course | support@guidely.in Page 6 of 14


Bank Po Mains PDF Course 2024
Quantitative Aptitude Day -5 (Eng)

e) 90 16) Total number of scooters sold in city F is


40% more than the total number of scooters sold
15) Number of 120cc electric scooters sold in city in city A. Ratio of electric scooter and non-
E is 55 which is equal to the number of 100cc electric scooter sold in city F is 4:3. Number of
non-electric scooters sold in the same city. Find electric scooters sold in city F is 40 more than
the percentage of 100cc electric scooters sold in the number of electric scooters sold in city A.
the city E out of the total number of non-electric Then find the number of non-electric scooters
scooters sold in city E? sold in city A?
a) 22% a) 80
b) 25% b) 110
c) 28% c) 160
d) 29% d) 105
e) None of these e) None of these

Directions (17-20): Read the data carefully and answer the following questions.
The bar graph below shows the total number of teachers and the number of students studying in five
different colleges A, S, M, P and V.

Note: Number of students in bar graph is 10% of its actual value.


The given table below shows the percentage of male in teachers and the ratio between boys to girls in
students.

Click Here For Bundle PDF Course | support@guidely.in Page 7 of 14


Bank Po Mains PDF Course 2024
Quantitative Aptitude Day -5 (Eng)

17. What is the difference between the total 19. How many extra teachers are required to
number of teachers in college M and P together assign all the students, If 8 students are
and the number of boys in college V and S assigned to each teacher in college V.
together? a) 4
a) 320 b) 8
b) 332 c) 3
c) 300 d) 5
d) 290 e) None of these
e) None of these
20. If 90% and 80% of students have cleared
18. The number of girl students in college M is semester exams in college A and S respectively,
approximately how much more or less and the ratio between boys to girls who cleared
percentage than the number of boy students in exams in college A and S are 4:3 and 11:5
college P. respectively. Find the number of girls cleared the
a) 10% exam in both colleges A and S.
b) 14% a) 375
c) 15.5% b) 255
d) 12.5% c) 235
e) None of these d) 225
e) None of these

Click Here For Bundle PDF Course | support@guidely.in Page 8 of 14


Bank Po Mains PDF Course 2024
Quantitative Aptitude Day -5 (Eng)

Click Here to Get the Detailed Video Solution for the above given Questions
Or Scan the QR Code to Get the Detailed Video Solutions

Answer Key with Explanation


1) Answer: E 3) Answer: A
By combining statement I and II Cannot find the average age of a family because
Total distance = 480 insufficient information about the age of Sundhar
Given time ratio A:B = 3:2 and also we don’t know that his father is the
So speed of train A is ‘2x’ and speed of train B is eldest person or his mother is the eldest person
‘3x’ in the family.
Let the train A travelled a distance be 4x before
the train B started to travel, Direction (4 - 8):
(480 – 4x) / 5x = 4 Flight speed of bird (without wind) on Monday
x = 20 from A to B = 15% of 2000 = 300 km/h
Speed of train A = 40km/hr Flight speed of bird (without wind) on Tuesday
Required answer = 480*(125/100)*40= 15h from A to B = 20% of 2000 = 400 km/h
Flight speed of bird (without wind) on
2) Answer: C Wednesday from A to B = 10% of 2000 = 200
Combining I, II and III, km/h
No. of shirts fold by a boy in one hour = Flight speed of bird (without wind) on Thursday
2000/(10*8) = 25 from A to B = 25% of 2000 = 500 km/h
No. of shirts fold by a woman in one hour = Flight speed of bird (without wind) on Friday from
1200 / (4*10) = 30 A to B = 30% of 2000 = 600 km/h
No. of shirts fold by a man in one hour = 3200 / Flight speed of bird (without wind) on Monday
(8 * 8) = 50 from B to A = 20% of 1600 = 320 km/h
Boy: woman: man = 25:30:50 = 5:6:10

Click Here For Bundle PDF Course | support@guidely.in Page 9 of 14


Bank Po Mains PDF Course 2024
Quantitative Aptitude Day -5 (Eng)

Flight speed of bird (without wind) on Tuesday Required difference = 340t – 100t = 180
from B to A = 30% of 1600 = 480 km/h 240t = 180
Flight speed of bird (without wind) on t = 0.75 hours
Wednesday from B to A = 25% of 1600 = 400 Distance between both the cities = 440 * 0.75 =
km/h 330 km
Flight speed of bird (without wind) on Thursday
from B to A = 15% of 1600 = 240 km/h 6) Answer: A
Flight speed of bird (without wind) on Friday from Time between 6: 00 AM in the morning to 3: 00
B to A = 10% of 1600 = 160 km/h PM in the evening on the same day = 9 hours =
540 minutes
Time taken in search of food = 7 hours 9 minutes
= (7 * 60) + 9 = 429 minutes
Time taken in flight = 540 – 429 = 111 minutes
Let the distance between cities A and B is ‘D’
km.
According to the question:
4) Answer: D (D/500) + (D/240) = (111/60)
Distance between cities A and B = 540 km (12D + 25D)/6000 = 111/60
Time taken by the bird to reach at city B from city 37D = (111/60) * 6000
A on Monday in morning = 540/200 = 2.7 hours 37D = 11100
Time taken by the bird reach at city A from city B D = 300
on Friday in evening = 540/240 = 2.25 hours Hence, distance between cities A and B is 300
Required difference = 2.7 – 2.25 = 0.45 hours = km.
27 minutes

7) Answer: B
5) Answer: C Let increment in the flight speed of bird = ‘x’
Flight speed (without wind) of another bird on km/h
Wednesday morning = 240 km/h Effective speed of bird on Tuesday evening
Flight speed (with wind) of another bird on while returning from B to A = (560 + 40 + x) =
Wednesday morning = 240 + 100 = 340 km/h (600 + x) km/h
Effective speed of both the birds = 100 + 340 = Calculated time taken by bird = (196/560) * 60 =
440 km/h 21 minutes
Let both the birds meet after travelling for ‘t’
hours.

Click Here For Bundle PDF Course | support@guidely.in Page 10 of 14


Bank Po Mains PDF Course 2024
Quantitative Aptitude Day -5 (Eng)

Actual time taken by bird = (21 – 4.2) = (196 * Couriers delivered to city B = 21.6(x + y)
60)/(600 + x) Couriers delivered to city C = 420
600 + x = 700 Couriers delivered to city D = 400
x = 100 Couriers delivered to city E = 360
Increment in the flight (without wind) speed of Couriers delivered to city F = 560
bird = x = 100 km/h According to question,
As, 25% x total couriers delivered = 695
8) Answer: C So,
Changed flight speed (without wind) of the bird = 25% x [500 + 21.6(x + y) + 420 + 400 + 360 +
120% of 500 = 600 km/h 560] = 695
Changed flight speed (with wind) of the bird = Thus, value of (x + y) = 25
600 – 100 = 500 km/h Hence, answer is option A
Original flight time of the bird = (160/400) +
(160/320) = 0.4 + 0.5 = 0.9 hours = 54 minutes 11) Answer: E
Original flight time of the bird = (160/500) + According to question,
(160/320) = 0.32 + 0.5 = 0.82 hours = 49.2 Couriers delivered to city A = 500
minutes Couriers delivered in good condition to city E =
Required less time = 54 – 49.2 = 4.8 minutes 40.5% x 120% x couriers delivered to city A
Thus,
9) Answer: D Couriers delivered in good condition to city E =
According to question, 40.5% x 120% x 500 = 243
Postcards delivered to city F = 150% x postcards Couriers delivered in good condition to city C =
delivered to city E 75% x 420 = 315
Postcards delivered to city F = 3/2 x 480 = 720 Couriers delivered in good condition to city D =
Total postcards and couriers delivered to city F = 65% x 400 = 260
720 + 560 = 1280 Couriers delivered in good condition to city F =
Total postcards and couriers delivered to city E = 60% x 560 = 336
480 + 360 = 840 Average of couriers delivered in good condition
Ratio = 1280:840 = 32:21 to cities C, D, E and F = (315 + 260 + 243 +
Hence, answer is option D 336)/4 = 1154/4 = 288.5
Hence, answer is option E
10) Answer: A
Couriers delivered to city A = 500 12) Answer: E

Click Here For Bundle PDF Course | support@guidely.in Page 11 of 14


Bank Po Mains PDF Course 2024
Quantitative Aptitude Day -5 (Eng)

Postcards delivered to city C = 630 14) Answer: D


Postcards delivered to city D = 8x + 15y Number of non-electric scooter sold in A
Postcards delivered to city E = 480 =160/2=80 and the number of electric scooter
20% of the average postcards delivered = 1/5 x sold in A = [240/2] = 120
[630 + (8x + 15y) + 480]/3 = (1110 + 8x + Number of non-electric scooter sold in C
15y)/15 =320/2=160 and number of electric scooter sold
Couriers delivered to city C = 420 in C = [420/2] =210
Couriers delivered to city D = 400 Total 100cc electric scooter sold in A and C
Couriers delivered to city E =360 together
10% of the total couriers delivered = 1/10 x (420 =120*(5/8)+210*(4/7) = 195
+ 400 + 360) = 118 Total 120cc non-electric scooter sold in A and C
According to question, together
118 - (1110 + 8x + 15y)/15 = 26 =80*(40/100)+160*(25/100)=72
So, So, difference = 195 – 72 =123
8x + 15y = 270
Thus, 15) Answer: B
270 postcards were delivered to city D Number of 100cc electric scooters sold in city E
Hence, answer is option E = 55*(5/11)=25
Total number of electric scooters sold in city E
13) Answer: C =25+55=80
Number of non electric scooter sold in city D Number of non-electric scooters sold in city E =
=280/2=140 and number of electric scooter sold 180-80=100
in city D = [240/2] =120 So, required percentage = [25/100]*100=25%
Number of non-electric scooter sold in city E
=200/2=100 and number of electric scooter sold 16) Answer: A
in city E = [160/2] =80 Number of non-electric scooter sold in A
Quantity I = [200-(200*(140/100)*(4/7)-40)] = 80
Required sum = 120*(3/5)+140*(30/100)=114
Quantity II Directions (17-20):
Required difference = [140+100] – [120+80] =40 For college A,
Quantity III: Number of teachers = 40
Required sum = 80*(11/16) + 100*(45/100)=100 Numbers of students = 35 * 100/10 = 350
Hence, Quantity I > Quantity II <Quantity III Ratio between boys to girls in students = 3:2

Click Here For Bundle PDF Course | support@guidely.in Page 12 of 14


Bank Po Mains PDF Course 2024
Quantitative Aptitude Day -5 (Eng)

Number of boys = 350*3/5 = 210 Number of teachers = 30


Number of girls = 350 – 210 = 140 Numbers of students = 28 * 100/10 = 280
Percentage of male in teachers =60% Ratio between boys to girls in students = 9:5
Number of male teachers= 40*60/100 = 24 Number of boys = 280 * 9/14 = 180
Number of female teachers = 40 -24 =16 Number of girls = 280 – 180 = 100
For college S Percentage of male in teachers = 80%
Number of teachers = 45 Number of male teachers= 30 * 80/100 = 24
Numbers of students = 40 * 100/10 = 400 Number of female teachers = 30 – 24 = 6
Ratio between boys to girls in students = 5:3
Number of boys = 400 * 5/8 = 250
Number of girls = 400 - 250 = 150
Percentage of male in teachers = 40%
Number of male teachers= 45 * 40/100 = 18
Number of female teachers = 45 – 18 = 27
For college M,
Number of teachers = 48
17. Answer: B
Numbers of students = 42 * 100/10 = 420
Number of teachers in college M and P = 48+50
Ratio between boys to girls in students = 4:3
=98
Number of boys = 420*4/7=240
Number of boy students in college V and S =
Number of girls = 420 - 240 = 180
180+250=430
Percentage of male in teachers = 50%
Required difference = 430 – 98 =332
Number of male teachers= 48 * 50/100 = 24
Number of female teachers = 48 – 24 = 24
18. Answer: D
For college P,
Number of girl students in college M = 180
Number of teachers = 50
Number of boy students in college P = 160
Numbers of students = 32 * 100/10 = 320
Required percentage = (180-160)/160*100 =
Ratio between boys to girls in students = 1:1
12.5%
Number of boys = 320 * 1/2 = 160
Number of girls = 320 – 160 = 160
19. Answer: D
Percentage of male in teachers = 70%
Number of students in college V = 280
Number of male teachers= 50 * 70/100 = 35
Number of teachers in college V = 30
Number of female teachers = 50 - 35 = 15
Number of students assigned = 30*8 =240
For college v,
Remaining students = 280 -240 =40

Click Here For Bundle PDF Course | support@guidely.in Page 13 of 14


Bank Po Mains PDF Course 2024
Quantitative Aptitude Day -5 (Eng)

Extra teachers needed = 40/8 =5 Number of students cleared in college S =


400*80/100 =320
20. Answer: C Number of girls who cleared in college A =
Number of students studying in college A = 350 315*3/7 =135
Number of students cleared in college A = Number of girls who cleared in college S =
350*90/100 = 315 320*5/16 =100
Number of students studying in college S = 400 Required total number of girls who cleared in
both colleges of A and S= 135+100 =235

Click Here For Bundle PDF Course | support@guidely.in Page 14 of 14


Bank Po Mains PDF Course 2024
English Day - 5

English Language
Directions (1-7): Read the following passage and with appropriate prescience and planning,
answer the questions that follow. Governments can manage the challenges from
Population ageing is a defining global trend of population ageing while enhancing opportunities
our time. People are living longer, and more are for all people to thrive and ensuring that no one
older than ever before. Spectacular is left behind. Population ageing needs to be
improvements in health and survival and widely understood as more than just a set of
diminution in fecundity have driven this discrete concerns mainly for one group of people
momentous shift, which has begun or is who have advanced beyond a given age. Ageing
expected to begin soon in all countries and touches all parts of economies and societies,
areas. This change brings both challenges and from health care and education to employment
opportunities as countries strive to achieve the and taxation. Each stage of life can contribute to
Sustainable Development Goals (SDGs). In or detract from well-being at older ages.
2022, the world marked the twentieth anniversary Population ageing signals our extraordinary
of the adoption of the Madrid International Plan collective success in improving living conditions
of Action on Ageing. To commemorate this for billions of people around the world. Better
landmark, the World Social Report 2023 explores sanitation and medical therapies, greater access
the economic and social entailments of the to education and family planning, and strides
ageing of the human population. It builds on the towards gender equality and women’s
Plan of Action’s framework for national policies to empowerment have all contributed to, and in
create equitable, compendious societies for some cases benefitted from, the steady move
people of all ages, providing recommendations to from high to low levels of fertility and mortality.
put the rights and well-being of older persons at These advances have ushered in an era where
the center, across the life course. Population rapid population growth is slowly coming to an
ageing is an ineluctable result of the end, accompanied by a gradual but permanent
demographic transition towards longer lives and shift towards older ages. Over several decades,
smaller families. While the shift towards older both the number and population share of older
populations is largely irredeemable, collective persons have risen globally, while the number
actions and policy decisions shape its path and and share of children and youth have begun to
consequences. Postponing critical measures that shrink. By 2050, the number of persons aged 65
allow societies to benefit from and adapt to years or older is expected to double, surpassing
population ageing would impose high social, 1.6 billion. Currently, population ageing is
economic, fiscal and health-related costs, for furthest along in Europe and Northern America,
both current and future generations. By contrast, Australia and New Zealand, and most of Eastern

Click Here For Bundle PDF Course | support@guidely.in Page 1 of 14


Bank Po Mains PDF Course 2024
English Day - 5

and South-Eastern Asia. In most countries of education. In 2020, the World Health
those regions, the proportion of older persons – Organization and the United Nations designated
by convention, those aged 65 years or older – 20212030 as the Decade of Healthy Ageing. Its
exceeds 10 per cent and in some cases 20 per purpose is to promote strategies, grounded in
cent of the total population. Most parts of sub- solid evidence, that support well-being among
Saharan Africa and Oceania (excluding Australia older people. It advocates for developing and
and New Zealand) are still in an early stage of maintaining functional abilities, recognizing that
this transition, while most countries in Central these depend on each individual’s immanent
and Southern Asia, Western Asia and Northern capacity, the surrounding environment and
Africa, and Latin America and the Caribbean are interactions between the two.
at an intermediate stage. Declining mortality 1. What is the tone of the passage?
throughout the life course has driven the a. subjective
increase of life expectancy at birth in most b. satire
countries and globally. Greater longevity has c. jocular
accompanied a narrowing of the age range in d. critical
which most deaths occur. In the past, death was e. None of these
common at all ages. Many children died from
infectious diseases, for example, and women 2. Which of the following is/are reason/reasons
frequently perished in childbirth. In most for people to live longer than before according to
countries today, “premature death” before age 60 the passage?
or 70 is relatively rare. Greater global life a. Reduction in the ability of the women to bear
expectancy reflects underlying improvements in and produce offspring.
health. In countries with available data, the b. Striking advancements in the fields of health
number of years lived in good health has and survival.
climbed, accounting for most of the increase in c. Decreased prevalence of death-inducing and
years lived overall. Statistical averages hide grievous diseases amongst mankind.
broad disparities in life expectancy, however, d. Only a and b
including by sex and socioeconomic status. In e. Options a, b, and c
almost all societies, women live longer than men
on average, and the rich longer than the poor. 3. Which of the following statement/statements
These differences stem partly from poor nutrition is/are TRUE according to the passage?
and exposures to environmental and a. The World Social Report 2023 digs into the
occupational hazards that are more common economic and social implications of the ageing of
among men and people with limited income and human population to commemorate the

Click Here For Bundle PDF Course | support@guidely.in Page 2 of 14


Bank Po Mains PDF Course 2024
English Day - 5

anniversary of the Madrid International Plan of b. excrescencies


Action on Ageing which was adopted in 2022. c. dereliction
b. Population ageing is an avoidable result of the d. foresight
demographic transition towards longer lives and e. blotches
smaller families.
c. The shift towards older populations is largely 7. Which of the following is/are the
irreversible. synonym/synonyms of the word ‘immanent’?
d. Only a and c a. inexorable
e. Options a, b, and c b. inadvertent
c. innate
4. Which of the following statement/statements d. inept
is/are FALSE according to the passage? e. None of the above
a. The advances in the living conditions for
billions of people around the world have paved Directions (8-12): Each of the following questions
the way to an era where rapid growth in has a sentence that is divided into 4 parts. One
population is slowly coming to a halt. of these parts may/may not contain a
b. Longer life-span has resulted in the shrinking grammatical error in it. Identify the part that is
of the age range between which most deaths are grammatically incorrect. If all the parts of the
recorded to occur. sentence are grammatically right, then mark
c. Countries like Australia and New Zealand are option ‘e’.
in the early stage of population ageing. 8. Kiran had been trying to (A)/ lead a happy and
d. Options a and c peaceful life since (B)/ he had less problems
e. None of the options a, b, and c is false when (C)/ compared to the others around him
(D)/ No error (E).
5. Which of the following is/are the a. A
antonym/antonyms for the word ‘compendious’? b. B
a. embracive c. C
b. panoramic d. D
c. comprehensive e. E
d. exhaustive
e. None of the above 9. Tanya knew very little of the abilities and (A)/
6. Which of the following is/are the competencies of the client with (B)/ who she had
synonym/synonyms of the word ‘prescience’? promised to venture into (C)/ new business deals
a. blemishes for the next few months (D)/ No error (E).

Click Here For Bundle PDF Course | support@guidely.in Page 3 of 14


Bank Po Mains PDF Course 2024
English Day - 5

a. A c. C
b. B d. D
c. C e. E
d. D
e. E Directions (13-17): In the following questions two
columns are given containing three
10. Vimal is not cognizant of no ways (A)/ to get Sentences/phrases each. A sentence/phrase
the job done quickly (B)/, but he is not willing to from the first column may or may not connect
(C)/ outsource professionals for the same (D)/ No with another sentence/phrase from the second
error (E). column to make a grammatically and
a. A contextually correct sentence. Each question has
b. B five options, four of which display the
c. C sequence(s) in which the sentences/phrases can
d. D be joined to form a grammatically and
e. E contextually correct sentence. If none of the
options given forms
11. Meena worked in her warehouse for (A)/ a correct sentence after combination, mark
hours yesterday, and thus, she (B)/ was unable option (e), i.e. “None of the above” as your
to make it to her appointment (C)/ with a new answer.
client on time (D)/ No error (E). 13.
a. A Column I Column II
b. B A. Markets regulator D. after considering
c. C SEBI barred IIFL some other factors that
d. D Securities, one of the are mentioned
e. E leading domestic elaborately in the order
broking houses, released in June this
12. The team will attend a meeting from noon year.
(A)/ to early evening tomorrow to discuss the B. SEBI had E. from taking on new
pros (B)/ and cons of investing a portion of considered cancellation clients for the next two
company shares (C)/ in ventures like real estate, of the certificate of the years for violating
interior designing, and many more (D)/ No error IIFL securities as a regulatory forms.
(E). broken but refrained
a. A from doing that
b. B

Click Here For Bundle PDF Course | support@guidely.in Page 4 of 14


Bank Po Mains PDF Course 2024
English Day - 5

C. IIFL Securities has F. using funds of one a. A-D, B-E, C-F


been penalized for client for meeting the b. A-F, B-E, C-D
indulging in unfair obligations of another. c. A-E, B-D, C-F
trading practices and d. A-F, B-D, C-E
a. A-D, B-E, C-F e. None of the above
b. A-F, B-E, C-D
c. A-E, B-D, C-F 15.
d. A-F, B-D, C-E Column I Column II
e. None of the above A. Describing Prime D. and talks about how
Minister Modi’s visit to defense industrial
14. the U.S. this June as a ecosystems could
Column I Column II milestone in India-U.S. cooperate much
A. Union tourism D. and focuses on relations, foreign better.
minister G Kishan shifting the contribution secretary said a
Reddy proposed that of its sectors towards roadmap for defense
the G20 members green, inclusive, and industrial cooperation
should take concrete circular economic B. The defense E. before departing for
steps growth. roadmap essentially Washington D.C.
B. While addressing E. should focus on focusses on aspects of where he held bilateral
the fourth G20 tourism green initiatives, defense co-production talks with the U.S.
working group meeting, inclusive strategies, and co-development president and
G Kishan Reddy and circular economic addressed a joint
proposed that the growth to promote sitting of the U.S.
tourism industry sustainability and congress.
renewable energy C. The prime minister F. will be among the
management. met some prominent key outcomes of what
C. The Indian tourism F. to drive a collective personalities in New will be Modi’s first
industry has enforced a change in how plastic York and participate in state visit to the
national strategy called pollution can be tackled a capacity-building country and his
the Swadesh tourism globally while giving event seventh as PM.
that acknowledges special attention to the a. A-D, B-E, C-F
guidelines for tourism industry of their b. A-F, B-E, C-D
sustainable tourism countries. c. A-E, B-D, C-F

Click Here For Bundle PDF Course | support@guidely.in Page 5 of 14


Bank Po Mains PDF Course 2024
English Day - 5

d. A-F, B-D, C-E 17.


e. None of the above Column I Column II
A. Victims of extensive D. but the construction
16. burns and acid attack of the same was
Column I Column II patients with deep, delayed due to the
A. The state D. in Srinagar, severe scars now sudden outbreak of the
investigation agency of Anantnag, Pulwama, have a fresh ray of pandemic three years
Jammu and Kashmir and Kupwara districts hope ago.
police carried out in a case pertaining to B. The proposal to set E. in the form of a
searches at several the misuse of social up a skin bank at the dedicated skin bank at
locations media platforms for Safdarjung hospital at center-run Safdarjung
unlawful and New Delhi was hospital in the nation’s
secessionist activities. initiated in 2012 after capital New Delhi.
B. The identified E. in Anantnag have gaining the approval
entities are suspected been found targeting of the health ministry
of conniving with government servants C. The initiative is the F. which are majorly
foreign associates to by hindering their first-of-its-kind in located in Maharashtra,
further their nefarious ability to perform their Delhi-NCR and north Odisha, Madhya
agenda and specifically lawful duties. India, while presently Pradesh, and the
the entities India retains 16 skin southern states like
C. The searches led to F. would be banks Tamil Nadu and
the seizure of meticulously analyzed Karnataka.
substantial digital and to build a strong case a. A-D, B-E, C-F
physical evidence, against the accused b. A-F, B-E, C-D
including mobile entities. c. A-E, B-D, C-F
phones and sim cards, d. A-D, B-F, C-E
which e. None of the above
a. A-D, B-E, C-F
b. A-F, B-E, C-D Directions (18-22): Given below is a word that is
c. A-E, B-D, C-F followed by 3 sentences. Each of the three
d. A-D, B-F, C-E sentences includes the word. Identify the
e. None of the above sentence/sentences that best express/expresses

Click Here For Bundle PDF Course | support@guidely.in Page 6 of 14


Bank Po Mains PDF Course 2024
English Day - 5

the meaning of the word. Choose option e when c. Only C


the word is not suitable for any of the sentences. d. All statements A, B, and C
18. HOBBLE e. None of the sentences A, B, and C is right
A. Poverty not only demeans our society but its
cost also hobbles the growth of our economy to a 20. DEPLORE
significant extent. A. We, as humans, should deplore the
B. Sita picked up her crutches and hobbled development of certain inventions like nuclear
across the room since she had retained a serious weapons to guarantee peace and harmony for
knee injury last week. the future generations.
C. Mules and horses typically required herding B. Early mornings and late evenings are some of
day and night and often had to be staked out on the best parts of the day to deplore the town and
a rope or hobbled to keep them from straying or get acquainted with its scenic beauty.
wandering. C. The leader deplored his actions and wilfully
a. Only A accepted to have created a toxic environment
b. Only B through his incompetent and biased governance.
c. Only C and A a. Only C
d. Options A, B, and C b. Only A and B
e. None of the sentences A, B, and C is right c. Only A and C
d. Only B and C
19. COMPORT e. None of the sentences A, B, and C is right
A. The employee was fired from his position
yesterday since his actions did not comport with 21. RESCIND
the guidelines and code of conduct of the A. Match tapes are routinely studied after the
company. final whistle to either get red cards rescinded on
B. The grieving relatives comported themselves appeal or charges brought for malicious tackles
with grace and dignity during the funeral that the officials miss.
proceeding and briefly expressed their B. The memories of Hera’s early childhood
condolences to the family of the deceased to let rescinded as she grew older and became mature
them mourn in peace and solitude. with time.
C. Free healthcare and education were of little C. With the right combination of a proven
comport amongst other obstacles when food treatment plan and natural remedies, a
rationing and poverty were all around. rescinding hairline can often grow back.
a. Only A and B a. Only C
b. Only B and C b. Only B and C

Click Here For Bundle PDF Course | support@guidely.in Page 7 of 14


Bank Po Mains PDF Course 2024
English Day - 5

c. All statements A, B and C are correct B. Because our company just recently opened its
d. Only A doors, we are inchoate and are not offering all of
e. None of the statements A, B, C is right our services yet.
C. The designs of the newly launched cars
22. INCHOATE successfully inchoate art and technology.
A. Since the power went out in the building, the a. Only C
electrical service has been inchoate, leaving b. Options A and B
many floors without lights. c. Options A and C
d. All statements A, B, and C
e. None of the statements A, B, and C is right
Click Here to Get the Detailed Video Solution for the above given Questions
Or Scan the QR Code to Get the Detailed Video Solutions

Answer Key with Explanation


1. Answer: E Critical - A fault-finding attitude of the author is
Subjective - A subjective tone uses words that frequently described by a critical tone, hence in a
describe feelings, judgments, or opinions. The negative meaning.
details are likely to include experiences, senses, The tone of the passage does not match with
feelings, and thoughts. any of the tones mentioned above. Hence,
Satire - uses sarcasm or irony to make fun of option e is the right answer.
something. 2. Answer: D
Jocular - When the author is humorous, he/she In the first paragraph of the passage, there is a
tries to make the context of the passage funny line which states that ‘People are living longer,
and amusing. and more are older than ever before.
Spectacular improvements in health and survival
and diminution in fecundity have driven this

Click Here For Bundle PDF Course | support@guidely.in Page 8 of 14


Bank Po Mains PDF Course 2024
English Day - 5

momentous shift’. Here, diminution means a Only option c is true.


decrease or reduction and fecundity means the
ability to produce offspring. Thus, options a and 4. Answer: C
b are the reasons for the momentous shift. Thus, Option a is true. There is a line in the second
option d is the right answer. paragraph of the passage that states that
‘Population ageing signals our extraordinary
3. Answer: C collective success in improving living conditions
In the first paragraph of the passage, there is a for billions of people around the world. Better
line which states that ‘In 2022, the world marked sanitation and medical therapies, greater access
the twentieth anniversary of the adoption of the to education and family planning, and strides
Madrid International Plan of Action on Ageing. towards gender equality and women’s
To commemorate this landmark, the World empowerment have all contributed to, and in
Social Report 2023 explores the economic and some cases benefitted from, the steady move
social entailments of the ageing of the human from high to low levels of fertility and mortality.
population’. Here, explores means to inquire or These advances have ushered in an era where
dig into; and entailments means implications. It rapid population growth is slowly coming to an
is also given than the Madrid international plan end’. Here the advances refer to the ones
celebrated its 20th anniversary in 2022. Thus, it described in the second line of the text within the
should have been adopted in 2002. Thus, option inverted commas. Here, urged means to steer or
a in inorrect. pave the way for something. Thus, option a is
In the first paragraph of the passage, there is a true.
line which states that ‘Population ageing is an Option b is true. There is a line in the second
ineluctable result of the demographic transition paragraph of the passage that states that
towards longer lives and smaller families’. Here, ‘Greater longevity has accompanied a narrowing
ineluctable means impossible to avoid or evade. of the age range in which most deaths occur’.
But the statement in option c does not match Here longevity means the time period during
with the statement mentioned in the passage. which something exists or lasts or lifespan.
Thus, option b is incorrect. Thus, option b is true.
In the first paragraph of the passage, there is a Option c is false. There is a line in the second
line which states that ‘the shift towards older paragraph of the passage which states that
populations is largely irredeemable’, here ‘Most parts of sub-Saharan Africa and Oceania
irredeemable means irreversible. Thus, option c (excluding Australia and New Zealand) are still in
is correct. an early stage of this transition’. Here, the

Click Here For Bundle PDF Course | support@guidely.in Page 9 of 14


Bank Po Mains PDF Course 2024
English Day - 5

countries Australia and New Zealand are Innate (adjective) – as in inherent; being a part of
excluded from the list of countries in the early the innermost nature of a person or thing
stage of this transition – population ageing. Inexorable (adjective) – as in inevitable; as in
Thus, option c is false. inevitable
Thus, option c is the right answer. Inadvertent (adjective) – Happening by chance,
unexpectedly or unintentionally
5. Answer: E Inept (adjective) - Generally incompetent and
Compendious, embracive, panoramic, ineffectual
comprehensive, and exhaustive are all Only option c is synonymous to the word
adjectives that unanimously mean to cover or immanent. Thus, option c is the right answer.
include everything or all important points. Thus,
none of these options is an antonym. 8. Answer: C
Thus, option e is the right answer. The use of the quantifier is wrong in part C.
‘Fewer’ means a quantifiable number and thus, it
6. Answer: D is used to denote a definite number but, ‘Less’ is
Prescience (noun) - T he special ability to see or used in a non-quantifiable situation to denote
know about events before they actually occur that something which is abstract is less in size or
Foresight (noun) - T he special ability to see or amount. Thus, ‘less’ should be replaced by
know about events before they actually occur ‘fewer’ is part C to make the sentence
Blemishes (noun) - something that spoils the grammatically right.
appearance or completeness of a thing; defects The right sentence is:
Blotches (noun) - something that spoils the Kiran had been trying to lead a happy and
appearance or completeness of a thing; defects peaceful life since he had fewer problems when
Excrescencies (noun) - something that spoils the compared to the others around him.
appearance or completeness of a thing; defects
Dereliction (noun) - Wilful negligence 9. Answer: C
Only option d is synonymous to prescience. Part C is grammatically incorrect. ‘Who’ is a
Thus, option d is the right answer. subjective pronoun, whereas ‘whom’ is an
objective pronoun. If the word, ‘he’ can be
7. Answer: C substituted into the sentence, use ‘who’. If the
Immanent (adjective) – as in inherent; being a word ‘him’ can be substituted into the sentence,
part of the innermost nature of a person or thing use ‘whom’. Thus, the word ‘who’ should be

Click Here For Bundle PDF Course | support@guidely.in Page 10 of 14


Bank Po Mains PDF Course 2024
English Day - 5

replaced by ‘whom’ to make the sentence Meena was working in her warehouse for hours
grammatically right. yesterday, and thus, she was unable to make it
The right sentence is; to her appointment with a new client on time.
Tanya knew very little of the abilities and
competencies of the client with whom she had 12. Answer: A
promised to venture into new business deals for Use the future continuous tense for future
the next few months. actions happening over a period of time,
especially when a specific time is mentioned.
10. Answer: A Thus, in part A, the phrase ‘will attend’ (simple
Option a is grammatically incorrect. Double future) should be replaced by ‘will be attending’
negation is grammatically incorrect: any negative (future continuous) to make the sentence
proposition must only contain one negative. grammatically correct.
Thus, one of the negatives should be removed to The right sentence is:
make the sentence grammatically right. Thus, The team will be attending a meeting from noon
replace ‘no’ by ‘any’. to early evening tomorrow to discuss the pros
The right sentence is: and cons of investing a portion of company
Vimal is not cognizant of any ways to get the job shares in ventures like real estate, interior
done quickly, but he is not willing to outsource designing, and many more.
professionals for the same.
13. Answer: C
11. Answer: A Among the given options, phrases A, B and C
The Past Continuous tense is used to describe connects well with phrases E, D, and F
actions that began in the past and often respectively to result in meaningful and
continued for a short period of time after the grammatically right sentences. Thus, option c is
action started. Thus, part A is incorrect here right.
because the action referred to here is expressed All other options do not result in grammatically
to have taken place for a specific amount of time right and coherent sentences.
in the past but the use of tense is incorrect The right sentences include:
(simple past tense is used). Thus, ‘worked’ Markets regulator SEBI barred IIFL Securities,
should be replaced by ‘was working’. one of the leading domestic broking houses,
The right sentence is: from taking on new clients for the next two years
for violating regulatory forms.

Click Here For Bundle PDF Course | support@guidely.in Page 11 of 14


Bank Po Mains PDF Course 2024
English Day - 5

SEBI had considered cancellation of the tourism and focuses on shifting the contribution
certificate of the IIFL securities as a broken but of its sectors towards green, inclusive, and
refrained from doing that after considering some circular economic growth.
other factors that are mentioned elaborately in
the order released in June this year. 15. Answer: D
IIFL Securities has been penalized for indulging Among the given options, phrases A, B and C
in unfair trading practices and using funds of one connects well with phrases F, D, and E
client for meeting the obligations of another. respectively to result in meaningful and
grammatically right sentences. Thus, option d is
14. Answer: B right.
Among the given options, phrases A, B and C All other options do not result in grammatically
connects well with phrases D, E, and F right and coherent sentences.
respectively to result in meaningful and The right sentences include:
grammatically right sentences. Thus, option b is Describing Prime Minister Modi’s visit to the U.S.
right. this June as a milestone in India-U.S. relations,
All other options do not result in grammatically foreign secretary said a roadmap for defense
right and coherent sentences. industrial cooperation will be among the key
The right sentences include: outcomes of what will be Modi’s first state visit to
Union tourism minister G Kishan Reddy the country and his seventh as PM.
proposed that the G20 members should take The defense roadmap essentially focusses on
concrete steps to drive a collective change in aspects of defense co-production and co-
how plastic pollution can be tackled globally development and talks about how defense
while giving special attention to the tourism industrial ecosystems could cooperate much
industry of their countries. better.
While addressing the fourth G20 tourism working The prime minister met some prominent
group meeting, G Kishan Reddy proposed that personalities in New York and participate in a
the tourism industry should focus on green capacity-building event before departing for
initiatives, inclusive strategies, and circular Washington D.C. where he held bilateral talks
economic growth to promote sustainability and with the U.S. president and addressed a joint
renewable energy management. sitting of the U.S. congress.
The Indian tourism industry has enforced a
national strategy called the Swadesh tourism 16. Answer: A
that acknowledges guidelines for sustainable

Click Here For Bundle PDF Course | support@guidely.in Page 12 of 14


Bank Po Mains PDF Course 2024
English Day - 5

Among the given options, phrases A, B and C All other options do not result in grammatically
connects well with phrases D, E, and F right and coherent sentences.
respectively to result in meaningful and The right sentences include:
grammatically right sentences. Thus, option a is Victims of extensive burns and acid attack
right. patients with deep, severe scars now have a
All other options do not result in grammatically fresh ray of hope in the form of a dedicated skin
right and coherent sentences. bank at center-run Safdarjung hospital in the
The right sentences include: nation’s capital New Delhi.
The state investigation agency of Jammu and The proposal to set up a skin bank at the
Kashmir police carried out searches at several Safdarjung hospital at New Delhi was initiated in
locations in Srinagar, Anantnag, Pulwama, and 2012 after gaining the approval of the health
Kupwara districts in a case pertaining to the ministry but the construction of the same was
misuse of social media platforms for unlawful delayed due to the sudden outbreak of the
and secessionist activities. pandemic three years ago.
The identified entities are suspected of conniving The initiative is the first-of-its-kind in Delhi-NCR
with foreign associates to further their nefarious and north India, while presently India retains 16
agenda and specifically the entities in Anantnag skin banks which are majorly located in
have been found targeting government servants Maharashtra, Odisha, Madhya Pradesh, and the
by hindering their ability to perform their lawful southern states like Tamil
duties. Nadu and Karnataka.
The searches led to the seizure of substantial
digital and physical evidence, including mobile 18. Answer: D
phones and sim cards, which would be i. Hobble (verb) - to slow the movement,
meticulously analyzed to build a strong case progress, or action of (someone or something)
against the accused entities. ii. Hobble (verb) - to walk with difficulty because
of injury or weakness; to limp
17. Answer: C iii. Hobble (verb) - to fasten together the legs of
Among the given options, phrases A, B and C (a horse, mule, etc.) by short lengths of rope to
connects well with phrases E, D, and F prevent free motion
respectively to result in meaningful and Sentences A, B, and C use the word ‘hobble’
grammatically right sentences. Thus, option a is rightly to denote meanings as described in i, ii,
right. and iii respectively. Thus, all sentences are right.
Thus, option d is the right answer.

Click Here For Bundle PDF Course | support@guidely.in Page 13 of 14


Bank Po Mains PDF Course 2024
English Day - 5

21. Answer: D
19. Answer: A Rescind (verb) – to cancel officially
i. Comport (verb) - to be fitting; Accord or agree Only statement A rightly uses the word ‘rescind’
with by denoting the meaning ‘to cancel officially.
ii. Comport (verb) - to behave in a manner Thus, only statement D is right.
conformable to what is right, proper, or expected Statement B can be made grammatically right
Sentences A and B use the word rightly to and meaningful if the word ‘rescinded’ is
denote meaning mentioned in i and ii replaced by ‘receded’.
respectively. Thus, only A and B is right. Statement C can be made grammatically right
Sentence C becomes grammatically right and and meaningful if the word ‘rescinding’ is
meaningful when the word ‘comport’ is replaced replaced by ‘receding’.
by ‘comfort’.
22. Answer: B
20. Answer: C i. Inchoate (adjective) – partly in existence
i. Deplore (verb) – to express strong disapproval ii. Inchoate (adjective) – imperfectly or recently
of formed
ii. Deplore (verb) - to regret strongly Statements A and B rightly use the word
Sentences A and C rightly use the word ‘deplore’ ‘inchoate’ by denoting meanings that are
to denote meanings as described in i and ii. described in i and ii respectively. Thus, only
Thus, sentences A and C are correct. statements A and B are correct.
Sentence B becomes grammatically right and Statement C can be made grammatically right
meaningful when the word ‘deplore’ is replaced and meaningful if the word ‘inchoate’ is replaced
by ‘explore’. by ‘integrate’.

Click Here For Bundle PDF Course | support@guidely.in Page 14 of 14


Bank Po Mains PDF Course 2024
Reasoning Day -6 (Eng)

Reasoning Aptitude
Directions (1-5): Study the following information d) Only III
carefully and answer the given questions. e) All I, II and III
Twelve persons - A, C, D, E, F, G, I, J, K, M, O
and P are attending the meeting on three 2. How many persons attend the meeting
different dates 6th, 13th and 24th of four different between A and I?
months viz., January, April, July and October of a) As many persons attend between P and D
the same year. b) Two
Note: If the place value of the person’s name is c) As many persons attend between G and J
an even number as per the English alphabetical d) Three
series, then the person must attend the meeting e) None
on an odd numbered date of a month.
D attends the meeting three months before J. 3. Which of the following statements is/are true
Only four persons attend the meeting between J according to the given arrangement?
and G. As many persons attend the meeting I. F attends the meeting in the month having 30
before C as after K. F attends the meeting two days
persons after C. Both K and F are not attending II. K attends the meeting before O
the meeting in the same month. A attends the III. C attends the meeting immediately after P
meeting after P on an even numbered date. Only a) Only III
one person attends the meeting between A and b) Only I and III
O. The number of persons attending the meeting c) Only I
between O and M is three less than the number d) Only II and III
of persons attending the meeting between M and e) All I, II and III
I. Both E and I attend the meeting on the same
date in different months. 4. K attends the meeting on which of the
1. If T attends the meeting two months after P, following date and month?
then T attends the meeting on which of the a) 13th January
following date and month? b) 24th July
I. 6th April c) 6th October
II. 24th June d) 6th July
III. 13th March e) 24th April
a) Only I and II
b) Only II 5. Which among the following pairs of persons
c) Only II and III attend the meeting on the even numbered date?

Click Here For Bundle PDF Course | support@guidely.in Page 1 of 10


Bank Po Mains PDF Course 2024
Reasoning Day -6 (Eng)

a) E and F c) Step 1
b) D and G d) Step 3
c) J and A e) Both a and d
d) O and M
e) P and I 7. If all the digits are arranged in descending
order from left to right in step 2, then what is the
Directions (6-10): Study the following information difference between the digits which are fifth from
carefully and answer the given questions. the right end and seventh from the left end?
74912845319751365731498275 a) 4
2649 b) 6
Note: 0 is neither an even digit nor an odd digit c) 5
Step 1: One is added to every fourth digit from d) 3
the left end and two is subtracted from the digits e) 2
in the prime numbered positions from the left end
in the given series. 8. If one is subtracted from all the odd digits and
Step 2: After completing step 1, the odd numbers one is added to all the even digits in the last
which are either immediately followed by an odd step, then which of the following digits appear
digit or immediately preceded by an even digit more than thrice?
but not both are to be written in descending order a) 0
after the last digit from the left in the series. b) 3
Step 3: After completing step 2, the digits which c) 2
are continuously repeated for more than two d) Both b and c
times are dropped from the series. e) All a, b and c
Step 4: After completing step 3, the first 12 digits
from the left end are arranged in ascending order 9. In which of the following step the gap between
from the left end followed by the remaining digits the first “3” from the right end and first “3” from
are arranged in descending order. the left end is Minimum?
Step 4 is the last and final step and the outcome I. Step 1
in this step is the final series. II. Step 3
6. In which of the following step the product of III. Step 4
the digits which is fourth from the left end and a) Only II
third from the right end is Minimum? b) Only I and II
a) Step 2 c) Only III
b) Step 4 d) Only II and III

Click Here For Bundle PDF Course | support@guidely.in Page 2 of 10


Bank Po Mains PDF Course 2024
Reasoning Day -6 (Eng)

e) All I, II and III H and senior to E. DOR are working in the same
designation. As many designations higher than J
10. In step 1, how many such 2’s are each of as lower than T. F is senior to N but junior to A. A
which is immediately preceded as well as is not working in the senior most designation. U
immediately followed by an odd digit? is senior to M but Junior to B. M is not working
a) Four with R. The number of designations higher than
b) Three B is one less than the number of designations
c) Two lower than K. I is senior to C and G.
d) One 11. Who among the following persons are
e) Five designated as Assistant Manager (AM)?
I. R
Directions (11-15): Study the following II. K
information carefully and answer the given III. I
questions. a) Only III
21 persons from A to U are working in a bank in b) Only I and II
different designations viz., Chairman, General c) Only II and III
Manager (GM), Assistant General Manager d) Only II
(AGM), Manager, Assistant Manager (AM) and e) Only I and III
Clerk. The designations are given in decreasing
order such that Chairman is the senior most and 12. L is working on which of the following
Clerk is the junior most designations. Only one designation as per the given arrangement?
person works in the senior most designation and a) Manager
two persons work in the second senior most b) The designation in which E works
designation and so on. The number of persons c) General Manager (GM)
working in each designation is one less than the d) The designation in which C works
number of persons working in their immediately e) None of these
lower designation.
Note 1: If X works with Y, then X and Y are 13. Who among the following pair of persons are
working in the same designation. working in the same designation as per the given
Note 2: Consecutive alphabetically named arrangement?
persons are not working in the same designation. I. U and R
The one who is designated as AGM is senior to II. F and D
Q. Q works with S and is not designated as AM. III. E and I
T works with E. H works only with J. P is junior to a) Only II and III

Click Here For Bundle PDF Course | support@guidely.in Page 3 of 10


Bank Po Mains PDF Course 2024
Reasoning Day -6 (Eng)

b) Only III Mumbai, Pune, Chennai, Lucknow, Kolkata and


c) Only I and III Ranchi.
d) Only I and II Note: None of the city names with an equal
e) Only II number of vowel counts are opposite to each
other.
14. If H is related to R and P is related to K in a S and the one who went for an interview to
certain way, then who among the following Ranchi are sitting second to the left of each
person is related to N? other. At least one person sits between the one
a) B who went to Ranchi and the one who went to
b) T Lucknow, when counted from both sides. S does
c) D not go to Lucknow. T faces the one who went to
d) J Pune. Q sits fourth to the right of T. The number
e) E of persons sitting between Q and the one who
went to Mumbai is two less than the number of
15. What is the position of the one who works persons sitting between P and the one who went
with O with respect to the one who works with to Chennai, when counted from the right of both
Q? P and Q. Both the immediate neighbours of the
a) Three designations higher one who went to Mumbai are facing away from
b) Two designations higher the centre. P faces the opposite direction of T. U
c) Three designations lower neither went to Lucknow nor sits at the corner of
d) Two designations lower the table. R is not facing the same direction as U.
e) None of these 16. Who among the following person sits second
to the left of the one who went to Mumbai?
Directions (16-20): Study the following I. The one who went to Kolkata
information carefully and answer the given II. Q
questions. III. The one who went to Chennai
Six persons - P, Q, R, S, T and U are sitting a) Only II
around an equilateral triangular table in such a b) Only I and III
way that three of them are sitting at the corners c) Only III
and the remaining are sitting in the middle of the d) Only II and III
side. Some of them are facing the centre while e) Only I
some of them are facing away from the centre.
They went for an interview to different cities viz., 17. If all the persons are facing opposite to their
original direction, then who among the following

Click Here For Bundle PDF Course | support@guidely.in Page 4 of 10


Bank Po Mains PDF Course 2024
Reasoning Day -6 (Eng)

pair of persons are sitting second to the right of 19. How many persons sit between the one who
each other? went to Pune and U, when counted from the right
a) R and S of U?
b) T and Q a) Four
c) P and U b) None
d) R and U c) Two
e) Both b and d d) Three
e) One
18. Four of the following five are alike in a certain
way based on the given arrangement and thus 20. Who among the following person faces the
form a group. Which one of the following does one who went to Mumbai?
not belongs to the group? a) T
a) The one who went to Lucknow b) The one who went to Chennai
b) Q c) R
c) The one who went to Kolkata d) The one who went to Pune
d) S e) U
e) The one who went to Mumbai
Click Here to Get the Detailed Video Solution for the above given Questions
Or Scan the QR Code to Get the Detailed Video Solutions

Answer Key with Explanation


Directions (1-5): 5. Answer: D
1. Answer: D Final arrangement:
2. Answer: C
3. Answer: B
4. Answer: C

Click Here For Bundle PDF Course | support@guidely.in Page 5 of 10


Bank Po Mains PDF Course 2024
Reasoning Day -6 (Eng)

Again, we have
We have,  A attends the meeting after P on an even
 D attends the meeting three months numbered date of a month.
before J.  Only one person attends the meeting
 Only four persons attend the meeting between A and O.
between J and G.  The number of persons attending the
After applying the above conditions, there are meeting between O and M is three less
three possibilities. than the number of persons attending the
meeting between M and I.
 Both E and I attend the meeting on the
same date in different months.
After applying the above conditions, there is
another possibility. But we apply only one person
attends the meeting between A and O and both
E and I attend the meeting on the same date in
different months, we can cancel case 1, case 2
and case 3a, hence case 3 shows the final
Again, we have arrangement.
 F attends the meeting two persons after
C.
 As many persons attend the meeting
before C as after K.
 Both K and F not attend the meeting in
the same month.

Click Here For Bundle PDF Course | support@guidely.in Page 6 of 10


Bank Po Mains PDF Course 2024
Reasoning Day -6 (Eng)

Step 2: 9 9 8 8 7 7 7 7 7 7 7 6 6 5 4 3 3 3 3 3 2 2
2 2 2 2 1 1 10
Difference = 7 – 2 = 5

8. Answer: E
Step 4: 0 1 2 2 2 3 3 6 7 7 8 8 9 9 6 5 4 3 3 3 2 2
211
Step 4: 0 0 3 3 3 2 2 7 6 6 9 9 8 8 7 4 52 2 2 3 3
300

9. Answer: C

Directions (6-10): Step 1: 7 2 7 2 0 8 2 6 3 1 7 8 3 1 3 7 3 7 1 2 4 9

74912845319751365731498275 6 3 7 5 2 7 2 9-> 14 digits

2649 Step 2: 7 2 2 0 8 2 6 3 7 8 3 1 2 4 6 3 5 2 2 9 9 7

Step 1: 7 2 7 2 0 8 2 6 3 1 7 8 3 1 3 7 3 7 1 2 4 9 7 7 7 7 3 3 1 1->19 digits

63752729 Step 3: 7 2 2 0 8 2 6 3 7 8 3 1 246352299

Step 2: 7 2 2 0 8 2 6 3 7 8 3 1 2 4 6 3 5 2 2 9 9 7 3 3 1 1-> 14 digits

77773311 Step 4: 0 1 2 2 2 3 3 6 7 7 8 8 996543332

Step 3: 7 2 2 0 8 2 6 3 7 8 3 1 2 4 6 3 5 2 2 9 9 3 2 2 1 1 -> 13 digits

311
Step 4: 0 1 2 2 2 3 3 6 7 7 8 8 9 9 6 5 4 3 3 3 2 2 10. Answer: B

211 Step 1: 7 2 7 2 0 8 2 6 3 1 7 8 3 1 3 7 3 7 1 2 4 9

6. Answer: E 63752729

Step 1: 2 * 7 = 14
Step 2: 0* 3 = 0 Directions (11-15):

Step 3: 0*3 = 0 11. Answer: C

Step 4: 2*2 = 4 12. Answer: D


13. Answer: A

7. Answer: C 14. Answer: C

Step 2: 7 2 2 0 8 2 6 3 7 8 3 1 2 4 6 3 5 2 2 9 9 7 15. Answer: B

77773311 Final arrangement:

Descending order:

Click Here For Bundle PDF Course | support@guidely.in Page 7 of 10


Bank Po Mains PDF Course 2024
Reasoning Day -6 (Eng)

We have, Again, we have


 The one who is designated as AGM is  The number of designations higher than B
senior to Q. is one less than the number of
 Q works with S and is not designated as designations lower than K.
AM.  I is senior to C and G.
 T works with E. After applying the above conditions, case 1 and
 H works only with J. case 2 get eliminated because consecutive
 P is junior to H and senior to E. alphabetically named persons are not working in
 DOR are working in the same the same designation, hence case 2a shows the
designation. final arrangement.
 As many designations higher than J as
lower than T.
After applying the above conditions, there are
three possibilities.

Directions (16-20):
16. Answer: D

Again, we have 17. Answer: B

 F is senior to N but junior to A. 18. Answer: C (Except option c, all are facing

 A is not working in the senior most away from the centre)

designation. 19. Answer: D

 U is senior to M but Junior to B. 20. Answer: E

 M is not working with R. Final arrangement:

After applying the above conditions, there is


another one possibility, but case 3 gets
eliminated because F and E are not working in
the same designation.

Click Here For Bundle PDF Course | support@guidely.in Page 8 of 10


Bank Po Mains PDF Course 2024
Reasoning Day -6 (Eng)

 The number of persons sitting between Q


and the one who went to Mumbai is two
less than the number of persons sitting
between P and the one who went to
Chennai, when counted from the right of
both P and Q.
 Both the immediate neighbours of the one
who went to Mumbai are facing away
We have, from the centre.
 S and the one who went for an interview  P faces the opposite direction of T.
to Ranchi are sitting second to the left of
each other.
 At least one person sits between the one
who went to Ranchi and the one who
went to Lucknow when counted from both
sides.
 S does not go to Lucknow.
After applying the above conditions, there are
four possibilities.

Again, we have
 U neither went to Lucknow nor sits at the
corner of the table.
 R is not facing the same direction as U.
After applying the above conditions, case 1,
case 2 and case 4 get eliminated because U
neither went to Lucknow nor sits at the corner of
the table, hence case 3 shows the final
Again, we have
arrangement.
 T faces the one who went to Pune.
 Q sits fourth to the right of T.

Click Here For Bundle PDF Course | support@guidely.in Page 9 of 10


Bank Po Mains PDF Course 2024
Reasoning Day -6 (Eng)

Click Here For Bundle PDF Course | support@guidely.in Page 10 of 10


Bank Po Mains PDF Course 2024
Quantitative Aptitude Day -6 (Eng)

Quantitative Aptitude

Directions (1-5): Study the following data Y: The distance covered by bike C at 50% of its
carefully and answer the questions: maximum speed in 1.2 hours is 10 km less than
The data given below is related to the maximum that covered by bike D at 50% of its maximum
speeds (in km/h) of 5 different bikes A, B, C, D speed in 1.5 hours.
and E and the maximum speeds (in km/h) of 5 Z: The average of the maximum speeds of cars
different cars P, Q, R, S and T. R, S and T is 210 km/h.
The table given below shows the maximum a) Only X
speeds of bikes B, C, D and E as percent more b) Only Y and Z
or less than that of those of bike A and the c) Only Y
maximum speeds of cars Q, R, S and T as d) Only A and Z
percent more or less than that of car P. e) None is true.

2) A person has to cover a distance of 345 km by


cars P, Q and S. First he travelled by car P at
48% of its maximum speed for 1.25 hours, then
he travelled by car Q at 45% of its maximum
speed for 1 hour. If he has to cover the
remaining distance in 1.5 hours, then find out at
what per cent of the maximum speed of car S
should he travel?
a) 45%
b) 35%
c) 50%
Note: d) 40%
1: Maximum speed of car Q is 100 km/h more e) 30%
than that of bike B.
2: The ratio of the distance covered by bike A at 3) If the ratio of distance covered by bike A at
64% of its maximum speed in 2.5 hours to that 72% of its original speed in 1.8 hours to that
covered by car P at 60% of its maximum speed covered by car R at 60% of its maximum speed
in 1.5 hours, is 10: 9. in 1.5 hours is M: N, then find that the value of
1) Which of the following is/are not true? (5M – 2N) is divisible by which of the following?
X: The ratio of the maximum speed of bike E to a) 8
that of car T is 23: 36.

Click Here For Bundle PDF Course | support@guidely.in Page 1 of 14


Bank Po Mains PDF Course 2024
Quantitative Aptitude Day -6 (Eng)

b) 7 Directions (6-8): Study the following data


c) 5 carefully and answer the questions:
d) Both (a) and (b) A shopkeeper has three articles A, B and C of
e) Both (a) and (c) different costs. The ratio of the cost price of
article A to that of article B is 2: 5 and the cost
4) The bike E is running ‘d’ km ahead of car P. If price of article C is ₹1500 less than that of article
bike E is running at 80% of its maximum speed, B. Article A is marked up by 75%, article B is
car P is running at 64% of its maximum speed marked up by ₹900 and the marked price of
and car P crosses bike E in 2.5 minutes, then article C is ₹1350 more than that of article A. If
find the time taken by bike B to cover ‘d’ km at article B is sold at 8% discount on its marked
75% of its maximum speed? price and article C is sold at 4% discount on its
a) 60 seconds marked price, then the selling price of article B
b) 75 seconds will be ₹648 more than that of article C.
c) 45 seconds 6) If article A is sold at a 20% discount on its
d) 50 seconds marked price, article B is sold at 10% discount
e) 80 seconds on its marked price and article C is sold at a 28%
discount on its marked price, then find the
5) The distance covered by bike B at ___% of its average of profit received on articles A, B and C?
maximum speed in 1.6 hours is 14 km more than a) ₹410
that covered by car T at ___% of its maximum b) ₹440
speed in 1.2 hours. c) ₹420
Find out which of the following can’t be filled in d) ₹450
the blanks in the same order? e) ₹430
X: 60, 40
Y: 75, 60 7) If article B is marked up by M% and article C
Z: , 50 is marked up by N%, then which of the following
a) Only X is/are true?
b) Only Y and Z P: The sum of M and N is divisible by a prime
c) All X, Y and Z number greater than 5.
d) Only Z Q: Both M and N are divisible by 5 but not by 10.
e) Only X and Y R: The value of (N2 – M2) is divisible by 11.
a) Only P and R
b) All P, Q and R
c) Only P

Click Here For Bundle PDF Course | support@guidely.in Page 2 of 14


Bank Po Mains PDF Course 2024
Quantitative Aptitude Day -6 (Eng)

d) Only Q B, then what will be the cost of the new mixture


e) Only R (in terms of M and N)?
a)
8) The cost price of article D is equal to the b)
average of the cost prices of articles A, B and C c)
and article D is marked up by 140%. If article D d)
is sold at ₹1085 discount on its marked price,
e) None of these
then find the ratio of the marked up per cent of
article D to the profit per cent received on article
10) If the ratio of milk in mixture C to that in
D?
mixture D is 3: 5 and the ratio of water in mixture
a) 4: 1
B to that in mixture D is 8: 5, then find which of
b) 5: 1
the following is/are true?
c) 10: 1
P: The ratio of milk in mixture A to that in mixture
d) 8: 1
D is 4: 5.
e) 2: 1
Q: The average quantity of water in mixtures A,
B, C and D is 9.5 L.
Directions (9-11): Study the following data
R: The cost of mixture D is ₹30 per L.
carefully and answer the questions:
a) All are true
Three mixtures A, B and C contain different
b) Only P
quantities (in L) of milk and water and the cost of
c) Only P and Q
pure milk is ₹45 per L.
d) Only P and R
The ratio of milk to water in mixture A is 4: 1 and
e) None is true.
the ratio of milk to water in mixture B is 3: 2.
When both the mixtures are mixed together, the
11) When 50% quantity of mixture A, 75%
cost of the new mixture becomes ₹30 per L. The
quantity of mixture B and 25% quantity of mixture
quantity of milk in mixture A is 8 L less than that
C are mixed together, then what will be the cost
in mixture B and also 4 L more than that in
of the new mixture?
mixture C. The cost of mixture C is ₹27 per L.
a) ₹28 per L
9) The cost of mixture A is ₹M per L and the cost
b) ₹24 per L
of mixture B is ₹N per L. If 15 L quantity of
c) ₹30 per L
mixture A is mixed with 10 L quantity of mixture
d) ₹29 per L
e) ₹32 per L

Click Here For Bundle PDF Course | support@guidely.in Page 3 of 14


Bank Po Mains PDF Course 2024
Quantitative Aptitude Day -6 (Eng)

Directions (12-16): Study the following data carefully and answer the questions:
The data given below is related to the number of satellites launched through 6 different rockets A, B, C,
D, E and F in a country and each rocket carried a different weight (in kg).
The bar graph given below shows the total weight carried by each of the rockets B, C, D, E and F as per
cent more/less than that carried by rocket A.

The table given below shows the number of satellites carried by each rocket.

Note:
1: The average weight of each satellite carried by rocket A is 640 kg more than the average weight of
each satellite carried by rocket B.
2:

Click Here For Bundle PDF Course | support@guidely.in Page 4 of 14


Bank Po Mains PDF Course 2024
Quantitative Aptitude Day -6 (Eng)

12) If the average weight of each satellite carried R: The difference between the average weight of
by rocket C is M% more/less than the average each satellite carried by rocket B and the
weight of each satellite carried by rocket A and average weight of each satellite carried by rocket
the average weight of each satellite carried by C is 380 kg.
rocket E is N% more/less than the average a) Only P
weight of each satellite carried by rocket D, then b) Only Q and R
find the ratio of M to N? c) Only Q
a) 10: 167 d) Only P and R
b) 5: 67 e) Only R
c) 40: 467
d) 20: 267 15) If the total weight carried by rocket G is 25%
e) None of these more than that carried by rocket C and the
average weight of each satellite carried by rocket
13) If the weight of rocket D without satellites is E is 76.8% of the average weight of each
12.5% more than the total weight of satellites satellite carried by rocket G, then find that the
carried by rocket D and the weight of rocket F number of satellites carried by rocket G is what
without satellites is 5% more than the total percent more/less than that carried by rocket A?
weight of satellites carried by rocket F, then find a) 16.67%
that the weight of rocket F without satellites is b) 20%
what percent more or less than that of rocket D c) 15%
without satellites? d) 12.5%
a) 5.78% e) 10%
b) 12.21%
c) 10% 16) If the total weight carried by rocket A were
d) 2.24% 4000 kg less and the new total weight carried by
e) 19.81% E is 20% more than the new total weight carried
by A. If the number of satellites carried by rocket
14) Which of the following is/are true? E were the same as initially, then the average
P: The ratio of the average weight of each weight of each satellite carried by rocket E would
satellite carried by rocket E to the average weight be what per cent less than the original average
of each satellite carried by rocket F is 48: 83. weight of each satellite carried by rocket E?
Q: The average of the total weight carried by a) 7.25%
rockets C, D and E is 60800 kg. b) 2.25%
c) 9.25%

Click Here For Bundle PDF Course | support@guidely.in Page 5 of 14


Bank Po Mains PDF Course 2024
Quantitative Aptitude Day -6 (Eng)

d) 8.25% 19) Around a rectangular park, there is a path


e) 6.25% running outside it of uniform width given in the
figure below.
17) A boat goes ‘D’ km upstream and ‘D + 10’ km
downstream and takes a total ‘17t’ min. If the
boat covers total ‘D + 10’ km upstream and ‘D’
km in downstream in ‘19t’ min. If the ratio of the
upstream speed to the downstream speed of the
boat is 1: 2, then which of the following values
can be determined?
I: Upstream speed of the boat.
II: Time taken by the same boat to go ‘D – 10’ km
upstream.
If the cost of flooring the shaded portion at the
III: Value of ‘D’.
rate ₹5/m2 is ₹2540 and the length and the width
a) Only statements I and II
of the park (without the path) are20 m and 15 m
b) Only statements I and III
respectively and the radius of the semi-circular
c) Only statement III
figure is 7 m, then find the width of the path.
d) None of the statements follows
a) 2.25 m
e) Only statement II
b) 3 m
c) 2 m
18) The time taken by persons A, B, and C alone
d) 2.5 m
to finish a work is (24 +32 – 1) days, (52 + 11)
e) 3.5 m
days, (52 – 32 + 2) days respectively. If A and B
started the work and after ‘6x’ days A is replaced
20) The ratio of milk to water in two mixtures A
by C and now B and C together can finish that
and B is 7: x and x: 13 respectively. These two
work in ‘x’ days. Find the value of ‘x2 + 9’.
mixtures are mixed in the ratio of 5: 8 and when
a) 52 – 32
the final mixture is sold at the cost of pure milk,
b) 32 +5
the profit percent earned would be 160%. The
c) 12 +9
ratio of milk to water in a mixture that is formed
d) 22 +7
after mixing mixtures A and B in the same
e) 42 – 3
quantity.
a) 61: 99
b) 71: 89

Click Here For Bundle PDF Course | support@guidely.in Page 6 of 14


Bank Po Mains PDF Course 2024
Quantitative Aptitude Day -6 (Eng)

c) 81: 79 e) 91: 69
d) 51: 109
Click Here to Get the Detailed Video Solution for the above given Questions
Or Scan the QR Code to Get the Detailed Video Solutions

Answer Key with Explanation


Directions (1-5): 176x – 96x = 100
Let the maximum speeds of bike A and car P are x = 1.25
100x km/h and 100y km/h respectively. From equation (2):
So, the maximum speeds of bikes B, C, D and E y=2
are 96x km/h, 120x km/h, 112x km/h and 92x
km/h.
And the maximum speeds of cars Q, R, S and T
are 110y km/h, 85y km/h, 120y km/h and 95y
km/h.
Since, the maximum speed of car Q is 100 km/h
more than that of bike B.
So,
110y – 96x = 100 -------------(1)
Since, the ratio of the distance covered by bike A
1) Answer: E
at 64% of its maximum speed in 2.5 hours to that
From X:
covered by car P at 60% of its maximum speed
Ratio of the maximum speed of bike E to that of
in 1.5 hours, is 10: 9.
car T = 115: 190 = 23: 38
So,
So, X is not true.
From Y:

Click Here For Bundle PDF Course | support@guidely.in Page 7 of 14


Bank Po Mains PDF Course 2024
Quantitative Aptitude Day -6 (Eng)

Distance covered by bike C at 50% of its 60% of 170 * 1.5 = 153 km


maximum speed in 1.2 hours: Required ratio = M: N = 162: 153 = 18: 17
50% of 150 * 1.2 = 90 km Now, the value of (5M – 2N) = 90 – 34 = 56
Distance covered by bike D at 50% of its And the value of (5M – 2N) is divisible by both 8
maximum speed in 1.5 hours: and 7.
50% of 140 * 1.5 = 105 km
Required difference = 105 – 90 = 15 km 4) Answer: A
So, Y is not true. The speed, at which bike E is running = 80% of
From Z: 115 = 92 km/h
Average of maximum speeds of cars R, S and T: The speed, at which car P is running = 64% of
200 = 128 km/h
So,
So, Z is not true.
Hence, none is true.
d = 1500 m = 1.5 km
2) Answer: B Now, the time taken by bike B to cover 1.5 km at
Distance covered by the person by car P at 48% 75% of its maximum speed:
of its maximum speed in 1.25 hours = 48% of
200 * 1.25 = 120 km
Distance covered by the persons by car Q at
45% of its maximum speed in 1 hour = 45% of 5) Answer: E
220 * 1 = 99 km From X:
Remaining distance = 126 km 60% of 120 * 1.6 – 40% of 190 * 1.2 = 115.2 –
The speed of the car S while covering the 91.2 ≠ 14 km
remaining distance = = 84 km/h So, the values in X can’t be filled in the blanks in
The maximum speed of car S = 240 km/h the same order.
Required percentage = = 35% From Y:
75% of 120 * 1.6 – 60% of 190 * 1.2 = 144 –

3) Answer: D 136.8 ≠ 14 km

The distance covered by bike A at 72% of its So, the values in Y can’t be filled in the blanks in

maximum speed in 1.8 hours: the same order.

72% of 125 * 1.8 = 162 km From Z:

The distance covered by car R at 60% of its 66(2/3)% of 120 * 1.6 – 50% of 190 * 1.2 = 128 –

maximum speed in 1.5 hours: 114 = 14 km

Click Here For Bundle PDF Course | support@guidely.in Page 8 of 14


Bank Po Mains PDF Course 2024
Quantitative Aptitude Day -6 (Eng)

So, the values in Z can be filled in the blanks in The SP of article C = 72% of 4500 = ₹3240
the same order. The profit received on article C = 3240 – 3000 =
Hence, the values in Z only can be filled in the ₹240
blanks in the same order. Required average = = ₹440

Directions (6-8): 7) Answer: C


Let the CPs of articles A and B be ₹2x and ₹5x The CP of article B = ₹4500
respectively. The MP of article B = ₹5400
So, the CP of article C = ₹(5x – 1500) So, M = = 20%
The MP of article A = 175% of 2x = ₹3.5x The CP of article C = ₹3000
The MP of article B = ₹(5x + 900) The MP of article C = ₹4500
And the MP of article C = ₹(3.5x + 1350) So, N = = 50%
If article B is sold at 8% discount on its marked From P:
price and article C is sold at 4% discount on its Sum of M and N = 20 + 50 = 70
marked price, then the selling price of article B Since, the sum of M and N is divisible by a prime
will be ₹648 more than that of article C. number 7, which is greater than 5.
So, So, P is true.
From Q:
460x + 82800 – 336x – 129600 = 64800 Since, both M and N are divisible by 5 and also
x = 900 divisible by 10.
The CP of article A = 2 * 900 = ₹1800 So, Q is not true.
The CP of article B = 5 * 900 = ₹4500 From R:
The CP of article C = 4500 – 1500 = ₹3000 Value of (N2 – M2) = 2500 – 400 = 2100
The MP of article A = 175% of 1800 = ₹3150 Since, 2100 is not divisible by 11.
The MP of article B = 4500 + 900 = ₹5400 So, R is not true.
The MP of article C = 3150 + 1350 = ₹4500 Hence, only P is true.
6) Answer: B
The SP of article A = 80% of 3150 = ₹2520 8) Answer: D
The profit received on article A = 2520 – 1800 = The CP of article D = = ₹3100
₹720 The MP of article D = 140% of 3100 = ₹4340
The SP of article B = 90% of 5400 = ₹4860 And the SP of article D = 4340 – 1085 = ₹3255
The profit received on article B = 4860 – 4500 = So, the profit per cent on article D =
₹360 = 5%

Click Here For Bundle PDF Course | support@guidely.in Page 9 of 14


Bank Po Mains PDF Course 2024
Quantitative Aptitude Day -6 (Eng)

Required ratio = 40: 5 = 8: 1 Cost of mixture A = M = = ₹36 per L


Cost of mixture B = N = = ₹27 per L
Directions (9-11): When 15 L quantity of mixture A is mixed with 10
Let the quantities of milk and water in mixture A L quantity of mixture B:
are ‘4a’ L and ‘a’ L respectively. Quantity of milk in 15 L quantity of mixture A =
Also let the quantities of milk and water in = 12 L
mixture B are ‘3b’ L and ‘2b’ L respectively. Quantity of water in 15 L quantity of mixture A =
So, 15 – 12 = 3 L
Quantity of milk in 10 L quantity of mixture B =

12a + 9b = 10a + 10b =6L

2a = b ------------(1) Quantity of water in 10 L quantity of mixture B =

Since, the quantity of milk in mixture A is 8 L less 10 – 6 = 4 L

than that in mixture B. Cost of new mixture = = ₹32.4 per L =

So,
3b – 4a = 8 ------------(2)
From equations (1) and (2): 10) Answer: A
3b – 2b = 8 Quantity of milk in mixture D = = 20 L
b=8 Quantity of water in mixture D = = 10 L
From equation (1): From P:
a=4 Ratio of milk in mixture A to that in mixture D =
Quantity of milk in mixture A = 4 * 4 = 16 L 16: 20 = 4: 5
Quantity of water in mixture A = 4 L So, P is true.
Quantity of milk in mixture B = 3 * 8 = 24 L From Q:
Quantity of water in mixture B = 2 * 8 = 16 L Average quantity of water in mixtures A, B, C
Quantity of milk in mixture C = 16 – 4 = 12 L and D:
Let the quantity of water in mixture C = ‘c’ L
And,
So, Q is true.
From R:
c=8 Cost of mixture D = = ₹30 per L
So, quantity of water in mixture C = 8 L
So, R is true.
Hence, all are true.
9) Answer: B

Click Here For Bundle PDF Course | support@guidely.in Page 10 of 14


Bank Po Mains PDF Course 2024
Quantitative Aptitude Day -6 (Eng)

11) Answer: D
Total quantity of milk in the new mixture:
50% of 16 + 75% of 24 + 25% of 12 = 8 + 18 + 3
= 29 L
Total quantity of water in the new mixture:
50% of 4 + 75% of 16 + 25% of 8 = 2 + 12 + 2 =
16 L
So, the cost of the new mixture:

Directions (12-16):
Let the total weight carried by rocket A is 100x
12) Answer: B
kg.
The average weight of each satellite carried by
So, the total weights carried by rockets B, C, D,
rocket A = 4000 kg
E and F are 105x kg, 90x kg, 75x kg, 120x kg
The average weight of each satellite carried by
and 85x kg respectively.
rocket C = 3840 kg
The average weight of each satellite carried by
The average weight of each satellite carried by
rocket A = kg
rocket D = 2000 kg
And the average weight of each satellite carried The average weight of each satellite carried by
by rocket B = kg rocket E = 3072 kg
Since, the average weight of each satellite So, M = = 4%
carried by rocket A is 640 kg more than the And, N = = 53.6%
average weight of each satellite carried by rocket
Required ratio = M: N = 4: 53.6 = 40: 536 = 5: 67
B.
So,
13) Answer: A
Total weight carried by rocket D = 48000 kg
1000x – 840x = 160 * 640 So, the weight of rocket D without satellites =
x = 640 112.5% of 48000 = 54000 kg
Total weight carried by rocket F = 54400 kg
So, the weight of rocket F without satellites =
105% of 54400 = 57120 kg

Click Here For Bundle PDF Course | support@guidely.in Page 11 of 14


Bank Po Mains PDF Course 2024
Quantitative Aptitude Day -6 (Eng)

Required percentage = = And the number of satellites carried by rocket G


5.78% = = 18
Since, the number of satellites carried by rocket
14) Answer: C A = 16
From P: So, required percentage = = 12.5%
Ratio of the average weight of each satellite
carried by rocket E to the average weight of 16) Answer: E
each satellite carried by rocket F = 3072: 5440 = The original total weight carried by rocket A =
48: 85 64000 kg
So, P is not true. So, the new total weight carried by rocket A
From Q: would be = 64000 – 4000 = 60000 kg
Average of total weight carried by rockets C, D And the new total weight carried by rocket E
and E: would be = 120% of 60000 = 72000 kg
If the number of satellites carried by rocket E

So, Q is true. were the same as initially:

From R: So, the average weight of each satellite carried

Difference between the average weight of each by rocket E would be:

satellite carried by rocket B and the average


weight of each satellite carried by rocket C: Since, original average weight of each satellite
3840 – 3360 = 480 kg carried by rocket E = 3072 kg
So, R is not true. So, required percentage = =
Hence, only Q is true. 6.25%

15) Answer: D 17) Answer: C


Since, the total weight carried by rocket C = Let the upstream speed and downstream speed
57600 kg of the boat is ‘x’ and ‘2x’ respectively.
So, the total weight carried by rocket G = 125% According to the question:
of 57600 = 72000 kg
Since, the average weight of each satellite
carried by rocket E = 3072 kg
So, the average weight of each satellite carried
by rocket G = = 4000 kg

Click Here For Bundle PDF Course | support@guidely.in Page 12 of 14


Bank Po Mains PDF Course 2024
Quantitative Aptitude Day -6 (Eng)

36x = 72
x=2
From (1) and (2):
Value of ‘x2 + 9’ = 22 + 9 = 13 = 42 – 3

19) Answer: D
57D + 190 = 51D + 340 Let the width of the path = ‘x’ m
6D = 150 Length of path with path = (20 + 2x) m
D = 25 Width of path with path = (15 + 2x) m
By putting the value of ‘D’ in equation (1) and Area of the shaded portion =
(2):

We can clearly see than we cannot find the


value of ‘x’ and ‘t’.
So, we cannot calculate the upstream and
downstream speed of the boat.
So, we also cannot calculate the time taken by
same boat to go ‘D – 10’ km in upstream.
Hence, only statement III can be determined.

Hence, the width of the path = 5/2 m = 2.5 m


18) Answer: E
Time taken by A alone to finish the work = 24 +
20) Answer: B
32 – 1 = 16 + 9 – 1 = 24 days
Part of milk in mixture A =
Time taken by B alone to finish the work = 52 +
Part of milk in mixture B =
11 = 25 + 11 = 36 days
Time taken by C alone to finish the work = 52 – Since the profit percent earned when the final

32+ 2 = 25 – 9 + 2 = 18 days mixture is sold at the cost of pure milk is 160%.

Total days for which A, B, and C work are ‘6x’ Which means quantity of water in the final

days, ‘7x’ days, and ‘x’ days respectively. mixture will be 160% of quantity of pure milk.

According to the question: Ratio of milk to water in the final mixture = 100:
160 = 5: 8
Part of milk in final mixture =

Click Here For Bundle PDF Course | support@guidely.in Page 13 of 14


Bank Po Mains PDF Course 2024
Quantitative Aptitude Day -6 (Eng)

By the rule of alligation: Let the part of milk in the final mixture when
mixtures A and B are mixed in equal quantity = P
By the rule of alligation:

5 8

1 1

Required ratio of milk to water = 71: (160 – 71) =


71: 89

Part of milk in mixture A =


Part of milk in mixture B =

Click Here For Bundle PDF Course | support@guidely.in Page 14 of 14


Bank Po Mains PDF Course 2024
English Day - 6

English Language
Directions (1-6): The following paragraph below A. Advantage
has certain words that have been deleted. The B. Leverage
blanks with labels (A), (B)………… up to (F) C. Command
indicate the missing words. You have to choose D. Clout
the best words from the options that will make E. Exploit
the paragraph coherent and meaningful
“Air India was the Holy Grail of public sector 2. Which of the following options will fill the blank
privatisation. The sentiment in the government labelled (B) in the paragraph given above?
was, ‘If we can successfully privatise Air India, A. Rivet
we can sell anything.’ Now that it’s done, it’s up B. Engross
to them to _____(A)_____ it.” The sale of Air India C. Fetch
is to _____(B)_____ the government ₹ 18,000 D. Relinquish
crore, and would give the buyer, Tata group, E. Forbear
ownership of Air India, its low-cost unit Air India
Express, and a 50 per cent stake in the airline’s 3. Choose the option that will fit in the blank (C)
ground and cargo handling subsidiary, Air India in order to complete the sentence
SATS Airport Services (AISATS). Privatisation is A. Tangled
a multi-layered, _____(C)_____ process. The dos B. Apparent
and don’ts from the Air India experience might C. Uniform
well serve as a template for future such deals. D. Intelligible
And the government _____(D)_____ needs the E. Complex
template. Add to that the fact that these days,
asset monetisation, disinvestment and 4. Which of the following options can complete
privatisation are often used interchangeably. the sentence with the blank (D) to fit coherently
(The government is looking at raising ₹ 6 lakh in the paragraph?
crore through asset monetisation alone. So, has A. Sorely
the government bitten off more than it can B. Distressfully
_____(E)_____? Not quite, say experts, who aver C. Hardly
that for a country with the world’s fifth-largest D. Merrily
economy and _____(E)_____ assets, it is E. Meagrely
achievable, provided certain conditions are met. 5. The most suitable word that would fill in the
1. Find the word from the options that will fit in blank labelled (E) from the given options is?
the blank (A) to complete the sentence A. Sell

Click Here For Bundle PDF Course | support@guidely.in Page 1 of 10


Bank Po Mains PDF Course 2024
English Day - 6

B. Chew to thunderous them into religious


C. Buy
explosions that ceremonies and festive
D. Cut
reverberate in the events.
E. None of the above
B. The history of E. they were initially

6. Which of the following words would fit best in fireworks dates back invented by alchemists

the last blank of the paragraph? to ancient China, searching for the elixir of

A. Varied where immortality.

B. Sublime
C. Over time, the art F. against the darkness,
C. Humongous
of creating fireworks igniting a sense of
D. Scarce
spread to different childlike wonder in our
E. Elite
parts of the world, hearts.
each culture
Directions (7-11): In the following questions two
A. A-F
columns are given containing three
B. A-E
Sentences/phrases each. In the first column,
C. B-E
sentences/phrases are A, B and C and in the
D. A-E and C-D
second column the sentences/phrases are D, E
E. None of these
and F. A sentence/phrase from the first column
may or may not connect with another
8.
sentence/phrase from the second column to
COLUMN 1 COLUMN 2
make a grammatically and contextually correct
sentence. Each question has five options, four of A. While conjunctivitis D. dirty hands, using
which display the sequence(s) in which the is generally not a contaminated eye
sentences/phrases can be joined to form a severe condition, it makeup, or swimming in
grammatically and contextually correct sentence. can be highly water with harmful
If none of the options given forms a correct contagious, bacteria.
sentence after combination, mark option (e), i.e. necessitating
“None of these” as your answer.
B. These infections E. self-medicate eye
7.
can lead to more infections, as improper
COLUMN 1 COLUMN 2
severe symptoms like treatment can
A. From brilliant D. create entertainment severe pain, blurred exacerbate the condition
cascades of sparkles and began incorporating vision, sensitivity or delay proper care.

Click Here For Bundle PDF Course | support@guidely.in Page 2 of 10


Bank Po Mains PDF Course 2024
English Day - 6

C. Eye infections can F. to light, and even A. A-E and C-F


B. A-D and C-E
be caused by various vision loss if left
C. B-F and C-D
factors, including untreated.
D. B-E and C-F
poor hygiene,
touching the eyes E. None of these

with
10.
A. A-D
COLUMN 1 COLUMN 2
B. B-F
A. In an age D. simple pleasure of
C. A-F and B-D
D. B-F and C-D dominated by traditional games, the

E. None of these screens and great outdoors offers an


technology, the expansive canvas for
allure of outdoor boundless fun and
9.
games remains as adventure.
COLUMN 1 COLUMN 2
strong as ever,
A. By being D. technology's benefits
positive role and ensuring healthy real- B. From soccer and E. and the thrill of
models and world interactions is a basketball to cricket scoring a point can
displaying qualities challenge parents face in and baseball, team create unforgettable
such as kindness, the digital age. sports have a unique memories with friends
way of fostering and family.
B. It is also crucial E. resilience, and honesty,
camaraderie,
for parents to parents impart invaluable
teaching
remember that no life lessons that extend far
one is infallible, beyond the classroom. C. Whether it's the F. bring people together,
and making excitement of team encouraging physical
mistakes sports, the thrill of activity and imaginative
running freely on play in equal measure.
C. The constant F. ample time to children
open fields, or the
juggling act can sometimes leave
A. A-F
between work, parents feeling
B. C-D
household overwhelmed and
responsibilities, exhausted. C. B-D and C-F
D. B-E and C-D
and giving
E. None of these

Click Here For Bundle PDF Course | support@guidely.in Page 3 of 10


Bank Po Mains PDF Course 2024
English Day - 6

11. rearranged if needed. If no rearrangement is


COLUMN 1 COLUMN 2 required then choose option E as your answer.
12. The ties between India and Nepal (a)/ came
A. Adulteration, a D. affects human health
under severe strain after Kathmandu published
deceptive practice but also tarnishes the
that involves adding reputation of businesses (b)/ a new political map in 2020 (c)/ Indian

inferior or harmful and industries. territories Limpiyadhura (d) that showed three

substances to food, (e), Kalapani and Lipulekh as part of Nepal (f).


A. abced
beverages, or other
products, poses a B. acdeb
C. bcdae
B. The history of E. adulteration has D. bedca
adulteration dates expanded to a wide range E. No rearrangement is required
back centuries, of products, including
where unscrupulous food items, 13. Under international law (a)/, ships are
individuals sought to pharmaceuticals, allowed to sail through foreign EEZs (b)/, but
profit at the expense cosmetics, and even unauthorised surveys are not permitted and
fuels. China's operations (c)/ in the South China Sea
have long been problematic (d)/ for countries in
C. Contaminated F. serious threat to public
the region, as Beijing claims most of the energy-
food and beverages health and undermines
rich sea (e)/, including foreign EEZs (f).
can lead to a host of the integrity of industries
A. bcedf
health issues, and businesses.
B. cdefb
ranging from mild
C. bdcef
discomfort and
D. edcab
A. A-F
E. No rearrangement is required
B. B-E
C. B-D and C-F
14. showed Monday (a)/ Republican former vice
D. B-E and C-F
president Mike Pence (b)/ has launched his
E. None of these
hotly-anticipated challenge (c)/ to his one-time
boss Donald Trump (d)/ for the party's 2024
Directions (12-16): Given below are a few
White House nomination (e)/, papers filed with
questions with jumbled parts in order to
the US Federal Election Commission (f).
rearrange them and make a contextually correct
A. abcdf
sentence. One of the part is fixed and rest to be

Click Here For Bundle PDF Course | support@guidely.in Page 4 of 10


Bank Po Mains PDF Course 2024
English Day - 6

B. acdfb 17. Do we prioritise the use of public funds to


C. bcdfa develop our human potential, or do we give them
D. fadcb away in corporate tax cuts in the hope that it will
E. No rearrangement is required set off a virtuous cycle of development?
A. Prefer, promiscuous
15. Years of financial mismanagement (a)/ have B. Unscrupulous, sequence
pushed Pakistan's economy to the limit (b)/, C. Favour, upstanding
exacerbated by a global energy crisis (c)/ that D. Negate, moral
submerged a third of the country (d)/ and E. Overlook, sullied
devastating floods (e)/ in 2022 (f).
A. abced 18. The 75th Independence Day speech marked
B. acdeb the creation of a gender-inclusive nation, based
C. adebc on the liberal ethos found in our cultural-
D. acedb civilisational values and new ideas for a post-
E. No rearrangement is required modern world.
A. Neutral, tolerant
16. Pakistan needs billions of dollars (a)/ in B. Exclusive, martinet
financing to service (b)/ staggering levels of C. Comprehensive, righteous
external debt (C)/, and foreign exchange D. Overall, plentiful
reserves (d)/ barely enough for a month of E. All of the above
imports (e)/ have dwindled to just $4.2 billion (f).
A. acdfe 19. We believe that no solution can be arrived at
B. adfec by shedding blood and at the cost of innocent
C. cedfa lives. We have emphasised right from the
D. cdeaf beginning of the conflict that the path of
E. No rearrangement is required diplomacy and dialogue should be the only viable
option.
Directions (17-21): In each question below, A. Slay, judicious
certain sentences have been given in which B. Slaughter, unserviceable
words have been highlighted. In the options C. Annihilate, teetering
below the same, the synonyms of the highlighted D. Extermination, superficial
words have been given. You are required to E. Slay, homicide
choose the option containing the correct
synonyms for the highlighted words.

Click Here For Bundle PDF Course | support@guidely.in Page 5 of 10


Bank Po Mains PDF Course 2024
English Day - 6

20. After stubbornly holding off from acting to


tame inflation, which has steadily eroded 21. The prime minister has not uttered a word so
consumers’ purchasing power and derailed far on these developments nor appealed for
broader economic momentum, the RBI’s rate peace amongst the warring groups, which raises
setting panel announced an ‘off-cycle’ increase in questions about his party and the government.
benchmark interest rates. A. Addressed, benevolent
A. Vitalize, thwart B. Spoken, diplomatic
B. Insipid, animate C. Express, pugnacious
C. Deprecate, put off D. Nonplussed, dissention
D. Rein, wreck E. Clarified, fringe
E. None of the above
Click Here to Get the Detailed Video Solution for the above given Questions
Or Scan the QR Code to Get the Detailed Video Solutions

Answer Key with Explanation


1. Answer: B doesn’t fit grammatically and hence can be
It is being explained that the government eliminated.
seemed to think that if this privatisation is Command and Exploit won’t fit contextually and
successfully carried out, it will gain confidence to thus can be ruled out.
move ahead in the direction and now that it has Clout is to use power or political influence for
already happened, the government should take getting something done.
advantage of the same (as per the author; the Therefore, the best option is (b).
government should capitalize on this now).
So, the best option is ‘Leverage: use 2. Answer: C
(something) to maximum advantage’ which can The sentence says that the sale of the carrier will
be kept above the other options. Advantage bring Rs. 18000 crores to the government. We

Click Here For Bundle PDF Course | support@guidely.in Page 6 of 10


Bank Po Mains PDF Course 2024
English Day - 6

need a word that would mean ‘to bring ‘desperately’ or ‘highly intensive’ should be
something’ and thus, the third option can fit well. correct.
Rivet means to hold (someone or something) Analysing the options, hardly and meagrely
fast so as to make them incapable of movement. mean negligible need of something which goes
Engross means to absorb or attract all the opposite to what is expected.
attention etc. Merrily is out of context and can be ruled out.
Relinquish and Forbearare similar in meaning Distressful means painful so even this is not
and mean to give up something voluntarily. needed here.
After understanding the meanings, we can easily Hence, option (a) is the correct answer.
mark (c) as the correct option.
5. Answer: B
3. Answer: E ‘Bite more than one can chew’ means taking up
Something is being termed as a multi-layered more work than can be accomplished.
process which means that it will cover a variety This is an easy one and just a little observation
of aspects and won’t be clear and easily will help us eliminate the other options and mark
understandable. So, the word ‘uniform’ can be (b) as the correct answer.
eliminated. 6. Answer: C
Apparent means very obvious and clearly Humongous means extraordinarily large or huge
understood so even this is not fit as per the number (of assets in this case)
context. Varied means variety of which is pointless in this
Intelligible means easily understood or context.
comprehensible so even this option can be ruled Sublime means pure, unparalleled
out. Scarce is opposite of what is intended
Out of tangled and complex, complex would fit Elite means rich and classy which again doesn’t
the blank better. go with the context
So, the best option is (e). So, the best option is (c).

4. Answer: A 7. Answer: C
The part where we have the fourth blank B-E forms a complete sentence which is both
discusses the intensity or the urgency of having meaningful and correct.
a template (a prototype or an example for the The other options have a combination of
further cases). Any word that would mean, statements/fragments which do not combine to
form a complete sentence.

Click Here For Bundle PDF Course | support@guidely.in Page 7 of 10


Bank Po Mains PDF Course 2024
English Day - 6

The sentence formed : The history of fireworks and honesty, parents impart invaluable life
dates back to ancient China, where they were lessons that extend far beyond the classroom.
initially invented by alchemists searching for the C-F : The constant juggling act between work,
elixir of immortality. household responsibilities, and giving ample
time to children can sometimes leave parents
8. Answer: D feeling overwhelmed and exhausted.
Both B-F and C-D are the correct combinations
which combine to form a complete sentence 10. Answer: B
which is meaningful and correct. The fragment C and D combine to form a
The other options are incorrect as those meaningful and complete sentence.
combinations of fragments/statements do not Even though the other statements also revolve
form a proper sentence. around the same topic they do not combine with
The sentences formed : each other to form a sentence that is meaningful
B-F : These infections can lead to more severe and correct.
symptoms like severe pain, blurred vision, The sentence formed : Whether it's the
sensitivity to light, and even vision loss if left excitement of team sports, the thrill of running
untreated. freely on open fields, or the simple pleasure of
C-D : Eye infections can be caused by various traditional games, the great outdoors offers an
factors, including poor hygiene, touching the expansive canvas for boundless fun and
eyes with dirty hands, using contaminated eye adventure.
makeup, or swimming in water with harmful
bacteria. 11. Answer: A
A-F is the correct combination that forms a
9. Answer: A complete sentence.
A-E forms a complete meaningful sentence The other given options are wrong as they don’t
similarly the combination C-F forms a complete contain statements that form a meaningful
sentence with meaning. Fragments B and D do sentence.
not combine with the other statements to The sentence formed : Adulteration, a deceptive
generate a sentence and hence are discarded. practice that involves adding inferior or harmful
The sentences formed : substances to food, beverages, or other
A-E : By being positive role models and products, poses a serious threat to public health
displaying qualities such as kindness, resilience, and undermines the integrity of industries and
businesses.

Click Here For Bundle PDF Course | support@guidely.in Page 8 of 10


Bank Po Mains PDF Course 2024
English Day - 6

12. Answer: A 16. Answer: A


Here, the first part after rearrangement will be a Here, the first part after rearrangement will be a
which is introductory part starting with the ties which gives introductory part as need of billions
between India and Nepal followed by b which of dollars for Pakistan followed by b which is
gives the reason for the stress mentioned fixed followed by c which gives reason for need
followed by c and then d and finally ends with e of money followed by d then f and finally ends
and f in which f is already fixed. So, option A is with e which shows how much amount is left with
the correct answer. them. So, option A is the correct answer.

13. Answer: E 17. Answer: C


Here, no rearrangement is required and The word prioritize means to give preference to
therefore, option E is the correct answer. one thing over the other as in the sentence,
giving preference to the development of human
14. Answer: C potential is being talked about. The words prefer
Here, the first part after rearrangement will be b and favour from the options can be taken as its
as it gives introductory part as former vice synonyms. Negate and overlook are the
president followed by c which gives information antonyms of the given word. Unscrupulous
about a challenge launched by him followed by d means being dishonest, cruel, or unfair in order
and then e which is fix followed by f which gives to get what you want.
information about details submitted by him and Virtuous is a positive word which means
finally ends with a which is ending part. So, behaving in a morally good way. In the sentence
option C is the correct answer. it means being righteous and moral when it
comes to development. From (a) and (c), (a)
15. Answer: A gets cancelled because, promiscuous means
Here, the first part after rearrangement will be a amoral or not morally right. This is opposite the
which is introductory part as it gives information required meaning.
about some mismanagement from years The best would be to go with option (c).
followed by b which gives information about
whom it is referring followed by c then e which 18. Answer: A
gives information about the floods and then Martinet means a strict disciplinarian which is
followed by d and finally ends with f which is opposite of liberal. This cancels option (b).
fixed. So, option A is the correct answer. The best option that carries the synonyms of the
given words is (a).

Click Here For Bundle PDF Course | support@guidely.in Page 9 of 10


Bank Po Mains PDF Course 2024
English Day - 6

19. Answer: A ‘animate’ is to bring to life. So, it is almost


All the first words given in the options are opposite to ‘derailed’.
synonyms of each other and all of these mean to Considering the above explanation, we can
kill or murder. So, any of these can be taken as eliminate options (a), (b) and (c) to mark option
the correct answer but, the second word ‘viable’ (d) as the correct answer.
which means practical or feasible should take a
synonym that means the same. 21. Answer: C
‘Judicious’ means sensible which fits the context The word ‘uttered’ is used for ‘speaking’ or
as the sentence talks about peace being the only ‘expressing something’. The sentence says that
sensible option. The other options do not carry the PM has not spoken a word about the recent
appropriate words. incidents of violence. ‘Warring groups’ means
Therefore, the best answer is option (a). the parties that are fighting with each other or
are in conflict.
20. Answer: D ‘Benevolent’ in the first option means friendly
To vitalize is to give energy or strength to things. and this doesn’t fit the context. So, the first
The sentence talks about ‘taming inflation’ that is option can be cancelled. In (b), diplomatic makes
controlling the same and not encouraging it. This no sense. ‘Pugnacious’ means eager to argue or
helps us cancel option (a). Insipid is something quarrel. Both words in option (c) are correct.
that lacks energy or colour etc. This is the Nonplussed is a state of being extremely
opposite of vitalize but it is still not the answer confused or bewildered. Dissention is
as, it doesn’t fit the context. disagreement so that can be eliminated. Words
Deprecate is to depreciate or lessen which is in (e) are irrelevant as well.
close but we have a better word ‘rein’ which So, the correct answer is option (c).
means to ‘control’ and that would fit better. To

Click Here For Bundle PDF Course | support@guidely.in Page 10 of 10


Bank Po Mains PDF Course 2024
Reasoning Day -7 (Eng)

Reasoning Aptitude
Directions (1-5): Study the following information 1) What is the position of F with respect to the
carefully and answer the given questions. one who sits opposite to ___?
Fourteen persons are sitting in two parallel rows a) Third to the right, T
containing seven persons each in a way that b) Second to the left, S
there is equal distance between adjacent c) Immediate left, R
persons. In row1 A, B, C, D, E, F, and G are d) Fourth to the right, P
seated and some of them facing north and some e) Immediate right, Q
of them facing south. In row2 P, Q, R, S, T, U,
and V are seated and some of them facing north 2) ___ sits ___ to the right of P and sits opposite
and some of them facing south. Row1 is to the to ___.
north of row2. a) U, immediate, D
Note: Not more than two persons facing the b) V, Second, F
same direction sit together. c) Q, third, B
E sits third from one of the extreme ends. The d) R, Second, C
one who sits opposite to E sits third to the left of e) S, Second, D
V. T sits third to the left of S, where both are
facing different directions. Neither T nor S sits 3) Who among the following person sits opposite
adjacent to V. D sits fourth to the right of F. F sits to E?
second to the left of A, who faces south. E and F a) U
are not immediate neighbours. U, who doesn’t sit b) The one who sits third to the left of T
at any of the extreme end, sits opposite to the c) The one who sits second to the right of R
one who sits third to the left of C. The one who d) V
sits adjacent to U sits opposite to G, who faces e) None of these
south. U and R are facing the same direction.
The number of persons sitting between B and E 4) Which of the following statement is not true
is one less than the number of persons sitting to with respect to the final arrangement?
the right of R. G doesn’t sit adjacent to F. The a) A sits immediate left of C
persons sitting at the extreme ends of each row b) P and Q face same direction
are facing opposite direction. P sits third to the c) Only two persons are sitting between F and
left of the one who sits immediate left of Q, who the one who sits opposite to S
face opposite direction of E. S doesn’t sit to the d) S sits second to the left of V
left of P. e) D doesn’t sit immediate left of E

Click Here For Bundle PDF Course | support@guidely.in Page 1 of 11


Bank Po Mains PDF Course 2024
Reasoning Day -7 (Eng)

5) Four of the five are alike in a certain way c) qbztyxp


based on the given arrangement and thus form a d) 64
group. Which one of the following doesn’t belong e) ytcjz
to the group?
a) The one who sits third to the right of A 7) What is the sum of the highest number in step
b) The one who sits opposite to F III and the lowest number in step II?
c) Q a) 79
d) The one who sits immediate right of U b) 82
e) B c) 67
d) 74
Directions (6-10): Study the following information e) 87
carefully and answer the given questions.
An alphanumeric machine when given an input 8) Which of the following element is second from
rearranges them by following a particular rule in the left end and second from the right end
each step. The following is an illustration of the respectively in step I?
input and steps rearrangement: a) 2473, tqwvgpc
Input: 4376 Work Round 2861 3586 Blink b) aqxgb, 9442
Service 4379 c) btgxc, 3636
Step I: 4821 zjgpi 3628 tqwpb 5624 uctxgac d) 9442, kcbgayj
2149 yqti e) tqwvhpd, 1824
Step II: 15 aqtkr 19 gjdky 17 fxgctzx 16 bjgr
Step III: 16 25 36 64 rktqa rgjb xztcgxf ykdjg 9) Which of the following sequence correctly
Step IV: af be cf fd kq gj zx kj represents the step V?
Step V: zx af kq be kj cf gj fd a) zy ag xg be xy di qp fd
Step V is the last and final step of the b) yz af xg be xy di qp fd
rearrangement. With the same rules followed in c) xg be xy zy ag di qp fd
the above arrangement, determine the steps for d) af xg yz be xy di qp fd
the following input e) tj af kj bx fd cf di cg
Input: 4396 Drive Covid Medical 2364 6974 10) If “aqxgb” is related to “3636” in step I and
Routine 5963 “fd” is related to”kj” in step IV, then which of the
6) Which of the following element is third from following element is related to “gjdetkx” in step
the right end in step III? II?
a) 25 a) ygtcx
b) xctgy b) 17

Click Here For Bundle PDF Course | support@guidely.in Page 2 of 11


Bank Po Mains PDF Course 2024
Reasoning Day -7 (Eng)

c) zjcty between O and R, who was born two days after


d) 19 E. G faces the one who was born on 24th and
e) btgxc sits immediate left of A. Z was born 16 days after
P, who sits immediate left of the one who is
Directions (11-15): Study the following eldest among all. The number of persons sitting
information carefully and answer the given between A and I is one less than the number of
questions. persons sitting between R and P, when counted
Ten persons – A, D, E, G, I, M, O, P, R, and Z from the left of A and right of R. I do not sit on the
are sitting around two concentric square tables in same side as G, who was not born on a prime
such a way that four persons are sitting at the number date. The one who faces the one who
inner table whereas six persons are sitting at the was born on a perfect square number date sits
outer table as shown in the below diagram. Each third to the right of M. The difference between the
person was born on different dates viz. - 2, 4, 6, birth date of M and E is a multiple of 5.M neither
5, 8, 9, 14, 17, 21, and 24 of the same month of sits adjacent to E nor sits at the inner table. Not
the same year. more than four persons are elder than D.
11) Who among the following person was born
on 14?
a) O
b) The one who sits immediate right of R
c) The one who sits adjacent to the eldest person
d) P
e) Z

12) ___ faces the one who was born on ___ and
Note I: The person who was born on an odd
sits two places away from ___.
numbered date is sitting at the inner table and
a) I, 2, A
the person who was born on an even numbered
b) M, 21, P
date is sitting at the outer table.
c) E, 17, P
Note II: The persons whose name is a vowel are
d) Both a and b
not sitting adjacent to each other.
e) Both a and c
Note III: If A is facing B, then both are sitting at
different tables.
13) Which among the following pair of persons
O faces the one who sits second to the right of
are sitting at the same side of the table?
the one who was born on 17th. One person sits
a) ZE

Click Here For Bundle PDF Course | support@guidely.in Page 3 of 11


Bank Po Mains PDF Course 2024
Reasoning Day -7 (Eng)

b) RO I. Only one day gap between the classes for P


c) GA and S, both have classes on different timings.
d) DP II. R has class after T, but not has the class
e) Both a and c adjacent to each other.
III. S doesn’t have class at last on any of the
14) Who among the following person is the days
second youngest? IV. Q has class along with P but not in morning
a) The one who sits second to the right of M on any of the days.
b) R The persons may have class only on the below
c) The one who faces O mentioned days.
d) G
e) The one who sits immediate right of Z

15) Which of the following combination is true?


a) I-5
b) G-9
c) D-2
d) P-6
e) Z-21 16) Q has class on which of the following day
and time?
Directions (16-20): Study the following a) 6PM-8PM, Monday
information carefully and answer the given b) 2PM-4PM, Saturday
questions. c) 2PM-4PM, Friday
Five persons have their classes on three different d) Both a and c
days in a week from Sunday to Saturday at three e) Both b and c
different timings- 8AM-10AM, 2PM-4PM, and
6PM-8PM. Each of them has classes at least 17) Who among the following person has class
once but not more than twice in a week. No one on Saturday in the same sequence?
has class twice in a day. Each of them has a) TQP
classes at different timings on all three days. b) SRT
There must be three persons or no person has c) PRT
class in a day. d) QTP
Note: e) No class

Click Here For Bundle PDF Course | support@guidely.in Page 4 of 11


Bank Po Mains PDF Course 2024
Reasoning Day -7 (Eng)

18) Which of the following combination is not a) Only III


true? b) Both I and II
a) T, 2PM-4PM, Monday c) Only II
b) R, 6PM-8PM, Wednesday d) Both II and III
c) T, 6PM-8PM, Wednesday e) Both I and III
d) S, 2PM-4PM, Wednesday
e) Q, 2PM-4PM, Friday 20) Which of the following statement is true with
respect to the final arrangement?
19) Who among the following person has classes a) R has class only on Monday
at 6PM-8PM time? b) R has class after S
I. Q c) S has class on Wednesday and Saturday
II. T d) T has class at 6PM-8PM
III. P e) None of the given statement is true
Click Here to Get the Detailed Video Solution for the above given Questions
Or Scan the QR Code to Get the Detailed Video Solutions

Answer Key with Explanation


Direction (1-5):
1. Answer: B
2. Answer: D
3. Answer: C
4. Answer: C
5. Answer: B (All the persons sit at the extreme
ends of the row, except option b)
We have,
Final arrangement
 E sits third from one of the extreme ends.

Click Here For Bundle PDF Course | support@guidely.in Page 5 of 11


Bank Po Mains PDF Course 2024
Reasoning Day -7 (Eng)

 The one who sits opposite to E sits third  The number of persons sitting between B
to the left of V. and E is one less than the number of
 T sits third to the left of S, where both are persons sitting to the right of R.
facing different directions.  G doesn’t sit adjacent to F.
 Neither T nor S sits adjacent to V. After applying the above conditions case-2 and
From the above conditions, there are four case-2a get eliminated because in row 2 more
possibiities than two persons facing the same direction sits
together.

Again, we have
 D sits fourth to the right of F.
 F sits second to the left of A, who faces
Again, we have
south.
 The persons sitting at the extreme ends of
 E and F are not immediate neighbours.
each row are facing opposite direction.
 P sits third to the left of the one who sits
immediate left of Q, who face opposite
direction of E.
 S doesn’t sit to the left of P.
After applying the above conditions case-1a gets
eliminated because S sits to the left of P, hence
Again, we have case-1 shows the final arrangement.
 U, who doesn’t sit at any of the extreme
end, sits opposite to the one who sits third
to the left of C.
 The one who sits adjacent to U sits
opposite to G, who faces south.
 U and R are facing the same direction.

Direction (6-10):

Click Here For Bundle PDF Course | support@guidely.in Page 6 of 11


Bank Po Mains PDF Course 2024
Reasoning Day -7 (Eng)

6. Answer: C whereas N is in the second half in the


7. Answer: A alphabetical series then it is changed to the
8. Answer: B second succeeding letter i.e.P
9. Answer: E Step I: 4821 zjgpi 3628 tqwpb 5624 uctxgac
10. Answer: D 2149 yqti
We have, For step II: All the digits are added within the
Input: 4376 Work Round 2861 3586 Blink numbers and all the letters are changed to their
Service 4379 complimentary pair as per the alphabetical order.
For step I: Firstly, all the numbers are added Step II: 15 aqtkr 19 gjdky 17 fxgctzx 16 bjgr
within the number then the resultant is arranged For step III: The square value of the difference of
in ascending order, such that if the resultant digits of numbers are taken and arranged in
number is an even then, product of the first and ascending order, followed by the words are
the third digit is written at the end and retained arranged in alphabetical order and then the
the remaining two digits of each number, else if letters within each word are written in reverse
the resultant number is an odd, then the product order.
of second and the third digit is written at the Step III: 16 25 36 64 rktqa rgjb xztcgxf ykdjg
beginning and retained the remaining two digits For step IV: The digits in each number are
of each number. changed with the place value of the letters as
4376-20; 2861-17; 3586-22; 4379-23 per English alphabetical order, whereas the
17 20 22 23 letter which is second from both the ends of
4821 3628 5624 2149 each word are taken.
First, all the words are arranged in alphabetical Step IV: af be cf fd kq gj zx kj
order from left to right and then the letter which For step V: Words are arranged in alternate way
comes in the first half of the alphabetical series such that the reverse alphabetical and
is changed to the second preceding letter alphabetical words are written.
whereas the letter which comes in the second Step V: zx af kq be kj cf gj fd
half of the alphabetical series is changed to the Based on the rule followed we have output for
second succeeding letters as per the English the given input.
alphabetical order. Input: 4396 Drive Covid Medical 2364 6974
Blink Round Service Work Routine 5963
For example in the word blink B is in the first half Step I: 1824 aqxgb 3636 btgxc 5453 kcbgayj
of the letter in the alphabetical series then it is 9442 tqwvgpc
changed to the second preceding letter i.e z, Step II: 15 zjcty 18 ygtcx 17 pxytzbq 19 gjdetkx

Click Here For Bundle PDF Course | support@guidely.in Page 7 of 11


Bank Po Mains PDF Course 2024
Reasoning Day -7 (Eng)

Step III: 16 36 49 64 xktedjg qbztyxp xctgy ytcjz


Step IV: af cf di fd kj bx cg tj
Step V: tj af kj bx fd cf di cg
Step V is the final output.

Direction (11-15):
11. Answer: C
12. Answer: E
13. Answer: E
14. Answer: C
15. Answer: B
Final arrangement

Again, we have
 G faces the one who was born on 24th
and sits immediate left of A.
 Z was born 16 days after P, who sits
immediate left of the one who is eldest
among all.
 The number of persons sitting between A
and I is one less than the number of
persons sitting between R and P, when
We have, counted from the left of A and right of R.
 O faces the one who sits second to the  I do not sit on the same side as G, who
right of the one who was born on 17th. was not born on a prime number date.
 One person sits between O and R, who After applying the above conditions case-2 gets
was born two days after E. eliminated because there is no possibility to
Since, R must sit at the outer table so, place P. Case 3 and 4 gets eliminated, because
only such possible combinations are (4, there is no possibility to place I.
6), (2, 4) and (6, 8).
From the above conditions, there are four
possibilities

Click Here For Bundle PDF Course | support@guidely.in Page 8 of 11


Bank Po Mains PDF Course 2024
Reasoning Day -7 (Eng)

Direction (16-20):
16. Answer: D
17. Answer: E
18. Answer: C
19. Answer: E
20. Answer: B

Again, we have Final arrangement

 The one who faces the one who was born


on a perfect square number date sits third
to the right of M.
 The difference between the birth date of
M and E is a multiple of 5.
Since, R was born two days after E, thus E must
born on 4.
Now, only possible birth date of M is 9/14.
 M neither sits adjacent to E nor sits at the
inner table.
 Not more than four persons are elder than
D.
Since, possible birth date of D is either 21 or 5,
thus D must born on 5.
After applying the above conditions case-1 gets We have,
eliminated because E and M can’t sit together, Note:
hence case-1a shows the final arrangement. I. Only one day gap between the classes for P
and S, both have classes on different timings.

Click Here For Bundle PDF Course | support@guidely.in Page 9 of 11


Bank Po Mains PDF Course 2024
Reasoning Day -7 (Eng)

II. R has class after T, but not has the class Case 1: S has class at 8AM-10AM, and P has
adjacent to each other. class at either 2PM-4PM or 6PM-8PM.
III. S doesn’t have class at last on any of the Case 2: S has class at 2PM-4PM, and P has
days class at either 8AM-10AM or 6PM-8PM.
IV. Q has class with P but not in morning on any Q has class with P but not in morning on any of
of the days. the days.
From the above conditions, there are four
possibilities

From above mentioned note and days we have


some conclusion

From above table now we can conclude that all


the persons have classes only on Monday,
Wednesday, and Friday.
Also, S has class only on Wednesday.
Also, we have
S doesn’t have class at last on any of the days
Now, again we have
Thus, S has class either at 8AM-10AM or 2PM-
R has class after T, but not has the class
4PM.
adjacent to each other.
Only one day gap between the classes for P and
Since, only possible day available for T is
S, both have classes on different timings.
Monday and Wednesday.
From the above conditions, there are two
possibilities

Click Here For Bundle PDF Course | support@guidely.in Page 10 of 11


Bank Po Mains PDF Course 2024
Reasoning Day -7 (Eng)

We know, S has class only on Wednesday, so


all the remaining persons must has class twice
as totally there are 9 classes.
After applying the above conditions Case (1) and
(1A) is not valid as R and T can’t take classes
together and case (2A) is not valid as T has
classes at same time, hence case-2 shows the
final arrangement.

Click Here For Bundle PDF Course | support@guidely.in Page 11 of 11


Bank Po Mains PDF Course 2024
Quantitative Aptitude Day -7 (Eng)

Quantitative Aptitude

Directions (1-5): Following questions contain two data provided in the statements are sufficient to
statements as statement I and statement II. You answer the question.
have to determine which statement/s is/are The total number of students in classes A, B and
necessary to answer the question and give C together is (4x + 28). The ratio of total number
answer as, of students in class A to class B is 5:3
1) The length of the rectangle is 19 cm more respectively. Find the value of x?
than its breadth and then find the area of the Statement I: The total number of students in
rectangle if the perimeter of the rectangle is class A is 24 more than the same in class C. If a
equal to the value of x. student is randomly chosen from classes A, B
Statement I: The average height of 25 students and C then the probability that the student
in school is 150 cm. 7 new students with an belongs to class B is 1/4.
average height of x cm joined in school and the Statement II: The total number of students in
average height becomes (x-25) cm. class C is 20% less than total number of students
Statement II: The breadth of the rectangle is in class A.
71.42% more than the radius of the cone and the A. The data in statement I alone is sufficient to
volume of the cone is 6930 cm3. answer the question while data given in
A. The data in statement I alone is sufficient to Statement II is not sufficient to answer the
answer the question, while the data in statement question.
II alone is not sufficient to answer the question B. The data in statement II alone is sufficient to
B. The data in statement II alone is sufficient to answer the question while data given in
answer the question, while the data in statement Statement I is not sufficient to answer the
I alone is not sufficient to answer the question question.
C. The data either in statement I alone or in C. The data in either statement I alone or
statement II alone is sufficient to answer the statement II alone is sufficient to answer the
question question.
D. The data given in both statements I and II D. The data given in both statements I and II
together are not sufficient to answer the question together are not sufficient to answer the
E. The data given in both statements I and II question.
together are necessary to answer the question. E. The data in both statements I and II together
are necessary to answer the question.
2) The question consists of two statements I and
II given below it. You have to decide whether the 3) Find the speed of boat A?

Click Here For Bundle PDF Course | support@guidely.in Page 1 of 15


Bank Po Mains PDF Course 2024
Quantitative Aptitude Day -7 (Eng)

Statement I: Speed of stream is 4 km/hr and boat B. The data in statement II alone is sufficient to
B covers 48 km upstream in 3 hours. answer the question, while the data in statement
Statement II: Speed of boat B in still water is 20 I alone is not sufficient to answer the question
km/hr and Boat A covers 52 km downstream in 2 C. The data either in statement I alone or in
hours. statement II alone is sufficient to answer the
A. The data in statement I alone is sufficient to question
answer the question, while the data in statement D. The data given in both statements I and II
II alone is not sufficient to answer the question together are not sufficient to answer the question
B. The data in statement II alone is sufficient to E. The data given in both statements I and II
answer the question, while the data in statement together are necessary to answer the question.
I alone is not sufficient to answer the question
C. The data either in statement I alone or in 5) Find the age of P?
statement II alone is sufficient to answer the Statement I: P is 5 years older than Q who is 10
question years younger than R. Average age of R and Q
D. The data given in both statements I and II is 25 years old.
together are not sufficient to answer the question Statement II: Ratio of age of R and P is
E. The data given in both statements I and II 4:7andratio of age of R and Q is 2:3.
together are necessary to answer the question. A. The data in statement I alone is sufficient to
answer the question, while the data in statement
4) In how many days can A alone complete the II alone is not sufficient to answer the question
work? B. The data in statement II alone is sufficient to
Statement I: Ratio of efficiency of A and B is 3:2. answer the question, while the data in statement
C alone can complete the work in 16 days which I alone is not sufficient to answer the question
is 10 days more than the number of days taken C. The data either in statement I alone or in
by D alone to complete the work. statement II alone is sufficient to answer the
Statement II: B alone can complete the work in question
12 days and B and D together can complete the D. The data given in both statements I and II
work in 4 days. Ratio of efficiency of A and D is together are not sufficient to answer the question
3:4. E. The data given in both statements I and II
A. The data in statement I alone is sufficient to together are necessary to answer the question.
answer the question, while the data in statement
II alone is not sufficient to answer the question Directions (6-10): Following questions have two
quantities as Quantity I and Quantity II. You have

Click Here For Bundle PDF Course | support@guidely.in Page 2 of 15


Bank Po Mains PDF Course 2024
Quantitative Aptitude Day -7 (Eng)

to determine the relationship between them and E. Quantity I ≤ Quantity II


give answer as,
6) 8)
Quantity I: Ratio of A’s present to B’s present Quantity I. Probability of not solving a question
age is 3: 2 and after 2 years, B’s age will be 70% by P, Q, and R is 0.25, 0.2, and 0.5 respectively.
of A’s age. If difference between A’s present age Find the probability that at most two of them
and C’s present age is 2 years, then find the C’s solve the question.
age after 4 years? Quantity II.A bag contains ten red pens and six
Quantity II: A boat is travelling in a river and black pens, if three pens are drawn at random
speed of stream is 20% of speed of boat in still from the bag. Then find the probability of getting
water. If time taken by boat to go 960 m at least one black pen.
downstream is 20 seconds less than that taken A. Quantity I > Quantity II
to come back, then find the speed of boat in still B. Quantity I < Quantity II
water? C. Quantity I = Quantity II or No relation
A. Quantity I > Quantity II D. Quantity I ≥ Quantity II
B. Quantity I < Quantity II E. Quantity I ≤ Quantity II
C. Quantity I ≥ Quantity II
D. Quantity I ≤ Quantity II 9) A, B and C together can complete a work in 8
E. Quantity I = Quantity II or relation can't be hours, while C alone can complete it in 24 hours.
determined Ratio of A’s efficiency to B’s efficiency is 2: 1.
Quantity I: In what time A alone will complete
7) 50% work?
Quantity I. M travelled from point P to Q with Quantity II: If difference between time taken by A
speed of 6 km/hr and then comes to point P with alone and D alone to complete the work is 9
speed of 8 km/hr. N travelled from point P to Q hours, then in what time A and D will complete
with speed of 16 km/hr. Total journey time of M is the work together?
22 hours more than that of N. Find the distance A. Quantity I > Quantity II
between P and Q. B. Quantity I < Quantity II
Quantity II. Find the value of 33K, where K is the C. Quantity I ≥ Quantity II
unit digit of 3353. D. Quantity I ≤ Quantity II
A. Quantity I > Quantity II E. Quantity I = Quantity II or relation can't be
B. Quantity I < Quantity II determined
C. Quantity I = Quantity II or No relation
D. Quantity I ≥ Quantity II

Click Here For Bundle PDF Course | support@guidely.in Page 3 of 15


Bank Po Mains PDF Course 2024
Quantitative Aptitude Day -7 (Eng)

10) In a mixture of milk and water, quantity of 25% above the cost price and allowed a discount
milk is 10 L more than that of water. Cost of pure of Rs. 125 on the kurti. She sold the saree for
milk is Rs.48 per L and the mixture is sold at Rs. _________ and the profit earned on the saree
Rs.36 per L at 20% profit. is equal to the profit earned on the kurti.
Quantity I: Find the total quantity of mixture? Which of the following option satisfies the two
Quantity II: If 8 L mixture is replaced with same blanks in the question?
quantity of milk, then what will be the new cost i. 1000 & 1200
per L of the mixture? ii. 1600 & 1850
A. Quantity I > Quantity II iii. 1300 & 1560
B. Quantity I < Quantity II iv. 1400 & 1650
C. Quantity I ≥ Quantity II A. Only i and iii
D. Quantity I ≤ Quantity II B. Only iii and iv
E. Quantity I = Quantity II or relation can't be C. Only ii and iv
determined D. Only i and ii
E. Only i, ii and iv
11) In a cone, if diameter is increased by __%
and height is increased by __% then the 13) The sum of the speeds of boat ‘A’ and ‘B’ in
volume will be increased by __%. Find the still water is 64 km/hr such that the speed of boat
correct optionto fill the blanks in order to make B in still water is ________ km/hr and the speed
the statement correct, if height and diameter of of the current is 16 km/hr. The time taken by the
cone are 32cm and 24cm, respectively? boat ‘A’ to cover 196 km in downstream is
i) 12, 25, 56.8 _______ hours.
ii) 20, 5, 51.2 The values given in which of the following
iii) 10, 20, 44 options will fill the blanks in the same order in
iv) 5, 20, 32.3 which is it given to make the statement true?
A. i only i. 24, 3.5
B. i and iii only ii. 31, 4
C. i, ii and iv only iii. 40, 4.9
D. i and ii only A. Only i and iii
E. ii and iv only B. Only i and ii
C. All i, ii and iii
12) Aishu bought a saree and a kurti for Rs. D. Only i
________ and Rs. 1500 respectively. She marked E. None of these
the saree 20% above the cost price and the kurti

Click Here For Bundle PDF Course | support@guidely.in Page 4 of 15


Bank Po Mains PDF Course 2024
Quantitative Aptitude Day -7 (Eng)

14) A vessel contains a mixture of milk and water Directions (16-20): In the questions the question
(200 ml) in which milk and water is in ratio of 4:1 is followed by four statements. Read all the
respectively if (K + 4) ml of mixture is removed statements carefully and find which of the
and replaced by the same amount of water then following statements cannot answer the
concentration of milk becomes Q%. question.
Find which of the following statement satisfied 16) A, B and C were travelling between cities X
the pair (K, Q) respectively. and Y by bus, car and train respectively. Speed
I. 36, 64 of the bus is 40 kmph. What is the distance
II. 16, 72 between city X and Y?
III. 46, 60 A. Ratio of speed of bus to train is 1:2 and
A.I and III only difference between time taken by them is 3 hours
B.II and III only B. If the bus runs 25% faster, it will take 20% less
C.I and II only time to cover the distance
D.I, II, and III only C. Average of speed of a bus and car is 50 kmph
E. None of these and the average time taken is 5 hrs.
D. If the car covered half of its journey at the
15) Anmol invested Rs. (2P + 1500) in a bank, speed of bus and remaining half at twice the
offers simple interest at 11.11% per annum. After speed of bus, time taken will be 4.5 hours
three years he withdraws all amount from bank E. None of these
and invested in a scheme which offered
compound interest at 50% per annum. Total 17) Ram, Shyam, Prakash and Ankush started a
amount received from scheme after two years is business and invested money in a ratio 10:12:8:9
Rs. (10.2P – 1800). respectively. They invested the amount for 8
Find which of the following statement(s) is/are months, 10 months, 6 months and 12 months
definitely true. respectively.What is the profit of the business at
I. Interest received from the scheme is (5.2P – the end of the year?
300). A. Profit of Ankush is Rs 12960.
II. Amount invested by Anmol in bank is Rs. 4500 B. Difference between share of Ram and
III. Interest received from the Bank is Rs. P Prakash is Rs 4000 and difference between their
A.II only shares out of profit is Rs 2112.
B.I and II only C. Average of profit of all of them is Rs 19,936.
C.I and III only D. All of these can answer the question.
D.II and III only E. None of these
E. None of these

Click Here For Bundle PDF Course | support@guidely.in Page 5 of 15


Bank Po Mains PDF Course 2024
Quantitative Aptitude Day -7 (Eng)

18) 16 men working 8 hours a day can complete B. When item C is sold at loss of 10%, number of
a work in 5 days. In how many days 8 men and items that can be bought for Rs 13230 is 10 less
10 women working 6 hours a day can finish the than when it is sold at CP
work? ( Efficiency of men = 3units per hour) C. Numerical average price of item A and B is
A. Ratio of efficiency of men to women is 3:2 less than Rs 100 and is a number that is twice
B.12 women and 12 men working 5 hours a day the square of a prime number less than 10 but
can finish the work in 6 2/5 days. more than 3.
C.8 women work 6 hours a day for 15 days. D. All of these can answer the question
Remaining work is finished by 12 men working 3 E. None of these
1/3 hrs days in 4 days.
D.12 men and 15 women working 5 hours a day 20) All odd multiples of a prime number less than
will finish the work faster than 9 men working 6 20 but more than 10 are arranged in ascending
hours a day and 16 women working 5 hours a order (up to 100) and then added up. What is the
day. average of all the odd multiples of the prime
E. None of these number?
A. The average of the first two even multiples is
19) The cost price of three items A, B and C is in 39.
the ratio 16:12:21 respectively. How many B. Number of odd multiples is one more than
numbers of items C can be bought in Rs 2646? number of even multiples
A. When item A is sold for Rs 140, profit on item C. If all the multiples are arranged in ascending
A is 10% less than profit on item B when it is sold order, the third multiple is less than 50 but the
at Rs 96.60 fourth multiple is more than 50.
D. Sum of all the multiples of the number is 364.
E. None of these

Click Here For Bundle PDF Course | support@guidely.in Page 6 of 15


Bank Po Mains PDF Course 2024
Quantitative Aptitude Day -7 (Eng)

Click Here to Get the Detailed Video Solution for the above given Questions
Or Scan the QR Code to Get the Detailed Video Solutions

Answer Key with Explanation


1) Answer: A 2) Answer: A
Let the breadth of the rectangle=a cm Let the number of students in class A and B is 5y
And the length of the rectangle=(a+19) cm and 3y respectively.
From statement I, Statement I:
25*150+7*x=32*(x-25) No. of students in class C = 5y – 24
3750+7x=32x-800 (4x+28) – (5y+3y) = 5y – 24
x=182 cm 13y – 4x = 52
2*(a+a+19)=182 4x = 13y – 52 --------→ 1
2a=72 3y/ (4x+28) = 1/4
a=36 cm 12y = 4x+28
The length of the rectangle=36+19=55 cm 12y = 13y – 52 + 28
The breadth of the rectangle=36 cm y = 24
The area of the rectangle=55*36=1980 cm2 x = (13*24 - 52)/4
So, Statement I alone is sufficient to answer the x = 65
question. So, the statement I alone is sufficient to answer
From statement II, the question.
The volume of the cone=6930 cm3 Statement II:
1/3*22/7*r2*h=6930 Total no. of students in class C = 4x + 28 – 8y
r2*h=735 4x + 28 – 8y = 5y*80%
So, Statement II alone is not sufficient to answer 4x + 28 - 8y = 4y
the question.

Click Here For Bundle PDF Course | support@guidely.in Page 7 of 15


Bank Po Mains PDF Course 2024
Quantitative Aptitude Day -7 (Eng)

Since, there are two variables in only one 5) Answer: A


equation. So, we cannot determine the value of From statement I
x. Difference age of R and Q is 10 years
So, the statement II alone is not sufficient to Sum of agesof R and Q is 25*2=50years
answer the question. From this we calculate age of R and Q. From
Hence, option a. age of Q, we can calculate age of P.
From statement II,
3) Answer: E We cannot calculate age of P because only ratio
From statement I is given no exact age is given.
We cannot calculate speed of boat A because Hence, the statement I alone is sufficient to
data is given about boat B. speed of stream is 4 answer the question.
km/hr.
From statement II, 6) Answer: C
Speed of boat A+ speed of stream = 52/2 = 26 Quantity I:
km/hr Let A’s present age and B’s present age are ‘3x’
From I and II years and ‘2x’ years respectively.
Speed of boat A is 26-4 = 22 km/hr. So, (2x + 2) = (3x + 2) * (70/100)
Hence, the data instatement I and II together are 20x + 20 = 21x + 14
sufficient to answer the question. x=6
A’s present age = 3 * 6 = 18 years
4) Answer: B Since, difference between A’s present age and
From statement I C’s present age is 2 years.
We cannot calculate the number of days taken So, C’s present age will be either 18 + 2 = 20
by A to complete the work because no direct years or 18 – 2 = 16 years.
information given. And C’s age after 4 years will be either 20 + 4 =
From statement II, 24 years or 16 + 4 = 20 years.
D alone complete the work in = 1/ (1/4-1/12) = 6 Quantity II:
days Let speed of boat in still water = ‘5x’ m/s
A alone complete the work in = 6*(4/3) = 8 days So, speed of stream = 20% of ‘5x’ = ‘x’ m/s
Hence, the statement II alone is sufficient to Downstream speed of boat = (5x + x) = ‘6x’ m/s
answer the question. Upstream speed of boat = (5x – x) = ‘4x’ m/s
Since, time taken by boat to go 960 m
downstream is 20 seconds less

Click Here For Bundle PDF Course | support@guidely.in Page 8 of 15


Bank Po Mains PDF Course 2024
Quantitative Aptitude Day -7 (Eng)

than that taken to come back. Probability (at least 1 black) = 1 – probability (no
So, (960/4x) – (960/6x) = 20 black Pen)
x=4 = 1 – [10C3 / 16C3]
So, speed of boat in still water = 5 * 4 = 20 m/s = 1 – [(10 x 9 x 8)/ (16 x 15 x 14)]
Hence, Quantity I ≥ Quantity II = 1 – 3/14
= 11/14 = 0.78 (approx.)
7) Answer: B So, Quantity I < Quantity II
Quantity I. Hence answer is option B
Average speed of M = 2 x (6 x 8)/ (6 + 8) = 48/7
km/hr 9) Answer: E
Speed of N = 16 km/hr (1/A) + (1/B) + (1/C) = 1/8
Let total time taken by M = a hours (1/A) + (1/B) = (1/8) – (1/24)
So, total time taken by N = (a - 22) hours (1/A) + (1/B) = 1/12 -----------(1)
Total distance travelled by M = 2 x total distance Since, ratio of A’s efficiency to B’s efficiency is 2:
travelled by N 1.
Now, (48/7) x a = 2 x 16 x (a - 22) So, ratio of time taken by A alone to B alone to
3a = 14a – 14 x 22 complete the work = 1: 2
So, value of a = (14 x 22 / 11) = 28 hours From equation (1):
Time taken by N = (28 – 22) = 6 hours (1/A) + (1/2A) = 1/12
Required distance = 16 x 6 = 96 km A = 18
Quantity II. Quantity I:
3353 = 334 x 13 + 1 Time taken by A alone to complete the whole
So, unit digit of 3353 = 3 work = 18 hours
Value of 33K = 33 x 3 = 99 So, time taken by A to complete 50% work =
So, Quantity I < Quantity II 18/2 = 9 hours.
Hence answer is option B Quantity II:
Time taken by A alone to complete the work = 18
8) Answer: B hours
Quantity I. So, time taken by D alone to complete the work
Required probability = 1 – [(1 – 0.25) x (1 – 0.2) x will either be 18 + 9 = 27 hours or 18 – 9 = 9
(1 – 0.5)] = 0.7 hours.
Quantity II. If time taken by D alone to complete the work =
27 hours

Click Here For Bundle PDF Course | support@guidely.in Page 9 of 15


Bank Po Mains PDF Course 2024
Quantitative Aptitude Day -7 (Eng)

(1/18) + (1/27) = (3 + 2)/54 = 1/10.8 New quantity of water in the mixture = 15 –


So, time taken by A and D together to complete 8(3/8) = 12 L
the work = 10.8 hours So, new cost of mixture = (28 * 48)/40 = Rs.33.6
If time taken by D alone to complete the work = 9 per L
hours Hence, Quantity I > Quantity II
(1/18) + (1/9) = (1 + 2)/18 = 1/6
So, time taken by A and D together to complete 11) Answer: C
the work = 6 hours According to question,
Hence, relation can't be determined. In a cone, diameter increases by __%
Height increases by __%
10) Answer: A Volume increases by __%
Let quantity of water in the mixture = ‘x’ L Percentage change does not depend on height
So, quantity of milk in the mixture = (x + 10) L and radius of cone
Since, mixture is sold at Rs.36 per L at 20% So, volume of cone = 1/3 ∏ (radius)² height
profit. i) 12, 25, 56.8
So, cost of the mixture = 36 * (100/120) = Rs.30 New volume = 1/3 ∏ (1.12 radius)² *(1.25 height)
per L = 1.568 x 1/3 ∏ (radius)² *height
Since, cost of pure milk is Rs.48 per L. Increase in volume = 56.8%
So, [(x + 10) * 48]/ (x + 10 + x) = 30 ii) 20, 5, 51.2
48x + 480 = 60x + 300 New volume = 1/3 ∏ (1.20 radius)² *(1.05 height)
x = 15 = 1.512 x 1/3 ∏ (radius)² *height
Quantity of water in the mixture = 15 L Increase in volume = 51.2%
Quantity of milk in the mixture = (15 + 10) = 25 L iii) 10, 20, 44
Quantity I: New volume = 1/3 ∏ (1.10 radius)² *(1.20 height)
Total quantity of the mixture = 25 + 15 = 40 L = 1.452 x 1/3 ∏ (radius)² *height
Quantity II: Increase in volume = 45.2%
Initial ratio of milk to water in the mixture = 25: So iii cannot follow
15 = 5: 3 iv) 5, 20, 32.3
If 8 L mixture is replaced with same quantity of New volume = 1/3 ∏ (1.05 radius)² *(1.20 height)
milk: = 1.323 x 1/3 ∏ (radius)² *height
New quantity of milk in the mixture = 25 – 8 * Increase in volume = 32.3%
(5/8) + 8 = 28 L Hence, answer is option C

Click Here For Bundle PDF Course | support@guidely.in Page 10 of 15


Bank Po Mains PDF Course 2024
Quantitative Aptitude Day -7 (Eng)

12) Answer: C ATQ, 196/ (33 + 16) = 196/49 = 4 hours


By using optional method, Option ii is correct.
Option i, Option iii,
Cost price of kurti = Rs.1500 Boat speed of B = 40 km/hr
Marked price of kurti = 1500 * 125% = Rs.1875 So, Boat speed of A in still water = 64 – 40 = 24
Profit = 1875 - 125 – 1500 = Rs.250 km/hr
Cost price of saree = Rs.1000 ATQ, 196/ (24 + 16) = 196/40 = 4.9 hours
ATQ, Profit of kurti = profit of saree Option iii is correct.
So, selling price of saree = Rs.1200 Hence, the correct answer is option c.
Profit of Saree = 1200 - 1000 = 200 ≠250
Option iis not true. So, we eliminate the option A, 14) Answer: D
D and E. Ratio of milk and water in vessel = 4:1
Option ii, Amount of milk initially = 4/5 x 200 = 160 ml
Cost price of saree = Rs.1600 Amount of water initially = 1/5 x 200 = 40 ml
ATQ, Profit of kurti = profit of saree I. 36, 64
Profit of Saree = 1850 - 1600 = Rs.250 When 40 litres of mixture are removed, ratio of
Hence, option iiis true. So, option C is the correct milk and water is same, 4:1
answer. Amount of milk in mixture = 160 - 32 = 128 ml
Amount of water in mixture = 40 – 8 = 32 ml
13) Answer: C On adding 40 ml of water, amount of water = 32
By using option method, + 40 = 72 ml
Given that, boat speed of A + B = 64 km/hr Required % = 128/200 = 64%
Water speed = 16 km/hr This statement is true
Option i, II. 16, 72
Boat speed of B = 24 km/hr When 20 ml of mixture are removed, ratio of milk
So, Boat speed of A in still water = 64 – 24 = 40 and water is same, 4:1
km/hr Amount of milk in mixture = 160 – 16 = 144 ml
ATQ, 196/ (40 + 16) = 196/56 = 3.5 hours Amount of water in mixture = 40 – 4 = 36 ml
Option i is correct. Now 20 ml of water added, so amount of milk in
Option ii, final mixture is same.
Boat speed of B = 31 km/hr Required % = 144/200 = 72%
So, Boat speed of A in still water = 64 – 31 = 33 This statement is true
km/hr III. 46, 60

Click Here For Bundle PDF Course | support@guidely.in Page 11 of 15


Bank Po Mains PDF Course 2024
Quantitative Aptitude Day -7 (Eng)

When 50 ml mixture removed This statement is true.


Amount of milk in mixture = 160 - 40 = 120 ml All statements are true
Amount of water in mixture = 40 – 10 + 50 = 80 Hence answer is option E
ml
On adding 50 ml of mixture, amount of milk 16) Answer: B
remains same. Speed of bus=40kmph
So, required % = 120/200 x 100 = 60% Let the distance between city X and Y be ‘D’ Km
This statement is true Time taken by bus=D/40 hr
All statements are true. From (a),
Hence answer is option D Speed of train=80 kmph
Time taken by train=D/80
15) Answer: E => D/40-D/80=3
Total amount received from Bank after three => D=240km
years = (2P + 1500) + (2P + 1500) x 1/9 x 3 = From (b), we get
4/3 x (2P + 1500) When bus runs 25% faster, speed of bus=50
Now, kmph
4/3 x (2P + 1500) x 3/2 x 3/2 = (10.2P – 1800) Time taken=D/50
6P + 4500 = 10.2P – 1800 => D/50=80/100*D/40=D/50
4.2P = 6300 This equation can’t be solved as variables equal
So, value of P = 1500 out on both sides.
I. Interest received from scheme is (5.2P – 300) From (c), we get
Money invested in Scheme = 4/3 x (2 x 1500 + Average speed of bus and car= 50 kmph
1500) = Rs. 6000 Speed of car=2*50-40=60 kmph
Required interest = 6000 x [3/2 x 3/2 – 1] = Rs. Time taken by car =D/60
7500 Total time taken=D/60+D/40=2*5
(5.2P – 300) = Rs. 7500 => (2D+3D)/120=10
This statement is true. => D=240 Km
II. Amount invested by Amol in bank is Rs. 4500 From (d), we get
Required amount = 2 x 1500 + 1500 = Rs. 4500 For half of distance, speed of car=40 kmph
This statement is true. For remaining half distance, speed of car=80
III. Interest received from Bank is Rs. P kmph
Interest received from bank = 4500 x 1/9 x 3 = Total time
Rs. 1500 taken=D/2/40+D/2/80=D/80+D/160=3D/160

Click Here For Bundle PDF Course | support@guidely.in Page 12 of 15


Bank Po Mains PDF Course 2024
Quantitative Aptitude Day -7 (Eng)

=> 3D/160=4.5 18) Answer: D


=> D=240 Km Given that 16 men working 8 hours a day can
complete a work in 5 days.
If a man does 3 units of work every hour, total
17) Answer: D work=16*8*5*3=1920
Let the amount invested by Ram, Shyam, From (a),
Prakash and Ankush be 10x, 12x, 8x and 9x Ratio of efficiency of men and women is 3:2.
respectively.They invested the amount for 8 Thus if a man does 3 units of work every hour, a
months, 10 months, 6 months and 12 months woman will do 2 units of work in an hour.
respectively. Thus, time taken for 8 men and 10 women
Ratio of profit in partnership working 6 hours a
=10x*8:12x*10:8x*6:9x:12 =80x:120x:48x:108x = day=1920/(8*3*6+10*6*2)=1920/264=7.27 days
20:30:12:27 From (b),
From (a), Work of 12 men working 5 hours can be
Profit of Ankush is Rs 12960. determined. Remaining work is finished by
=> 27/89 of total profit=12960 women
Total profit=Rs 42720 Thus, efficiency of women can be determined
From (b), and the required value in the question can be
Difference between share of Ram and determined.
Prakash=2x=4000 From (C),
Difference between share of profit=2112 Work of 12 men working 3 1/3 hrs days in 4 days
=> 8/89 of total profit=2112 can be determined. Remaining work was already
Thus, total profit can be determined. finished by women. Thus, efficiency of a woman
From (c), can be determined
Average of total profit=19936 Thus, the required value in the question can be
=> 1/4(20+30+12+27) of total profit=19936 determined
=> 89/4 of total profit=19936 From (d),
22.25=19936 12 men and 15 women working 5 hours a day
89=x will finish the work faster than 9 men working 6
X=79744 hours a day and 16 women working 5 hours a
Thus, total profit can be determined day
This is a comparison and the value of exact time
taken is not given.

Click Here For Bundle PDF Course | support@guidely.in Page 13 of 15


Bank Po Mains PDF Course 2024
Quantitative Aptitude Day -7 (Eng)

So, it can’t be determined. items of C that can be bought in Rs 2646 can


also not be determined.
19) Answer: C
Let the price of the items A, B and C be 16p, 12p 20) Answer: B
and Rs 21p respectively Prime number is less than 20 but more than 10.
From (a), Possible numbers are 11, 13, 17 and 19.
Profit on A when it is sold at Rs 140=140-16p From (a),
Profit on item B when it is sold at Rs First two even multiples average is 39. It
96.60=96.60-12p corresponds to 13’s multiples 26 and 52
Profit % on B- Profit % on A=10 Average of thus all odd multiples can be
=> [(96.60-12p)/12p-(140-16p)/16p]*100 %=10 evaluated.
% From (b),
The equation is solved to find the value of p. Number of odd multiples is one more than the
So, CP of C can be determined and thus number number of even multiples.
of items of C that can be bought in Rs 2646. For 11, multiples are 11, 22, 33, 44, 55, 66, 77,
From (b), 88, 99.
When C is sold at loss of 10%, For 13, multiples are 13, 26, 39, 52, 65, 78 and
SP=21p*0.9=18.9p 91.
So, 13230/18.9p-13230/21p=10 For 17 multiples are 17, 34, 51, 68, and 85
The equation is solved to find value of p. For 19, multiples are 19, 38,57,76,95
So, CP of C can be determined and thus number All of them satisfy the condition.
of items of C that can be bought in Rs 2646. No conclusion about the number can be
From (c), obtained
Numerical average price of item A and B is less From (c),
than Rs 100 and is a number that is twice the When all multiples are arranged in ascending
square of a prime number less than 10 but more order, 3rd multiple is less than 50 but the fourth
than 3 multiple is more than 50.
=> (16p+12p)/2 i.e., 14p <100 Only 13 satisfies this condition
Prime number less than 10 but more than 3 are Average of thus all odd multiples can be
5 and 7. evaluated.
Thus, 14p=2*52=50 or 2*72=98 From (d),
Thus, value of p can’t be determined and so CP Sum of all the multiples of the number is 364.
of C can’t be determined and thus number of Only multiples of 13 satisfies this condition

Click Here For Bundle PDF Course | support@guidely.in Page 14 of 15


Bank Po Mains PDF Course 2024
Quantitative Aptitude Day -7 (Eng)

Average of thus all odd multiples can be evaluated.

Click Here For Bundle PDF Course | support@guidely.in Page 15 of 15


Bank Po Mains PDF Course 2024
English Day - 7

English Language
Direction (1-5): Rearrange the following five D) cdaeb
sentences in a proper sequence to form a E) bdcae
coherent paragraph and then answer the
questions given below. 2) Pick out the erroneous part in sentence A (if
A. Services exports for last month will be known any)
later, but Commerce Ministry extrapolations A) A
suggest that this engine which has been resilient B) B
so far through the global economic turmoil, is C) C
also beginning to feel the heat. D) D
B. For now, the 0.4% drop expected in services E) No error
exports in August is minor and with intangible
imports also likely dropping at a sharper pace, 3) Which of the following is the SYNONYM of the
this will not exacerbate the trade balance yet. word RESILIENT as mentioned in sentence C?
C. India’s goods exports shrank for the seventh A) Fragile
consecutive time and the ninth time in 11 months B) Enduring
this August, while imports surged to hit the C) Vulnerable
highest level since March this year. D) Frail
D. At $58.6 billion, inbound shipments were still E) Susceptible
(A)/ 5.2% below last August’s levels, (B)/ but
exports fell by a relatively steeper 6.9%, (C)/ 4) Which of the following is the ANTONYM of the
leading to a $24.2 billion trade deficit — a widest word INTANGIBLE used in sentence D?
since October 2022. (D) A) Impalpable
E. However, shrinking services exports imply that B) Ineffable
their ability to overcome the goods trade deficits C) Ethereal
that were up sharply last year will be restricted, D) Incorporeal
thus rolling the possibility of wider current E) Perceptible
account deficits from this quarter.
1) Which of the following is the correct sequence 5) Replace the highlighted words in sentence E
of rearrangement that forms a coherent (if necessary).
passage? A) bridge, raising
A) abdec B) extend, lifting
B) acdeb C) prolong, rising
C) cdabe D) build, laying

Click Here For Bundle PDF Course | support@guidely.in Page 1 of 11


Bank Po Mains PDF Course 2024
English Day - 7

E) reconcile, rising of existence with a sense of serenity and


profound __________.
Directions (6-10): In the following questions, a A) stagnation, discord
sentence is given with two blanks. Identify the B) mediocrity, jubilation
correct pair of words that fit in the sentence to C) elevation, equanimity
make it grammatically and contextually correct. D) aberration, ambiguity
6) The enigmatic artist's work not only E) complacency, apathy
__________ the boundaries of traditional art but
also __________ a profound sense of wonder in 9) In the world of advanced technology, where
those who encounter it, leaving an indelible mark creativity and innovation are the __________,
on the world of creativity. companies must constantly seek ways to
A) defied, diminished __________ their products and services, or risk
B) confined, enlightened becoming obsolete in an ever-evolving market.
C) expanded, subsided A) norm, stagnate
D) transcended, evoked B) exception, rejuvenate
E) obscured, stimulated C) paradigm, standard
D) antithesis, streamline
7) As the archaeologists excavated the ancient E) benchmark, upgrade
tomb, they discovered a trove of __________
artifacts that shed light on the civilization's 10) In the realm of cutting-edge scientific
enigmatic rituals and customs. The team was research, where breakthroughs and discoveries
both __________ and overwhelmed by the are the __________, scientists must continually
magnitude of their find. explore uncharted territories, pushing the
A) esoteric, elated boundaries of knowledge to unveil the
B) rudimentary, perturbed __________ mysteries of the universe.
C) mundane, despondent A) exception, obscure
D) antiquated, ecstatic B) norm, enigmatic
E) abstruse, apprehensive C) outlier, apparent
D) paradigm, convoluted
8) Amidst the cacophony of modern life, where E) antithesis, cryptic
the relentless pursuit of material wealth often
overshadows the quest for inner __________, Directions (11-15): In each of the following
many find solace in moments of quiet questions a short passage is given with one of
contemplation, seeking to pacify the wild currents the lines in the passage missing and represented

Click Here For Bundle PDF Course | support@guidely.in Page 2 of 11


Bank Po Mains PDF Course 2024
English Day - 7

by a blank. Select the best out of the given competitive advantage. However,
options, to make the passage complete and ___________________. As organizations continue
coherent. to amass vast amounts of data, it becomes
11) In recent years, the field of artificial imperative to strike a balance between
intelligence (AI) has witnessed remarkable harnessing its potential for innovation and
advancements. AI has become an integral part of ensuring that data is handled responsibly. To
various industries, from healthcare to finance, achieve this, companies must not only implement
revolutionizing the way we operate. However, state-of-the-art data protection measures but
these advancements also raise ethical concerns also allocate resources for data monetization
about the potential misuse of AI. It is crucial for strategies.
society to strike a balance between embracing A) organizations should disregard data privacy
the benefits of AI and ensuring that AI concerns in the pursuit of innovation.
technologies are developed and deployed in B) the digital landscape remains largely
ways that respect fundamental human rights and unaffected by data-related challenges.
privacy. This requires not only C) this increased reliance on data also raises
______________________ but also an ongoing concerns about data privacy and security.
dialogue involving experts, policymakers, and the D) data protection measures have become
public to establish clear guidelines and ethical obsolete in the age of digital transformation.
standards for AI. E) data handling has become a secondary
A) fostering collaboration among AI developers concern for most organizations.
and industry stakeholders
B) advocating for the complete ban of AI in 13) In the age of information overload, staying
sensitive sectors well-informed is a formidable challenge. With the
C) relying solely on government regulations to constant barrage of news, social media updates,
manage AI ethics and online content, individuals often struggle to
D) increasing investment in AI without filter through the noise and discern credible
considering ethical implications information from misinformation. This has led to
E) ignoring public concerns and focusing solely a growing need for media literacy education,
on AI development which empowers people to critically analyze
sources, evaluate information, and make
12) In the era of digital transformation, data has informed decisions.
emerged as the lifeblood of organizations across ___________________________.
industries. The ability to collect, analyze, and A) The government should control media content
leverage data efficiently has become a key to combat misinformation.

Click Here For Bundle PDF Course | support@guidely.in Page 3 of 11


Bank Po Mains PDF Course 2024
English Day - 7

B) Media literacy education helps individuals 15) In today's dynamic and hyperconnected
navigate the complex information landscape but business landscape, the traditional models of
also fosters a sense of digital responsibility. leadership are facing unprecedented challenges.
C) Media literacy education has little impact on Leaders must adapt to rapidly changing markets,
improving information discernment. emerging technologies, and diverse workforces.
D) Misinformation is not a significant issue in the As a result, there is a growing recognition that
digital age. effective leadership requires not only a deep
E) Media literacy education fosters the spread of understanding of business strategies but also the
misinformation and fake news. ability to _________________________."
A) limit communication to preserve leadership
14) In today's fast-paced and interconnected authority.
world, the concept of work-life balance has B) maintain strict hierarchical structures to
undergone a significant transformation. With ensure efficient decision-making.
technology enabling constant connectivity, many C) prioritize individual achievements over team
individuals find it challenging to disconnect from dynamics.
work even during personal time. This shift has D) rely solely on established leadership theories
given rise to discussions about the need to without flexibility.
redefine work-life balance to ensure that E) foster a culture of innovation and collaboration
individuals can maintain their well-being while that empowers team members.
pursuing their careers. While technology has
blurred the lines between work and personal life, Directions (16-20): In each of the questions given
it has also created opportunities for below, four words are given in bold. These four
_________________________. words may or may not be in their correct
A) achieving greater productivity and flexibility in position. The sentence is then followed by
work arrangements. options with the correct combination of words
B) imposing stricter boundaries between work that should replace each other in order to make
and leisure. the sentence grammatically and contextually
C) discouraging remote work and flexible correct. Find the correct combination of words
schedules. that replace each other. If the sentence is correct
D) reducing the overall reliance on technology in as it is, select ‘E’ as your option.
the workplace. 16) Amidst the fervent (A) discussions at the
E) eliminating the need for traditional office international summit, there emerged a view that
spaces. the devaluation (B) of the nation's currency had
led to the unexpected appreciation of its exports,

Click Here For Bundle PDF Course | support@guidely.in Page 4 of 11


Bank Po Mains PDF Course 2024
English Day - 7

thus becoming a general result of the ongoing A) Only A-C


fluctuations (C) in the global economy and the B) Both B-C and A-D
dollar depreciation (D) against multiple major C) Only B-D
currencies. D) Both A-B and C-D
A) Only A-D E) The sentence is correct
B) Both B-C and A-D
C) Only B-D 19) In the wake of the financial crisis, economists
D) Both A-B and C-D endeavoured (A) to analyze the resilient (B) web
E) The sentence is correct of factors that contributed to the market turmoil
(C), scrutinizing the interplay of monetary policy,
17) In the midst of the grand (A) celebrations at and regulatory oversight in order to formulate
the royal palace, there emerged a belief that the more intricate (D) economic frameworks.
rejuvenation (B) of the kingdom's infrastructure A) Only B-D
had led to the unexpected prosperity of its B) Both B-C and A-D
citizens, thus becoming a general consequence C) Only A-D
of the ongoing innovations (C) in governance D) Both A-B and C-D
and the economic modernization (D) of the E) The sentence is correct
nation.
A) Both A-D and C-B 20) Against the backdrop of a navigate (A)
B) Both B-C and A-D evolving technological landscape, relentless (B)
C) Only B-D find themselves at the crossroads of
D) Both A-B and C-D transformation, where they must rapidly (C) the
E) The sentence is correct complexities of patent law, and intellectual
property rights while striving to cultivate a culture
18) Amidst the widespread (A) tides of of entrepreneurs (D) innovation.
geopolitical adroit (B), the diplomat's upheaval A) Both A-D and C-B
(C) negotiation skills not only averted a potential B) Both B-C and A-D
international crisis but also garnered tumultuous C) Only B-D
(D) acclaim for his skilful handling of the delicate D) Both A-C and B-D
situation. E) The sentence is correct

Click Here For Bundle PDF Course | support@guidely.in Page 5 of 11


Bank Po Mains PDF Course 2024
English Day - 7

Click Here to Get the Detailed Video Solution for the above given Questions
Or Scan the QR Code to Get the Detailed Video Solutions

Answer Key with Explanation


1) Answer: C exports on the overall trade balance and current
The correct sequence is: cdabe account deficits, providing a final insight into the
● Sentence C sets the stage by highlighting the economic situation being discussed.
recent trends in India's goods exports and Therefore, option C is the correct answer.
imports. It establishes the key theme of the
passage, which is the trade balance of goods in 2) Answer: D
India during the month of August. The word ‘widest’ is a superlative adjective.
● Sentence D correctly follows as it provides Superlative adjectives are usually preceded by
specific numerical data and context regarding the definite article ‘the’. So, ‘the widest’ is the
the trade balance. correct usage.
● Sentence A will be the third sentence as it Therefore, option D is the correct answer.
provides an introduction to the topic of services
exports and their changing dynamics, following 3) Answer: B
the information about goods exports and Let’s learn the meanings of the words:
imports. ● Resilient - recover quickly from adversity.
● Sentence B correctly follows sentence A as it ● Fragile - Easily broken or damaged.
offers further analysis and context to the ● Enduring - Capable of lasting long.
preceding information about goods exports and ● Vulnerable - Open to harm or attack.
imports. ● Frail - Physically weak and delicate.
● Sentence E will be the last sentence as it ● Susceptible - Easily influenced or affected.
concludes the passage by explaining the
potential consequences of the shrinking services

Click Here For Bundle PDF Course | support@guidely.in Page 6 of 11


Bank Po Mains PDF Course 2024
English Day - 7

From the given meanings, we can understand The first blank should describe how the
that ‘enduring’ is the synonym of the word enigmatic artist's work relates to traditional art.
‘resilient’. We need a word that indicates that the artist's
Therefore, option B is the correct answer. work goes beyond or surpasses the boundaries
of traditional art.
4) Answer: E The word "Transcended" fits this meaning
Let’s learn the meanings of the words: perfectly as it means ‘’.
● Intangible - Not physical, cannot be perceived The second blank should describe the effect the
● Impalpable - Cannot be touched. artist's work has on those who encounter it. We
● Ineffable - Too great to be expressed. need a word that suggests the work inspires a
● Incorporeal - Lacking physical substance. deep sense of wonder or amazement.
● Ethereal - delicate, light, or heavenly in nature. The word "Evoked" is a suitable word choice as
● Perceptible - Able to be perceived. it means a response or reaction that is triggered
From the given meanings, we can understand or brought about by something else.
that the word ‘perceptible’ is the antonym of the All other pairs of words do not make the
word ‘intangible’. sentence contextually correct.
Therefore, option E is the correct answer. Therefore, option D is the correct answer.

5) Answer: A 7) Answer: A
In the given sentence, “ability to bridge" is the The first blank should describe the artifacts
correct usage as it means the capacity to found in the ancient tomb. We need a word that
balance or make up for something. suggests these artifacts are mysterious,
"Raising" means making something more likely understood by only a select few, and related to
or increasing its chances. enigmatic rituals and customs. "Esoteric" fits this
‘Raising’ is the correct usage as it suggests that meaning precisely as it means ‘understood or
the shrinking services exports are leading to a known by a select or specialized group of people
situation where the possibility of wider current and not easily accessible or comprehensible by
account deficits is becoming more likely or the general public’.
increasing. The second blank should describe the emotions
Therefore, option A is the correct answer. of the archaeologists upon discovering these
artifacts.
6) Answer: D We need a word that conveys a sense of joy,
excitement, and happiness, as well as being

Click Here For Bundle PDF Course | support@guidely.in Page 7 of 11


Bank Po Mains PDF Course 2024
English Day - 7

overwhelmed. "Elated" captures these emotions The second blank should describe what
effectively as it means ‘extremely happy, companies must do to stay relevant in this
delighted’. competitive environment. "Upgrade" means to
All other pairs of words do not make the improve or enhance, and in the context of the
sentence contextually correct. sentence, companies need to continually seek
Therefore, option A is the correct answer. ways to enhance and improve their products and
services to stay competitive in an ever-evolving
8) Answer: C market.
The first blank should describe the pursuit of All other pairs of words do not make the
inner peace amidst the chaos of modern life. sentence contextually correct.
In this context, the word "elevation" refers to the Therefore, option E is the correct answer.
idea of raising one's inner self, achieving a
higher state of consciousness, or seeking a 10) Answer: D
more spiritually or morally elevated state. It The first blank should describe the prevailing
implies a pursuit of something greater than conditions in the realm of cutting-edge scientific
material wealth, such as personal growth, research. "Paradigm" implies a widely
wisdom, or inner peace. recognized model or example, and in this
The second blank should describe the state of context, it reflects that breakthroughs and
mind one achieves through quiet contemplation. discoveries are the standard or usual
The word "equanimity," meaning mental occurrences.
calmness and composure, is an ideal choice for The second blank should describe the nature of
this purpose. the mysteries scientists seek to unveil.
All other pairs of words do not make the "Convoluted" means intricate or complex, and it
sentence contextually correct. accurately describes the idea that these
Therefore, option C is the correct answer. mysteries are not straightforward but rather
intricate and challenging to unravel.
9) Answer: E All other pairs of words do not make the
The first blank should describe the prevailing sentence contextually correct.
conditions in the world of advanced technology. Therefore, option D is the correct answer.
"Benchmark" implies a standard or a level of
quality that is widely recognized and accepted, 11) Answer: A
which accurately reflects the competitive and The missing sentence should provide a coherent
innovative nature of the technology industry. and contextually relevant continuation of the

Click Here For Bundle PDF Course | support@guidely.in Page 8 of 11


Bank Po Mains PDF Course 2024
English Day - 7

paragraph, emphasizing the importance of It highlights the benefits of such education,


addressing the ethical concerns related to AI emphasizing its role in both navigating the
advancements. information landscape and promoting digital
Option A) "fostering collaboration among AI responsibility.
developers and industry stakeholders" is the Therefore, option B is the correct answer.
correct choice because it suggests a proactive
approach to handling AI ethics. It implies that AI 14) Answer: A
developers, along with other key players in the The conjunction ‘while’ indicates that two
industry, should work together to find ethical opposing ideas are being connected. The first
solutions. This aligns with the idea that idea is negative, so, we need a positive
addressing AI's ethical implications is not the sentence in the given blank.
sole responsibility of one group or entity but Sentence A provides a logical continuation of the
requires collective effort. paragraph by highlighting the positive aspect of
Therefore, option A is the correct answer. technology's impact on work-life balance, which
is the potential for greater productivity and
12) Answer: C flexibility in work arrangements due to
The contrasting conjunction ‘however’ suggests technological advancements.
that there should be an opposite idea in the It aligns with the idea that while technology has
blank to that of the previous sentence. Sentence posed challenges to work-life balance, it has
C is the correct choice as it provides a also presented opportunities for improved
meaningful and contextually appropriate working conditions.
continuation of the paragraph, emphasizing the Therefore, option A is the correct answer.
concerns about data privacy and security in the
era of digital transformation. 15) Answer: E
Therefore, option C is the correct answer. Option E provides a meaningful continuation of
the paragraph by emphasizing the importance of
13) Answer: B leadership that encourages innovation and
Sentence B aligns with the central theme of the collaboration while empowering team members.
paragraph, which discusses the importance of It aligns with the idea that effective leadership in
media literacy education in addressing the today's dynamic business landscape goes
challenges posed by information overload and beyond traditional approaches and requires a
misinformation. focus on fostering a culture of creativity and
collaboration.

Click Here For Bundle PDF Course | support@guidely.in Page 9 of 11


Bank Po Mains PDF Course 2024
English Day - 7

Therefore, option E is the correct answer. Thus, the correct sentence is:
In the midst of the grand (A) celebrations at the
16) Answer: E royal palace, there emerged a belief that the
In the given sentence, the words are modernization (B) of the kingdom's infrastructure
appropriately positioned and serve their intended had led to the unexpected prosperity of its
purpose. There are no grammatical or contextual citizens, thus becoming a general consequence
errors that require word swaps. of the ongoing innovations (C) in governance
The word ‘fervent’ means ‘having or displaying a and the economic rejuvenation (D) of the nation.
passionate intensity. ‘Fervent discussions’ Therefore, option C is the correct answer.
appropriately describes the tone and nature of
the conversations at the international summit. 18) Answer: B
The word ‘Devaluation’ means the reduction or The sentence conveys that in the midst of a
underestimation of the worth. challenging and chaotic period marked by
‘Ongoing fluctuations’ represents the continuous geopolitical turmoil, the diplomat's skilful and
and varied changes in the global economy. good negotiation skills played a crucial role in
"Dollar depreciation against multiple major preventing a potential international crisis. His
currencies" This phrase correctly indicates the adept handling of the delicate situation earned
depreciation of the U.S. dollar concerning him widespread credit.
several significant currencies, which contributes The words ‘A-D and B-C’ should be
to the appreciation of the nation's currency. interchanged in order to make the sentence
Therefore, option E is the correct answer. meaningful.
Thus, the correct sentence is:
17) Answer: C Amidst the tumultuous (A) tides of geopolitical
The given sentence means that improvements in upheaval (B), the diplomat's adroit (C)
the kingdom's infrastructure have led to negotiation skills not only averted a potential
unexpected prosperity, and this is a international crisis but also garnered widespread
consequence of ongoing innovations in (D) acclaim for his skilful handling of the delicate
governance and the nation. situation.
The words B and D should be interchanged as Therefore, option B is the correct answer.
"economic rejuvenation" refers to the
revitalization or improvement of the nation's 19) Answer: A
economy. This maintains the sentence's original The sentence discusses how economists,
meaning, so this is the correct option. following the financial crisis, made an effort

Click Here For Bundle PDF Course | support@guidely.in Page 10 of 11


Bank Po Mains PDF Course 2024
English Day - 7

(endeavored) to examine the complex factors The sentence discusses how entrepreneurs face
(intricate web) that led to the market chaos the challenge of managing the intricacies of
(turmoil). They carefully examined how monetary patent law, etc, while simultaneously
policy and regulatory oversight interacted to emphasizing a culture of continuous innovation
create more powerful economic systems in the middle of a fast-changing technological
(resilient economic frameworks). environment (rapidly evolving technological
In order to make the sentence meaningful, B and landscape).
D should be interchanged. In order to make the sentence meaningful, A-C
Thus, the correct sentence is: and B-D should be interchanged.
In the wake of the financial crisis, economists Thus, the correct sentence is:
endeavoured (A) to analyze the intricate (B) web Against the backdrop of a rapidly(A) evolving
of factors that contributed to the market turmoil technological landscape, entrepreneurs (B) find
(C), scrutinizing the interplay of monetary policy, themselves at the crossroads of transformation,
and regulatory oversight in order to formulate where they must navigate (C) the complexities of
more resilient (D) economic frameworks. patent law, and intellectual property rights while
Therefore, option A is the correct answer. striving to cultivate a culture of relentless (D)
innovation.
20) Answer: D Therefore, option D is the correct answer.

Click Here For Bundle PDF Course | support@guidely.in Page 11 of 11


Bank Po Mains PDF Course 2024
Reasoning Day -8 (Eng)

Reasoning Aptitude
Directions (1-5): Study the following information b) E and R; R and P
carefully and answer the given questions. c) R and D; P and B
Eight persons - P, B, R, D, E, K, G and M joined d) P and G; M and D
the school as a teacher in different Months from e) None of these
January to September of the same year. Only
one person joined the school in each month. 2) Which among the following pair of persons
They are teaching different Subjects viz. joined in the month which has an odd number of
Mathematics, English, Tamil, Physics, Chemistry, days?
Biology, History and Accounts. There is only one I. EG
vacant month in which nobody joined the School. II. BM
Only three persons joined the school before K, III. RK
who taught Tamil. R joined three months after IV. BG
the one who taught Tamil. The one who taught a) Only I, II and III
Physics joined the school in the month which has b) Only I, II and IV
the least number of days. Only two persons c) Only II and IV
joined between the one who taught Physics and d) Only II, III and IV
G. M taught Chemistry but neither joined in the e) Only IV
month which has an even number of days nor
joined before May. As many months after M as 3) If all the subject names are arranged in the
before D. Only one person joined between the dictionary order from January to September in
one who taught Biology and B, who did not join the occupied place, then which among the
after the one who taught physics. E joined one of following combination is false with respect to the
the adjacent months of K. The number of new arrangement?
persons joined between B and G is one more a) E – March – Chemistry
than the number of persons joined between G b) R – July – Mathematics
and P. Either E or R taught History. B taught c) G – May – History
neither Mathematics nor Accounts. The one who d) M – August – Physics
taught Accounts did not join in the month which e) All are true
has an even number of days.
1) The number of persons joined between __ and 4) __ joined three months before G and __
__ is two less than the number of persons joined joined three persons after K respectively.
between __ and __ respectively. a) The one who taught Physics and R
a) D and G; M and E b) M and the one who taught Biology

Click Here For Bundle PDF Course | support@guidely.in Page 1 of 10


Bank Po Mains PDF Course 2024
Reasoning Day -8 (Eng)

c) D and the one who taught Chemistry immediately before or immediately after U but
d) E and the one who taught Mathematics doesn’t like Kheer. The number of years between
e) None of these the years in which R and T were promoted is
same as the number of years between the years
5) If M and G did not join due to some reason in which T and U were promoted. Only two
and their subject was additionally taught by K persons were promoted between the one who
and D respectively, then which among the two likes Peda and the one who likes Barfi.
subjects were taught by D with respect to the 6) Who among the following person was
new arrangement? promoted in the prime numbered year?
a) Physics and Accounts a) The one who likes Kheer
b) Physics and Chemistry b) The one who likes Halwa
c) Tamil and Chemistry c) The one who was promoted immediately
d) Tamil and Accounts before T
e) Chemistry and History d) The one who was promoted three persons
before R
Directions (6-10): Study the following information e) None of these
carefully and answer the given questions.
SBI promoted their seven employees viz.- P, Q, 7) Who among the following person was
R, S, T, U, and V during eight different years viz.- promoted in the year, which is exactly divisible
1993, 1996, 1998, 2001, 2005, 2009, 2012, and by 3?
2015. Only one person was promoted in each a) The one who likes Rabri
year and no person was promoted in one of the b) The one who was promoted immediately
given years. Each person likes different sweets before P
viz.- Laddu, Peda, Kheer, Rabri, Kulfi, Barfi, and c) The one who likes Barfi
Halwa. d) Both a and c
P was promoted in 2015 but neither likes Barfi e) Both a and b
nor Kheer. Only three persons were promoted
between P and the one who likes Rabri. Q, who 8) As many years gap between U and the one
likes Laddu, was promoted one of the leap years. who likes Barfi as between __ and __.
The one who likes Halwa was promoted three a) S and the one who likes Laddu
years before Q. T likes Kulfi and was promoted in b) T and the one who likes Kheer
the year whose sum of the digits is a multiple of c) P and the one who likes Kulfi
seven. R was promoted after T but not promoted d) U and the one who likes Halwa
in an odd numbered year. V was promoted either e) V and Q

Click Here For Bundle PDF Course | support@guidely.in Page 2 of 10


Bank Po Mains PDF Course 2024
Reasoning Day -8 (Eng)

9) How many persons were promoted between factor nor a common multiple. For example: If P,
the one who likes Kheer and U? Q and R are sitting together, then the sum of the
a) As many persons promoted between Q and T marks scored by PQ and QR is neither a
b) As many persons promoted between R and V common factor nor a common multiple.
c) Two A sits second to the right of the one who scored
d) Both a and b 24 marks, both are not facing the same direction.
e) Both a and c Two persons sit between A and C, who scored
31 marks. C is facing the centre. E sits third to
10) If all the persons and sweet names are the left of B, where neither of them sits adjacent
arranged in the dictionary order from 1993 to to A. B scored 8 marks less than A. D sits
2015 in the occupied position, then which of the second to the right of B, where both of them are
following persons were promoted after the facing the same direction. C neither sits adjacent
vacant year? to H nor D. H sits second to the right of the one
I.V - Rabri who scored 32 marks. H faces the same
II. R - Kheer direction as A. The persons sitting adjacent to
III. U - Peda the one who scored 32 marks are facing opposite
a) Only II directions. The one who scored 43 marks sits
b) Only I and II third to the left of the one who scored 76 marks.
c) Only I and III G scored less marks than B but doesn’t face
d) Only II and III away from the centre.
e) Only III 11) What is the position of F with respect to the
one who scored 24 marks?
Directions (11-15): Study the following a) Second to the left
information carefully and answer the given b) Third to the left
questions. c) Immediate left
Eight persons A, B, C, D, E, F, G and H are d) Fourth to the right
sitting around a circular table in such a way that e) None of these
some are facing the centre whereas others are
facing away from the centre. Each person scored 12) Who among the following person sits second
different marks in maths subjects viz. 24, 25, 31, to the right of G?
32, 43, 54, 62 and 76. Not more than three a) The one who scored 62 marks
persons sit together facing the same direction. b) The one who sits immediate right of H
Note: The sum of the marks scored by pair of c) A
persons sitting together is neither a common d) The one who scored 76 marks

Click Here For Bundle PDF Course | support@guidely.in Page 3 of 10


Bank Po Mains PDF Course 2024
Reasoning Day -8 (Eng)

e) None of these “A@B” means “A is the daughter of B”.


“A&B” means “B is the father of A”.
13) If the one who scored 24 is related to H, “A%B” means “A is the sister of B”.
similarly the one who scored 54 is related to G, “A#B” means “B is the son of A”.
then who among the following person is related “A$B” means “A is the mother of B”.
to D? “A*B” means “B is the wife of A”.
a) The one who scored 31 marks “A!B” means “A is the brother of B”.
b) The one who scored 62 marks 16) If P@J*U, then how J is related to M and
c) The one who sits second to the left of F how many female members are there in this
d) The one who scored 43 marks family?
e) None of these K%G$L, U&M, U@T%L@V
a) Son-in-law and Five
14) How many persons are sitting between H b) Father-in-law and Six
and the one who scored 25 marks when counted c) Brother-in-law and Five
to the right of H? d) Son-in-law and Six
a) Two e) Father-in-law and Four
b) Three
c) Four 17) What comes in the blank space respectively
d) One such that P is the daughter-in-law of M?
e) No one P_G@T_H!K%Q; H*M_T!L
a)$, %, and #
15) Which of the following statement is/are true b) $, &, and #
with respect to the final arrangement? c) %, % and $
a) C sits immediate left of D d) $, % and &
b) The one who scored 54 marks sits third to the e) #, $ and #
right H
c) F scored 24 marks 18) Which of the following correctly represents B
d) H sits immediate right of B is the brother-in-law of F?
e) E sits opposite to the one who scored 25 a) M#B*L%F@Q
marks b) F@M*Q$L%B
c) A&B*M@Q$F
Directions (16-18): Study the following d) Both a and b
information carefully and answer the given e) Both a and c
questions.

Click Here For Bundle PDF Course | support@guidely.in Page 4 of 10


Bank Po Mains PDF Course 2024
Reasoning Day -8 (Eng)

Directions (19-20): Study the following 19) If all the unknown genders are female, then
information carefully and answer the given which of the following statement(s) is/are true?
questions. a) J+Q
‘A @ B’ means ‘B is the father of A’ b) D$∞M@R
‘A # B’ means ‘A is the mother of B’ c) Z@∞J+K
‘A $ B’ means ‘B is the sister of A’ d) Both a and c
‘A % B’ means ‘A is the brother of B’ e) Both b and c
‘A & B’ means ‘B is the wife of A’
‘A * B’ means ‘A is the husband of B’ 20) If M%∞D#∞F, then how is F related to P?
‘A + B’ means ‘B is the daughter of A’ a) Niece’s husband
‘A ^ B’ means ‘B is the son of A’ b) Nephew’s son-in-law
‘A +∞ B’ means ‘A is the daughter-in-law of B’ c) Nephew’s daughter-in-law
“∞” means in-law d) Sister’s grand-daughter
O#P$K@J; N%M@R*K; Z#Q@P; B@N*D e) Either b or c
Click Here to Get the Detailed Video Solution for the above given Questions
Or Scan the QR Code to Get the Detailed Video Solutions

Answer Key with Explanation


Directions (1-5):
1) Answer: D
2) Answer: B
3) Answer: E
4) Answer: C
5) Answer: A
Final arrangement

Click Here For Bundle PDF Course | support@guidely.in Page 5 of 10


Bank Po Mains PDF Course 2024
Reasoning Day -8 (Eng)

We have,
Again we have,
 Only three persons joined the school
 Only one person joined between the one
before K, who taught Tamil.
who taught Biology and B, who did not
 R joined three months after the one who
join after the one who taught physics.
taught Tamil.
 E joined one of the adjacent months of K.
 The one who taught Physics joined the
 The number of persons joined between B
school in the month which has the least
and G is one more than the number of
number of days.
persons joined between G and P.
 Only two persons joined between the one
 Either E or R taught History. B taught
who taught Physics and G.
neither Mathematics nor Accounts.
 M taught Chemistry but neither joined in
 The one who taught Accounts did not join
the month which has an even number of
in the month which has an even number
days nor joined before May.
of days.
 As many months after M as before D.
From the above condition case-2 gets eliminated
From the above conditions, there are three
because we cannot place E, vacant place and
possibilities
Case-1(a) also eliminated because we cannot
place the one who taught Accounts.
Hence Case-1 shows the final answer.

Click Here For Bundle PDF Course | support@guidely.in Page 6 of 10


Bank Po Mains PDF Course 2024
Reasoning Day -8 (Eng)

We have,
 P was promoted in 2015 but neither likes
Barfi nor Kheer.
 Only three persons were promoted
between P and the one who likes Rabri.
 Q, who likes Laddu, was promoted one of
the leap years.
 The one who likes Halva was promoted
three years before Q.
 T likes Kulfi and was promoted in the year
whose sum of the digits is a multiple of
seven.
From the above conditions, there are four
possibilities

Directions (6-10):
6) Answer: B
7) Answer: D
8) Answer: A
9) Answer: E
10) Answer: C
Final arrangement

Again we have,
 R was promoted after T but in an odd
numbered year.
 V was promoted either immediately
before or immediately after U but doesn’t
like Kheer..
 The number of years between the years
in which R and T were promoted is same

Click Here For Bundle PDF Course | support@guidely.in Page 7 of 10


Bank Po Mains PDF Course 2024
Reasoning Day -8 (Eng)

as the number of years between the years


in which T and U were promoted.
 Only two persons were promoted
between the one who likes Peda and the
one who likes Barfi.
From the above conditions, Case 1 gets
eliminated because we cannot place V and U,
Case 1(a) and Case 2(a) get eliminated because
we cannot place R We have,
Hence, Case-2 shows the final answer.  A sits second to the right of the one who
scored 24 marks, both are not facing the
same direction.
 Two persons sit between A and C, who
scored 31 marks.
 C is facing the centre.

From the above condition, there are four


possibilities

Directions (11-15):
11) Answer: A
12) Answer: D
Again we have,
13) Answer: B
 E sits third to the left of B, where neither
14) Answer: C
of them sits adjacent to A.
15) Answer: E
 B scored 8 marks less than A. (The
Final arrangement
possible outcome of B’s score is either 24
or 54 and A’s score is either 32 or 62)

Click Here For Bundle PDF Course | support@guidely.in Page 8 of 10


Bank Po Mains PDF Course 2024
Reasoning Day -8 (Eng)

 D sits second to the right of B, where both who score 76 marks sits between B and C.
of them are facing the same direction. Similarly for Case-2
 C neither sits adjacent to H nor D. So Case-1 gets eliminated, hence Case-2 shows
 H sits second to the right of the one who the final arrangement
scored 32 marks.
 H faces the same direction as A.
 The persons sitting adjacent to the one
who scored 32 marks are facing opposite
directions.
Here Case-1a gets eliminated because as if D’s
score is 32 then A+D and D+ the one who sits Directions (16-18):
16) Answer: D
immediate left of D have common factors,
similarly for Case-2a

17) Answer: B

Again we have,
 The one who scored 43 marks sits third to
the left of the one who scored 76 marks.
 G scored less marks than B but doesn’t
face away from the centre.
In Case-1 H doesn’t score 76 marks, as if score 18) Answer: E

76, then AH and HE have common factors and if


the person sits between D and B score 76
marks, then D and the one who score 76 marks
and B and the one who score 76 marks have
common factors. So, the only possible is the one
Directions (19-20):

Click Here For Bundle PDF Course | support@guidely.in Page 9 of 10


Bank Po Mains PDF Course 2024
Reasoning Day -8 (Eng)

19) Answer: E
20) Answer: E

Click Here For Bundle PDF Course | support@guidely.in Page 10 of 10


Bank Po Mains PDF Course 2024
Quantitative Aptitude Day -8 (Eng)

Quantitative Aptitude

Directions (1- 4): Study the following information carefully and answer the questions given below.
The given pie chart shows the percentage distribution of the number of five different items [ice cream,
pizza, burger, sandwich, and cake] sold in shop A.

Note –I.The number of eachitems sold in shop B is 75% of the number of respectiveitems sold in shop A.
II. Difference between 50% number of burgers and 25% of the number of sandwiches sold in shop A is
44.
1) Ratio of the number of vanilla and chocolate 2) Out of the total burgers sold in shop A, 55%
ice creams sold in shop A is 3:1, and the ratio of are veg burgers and the rest are non-veg
the number of vanilla and chocolate ice creams burgers. If the number of veg burgers sold in
sold in shop B is 6:5. If average number of shops A and B is the same, then find the
vanilla ice creams sold in shops A, B and C is percentage of non-veg burgers sold in shop B
84, and the average number of chocolate ice out of the total number of burgers and pizzas
creams sold in shop A, B and C is 62, Find the sold in shop B?
total number of ice creams sold in shop C? a) 11.11%
a) 145 b) 19.33%
b) 167 c) 10.25%
c) 130 d)16.66%
d) 152 e) 15.33%
e) 197

Click Here For Bundle PDF Course | support@guidely.in Page 1 of 10


Bank Po Mains PDF Course 2024
Quantitative Aptitude Day -8 (Eng)

3) In shop A, 75% of the veg sandwiches are the number of cakes sold in shop B is m more
sold and the rest are non-veg sandwiches. In than the original number of cake sold in shop B,
shop B, 50% of the veg sandwiches are sold and then the ratio of the number of cake sold in shop
the rest are non-veg sandwiches. The Price of A (original) and B (new) is 28:25. Find the value
the veg sandwich in shops A and B is Rs.60 and of n+m=?
Rs.75 respectively, and the price of the non-veg a) 48
sandwich in shops A and B is Rs.80 and Rs.95 b) 42
respectively. Find the difference between the c) 65
total revenue earned by shops A and B by selling d) 55
sandwiches? e) None of these
a) Rs.124
b) Rs.180 5) Series I and Series II follow the same logic.
c) Rs.158 Find the value of d?
d) Rs.167 Series I – 145, 158, 192, 249, a
e) None of these Series II – a, b, c, d, e
a) 445
4) If the number of cakes sold in shop A is n b) 454
more than the original number of cakes sold in c) 494
shop A, then the ratio of the number of cakes d) 432
sold in shop A (new) and B (original) is 10:7. If e) None of these

Directions (6 -10): Study the following information carefully and answer the questions given below.
The table shows the number of candidates selected in BANK, SSC and Railway in five different, [A, B, C,
D, and E] institutes.

Note- Total number of candidates selected from A and B is 432 and 465 respectively and x:z=3:2.
6) The ratio of the number of candidates selected candidates selected as bank PO from B is 20%
as a bank PO and clerk from A is 5:3. Number of more than the number of candidates selected as

Click Here For Bundle PDF Course | support@guidely.in Page 2 of 10


Bank Po Mains PDF Course 2024
Quantitative Aptitude Day -8 (Eng)

bank PO from A. Find the percentage of number III. Difference between the number of candidates
of candidates selected asbank clerk from A to selected in the bank from D and E is 4.
that from B? a) only I is true
a) 74% b) only III is true
b) 61% c) only I and II is true
c) 75% d) only I and III are false
d) 66% e) None of these
e) 84%
9) Find the value of 2 * (X + Y + Z)
7) 60% and 65% of the candidates selected in a) 70
SSC from C and D are selected in the b) 78
CGL exam, and the rest are selected in the c) 88
CHSL exam. If the average number of d) 90
candidates selected in the SSC CGL exam from e) None of these
C, D and E is 95 then find the number of
candidates selected in the CHSL exam from E, C 10) The ratio of boys and girls selected in railway
and D together? from A is 5:2, and the ratio of boys and girls
a) 241 selected in railway from E is 7:4, If ratio of the
b) 252 number of boys and girls selected in railway from
c) 255 A, E and B together is 3:1, Find the difference
d) 285 between boys and girls selected in railway from
e) 197 B?
a) 152
8) Find which of the following is true? b) 142
I. Difference between the number of candidates c) 165
selected in the railway from A and B is 24. d) 187
II.The sum of candidates selected in all three e) None of these
exams together in E is 624.

Directions (11 -13): Study the following information carefully and answer the questions given below.
The line graph shows the average runs scored by five batsmen after thirty matches.

Click Here For Bundle PDF Course | support@guidely.in Page 3 of 10


Bank Po Mains PDF Course 2024
Quantitative Aptitude Day -8 (Eng)

11) In the next five matches, A scored 35, 42, 5, e) 8


85, and 22 runs, and B scored 112, 88, 2, 8, and
30 runs. If the difference between the average 13) Find the difference between 55% of total
runs of A and B after the 35th match is r, then find runs scored by A and E inthirty matches together
the value of 2r+37.5(upto two decimal)? and 47% of total runs scored by B and D
a) 61.33 together in thirty matches?
b) 68.22 a) 254.3
c) 60.99 b) 234.6
d) 49.32 c) 215.3
e) 49.24 d) 211.6
e) None of these
12) After the 31st match, the average of C is 36,
if in the 31st match C takes, 24 singles, 10 14) Find the sum of the missing number of two
doubles and the rest of the runs are score in the series?
boundary(4’s and 6’s), if he hit two 6's then find Series I – 85,87, 177, 535,?
the number of 4's he hit? Series II – 56,65.8,85.2,114,?
a) 12 a) 2297
b) 14 b) 2456
c) 16 c) 2415
d) 10 d) 2150

Click Here For Bundle PDF Course | support@guidely.in Page 4 of 10


Bank Po Mains PDF Course 2024
Quantitative Aptitude Day -8 (Eng)

e) None of these and dairy milk but not kitkat. The number of
people who like both KitKat and dairy milk but not
15) A and B is the wrong number of the series. milky bar is 38.
Find the relation between A and B? 16) If x is the difference between the total
Series I – 74,198,415,758,1270 number of people like KitKat and dairy milkand y
Series II – 85,88.6,93.2,98.8,104.5 is the difference between the total number of
a) 2A+3B=745 people like dairy milk and milky bar(dairy milk-
b) 2A-B=56 milky bar), Find the value of x*y=?
c)A-B=93.5 a) 2450
d) 3A+7B=745 b) 2410
e) None of these c) 2000
d) 2260
Directions (16 -20): Study the following e) 2750
information carefully and answer the questions
given below. 17) The ratio of the number of males and
A survey conducted on the number of people like females like only dairy milk is 5:4, and the ratio of
KitKat, dairy milk and milky bar chocolate. The the number of males and females like only KitKat
number of people who like only KitKat is four is 3:1. Find the difference between the total
times the number of people who like both dairy number of males who like dairy milk and KitKat
milk and KitKat but not a milky bar. The number together and the total number of females who
of people who like both KitKat and dairy milk but like dairy milk and KitKat together?
not milky bar is 5.55% more than the number of a) 84
people who like dairy milk and milky bar but not b) 82
KitKat. The number of people who like dairy milk c) 83
and milky bar but not kitkat is 5.88% more than d) 88
the number of people who like all three e) 81
chocolates. The number of people who like only
milky bars is double the number of people who 18)
like both KitKat and milky bars but not dairy milk. Quantity I: The number of people who like both
The number of people who like KitKat and milky KitKat and dairy milk but not the milky bar is what
bar but not diary milk is 5.88% less than the percent of the number of people who like both
number of people who like all three. The number KitKat and milky bar but not dairy milk?
of people who like only dairy milk is three times
the number of people who like both milky bars

Click Here For Bundle PDF Course | support@guidely.in Page 5 of 10


Bank Po Mains PDF Course 2024
Quantitative Aptitude Day -8 (Eng)

Quantity II: Total number of people who like dairy c) 67


milk What percent of the total number of people d) 61
like milky bar? e) None of these
a) Quantity: I Quantity: II
b) Quantity: I > Quantity: II 20) The number of people who like only Perk is
c) Quantity: I < Quantity: II 25% more than the number of people who like
d) Quantity: I ≤ Quantity: II only milky bar. The number of people who like
e) Quantity I = Quantity II only 5 stars is 30% more than the number of
people who like only the perk. Find the number
19) Out of the number of people who like only of people like only the 5-star?
dairy milk, 25% like only silk, and out of the total a) 104
number of people who like only KitKat, 25% like b) 125
KitKat Hazelnut. Find the total number of people c) 124
who like the dairy milk silk and KitKat hazelnuts? d) 156
a) 65 e) None of these
b) 62
Click Here to Get the Detailed Video Solution for the above given Questions
Or Scan the QR Code to Get the Detailed Video Solutions

Answer Key with Explanation


Directions (1-4): 5.5x=44
Differencebetween50% of the number of burgers x=44*10/55=8
and 25% of the number of sandwiches sold in So, the number of items sold in shop A is
shop A is 44. 100*8=800.
Let the number of items sold in shop A be 100x. Number of Ice creams sold in shop A = 800 *
So, 20x*50/100-18x*25/100=44 22/100 = 176
10x-4.5x=44

Click Here For Bundle PDF Course | support@guidely.in Page 6 of 10


Bank Po Mains PDF Course 2024
Quantitative Aptitude Day -8 (Eng)

Number of items sold in shop B = 176 * 75/100 = The new number of cakes sold in A is =
132 168*10/7=240
So, 240-224=16=n
The new number of cakes in B is =
224*25/28=200
So, 200-168=32=m
So, n+m=16+32=48

5. Answer: A
1. Answer: C Series I – 145, 158, 192,
The total number of ice creams sold in shop C is 249, a=341
= [84*3-176*3/4-132*6/11] + [62*3-176*1/4- +13*1 +17*2 +19*3 +23*4
132*5/11] Series II – a=341, b=341+13*1=354,
= 48+82=130 c=354+17*2=388, d=388+19*3=445
Directions (6 -10):
2. Answer: D So, 10x+12x+14x=432
The number of veg burgers sold in A is = Or, x=432/36=12
160*55/100=88 10y+9y+12y=465
The number of non-veg burgers sold in A is = Or, 31y=465, y=465/31=15
160-88=72 So, z=12*2/3=8
The number of the veg burger sold in B is = 88
The number of the non-veg burger sold in B is =
120-88=32
So, required percentage = [32/192]*100=16.66%

3. Answer: B
Required difference
6. Answer: C
=[[144*75/100]*60+[144*25/100]*80]-
The number of candidates selected as PO from
[{108*50/100}*75+{108*50/100}*95
A is = 120*5/8=75
= [6480+2880]-[4050+5130]=Rs.180
Number of candidates selected as PO from B is
=75*120/100=90
4. Answer: A
The number of candidates selected as clerks
from A is =120-75=45

Click Here For Bundle PDF Course | support@guidely.in Page 7 of 10


Bank Po Mains PDF Course 2024
Quantitative Aptitude Day -8 (Eng)

The number of candidates selected as clerks So, difference =166-14=152


from B is =150-90=60
Required percent = [45/60]*100=75% 11. Answer: C
The average of A after the 35th match is
7. Answer: D =[30*44+35+42+5+85+22]/35=43.11
The total number of candidates selected in the The average of B after the 35th match is
SSC CHSL exam from E,C and D is =[30*56+112+88+2+8+30]/35=54.85
={220-[95*3-(170*60/100)- So, difference = 54.85-43.11=11.74
(180*65/100)]}+[(170*40/100)]+[(180*35/100)] So, 2r+37.5=2(11.74)+37.5=23.49 + 37.5 =
=154+68+63=285 60.99

8. Answer: B 12. Answer: D


I. Difference between the number of candidates Runs score in 31th match is = 36*31-34*30=96
selected in railway from A and B is = 180- Runs score in 4’s is = 96-24*1-10*2-2*6=40
168=12. The number of 4's he hit =40/4=10
II. The sum of candidates selected in all three
exams together in E is =144+220+220=584. 13. Answer: B
III. Difference between the number of candidates Total runs score by A and E is
selected in the bank from D and E is 144-140=4. =44*30+42*30=2580
Total runs scored by B and D is =
9. Answer: A 28*30+56*30=2520
2*(12 + 15 + 8) = 70 So, 55*2580/100 – 47*2520/100 = 234.6

10. Answer: A 14. Answer: A


The total number of boys selected from A, E and
B is
= [168+180+220]*3/4=568*3/4=426
The total number of girls selected from A, E and
B is = 568*1/4=142 So, the required sum = 2145+152=2297
Number of boys selected in B = 426-168*5 /7-
220*7/11=166 15. Answer: C
The number of girls selected in B is =142-
168*2/7-220*4/11=14

Click Here For Bundle PDF Course | support@guidely.in Page 8 of 10


Bank Po Mains PDF Course 2024
Quantitative Aptitude Day -8 (Eng)

Directions (16-20):
Let the number of people who like all three be
17x.
So, the number of people who like both dairy
milk and milky bar but not kitkat is
17x*105.88/100=18x
The number of people who like both dairy milk 16. Answer: C
and KitKat but not milky bar is = So, x=152+38+34+32-108-38-36-34=40
18x*105.55/100=19x Y=108+38+36+34-64-32-34-36=50
The number of people who like both KitKat and So, x*y=50*40=2000
milky bar but not diary milk is =17x*[100-
5.88]/100=16x 17. Answer: D
So, 19x=38, x=2 So, required difference = [108*5/9]+[152*3/4]-
The number of people who like all three is [108*4/9]-[152*1/4]=174-48-38=88
17x=34
So, the number of people who like both dairy 18. Answer: C
milk and milky bar but not kitkat =18*2=36 Quantity I
The number of people who like both dairy milk Required percentage = [38/32]*100=118.75%
and KitKat but not milky bar = 38 Quantity II
The number of people who like both KitKat and Required
milky bar but not diary milk is =16x=32 percentage=[108+38+34+36]*100/[64+32+34+36
The number of people who like only KitKat = ] =130.12%
38*4=152 Quantity I < Quantity II
The number of people who like only dairy milk =
36*3=108 19. Answer: A
The number of people who like only milky bar = The Total number of people who like silk and
32*2=64 Hazelnuts is
=152*25/100+108*25/100=65

20. Answer: A

Click Here For Bundle PDF Course | support@guidely.in Page 9 of 10


Bank Po Mains PDF Course 2024
Quantitative Aptitude Day -8 (Eng)

Number of people who like only 5star = 64*(125/100)*(130/100)=104

Click Here For Bundle PDF Course | support@guidely.in Page 10 of 10


Bank Po Mains PDF Course 2024
English Day - 8

English Language
Directions (1-5): The questions below carry a Investors ___________ whether they want to
paragraph from which a part has been deleted. invest or not.
You must read the same carefully and choose an (a) can survive downturns in the economy
option that correctly fits the given paragraph to (b) value and trust the user over rapid
complete the same in the most appropriate development
manner. (c) analyse the types of start-ups and then
1) Monsoon is here and so is the season for decide
numerous hair problems. From frizz to fall, the (d) grow quickly and draw substantial
change in weather and increase in humidity investments
brings with it a hair-related crisis of sorts. Hairfall (e) None of these
during monsoon may also be caused by
hormonal changes triggered by high 3) Layoffs are on the rise and with it, people are
temperatures and sun exposure during summer. also becoming victims of various kinds of job
Dust, sweat and oil may ________ haircare scams. Recently in Delhi, the police arrested
products leading to residue build-up that leads to impostors ________ a job in the Airports Authority
more hair fall. At times, increased humidity can of India. In another case, a Maharashtra woman
also dry the scalp leaving the follicles weak and lost Rs 15 lakh in a work-from-home job scam.
deprived of nutrients. And mind you, these scams are not just
(a) avoid hair fall in during the rainy season restricted to India.
(b) also accumulate on the scalp along with (a) created various fake job postings for
(c) absorb excess water from your hair Singapore
(d) maintain scalp health during monsoon (b) know the times are tough and the job market
(e) None of these is brutal
(c) talking to people from the same field
2) It might sound strange today, but tech start- (d) who duped people on the pretext of providing
ups were called just "start-ups" initially. (e) None of these
Sometimes venture investors referred to them as
"portfolio companies", while customers called 4) Twitter users have asserted that they are able
them "tech companies", but "start-up" remained to access the old TweetDeck along with the free
to be the basic term. In 2013, start-ups began API access. This free API made__________.
their Animorphs journey, and that continues. Days ago, Twitter introduced rate-limits and
Several animal names are used as terms and restrictions on tweets, along with killing the
phrases in the start-up world on a regular basis. legacy APIs that facilitated the above-mentioned

Click Here For Bundle PDF Course | support@guidely.in Page 1 of 9


Bank Po Mains PDF Course 2024
English Day - 8

feature. Access to third-party apps, meanwhile, 6) These channels offer (A) engage opportunities
was banned back in January. for brands to (B) unprecedented with their
(a) accessing third-party apps possible customers in real-time, (C) deliver personalised
(b) the decision to limit the number of tweets content, and (D) neglect meaningful interactions.
(c) to do anything to encourage those verticals (a) A-D; suppress
(d) that the new Tweet Deck would have been (b) A-B; foster
accessible (c) B-C; create
(e) None of these (d) B-D; multiple
(e) No swapping needed
5) In today’s fast-paced digital landscape,
technology has emerged as a driving force of 7) In today’s fast- (A) regulated digital landscape,
transformation across various industries. One technology has (B) transformation as a driving
sector that has experienced a seismic shift is the force of (C) emerged across various (D)
advertising industry, where technological industries.
advancements have _____ their target audience. (a) B-D; transformed
Technology is changing the advertising (b) A-C; growing
landscape, empowering brands to create (c) B-C; paced
impactful and personalised experiences for their (d) A-D; companies
customers. (e) No swapping needed
(a) spark curiosity, and leave a lasting
impression 8) State-owned Bank of India is (A) exploring the
(b) revolutionised advertising (B) possibility of share sale to investors over the
(c) has experienced a remarkable revolution next one year to meet the (C) minimum public
(d) reshaped the way brands connect with holding (D) requirement of 25 per cent.
(e) None of these (a) A-D; maximum
(b) B-C; discovering
Directions (6-10): The questions given below (c) A-B; target
carry highlighted words which might be placed (d) B-D; business
incorrectly. One of these words would be (e) No swapping needed
inappropriate in the context and need to be
replaced. You must choose an option that carries 9) Suzlon is (A) committed to partner with an
the pair of words that should be swapped and increasing number of Indian industries, (B)
the word that should replace the inappropriate driving them toward their net‐zero (C) sum while
word as well. powering the nation with (D) targets energy.

Click Here For Bundle PDF Course | support@guidely.in Page 2 of 9


Bank Po Mains PDF Course 2024
English Day - 8

(a) C-D; sustainable because basic human nature has ________


(b) A-B; slated changed.
(c) B-C; pushing (a) Happened, hardly
(d) B-D; energy (b) Came about, not much
(e) No swapping needed (c) Materialised, bare
(d) Hit upon, slightly
10) Demonetisation (A) continue confidence in (e) None of these
the rupee, (B) especially in Bhutan and Nepal,
and both countries (C) shook to fear additional 13) While Assam is the most _________ example,
(D) relevant policy changes. several states in the Northeast have witnessed a
(a) B-C; refer significant ________ from across the border.
(b) B-D; specially (a) Serious, migrants
(c) A-C; sudden (b) Obvious, influx
(d) A-B; ensured (c) Evident, exodus
(e) No swapping needed (d) Apparent, arrival
(e) Both (c) and (d)
Directions (11-15): The questions given below
carry sentences in which words from the options 14) This fact is _________ since about 96% of
given below them can be filled to complete the China’s container trade with Europe went
same grammatically and meaningfully. You must ________ sea routes.
choose the most appropriate option for each (a) Importance, across
sentence as your answer. (b) Venerable, haywire
11) Our regulators have had a record of ______ (c) Crucial, via
and poor oversight over unlisted companies’ (d) Paramount, though
_________ startups. (e) None of these
(a) Negligible, of
(b) Meticulous, preferring 15) A _________ search operation was set in
(c) Careful, excluding motion and hours later the vessel was found
(d) Lax, including _________ with deceased individuals.
(e) Careless, counted as (a) Evacuation, mended
(b) Massive, imploded
12) What __________ nearly 4,000 years ago in (c) Tremendous, splintered
the court of the Kauravas is still happening (d) Vast, burnt
(e) Immense, sunk

Click Here For Bundle PDF Course | support@guidely.in Page 3 of 9


Bank Po Mains PDF Course 2024
English Day - 8

(c) Only III


Directions (16-20): The questions given below (d) Only I
carry tables in which a sentence has been given (e) Both II and III
in the first column. A part of the sentence has
been deleted. You are required to choose an 18)
option from the second column that can Column I Column II
complete the given sentence meaningfully and When you make I. what’s
grammatically. organisations safe for heartbreaking is the
16) women, derision
Column I Column II ________________ for II. you make them
Knowledge centres I. by hire and fire, everybody. safer
_____________________ of stick and carrot III. it is the job of the
productivity and II. do not operate IC to identify those
accountability. by tactics of the (a) Only III
market (b) Both I and II
III. call for rigorous (c) Only I
standards (d) Both I and III
(a) Only I (e) Only II
(b) Only II
(c) Only III 19)
(d) Both I and III Column I Column II
(e) None matches Flipkart customers I. will now be able to
_______________ get II. has entered a
17) personal loans from Axis partnership to
Column I Column II Bank. III. will also provide
Simply having the largest I. is not enough in customers
labour force an (a) Only I
___________________ II. has never been (b) Both II and III
environment of fast- enough in (c) Only II
changing production III. will not be (d) Both I and II
processes. considered an (e) None of these
(a) Only II 20)
(b) Both I and III Column I Column II

Click Here For Bundle PDF Course | support@guidely.in Page 4 of 9


Bank Po Mains PDF Course 2024
English Day - 8

For 56%, a I. familiar with or the (a) Only I


recommendation from brands (b) Only II
friends and II. consumers (c) Only III
________________ for indicated that (d) Both I and II
purchasing health III. family was the (e) None
insurance. main trigger
Click Here to Get the Detailed Video Solution for the above given Questions
Or Scan the QR Code to Get the Detailed Video Solutions

Answer Key with Explanation


1) Answer: B the investors properly analyse before they
The sentence with the blank talks about dust, invest…
sweat etc that would harm the hair obviously so, The first option mentions a new aspect and does
the first option that says ‘avoid hair fall’ will go not match the discussion so it can be ruled out.
against the context. The second option makes The second option looks incomplete. The third
sense as it says these things might get collected option completes the idea that we discussed
on the scalp (leading to residue build-up)… The above. Why would investors draw investments?
third option is irrelevant as these things do not This makes the fourth option irrelevant.
absorb water. The fourth option; like the first one The best would be to mark option (c) as the
is also irrelevant. answer.
This makes option (b) the most logical answer
choice. 3) Answer: D
The first and the second options do not continue
2) Answer: C the sentence grammatically so these can be
The part of the sentence after the blank says; ruled out.
‘whether they want to invest or not’ which means

Click Here For Bundle PDF Course | support@guidely.in Page 5 of 9


Bank Po Mains PDF Course 2024
English Day - 8

The third option says people from the same field sentence is continued with their target
which makes it irrelevant as nothing of the sort audience…
has been discussed. The best would be to mark option (d) as the
The fourth option continues well as imposters answer.
are people who pretend to be someone else so
they can dupe that is cheat or trick people. 6) Answer: B
This makes option (d) the best choice. The word engage does not fit well before
opportunities and we need an adjective in its
4) Answer: A place. What kind of opportunities?
The latter part of the paragraph tells us that the Unprecedented opportunities means those that
access to third-party apps was banned in have been seen for the first time and were been
January which now makes the first option presented with ever before…This tells us that
logically support the idea that the API would the words in A and B need to be swapped.
make accessing the third-party apps possible. Also, the word neglect has incorrectly been used
The second option talks about limiting the because the sentence intends to promote or
number of tweets which has again been talked nurture meaningful interactions and not neglect
about in the next line making it repetitive and them. So, the word foster would best replace the
redundant. (c) is unrelated and vague so can be same.
ruled out. Same logic eliminates the fourth option This makes option (b) the most appropriate
as well. answer choice.
The best would hence, be to mark option (a) as
the answer. 7) Answer: C
The word regulated in the given sentence neither
5) Answer: D fits in A nor anywhere else in the sentence. This
The first option is incorrect as have has been clarifies that this word needs to be replaced. The
used before the blank and spark begins the other words highlighted in the sentence cannot
option making it incorrect grammatically. Have be used after the word fast which makes it
should be followed by the third form of the verb. necessary for us to choose a suitable word from
The second option is incomplete and would not the options. Fast-paced makes a correct phrase
fit in the sentence. The third option goes wrong as it means moving or developing very quickly…
for using the verb has (which would go wrong This is a hint that the words in B and C should
immediately after have). The fourth option swapped as well for the sentence to be correct.
makes sense as; it ends in with and the

Click Here For Bundle PDF Course | support@guidely.in Page 6 of 9


Bank Po Mains PDF Course 2024
English Day - 8

Therefore, the best would be to mark option (c) The word relevant is positive but such policy
as the answer. changes would not create fear. So, this word is
contextually incorrect.
8) Answer: E The best would be to mark option (c) as the
The given sentence uses all the words correctly answer.
and no swapping or rearrangement is needed.
This makes option (e) the most logical answer 11) Answer: D
choice. The word negligible means very small and
therefore not important. This can be taken as
9) Answer: A correct but the second word of makes no sense
The first two words seem correct as given. in the context. The word meticulous is same as
The word sum does not fit in the sentence at any careful but both cannot fill the first blank as they
place so it should be changed. will go contextually incorrect. The choice for the
The word targets should come after net-zero second blank in the last option makes it
which means the words in C and D should be incorrect.
interchanged for the sentence to make sense. Lax means not having high standards; not strict.
Sustainable means involving the use of natural The words given in option (d) fill the sentence
products and energy in a way that does not harm correctly.
the environment. This word fits best before the
word energy. 12) Answer: A
So, the correct option should be (a). The first blank can take the word happened as
the sentence itself gives a hint by using the
10) Answer: C phrase ‘still happening’ in the later part of the
The word continue doesn’t fit well after the sentence.
singular subject Demonetisation so it needs to The phrase came about also means happened
be swapped for sure… The sentence talks about but the choice for the second blank does not fit
fear (of countries in relation to the currency) in.
which would shake the confidence of these Materialised means to become real; to happen…
countries. So, the word shook should come in bare means uncovered. Both these words make
place of A. This tells us that A and C must be no sense in the given sentence.
interchanged. Hit upon means discover or think of something,
especially by chance… This word cancels the
fourth option as well.

Click Here For Bundle PDF Course | support@guidely.in Page 7 of 9


Bank Po Mains PDF Course 2024
English Day - 8

The only option carrying both the correct words carries the word search after the first blank
is (a). which would make the given word redundant.
Mended means to repair something that is
13) Answer: B damaged or broken… imploded means
The first option can be ruled out as the word collapsed. This word describes the state of the
migrants does not fit in grammatically. vessel mentioned in the sentence. Tremendous
The word obvious means easily seen or is very large or great; very good but this carries a
understood; clear. Influx means large numbers positive tone making it unfit for the sentence
of people or things arriving suddenly… (this fits given above. Splintered means to split or rend
as it describes people coming from across the into long thin pieces.
border). Exodus is the opposite of influx as it Of all the given options, the pair of words that
means the mass departure of people. Apparent make the sentence grammatically and
means the same as obvious or evident but meaningfully correct is (b).
arrival is not fit here. It is best used with respect
to railway stations or airports etc. 16) Answer: C
The only pair of words that fits correctly can be The sentence does not take the first option
found in option (b). correctly as it is a misfit in the given context. If
we take the second option, the preposition ‘of’
14) Answer: C will make it an incorrect continuation.
The first blank can take the words in the third The third option is correct as it completes the
and the fourth options only and not the others. idea that knowledge centres make rigorous
The word venerable means highly respectable… standards of productivity necessary.
The word haywire means erratic; out of control… Therefore, the correct answer is option (c).
important could have been a better choice
grammatically for choosing the first option. The 17) Answer: D
fourth option gets eliminated for the usage of the The sentence starts with the word simply which
word though… means used to emphasize how easy or basic
The best combination of words that can be found something is. This means that having the largest
in the options hence, is (c). labour force is just a basic thing and much more
is needed.
15) Answer: B The first option completes the above meaning
Evacuation means the removal of persons or contextually. Also, it uses an which will be
things from an endangered area. The sentence

Click Here For Bundle PDF Course | support@guidely.in Page 8 of 9


Bank Po Mains PDF Course 2024
English Day - 8

correct before environment. The absence of the This makes the fist option the best answer
article makes the second option incorrect. choice.
The third option is incorrect as it makes the
sentence contextually incorrect. 19) Answer: A
This makes option (d) the most logical answer The only option that fits the blank grammatically
choice. is the first one. The second option uses has
incorrectly for the plural subject customers which
18) Answer: E makes it a misfit. The third option does not fit in
The first option mentions the word derision which grammatically.
means the situation in which someone or This makes option (a) the best answer choice.
something is laughed at and considered stupid
or of no value… making organisations safe will 20) Answer: C
certainly not create any such situation. The third option continues the given sentence
The second option fits perfectly because it most appropriately by completing the phrase
continues the idea of organisations becoming ‘…friends and family…’
safe for women and everyone else. The other two options can be eliminated for
The third option brings in the IC which is a new being unrelated and grammatically incorrect.
perspective as per the sentence so it can be Thus, the correct answer is option (c).
cancelled.

Click Here For Bundle PDF Course | support@guidely.in Page 9 of 9


Bank Po Mains PDF Course 2024
Reasoning Day -9 (Eng)

Reasoning Aptitude
Directions (1-5): Study the following information Assistant Manager. U neither likes Biba nor Allen
carefully and answer the below questions. Solly. The number of persons junior to U is one
Seven persons – P, Q, R, S, T, U, and V are less than the number of persons senior to the
working at nine different designations viz.- one who likes H&M. The one who likes Allen
Managing Director (MD), General Manager (GM), Solly is not immediately junior position to P. Only
Additional General Manager (AGM), Chief two persons are designated between T and the
Executive Officer (CEO), Sales Officer (SO), one who likes Allen Solly. Neither T nor Q likes
Manager, Assistant Manager (AM), Clerk, and Biba.
Personal Assistant (PA). Each person likes 1) Who among the following person likes Mufti?
different cloth brands viz.- Levis, Mufti, H&M, a) R
Zara, Allen Solly, Biba, and Raymond. All the b) The one who is immediately senior to V
information is not necessary in the same order. c) The one who is designated as Manager
Note: I. There is no person designated at two d) U
positions but neither at Managing Director nor e) None of these
Personal Assistant.
II. No two consecutive positions are vacant. 2) How many persons are designated between R
III. The managing director is the seniormost and the one who likes Biba?
designated person whereas the personal a) As many persons senior to the one who likes
assistant is the juniormost designated person. H&M
IV. Each person can be terminated by any higher b) Three
designated person but not immediately higher c) As many persons junior to the one who likes
designated person. Levi’s
The one who likes Zara is the Manager. Only two d) Four
persons are designated between the one who e) Either A or C
likes Zara and V, who likes Raymond. V is
neither designated as Managing Director nor 3) Who among the following person is
Personal Assistant. R is neither designated as designated as Sales officer?
Additional General Manager nor Chief executive a) Q
officer. R is terminated by the one who likes b) The one who likes H&M
Mufti, who is not designated as the General c) The one who likes Allen Solly
Manager. The one who likes Biba is four d) V
positions junior to R. P, who likes H&M, who can e) None of these
terminate U. Neither U nor T is designated as an

Click Here For Bundle PDF Course | support@guidely.in Page 1 of 11


Bank Po Mains PDF Course 2024
Reasoning Day -9 (Eng)

4) Who among the following is designated two Train P is parked second to the right of the one
persons junior to R? whose length is 41m but none of them is parked
a) T at the extreme ends of the yard. Train A whose
b) The one who is immediately senior to S width is 12m and is parked third from the left end.
c) The one who likes Zara Train A is parked second to the left of the train
d) V whose width is 13m. The width of train S is 14m
e) None of these and parked adjacent to train P. Only one train is
parked between train S and the train whose
5) Which of the following statement is/are not width is 16m. The length of train S is equal to the
false? square of the difference between the widths of
a) R is designated immediately senior to the one train Q and C. The length of train C is thrice the
who likes Biba width of train R, whose length is 25m. Only one
b) The one who likes Raymond is designated as train is parked between train R and the one
General Manager whose length is 35m. Neither R nor E is parked
c) Two persons are designated between T and at the ends. Train C is parked third to the right of
the one who likes H&M the train whose length is 45m. Only one train is
d) U is designated as Manager parked between the one whose width is 15m and
e) None of the above statements are true train E, whose length is 12m less than the length
of train S. The width of train T is 9m more than
Directions (6-10): Study the following information the width of train Q, whose length is 10m less
carefully and answer the below questions. than the length of train B. Train D is parked
Ten trains of different lengths and different immediate right of the train whose length is 48m.
widths are parked from west to east in a yard of The width of train D is nine meters more than the
width 77m in such a way that in row1 – P, Q, R, width of train P. The width of train T is not an odd
S, and T are parked facing south whereas in number and the length of train T is 14m more
row2 – A, B, C, D, and E are parked facing north. than the length of train D. The width of train B is
The width gap between two adjacent trains is one meter less than the width of train E. The
4m. length of train P is 1m less than twice the width
Note: I. The length of each train is neither less of train E, whose length is 8m less than the
than 20m nor more than 50m. length of the train which is parked immediate left
II. The width of each train is neither less than 4m of train C.
nor more than 18m. 6) Width of which of the following train is16m?
III. There is no space left at the left and right end a) T
of the yard. b) The one whose length is 24m

Click Here For Bundle PDF Course | support@guidely.in Page 2 of 11


Bank Po Mains PDF Course 2024
Reasoning Day -9 (Eng)

c) Q c) 58m
d) The one whose length is 25m d) 65m
e) None of these e) None of these

7) What is the position of train A with respect to Directions (11-15) Two machine rearranges a
the one whose length is 48m? given word and number step by step to a final
a) Third to the right output following same rule. Study the following
b) Immediate left information carefully and answer the below
c) Second to the right questions.
d) Fourth to the left Machine 1:
e) None of these Input: Social 372 473 Open Under 564 Network
385
8) Length of which of the following train is 27m? Step I:
a) The train which parked is second to the right _(A)__(B)__(C)__(D)__(E)__(F)__(G)__(H)_
of A Step II: bhknrz 654 djmnqsv 732 473 Open
b) E Under 385
c) The one whose width is 18m Step III:
d) A _(A)__(B)__(C)__(D)__(E)__(F)__(G)__(H)_
e) None of these Step IV: vsofe 374 qpof 583 bhknrz 654 djmnqsv
732
9) Four of the following five are alike in a certain Machine 2:
way based on the given arrangement and thus Input: 381 Travel 468 Magic 624 Above Entry
form a group. Which one of the following does 531
not belong to the group? Step I: bfhlz 648 381 Travel 624 Above Entry 531
a) The one whose length is 36m Step II:
b) The one whose width is 4m _(A)__(B)__(C)__(D)__(E)__(F)__(G)__(H)_
c) The one which is parked immediate right of C Step III: wpfcb 183 dkqsuz 264 bfhlz 648 Entry
d) B 531
e) The one whose length is 35m Step IV:
_(A)__(B)__(C)__(D)__(E)__(F)__(G)__(H)_
10) What is the sum of the lengths of trains E 11) What comes in place of B in step III of
and T? machine 1?
a) 75m a) qpof
b) 72m b) 583

Click Here For Bundle PDF Course | support@guidely.in Page 3 of 11


Bank Po Mains PDF Course 2024
Reasoning Day -9 (Eng)

c) djmnqsv d) 372
d) 473 e) None of these
e) None of these
Directions (16-20) Study the following
12) What comes in place of D in step I of information carefully and answer the below
machine 1? questions.
a) 732 In a zoo different animal cages are kept at
b) Social different points. V is north of T, which is 13m
c) 473 northeast of Q. The distance between RX is
d) Under same as the distance between VT. The distance
e) None of these between UX is 4m more than the distance
between CR. X is 12m north of U. R is to the east
13) What comes in place of C in step IV of of C and west of X.S is 5m southeast of C. Z is
machine 2? 12m north of P, which is 18m west of Q. Z is
a) wpfcb west of T. S is 10m west of V, which is 8m north
b) 264 of T.
c) bfmsuw 16) What is the position of C with respect to Z?
d) 183 a) North
e) None of these b) Southwest
c) West
14) What comes in place of G in step II of d) Northeast
machine 2? e) None of these
a) bfhlz
b) 531 17) What is the position of T with respect to U?
c) Magic a) West
d) Entry b) Northeast
e) Above c) Southwest
d) South
15) What is the difference between the second e) None of these
highest number in step IV of machine 2 and the
second lowest number in step III of machine 1? 18) If point A is east of point Q and south of point
a) 319 T, then what is the distance between Q and A?
b) 275 a) 5m
c) 285 b) 2m

Click Here For Bundle PDF Course | support@guidely.in Page 4 of 11


Bank Po Mains PDF Course 2024
Reasoning Day -9 (Eng)

c) 7m d) PT
d) 6m e) QZ
e) None of these
20) If point Y is 12m north of Q, then what is the
19) Four of the following five are alike in a certain position and distance of point Y with respect to
way based on the directions of the points in the Z?
given arrangement and thus form a group. Which a) Southeast,12m
one of the following doesn’t belong to the group? b) East, 18m
a) QC c) North, 20m
b) UR d) Northeast, 8m
c) SC e) None of these
Click Here to Get the Detailed Video Solution for the above given Questions
Or Scan the QR Code to Get the Detailed Video Solutions

Answer Key with Explanation


Directions (1-5):
1) Answer: B
2) Answer: A
3) Answer: C
4) Answer: B
5) Answer: B

Click Here For Bundle PDF Course | support@guidely.in Page 5 of 11


Bank Po Mains PDF Course 2024
Reasoning Day -9 (Eng)

We have:
 The one who likes Zara is the Manager.
 Only two persons are designated between
the one who likes Zara and V, who likes
Raymond.
 V is neither designated as Managing
Director nor Personal Assistant.
That means, in case (1) V is designated
as an assistant general manager, in case
(2) V is designated as the general
Manager.
 R is neither designated as Additional
General Manager nor Chief executive
officer.
 The one who likes Biba is four positions
junior to R.
 R is terminated by the one who likes
Mufti, who is not designated as the
General Manager.
Since, one person can be terminated by Again, we have:
higher designated person but not  P, who likes H&M, who can terminate U.
immediately high rank.  Neither U nor T is designated as an
Thus, R can’t be a General Manager. Assistant Manager.
That means, in case (1) & case (2a) the  U neither likes Biba nor Allen Solly.
one who likes Mufti is designated as the  The number of persons junior to U is one
Managing director, in case (2) the one less than the number of persons senior to
who likes Mufti is designated as the the one who likes H&M.
additional general manager. That means, in case (1) & case (2b) P is
Based on the above given information we have: designated as the Chief executive officer,
in case (2a) P is designated as an
additional general manager, case (2) is
not valid.
Based on the above given information we have:

Click Here For Bundle PDF Course | support@guidely.in Page 6 of 11


Bank Po Mains PDF Course 2024
Reasoning Day -9 (Eng)

Case (2) is not valid as the number of persons Case (1) & case (2b) are not valid as the one
junior to U is one less than the number of who likes Allen Solly is not immediately junior
persons senior to the one who likes H&M. position to P
Again, we have: Parallel arrangement
 The one who likes Allen Solly is not
immediately junior position to P. Directions (6-10):
 Only two persons are designated between 6) Answer: D
T and the one who likes Allen Solly. 7) Answer: B
 Neither T nor Q likes Biba. 8) Answer: A
That means, in case (2a) T is designated 9) Answer: B
as the Managing Director, case (1) & case 10) Answer: D
(2b) are not valid.
Based on the above given information we have:

Click Here For Bundle PDF Course | support@guidely.in Page 7 of 11


Bank Po Mains PDF Course 2024
Reasoning Day -9 (Eng)

 The length of train C is thrice the width of


train R, whose length is 25m.
Since, the length of none of the trains is
less than 20m not more than 50m, thus
only possible length of train S is 36m or
49m.
We have:
 Only one train is parked between train R
 Train P is parked second to the right of
and the one whose length is 35m.
the one whose length is 41m but none of
 The length of train C is thrice the width of
them is parked at the extreme ends of the
train R, whose length is 25m.
yard.
 Neither R nor E is parked at the end.
 Train A whose width is 12m and is parked
 Train C is parked third to the right of the
third from the left end.
train whose length is 45m.
 Train A is parked second to the left of the
That means, in case (1) C is parked at the
train whose width is 13m.
right end of row2, in case (1a) C is parked
 The width of train S is 14m and parked
second from the right end of row2, case
adjacent to train P.
(2) & case (3) are not valid.
 Only one train is parked between train S
Based on the above given information we have:
and the train whose width is 16m.
That means, in case (1) S is parked
immediate right of P, in case (2) & case
(3) S is parked immediate left of P.
Based on the above given information we have:

Case (2) & case (3) are not valid as only one
train is parked between train R and the one
whose length is 35m and R’s length is 25m.
Again, we have:
 Only one train is parked between the one
Again, we have: whose width is 15m and train E, whose
 The length of train S is equal to the length is 12m less than the length of train
square of the difference between the S.
widths of train Q and C.

Click Here For Bundle PDF Course | support@guidely.in Page 8 of 11


Bank Po Mains PDF Course 2024
Reasoning Day -9 (Eng)

 The width of train T is 9m more than the  The length of train P is 1m less than twice
with of train Q, whose length is 10m less the width of train E, whose length is 8m
than the length of train B. less than the length of the train which is
 Train D is parked immediate right of the parked immediate left of train C.
train whose length is 48m. That means, the length of train P is 21m,
That means, case (1) is not valid. and the length of train A is 32m.
 The width of train D is nine meters more Based on the above given information we have:
than the width of train P.
That means, width of the train P must be
4m.
 The width of train T is not an odd number
and the length of train T is 14m more than
the length of train D.
Directions (11-15):
So length of train D becomes 27.
11) Answer: B
Since The length of train S is equal to the
12) Answer: C
square of the difference between the
13) Answer: A
widths of train Q and C.
14) Answer: D
Length of S becomes 36 and width of Q
15) Answer: A
becomes 9.
We have:
Based on the above given information we have:
Words are arranged in such a way that words
starting with consonants are rearranged first
followed by words starts with vowels in
alphabetical order.
For words: If the word starts with a vowel then,
change all letters with its immediate next letter.
Else, if the word starts with a consonant then,
change all letters with its immediate preceding
Case (1) is not valid as Train D is parked letter. Also, after changing letters, letters of the
immediate right of the train whose length is 48m. word start with vowel are changed in reverse
Again, we have: alphabetical order, and the word starts with
 The width of train B is one meter less than consonants are rearranged in alphabetical order.
the width of train E.

Click Here For Bundle PDF Course | support@guidely.in Page 9 of 11


Bank Po Mains PDF Course 2024
Reasoning Day -9 (Eng)

Numbers are arranged in such a way that even 17) Answer: C


numbers are arranged first followed by odd 18) Answer: A
numbers in ascending order. 19) Answer: D (In all pairs first point is at south
For number: If a number is odd, then east of second point except option d)
interchange the first and the last digit, else if a 20) Answer: B
number is even then interchange the first and We have:
the second digit. Z is 12m north of P, which is 18m west of Q. Z is
Based on the above given information we have: west of T.
Machine 1: S is 10m west of V, which is 8m north of T.
Input: Social 372 473 Open Under 564 Network
385
Step I: djmnqsv 732 Social 473 Open Under 564
385
Step II: bhknrz 654djmnqsv 732 473 Open
Under 385
Step III: qpof 583 bhknrz 654djmnqsv 732 473
Under
Step IV: vsofe 374 qpof 583 bhknrz 654 djmnqsv
Again, we have:
732
V is north of T, which is 13m northeast of Q. S is
Machine 2:
5m southeast of C.
Input: 381 Travel 468 Magic 624 Above Entry
531
Step I: bfhlz 648 381 Travel 624 Above Entry
531
Step II: dkqsuz 264 bfhlz 648 381 Above Entry
531
Step III: wpfcb 183 dkqsuz 264 bfhlz 648 Entry
531
Step IV: zusof 135 wpfcb 183 dkqsuz 264 bfhlz
648

Directions (16-20): Again, we have:


16) Answer: D

Click Here For Bundle PDF Course | support@guidely.in Page 10 of 11


Bank Po Mains PDF Course 2024
Reasoning Day -9 (Eng)

The distance between RX is same as the


distance between VT. The distance between UX
is 4m more than the distance between CR.R is
to the east of C and west of X. X is 12m north of
U.
Based on the above given information we have:

Click Here For Bundle PDF Course | support@guidely.in Page 11 of 11


Bank Po Mains PDF Course 2024
Quantitative Aptitude Day -9 (Eng)

Quantitative Aptitude

Directions (01 - 04): Study the following c) only III true


information carefully and answer the questions d) only I and II true
given below. e) all false
Length of train A is 12x m and speed of train A is
10y km/hr. When train A is not moving, then train 3) If train B cross _______ m long bridge in 25
C cross train A in 34.66 sec. Train A cross train B sec then trains C cross same bridge in
in 9.33 sec when both trains moving in opposite _______sec.
direction. Length of train B is 18z. Train C cross Find which of the following the satisfy the blank.
a 240 m long bridge in 34.66 sec. Train A cross a a) 340, 48.5
Pole in 9.6 sec. Ratio of speed of train A and b) 320,40
train B is 5:4. Ratio of length of train A and train c) 300, 42.5
C is 6:7. d) 360, 40.5
1) Find which one of the following is true. e) None of these
I. Train A cross a 120 m long bridge in 12.4 sec.
II. Train B cross a 260 m long bridge in 20.6 sec. 4) If value of k equal to the time taken by train A
III. Train C cross a 190 m long platform in 43.33 and C to cross each other when moving in same
sec. direction and j equal to the time taken by train B
a) All true and C to cross each other when moving in same
b) II and III true direction.
c) All false Find the correct relation between numerical
d) only II false value of x, k , y and j.
e) Only III false a) k>x, x*y>k+j
b) j>y, x*y<k+j
2) What is the possible value of difference of c) k>y, x*y<k+j
time taken by train A and train B to cross 28z m d) k>x, x>k+j
long bridge and 12x m long platform e) None of these
respectively?
I. x/100 5) Number of people like coffee in complex A is
II. z/50 20 more than the number of people like tea in B.
III. 2y/3 Number of people like coffee in C is 20% more
a) only I true than the number of people like cold drinks in
b) only II and III true same. Number of people like cold drinks in C is

Click Here For Bundle PDF Course | support@guidely.in Page 1 of 11


Bank Po Mains PDF Course 2024
Quantitative Aptitude Day -9 (Eng)

20% more than the number of people like tea in Quantity I: Total number people in A?
B. Number of people like cold drinks in C is 30. Quantity II: Total number of people in B?
Number of people like tea in C is 25. Number of Quantity III: Total number of people in C?
people like coffee in A is 15 more than number of a) Quantity I>Quantity II>Quantity III
people like coffee in B. Number of people like b) Quantity I>Quantity II<Quantity III
cold drinks is same in A and B which is 5 more c) Quantity I>Quantity II= Quantity III
than number of people like tea in B. Number of d) Quantity I>Quantity II>Quantity III
people like Tea in A is 7 more than the number e) Quantity I =Quantity II =Quantity III
of people like cold drinks in A.

Directions (06 - 09): Study the following information carefully and answer the questions given below.
The pie chart shows the percentage distribution of number of days taken by different combination of
persons.

Note – I. Total work is 240 units.


II. Ratio of efficiency of D and E is 3:4.
III. Value of total pie chart is 132.
IV. Ratio of efficiency of B and F is 17:13.
6) Find the correct relation with value of number III. B, C and F together.
of days taken by below combination. a) I=14.77>II=12.48>III=11.03
I. A, B and C together. b) I=17.55>II=15.84>III=12.25
II. A, D and E together. c) I=17.77>II=15.48>III=12.97

Click Here For Bundle PDF Course | support@guidely.in Page 2 of 11


Bank Po Mains PDF Course 2024
Quantitative Aptitude Day -9 (Eng)

d) I=16.97>II=13.48>III=10 d) The data given in both statements I and II


e) I=17.77>II=13.48>III=12 together are not sufficient to answer the question
e) The data given in both statements I and II
7) A and E together start the work, after ______ together are necessary to answer the question.
days they left the work and B and D joined the
work and after ______ days B and D left the 9)
work and A finished the work. If the total work is Quantity I: A and D started the work and after 5
finished in ______ days then find which of the days they left, rest of the work is finished by E
following combinations satisfy the blanks? alone. In how many days, they will the whole
a) 15,6,29 work?
b) 18,6,33 Quantity II: In how many days B and C together
c) 19,12,36 complete the work when B working 20% less
d) 14,6,33.33 efficient?
e) 11,11,33 a) Quantity: I Quantity: II
b) Quantity: I > Quantity: II
8) Find the number of days taken by P to c) Quantity: I < Quantity: II
complete the work alone. d) Quantity: I ≤ Quantity: II
Statement I: P and Q together complete the e) Quantity I = Quantity II
work in 20 days. Q is more efficient than D but
less efficient than E. 10) x,y and z are three numbers which are less
Statement II: Difference of efficiency of P and D than 10 and not equal to zero and also x>y<z.
is 1. Ratio of efficiency of C and P is 7:10. (ii) 4xy=15z.
a) The data in statement I alone is sufficient to (iii) z is the perfect square number and also
answer the question, while the data in statement square root of a even number.
II alone is not sufficient to answer the question Quantity I: Find the value of 3x+4y.
b) The data in statement II alone is sufficient to Quantity II: Find the value 6z-2y=?
answer the question, while the data in statement Quantity III: Find the value of 3x+7y-3z=?
I alone is not sufficient to answer the question a) Quantity I>Quantity II>Quantity III
c) The data either in statement I alone or in b) Quantity I>Quantity II<Quantity III
statement II alone is sufficient to answer the c) Quantity I>Quantity II= Quantity III
question d) Quantity I>Quantity II>Quantity III
e) Quantity I =Quantity II =Quantity III

Directions (11 - 14): Study the following information carefully and answer the questions given below.

Click Here For Bundle PDF Course | support@guidely.in Page 3 of 11


Bank Po Mains PDF Course 2024
Quantitative Aptitude Day -9 (Eng)

The table given below shows the number of people of five cities using public (bus and train) and private
(bike and car) transports.

Note – I) Total number of people using public transport from A and B is 210 and difference between the
total number of people using public transport from B and E is 60. x is 50% more than z.
II) Public transport of E > Public transport of B
11) Ratio of number people using bus and train 12) Difference of number of male and female
out of total number of people using public [Male>female] using public transport in city C is z
transport in city A is 5:3 and ratio of number of and the difference of number of male and female
people using bike and car out of total number of [female>male] using private transport in city C is
people using private transport in A is 4:3. y-10. Then, difference of total male and total
Number people using bus in city B is [z/2]% less female using transport in C is equal to
than the same in A and number of people using I. x-z
car in city B is y% less than the same in city A. II. [x+y]-35
Find the difference between the number of III. 2*[3y-2z]
people using train in city B and number of people a) Both I and II
using bike in city B? b) All three
a) y+3 c) Only I
b) x+3 d) both I and III
c) y-4 e) only III
d) z-4
e) y+2 13) Number of people using public transport in F
is ______ more than the number of people using

Click Here For Bundle PDF Course | support@guidely.in Page 4 of 11


Bank Po Mains PDF Course 2024
Quantitative Aptitude Day -9 (Eng)

public transport in E and number of people using c) 36.36 years


private transport in F is ______ less than the d) 39.27 years
number of people using private transport in D. e) None of these
Total number of people using transport in F is
______. 15) Quantity I: Ratio of speed of boat in still
Find the correct options which satisfy the blank? water and upstream speed is 5:4. Boat cover 48
a) 10,20,220 km downstream in two hours. In n hours, boat
b) y/2,z+8,7y-12 covers 64km upstream and 96km downstream.
c) x-3,z+8,5x-1 Find the value of n.
d) z/2,y,7z Quantity II: m2-10m+24=0. Find the value of m.
e) More than one option is true. Quantity III: Average of downstream and
upstream speed is 20 km/hr. When speed of
14) Average age of number of people using stream is double then boat Can cover 72 km
private transport in D and E is 32 years and 48 downstream in 3 hours. If O is the time taken by
years respectively and average age of number of boat to cover 216 km upstream(with actual
people using public transport in D and E is 28 speed of stream). Find the value of O.
and 36 years respectively. Find the average age a) Quantity I>Quantity II>Quantity III
of all the people using transport in D and E b) Quantity I>Quantity II<Quantity III
together? c)Quantity I>Quantity II= Quantity III
a) 35.29 years d)Quantity I>Quantity II>Quantity III
b) 27.27 years e) Quantity I =Quantity II =Quantity III

Directions (16 - 18): Study the following information carefully and answer the questions given below.
The line graph shows the number of red, green and blue balls in five different bags.

Click Here For Bundle PDF Course | support@guidely.in Page 5 of 11


Bank Po Mains PDF Course 2024
Quantitative Aptitude Day -9 (Eng)

16) Quantity I: Find the difference of probability III. Probability of getting two red balls from bag A
of getting two red balls from bag B and is 1/3.
probability of getting 2 green balls from bag C? a) all true
Quantity II: Find difference of probability of b) only I true
getting three red balls from bag C and probability c) only I false
of getting 2 green balls from bag E? d) all false
a) Quantity: I Quantity: II e) only II false
b) Quantity: I > Quantity: II
c) Quantity: I < Quantity: II 18) x is probability of getting 2 blue balls from
d) Quantity: I ≤ Quantity: II bag B and y is probability of getting 3 blue balls
e) Quantity I = Quantity II from bag C. Find the relation between x and y.
a) x=y
17) Find which one of the following is true. b) x>y
I. Probability of getting three blue balls from bag c) x<y
E is 7/139. d) x>2y
II. Probability of getting two green balls from bag e) None of these
D is 14/97.

Directions (19-20): Study the following information carefully and answer the questions given below.
Bar graph shows the number of right questions and number of wrong questions of four students in an
exam of math and English sections.

Click Here For Bundle PDF Course | support@guidely.in Page 6 of 11


Bank Po Mains PDF Course 2024
Quantitative Aptitude Day -9 (Eng)

Note – I. Total number of questions in Math and English section is 35 and 30 respectively. II. For each
right answer in math a student gets 4 marks and 2 marks deducted for wrong answer.
III. For each right answer in English a student c) only I true
gets 3 marks and 1 mark deducted for wrong d) all true
answer. e) None of these
19) Find which of the following is true.
I. Total marks scored by A and B in Math section 20) Find which of the following is false?
is 150. a) Total marks of B and C together is 311.
II. Difference between the marks of math and b) Total negative marks of B is 19.
English section by student C is 38. c) Difference of math marks of A and B is 8.
III. Total marks of D is 143. d) Total marks in English of C and D is 136
a) All false e) None of these
b) only I false

Click Here For Bundle PDF Course | support@guidely.in Page 7 of 11


Bank Po Mains PDF Course 2024
Quantitative Aptitude Day -9 (Eng)

Click Here to Get the Detailed Video Solution for the above given Questions
Or Scan the QR Code to Get the Detailed Video Solutions

Answer Key with Explanation


Directions (01 - 04): I. Train A cross a 120 m long bridge in
In 34.66 sec train C cross a 240 long bridge and [240+120]/25=14.4 sec.
also cross train A when train A is not moving. II. Train B cross a 260 m long bridge in
Therefore, length of train A = 12x=240, x=20 [260+180]/20=22 sec.
Further, 240/(10y*5/18)=9.6 III. Train C cross a 190 m long platform in
Or, y=[240*18/50*9.6]=9 [190+280]/15=31.33 sec.
So, speed of train A is 90 km/hr = 25 m/sec
Speed of train B is 25*4/5=20 m/sec 2) Answer: D
Length of train C is 240*7/6=280 Required time taken by A = [28*10+240]/25=20.8
So, 240+18z=9.33*45 sec
Note : 9.33 = 6 + 3.33 Required time taken by B =[12*20+180]/20=21
= 6 + (10/3) sec
=28/3 So, difference is 21-20.8=0.2, x/100 and z/50
240 + 18z = 28/3 * 45
Or, z=180/18=10 3) Answer: B
Length of train B (18z) = 180m By checking option B, we see,
Speed of train C is = [240+280]/34.66 B cross 320 m long bridge in 25 sec.
Here, 34.66 can be written as = 28+6.66 = [320+180]/20=25
28+(20/3) =104/3 Therefore, time taken by train C to cross 320 m
Therefore, Speed of train C = 520/(104/3) = long bridge is [320+280]/15=40 sec
15m/s
1) Answer: C 4) Answer: A

Click Here For Bundle PDF Course | support@guidely.in Page 8 of 11


Bank Po Mains PDF Course 2024
Quantitative Aptitude Day -9 (Eng)

k= [240+280]/(25-15)=52 Efficiency of D is 240/40=6


j= [180+280] / (20-15) =92 Efficiency of E is 6*4/3=8
So, correct relation from given option is k>x, Efficiency of F is 8.5*13/17=6.5
x*y>k+j 6) Answer: C
I. A, B and C together complete =
5) Answer: B [240/(3.5+8.5+1.5)]=17.77
Number of people like cold drinks in C is 30. II. A, D and E together complete =
Number of people like tea in B is 30*100/120=25 [240/(1.5+6+8)]=15.48
Number of people like coffee in C is = III. B, C and F together complete
30*120/100=36 =[240/(8.5+3.5+6.5)]=12.97
Total number of people in C is 30+25+36=91 So, I=17.77>II=15.48>III=12.97
Coffee in A is 25+20=45
Coffee in B is 45-15=30 7) Answer: D
Cold drinks in B = A = 25+5=30 Let check option D,
Tea in A = 30+7=37 [8+1.5]*14+[8.5+6]*6+[1.5]*[33.33-20]=240
Quantity I: total number of people in A is So, it is satisfied.
37+30+45=112
Quantity II: total number of people in B is 8) Answer: B
30+25+30=85 From statement I,
Quantity III: 91 We cannot calculate the number of days taken
Quantity I >Quantity II <Quantity III by P to complete the work because no direct
relation is given.
Directions (06 - 09): From statement II,
A and B together complete the work in Ratio of efficiency of number of days taken by C
132*18.18/100=24 days and P given.
Similarly, B and C together, A and C together So, from that we can calculate the number of
and D alone complete the work in 20,48 and 40 days taken by P.
days respectively. So, efficiency of P 3.5*10/7=5
Efficiency of A+B, B+C and A+C is 10,12, and 5 So, number of days taken by P = 240/5=48
respectively.
So, efficiency of C [27/2]-10=3.5 9) Answer: B
Similarly, efficiency of B and A is 8.5 and 1.5 Quantity I
respectively.

Click Here For Bundle PDF Course | support@guidely.in Page 9 of 11


Bank Po Mains PDF Course 2024
Quantitative Aptitude Day -9 (Eng)

Time taken to complete the whole work =[240- =90-60=30


5*(1.5+6)]/8+5=30.3125 days Number of people using bike in city B is =[108-
Quantity II [140*3/7]*70/100]
Time taken to complete the whole work = =66
[240/[(8.5*80/100)+3.5] =240/10.3=23.30 So, difference is = 66-30 = 36 =Z-4
Quantity I > Quantity II
12) Answer: D
10) Answer: B Total male in city C is
So, we can say z is 4 because only 4 is perfect = [200+40]/2 + [160-20]/2=120+70=190
square and square root of even number (i.e 16) Total female in city C is = [200-40]/2 +
below 10. [160+20]/2
So, xy=15*4/4=15 =80+90=170
So, xy=15 So, difference is 190-170=20=x-z=2*[3y-2z]
And value of x and y must be 5 and 3
respectively because x>y. 13) Answer: E
Quantity I: 3x+4y=3*5+4*3=27 If we check option C and option D values ,both
Quantity II: 6*4-2*3=18 are true.
Quantity III: 3*5+7*3-3*4=24 150+[(x-3)=57]+140-[(z+8)=48] =[5x-1]=299
Quantity I >Quantity II <Quantity III And 150+[20=z/2] +140-[y=30]=280

Directions (11 - 14): 14) Answer: A


So, 2x+3y=210 and 2y+1.5x-3y=60 The average age of all the people using
By solving above equations, we get, x=60 and transport in D and E together =
y=30 [32*140+48*120+28*160+36*150]/[160+150+140
+120]
=20120/570=35.29

15) Answer: B
Quantity I:
So, speed of boat is 5x and upstream speed is
4x. so, speed of stream is x. 48/2=24=5x+x=6x
11) Answer: D Or, x=24/6=4
Number of people using train in city B is =[90- Therefore, n =[64/(20-4)]+[96/(20+4)]=4+4=8
[120*5/8]*[80/100]]

Click Here For Bundle PDF Course | support@guidely.in Page 10 of 11


Bank Po Mains PDF Course 2024
Quantitative Aptitude Day -9 (Eng)

Quantity II: So, x=5C2/14C2=10/91=0.109


m2-10m+24=0 So, y=6C3/18C3=20/816=0.024
or, m2-6m-4m+24=0 So, B is true
or, (m-6)(m-4)=0, m=6,4
Quantity III: Directions (19-20):
Average of downstream and upstream speed is
20 = speed of boat in still water.
So, original speed of stream is = [[72/3]-
20]/2=[24-20]/2=2
Therefore, O = 216/18=12
Quantity I >Quantity II <Quantity III

19) Answer: A
16) Answer: C
Quantity I I. Total marks scored by A and B in Math section

Required difference =[5C2/14C2] – [6C2/18C2] is 154.


II. Difference of marks of math and English
= [10/91]-[15/153]=0.0118
section C of is 39.
Quantity II
III. Total marks of D is 145.
Required difference = [4C2/17C2]-[6C3/18C3] =
[6/136]-[20/816]=0.0196
20) Answer: C
Quantity I < Quantity II
So, option C is false.
Difference of math marks of A and B is 6.
17) Answer: D
I. Probability of getting three blue balls from bag
E is =35/680=7/136
II. Probability of getting two green balls from bag
D is =28/190=14/95.
III. Probability of getting two red balls from bag A
is= 6/36=1/6.

18) Answer: B

Click Here For Bundle PDF Course | support@guidely.in Page 11 of 11


Bank Po Mains PDF Course 2024
English Day - 9

English Language
Directions (1-5): The following paragraph given pensioners will receive, should their applications
below has five blanks in which words from be accepted. Their _______(C) is understandable
options given in each question would be filled in as they — particularly those still in service —
order to complete the paragraph meaningfully. give their consent to transfer a substantial portion
You must read the same carefully and choose of their PF savings to the Pension Fund. Though
the correct answer for each question. pensioners will have to make payments
The move by the Employees’ Provident Fund separately to be considered eligible for higher
Organisation (EPFO) to extend the deadline, for pension, they would also be keen to know how
a third time, for employees and pensioners to much pension they would get. It is against this
exercise the joint option for higher pension is of backdrop that the EPFO has sent demand letters
limited relief, as several issues to about 1,000 pensioners and employees for
remain__________(A). As the entire process is an collecting arrears. As the calculator on the
outcome of the Supreme Court judgement of EPFO’s website only indicates the amount to be
November 2022 on the validity of amendments transferred, it is time that the EPFO gave an
made in 2014 to the Employees’ Pension indicative figure of pension, at least to applicants
Scheme (EPS), the matter ________(B) to two who have been issued the letters. In the case of
categories of applicants — those who retired the pre-2014 retirees, it appears that the PF
from service prior to September 1, 2014, and authorities have not yet officially commenced
those who left service after the date and those communication on the status of their
who are still in service. Nearly 16 lakh applications, even though, under the rules, most
applications have been received so far. Yet of these may not stand the test of________(D).
another extension cannot be ruled out. After all, it Employers have the _________(E)task of
is too early to indicate that higher pension producing physical records for every applicant.
disbursal will begin this financial year, even if As not all establishments will have these records,
applicants meet all the terms and conditions. The the sensible option for the EPFO would be to
Union Labour and Employment Ministry, which share its database with employers for the limited
oversees the EPFO, told the standing purpose of establishing applicant authenticity.
parliamentary committee concerned early this The situation is more complicated for
year that the implications of the Court’s verdict establishments that are no longer in existence —
on the EPS had not been factored in, while there appears to be no way out for their
preparing the Budget estimates for 2023-24.The employees and pensioners to apply for higher
most important issue of concern is the lack of pension. Given that the spirit of the judgement is
clarity on the amount pension members and to provide a better social security net, the Union

Click Here For Bundle PDF Course | support@guidely.in Page 1 of 11


Bank Po Mains PDF Course 2024
English Day - 9

Ministry and the EPFO should be proactive in b) study


simplifying the process and ensuring that every c) scrutiny
deserving person gets the benefit. d) inspect
1) Which of the following words fits in ‘blank A’ e) None of these
given in the passage and the same word should
fill in the blank given in the sentence below ? 5) Which of the following words fits in ‘blank E’
The system issue still remains ________(A) and given in the passage and the antonym of the
that’s why I was not able to complete my task on word fills the blank in the given sentence below ?
time. She does all the household chores in an ___way.
a) addressed a) simple
b) solved b) herculean
c) unresolved c) easiest
d) fixed d) effortless
e) None of these e) None of these

2) Which of the following words fits in ‘blank B’ Directions (6-10) : In each question a sentence is
given in the passage ? given followed by a blank. You are provided with
a) concerning three fragments I, II and III. You have to identify
b) disconnects which statement/statements can carry forward
c) parts the given sentence in the most logical way so as
d) pertains to make the sentence coherent and contextually
e) None of these correct.
6) CNG typically costs less than gasoline or
3) Which of the following words fits in ‘blank C’ diesel and is often more economically viable for
given in the passage ? fleet operators or individuals with CNG-powered
a) anxiety vehicles, as _____.
b) happiness I. CNG stations are present in every nook and
c) joy corner.
d) anxious II. it can result in significant fuel cost savings
e) None of these over time.
III. gasoline and diesel are dangerous.
4) Which of the following words fits in ‘blank D’ a) Only I
given in the passage ? b) Only II
a) attention c) Only III

Click Here For Bundle PDF Course | support@guidely.in Page 2 of 11


Bank Po Mains PDF Course 2024
English Day - 9

d) Both I and II 9) In order to resolve all the client issues that


e) Both II and III followed the code production this week all the
testing and development leads_______.
7) Data science is a multidisciplinary field that I. agreed to work at night and extend their
combines statistical analysis, machine learning, support during the weekends too.
data visualisation, and domain II. came up with a solution to automate the
expertise____________and unstructured data. handling of frequently reported issues so that the
I. to combine structured team can concertare on new and difficult ones.
II. to store various data in the form of structured III. went on a two week long vacation to escape
III. to extract meaningful insights and knowledge from the workload.
from structured a) Only I
a) Only I b) Only II
b) Only II c) Both I and II
c) Only III d) Both II and III
d) Both I and II e) All I, II and III
e) Both I and III
10) ___________the visit was the MoU on India
8) ___________the three service chiefs who importing 10,000 MW of electricity from Nepal in
continue to operate in their respective domains, 10 years and exporting Nepali electricity to
but the role and function of the CDS is still in the Bangladesh via the Indian territory.
making.
I. Even Though they belong to three different I. Of the various agreements signed, the one that
forces there must be a coordination among was projected as the highlight of
II. The huge arms and ammunition of the country II. Many important decisions were made during
must be shared by the Nepal’s PM visit to India but the most
III. The Chief of Defence Staff, who functions as significant thing about
Principal Military Adviser to the Defence Minister, III. Nepal’s Prime Minister shared on his official
is expected to work in tandem with page the importance of
a) Only I a) Only I
b) Only II b) Only III
c) Both I and II c) Both I and II
d) Only III d) Both I and III
e) Both I and III e) Both II and III

Click Here For Bundle PDF Course | support@guidely.in Page 3 of 11


Bank Po Mains PDF Course 2024
English Day - 9

Directions (11-15) : Rearrange the following b) B


sentences in the proper sequence to form a c) C
meaningful paragraph and then answer the d) D
questions given below. Sentence E is the fifth e) F
sentence of the coherent paragraph and is fixed.
Rearrange the remaining sentences (A), (B), (C), 12) Fill in the blanks with correct alphabets in
(D) and (F). accordance with the rearranged sequence.
Note : E is the fifth sentence after the CF AB BD F_
rearrangement and is fixed. CD AE F_
(A) Red sanders is a native and endemic to a) A, E
India and can only be found in the Southern parts b) B, D
of the Eastern Ghats. c) C, F
(B) There is a huge demand for this timber in the d) D, A
domestic and international market, especially in e) E, B
East Asian Countries.
(C) The Andhra Pradesh forest department 13) Which of the following immediately follows
represented by Principal Chief Conservator of sentence ‘E’ in the coherent paragraph formed ?
Forest & Head of Forest Force have submitted a) A
an application seeking geographical indication b) B
(GI) tag for the famous Andhra Pradesh Red c) C
Sanders. d) D
(D) In addition, the timber is also exploited for the e) F
extraction of Santalin (a red pigment used as dye
and colourant in food), medicine and cosmetics. 14) Which of the following is the ODD ONE OUT.
(E) Red Sanders timber is used for carving a) CAB
furniture, poles, and house posts and the rare b) BEA
“wavy” grain variant is highly valued in Japan for c) FBD
its acoustic properties and is used to make d) AFE
musical instruments. e) EDA
(F) It is a small tree that grows to 5-8 metres in
height with a trunk 50-150 cm diameter. 15) Which of the following is an adjoining pair
11) Which of the following is the introductory after the rearrangement ?
sentence after the rearrangement ? a) AB
a) A b) BE

Click Here For Bundle PDF Course | support@guidely.in Page 4 of 11


Bank Po Mains PDF Course 2024
English Day - 9

c) CD meeting that the administration had scrapped


d) DF “obsolete land rules” in J&K.
e) EA a) bloggers, keep
b) influencers, dispose
Directions (16-20) : In each of the questions c) infamous, preserve
below a statement is given in which two words d) unpopular, accept
have been highlighted. Below each question five e) None of these
options are given each containing a pair of
words. Choose the pair which contains the 19) Despite desperate attempts by interested
synonyms of the words respectively. parties to paint the expert committee’s report as
16) Nearly a decade after the Big Three’s hijack a ‘clean chit’, all the actions taken by SEBI
bid, the ICC has proposed a similar surplus indicate an admission of guilt and a belated
revenue-sharing model. attempt to increase transparency regarding
a) control, excess financial flows.
b) give, deficit a) satisfied, early
c) defence, equal b) cheerful, premature
d) free, less c) hopeless, delayed
e) None of these d) composed, overdue
e) None of these
17) Any non-compliance of the applicable
regulation shall be dealt with strictly and may 20) A local court in Imphal on Friday took
also invite stringent enforcement action,” read cognizance of charges filed against two Kuki-
the advisory issued by the DGCA. Zomi leaders, accusing them of promoting enmity
a) disagree, lenient between two groups.
b) denial, easy a) ignorance, love
c) rebellion, flexible b) knowledgeable, hostility
d) abide, rigorous c) negligence, friendly
e) None of these d) heedlessness, grudge
18) The L-G of Jammu and Kashmir has e) None of these
informed the legal luminaries who attended the

Click Here For Bundle PDF Course | support@guidely.in Page 5 of 11


Bank Po Mains PDF Course 2024
English Day - 9

Click Here to Get the Detailed Video Solution for the above given Questions
Or Scan the QR Code to Get the Detailed Video Solutions

Answer Key with Explanation


1) Answer: C Concerning can be eliminated for it being a -ing
The issues remain unresolved and thus the word which is not suitable to the sentence.
EPFO board has extended the deadline. Parts and disconnects have different meanings
Addressed, solved and fixed are the opposite in that vary from the context of the sentence and
meaning to the word what the blank needs. hence cannot fit the blank.
Resolve - to fix something The sentence:
Unresolve - to not fix something As the entire process is an outcome of the
The sentences : Supreme Court judgement of November 2022 on
The move by the Employees’ Provident Fund the validity of amendments made in 2014 to the
Organisation (EPFO) to extend the deadline, for Employees’ Pension Scheme (EPS), the matter
a third time, for employees and pensioners to pertains to two categories of applicants — those
exercise the joint option for higher pension is of who retired from service prior to September 1,
limited relief, as several issues remain 2014, and those who left service after the date
unresolved. and those who are still in service.
The system issue still remains unresolved and
that is why I was not able to complete my task 3) Answer: A
on time. Clearly there is confusion and lack of clarity
regarding the pension numbers - this is the
2) Answer: D context of the previous sentence to the sentence
Pertains is the correct word that fits in the given which holds the blank. And hence the following
blank ‘B’ and makes the sentence meaningful. sentence must also follow the same tone and
Pertains - applies to

Click Here For Bundle PDF Course | support@guidely.in Page 6 of 11


Bank Po Mains PDF Course 2024
English Day - 9

context, from the given options only ‘anxiety’ is Herculean - task which needs great efforts and
the right word that can fit the blank properly. strength.
Anxiety - worry/fear The sentences :
Joy and happiness are opposite in meaning to Employers have the herculean task of producing
anxiety physical records for every applicant.
The sentence : She does all the household chores in an
Their anxiety is understandable as they — effortless way.
particularly those still in service — give their 6) Answer: B
consent to transfer a substantial portion of their The context here is the cost of gasoline, diesel
PF savings to the Pension Fund. and CNG, CNG is more affordable than the other
two. So, the right fragment to complete the
4) Answer: C sentence would be II.
Among the given words ‘Scrutiny’ is the correct I - talks about CNG station which is inappropriate
word. to the context and similarly II talks about the
Scrutiny - watching something closely/examining effects of usage which also seems inappropriate.
Inspect, attention are words with similar meaning The sentence:
but scrutiny is a better word that fits the blank. CNG typically costs less than gasoline or diesel
Study is inappropriate to the context. and is often more economically viable for fleet
The authorities haven’t communicated to the operators or individuals with CNG-powered
2014 retirees however this has not been a part vehicles, as it can result in significant fuel cost
of the test of scrutiny. savings over time.
The sentence: In the case of the pre-2014
retirees, it appears that the PF authorities have 7) Answer: C
not yet officially commenced communication on Statement III is the correct pick to complete the
the status of their applications, even though, sentence in a meaningful way.
under the rules, most of these may not stand the To combine structured and unstructured data -
test of scrutiny. data when organised in different forms is always
easy to use and there is no need to combine the
5) Answer: B data back and make it difficult
Producing physical record of every applicants is Data science does not deal with storage so
really hard and tiring so the right word would be statement II is also wrong.
‘herculean’ The sentence:

Click Here For Bundle PDF Course | support@guidely.in Page 7 of 11


Bank Po Mains PDF Course 2024
English Day - 9

Data science is a multidisciplinary field that The sentences :


combines statistical analysis, machine learning, In order to resolve all the client issues that
data visualisation, and domain expertise to followed the code production this week all the
extract meaningful insights and knowledge from testing and development leads agreed to work at
structured and unstructured data. night and extend their support during the
weekends too.
8) Answer: D In order to resolve all the client issues that
followed the code production this week all the
III is the correct statement that can be used to testing and development leads came up with a
complete the given sentence. solution to automate the handling of frequently
The second half of the sentence talks about a reported issues so that the team can concertare
single officer(the CDS), so I and II can be on new and difficult ones.
eliminated because they talk about arms,
ammunition and coordination and about a 10) Answer: C
person/officer. Both the statements I and II are correct and fit
Only III describes the role of CDS-Chief Defence the blank in a perfect way adding meaning to the
Officer and hence it is the correct fragment that sentence.
fits the given blank. III is inappropriate and incomplete making the
The sentence : sentence incorrect.
The Chief of Defence Staff, who functions as The sentences:
Principal Military Adviser to the Defence Of the various agreements signed, the one that
Minister, is expected to work in tandem with the was projected as the highlight of the visit was the
three service chiefs who continue to operate in MoU on India importing 10,000 MW of electricity
their respective domains, but the role and from Nepal in 10 years and exporting Nepali
function of the CDS is still in the making. electricity to Bangladesh via the Indian territory.
Many important decisions were made during the
9) Answer: C Nepal’s PM visit to India but the most significant
Both the statements I and II fit the blank to make thing about the visit was the MoU on India
the sentence correct and meaningful. importing 10,000 MW of electricity from Nepal in
Statement III is inappropriate to the context of 10 years and exporting Nepali electricity to
the sentence because to solve the production Bangladesh via the Indian territory.
issues the employees usually work and do not
go on a vacation. 11) Answer: C

Click Here For Bundle PDF Course | support@guidely.in Page 8 of 11


Bank Po Mains PDF Course 2024
English Day - 9

C is the first/introductory sentence after the sanders’. Now what is red sanders ? and the
rearrangement. answer is sentence ‘A’ and it is obvious that it
The rearranged sequence is CAFBED. will be the second sentence in the
Sentence ‘C’ is the perfect introduction to this rearrangement. Sentence ‘F’ tells more about
passage as it contains the information ‘AP the specifications of the red sanders, so place it
government has applied for a GI tag for its Red as the third sentence. Now, look at the fixed
sanders’. Now what is red sanders ? and the sentence ‘E’ which talks about the
answer is sentence ‘A’ and it is obvious that it usage/applications of red sanders and ‘D’ has
will be the second sentence in the the phrase ‘in addition’ and continues to discuss
rearrangement. Sentence ‘F’ tells more about red sander’s usage. So, ED is a pair. The left out
the specifications of the red sanders, so place it sentence is ‘B’ which sits right after CAF and
as the third sentence. Now, look at the fixed before ED. The final arrangement is CAFBED.
sentence ‘E’ which talks about the
usage/applications of red sanders and ‘D’ has 13) Answer: D
the phrase ‘in addition’ and continues to discuss ED is a pair, they revolve around the same
red sander’s usage. So, ED is a pair. The left out context.
sentence is ‘B’ which sits right after CAF and The rearranged sequence is CAFBED.
before ED. The final arrangement is CAFBED. Sentence ‘C’ is the perfect introduction to this
passage as it contains the information ‘AP
12) Answer: E government has applied for a GI tag for its Red
CF AB BD - F is two places away from A, B is sanders’. Now what is red sanders ? and the
two places away from A similarly D is two places answer is sentence ‘A’ and it is obvious that it
away from B. will be the second sentence in the
According to the pattern followed, E is two rearrangement. Sentence ‘F’ tells more about
places away from F. So the first blank should be the specifications of the red sanders, so place it
filled with ‘E’ as the third sentence. Now, look at the fixed
Second - CD first and last, AE- second and fifth, sentence ‘E’ which talks about the
FB- third and fourth usage/applications of red sanders and ‘D’ has
‘B’ fills the second blank. the phrase ‘in addition’ and continues to discuss
The rearranged sequence is CAFBED. red sander’s usage. So, ED is a pair. The left out
Sentence ‘C’ is the perfect introduction to this sentence is ‘B’ which sits right after CAF and
passage as it contains the information ‘AP before ED. The final arrangement is CAFBED.
government has applied for a GI tag for its Red

Click Here For Bundle PDF Course | support@guidely.in Page 9 of 11


Bank Po Mains PDF Course 2024
English Day - 9

14) Answer: B rearrangement. Sentence ‘F’ tells more about


CA is a pair in the arrangement and B is two the specifications of the red sanders, so place it
places away from A. FB - pair and D is two as the third sentence. Now, look at the fixed
places away from B. A Similar pattern is followed sentence ‘E’ which talks about the
in AFE and EDA. usage/applications of red sanders and ‘D’ has
But in BEA the pattern is missed - rather BEC the phrase ‘in addition’ and continues to discuss
would be correct. red sander’s usage. So, ED is a pair. The left out
The rearranged sequence is CAFBED. sentence is ‘B’ which sits right after CAF and
Sentence ‘C’ is the perfect introduction to this before ED. The final arrangement is CAFBED.
passage as it contains the information ‘AP
government has applied for a GI tag for its Red 16) Answer: A
sanders’. Now what is red sanders ? and the The words in option a are the correct synonyms
answer is sentence ‘A’ and it is obvious that it of the words hijack and surplus respectively.
will be the second sentence in the Hijack - to take control of a plane
rearrangement. Sentence ‘F’ tells more about Surplus - something that is in excess amount
the specifications of the red sanders, so place it Control and excess is the right pair of synonyms
as the third sentence. Now, look at the fixed Other given pairs are either antonyms or words
sentence ‘E’ which talks about the with different meanings.
usage/applications of red sanders and ‘D’ has
the phrase ‘in addition’ and continues to discuss 17) Answer: D
red sander’s usage. So, ED is a pair. The left out The words in option d are the correct synonyms
sentence is ‘B’ which sits right after CAF and of the words compliance and stringent
before ED. The final arrangement is CAFBED. respectively.
Compliance - in accordance with rules and
15) Answer: B regulation/to follow laws or rules
BE is a pair in the rearranged sequence. Stringent - very strict
The rearranged sequence is CAFBED. Abide and rigorous is the right pair of synonyms
Sentence ‘C’ is the perfect introduction to this for the words compliance and stringent.
passage as it contains the information ‘AP The other given options are not the synonyms or
government has applied for a GI tag for its Red meanings of the words highlighted.
sanders’. Now what is red sanders ? and the Rebellion - fighting
answer is sentence ‘A’ and it is obvious that it Lenient - not strict
will be the second sentence in the

Click Here For Bundle PDF Course | support@guidely.in Page 10 of 11


Bank Po Mains PDF Course 2024
English Day - 9

18) Answer: B Our requirement is to choose synonyms for both


The words in option b are the correct synonyms the highlighted words.
of the words luminaries and scrapped Composed - calm
respectively. Overdue - late in arriving
Luminary - a person who inspires or influence
others 20) Answer: B
Scrap - to get rid of something The words in option b are the correct synonyms
Influencers and dispose are the correct of the words cognisance and enmity
synonyms and other options given either have respectively.
different meaning or opposite to the given words. Cognizance - detailed knowledge about
something
19) Answer: C Enmity - hatred
The words in option c are the correct synonyms The other options either contain words which are
of the words desperate and belated respectively. antonyms or pairs of ‘synonym and antonym’.
Desperate - out of control, no hope Our requirement is to choose synonyms for both
Belated - late the highlighted words.
Hopeless and delayed is the best pair of words Hostility - strong feeling against
that can be picked. something/somebody
The other options either contain words which are Grudge - unfriendly
antonyms or pairs of ‘synonym and antonym’. Heedlessness - not considering something

Click Here For Bundle PDF Course | support@guidely.in Page 11 of 11


Bank Po Mains PDF Course 2024
Reasoning Day -10 (Eng)

Reasoning Aptitude
Octagon shape box is kept immediate northwest
Directions (1-5): Study the following information of the Cube shape box.
carefully and answer the questions given below. 1. Which of the following box is kept on Shelf 3 of
Eight boxes D, F, J, L, N, Q, T and W are kept on Stack 2?
four shelves. Each Box is different in shapes viz. a) The box which is kept immediately below box
Cone, Square, Rectangle, Circle, Oval, Sphere, J
Cube and Octagon but not necessarily in the b) Box N
same order. c) The box which is square in shape
Note: d) Box L
I. Each shelf has two stacks viz., Stack 1 and e) The box which is rectangle in shape
Stack 2. Stack 1 is exactly to the west of Stack 2.
Only one box is kept on each stack of each shelf. 2. On which of the following shelf and stack does
II. Stack 1 of shelf 2 is immediately above Stack the cube shape box is kept?
1 of Shelf 1 whereas Stack 2 of Shelf 2 is a) Shelf 4, Stack 1
immediately above Stack 2 of Shelf 1 and so on. b) Shelf 2, Stack 1
III. The shelves are numbered 1 to 4 from the c) Shelf 2, Stack 2
bottom to top respectively. d) Shelf 3, Stack 1
Box W is kept on an even numbered Shelf. Oval e) Shelf 4, Stack 2
shape box is kept immediately below box W in
different stacks. The number of shelves above 3. Which of the following box is kept two shelves
the Oval shape box is one more than the number above Oval shape box?
of shelves below box D. Square shape box is a) Box T
kept immediate northeast of box D. Box J is kept b) The sphere shaped box
two shelves below the Cone shape box. Sphere c) The Cone shaped box
shape box is kept southwest of box J. Cone d) Box Q
shape box is kept northwest of box F, which is e) The Octagon shaped box
not oval shaped box. Circle shape box is kept
immediately below box F in different stacks. Box 4. Four of the following five are alike in a certain
Q is kept southeast of box L, which is neither in way and thus form a group as per the given
circle shape nor in Oval shape. Either box F or arrangement. Which of the following does not
box Q is rectangle in shape. Box T is kept belong to that group?
immediately above the Rectangle shape box. a) Box T
b) The box which is in Circle shape

Click Here For Bundle PDF Course | support@guidely.in Page 1 of 10


Bank Po Mains PDF Course 2024
Reasoning Day -10 (Eng)

c) Box F who faces Q and S, who sits immediate left of T.


d) The box which is in Oval shape U is the father of P’s husband and sits second to
e) Box J the right of the one who is facing R. W is the only
daughter of U’s only son. X faces the one who
5. Which of the following statement(s) is/are true sits immediate left of S, who is the brother of W.
with respect to the final arrangement? Z, who is the son-in-law of U, faces the one who
I. Boxes D and J are kept in different stacks sits fourth to the left of S. S is the husband of T.
II. Box J is in Square shape Y is the niece of O, who is the brother-in-law of
III. Box N is kept immediately below box L in the Q. V sits immediate right of X, who is the brother-
same stack in-law of Z. Neither O nor P faces V. Both R and
a) Only I V are siblings, where R is the son of Y and sits
b) Only II and III adjacent to his mother. O neither faces away
c) Only III from the centre nor the married person.
d) Only I and III 6) How X is related to the one who sits second to
e) All I, II and III the right of U?
a) Niece
Directions (6-10): Study the following information b) Son
carefully and answer the given question. c) Nephew
Twelve persons – O, P, Q, R, S, T, U, V, W, X, Y d) Father
and Z of three generations family are sitting e) None of these.
around two concentric circular tables facing each
other in such a way that six persons are sitting at 7) Who among the following person sits third to
each table. The person sitting at the outer the left of S’s wife?
circular table is facing the centre and the person a) The one who sits third to the left of Q
sitting at the inner circular table is facing away b) U
from the centre. c) The one who faces the one who sits
Note: I-Not any single parents have a child. immediate right of X.
II- Not any couple faces to each other. d) P
III- If it is given that A and B face each other, e) O
then both are sitting at different tables and if it is
given that A and B sit opposite to each other, 8) If V is the sister of R, then how V is related to
then both are sitting at the same table. Q?
Q is the mother-in-law of P and sits third to the a) Niece
left of R. Only two persons sit between the one b) Granddaughter

Click Here For Bundle PDF Course | support@guidely.in Page 2 of 10


Bank Po Mains PDF Course 2024
Reasoning Day -10 (Eng)

c) Daughter Step I: 9764 8532 9731 8630 8543 5321


d) Aunt Step II: 1313 1008 1010 0809 1109 0605
e) Sister Step III: 19 164 10 81 181 25
Step IV: 57 328 20 243 543 75
9) The number of persons sitting between Q’s Step V: 74 19 02 15 38 74
husband and O is same as the number of Step V is the last step of the input.
persons sitting between_______ Input: 3276 5891 8423 9376 6512 2497
a) Z and his brother-in-law 11) How many even digits are there in the final
b) R and his sister output?
c) U and S a) Three
d) Z and R b) Five
e) None of these c) Four
d) Six
10) Which of the following statement(s) is/are not e) More than six
true as per the given arrangement?
I. Q is the grandmother of Y’s son. 12) What is the sum of the numbers which is
II. Y is not an immediate neighbour of her third from the left end in step II and fifth from the
husband. right end in step III?
III. Only two persons sit between T and X’s a) 1246
daughter b) 1162
a) Only I and III c) 1076
b) Only II d) 1026
c) Only III e) None of these
d) Only II and III
e) Only I 13) What is the 3rd step of the number "8693”?
a) 216
Directions (11-15): Study the following b) 116
information carefully and answer the given c) 146
questions. Number arrangement machines when d) 236
given an input line of numbers rearrange them e) 106
following a particular rule in each step. The
following is an illustration of input and 14) What is the sum of all the prime digits in the
rearrangement. second step?
Input: 4679 3285 9731 6830 4583 5321 a) 31

Click Here For Bundle PDF Course | support@guidely.in Page 3 of 10


Bank Po Mains PDF Course 2024
Reasoning Day -10 (Eng)

b) 28 Vistara, which is not D. Only two flights take off


c) 46 between A and the one which belongs to
d) 29 Emirates, which doesn’t take off before B. The
e) None of these flight which belongs to Qatar Airways takes off
immediately after Air Asia. F takes off two flights
15) What is the difference between the number after C, which doesn’t belong to Indigo.
which is second from the left end and third from 16) Which of the following flight takes off
the right end in step IV? immediately after E?
a) 0 a) The flight which takes off two flights before B.
b) 10 b) C
c) 15 c) A
d) 13 d) The one which belongs to Qatar Airways.
e) 09 e) None of these

Directions (16-20): Study the following 17) Which of the following flight belongs to Air
information carefully and answer the given Asia?
questions. a) B
Eight flights - A, B, C, D, E, F, G and H take off in b) The one which takes off immediately before D.
a sequence one after another. They belong to c) E
eight different airlines- Air India, Indigo, Spice-jet, d) The one which takes off two flights after A.
Air Asia, Go-first, Vistara, Emirates and Qatar e) H
Airways.
Note: The flight whose name starts with a vowel 18) How many flights take off between the one
neither takes off consecutively nor belongs to the which belongs to Vistara and Flight A?
airlines whose name starts with a vowel. a) Two
B takes off three flights before the one which b) As many flights take off between G and the
belongs to Go-first. Only two flights take off one which belongs to Spice- jet.
between the one which belongs to Air India and c)As many flights take off between C and the one
D, which takes off immediately before B.E takes which belongs to Indigo.
off four flights after G, neither of them belongs to d) Three
Air India and Go-first. G doesn’t take off e) None
immediately before D. The number of flights
takes off before E is one less than the number of 19) Which of the following statement is/are not
flights takes off after the flight which belongs to true as per the given arrangement?

Click Here For Bundle PDF Course | support@guidely.in Page 4 of 10


Bank Po Mains PDF Course 2024
Reasoning Day -10 (Eng)

I. Flight B takes off three flights before the one 20) If Flight B is related to the one which belongs
which belongs to Go-first. to Air Asia and Flight H is related to the one
II. Two flights take off between one which which belongs to Emirates in a certain way, then
belongs to Air Asia and Spice-jet. Flight F is related to which of the following flight?
III. B doesn’t belong to Emirates a) The one which belongs to Go-first
a) Only I and III b) The one which belongs to Vistara.
b) Only III c) The one which belongs to Indigo.
c) Only I d) The one which belongs to Air India
d) Only II and III e) The one which belongs to Spice Jet
e) Only II
Click Here to Get the Detailed Video Solution for the above given Questions
Or Scan the QR Code to Get the Detailed Video Solutions

Answer Key with Explanation


Directions (1-5):
1. Answer: E
2. Answer: C
3. Answer: E
4. Answer: B (All boxes are kept in stack 2
except option b) We have,
5. Answer: A  Box W is kept on an even numbered
Final arrangement Shelf.
 Oval shape box is kept immediately below
the box W in different stacks.
From the above conditions, there are four
possibilities

Click Here For Bundle PDF Course | support@guidely.in Page 5 of 10


Bank Po Mains PDF Course 2024
Reasoning Day -10 (Eng)

 Box Q is kept southeast of box L, which is


neither in circle shape nor in Oval shape.
 Either box F or box Q is rectangle in
shape.
 Box T is kept immediately above the
Rectangle shape box.
Again we have,  Octagon shape box is kept immediate
 The number of shelves above the Oval northwest of the Cube shape box.
shape box is one more than the number After applying above conditionsCase2 gets
of shelves below box D. eliminated, Hence Case3 shows the final
 Square shape box is kept immediate arrangement
northeast of box D.
 Box J is kept two shelves below the Cone
shape box.
 Sphere shape box is kept southwest of
box J.
 Cone shape box is kept northwest of box
F, which is not oval shaped box.
 Circle shape box is kept immediately Direction (6-10):
below box F in different stacks. 6. Answer: D
After applying above conditionsCase1 and 7. Answer: C
Case4 get eliminated 8. Answer: B
9. Answer: A
10. Answer: B
Final Arrangement

Again we have,

Click Here For Bundle PDF Course | support@guidely.in Page 6 of 10


Bank Po Mains PDF Course 2024
Reasoning Day -10 (Eng)

Q-

P= (+/-)
Again we have,
 U is the father of P’s husband and sits
second to the right of the one who is facing
R.
 W is the only daughter of U’s only son.
 X faces the one who sits immediate left of
S, who is the brother of W.
Blood relation:
 Z, who is the son-in-law of U, faces the
one who sits fourth to the left of S.
 S is the husband of T.
 Y is the niece of O, who is the brother-in-
law of Q.
 V sits immediate right of X, who is the
brother-in-law of Z.

We have,
 Q is the mother-in-law of P and sits third to
the left of R.
 Only two persons sit between the one who
faces Q and S, who sits immediate left of T.
From the above conditions, we have two
possibilities:
Blood relation:
Q-=U+ O+

P-=X+ Y-=Z+

W- S+=T-
Again we have,
B Blood relations:  Neither O nor P faces V.

Click Here For Bundle PDF Course | support@guidely.in Page 7 of 10


Bank Po Mains PDF Course 2024
Reasoning Day -10 (Eng)

 Both R and V are siblings, where R is the Step I: 7632 9851 8432 9763 6521 9742
son of Y and sits adjacent to his mother. Step II: 0909 1013 1007 1213 0707 1111
 O neither faces away from the centre nor Step III: 81 19 149 19 49 11
married person. Step IV: 243 57 447 57 147 33
From the above conditions, case 1 gets Step V: 15 74 57 74 54 18
eliminated because O faces away from the Step I: All the digits are arranged in descending
centre. Hence case 2 shows the final order within the numbers.
arrangement. Step II: Sum of the first and fourth digits is
written first and then sum of the second and third
digits is written.
Step III: Square of the first and last digits of the
numbers and remove the second and third digits.
Step IV: All the even numbers are multiplied by 2
and all the odd numbers are multiplied by 3.
Step V: Add all the even digits and the square of
Blood relation
all the odd digits within the numbers.

Directions (16-20):
16) Answer: E
17) Answer: B
18) Answer: C
19) Answer: B
20) Answer: D
Final arrangement

Directions (11-15):
11) Answer: C
12) Answer: D
13) Answer: B
14) Answer: D
15) Answer: A
Final arrangement
Input: 3276 5891 8432 9376 6512 2497

Click Here For Bundle PDF Course | support@guidely.in Page 8 of 10


Bank Po Mains PDF Course 2024
Reasoning Day -10 (Eng)

after the flight which belongs to Vistara,


which is not D.
 Only two flights take off between A and the
one which belongs to Emirates, which
doesn’t take off before B.

We have,
 B takes off three flights before the one which
belongs to Go-first.
 Only two flights take off between the one
Again we have,
which belongs to Air India and D, which takes off
 The flight which belongs to Qatar Airways
immediately before B.
takes off immediately after Air Asia.
From the above conditions, we have two
 F takes off two flights after C, which doesn’t
possibilities:
belongs to Indigo
From the above condition case 2 gets eliminated
because C belongs to Indigo. Hence case 1
shows the final arrangement.

Again we have,
 E takes off four flights after G, neither of them
belongs to Air India and Go-first. G doesn’t
take off immediately before D.
 The number of flights takes off before E is
one less than the number of flights takes off

Click Here For Bundle PDF Course | support@guidely.in Page 9 of 10


Bank Po Mains PDF Course 2024
Reasoning Day -10 (Eng)

Click Here For Bundle PDF Course | support@guidely.in Page 10 of 10


Bank Po Mains PDF Course 2024
Quantitative Aptitude Day -10 (Eng)

Quantitative Aptitude

Directions (01 - 05): Study the following information carefully and answer the questions given below.
The given pie chart shows the percentage distribution of the number of selling target (kg) of five fruits
[Apple, orange, grapes, Papaya, Mango] by a fruit seller, producer company.

Note- Red, orange, black, green, and yellow colors represent fruit apple, orange, grapes, papaya, and
mango respectively. The percentage of actual sales of fruit apples, Orange, grapes, papaya, and mango
out of target sales is 80%, 70%, 60%, 80%, and 90% respectively. A total of 180 kg of grapes were sold
by the company.
1) Cost of per kg apple sold by the company is fulfilled for fruit strawberries then find the total
Rs.80, Per kg orange sold by the company is target sales for strawberries and Papaya
Rs.60 and Per kg Mango sold by the company is together?
Rs.40. Find the total revenue earned by the a) 652kg
company by selling these three fruits? b) 852kg
a) Rs.28400 c) 752kg
b) Rs.25800 d) 725kg
c) Rs.29700 e) 951kg
d) Rs.27600
e) Rs.22300 3) Find the ratio between the total number of
sales of the target not achieved for fruit apples
2) Ratio of the actual sale of fruit strawberries and grapes together and for fruit oranges and
and papaya is 14:13. If 70% of the target is papaya together?

Click Here For Bundle PDF Course | support@guidely.in Page 1 of 9


Bank Po Mains PDF Course 2024
Quantitative Aptitude Day -10 (Eng)

a) 2:3 d) 135kg
b) 3:2 e) None of these
c) 3:5
d) 3:7 5) Find the difference between central angle
e) None of these formed by actual sales of oranges and actual
sales of mango if the total actual sales of fruit are
4) Find the difference between the actual sales represented in the pie chart?
of apple and orange fruit together and the actual a) 45.480
sales of grapes and papaya fruit together? b) 62.480
a) 155kg c) 55.480
b) 120kg d) 35.480
c) 165kg e) None of these

Directions (06 - 10): Study the following information carefully and answer the questions given below.
The given bar graphs show the number of test matches played, no of test matches drawn and no of
matches lost by the five teams is given.

Note – B+C=80, Total number matches played= Number of matches won+ Number of matches drawn +
Number of matches lost.
Win% = [Total Number of the match won/total Number of matches played] *100

Click Here For Bundle PDF Course | support@guidely.in Page 2 of 9


Bank Po Mains PDF Course 2024
Quantitative Aptitude Day -10 (Eng)

6) Find the difference between the winning a) 11:7


percentage of team A and the winning b) 10:7
percentage of team E? c) 19:12
a) 9.35% d) 18:11
b) 2.58% e) None of these
c) 3.75%
d) 1.25% 9) Find the value 3A+4B-B% of 320+3C-C% of
e) 6.35% 420+325=?
a) 409
7) Team F wins 10 matches more than team C. b) 502
Ratio of total matches played by team C and c) 650
team F is 5:6. Find the winning percentage of d) 582
team F? e) None of these
a) 30%
b) 38% 10) In the next 50 matches team D lost 15
c) 37% matches and draw 9 matches. Find the increase
d) 33.33% or decrease in winning percentage for team D
e) 39% after and before 50 matches?
a) 0.58%
8) Find the ratio between the total number of b) 0.68%
matches won by teams A, B, and E together and c) 0.21%
the total number of matches drawn by teams C, d) 0.32%
D, and E together? e) None of these

Directions (11 - 15): Study the following information carefully and answer the questions given below.
The given table shows the expenditure on rent, grocery, transport, bill and savings of five people is given.

Click Here For Bundle PDF Course | support@guidely.in Page 3 of 9


Bank Po Mains PDF Course 2024
Quantitative Aptitude Day -10 (Eng)

Note – Total income of a person is the sum of expenditure in rent, grocery, transport, bill, and saving
together.
11) Expenditure of D in rent, grocery transport, b) Rs.4520
and bill are 10%,20%,10%, and 20% more than c) Rs.6520
that of B. If the total income of D is Rs. 32500 d) Rs.6540
then find the saving of D? e) Rs.6320
a) Rs.5420
b) Rs.6820 13) Find which one is true?
c) Rs.8720 I. Total expenditure in the grocery of all three
d) Rs.8520 together is Rs.17200.
e) Rs.9520 II. Total expenditure in rent of all three together is
Rs.13900.
12) Find the difference between the average III. Total expenditure in transport of all three
income of A and B together and the total saving together is Rs.12800.
of A, B, and C together? a) Only III is true
a) Rs.5200 b) None is true

Click Here For Bundle PDF Course | support@guidely.in Page 4 of 9


Bank Po Mains PDF Course 2024
Quantitative Aptitude Day -10 (Eng)

c) Only II is true 40% more than the number of boys in the same
d) Only II and III are true class and number of Number of girls in class E is
e) None of these 20% more than the number of boys in class E.
The total number of boys in classes D and C is
14) Find the ratio of total expenditure in transport 57 and the number of boys in all schools
and rent of B and total expenditure in grocery together is 139.
and bill of C? 16) Total number of students in classes A and B
a) 94:101 together is what percent of the total number of
b) 97:112 students in classes C and D together?
c) 98:105 a) 45.35%
d) 91:101 b) 68.36%
e) None of these c) 84.28%
d) 78.33%
15) If the income of A and B is increased by 15% e) 55.32%
and 20% respectively. then find the new average
income of A and B is approximately what percent 17) Number of students in class F is 15 more
of the income of C? than the number of students in class C. The ratio
a) 130% of boys and girls in class F is 3:2. Find the
b) 140% number of girls in class F?
c) 160% a) 32
d) 110% b) 30
e) None of these c) 48
d) 38
Directions (16 - 18): Study the following e) 42
information carefully and answer the questions
given below. 18) Find the difference between the total number
The total number of boys in classes A and B of girls in all classes together and the total
together is 52 and the number of girls in class number of boys in all classes together?
classes B and C together is 84. The total number a) 51
of boys in classes B and C together is 64 and the b) 46
total number of girls in A and B together is 66. c) 52
The ratio of the number of boys and girls in d) 56
classes A, B, and C is 2:3,8:9, and 2:3 e) None of these
respectively. The number of girls in class D is

Click Here For Bundle PDF Course | support@guidely.in Page 5 of 9


Bank Po Mains PDF Course 2024
Quantitative Aptitude Day -10 (Eng)

Directions (19 - 20): Study the following b) 42L


information carefully and answer the questions c) 25L
given below. d) 38L
The ratio of milk and water in container A is 4:3 e) None of these
and the ratio of milk and water in container B is
5:3. Container C contains 40 L pure milk and 20) At first 20L of milk is mixed in container D
container D contains 30 L water. The amount of then 75% mixture from container A is mixed in
milk in container A is 50% more than the amount container D. After that 1.25 L water is added to
of milk in container C and the ratio of the amount the mixture. Find the ratio of milk and water at
of water in containers B and D is 3:5. the end of container D?
19) If 20% mixture from container A and 33.33% a) 1:1
mixture from container B is mixed in container C, b) 2:1
find the difference between the amount of milk c)3:2
and water in container C at the end? d) 1:2
a) 47L e) None of these
Click Here to Get the Detailed Video Solution for the above given Questions
Or Scan the QR Code to Get the Detailed Video Solutions

Answer Key with Explanation


Directions (01 - 05):
So, actual sales of grapes is 60%, so, 60%=180,
total target sales is 100%=[180/60]*100=300
So, 24%=300, 100%=[300/24]*100=1250

1) Answer: C
Required revenue =
180*80+105*60+225*40=Rs.29700

Click Here For Bundle PDF Course | support@guidely.in Page 6 of 9


Bank Po Mains PDF Course 2024
Quantitative Aptitude Day -10 (Eng)

2) Answer: D For team A,


Total actual sales of fruit strawberry are The Number of matches played = 4*C=4*50=200
260*14/13=280kg The Number of matches drawn = 2*40=80
So, target sales for strawberries are The number of matches lost=40
[280/70]*100=400kg The number of matches won=200-80-40=80
Total target sales of strawberry and papaya are So, the winning percentage of team A is
400+325=725kg [80/200]*100=40%
Similarly, we can calculate the value of the other
3) Answer: B also.
Required ratio
=[(225-180)+(300-180)] : [(150-105)+(325-
260)]=165:110=3:2

4) Answer: A
Required difference =[180+260]- 6) Answer: C
[180+105]=155kg Required difference = [43.75-40]%=3.75%

5) Answer: A 7) Answer: D
Actual sales of orange is = The total number of the match won by team F is
[105/(180+105+180+260+225)]*360=39.780 30+10=40
Actual sales of Mango is = The total number of matches played by team F is
[225/(180+105+180+260+225)]*360=85.260 =100*6/5=120
So, the difference is = 85.26-39.78=45.480 So, winning percentage = [40/120]*100=33.33%

Directions (06 - 10): 8) Answer: B


From the graph, we see that the number of Required ratio = [80+50+70]
matches lost by team A and the number of :[50+40+50]=200:140=10:7
matches drawn by the team B are in the same
line. So, we can say A=40. 9) Answer: A
In the same way number of matches drawn by 3A+4B-B% of 320+3C-C% of 420+325
teams, C and E are the same, i.e., 50. =3*40+4*30-30*320/100+3*50-
So, C=50 50*420/100+325=409
B+C=80, B=30

Click Here For Bundle PDF Course | support@guidely.in Page 7 of 9


Bank Po Mains PDF Course 2024
Quantitative Aptitude Day -10 (Eng)

10) Answer: A III. Total expenditure in transport of all three


After the next 50 matches the total number of together is Rs.11700.
winning the match is 60+[50-15-9]=86 So, None is true.
New winning percentage =
[86/170]*100=50.58% 14) Answer: A
Increase by 50.58-50=0.58% Required ratio = [4000+5400] : [5600+4500]=
9400:10100=94:101
Directions (11 - 15):
15) Answer: A
The new average income of A and B is
[28000*115/100+27800*120/100]/2=32780
Required percentage =
[32780/25360]*100=129.26%=130%

Directions (16 - 18):


Let, the number of boys and girls in class A be
2x and 3x.
11) Answer: C
Let, the number of boys and girls in class A be
The total expenditure of D is
8y and 9y.
=5400*110/100+5200*120/100+4000*110/100+6
Let, the number of boys and girls in class A be
000*120/100=Rs.23780
2z and 3z.
Total saving of D is =32500-23780= Rs.8720
2x+8y=52,-i 3x+9y=66,-ii 9y+3z=84 -iii
By solving i and ii we get, x=10 and y=4
12) Answer: D
So, 9*4+3z=84, z=[84-36]/3=16
The average income of A and B is
So, the number of boys and girls in class A is
[28000+27800]/2=55800/2=27900
2*10=20 and 3*10=30
Total saving is = 8400+7200+5760=21360
The number of boys and girls in class B is
Required difference=27900-21360=Rs. 6540
8*4=32 and 9*4=36
The number of boys and girls in class C is
13) Answer: B
2*16=32 and 3*16=48
I. Total expenditure in the grocery of all three
The Number of boys in D is 57-32=25
together is Rs.17300.
The Number of boys in E is 139-20-32-32-25=30
II. Total expenditure in rent of all three together
The Number of girls in D is 25*140/100=35
is Rs.14900.

Click Here For Bundle PDF Course | support@guidely.in Page 8 of 9


Bank Po Mains PDF Course 2024
Quantitative Aptitude Day -10 (Eng)

The Number of girls in E is 30*120/100=36 Water in container D is 30 L.


16) Answer: C Water in container B is 30*3/5=18L
Required percentage Milk in container B is 18*5/3=30L
=[(50+68)/(80+60)]*100=84.28%
19) Answer: A
17) Answer: D Milk in container C is
The number of students in class F is 40+60*20/100+30*33.33/100=40+12+10=62L
32+48+15=95 Water in container C is
The number of girls in class F is 95*2/5=38 45*20/100+18*33.33/100=9+6=15L
Required difference = 62-15=47L
18) Answer: B
Required difference 20) Answer: A
=[30+36+48+35+36]- 139=46 The final amount of milk in D is
20+60*75/100=65L
Directions (19 - 20): The final amount of water in D is
Milk in container C is 40 L. 30+45*75/100+1.25=65L
Milk in container A is 40*150/100=60L The required ratio is 65:65=1:1
Water in container A is 60*3/4=45L

Click Here For Bundle PDF Course | support@guidely.in Page 9 of 9


Bank Po Mains PDF Course 2024
English Day - 10

English Language
Directions (1-7): Read the given passage Hunger Index (SHI) is calculated using the same
carefully and answer the following questions indicators except calorie undernourishment,
based on the passage. Some words are which is replaced by body mass index (BMI)
highlighted to help you locate while answering undernourishment among the working-age
the questions. population, as data on calorie undernourishment
Despite being a major food producer with are not available since 2012. Data for stunting,
extensive food security schemes and the largest wasting, and mortality among children below the
public distribution system in the world, India still age of five are sourced from the fifth round of the
grapples with significant levels of food insecurity, National Family Health Survey (NFHS-5), while
hunger, and child malnutrition. The Global the prevalence of BMI undernourishment is
Hunger Index (GHI), 2022, ranked India 107 computed using NFHS-5 (2019-21) and Wave 1
among 121 countries, behind Nigeria (103) and of the Longitudinal Ageing Study in India (2017-
Pakistan (99). The GHI provides a composite 18). The calculation of the SHI score involves
measurement and tracks undernourishment and combining the normalised values of the four
hunger at the national level across three indicators using the techniques recommended by
dimensions: calorie undernourishment, child the GHI. The SHI scores range between 0 and
malnutrition, and under-five mortality. According 100, with higher scores indicating more hunger.
to the State of Food Security and Nutrition in the Scores below 10 signify low hunger, 10-20
World report of 2022, India is home to 224.3 moderate, 20-30 serious, 30-40 alarming, and 50
million undernourished people. Disparities are or above extremely alarming.
evident among States. Leveraging subnational In the SHI, Bihar, Jharkhand, and Chhattisgarh
data that encompasses the three dimensions of scored 35, which places them in the ‘alarming’
the GHI enables the development of an India- category. Gujarat, Uttar Pradesh, Assam,
specific hunger index at the level of States and Odisha, Madhya Pradesh, Tripura, Maharashtra,
Union Territories. This plays a pivotal role in and West Bengal all scored above the national
evaluating the extent of undernourishment at a average (29). The performance of these States
more localised scale, which is critical for meeting resembles that of African nations such as Haiti,
the Sustainable Development Goals of Niger, Liberia, and Sierra Leone. On the other
eradicating hunger and malnutrition. The GHI is hand, Chandigarh scored 12, and Sikkim,
computed using four indicators — the prevalence Puducherry, and Kerala all scored below 16.
of calorie undernourishment; stunting, wasting, These States, along with Manipur, Mizoram,
and mortality among children below the age of Punjab, Delhi, Arunachal Pradesh, Andaman and
five; and under-five mortality rate. The State Nicobar Islands, and Tamil Nadu, fall under the

Click Here For Bundle PDF Course | support@guidely.in Page 1 of 10


Bank Po Mains PDF Course 2024
English Day - 10

‘moderate hunger’ category. All the other States, conducted in 2020-21, four key questions were
which scored below the national average and included to gauge household food insecurity.
above 20, have a problem of ‘serious hunger’. No Unfortunately, information on these is missing
State falls under the ‘low hunger’ category. The from the NSS report. While the GHI has faced
impact of COVID-19 on the SHI is not captured significant criticism from experts regarding its
here since post-pandemic estimates are not yet conceptualisation, indicator selection, and
available. In 2008, Purnima Menon, Anil aggregation methods, it does provide critical
Deolalikar, and Anjor Bhaskar made a insight into the state of undernourishment and
comparable effort to grasp the variation in hunger child nutrition. India’s poor performance in the
at a subnational level using the then GHI is primarily attributed to its high prevalence
methodology of computing GHI. Among the 17 of undernourishment and child malnutrition. India
States they assessed, Punjab led the list, with ranks unfavourably in child wasting, performing
Kerala and Andhra Pradesh following closely as worse than many low-income African nations.
top performers. On the other hand, Madhya The NFHS-5 indicated that one-third of children
Pradesh, Jharkhand, Bihar, and Chhattisgarh under the age of five are stunted and
were ranked as the least-performing States. Over underweight, while every fifth child suffers from
the last half a decade, India’s GHI score has wasting. Despite India’s notable progress in
deteriorated primarily due to the increasing alleviating extreme poverty over the last 15
prevalence of calorie undernourishment. years, as indicated by the recent National
According to the Food and Agriculture Multidimensional Poverty Index, challenges
Organization, the proportion of calorie persist in addressing the disparity in food
undernourishment in India has been escalating insecurity, hunger, and child malnutrition
since 2017, reaching 16.3% in 2020, equivalent 1) Global Hunger Index (GHI) faced criticisms
to the 2009 statistic. The Indian government has from experts regarding which of the following
disputed these conclusions by raising concerns listed items, as mentioned in the passage ?
about the data and methodology used in I. Indicator selection
calculating the GHI. However, it has not been II. Conceptualization
able to provide empirical evidence to support its III. Aggregation methods
claims. Notably, no National Sample Survey IV. Documents and procedures
(NSS) round on nutritional intake has been a) Both I and III
conducted by the government since 2011-12, b) Both II and III
which used to offer insights into the prevalence c) Both I and IV
of calorie undernourishment at national and d) I, II and III
subnational levels. In the 78th round of the NSS e) I, III and IV

Click Here For Bundle PDF Course | support@guidely.in Page 2 of 10


Bank Po Mains PDF Course 2024
English Day - 10

2) According to the passage, what is India's c) I, II and III


performance in child wasting when compared to d) II, III and IV
that of low-income African nations ? e) I, II, III and IV
a) India performs worse in child wasting
compared to many low-income African nations,as 4) What are the challenges that India is still
indicated by the NFHS-5 data. facing as stated in the passage?
b) India performs better in child wasting a) India still faces challenges in economic
compared to many low-income African nations, growth, education and health care.
as indicated by the NFHS-5 data. b) India still faces challenges in social inequality
c) India’s performance is the same in child and political stability.
wasting as compared to many low-income c) India still faces challenges in addressing food
African nations, as indicated by the NFHS-5 insecurity, hunger, and child malnutrition.
data. d) India still faces challenges in addressing
d) There is no proper data on India’s demographic changes, cultural and social
performance in child wasting. challenges.
e) None of the above e) India still faces challenges in addressing
natural disasters and health crises.
3) Which of the following statements is/are true
according to the given passage? 5) Which of the following is the SYNONYM of the
I. The Global Hunger Index (GHI) is a word ‘LEVERAGING’ as mentioned in the
measurement that tracks undernourishment and passage ?
hunger at the national level across three a) working
dimensions. b) helplessness
II. According to the passage, India is home to c) impotence
224.3 million undernourished people. d) weakening
III. One-third of children under the age of five in e) dropping
India are stunted and underweight, and every
fifth child suffers from wasting. 6) Which of the following is the ANTONYM of the
IV. The prevalence of calorie undernourishment word ‘GRASP’ as mentioned in the passage?
in India has been increasing since 2017, a) comprehend
reaching 16.3% in 2020, which is equivalent to b) understand
the 2009 statistic. c) confusion
a) Both I and III d) perceive
b) Both I and IV e) observe

Click Here For Bundle PDF Course | support@guidely.in Page 3 of 10


Bank Po Mains PDF Course 2024
English Day - 10

(E) Roughly a week after the Indian Space


7) Which of the following is the SYNONYM of the Research Organisation (ISRO) soft-landed a
word ‘EMPIRICAL’ as mentioned in the robotic lander and rover on the moon’s south
passage? polar region, it launched India’s first space
a) theoretical mission dedicated to studying the sun, in the
b) visionary form of a spacecraft named Aditya-L1.
c) unproven 8) Which of the following is the first sentence
d) factual after rearrangement ?
e) hypothetical a) A
b) B
Directions (8-12): Rearrange the following five c) C
sentences (A), (B), (C), (D) and (E) in the proper d) D
sequence to form a coherent paragraph and then e) E
answer the questions given below.
(A) Aditya-L1 will study the sun in multiple 9) Which of the following is the second sentence
wavelengths with its suite of seven instruments: after rearrangement ?
four remote-sensing and three in situ. a) A
(B) This vignette illustrates a certain maturity on b) B
the part of the Indian space programme: it enters c) C
a phase where its growth from strength to d) D
strength becomes indistinguishable from e) E
international leadership in space exploration,
even as it carries forward an old tradition of 10) Which of the following is the third sentence
studying the sun, exemplified by the Kodaikanal after rearrangement ?
Solar Observatory. a) A
(C) Barely half a day after the launch, ISRO b) B
announced that its lunar rover had completed the c) C
surface studies it had planned and that it had d) D
been “parked” ahead of the two-week-long lunar e) E
night.
(D) Even though it is the star closest to the earth 11) Which of the following is the fourth sentence
and has been and continues to be observed by a after rearrangement ?
panoply of telescopes, the sun still holds many a) A
secrets. b) B

Click Here For Bundle PDF Course | support@guidely.in Page 4 of 10


Bank Po Mains PDF Course 2024
English Day - 10

c) C I. despite facing numerous obstacles, she


d) D remained determined to
e) E II. complete her doctoral dissertation and achieve
her academic goals.
12) Which of the following is the fifth sentence 1. In spite of,
after rearrangement ? 2. Nevertheless,
a) A 3. Yet,
b) B a) Only 2
c) C b) Only 3
d) D c) Both 1 and 2
e) E d) Both 1 and 3
e) All 1, 2 and 3
Directions (13-15) : Select the phrase/connector
out of three phrases/connectors given as (1), (2) 15)
and (3) which can be used in the beginning (to I. the project's success was largely
start the sentence) to form a single sentence II. due to the collaborative efforts of the team.
from the two statements given while implying the 1. However,
same meaning as expressed in the statement 2. Still,
sentences. 3. On the other hand,
13) a) Only 1
I. all employees must complete the new b) Only 3
cybersecurity training program c) Both 1 and 2
II. before gaining access to sensitive company d) Both 1 and 3
data. e) All 1, 2 and 3
1. Hereafter,
2. Therefore, Directions (16-20): In each of the following
3. Aftermath, questions, a highlighted word is given followed
a) Only 1 by three sentences. Choose the sentence(s) that
b) Only 2 has/have the wrong/inappropriate usage of the
c) Both 1 and 2 highlighted word.
d) Both 1 and 3 16) UBIQUITOUS
e) All 1, 2 and 3 A. The smartphone has become ubiquitous in
14) modern society, with nearly everyone owning
one.

Click Here For Bundle PDF Course | support@guidely.in Page 5 of 10


Bank Po Mains PDF Course 2024
English Day - 10

B. The scent of fresh flowers was ubiquitous in B. The band's cacophony of instruments clashed
the bustling farmers' market. discordantly, leaving the audience covering their
C. In the digital age, information is ubiquitously ears in discomfort.
accessible, thanks to the internet and mobile C. The cacophonies of voices in the crowded
devices. cafeteria made it challenging to hold a
a) Only A conversation.
b) Only B a) Only A
c) Both A and C b) Only C
d) Both B and C c) Both A and B
e) All correct d) Both A and C
e) All correct
17) QUINTESSENTIAL
A. The Eiffel Tower is often considered the 19) ELOQUENT
quintessentialled symbol of Paris. A. Even though he was a man of few words, his
B. The cozy, old-fashioned bookstore was eyes were eloquented, conveying a depth of
quintessentially charming, with its creaky emotion that words could not express.
wooden floors and shelves lined with leather- B. He eloquently expressed his views on social
bound classics. justice, captivating the audience with his
C. Her elegant and graceful dance performance articulate and persuasive words.
was the quintessential example of classical C. Her eloquented speech moved the audience
ballet. to tears as she spoke passionately about the
a) Only A importance of preserving the natural world for
b) Only C future generations.
c) Both A and B a) Only B
d) Both B and C b) Only C
e) All correct c) Both A and C
d) Both B and C
18) CACOPHONY e) All correct
A. Amidst the cacophony of car horns, sirens,
and shouting pedestrians, it was nearly 20) CLARIFY
impossible to have a peaceful moment in the A. Please clarify your question so that I can
bustling city. provide you with a more accurate answer.

Click Here For Bundle PDF Course | support@guidely.in Page 6 of 10


Bank Po Mains PDF Course 2024
English Day - 10

B. After a lengthy discussion, they clarify their a) Only B


misunderstandings and reached a mutual b) Only C
agreement. c) Both A and B
C. The new policy document clarifies the d) Both A and C
company's stance on employee benefits and e) All correct
outlines the changes that will be implemented.
Click Here to Get the Detailed Video Solution for the above given Questions
Or Scan the QR Code to Get the Detailed Video Solutions

Answer Key with Explanation


1) Answer: D 4) Answer: C
Global Hunger Index faced criticism from experts According to the passage option c is the correct
regarding its Indicator selection, answer.
conceptualization and aggregation methods.
IV - document and procedures - not mentioned 5) Answer: A
in the passage. ‘Working’ is the correct synonym of the given
word ‘leveraging’
2) Answer: A Leveraging - use (something) to maximum
According to the passage statement given in advantage.
option a is correct regarding the performance of Working is a word with similar meaning hence
India in terms of child wasting when compared to we choose ‘option a’ as our answer.
low income African nations. Helplessness, impotence, weakening and
dropping are words with an opposite meaning to
3) Answer: E the given word - hence discarded.
All the given four statements are true according
to the given passage. 6) Answer: C

Click Here For Bundle PDF Course | support@guidely.in Page 7 of 10


Bank Po Mains PDF Course 2024
English Day - 10

‘Confusion’ is the antonym of the word ‘GRASP’ is A which talks about the mission of Adithya -
Grasp - to understand something completely L1. The last/final sentence is D.
Other given words such as comprehend,
understand, perceive and observe are the exact 10) Answer: B
meaning of the given word ‘grasp’. The third sentence after rearrangement is B.
The correct sequence of rearrangement is
7) Answer: D ECBAD.
Factual is the right synonym of the word E is the first sentence/starter sentence. E is
empirical. followed by sentence C which adds more
Empirical - based on experiments and practical information on the rover and lander launched.
experience, not on ideas Third sentence would be B which is a
continuation of sentence C. The fourth sentence
8) Answer: E is A which talks about the mission of Adithya -
The first sentence after rearrangement is E. L1. The last/final sentence is D.
The correct sequence of rearrangement is
ECBAD. 11) Answer: A
E is the first sentence/starter sentence. E is The fourth sentence after rearrangement is A.
followed by sentence C which adds more The correct sequence of rearrangement is
information on the rover and lander launched. ECBAD.
Third sentence would be B which is a E is the first sentence/starter sentence. E is
continuation of sentence C. The fourth sentence followed by sentence C which adds more
is A which talks about the mission of Adithya - information on the rover and lander launched.
L1. The last/final sentence is D. Third sentence would be B which is a
continuation of sentence C. The fourth sentence
9) Answer: C is A which talks about the mission of Adithya -
The second sentence after rearrangement is C. L1. The last/final sentence is D.
The correct sequence of rearrangement is
ECBAD. 12) Answer: D
E is the first sentence/starter sentence. E is The fifth sentence after rearrangement is D.
followed by sentence C which adds more The correct sequence of rearrangement is
information on the rover and lander launched. ECBAD.
Third sentence would be B which is a E is the first sentence/starter sentence. E is
continuation of sentence C. The fourth sentence followed by sentence C which adds more

Click Here For Bundle PDF Course | support@guidely.in Page 8 of 10


Bank Po Mains PDF Course 2024
English Day - 10

information on the rover and lander launched. On the other hand, the project's success was
Third sentence would be B which is a largely due to the
continuation of sentence C. The fourth sentence collaborative efforts of the team.
is A which talks about the mission of Adithya -
L1. The last/final sentence is D. 16) Answer: E
All the sentences have used the given word in
13) Answer: C the most appropriate way possible.
Both 1 and 2 are appropriate starters. Ubiquitous - seeming to be everywhere or in
The sentences : several places at the same time; very common
Hereafter, all employees must complete the new Hence we go for option e as our answer.
cybersecurity training program before gaining
access to sensitive company data. 17) Answer: A
Therefore, all employees must complete the new Sentence A has used the word in a wrong way.
cybersecurity training program before gaining It must be ‘quintessential’ not ‘quintessentialled’
access to sensitive company data. (incorrect)
Quintessential - being the perfect example of
14) Answer: A something
Only ‘nevertheless’ is the correct starter that The sentence : The Eiffel Tower is often
connects both the fragments and makes a considered the quintessential
complete meaningful sentence. symbol of Paris.
The sentence:
Nevertheless, despite facing numerous 18) Answer: B
obstacles, she remained determined to complete Sentence C has used the given way
her doctoral dissertation and achieve her inappropriately.
academic goals. Replace ‘cacophonies’ with ‘cacophony’ to make
it correct.
15) Answer: D Cacophony - a mixture of loud, unpleasant
‘However’ and ‘On the other hand’ are the right sounds; noise
starters. The sentence : The cacophony of voices in the
The sentences : crowded cafeteria made it challenging to hold a
However, the project's success was largely due conversation.
to the collaborative efforts of the team. 19) Answer: C

Click Here For Bundle PDF Course | support@guidely.in Page 9 of 10


Bank Po Mains PDF Course 2024
English Day - 10

Both A and C have used the word in a importance of preserving the natural world for
wrong/inappropriate way. future generations.
Replace the word ‘eloquented’ with ‘eloquent’ in
both the sentences to make them correct and 20) Answer: A
meaningful. Only sentence B has used the word in an
Eloquent - able to use language and express inappropriate way.
your opinions well, especially when you speak in Change the word from ‘clarify’ to ‘clarity’ to make
public the sentence correct and meaningful.
The sentences : Clarify - to make something become clear and
Even though he was a man of few words, his easier to understand
eyes were eloquently, conveying a depth of The sentence:
emotion that words could not express. After a lengthy discussion, they clarified their
Her eloquent speech moved the audience to misunderstandings and reached a mutual
tears as she spoke passionately about the agreement.

Click Here For Bundle PDF Course | support@guidely.in Page 10 of 10


Bank Po Mains PDF Course 2024
Reasoning Day -11 (Eng)

Reasoning Aptitude
Directions (1-5): Study the following information multiple of 6, then “P” is named to the second
carefully and answer the given questions. stoppage reached by the bus.
There are two buses – X and Y started travelling Condition II: If the stoppage is either north-east
from a common depot. Both the buses travel or south-east from the depot, then the stoppage
through different stoppages. is named as Q.
Bus X started moving from depot towards south Condition III: If the stoppage is either north-west
for 8m to reach stoppage 1. Then it turns towards or south west from the depot, then the stoppage
left and travels for 12m to reach stoppage S2. is named as R.
Then from stoppage S2, it again turns towards Condition IV: If the distance between the
the left and travels for 16m to reach stoppage consecutive stoppages reached by bus is
S3. Stoppage S4 is 18m west of stoppage S3. multiple of 4, then “S” is named to the first
Then, it again takes a right turn and travels for stoppage reached by the bus.
16m to reach stoppage S5. Stoppage S6 is 24m 1) If bus A travels for 12m towards the north from
east of stoppage S5. the depot to reach stoppage 14, then the
Bus Y started moving from depot towards east distance between stoppages S14 and S9 is two
for 14m to reach stoppage S7. Then it turns more than the distance between stoppages
towards the left and travels for 12m to reach __and__.
stoppage S8. Then from stoppage S8 it takes a a) S8 and S9
right turn and travels for 12m to reach stoppage b) S5 and S6
S9. Then, it again takes a right turn and travels c) Depot and S11
for 8m to reach stoppage S10. Stoppage S11 is d) S11 and S12
8m west of stoppage S10. From stoppage S11, it e) S6 and S7
takes a left turn and travels for 18m to reach
Stoppage S12. The stoppage S13 is 6m west of 2) What is the shortest distance and direction of
stoppage S12. S11 with respect to S2?
Note: a) 5m, South-east
I. Stoppage 1 is named as S1 similarly for others. b) 3√7m, North-west
II. Depot is not considered as stoppage. c) 4√3m, North
These stoppages are assigned with different d) 6√5m, North-East
names according to the below given conditions. e) 7m, South-west
Condition I: If the distance between the
consecutive stoppages reached by bus is 3) What is the direction of S13 with respect to
Depot?

Click Here For Bundle PDF Course | support@guidely.in Page 1 of 11


Bank Po Mains PDF Course 2024
Reasoning Day -11 (Eng)

a) Northeast “P$Q” means “P is neither greater than nor equal


b) West to Q”.
c) Southeast “P#Q” means “P is neither less than nor equal to
d) Southwest Q”.
e) None of these “P&Q” means “P is neither less than nor greater
than Q”.
4) Stoppages S6 and S10 are assigned with __ 6) Statements:
and __ names respectively. P#L%M#X, L@H$T, H#W@S
a) PQ and QS Conclusion:
b) RS and PQS I. S$P
c) QS and RS II. M@H
d) Q and SQ III. T#X
e) RPQ and PRS a) Both conclusions I and III are true
b) Only conclusion II is true
5) P is assigned to which of the following c) Both conclusions I and II are true
stoppages? d) Only conclusion III is true
I. S2, S6, S13 e) None is true
II. S6, S4, S7
III. S8, S9, S12 7) Statements:
a) Only III W#U%F$T, F$M&V%X, U@G$S@B
b) Both II and III Conclusion:
c) Both I and III I. X@U
d) Only I II. T#G
e) All I, II and III III. V$W
a) Both conclusions I and III are true
Directions (6-10): In the given questions, certain b) Only conclusion III is true
symbols have been used to indicate the c) Both conclusions I and II are true
relationships between different elements in the d) Only conclusion II is true
statements followed by some conclusions as e) None is true
follows. Find the conclusion(s) which is/are
definitely true. 8) Statements:
“P@Q” means “P is not less than Q”. G$T%S#M@Z, M$W%Q#P, Q&R$L
“P%Q” means “P is not greater than Q”. Conclusion:
I. Q#G

Click Here For Bundle PDF Course | support@guidely.in Page 2 of 11


Bank Po Mains PDF Course 2024
Reasoning Day -11 (Eng)

II. Z$R Directions (11-15): Study the following


III. G@R information carefully and answer the given
a) Either conclusion I or III is true questions.
b) Only conclusion III is true Ten persons – B, D, F, G, I, L, M, P, T, and Y are
c) Either conclusion I or III and conclusion II is sitting in two circular tables, such that one
true circular table is inscribed in another circular
d) Only conclusion II is true table. Five persons are sitting in the outer circular
e) All conclusions I, II and III are true table and facing towards the centre whereas five
persons are sitting in the inner circular table and
9) Statements: facing away from the centre. There is an equal
Q%B$W, K$W#U&T$R, T@Z#M%S distance between the adjacent persons and each
Conclusion: person in the inner circular table faces exactly
I. T#Q one person in the outer circular table.
II. U%S Note: If X faces Y, then both are sitting at
III. R#K different circular tables.
a) Both conclusions II and III are true M sits second to the left of the one who faces D,
b) Only conclusion III is true who sits second to the right of Y. F sits third to
c) Both conclusions I and III are true the left of T. Y does not face T. F sits adjacent to
d) All conclusions I, II, and III are true neither Y nor D. The number of persons sitting
e) None is true between F and B is one less than the number of
persons sitting between P and D, when counted
10) Statements: from the left of both P and F. B and M are not
M#S%K$H, Z#H@L$F%G, F$T%E#Q immediate neighbours. One person sits between
Conclusion: I and L, who faces away from the centre. The
I. G#Q one who faces G sits immediate left of L.
II. S$Z Final arrangement:
III. E@L Now, the persons changed their position with
a) Only conclusion II is true respect to the closest previous alphabetical
b) Only conclusion III is true name of the person. The vacant seat at the last
c) Both conclusions I and III are true will be occupied by B.
d) All conclusions I, II, and III are true 11) What is the position of B with respect to M in
e) None is true the final arrangement?
a) Third to the left
b) Immediate right

Click Here For Bundle PDF Course | support@guidely.in Page 3 of 11


Bank Po Mains PDF Course 2024
Reasoning Day -11 (Eng)

c) Second to the right 15) If Y is related to T and F is related to G, then


d) Immediate left who among the following person is related to B in
e) Fourth to the left the final arrangement?
a) L
12) __ sits __ to the right of __ in the final b) The one who sits second to the left of D
arrangement. c) M
a) P, Third, G d) The one who faces I
b) F, second, L e) Either a or d
c) T, Second, F
d) Y, third, I Directions (16-20): Study the following
e) None of these information carefully and answer the given
questions.
13) Who among the following persons are sitting Twelve persons are sitting in a linear row and
between M and T when, counted from the left of facing north. Each person has some number of
T in the final arrangement? coins which is shown with the help of inequality.
I. L T=P≥L>R<M=U≤B<F≥Q=S≤Z<G
II. G Note: If it is given that A has definitely either
III. F equal or more number of coins that B, then it
a) Only III should be consider as A≥B.
b) Both II and III T sits fourth to the left of the one who has
c) Both I and III definitely less number of coins than T. At-most
d) All I, II, and III one person sits to the right of R. Only one person
e) Only I sits between R and S. The one who has
definitely either equal or more number of coins
14) Who among the following person sits second than S sits third to the left of S. Only three
to the right of G in the final arrangement? persons sit between M and Q, who is an
a) The one who faces T immediate neighbour of S. Only one person sits
b) Y between Q and the one who has definitely more
c) The one who sits second to the right of Y number of coins than Q. The one who has either
d) L equal or more number of coins than M sits
e) M second to the left of M. P and F don’t sits
adjacent to each other. As many persons sit
between B and P as between R and Q. Only one
person sits between U and the one who has

Click Here For Bundle PDF Course | support@guidely.in Page 4 of 11


Bank Po Mains PDF Course 2024
Reasoning Day -11 (Eng)

definitely more number of coins than U. At least b) UTM


four persons sit between L and F, who doesn’t sit c) GBZ
at the end. d) RST
16) What is the position of B with respect to P? e) FPM
a) Third to the right
b) Second to the right 19) What is the sum of the number of persons sit
c) Immediate left to the right of B and the number of persons
d) Fourth to the left sitting between M and U?
e) Third to the left a) 20
b) 9
17) How many persons are sitting between L and c) 12
Q? d) 15
a) One e) 10
b) Three
c) Four 20) The number of persons sitting between Z
d) Two and S is __the number of persons sitting
e) None of these between R and P.
a) Three less than
18) In which of the following option, third person b) same as
is not sitting exactly between the first and second c) Two more than
persons? d) One more than
a) PST e) Two less than
Click Here to Get the Detailed Video Solution for the above given Questions
Or Scan the QR Code to Get the Detailed Video Solutions

Answer Key with Explanation


Directions (1-5): 1) Answer: B

Click Here For Bundle PDF Course | support@guidely.in Page 5 of 11


Bank Po Mains PDF Course 2024
Reasoning Day -11 (Eng)

2) Answer: D 8) Answer: C
3) Answer: C G<T≤S>M≥Z; M<W≤Q>P; Q=R<L
4) Answer: A I. Q > G (R = Q ≥ W > M < S ≥ T > G)  False
5) Answer: C II. Z < R (Z ≤ M < W ≤ Q = R)  True
Final arrangement III. G ≥ R (R = Q ≥ W > M < S ≥ T > G)  False
9) Answer: E
Q≤B<W; K<W>U=T<R; T≥Z>M≤S
I. T > Q (T=U<W>B≥Q)  False
II. U ≤ S (U=T≥Z>M≤S)  False
III. R > K (R>T=U<W>K)  False

10) Answer: A
M>S≤K<H; Z>H≥L<F≤G; F<T≤E>Q
I. G > Q (G≥F<T≤E>Q)  False
II. S < Z (S≤K<H<Z)  True
III. E ≥ L (E≥T>F>L)  False

Directions (6-10): Directions (11-15):


11) Answer: D
12) Answer: C
13) Answer: C
6) Answer: C 14) Answer: A
P>L≤M>X; L≥H<T; H>W≥S 15) Answer: B
I. S < P (S≤W<H≤L<P)  True Final arrangement
II. M ≥ H (M≥L≥H)  True
III. T > X (T>H≤L≤M>X)  False

7) Answer: D
W>U≤F<T; F<M=V≤X; U≥G<S≥B
I. X ≥ U (X≥V=M>F≥U)  False
II. T > G (T>F≥U≥G)  True
III. V < W (V=M>F≥U<W)  False

Click Here For Bundle PDF Course | support@guidely.in Page 6 of 11


Bank Po Mains PDF Course 2024
Reasoning Day -11 (Eng)

We have:
 M sits second to the left of the one who
faces D, who sits second to the right of Y.
From the above conditions, we have two
possibilities.

Again, we have
 F sits third to the left of T.
 Y does not face T.
 F sits adjacent to neither Y nor D.
 The number of persons sitting between F
and B is one less than the number of
persons sitting between P and D, when
counted from the left of both P and F.
 B and M are not immediate neighbours.
After applying the above conditions extra two
possibilities are added for case-1 and case-2

Click Here For Bundle PDF Course | support@guidely.in Page 7 of 11


Bank Po Mains PDF Course 2024
Reasoning Day -11 (Eng)

Again, we have
 One person sits between I and L, who
faces away from the centre.
 The one who faces G sits immediate left
of L.
After applying the above conditions case-2 and
case-2a get eliminated because there is no
possibility to place I and L in inner circle (facing
away from the centre) and case 1 gets

Click Here For Bundle PDF Course | support@guidely.in Page 8 of 11


Bank Po Mains PDF Course 2024
Reasoning Day -11 (Eng)

eliminated, because the one who faces G sits


immediate left of P is not possible.

Now, the persons changed their position with


respect to the closest previous alphabetical
name of the person. The vacant seat at the last
will be occupied by B.
This shows the final arrangement

Click Here For Bundle PDF Course | support@guidely.in Page 9 of 11


Bank Po Mains PDF Course 2024
Reasoning Day -11 (Eng)

Thus, two persons F & Z have definitely either


equal or more coins than S.
From the above conditions, there are four
possibilities

Directions (16-20):
16) Answer: E
17) Answer: B
18) Answer: D Again, we have
19) Answer: C  Only three persons sit between M and Q,
20) Answer: A who is an immediate neighbour of S.
Final arrangement  One person sits between Q and the one
who has definitely more number of coins
than Q.
G has definitely more number of
coins than Q.
We have,
 The one who has either equal or more
T=P≥L>R<M=U≤B<F≥Q=S≤Z<G
number of coins than M sits second to the
 T sits fourth to the left of the one who has
left of M.
definitely lesser number of coins than T.
Only B has either equal or more
Only R has definitely lesser number of coins
number of coins than M.
than T.
 P and F don’t sits adjacent to each other.
 At-most one person sits to the right of R.
 As many persons sit between B and P as
 Only one person sits between R and S.
between R and Q.
 The one who has definitely either equal or
more number of coins than S sits third to
the left of S.

Click Here For Bundle PDF Course | support@guidely.in Page 10 of 11


Bank Po Mains PDF Course 2024
Reasoning Day -11 (Eng)

After applying the above conditions case-1a and  Only one person sits between U and the
case-2a get eliminated because P and F sit one who has definitely more number of
adjacent to each other. coins than U.
Only F has more number of coins
than U.
 At least four persons sit between L and F,
who doesn’t sit at the end.
After applying the above conditions case-1b gets
eliminated because there is no possibility to
place L, hence case-2b shows the final
arrangement.

Again, we have

Click Here For Bundle PDF Course | support@guidely.in Page 11 of 11


Bank Po Mains PDF Course 2024
Quantitative Aptitude Day -11 (Eng)

Quantitative Aptitude

1) In a mixture of milk and water, the quantity of efficiencies of A, B and C are respectively 20%,
milk is 20 L more than that of water and the cost 30% and 50% of the efficiency of D, then find the
of pure milk is ₹ 48 per L. If the cost of pure milk value which can be filled in the blank?
were 12.5% less, then the cost of the mixture a) Can’t be determined
was ₹ 4.5 per L less than the original cost of the b) 5 days
mixture. When 8 L quantity of this mixture is c)3 days
replaced with the same quantity of milk, then d) 8 days
what will be the cost of the new mixture? e) 6 days
a) ₹ 36.4 per L
b) ₹ 40.4 per L 4) A boat can cover ___ m downstream in
c) ₹ 38.4 per L 25seconds and ___ m upstream in 35 seconds.
d) ₹ 42.4 per L The time taken by the boat to go 540 m
e) ₹ 34.4 per L downstream is 20% more than that taken by the
boat to go 300 m upstream. If the speed of the
2) Ratio of the difference between compound boat in still water were 20% more, then its
interest and simple interest on ₹ X at R% rate downstream speed was 21 m/s.
after 2 years to the difference between Quantity I: Find the value which will be filled in
compound interest and simple interest on ₹ X at the 1st blank?
R% rate after 3 years is 25: 76. If the difference Quantity II: Find the value which will be filled in
between compound interest and simple interest the 2nd blank?
on ₹ (X – 7000) at (R + 1)% rate after 2 years is a) Quantity I > Quantity II
₹ 45, then find the simple interest received on b) Quantity I = Quantity II or the relation can’t be
at (R + 4)% rate after 2 years? established
a) ₹ 1250 c)Quantity I ≤ Quantity II
b) ₹ 2500 d) Quantity I > Quantity II
c)₹ 1750 e) Quantity I ≥ Quantity II
d) ₹ 1500
e) ₹ 2000 5) Average of present ages of A, B and C is 23
years and A’s present age is ___% of D's present
3) Workers A, B and C together can complete a age. After 6 years, the ratio of A’s age to B’s age
work in 9 days and workers B, C and D together will be 2: 3 and C’s present age is 70% of B’s
can complete the same work in ___ days. If the

Click Here For Bundle PDF Course | support@guidely.in Page 1 of 16


Bank Po Mains PDF Course 2024
Quantitative Aptitude Day -11 (Eng)

present age. The difference between the seconds and (t + 2) seconds respectively. The
present ages of B and D is 10 years. speed of the car is 50% more than that of the
Quantity I: Find the value which will be filled in bike and also 4 m/s more than that of the bus.
the blank? Quantity I: If the speed of the bus were ___% of
Quantity II:A’s present age is what per cent of its original speed, it would have crossed the
B’s present age? bridge in 16 seconds.
a) Quantity I > Quantity II Quantity II:If the speed of the bike were ___% of
b) Quantity I = Quantity II or the relation can’t be its original speed, it would have crossed the
established bridge in 20 seconds.
c)Quantity I ≤ Quantity II a) Quantity I > Quantity II
d) Quantity I > Quantity II b) Quantity I ≤ Quantity II
e) Quantity I ≥ Quantity II c)Quantity I > Quantity II
d) Quantity I = Quantity II or the relation can’t be
6)Time taken by a car, a bike and a bus to cross established
a 240 m long bridge is ‘t’ seconds, (t + 5) e) Quantity I ≥ Quantity II

Direction (7-10): Study the following data carefully and answer the questions:
There are three tanks P, Q and R of different capacities and each tank has an inlet pipe and an outlet
pipe connected to it.
Inlet pipe A and outlet pipe M are connected to tank P.
Inlet pipe B and outlet pipe N are connected to tank Q.
Inlet pipe C and outlet pipe O are connected to tank R.
The table given below shows the following data:

Note:
1) The ratio of the efficiency of pipe M to that of pipe N is 3: 2 and the ratio of the efficiency of the pipe M
to that of pipe O is 4: 3.

Click Here For Bundle PDF Course | support@guidely.in Page 2 of 16


Bank Po Mains PDF Course 2024
Quantitative Aptitude Day -11 (Eng)

7) The flowing rate of water from pipe M is 4.5 L 9) If the ratio between capacities of tanks P, Q
per minute, which is 90% of the flowing rate of and R is 7: 6: 8 respectively, the difference
water from pipe N. If the flowing rate of water between the capacities of tanks P and Q is 30 L
from pipe A is ‘x’ L per minute and that from pipe and only pipes A, B and C are connected to tank
B is ‘y’ L per minute, then find the value of ? R, then in what time tank R will be filled?
a) 5.4 a) 8 minutes
b) 6.8 b) 12 minutes
c)5.6 c)10 minutes
d) 7.5 d) 6 minutes
e) 4.9 e) 15 minutes

8) If pipes A and M work with 125% and of 10) Which of the following is/are true?

their respective efficiencies and pipes B and N X: The average time, in which pipe A alone, pipe

work with 125% and 93.75% of their respective B alone and pipe C alone can fill tanks P, Q and

efficiencies, then find that the time, in which R respectively, is 22 minutes.

pipes A and M together can fill tank P with their Y: If the capacity of tank R is 210 L, then the

new efficiencies, will be what per cent of the flowing rate of water from pipe O is 5.75 L.

time, in which pipes B and N together can fill Z: If the difference between the flowing rates of

tank Q with their new efficiencies? water from pipes B and N is 3 L per minute, then

a) 33.33% the capacity of tank Q is 300 L.

b) 40% a) Both X and Y

c)30% b) Both X and Z

d) 37.5% c) None is true

e) 44.44% d) Both Y and Z


e) All are true

Direction (11-14): Study the following data carefully and answer the questions:
The data given below is related to the total number of persons (male + female) who visited a museum
on 4 different days Mon, Tue, Wed and Thu.
The pie chart given below shows the percentage distribution of the total number of persons who visited
the museum on each day.

Click Here For Bundle PDF Course | support@guidely.in Page 3 of 16


Bank Po Mains PDF Course 2024
Quantitative Aptitude Day -11 (Eng)

The Pie chart given below shows the percentage distribution of the number of males, who visited the
museum on each day.

Note:
1) The number of females, who visited the museum on Tue, is 110, which is 175% more than the
females , who visited the museum on Mon.
11) The Cost of ticket for adults is ₹ 50 per adult child. If on Mon, 45% of the total persons were
and the cost of the ticket for children is ₹ 20 per adults and on Wed, 65% of the total persons

Click Here For Bundle PDF Course | support@guidely.in Page 4 of 16


Bank Po Mains PDF Course 2024
Quantitative Aptitude Day -11 (Eng)

were adults, then find the difference between the a) Only B


total amount collected by the museum by selling b) Only B and C
adult’s tickets on Mon and Wed together and that c) Only A and B
collected by selling children’s tickets on Mon and d) Only A and C
Wed together? e) Only C
a) ₹ 9100
b) ₹ 9900 14) If 40% of total males, who visited the
c) ₹ 9500 museum on Mon, were children, 20% of total
d) ₹ 9000 males, who visited the museum on Tue, were
e) ₹ 9300 children, 30% of total males, who visited the
museum on Wed, were children and 45% of total
12)If 74% of the total males, who visited the males, who visited the museum on Thu, were
museum on Tue, were Indian and the remaining children, then find the average of the number of
were foreigners and 90% of the total females, male children, who visited the museum on each
who visited the museum on Tue, were Indian and of the given days?
the remaining were foreigners, then find what a) 40
per cent of the total number of persons, who b) 55
visited the museum on Tue, were foreigners? c) 45
a) 12.5% d) 50
b) 25% e) 60
c) 20%
d) 16.67% Direction (15-18): Study the following data
e) 15% carefully and answer the questions:
The data given below is related to the number of
13) If the number of females, who visited the persons, who took membership in three different
museum on each day, is described in a circle sports (Tennis, Swimming, and Golf) in a sports
and the degree distribution of the number of club. Some took membership in only one sport,
females, who visited the museum on Wed and some took membership in only two sports and
Thu are M° and N° respectively, then find that the remaining took membership in all three
which of the following is/are true? sports.
A: Both M and N are multiple of 24. The number of people, who took membership in
B: The Difference of M and N is a multiple of 12. swimming only, is 80% of those, who took
C:The Value of is 2.5. membership in Tennis only and also 2 more than
those, who took membership in Golf only. The

Click Here For Bundle PDF Course | support@guidely.in Page 5 of 16


Bank Po Mains PDF Course 2024
Quantitative Aptitude Day -11 (Eng)

ratio of the number of persons, who took a) ₹ 2290


membership in both Tennis and Swimming but b) ₹ 2490
not Golf to those, who took membership in both c) ₹ 2590
Tennis and Golf but not Swimming, is 2: 3 and d) ₹ 2190
the number of persons, who took membership in e) ₹ 2390
both Swimming and Golf but not Tennis, is 3
more than those, who took membership of both 17) If the total number of persons, who took
Tennis and Golf but not Swimming. The number membership of only one sports, is M% of the
of persons, who took membership of both total number of persons in the sports club and
Swimming and Golf but not Tennis, is 60% of the total number of persons, who took
those, who took membership of Golf only. The membership of only two sports, is N% of the total
number of persons, who took membership in all number of persons in the sports club, then find
the three sports, is 5, which is 12.5% of those, thatthe value of (2M – 3N) is divisible by which of
who took membership in Tennis only. the following?
15) If the ratio of males to females, who took a) 10
membership of Tennis only, is 11: 9, the ratio of b) 5
males to females, who took membership of c) 15
Swimming only, is 5: 3 and the ratio of males to d) Both (a) and (b)
female, who took membership of Golf only, is 1: e) Both (b) and (c)
2, then find the ratio of total males to total
females, who took membership of only one 18) If 56% of the total number of persons in the
sport? sports club are male, out of which, took
a) 52: 51 membership of at-least two sports, then find that
b) 4: 3 what per cent of total females took membership
c) 104: 103 of only one sport?
d) 13: 12 a) 63.63%
e) 26: 25 b) 57.5%
c) 66.67%
16)If the cost of a Swimming membership is d) 53.33%
₹ 150 per person and the cost of a Golf e) 60%
membership is ₹ 180 per person, then find the
difference between the amount collected by the Direction (19-20): Study the following data
sports club by giving Swimming membership and carefully and answer the questions:
that collected by giving Golf membership?

Click Here For Bundle PDF Course | support@guidely.in Page 6 of 16


Bank Po Mains PDF Course 2024
Quantitative Aptitude Day -11 (Eng)

I: e) Only A and B
II:
III: 20) Find the roots of the equation: px2 + qx + (r –

IV: b 3) = 0, where:

19) If the sum of bigger roots of all the four p = Difference between smaller roots of

equations together is ‘P’, then which of the equations I and III.

following regarding P is TRUE? q = Sum of smaller roots of equations II and IV.

A: ‘P’ is a prime number. r = Smaller root of equation III.

B: ‘P + 2’ is a prime number. a) 4 and 2

C: ‘P – 2’ is a prime number. b) -4 and -2

a) Only A c) -4 and 2

b) Only A and C d) 4 and -2

c) Only C e) None of these

d) Only B and C
Click Here to Get the Detailed Video Solution for the above given Questions
Or Scan the QR Code to Get the Detailed Video Solutions

Answer Key with Explanation


1) Answer: C The new cost of the mixture will also be
Let the initial quantity of water = ‘x’ L 12.5%less than the original cost of the mixture.
So, the initial quantity of milk = (x + 20) L So, the original cost of the mixture = y =
Let the initial cost of the mixture = ₹ ‘y’ per L = ₹ 36 per L
So, From equation (1):

If the cost of pure milk were 12.5% less: 4x + 80 = 6x + 60


x = 10

Click Here For Bundle PDF Course | support@guidely.in Page 7 of 16


Bank Po Mains PDF Course 2024
Quantitative Aptitude Day -11 (Eng)

Initial quantity of water = 10 L Now, the SI received on at (R + 4)% rate


Initial quantity of milk = (10 + 20) = 30 L after 2 years:
Ratio of milk to water in the mixture = 30: 10 = 3:
1
When 8 L quantity of the mixture is replaced with
the same quantity of milk: 3) Answer: B
Let the time, in which D alone can complete the
The new quantity of milk in the mixture =
work is ‘3t’ days.
= 32 L
So, the time, in which A alone can complete the
And the new quantity of water in the mixture = 40
work = = ‘15t’ days
– 32 = 8 L
So, the new cost of the mixture = = ₹ 38.4 The time, in which B alone can complete the
work = = ‘10t’ days
per L
And the time, in which C alone can complete the

2) Answer: E work = = ‘6t’ days

Since, the difference between CI and SI after 2 Since, A, B and C together can complete the

years: work in 9 days.


So,

And the difference between CI and SI after 3


years:
t=3
Now, the time, in which B alone can complete
So,
the work = 10 * 3 = 30 days
The time, in which C alone can complete the
work = 6 * 3 = 18 days
And the time, in which D alone can complete the

R = 4% work = 3 * 3 = 9 days

Since, the difference between compound interest Since, the part of work completed by B, C and D

and simple interest on ₹ (X – 7000) at (R + 1)% together in 1 day:

rate after 2 years is ₹ 45.


So, So, the time, in which B, C and D together can
complete the work = 5 days

X = 25000
4) Answer: A

Click Here For Bundle PDF Course | support@guidely.in Page 8 of 16


Bank Po Mains PDF Course 2024
Quantitative Aptitude Day -11 (Eng)

Let the speed of the boat in still water is ‘5x’ m/s Since, the average of present ages of A, B and
and the speed of the stream is ‘y’ m/s. C is 23 years.
Since, the time taken by the boat to go 540 m So,
downstream is 20% more than that taken by the A + B + C = 69 -------------(1)
boat to go 300 m upstream. Since, the ratio of A’s age to B’s age after 6
So, years will be 2: 3.
So,

3A + 18 = 2B + 12
15x – 3y = 10x + 2y
2B – 3A = 6 -------------(2)
x = y ------------(1)
By equation (1) * 3 + equation (2):
If the speed of the boat in still water were 20%
3A + 3B + 3C + 2B – 3A = 207 + 6
more:
5B + 3C = 213 -------------(3)
The new speed of boat in still water = 120% of
Since, C’s present age is 70% of B’s present
5x = ‘6x’ m/s
age.
And
So,
6x + y = 21
From equation (1):
6x + x = 21 From equation (1):

x = 3, y = 3
The speed of the boat in still water = 5 * 3 = 15 B = 30
m/s From equation (2):
The speed of the stream = 3 m/s A = 18
Quantity I: From equation (1):
The downstream distance covered by the boat in C = 21
25 seconds: Since, the difference between B’s present age
(15 + 3) * 25 = 450 m and D’s present age is 10 years.
Quantity II: So, D’s present age will be either 30 + 10 = 40
The upstream distance covered by the boat in 35 years or 30 – 10 = 20 years.
seconds: Quantity I:
(15 – 3) * 35 = 420 m A’s present age = 18 years
Hence Quantity I > Quantity II If D’s present age is 40 years:
So, the value, which will be filled in the blank =
5) Answer: B = 45%

Click Here For Bundle PDF Course | support@guidely.in Page 9 of 16


Bank Po Mains PDF Course 2024
Quantitative Aptitude Day -11 (Eng)

If D’s present age is 20 years: So, the new speed of the bus = = 15 m/s
So, the value, which will be filled in the blank = Since, the original speed of the bus = 20 m/s
= 90% So, the value which will be filled in the blank =
Quantity II: = 75%
A’s present age = 18 years Quantity II:
B’s present age = 30 years The new time, in which the bike will cross the
Required percentage = = 60% bridge = 20 seconds
Hence, the relation can’t be established. So, the new speed of the bike = = 12 m/s
Since, the original speed of the bike = 16 m/s
6) Answer: D So, the value which will be filled in the blank =
Let the speed of the bike = ‘2x’ m/s = 75%
So, the speed of the car = 150 of ‘2x’ = ‘3x’ m/s Hence, Quantity I = Quantity II
And the speed of the Bus = (3x – 4) m/s
Since, the time taken by the car to cross the Direction (7-10):
bridge is ‘t’ seconds. Let the time, in which pipe M alone can empty
So, tank P is ‘6m’ minutes.
So, the time, in which pipe N alone can empty
Since, the time taken by the bike to cross the tank Q = = ‘9m’ minutes
bridge is (t + 5) seconds. And the time, in which pipe O alone can empty
So, tank R = = ‘8m’ minutes
Since, the time, in which pipes A and M together

From equations (1) and (2): can fill tank P is 30 minutes.


So,

2t + 10 = 3t
t = 10 Since, the time, in which pipes B and N together

From equation (1): can fill tank Q, is 90 minutes.

x=8 So,

Speed of the bike = 2 * 8 = 16 m/s


Speed of the car = 3 * 8 = 24 m/s From equations (1) and (2):
Speed of the bus = 24 – 4 = 20 m/s
Quantity I:
The new time, in which the bus will cross the
bridge = 16 seconds

Click Here For Bundle PDF Course | support@guidely.in Page 10 of 16


Bank Po Mains PDF Course 2024
Quantitative Aptitude Day -11 (Eng)

m=5 8) Answer: D
From equation (1): The time, in which pipe A alone can fill tank P
with 125% of its original efficiency = =

t = 15 12 minutes
The time, in which pipe M alone can empty tank
P with of its original efficiency = =
36 minutes
Now, the part of tank P filled by pipes A and M
together in 1 minute:

So, the time, in which pipes A and M together

7) Answer: B will fill tank P = 18 minutes

Since, the flowing rate of water from pipe M is The time, in which pipe B alone can fill tank Q

4.5 L per minute. with 125% of its original efficiency = =

And the time, in which pipe M alone can empty 24 minutes

tank P = 30 minutes The time, in which pipe N alone can empty tank

So, the capacity of tank P = 4.5 * 30 = 135 L Q with 93.75% of its original efficiency =

Since, the time, in which pipe A alone can fill = 48 minutes

tank P = 15 minutes Now, the part of tank Q filled by pipes B and N


So, the flowing rate of water from pipe A = x = together in 1 minute:
= 9 L per minute
Since, the flowing rate of water from pipe M = So, the time, in which pipes B and N together will
= 5 L per minute fill tank Q = 48 minutes
And the time, in which pipe N alone can empty Required percentage = = 37.5%
tank Q = 45 minutes
So, the capacity of tank Q = 5 * 45 = 225 L 9) Answer: A
Since, the time, in which pipe B alone can fill Since, the difference between the capacities of
tank Q = 30 minutes tanks P and Q is 30 L.
So, the flowing rate of water from pipe B = y = So, the capacity of tank P = = 210 L
= 7.5 L per minute The capacity of tank Q = = 180 L
So, the value of = = 6.8 And the capacity of tank R = = 240 L

Click Here For Bundle PDF Course | support@guidely.in Page 11 of 16


Bank Po Mains PDF Course 2024
Quantitative Aptitude Day -11 (Eng)

Now, the flowing rate of water from pipe A = Direction (11-14):


= 14 L per minute From the 1st pie chart:
The flowing rate of water from pipe B = =6L X + (Y + 20) = 100 – 30 – 15
per minute X + Y = 35 -------------(1)
And the flowing rate of water from pipe C = = From the 2nd pie chart:

10 L per minute
So, the time, in which pipes A, B and C together X + 4Y = 80 -------------(2)
will fill tank R: From equations (1) and (2):
35 – Y = 80 – 4Y
Y = 15

10) Answer: C From equation (1):

From X: X = 20

Average of time, in which pipe A alone, pipe B


alone and pipe C alone can fill tanks P, Q and R
respectively:

So, X is not true.


From Y:
If the capacity of tank R is 210 L:
Let the total number of persons, who visited the
And the time, in which pipe O alone can empty
museum on Mon, Tue, Wed and Thu are 30t,
tank R = 40 minutes
20t, 35t and 15t respectively.
So, the flowing rate of water from pipe O = =
Also, let the number of males, who visited the
5.25 L per minute
museum on Mon, Tue, Wed and Thu are 40m,
So, Y is not true.
10m, 30m and 20m respectively.
From Z:
Since, the number of females, who visited the
If the difference between the flowing rates of
museum on Tue, is 110.
water from pipes B and N is 3 L per minute:
So,
And pipes B and N together can fill tank Q in 90
20t – 10m = 110
minutes.
2t – m = 11 ----------------(3)
So, the capacity of tank Q = 3 * 90 = 270 L
Since, the number of females, who visited the
So, Z is not true.
museum on Tue, is 175% more than the
Hence, none is true.
females, who visited the museum on Mon.

Click Here For Bundle PDF Course | support@guidely.in Page 12 of 16


Bank Po Mains PDF Course 2024
Quantitative Aptitude Day -11 (Eng)

So, Total females, who visited the museum on Tue =


110

3t – 4m = 4 ---------------- (4) So, the number of female foreigners, who visited

From equations (3) and (4): the museum on Tue = 10% of 110 = 11

t = 8, m = 5 Total number of foreigners, who visited the


museum on Tue = 13 + 11 = 24
Total number of persons, who visited the
museum on Tue = 160
Required percentage = = 15%

13) Answer: A
11) Answer: B Total number of females, who visited the
Total number of persons, who visited the museum in all the 4 days together = 40 + 110 +
museum on Mon = 240 130 + 20 = 300
So, the number of adults in the museum on Mon So, M = = 156°
= 45% of 240 = 108 And, N = = 24°
And the number of children in the museum on From A:
Mon = 240 – 108 = 132 Factors of M = 156 = 12 * 13
Total number of persons, who visited the Factors of N = 24 = 12 * 2
museum on Wed = 280 Since, both M and N are multiple of 12, but not
So, the number of adults in the museum on Wed 24.
= 65% of 280 = 182 So, A is not true.
And the number of children in the museum on From B:
Wed = 280 – 182 = 98 Difference of M and N = 156 – 24 = 132
Required difference = (108 + 182) * 50 – (132 + Since, 132 is a multiple of 12.
98) * 20 = ₹ 9900 So, B is true.
From C:
12) Answer: E Value of = = 1.5
Total males, who visited the museum on Tue = So, C is not true.
50 Hence, only B is true.
So, the number of male foreigners, who visited
the museum on Tue = 26% of 50 = 13 14) Answer: C

Click Here For Bundle PDF Course | support@guidely.in Page 13 of 16


Bank Po Mains PDF Course 2024
Quantitative Aptitude Day -11 (Eng)

The number of male children, who visited the


museum on Mon = 40% of 200 = 80
The number of male children, who visited the
museum on Tue = 20% of 50 = 10
The number of male children, who visited the
museum on Wed = 30% of 150 = 45
The number of male children, who visited the
museum on Thu = 45% of 100 = 45 15) Answer: E
Required average = = 45 Number of males, who took membership in
Tennis only = = 22
Direction (15-18): Number of females, who took membership in
Number of persons, who took membership in all Tennis only = 40 – 22 = 18
the three sports = 5 Number of males, who took membership in
Number of persons, who took membership in Swimming only = = 20
Tennis only = = 40
Number of females, who took membership in
Number of persons, who took membership in Swimming only = 32 – 20 = 12
Swimming only = 80% of 40 = 32 Number of males, who took membership in Golf
Number of persons, who took membership in only = = 10
Golf only = 32 – 2 = 30
Number of females, who took membership in
Number of persons, who took membership in
Golf only = 30 – 10 = 20
both Swimming and Golf, but not Tennis = 60%
Required ratio = (22 + 20 + 10): (18 + 12 + 20) =
of 30 = 18
52: 50 = 26: 25
Number of persons, who took membership in
both Tennis and Golf, but not Swimming = 18 – 3
16) Answer: B
= 15
Total number of persons, who took Swimming
Number of persons, who took membership in
membership:
both Tennis and Swimming, but not Golf =
32 + 10 + 18 + 5 = 65
= 10
So, the total amount collected by the sports club
by giving Swimming membership = 65 * 150 =
₹ 9750
Total number of persons, who took Golf
membership:
30 + 15 + 18 + 5 = 68

Click Here For Bundle PDF Course | support@guidely.in Page 14 of 16


Bank Po Mains PDF Course 2024
Quantitative Aptitude Day -11 (Eng)

So, the total amount collected by the sports club So, the number of females, who took
by giving Golf membership = 68 * 180 = ₹ 12240 membership in only one sport = 102 – 60 = 42
Required difference = 12240 – 9750 = ₹ 2490 Required percentage = = 63.63%

17) Answer: D Direction (19-20):


Total number of persons in the sports club: I:
40 + 32 + 30 + 10 + 18 + 15 + 5 = 150
Total number of persons, who took membership
a2 + 5a – 66=0
of only one sport:
a2 + 11a – 6a – 66 0
40 + 32 + 30 = 102
a (a + 11) – 6 (a + 11) = 0
So, M = = 68%
(a + 11) (a – 6) = 0
Total number of persons, who took membership
a = -11 and 6
of only two sports:
II:
10 + 18 + 15 = 43
So, N =
So, the value of (2M – 3N) = (136 – 86) = 50 2b2 – 3b – 9 = 0
[Which is divisible by both 10 and 5 but not by 2b2 – 6b + 3b – 9 = 0
15]. 2b (b – 3) + 3 (b – 3) = 0
(b – 3) (2b + 3) = 0
18) Answer: A b = 3 and -3/2
Total number of members in the sports club: III:
40 + 32 + 30 + 10 + 18 + 15 + 5 = 150
So, the total number of males in the sports club 3c2 – 4c = 221 + 2c2
= 56% of 150 = 84 c2 – 4c – 221 = 0
And the total number of females in the sports c2 – 17c + 13c – 221 = 0
club = 150 – 84 = 66 c (c – 17) + 13 (c – 17) = 0
Total number of males, who took membership in (c – 17) (c + 13) = 0
at-least two sports = of 84 = 24 c = 17 and -13
So, the number of males, who took membership IV:
in only one sport = 84 – 24 = 60 3d2 – d2 = d + 15
Total number of persons, who took membership 2d2 – d – 15 = 0
in only one sport = 40 + 32 + 30 = 102 2d2 – 6d + 5d – 15 = 0
2d (d – 3) + 5 (d – 3) = 0

Click Here For Bundle PDF Course | support@guidely.in Page 15 of 16


Bank Po Mains PDF Course 2024
Quantitative Aptitude Day -11 (Eng)

(d – 3) (2d + 5) = 0 p = Difference between smaller roots of


d = 3 and -5/2 equations I and III
p = -11 – (-13)
p=2
q = Sum of smaller roots of equations II and IV
q = (-3/2) + (-5/2) = (-3 – 5)/2 =
q = -4
r = Smaller root of equation III
r = -13
Now equation:
19) Answer: E
px2 + qx + (r – 3) = 0
Bigger root of equation I = 6
2x2 + (-4)x + (-13 – 3) = 0
Bigger root of equation II = 3
2x2– 4x – 16 = 0
Bigger root of equation III = 17
x2– 2x – 8 = 0
Bigger root of equation IV = 3
x2– 4x + 2x – 8 = 0
Required sum = P = 6 + 3 + 17 + 3 = 29
x (x – 4) + 2 (x – 4) = 0
P + 2 = 31 and P – 2 = 27
(x – 4) (x + 2) = 0
We can clearly see that, P and ‘P + 2’ are prime
x = 4 and -2
numbers while ‘P – 2’ is not a prime number.

20) Answer: D

Click Here For Bundle PDF Course | support@guidely.in Page 16 of 16


Bank Po Mains PDF Course 2024
English Day - 11

English Language
Directions (1-5): Given below are a few 1. Russia said a. while the Kremlin
questions with a table given with two columns Tuesday it had demanded the military
with highlighted connectors to be matched in deployed jets and prevent any repeat
order to make sentences contextually correct as artillery to destroy an attack.
per the rules of connectors. If none of the options armed group that
are correct then choose option E as your answer. penetrated the border
1. from Ukraine,
Column 1 Column 2 2. Moscow reported b. which has previously
1. The new outbreak a. since China Russian forces killed faced shelling attacks
could be the largest scrapped its zero- more than 70 that have killed dozens
wave of infections Covid policy in Ukrainian fighters and of people since
December 2022. destroyed four Moscow launched its
2. Aleksandr b. who the SBU armored vehicles, offensive last year.
Skachkov, arrested in 2020 during 3. Belgorod's c. but AFP was unable
a raid on people selling Governor Vyacheslav to independently verify
translated versions of Gladkov said civilians the claims.
the Christchurch were evacuated from
Shooter’s Manifesto. nine border villages in
the region,
3. Iran has expanded c. despite opposition A. 1-a, 2-b, 3-c
its missile programme, from the United States B. 1-a, 2-c, 3-b
particularly its ballistic and expressions of C. 1-c, 2-b, 3-a
missiles, concern by European D. 1-b, 2-c, 3-a
countries. E. None of the above

A. 1-a, 2-b, 3-c 3.


B. 1-a, 2-c, 3-b Column 1 Column 2
C. 1-c, 2-b, 3-a 1. The annual a. but Ireland's
D. 1-b, 2-c, 3-a assembly in Geneva regulations are intended
E. None of the above decided not to to be more
extend Taiwan an comprehensive.
2. invitation to the
Column 1 Column 2 event,

Click Here For Bundle PDF Course | support@guidely.in Page 1 of 11


Bank Po Mains PDF Course 2024
English Day - 11

2. Ireland will b. which runs from May usually not released as


introduce mandatory 21-30. legal tender,
health labelling of 3. The Battlegrounds c. However, users will
alcoholic drinks, Mobile India, which only be able to play
warning of the links has been developed the game from May
to cancer, liver by Kraft, can now be 29.
disease and the risk preloaded by all
of drinking Android users starting
3. Other countries c. while pregnant. from today, May 27.
include warnings on A. 1-a, 2-b, 3-c
alcohol products, B. 1-a, 2-c, 3-b
A. 1-a, 2-b, 3-c C. 1-c, 2-b, 3-a
B. 1-a, 2-c, 3-b D. 1-b, 2-c, 3-a
C. 1-c, 2-b, 3-a E. None of the above
D. 1-b, 2-c, 3-a
E. None of the above 5.
Column 1 Column 2
4. 1. The previous version a. which generally
Column 1 Column 2 of the game, PUBG lowers global
1. The question for a. especially given Mobile, continues to be temperatures slightly,
investors is whether to mounting concern banned in India for the world will
jump on the AI train amongst regulators similar security experience a return to
now, or exercise about the technology's reasons and its El Nino, the warmer
caution, potentially disruptive associations with counterpart, later this
impact. China. year.
2. Higher b. On the other hand, 2. Further research on b. In addition, certain
denomination lower denomination X-rays from lawmakers expressed
commemorative coins, commemorative coins supernovae is valuable their opposition to the
which have precious usually stay in not just for availability of the
metals like gold and circulation for a limited understanding the life game in India, citing
silver in them, are time. cycle of stars, concerns about its
usually treated as impact on children.
collectibles and are

Click Here For Bundle PDF Course | support@guidely.in Page 2 of 11


Bank Po Mains PDF Course 2024
English Day - 11

3. Climate models c. but also has manufacturing, utility, transportation,


suggest that after three implications for fields construction, maritime, government, information
years of the La Nina like astrobiology, technology, and education.
weather pattern in the paleontology, and the A. Only a
Pacific Ocean, earth and planetary B. Only b
sciences. C. Only c
A. 1-a, 2-b, 3-c D. Both A&B
B. 1-a, 2-c, 3-b E. No error
C. 1-c, 2-b, 3-a
D. 1-b, 2-c, 3-a 7.
E. None of the above a. Under the new Twitter Blue program rolled out
last month, individuals can pay $8 a month for a
Directions (6-10): Given below are a few blue check mark.
questions with three sentences in each of them b. Under its new owner, Elon Musk, Twitter has
which may or may not contain errors in them. allowed paid subscriptions for anyone to obtain a
You have to find the sentence which has or have verified account in exchange for a monthly
errors in them and mark your answers charge.
accordingly. If none of the sentence contain c. The finance ministry said the new Rs 75 coin
errors then choose option E as your answer. will bear the inscription of the Parliament
6. complex and have the image of the new
a. Gripped by economic turmoil and suffering a Parliament building.
balance of payments crisis, Pakistan is trying to A. Only a
reach agreement with the International Monetary B. Only b
Fund (IMF) to disburse the stalled final $1.1 C. Only c
billion from a $6.5 billion bailout agreed in 2019. D. Both A&C
b. As China has stepped up military and E. No error
diplomatic pressure in its claim to democratically
governed Taiwan, US President Joe Biden has 8.
said he would be willing to use force to defend a. Life on Earth in itself is fragile and was a result
Taiwan. of millions of coincidences coming together on
c. Microsoft said the Chinese hacking group has the planet for us to emerge and survive.
been active since at least 2021 and was targeted b. A team of explorers announced it found a
several industries including communications, sunken Japanese ship that was transporting
Allied prisoners of war when it was torpedoed off

Click Here For Bundle PDF Course | support@guidely.in Page 3 of 11


Bank Po Mains PDF Course 2024
English Day - 11

the coast of the Philippines in 1942, resulting in 10.


Australia’s largest maritime wartime loss with a a. The world's hot year on record so far was
total of 1,080 lives. 2016, coinciding with a strong El Nino - although
c. The launch pad on the Satish Dhawan Space climate change has fuelled extreme
Centre is set to be in a busy mode for the next temperatures even in years without the
few months. The Indian Space Research phenomenon.
Organisation (Isro) is all set to conduct several b. Europe experienced its hottest summer on
big launches in a year it has decided to not just record in 2022, while climate change-fuelled
speed up its operations but also its turnaround extreme rain caused disastrous flooding in
time for rockets. Pakistan, and in February, Antarctic sea ice
A. Only a levels hit a record low.
B. Only b c. Despite most of the world's major emitters
C. Only c pledging to eventually slash their net emissions
D. Both A&B to zero, global CO2 emissions last year
E. No error continued to rise.
A. Only a
9. B. Only b
a. Isro said that it is gearing up for several bug C. Only c
missions in mid-2023. The missions will not only D. Both A&B
explore deep space but also the Sun and the E. No error
Moon.
b. ISRO will launch the much-awaited Directions (11-15): Given below are a few
Chandrayaan-3 mission to the Moon with the question with two columns given and both
next couple of months. contains parts of sentences to be matched in
c. The world could breach a new average order to make the sentence contextually correct.
temperature record in 2023 or 2024, fuelled by If none of the options are correct then choose
climate change and the anticipated return of the option E as your answer.
El Nino weather phenomenon, climate scientists 11.
say. Column 1 Column 2
A. Only a 1. Drugs are often a. while in Europe,
B. Only b identified with a symbol captured leaders
C. Only c representing stood trial for crimes
D. Both A&B against humanity.
E. No error

Click Here For Bundle PDF Course | support@guidely.in Page 4 of 11


Bank Po Mains PDF Course 2024
English Day - 11

2. Peru and Colombia b. the group which 3. For about a year, c. due to recurring pain
are the largest makes or moves the the pope has had to in his knee that he has
producers of coca leaf product. rely on a wheelchair said cannot be treated
through surgery.
3. After World War II, c. and cocaine in the
thousands of high- world, according to A. 1-a, 2-b, 3-c
ranking Nazi officers the United Nations. B. 1-a, 2-c, 3-b
sought refuge in South C. 1-c, 2-b, 3-a
America, D. 1-b, 2-c, 3-a
A. 1-a, 2-b, 3-c E. None of the above
B. 1-a, 2-c, 3-b
C. 1-c, 2-b, 3-a 13.
D. 1-b, 2-c, 3-a Column 1 Column 2
E. None of the above 1. Suffering a massive a. of nationalism that
economic crisis and has impacted the fate
12. surrounded by of its biggest election of
Column 1 Column 2 countries at war, modern times.
1. The pope's morning a. heads of states, Turkey is witnessing a
audiences are usually associations and resurgence
reserved for clerics, while his 2. Often referred to as b. illustrating the
afternoons are devoted the ‘world’s most continued demand
to work and private notorious’ spyware, among governments
meetings. and the lack of joint
international efforts to
2. Francis, who has b. for a decade, has limit the use of such
been the leader of the suffered increasing tools.
world's 1.3 billion health issues over the
Catholics past year, from 3. It was found to c. Pegasus often found
persistent pain in his have been used in its traces in the circles
right knee to sciatica to another dozen of journalists,
his recent hospital stay countries since 2021, oppositions and critics.
for bronchitis.

Click Here For Bundle PDF Course | support@guidely.in Page 5 of 11


Bank Po Mains PDF Course 2024
English Day - 11

A. 1-a, 2-b, 3-c B. 1-a, 2-c, 3-b


B. 1-a, 2-c, 3-b C. 1-c, 2-b, 3-a
C. 1-c, 2-b, 3-a D. 1-b, 2-c, 3-a
D. 1-b, 2-c, 3-a E. None of the above
E. None of the above
15.
14. Column 1 Column 2
Column 1 Column 2 1. WhatsApp, which is a. to defend itself, and
1. Sometime in mid- a. the Pegasus Project owned by Meta, is he has cast the
2021, Anna initiative investigated among a number of Ukraine war as a battle
Naghdalyan who is the the use of spyware by for the survival of
spokesperson governments on Russia against an
journalists, opposition aggressive West.
politicians, activists
and business people. 2. Russian President b. tech companies
Vladimir Putin says the pursuing legal action
2. Citizen Lab, a b. suspected Pegasus United States and its against the NSO group
Toronto-based operators in allies since 2019.
research organisation Azerbaijan that they
identified at least two call “BOZBASH” and 3. Putin has repeatedly c. are fighting an
“YANAR” by their warned that Russia, expanding proxy war
ongoing internet which has more against Russia after
scanning and DNS nuclear weapons than the Kremlin chief sent
cache probing. any other country, will troops into Ukraine 15
use all means months ago.
3. In a collaborative c. for Armenia’s
investigation by 17 foreign affairs agency, A. 1-a, 2-b, 3-c
media agencies was alerted by Apple B. 1-a, 2-c, 3-b
including Forbidden that her iPhone was C. 1-c, 2-b, 3-a
Stories, under attack by a D. 1-b, 2-c, 3-a
foreign government. E. None of the above

A. 1-a, 2-b, 3-c

Click Here For Bundle PDF Course | support@guidely.in Page 6 of 11


Bank Po Mains PDF Course 2024
English Day - 11

Directions (16-20): Given below are a few 18. For years, Armenia and Azerbaijan has been
questions with a highlighted phrase in each of at loggerheads over the dispute Nagorno-
them which may or may not require replacement. Karabakh region. While it’s internationally
If replacement is required then choose a proper recognised as part of Azerbaijan, many of its
replacement from the options. If no replacement residents are Armenian nationals.
is required then choose option E as your answer. A. Armenia or Azerbaijan have been at
16. NSO Group’s infamous Pegasus spyware, loggerheads over the disputed
which has been earlier in the news for its misuse B. Armenia and Azerbaijan has been at
against political and civil rights individuals, has loggerheads over the disputed
found its use in a war zone. C. Armenia and Azerbaijan have been at
A. which had been earlier in the news for its loggerheads over the disputed
misuse against D. Armenia and Azerbaijan had been at
B. which have been earlier in the news for its loggerheads over the disputed
misuse against E. No replacement is required
C. who has been earlier in the news for its
misuse against 19. Russia had been an huge numerical
D. who had been earlier in the news for its superiority over the United States and the NATO
misuse against military alliance when it comes to tactical nuclear
E. No replacement is required weapons: the United States believes Russia has
around 2,000 such working tactical warheads.
17. Global Intelligence and cyberwarfare in the A. Russia have a huge numerical superiority over
digital age altered forever when an Israeli the United States and
company, the NSO Group introduce its military- B. Russia has an huge numerical superiority over
grade spyware Pegasus in 2011 to the global the United States and
market. C. Russia had a huge numerical superiority over
A. the NSO Group introduced his military-grade the United States and
spyware D. Russia has a huge numerical superiority over
B. the NSO Group introduced her military-grade the United States and
spyware E. No replacement is required
C. the NSO Group introduce their military-grade
spyware 20. Vietnam and China have long been
D. the NSO Group introduced its military-grade embroiled in a territorial dispute over a potentially
spyware energy-rich stretch in South China Sea, a
E. No replacement is required

Click Here For Bundle PDF Course | support@guidely.in Page 7 of 11


Bank Po Mains PDF Course 2024
English Day - 11

strategic waterway through which more than $3 C. in a territorial dispute over a potentially
trillion of commerce passes annually. energy-rich stretch in the South China Sea
A. in a territorial dispute over a potentially D. in a territorial dispute over an potentially
energy-rich stretch in a South China Sea energy-rich stretch in the South China Sea
B. in a territorial dispute over a potentially E. No replacement is required
energy-rich stretch at the South China Sea
Click Here to Get the Detailed Video Solution for the above given Questions
Or Scan the QR Code to Get the Detailed Video Solutions

Answer Key with Explanation


1. Answer: A 3. Answer: D
Here, the first part of column 1 will be matched Here, the first part of column 1 will be matched
wthe ith first part of column 2 as it is connected with the second part of column 2 as it is
by since as we use it to give a reason for connected by which, the second part of column 1
something and here the same can be seen, the will be connected with the third part of column 2
second part of column 1 will be matched with and the final part of the first column will be
second part of column 2 and finally the leftover matched with the first part of column 1. So,
will be matched. So, option A is the correct option D is the correct answer.
answer.
4. Answer: A
2. Answer: B Here, all of the parts of column 1 will be matched
Here, the first part of column 1 will be matched with the parts which are just in opposite of them
with a and second part with c and the final means with the same parts of column 2. So,
leftover part will be matched with b. So, option B option A is the correct answer.
is the correct answer. 5. Answer: D

Click Here For Bundle PDF Course | support@guidely.in Page 8 of 11


Bank Po Mains PDF Course 2024
English Day - 11

Here, the first part of column 1 will be matched Here, the error lies in sentence b as use of with
with the second part of column 2 as both are before the next is inappropriate as within should
connected by in addition to as the same context have been used as within is used in the context
with further information is given, the second part of time as it means under certain limitations of
of column 1 will be matched with the third part of time given and here the same can be seen. So,
column 2 as both are connected with but also option B is the correct answer.
which is used to add emphasis when writing
about two related things and finally the third part 10. Answer: A
of column 1 will be matched with the first part of Here, the error lies in sentence a as use of hot is
column 2 which is connected by which. So, inappropriate because as per the context of the
option D is the correct answer. sentence the superlative degree should have
been used which is hottest. The superlative
6. Answer: C degrees are used when the noun in a particular
Here, the error lies in part c as use of was before context has the greatest or least degree of the
targeted is inappropriate as here sentence is in quality and here greatest quality is there. So,
perfect continuous tense from starting and option A is the correct answer.
second part is in present perfect tense and
therefore has should have been used here due 11. Answer: D
to perfect tense and singular subject. So, option Here, the first part of column 1 which is about
C is the correct answer. drugs should be matched with second part of
column 2 and second part of column 1 which is
7. Answer: E about the peru and Colombia being largest
Here, none of the sentence contains error. So, producer should be matched with third part of
option E is the correct answer. column 2 and finally the third part of column one
will be matched with first part of column 2. So,
8. Answer: A option D is the correct answer.
Here, the error lies in part a as use of was before
a result is inappropriate as sentence is in 12. Answer: A
present tense and therefore is should have been Here, all of the parts given in first column will be
used. So, option A is the correct answer. matched with the part given same in the column
2. So, option A is the correct answer.
9. Answer: B
13. Answer: B

Click Here For Bundle PDF Course | support@guidely.in Page 9 of 11


Bank Po Mains PDF Course 2024
English Day - 11

Here, the first part of column 1 which is about Here, the phrase highlighted is grammatically
turkey experiencing economic crisis should be correct and no replacement is required. So,
matched with first part of column 2, the second option E is the correct answer.
part of column 1 which is about notorious
spyware should be matched with the third part of 17. Answer: D
column 2 and finally the third part of column 3 Here, there should be introduced in place of
which is about the use of something by countries introduce as the sentence is in past tense and so
should be matched with the second part of the second form of verb will be used in case of
column 2. So, option B is the correct answer. past simple tense. So, option D is the correct
answer.
14. Answer: C
Here, the first part of column 1 which is about 18. Answer: C
some spokesperson should be matched with the Here, the use of has been is inappropriate as
third part of column 2, the second part of column subject is plural and therefore have been will be
1 which is about some research being done used and also the dispute should be changes
should be matched with the second part of into disputed as the second part of the sentence
column 2 and finally the third part of column 1 is in past simple tense and therefore it will use
which is about some investigations should be second form of the verb. So, option C is the
matched with the first part of column 2. So, correct answer.
option C is the correct answer.
19. Answer: D
15. Answer: D Here, the use of had been is inappropriate as
Here, the first part of column 1 which is about has should have been used as sentence is in
whatsapp should be matched with the second present perfect continuous tense and also the
part of column 2, the second part of column 1 use of an is also inappropriate as huge doesn’t
which is about putting and his statement about sound like a vowel and also h is not a vowel and
US should be matched with the third part of therefore, a should have been used. So, option
column 2 and finally the third part of column 1 D is the correct answer.
which is about warning given by Putin should be
matched with the first part of column 2. So, 20. Answer: C
option D is the correct answer. Here, there should be use of ‘the’ in between in
and South China sea as the article the is used
16. Answer: E before seas and here the sea is mentioned and

Click Here For Bundle PDF Course | support@guidely.in Page 10 of 11


Bank Po Mains PDF Course 2024
English Day - 11

therefore, the is required as an article. "The" is oceans and seas. So, option C is the correct
used in these case before the names of deserts, answer.

Click Here For Bundle PDF Course | support@guidely.in Page 11 of 11


Bank Po Mains PDF Course 2024
Reasoning Day -12 (Eng)

Reasoning Aptitude
Directions (1-5) Study the following information designated senior to N, who is neither working
carefully and answer the given questions. with O nor R.
Twenty one persons are working in an All the persons except senior to Manager are
organization at different designations viz.- transferred to three different departments viz.-
Director, General Manager (GM), Manager, Production, Marketing, and Sales as per below
Assistant Manager (AM), Probationary Officer conditions:
(PO), and Clerk. The hierarchy of designations Condition I: All persons whose name comes
are given in decreasing order such that before ‘M’ in alphabetical series and even
Chairman is the highest designation whereas number of persons working along with them are
clerk is the lowest designation. transferred to Production department.
Note: I. The number of persons working at each Condition II: All persons whose name comes
designation is one less than the number of after ‘M’ in alphabetical series and odd number of
persons working its immediate lower designation. persons working along with them are transferred
II. If said that A works with B or A and B are to Marketing department.
working together, then both A and B are working Condition III: All the remaining persons are
at the same designation. transferred to Sales departement.
B, I, and T are working together but are not The persons senior to Manager are appointed as
designated as Assistant managers. Odd number supervisors of all three departments viz.-
of persons are designated between T and F, who Production, Marketing, and Sales in alphabetical
is immediately junior to H. Both H and D are order respectively.
neither designated as probationary officer nor the 1) Which among the following pair of persons are
General Manager. D and O are working together not supervised by H?
and designated junior to F. Odd number of I. TQD
persons are working along with O. The number II. BIL
of persons senior to G is the same as the III. ORP
number of persons junior to Q, who is two a) Only I and III
designations junior to M. Neither M nor A working b) Only II and III
with H. Odd number of persons are designated c) Only III
between A and R, who is designated immediately d) Only I
senior to L. Both A and J are designated senior e) All I, II and III
to assistant manager. P, S and K are working
together and designated junior to L. Both E and 2) _____ persons works with _____ who works
U are not senior to assistant manager. C is as _______

Click Here For Bundle PDF Course | support@guidely.in Page 1 of 12


Bank Po Mains PDF Course 2024
Reasoning Day -12 (Eng)

a) Four, T, PO Eight persons – Rimii, Kimmi, Simmi, Timmy,


b) Three, C, Manager Pummy, Summy, Gummy and Rummy were born
c) Three, G, AM in different years. The age of persons is
d) Both a and c calculated on the base year 2023. Each person
e) Both a and b watched different OTT platforms viz.-Hotstar,
Arre, TVF, Zee5, Amazon, Netflix, Voot and Sony
3) _______ persons are working as senior to Liv.
_____ Note: I. If it is given A’s age is equal to the last
a) Three, Q two digits of B’s birth year, then it may be in any
b) Ten, E order. For example, if B was born in 1965, then
c) Six, R A’s age is either 65 or 56.
d) Three, A II. None of the persons were born after 2012 and
e) One, J born before 1950.
Rummy was born in 2008. Only two persons
4) Which of the following pair of persons are not were born between Rummy and the one who
working together? watched Amazon. The difference between the
I. GDR ages of Rummy and Simmi is 15 years. The one
II. BIJ who watched Amazon was born immediately
III. PEN before Simmi but not after 1986. The age of
a) Only I and III Timmy, who watched Hotstar, is the last two
b) Only II and III digits of the birth year of Simmi. The difference
c) Only III between the ages of Timmy and the one who
d) Only I watched Zee5 is 28 years. The difference
e) Only II between the ages of Pummy and the one who
5) Which of the following statements is/are not watched Zee5 is 17 years. Only one person was
true? born between Pummy and the one who was born
a) J and C transferred to production in 1986. The difference between the ages of
b) O and S are supervised by H Pummy and Timmy is the same as the difference
c) E, P and U are working together between the ages of Timmy and Kimmi. Rimmi’s
d) Only three persons are designated senior to C age is equal to the last two digits of the birth year
e) All the above statements are true of Kimmi, who was born immediately after the
one who watched Netflix. Rimmi was neither
Directions (6-10) Study the following information born in 1964 nor watched Netflix. The number of
carefully and answer the given questions. persons born after the one who watched Voot is

Click Here For Bundle PDF Course | support@guidely.in Page 2 of 12


Bank Po Mains PDF Course 2024
Reasoning Day -12 (Eng)

one more than the number of persons born 9) Who among the following person watched
before the one who watched Tvf. No one was Voot?
born between the one who watched Tvf and a) The one who was born immediately after
Summy. The difference between the ages of Kimmi
Summy and the one who was born immediately b) Timmy
before the one who watched Sony Liv is 18 c) The one who was born in 1993
years. d) Pummy
6) Who among the following person watched e) None of these
Tvf?
I. The one who was born in 1973 10) Which of the following statements is/are not
II. The one who was born immediately after true as per the given arrangement?
Summy a) Gummy was born in 2012
III. Kimmi b) Simmi Watched Sony Liv
a) Only I and III c) Only two persons were born before Timmy
b) Only II and III d) The one who watched Amazon was born in
c) Only III 1984
d) Only I e) All the above statements are true
e) All I, II, and III
Directions (11-15) Study the following
7) _____ watched ____ and was born _____ information carefully and answer the given
persons after Timmy questions.
a) Gummy, Zee5, Two In a certain code language following statements
b) Kimmi, Hotstar, Two are coded as follows:
c) Pummy, Voot, Three “Perfect Driver Safety Release Counter” is coded
d) Summy, Tvf, Two as “%8G %12G @10E &8V $12U”
e) None of these “Clint Party Manual Define Campaign” is coded
as “@4P *9C $9P #15I &4R”
8) The one who watched ______ was born in “Poverty Crown Mobile Several Bought” is coded
_____ as “&10V #4Y __(A)___ *12C @8J”.
a) Hotstar, 1984 “Action Polar Count Flight Service” is coded as
b) Netflix, 1973 “%6C @6P #9Q __(B)__ $12E”.
c) Amazon, 1985 11) What comes in place of ___(A)____?
d) Voot, 1993 a) &12U
e) Both b and d b) #9V

Click Here For Bundle PDF Course | support@guidely.in Page 3 of 12


Bank Po Mains PDF Course 2024
Reasoning Day -12 (Eng)

c) $9N
d) @4T Directions (16-20) Study the following
e) None of these information carefully and answer the given
questions.
12) Code of which of the following word is Seven different items viz.-Cookies, Cake,
incorrect in the bold highlighted codes? Biscuits, Chocolate, Cream Roll, Pastry, and Ice
a) Clint - @4P Cream are scheduled to be delivered one by one
b) Define - $9P in different apartments viz.- A, B, C, D, E, F, G
c) Party - &4R and H. No item was delivered in one of the
d) Campaign - #15I apartments due to cancelation of order. Atmost
e) None of these one item was delivered in each apartment.
Note: The apartment in which no item was
13) What comes in place of ___(B)___? delivered is scheduled to be delivered neither
a) @5J first nor last.
b) $6M Only three items were delivered between Cake
c) @7F and the item which was delivered in apartment
d) &5U G, which was delivered after Cake. Only one
e) None of these item was delivered between the item which was
delivered in apartment G and Cream Roll. Cream
Roll was delivered in apartment A and delivered
14) What is the code of “Safety Release”? before the item which was delivered in apartment
a) &8V $12U G. Chocolate was delivered immediately before
b) %12G &8R the item which was delivered in apartment A.
c) $12Q @4L Only two items were delivered between
d) @4L $12Q Chocolate and the item which was delivered in
e) None of these apartment F, which was delivered before
Chocolate. Neither Pastry nor Biscuit delivered in
15) Which of the following combination of codes apartment F. The number of items delivered
are not true? before Biscuit is one more than the number of
a) Perfect - @10E items delivered after Cookies. Cookies was
b) Action - #9Q delivered immediately after Pastry. Neither
c) Define - $9P Biscuit nor the item which was delivered in
d) Several - &10R apartment H was delivered immediately before or
e) Bought - @8J immediately after the apartment in which no item

Click Here For Bundle PDF Course | support@guidely.in Page 4 of 12


Bank Po Mains PDF Course 2024
Reasoning Day -12 (Eng)

was delivered. Neither Cake nor Pastry was a) Three


delivered in both apartments E and H. The item b) One
which was delivered in apartment D was c) Four
delivered immediately after the item which was d) Two
delivered in apartment B. Chocolate was neither e) None
delivered in apartment C nor D. At least one item
was delivered in apartments H and C. 19) Which among the following item was
16) Biscuit was delivered in which of the delivered after Cream Roll?
following apartment? I. Pastry
a) F II. Cookies
b) B III. Cake
c) A a) Only II
d) E b) Only II and III
e) None of these c) Only I and II
d) Only III
17) Which of the following combinations of e) Only I
apartment and item delivered is true?
a) G – Pastry 20) In which of the following apartment no item
b) D – Cake was delivered?
c) B – Ice cream a) C
d) H – Biscuit b) A
e) E – Cookies c) E
d) G
18) How many items were delivered before the e) H
item which was delivered in apartment D?

Click Here For Bundle PDF Course | support@guidely.in Page 5 of 12


Bank Po Mains PDF Course 2024
Reasoning Day -12 (Eng)

Click Here to Get the Detailed Video Solution for the above given Questions
Or Scan the QR Code to Get the Detailed Video Solutions

Answer Key with Explanation


Directions (1-5):  Odd number of persons are designated
1. Answer: B between T and F, who is immediately
2. Answer: D junior to H.
3. Answer: C  Both H and D are neither designated as
4. Answer: E probationary officer nor the General
5. Answer: B Manager.
That means, in case (1) B, I and T are
working as probationary officer, in case
(2) B, I, and T are working as clerk.
Based on the above given information we have:

After rearrangement:

Again, we have:
 D and O are working together and
We have: designated junior to F.
 B, I, and T are working together but are  Odd number of persons are working along
not designated as Assistant manager. with O.

Click Here For Bundle PDF Course | support@guidely.in Page 6 of 12


Bank Po Mains PDF Course 2024
Reasoning Day -12 (Eng)

Since, odd number of persons working


along with O, then even number of
persons are working on O’s designation.
That means, in case (1) O working as an
assistant manager, in case (1a) & case
(2) O working as clerk.
Based on the above given information we have: Case (2) is not valid as PSK working together
and junior to L but no such position available.
Again, we have:
 Both E and U are not senior to assistant
manager.
 C is designated senior to N, who is
neither working with O nor R.

Again, we have: That means, in case (1) E and U working

 The number of persons senior to G is the as clerk, case (1a) is not valid.

same as the number of persons junior to Based on the above given information we have:

Q, who is two designations junior to M.


 Neither M nor A working with H.
That means, in case (1) and in case (1a)
G is designated as an assistant manager
& in case (2) G is designated as clerk.
 Odd number of persons designated
between A and R, who is designated
All the persons except senior to Manager are
immediately senior to L.
transferred to three different departments viz.-
 Both A and J are designated senior to
Production, Marketing, and Sales as per below
assistant manager.
conditions:
 P, S and K are working together and
Condition I: All persons whose name comes
designated junior to L.
before ‘M’ in alphabetical series and even
That means, in case (1) & case (1a) A is
number of persons working along with them are
working with F, and case (2) is not valid.
transferred to Production department.
Based on the above given information we have:
Condition II: All persons whose name comes
after ‘M’ in alphabetical series and odd number

Click Here For Bundle PDF Course | support@guidely.in Page 7 of 12


Bank Po Mains PDF Course 2024
Reasoning Day -12 (Eng)

of persons working along with them are We have:


transferred to Marketing department.  Rummy was born in 2008.
Condition III: All the remaining persons are  Only two persons were born between
transferred to Sales department. Rummy and the one who watched
The persons senior to Manager are appointed as Amazon.
supervisors of all three departments viz.-  The difference between the ages of
Production, Marketing, and Sales in alphabetical Rummy and Simmi is 15 years.
order respectively.  The one who watched Amazon was born
Based on the above given information we have: immediately before Simmi but not after
1986.
Thus, Simmi was born in 1993.
 The age of Timmy, who watched Hotstar,
is the last two digits of the birth year of
Simmi.
Since, Simmi was born in 1993, and no
Directions (6-10): one was born before 1950, thus age of
6. Answer: E Timmy must be 39 years.
7. Answer: C Based on the above given information we have:
8. Answer: A
9. Answer: D
10. Answer: D

Again, we have:
 The difference between the ages of
Timmy and the one who watched Zee5 is
28 years.
Since, Timmy was born in 1984, thus the
one who watched Zee5 must born in 1956
or 2012.

Click Here For Bundle PDF Course | support@guidely.in Page 8 of 12


Bank Po Mains PDF Course 2024
Reasoning Day -12 (Eng)

That means, in case (1) the one who Case (1) is not valid as Rimmi was not
watched Zee5 was born in 1956, in case born in 1964.
(2) the one who watched Zee5 was born Based on the above given information we have:
in 2012.
 The difference between the ages of
Pummy and the one who watched Zee5 is
17 years.
 Only one person was born between
Pummy and the one who was born in
1986.
Based on the above given information we have:

Case (1) is not valid as Rimmi was not born in


1964.
Again, we have:
 The number of persons born after the one
who watched Voot is one more than the
number of persons born before the one
who watched Tvf.
Again, we have:  No one was born between the one who
 The difference between the ages of watches Tvf and Summy.
Pummy and Timmy is the same as the  The difference between the ages of
difference between the ages of Timmy Summy and the one who was born
and Kimmi. immediately before the one who watched
That means, in case (1) Kimmi was born Sony Liv is 18 years.
in 1995, in (2) Kimmi was born in 1973. Since, Summy was born before 1973,
 Rimmi’s age is equal to the last two digits thus only possible person who watched
of the birth year of Kimmi, who was born Sony Liv is Simmy, thus Summy must
immediately after the one who watched born in 1968.
Netflix. Based on the above given information we have:
 Rimmi was neither born in 1964 nor
watched Netflix.

Click Here For Bundle PDF Course | support@guidely.in Page 9 of 12


Bank Po Mains PDF Course 2024
Reasoning Day -12 (Eng)

Directions (16-20):
16. Answer: B
17. Answer: B
18. Answer: D
19. Answer: C
20. Answer: C
Directions (11-15):
11. Answer: C
12. Answer: C
13. Answer: A
14. Answer: A
15. Answer: D
We have:
For symbols: last letters of the word represents
symbol such as:
T  @, N  #, R  %, Y &, E  $, L  *
For number: product of the number of vowels
and consonants in the words are taken.
For letters: Second succeeding letter of the
second last letter of the word in alphabetical We have:
order are taken.  Only three items were delivered between
For example: Cake and the item which was delivered
Perfect: @10E in apartment G, which was delivered
Based on the rule followed we have: after Cake.

Click Here For Bundle PDF Course | support@guidely.in Page 10 of 12


Bank Po Mains PDF Course 2024
Reasoning Day -12 (Eng)

 Only one item was delivered between the  Neither Pastry nor Biscuit delivered in
item which was delivered in apartment G apartment F.
and Cream Roll.  The number of items delivered before
 Cream Roll was delivered in apartment A Biscuit is one more than the number of
and delivered before the item which was items delivered after Cookies.
delivered in apartment G.  Cookies was delivered immediately after
That means, in case (1) & case (3) Pastry.
Cream roll was delivered two  Neither Biscuit nor the item which was
apartments before G, in case (2) Cream delivered in apartment H was delivered
roll was delivered three apartments immediately before or immediately after
before G. the apartment in which no item was
Based on the above given information we have: delivered.
That means, in case (2) & case (3) Biscuit
was delivered immediately after item
which was delivered in apartment F, case
(1) is not valid.
Based on the above given information we have:

Again, we have:
 Chocolate was delivered immediately
before the item which was delivered in
apartment A.
 Only two items were delivered between
Chocolate and the item which was Case (1) is not valid as no place available for
delivered in apartment F, which was Biscuit.
delivered before Chocolate. Again, we have:
Since, the apartment in which no item is  Neither Cake nor Pastry delivered in both
delivered is between any two apartments. apartments E and H.
Thus, item in apartment F is delivered  The item which was delivered in
first. apartment D was delivered immediately

Click Here For Bundle PDF Course | support@guidely.in Page 11 of 12


Bank Po Mains PDF Course 2024
Reasoning Day -12 (Eng)

after item which was delivered in


apartment B.
 Chocolate was neither delivered in
apartments C nor D.
That means, Biscuit was delivered in
apartment B.
 At least one item was delivered in
apartments H and C.
That means, in case (2) Chocolate was
delivered in apartment H, case (3) is not
valid.
Case (3) is not valid as Neither Biscuit nor the
Based on the above given information we have:
item which was delivered in apartment H was
delivered immediately before or immediately
after the apartment in which no item was
delivered.

Click Here For Bundle PDF Course | support@guidely.in Page 12 of 12


Bank Po Mains PDF Course 2024
Quantitative Aptitude Day -12 (Eng)

Quantitative Aptitude

Directions (1 - 4): Study the following information carefully and answer the questions given below.
Three types of elections are conducted in panchayat [Gram panchayat (village level), Panchayat samite
(block level) and District Panchayat (District level)] in a single day. The given table shows the number of
blocks in four different states and, the average number of registered voters in each village. Only party X,
Y and Z contests in the election.

Note – I. There are two villages in each block. Total registered Voters of each block = Total registered
voters in all villages in the block. Total registered voters in each district = Total registered voters in all
villages together. Each registered voter who comes to give vote should vote in all three elections [i.e., the
Total cast vote should be equal in all three types of the election] and their vote may differ in each
election, [ i.e., If a voter gives the vote to party X in village election he or she may or may not be give vote
to party X in others two types of election].
II. In each village only one candidate for each party contests the election, in each block only one
candidate for each party contests the election, and for each district only one candidate for each party
contests the election.
1) In the District panchayat election of state A, c) 2800
Party Z got 25% of registered voters and got 3 rd d) 2500
position, and Party Y got 2900 votes which is e) 2900
100 votes less than Party X. In the block election
of the same state, party X got 30% of vote of 2) In each village of B, 10% of registered voters
total casted votes and party Y and Z got equal did not cast their vote and 10% of casted votes
numbers of votes. The sum of votes of party X were declared as invalid. In the village election
at village level and block level is 4400 and for party X got 7530 votes and the rest of the votes
party Y is 6000. Find the number of votes got by were casted to Y and Z equally. Block Parties X,
party Z in village elections? [All cast vote is valid] Y and Z got votes in the ratio of 3:2:4. In total,
a) 2240 Party Y got 30% of votes in all the formats
b) 2600 together. Find the difference between the total

Click Here For Bundle PDF Course | support@guidely.in Page 1 of 12


Bank Po Mains PDF Course 2024
Quantitative Aptitude Day -12 (Eng)

vote of party Z in all the three types and total d) 2800


vote of party Y in all the three types of election? e) None of these
a) 2410
b) 3890 4) In District D, 80% of people are registered
c) 2530 voters. The ratio of males and females out of the
d) 3020 total population in D is 3:2. 70% of female voters
e) CND are registered voters and all the male voters cast
their vote in favor of party X in district and village
3) 25% of registered voters did not cast their elections and in favor of party Y in block
votes in District C and 400, 1600 and 1200 votes elections. Females cast their vote in favor of
of village, block and district elections were Party Z in all elections. Find the difference
declared invalid. The ratio of the total vote of between Party X's vote in the district and Party Z
parties X, Y and Z in all villages together, all in the block election. [All voters cast their votes
blocks together and district elections are 15:8:15, and all votes are valid]
7:5:5 and 1:2:1 respectively. Find the total votes a) 2520
of party Y in the block election and Z in the b) 2240
district election? c) 2630
a) 2000 d) 2120
b) 3000 e) None of these
c) 2500

Directions (5 - 8): Study the following information carefully and answer the questions given below.
The pie chart shows the percentage distribution of the total number of staff in five different factories [A, B,
C, D, and E].

Click Here For Bundle PDF Course | support@guidely.in Page 2 of 12


Bank Po Mains PDF Course 2024
Quantitative Aptitude Day -12 (Eng)

The pie chart shows the percentage distribution of the total number of graduate staff in five different
factories [A, B, C, D, and E].

Note –
I. R-Q=8%. P is 50% more than Q.

Click Here For Bundle PDF Course | support@guidely.in Page 3 of 12


Bank Po Mains PDF Course 2024
Quantitative Aptitude Day -12 (Eng)

II. the Ratio of Q and S is 1:2.


III. The Total Number of graduate staff in all factories together is 5600.
IV. The total number of non-graduate staff in E is 660.
5) Out of the total number of staff in C, T% are ratio of the number of staff using private
female and the rest are male. Out of total male transport in B and D is 29:48. Find the difference
(4R-5) % are graduates. Find the possible value between the number of employees using public
of non-graduate female staff in C? transport in B and D together and number of
I. 9R+55 employees using private transport in B and D
II.7S+25 together?
III.9T+1 a) 2560
a) Only I b) 2740
b) only I and II c) 2140
c) Only I and III d) 2470
d) only II and III e) None of these
e) only III
8) The total number of staff in F is R% more than
6) Out of the total graduates in A, (Q+13)% are the number of staff in E and the total number of
officers and the rest are clerks. The average staff in G is (P+2)% more than the number of
salary of an officer is Rs.45000 and the average graduate staff in E. 25% of the total number of
salary of a clerk is Rs.29000. Average salary of employee of F and G together work in night shift.
all non-graduate staff is Rs.22000. Find the Find the possible value of the total number of
approximate average salary of all the staff of A? staff in F and G together work in night shift?
a) Rs.22884 I.50.25S
b) Rs.29854 II. 4.5P
c) Rs.29544 III.75.375(Q+4)
d) Rs.28844 a) only I
e) Rs.29484 b) only I and III
c) only II and III
7) The ratio of the number of staff using public d) only I and II
transport in factories B and D is 5:12 and the e) None of these

Directions (09 - 12): Study the following information carefully and answer the questions given below.

Click Here For Bundle PDF Course | support@guidely.in Page 4 of 12


Bank Po Mains PDF Course 2024
Quantitative Aptitude Day -12 (Eng)

The given bar graph shows the amount of investment by A, B, and C, in three different business [P, Q,
and R] and the difference between the amount of investment by A and B and B and C in each business is
also given.

9) In business P, after 6 months, 8 months and Rs.2000. The ratio of profit share of A, B and C
10 months A, B and C invest Rs.8000, Rs.6000 is 108:105:116. Find the value of n?
and Rs.2000 respectively. If after one year the a) 6
total profit of P is Rs. 11041 Find the share of b) 4
profit of B? c) 8
a) z-260 d) 2
b) 2x+360 e) 7
c) y+660
d) x+y-2560 11) All the person invests their money for 12
e) 2x+240 months and the total profit in P, Q and R is
10) In business Q, after 4 months A invests Rs.8100, Rs.9065 and Rs. 11000. Find which of
Rs.6000 more and after 2 months B invests the following is true?
Rs.5000 more and after n months C withdraws a) The sum of the share of A in all three is 7852.

Click Here For Bundle PDF Course | support@guidely.in Page 5 of 12


Bank Po Mains PDF Course 2024
Quantitative Aptitude Day -12 (Eng)

b) The sum of the share of B in all three is 9975. scored by C is 15:16. D scored 6.66% more
c) The difference of share of A and C in P and Q marks than C in math. Marks of C in math are
is 654. 25% more than B in chemistry. The ratio of
d) The difference of share of C in P and Q is 458. marks of A in math and physics is 16:17. The
e) None of these ratio of marks scored by B in physics and
chemistry is 7:6. Total marks scored by C in all
12) Total profit from Business P is Rs.13500. the subjects together is 230. The ratio of marks
With his share, A bought a watch and then sold it of C and D in physics is 5:6.
at 20% profit. Find the possible selling price of 13)If the marks of A in English is 20% more than
the watch? the marks of A in math. The ratio of marks of A
I.5.6X-7600 and B in English is 4:3. Marks of B in Biology is
II.4.5X-9900 25% more than his marks in English. Which of
III.X+Y the following can be the ratio of marks of the
a) only I and II biology of B and A?
b) only II and III I. 9:10. II. 9:11, III. 5:6
c) only I a) only II
d) all three b) only III
e) None of these c) only I
d) only I and II
Directions (13- 16): Study the following e) only II and III
information carefully and answer the questions
given below. 14) The ratio of marks of A and E in Math is 4:3.
There are four students: [A, B, C and D]. Each The average marks of B and E in chemistry is 72
student scores different marks in three different and the average marks of C and E in physics is
subjects: math, physics, and chemistry. The ratio 78. Find the total marks scored by E?
of marks of A and B in math is 16:17. The total a) 245
marks in each subject are 100. In chemistry, A b) 258
scored 20 marks less than his score in physics. c) 256
Marks of D in chemistry are 16.66% more than d) 225
marks of B in the same subject. The marks of B e) 268
in math are 13.33% more than the marks of C in
physics. Marks of C in Chemistry are 33.33% 15) If A got x% percentage marks in three
more than the same scored by B in chemistry. subjects and B got y% percentage marks in three
The ratio of marks in physics and chemistry

Click Here For Bundle PDF Course | support@guidely.in Page 6 of 12


Bank Po Mains PDF Course 2024
Quantitative Aptitude Day -12 (Eng)

subjects. Find the percentage of marks C and D Quantity: II If Mark up percentage of item B is
get? 60% above the cost price and the discount
a) y%, x% amount is Rs.80 then find the profit percentage?
b) (y+5)%, (x+3.34)% A. Quantity: I < Quantity: II
c) (x+2.5)%, (x-2.5)% B. Quantity: I ≥ Quantity: II
d) (x+3.66)%, (y-1.66)% C. Quantity: II ≥ Quantity: I
e) None of these D. Quantity: I > Quantity: II
E. Quantity I = Quantity II or relation can't be
16) established
Quantity: I Total marks scored by all the students
together in physics is what percent of the total 18) The ratio of the present age of A and B is
marks scored by all the students together in 2:3. The ratio of the age of A after 5 years and
maths? the present age of C is 5:7. B is 5 years younger
Quantity: II Total marks scored by all the than C.
students together in chemistry is what percent of Quantity: I If the Age of D is the average age of A
the total marks scored by all the students and B then find the average age of C and D?
together in maths? Quantity: II If the age of E is 2 years younger
A. Quantity: I < Quantity: II than B and the Age of F is 3 years younger than
B. Quantity: I ≥ Quantity: II C. Find the average age of E and F?
C. Quantity: II ≥ Quantity: I A. Quantity: I < Quantity: II
D. Quantity: I > Quantity: II B. Quantity: I ≥ Quantity: II
E. Quantity I = Quantity II or relation can't be C. Quantity: II ≥ Quantity: I
established D. Quantity: I > Quantity: II
E. Quantity I = Quantity II or relation can't be
17) The ratio of the cost price of items A and B is established
4:5. Item A and B is marked up by 60% and 70%
above the cost price. Item A and B sold at Rs.40 19) SERIES I and SERIES II follow the same
and Rs.60 discount. The profit percentage of logic. Find the value of D?
item A is 56.66%. Series I – 158, 290, 446, 628, 838, A
Quantity: I If Mark up percentage of item A is Series II – A, B, C, D, E
50% above the cost price and the discount a) 1478
amount is the same then find the profit b) 1548
percentage? c) 1684
d) 1984

Click Here For Bundle PDF Course | support@guidely.in Page 7 of 12


Bank Po Mains PDF Course 2024
Quantitative Aptitude Day -12 (Eng)

e) None of these a) 1940


b) 1480
20) Find the Sum of the missing number of the c) 1570
series? d) 1640
Series I –78,180,408,840, ? e) None of these
Series II- 145,155,166,179,196, ?
Click Here to Get the Detailed Video Solution for the above given Questions
Or Scan the QR Code to Get the Detailed Video Solutions

Answer Key with Explanation


1. Answer: C So, the total votes of party Z in the village
The total number of blocks in A is 4. election = 8000-2000-3200=2800
The total number of villages in A is 4*2=8
The total number of registered voters in A is 2. Answer: E
8*1050=8400 Total villages in B are = 5*2=10
Total cast vote is 8400*25/100 + 2900 The total population in B is = 10*1300=13000
+2900+100=8000 Total valid vote in village of B = 13000*90*90/

Total votes party X in block election = [100*100] =10530

8000*30/100=2400 Total votes got by party Y in all the elections

Total votes of party Y in block election = [8000- together


2400]/2=2800 = Total votes of party Z in block = (30% of (10530*3)
election. =9477

Total votes of party X in village election = 4400- Total votes got by party Z in all the elections

2400=2000 together – From the given data, we couldn’t find


Total votes of party Y in village election = 6000- votes got by Z in district elections.
2800=3200 Answer: Can’t be determined

Click Here For Bundle PDF Course | support@guidely.in Page 8 of 12


Bank Po Mains PDF Course 2024
Quantitative Aptitude Day -12 (Eng)

3. Answer: B From the 2nd pie chart, R%+Q%=100-16-22-


Total voters in district C is = 6*2*8000=9600 30=32
The total cast votes in C is 9600*3/4=7200 And ATQ, R-Q=8%
Total valid votes of village elections of all villages So, R= [32+8]/2=20% and Q=[32-8]/2=12%
together = 7200-400=6800 P=12*150/100=18%
Total valid votes of village elections of all villages S=12*2/1=24%
together = 7200-1600=5600 From 1st pie chart, T=100%-18%-12%-20%-
Total valid votes of village elections of all villages 24%=26%
together = 7200-1200=6000 The total number of graduate staff in E is =
Let party X vote in village, block and district 5600*30/100=1680
elections is 15a, 8a and 15a respectively. The total number of staff in E is =
Let party Y votes in village, block and district 1680+660=2340
elections are 7b, 5b and 5b respectively. So, the total number of staff in all company
Let party Z votes in village, block and district together is
election is c, 2c and c respectively. =[2340/26]*100=9000
So, 15a+7b+c=6800, 8a+5b+2c=5600 and 5. Answer: C
15a+5b+c=6000 Total female in C is = 1800*26/100=468
By solving the above equations, we get a=200, So, male is = 1800-468=1332
b=400, c=1000 So, male graduate is = 1332*75/100=999
Required sums = 5*400+1000=3000 So, the female graduate is =1232-999=233
So, female non- graduate is = 468-
233=235=9R+55=9T+1
4. Answer: A
Total registered voters in D is = 3*2*1400=8400 6. Answer: D
Total number of people in D is = Total officer is = 896*25/100=224
8400*100/80=10500 Total clerk is = 896-224=672
Total female voters is = So, total salary is =
[10500*2/5]*70/100=2940 224*45000+672*29000+544*22000
Total male voters is = 8400-2940=5460 =41536000
Required difference = 5460-2940 =2520 So, average salary =
[41536000/1440]=28844.44=28844
Directions (5 - 8):
7. Answer: B

Click Here For Bundle PDF Course | support@guidely.in Page 9 of 12


Bank Po Mains PDF Course 2024
Quantitative Aptitude Day -12 (Eng)

Let number of staff use public transport for B and Or, Z=5000
D is 5x and 12x. So, X=[5000+3000]/4=8000/4=2000
The number of staff use private transport of B Now, Y=3*2000/2=3000
and D is 29y and 48y.
So, 5x+29y=1260(i) and 12x+48y=2160,
x+4y=180(ii)
By solving (i) and (ii) we get x =20 and y=40.
9. Answer: C
So, the total number of staff from B and D using
Investment ratio of A, B and C,
public transport is
[12000*6+20000*6]:[18000*8+24000*4]:[24000*
= 100+240=340
10+26000*2]
The total number of staff from B and D use
=192:240:292=48:60:73
private transport is
So, share of profit of B is =
= 1160 + 1920 = 3080
11041*60/181=3660=y+660
So, required difference 3080 - 340=2740
8. Answer: B
10. Answer: D
The total number of staff in F is =
So, their investment ratio
2340*120/100=2808
[14000*4+20000*8] :
Total number of staffs in G is =
[15000*6+20000*6]:[20000*(6+n)+18000*(12-
1680*120/100=2016
(6+n)] = 108:105:116
So, the total number of staff in F and G is =
108 : 105 : n+114 = 108 : 105 : 116
2808+2016=4824
n+114 = 116
The total number of employees working night
n=2
shifts is = 4824*25/100
=1206=50.25S=75.375(Q+4)
11. Answer: B
Invest ratio of A, B and C in P = 12:18:24=2:3:4
Directions (09 - 12):
Invest ratio of A, B and C in Q =
From the graphs, we can see in business P, [9X-
14:15:20=14:15:20
6X]=2Y
Invest ratio of A, B and C in R =
Or, 3X=2Y
10:18:16=10:18:16
Again, in business Q, 3Z-[8X-2000] =1000,
So, the sum of share of B is
In business R, 3Z+3000-2Z=4X
=8100*3/9+9065*15/49+11000*18/44=9975
Or, Z+3000=4X
So, B is true.
Now, 3Z-[2*(Z+3000)-2000] =1000

Click Here For Bundle PDF Course | support@guidely.in Page 10 of 12


Bank Po Mains PDF Course 2024
Quantitative Aptitude Day -12 (Eng)

12. Answer: C 90*6/5=108 - not possible


Invest ratio of A, B and C in P = 12:18:24=2:3:4
So, A’s share is = 13500*2/9=3000 14. Answer: D
So, the selling price of watch is Total marks of E is = 80*3/4+72*2-60+78*2-
3000*120/100=3600=5.6X-7600 75=225

Directions (13- 16): 15. Answer: B


Let marks of C in physics and chemistry be 15x A’s percentage = [230/300]*100=76.66%
and 16x, respectively. B’s percentage = [215/300]*100=71.66%
So, the mark of B in chemistry is 16x*100/133.33 C’s percentage =
= 12x. [230/300]*100=76.66%=[y+5]%
So, the mark of C in math is 12x*125/100 = 15x. D’s percentage =
So, 15x+16x+15x=230 [240/300]*100=80%=[x+3.34]%
Or, x=230/46=5
Marks of C in physics and chemistry are 15* 5 = 16. Answer: D
75 and 16* 5 = 80. Quantity I
So, marks of B in chemistry are 12 * 5 = 60. Required percentage =
So, the mark of C in math is 75. [(85+70+75+90)/(80+85+75+80)]*100=100%
Marks of B in math are = 75*113.33/100 = 85. Quantity II
Marks of A in math are = 85*16/17 = 80. Required percentage =
Marks of A in physics are = 80*17/16 = 85. [(65+60+80+70)/(80+85+75+80)]*100
Marks of A in chemistry are = 85-20 = 65. =85.9375%
Marks of B in Physics are 60*7/6 = 70. Quantity I>Quantity II
Marks of D in math are = 75*106.66/100 = 80.
Marks of D in chemistry are 60*116.66/100 = 70. 17. Answer: A
Marks of D in Physics are = 75*6/5 = 90. Let the cost price of A and B is 400x and 500x
13. Answer: C So, 400x*160/100 -40 =400x*156.66/100
Marks of A in English is = 80*120/100=96 Or, 13.36x=40
Marks of B in English is = 96*3/4=72 Or, x=40/13.36=3
Marks of B in Biology is = 72*125/100=90 So, the cost price of item A is 400*3=1200 cost
So, from I, marks of A in biology are = price of item B is 500*3=1500
90*10/9=100  possible Quantity I
90*11/9=110  not possible

Click Here For Bundle PDF Course | support@guidely.in Page 11 of 12


Bank Po Mains PDF Course 2024
Quantitative Aptitude Day -12 (Eng)

So, selling price of A is = 1200*150/100 - Quantity II


40=1760 Age of E is = 30-2=28 years
So, profit percentage = [1760- Age of F is = 35-3=32
1200]*100/1200=46.66% Average age of E and F is = [28+32]/2=30 years
Quantity II Quantity I = Quantity II
Selling price of item B is = 1500*160/100 -
80=2320 19. Answer: B
So, profit percentage = [2320- Series I – 158, 290, 446, 628, 838, A
1500]*100/1500=54.66% 158+121+11=290, 290+144+12=446,
Quantity I < Quantity II 446+169+13=628, 628+196+14=838,
838+225+15=1078=A
18. Answer: E Series II – A=1078, B=1078+121+11=1210,
Let the ages of A and B be 2x and 3x. C=1210+144+12=1366, D=1366+169+13=1548
Age of C is 3x+5
So, 2x+5/3x+5=5/7 20. Answer: A
Or, 14x+35=15x+25 Series I –78,180,408,840, ?
Or, x=10 78*2+24=180, 180*2+48=408, 408*2+24=840,
So, the age of A and B is 20 and 30 years 840*2+48=1728
respectively. Series II- 145,155,166,179,196, ?
Age of C is 30+5=35 years. 145+1+4+5=155, 155+1+5+5=166,
Quantity I 166+1+6+6=179, 179+1+7+9=196,
Age of D is = [20+30]/2=25 years 196+1+9+6=212
Average age of C and D is = [25+35]/2=30 years So, the required sum = 1728+212=1940

Click Here For Bundle PDF Course | support@guidely.in Page 12 of 12


Bank Po Mains PDF Course 2024
English Day - 12

English Language
Directions (1-4): In the questions given below, B. As the sun dipped below the horizon, the city's
four statements are given that are labelled as skyline became a mesmerizing tapestry of
(A),(B),(C) and (D), with one statement being twinkling lights.
grammatically and contextually incorrect. You C. The old oak tree in the park provided shade
are required to choose that sentence which is and solace to those seeking refuge from the
erroneous. If all sentences are correct, then scorching summer sun.
choose ‘option E’ as your answer. D. With a backpack full of camping gear and a
1. sense of adventure, the journey embarked into
A. A sudden downpour transformed the dusty the heart of the wilderness.
streets into glistening ribbons of water, bringing A. A
relief to the parched earth and rejuvenating the B. B
surrounding landscape. C. C
B. After a long day at work, he sank into his D. D
favourite armchair, sips on a hot cup of tea, and E. All are correct
savours the quiet moments of solitude.
C. In the heart of the bustling city, a small park 3.
provided a peaceful oasis where people could A. The gentle chirping of the insects on the
escape the urban chaos and unwind amidst summer night provided a soothing backdrop to
nature. their campfire stories.
D. The ancient ruins of the Mayan civilization B. The vibrant flowers in the garden swayed in
stood as a testament to the ingenuity and the breeze, creating a symphony of colour and
architectural prowess of a bygone era. motion.
A. A C. With determination and a clear plan in mind,
B. B she embarked a journey to achieve her long-held
C. C dreams.
D. D D. As the first snowflake touched the ground, a
E. All are correct sense of wonder and anticipation filled the
children's hearts.
2. A. A
A. The aroma of freshly baked cookies wafted B. B
through the house, tempting everyone to take a C. C
sweet break. D. D
E. All are correct

Click Here For Bundle PDF Course | support@guidely.in Page 1 of 12


Bank Po Mains PDF Course 2024
English Day - 12

outcomes was the decision not to exercise the


4. Council’s power to tax extra neutral alcohol
A. The scent of blooming flowers and the (ENA) used for alcoholic liquor. With alcohol for
chirping of birds signalled the arrival of spring in human consumption still outside the GST net, the
the quiet, suburban neighbourhood. indirect tax levy on ENA or high strength potable
B. With a canvas and a palette of vibrant colours, alcohol — a key ingredient — could not be set off
she set out to create a masterpiece that would against State levies on the final product. Industry
expressed her deepest emotions. had been seeking for clarity on this vexed issue
C. The sun dipped below the horizon, casting for years, with courts taking varying positions.
long shadows across the tranquil meadow. It is heartening that the Council, that met just
D. After a long day of hiking, they gathered twice in 2022, has met four times this year, and
around the campfire, shared stories, and roasted thrice in just four months, even if a few agenda
marshmallows under the starry night sky. items pertained to fixing anomalies in recent
A. A decisions. With the age norms for the president
B. B and members of the long-awaited GST Appellate
C. C Tribunals now harmonised with other tribunals —
D. D a clearly avoidable oversight — one hopes they
E. All are correct will become operational soon. For consumers
and producers, however, the biggest matter of
Directions (5-11): Read the given passage concern should be the Council’s resolve to meet
carefully and answer the following questions. at a future date exclusively to discuss what
Some words are highlighted to help you locate Finance Minister Nirmala Sitharaman termed
them while answering the questions. ‘perspective planning’ on the GST Compensation
The Goods and Services Tax (GST) Council last Cess and what kind of surcharge it could be
Saturday lifted the haze on about a dozen tax replaced with. Originally packaged as a time-
treatment ambiguities, some of which have bound levy on top of a ‘Good and Simple Tax’ to
lingered since the indirect tax regime’s launch in compensate States for revenue losses for the
July 2017, such as the tax on corporate and first five years of GST, the COVID-19 pandemic’s
personal guarantees for bank loans. It slashed hit on tax collections had triggered an extension
the GST on molasses from 28% to 5%, with a of the Cess levied on so-called demerit goods
view to lower cattle feed costs and ease up cash such as aerated drinks, tobacco products and
flows for sugar mills so they may pay farmers’ automobiles, till March 2026. Discouraging some
dues faster. Rate tweaks and spring-cleaning sin goods may be desirable. However, ringing in
clarifications apart, one of the significant a new cess must not be done in isolation, but as

Click Here For Bundle PDF Course | support@guidely.in Page 2 of 12


Bank Po Mains PDF Course 2024
English Day - 12

a part of the broader rationalisation of GST’s 7. According to the passage, what is one of the
complex multiple-rate structure. That key issues with the current GST regime that
rationalisation exercise, initiated two years ago, needs attention, leading to discussions in the
unfortunately remains off the table despite robust passage?
revenue inflows in recent times. Frequent tweaks A. The need to promote the consumption of
of irritants aside, the GST regime needs a holistic demerit goods, despite their negative impact on
reform plan, including a road map to bring in health.
excluded items such as electricity, petroleum and B. The extension of Cess on all goods,
alcohol. particularly in light of the ongoing COVID-19
5. What is one of the noteworthy results of the pandemic.
most recent GST Council meeting, as indicated C. The complexity of the multiple-rate structure,
in the passage? which requires simplification and rationalization.
A. The reduction of GST on molasses from 5% to D. The absence of clarity on the taxation of ENA,
28%. a critical ingredient in alcoholic liquor.
B. The Council's choice not to impose taxes on E. The increase in tax collections during the
extra neutral alcohol (ENA). pandemic, necessitating a reevaluation of
C. The decision to tax corporate guarantees for revenue sources.
bank loans.
D. The clarification of tax treatment ambiguities 8. Which action, as per the author, is considered
related to GST. positive or encouraging?
E. The inclusion of alcohol for human A. The Council has decided to discontinue
consumption in the GST net. meetings due to various anomalies.
B. The Council has harmonized age norms for
6. Why was the Cess on demerit goods extended other tribunals.
until March 2026, as mentioned in the passage? C. The implementation new policies without
A. To discourage the consumption of sin goods. discussion.
B. To provide additional revenue for the D. The Council has decided to increase the age
government. limits for GST Appellate Tribunal members.
C. To simplify the GST's multiple-rate structure. E. The increased frequency of meetings by the
D. To compensate states for revenue losses due GST Council in the current year.
to the GST.
E. To promote the consumption of aerated drinks 9. What is the tone of the given passage?
and tobacco products. A. Critical
B. Derisive

Click Here For Bundle PDF Course | support@guidely.in Page 3 of 12


Bank Po Mains PDF Course 2024
English Day - 12

C. Obsequious I. Despite, but also


D. Sarcastic II. In spite of, and also
E. Hypothetical III. Meanwhile, but also
IV. As a result of, on the other hand
10. What is the antonym of the word A. Only II
‘ANOMALIES’ as used in the passage? B. Only III
A. Irregularities C. Only I
B. Conformities D. Both I and IV
C. Discrepancies E. None of these
D. Inconsistencies
E. Aberrations 13.
A. John thought he had made a mistake
11. What is the meaning of the phrase ‘off the B. his decision turned out to be the right one in
table’ as used in the passage? the end
A. No longer relevant C. it pleasantly surprised everyone.
B. Still under consideration I. since, but
C. Highly recommended II. hence, and on the other hand
D. Currently in progress III. conversely, but
E. Open for discussion IV. but, and as a matter of fact
Directions (12-15): In the following questions, A. Only III
three phrases/sentences are given (A, B and C) B. Only IV
followed by four connectors (I, II, II and IV). C. Only I
Choose the option that represents the correct D. Both II and IV
combination of connectors that can be used to E. None of these
join the given sentences without changing their
original meaning. If none of the connectors is 14.
correct, choose option E as your answer. A. the rain was pouring down relentlessly from
12. the overcast sky,
A. the challenging economic conditions, B. I decided to go for a run in the park,
B. the company not only managed to maintain its determined to stay committed to my daily
market share exercise routine
C. expanded its global operations, launching new C. I soon regretted it when I got completely
products and ultimately securing its position as a soaked.
market leader A. Although, but

Click Here For Bundle PDF Course | support@guidely.in Page 4 of 12


Bank Po Mains PDF Course 2024
English Day - 12

B. Nevertheless, but 16. The government's decision to revise tax


C. Due to, in contrast policies has ___________ a substantial debate
D. Besides, subsequently among economists, with some arguing that the
A. Only III changes could ___________ economic growth.
B. Only IV A. stimulated
C. Both I and IV B. overhauled
D. Both I and II C. prompted
E. None of these D. hinder
E. stimulate
15. F. impair
A. Eager to enjoy the festivities and spend time A. AD
with friends, I had a strong desire to attend the B. BF
party. C. CE
B. My responsibilities were piling up, and my D. CD
workload had reached a point where it was E. None of these
simply impossible to set them aside.
C. I had no choice but to decline the invitation 17. The company's commitment to sustainability
with a heavy heart. goes beyond its environmental efforts; it also
I. Nonetheless, so extends to its _______ practices and its _______
II. Meanwhile, provided that in local community initiatives.
III. Furthermore, yet A. ethical
IV. Conversely, thus B. profitable
A. Only I C. innovative
B. Only III D. engagement
C. Both I and IV E. alienation
D. Both II and III F. participation
E. None of these A. AE
Directions (16-20): The given sentences carry B. AD
two blanks each which can be filled with the C. BF
words from the choices given below them. Read D. CE
them carefully and choose the most appropriate E. None of these
combination of words that makes the sentence
complete and meaningful. 18. The company's commitment to research and
development is commendable; it continuously

Click Here For Bundle PDF Course | support@guidely.in Page 5 of 12


Bank Po Mains PDF Course 2024
English Day - 12

invests in cutting-edge technology to ________ its F. forge


products and ________ its market position. A. AD
A. streamline B. BE
B. enhance C. CF
C. jeopardize D. Both BE and CF
D. bolster E. None of these
E. stagnate
F. sustain
A. AE 20. The chef's dedication to using fresh, locally-
B. CD sourced ingredients is what ________ the quality
C. BD of the restaurant's dishes and ________ its
D. CF reputation for excellence.
E. None of these A. elevates
B. sustains
19. The author's ability to craft compelling C. stabilizes
characters and weave intricate plotlines is what D. amplifies
________ readers to immerse themselves in the E. erodes
story and ________ a deep emotional connection. F. tarnishes
A. entices A. AD
B. fascinates B. BE
C. captivates C. CF
D. repels D. Both AD and CF
E. sever E. None of these

Click Here For Bundle PDF Course | support@guidely.in Page 6 of 12


Bank Po Mains PDF Course 2024
English Day - 12

Click Here to Get the Detailed Video Solution for the above given Questions
Or Scan the QR Code to Get the Detailed Video Solutions

Answer Key with Explanation


1. Answer: B The introductory phrase "With a backpack full of
The error is in sentence B in the wrong usage of camping gear and a sense of adventure" is
the verb forms. meant to describe the person or group
Parallelism or parallel construction is a embarking on the journey, but it's not properly
grammatical and stylistic rule that requires the connected to the subject of the sentence.
elements in a series or list to have the same The correct sentence is:
grammatical form. With a backpack full of camping gear and a
The parallelism rule for this sentence would sense of adventure, they embarked on a journey
require consistency in verb tense for the items in into the heart of the wilderness.
the series. In the corrected sentence, all verbs In this corrected version, the introductory phrase
should be in the past tense to maintain parallel clearly modifies the subject "they," and the
structure. modifier is properly connected to the subject,
Thus, the correct sentence is: resolving the dangling modifier problem. Thank
"After a long day at work, he sank into his you for pointing that out.
favourite armchair, sipped on a hot cup of tea, Therefore, option D is the correct answer.
and savoured the quiet moments of solitude."
Therefore, option B is the correct answer. 3. Answer: C
The error lies in sentence C as did not use the
2. Answer: D preposition ‘on’ after the verb ‘embarked’.
The error in the sentence D is a misplaced The word "embarked" is often followed by the
modifier, leading to a dangling construction. preposition "on" when it is used to mean "to start
or begin a journey or a significant endeavour."

Click Here For Bundle PDF Course | support@guidely.in Page 7 of 12


Bank Po Mains PDF Course 2024
English Day - 12

When you say someone "embarked on" neutral alcohol (ENA) when it is used in the
something, it indicates that they have started or production of alcoholic liquor.
undertaken a particular course of action, project, Therefore, option B is the correct answer.
or journey.
For example, She embarked on a new career in 6. Answer: D
medicine. In the passage, it is mentioned that the Cess on
Thus, the correct sentence is: demerit goods (such as aerated drinks, tobacco
With determination and a clear plan in mind, she products, and automobiles) was originally
embarked on a journey to achieve her long-held introduced as a time-bound levy to compensate
dreams. states for revenue losses during the first five
Therefore, option C is the correct answer. years of the Goods and Services Tax (GST)
regime. This compensation was necessary
4. Answer: B because the GST was expected to lead to
The error in sentence B is in the wrong usage of revenue losses for states initially.
modal-verb agreement. However, due to the impact of the COVID-19
Modal verbs like "would," "could," "should," pandemic on tax collections, the Cess on
"might," and "can" are followed by the base form demerit goods was extended until March 2026.
of the main verb (the infinitive form without "to"). This extension was primarily aimed at
Thus, the correct sentence is: compensating states for the revenue losses they
With a canvas and a palette of vibrant colours, experienced.
she set out to create a masterpiece that would Therefore, the correct answer is option D.
express her deepest emotions.
Therefore, option B is the correct answer. 7. Answer: C
In the passage, the author discusses the
5. Answer: B complexities of the current Goods and Services
In the given passage, it is mentioned that “Rate Tax (GST) regime, emphasizing that there is a
tweaks and spring-cleaning clarifications apart, need for reform and simplification. This includes
one of the significant outcomes was the decision addressing the issue of a multiple-rate structure,
not to exercise the Council’s power to tax extra which has been a subject of concern. The
neutral alcohol (ENA) used for alcoholic liquor.” passage suggests that the complex multiple-rate
Thus we can understand that the passage structure needs attention, and it is one of the key
mentions a significant decision made by the GST issues with the GST system that is highlighted in
Council, which is not to impose taxes on extra the passage.

Click Here For Bundle PDF Course | support@guidely.in Page 8 of 12


Bank Po Mains PDF Course 2024
English Day - 12

Therefore, option C is the correct answer.  Derisive: Mocking or scornful in a


disrespectful manner.
8. Answer: E  Obsequious: Excessively obedient or
It is mentioned in the passage that “The author servile, often to gain favor.
considers this action to be positive or  Sarcastic: Using irony or mock praise to
encouraging because it signifies that the GST convey contempt or humor.
Council is actively addressing issues and  Hypothetical: Involving a supposition or
challenges.” The phrase ‘it is heartening’ means assumption, not necessarily based on
‘The phrase "it is heartening" means providing fact.
comfort, encouragement or cheer to someone. It
is often used to express gratitude or relief. 10. Answer: B
The author considers this action to be positive or "Anomalies" refers to irregularities or deviations
encouraging because it signifies that the GST from what is considered normal or expected.
Council is actively addressing issues and Let’s learn the meanings of the given words:
challenges.  Irregularities: Departures from the norm or
Therefore, option E is the correct answer. standard.
 Conformities: Acts of complying with rules
9. Answer: A or standards.
The passage contains a critical assessment of  Discrepancies: Differences or
various aspects of the Goods and Services Tax inconsistencies between two or more
(GST) regime, including its complexity, the need things.
for rationalization, and the extension of the Cess  Inconsistencies: Lack of harmony or
on demerit goods. The author points out areas uniformity; contradictions.
where there is room for improvement and reform  Aberrations: Departures from what is
in the GST system. considered normal or typical.
Overall, the tone combines the presentation of Thus, we can see that ‘conformities’ is the
information with a critical assessment of the antonym of the given word.
current GST system and a cautious sense of Therefore, option B is the correct answer.
optimism for future improvements.
The meanings of the given words: 11. Answer: A
 Critical: Expressing disapproval or The phrase "off the table" is an idiomatic
pointing out flaws and shortcomings. expression that means an option or possibility is
no longer being considered or is not currently

Click Here For Bundle PDF Course | support@guidely.in Page 9 of 12


Bank Po Mains PDF Course 2024
English Day - 12

available for discussion or consideration. It belief that he made a mistake, his decision
signifies that a particular idea or proposal has turned out to be the right one, and this was
been excluded or dismissed from consideration. pleasantly surprising. The "as a matter of fact"
Therefore, option A is the correct answer. part adds an emphasis on the surprising aspect.
Thus, the sentence formed is:
12. Answer: C John thought he had made a mistake but his
The phrase "Despite" effectively connects the decision turned out to be the right one in the end
introductory phrase to the subsequent actions and as a matter of fact, it pleasantly surprised
and outcomes, showing a contrast between the everyone.
challenging conditions and the company's Therefore, option B is the correct answer.
achievements. "But also" is an appropriate
connector that indicates that in addition to 14. Answer: D
maintaining market share, the company ‘Nevertheless, although, but’ all three are
expanded its operations and secured its position contrasting connectors and are used to connect
as a market leader. two opposite ideas or sentences. Here, ‘although
The other options are not suitable connectors for and nevertheless’ come at the beginning of
the given sentence or do not maintain the sentence A, to convey the idea that despite the
intended meaning and coherence of the adverse weather conditions, the person decided
passage. to go for a run.
Thus, the sentence formed is: ‘But’ is used to connect sentences B and C to
Despite the challenging economic conditions, the highlight the contrast between the person’s
company not only managed to maintain its determination to exercise and the regret that
market share but also expanded its global followed as a consequence of their decision.
operations, launching new products and Thus, the sentence formed is:
ultimately securing its position as a market Although/ nevertheless the rain was pouring
leader. down relentlessly from the overcast sky, I
Therefore, option C is the correct answer. decided to go for a run in the park, determined to
stay committed to my daily exercise routine but I
13. Answer: B soon regretted it when I got completely soaked.
"But" introduces a contrast, and "and as a matter Therefore, option D is the correct answer.
of fact" is used to provide additional information
or clarification. In this context, this combination 15. Answer: A
works well. It shows that despite John's initial

Click Here For Bundle PDF Course | support@guidely.in Page 10 of 12


Bank Po Mains PDF Course 2024
English Day - 12

"Nonetheless" is used to indicate a contrast or changes in tax policies could have a negative
contradiction between statements I and II. In this effect on economic growth. "Hinder" is the
context, it conveys the idea that despite the correct word for this context, as it means to
strong desire to attend the party, there were create obstacles or restrictions that slow down or
obstacles (piling responsibilities and an impede progress.
overwhelming workload) that made it Therefore, option D is the correct answer.
impossible.
"so" is used to show the cause-and-effect 17. Answer: B
relationship between the increasing The first blank should be filled with a word that
responsibilities and the necessity to decline the describes the company's practices as being in
party invitation. It conveys that because the line with moral principles or ethical standards.
workload became unmanageable, the person "Ethical" is the appropriate word for this context.
had no other option but to decline. The second blank should be filled with a word
Thus, the sentence formed is: that indicates the company's active involvement
Eager to enjoy the festivities and spend time with or participation in local community initiatives.
friends, I had a strong desire to attend the party. "Engagement" is the correct word for this
Nonetheless, my responsibilities were piling up, context.
and my workload had reached a point where it Therefore, option B is the correct answer.
was simply impossible to set them aside. So, I
had no choice but to decline the invitation with a 18. Answer: C
heavy heart The first blank should be filled with a word that
Therefore, option A is the correct answer. indicates the company's efforts to improve its
products. "Enhance" is the correct word in this
16. Answer: D context as it means to improve, elevate, or make
In the given sentence, the first blank should be better.
filled with a word that describes the The second blank should be filled with a word
government's decision as a catalyst for or the that suggests strengthening or fortifying the
cause of the substantial debate among company's market position. "Bolster" is the right
economists. "Prompted" is the appropriate word word for this context as it means to support or
here, as it means to cause or initiate a reinforce.
discussion or action. Therefore, option C is the correct answer.
In the second blank, you need a word that
indicates that some economists believe that the

Click Here For Bundle PDF Course | support@guidely.in Page 11 of 12


Bank Po Mains PDF Course 2024
English Day - 12

19. Answer: C The word ‘elevates’ in this context, means ‘to


The word ‘captivates’ means to attract and hold raise or lift up, and it is used to emphasize how
the interest and attention of someone. In the the chef's dedication significantly improves or
context of the sentence, it's used to describe raises the quality of the restaurant's dishes’. It
how the author's ability to craft compelling conveys the idea that the use of fresh, locally-
characters and intricate plotlines attracts and sourced ingredients takes the quality to a higher
holds the interest of readers. level.
The word ‘forge’ in this context, means to create The word "amplifies" means to increase the
or develop a deep emotional connection. It's strength, intensity, or significance of something.
used to convey that the author's storytelling skills In this sentence, it is used to emphasize how the
lead readers to form or create a strong and deep chef's dedication enhances or intensifies the
emotional connection with the story or its restaurant's reputation for excellence.
characters. It conveys the idea that the restaurant's excellent
Therefore, option C is the correct answer. reputation becomes even more prominent or
significant due to the chef's commitment to high-
20. Answer: A quality ingredients.
Therefore, option A is the correct answer.

Click Here For Bundle PDF Course | support@guidely.in Page 12 of 12


Bank Po Mains PDF Course 2024
Reasoning Day -13 (Eng)

Reasoning Aptitude
Directions (1-5): A string of letters are given as 2) How many letters are there in the English
input. Some conditions are given in four different alphabetical series between the sixth letters from
steps. Study the following information carefully both ends in step III?
and answer the given questions. a) Eight
Input: D G A F L I W P E Y S U H C Z O M J X B b) Fifteen
RKNTVQ c) Twelve
Step I: The first half of the letters is written in d) Three
reverse order after the second half of the letters. e) None of these
Step II: All the letters are changed to their
corresponding reverse letter as per the 3) How many consonants from the second half of
alphabetical series. the English alphabetical series are in step IV,
Step III: If the letters which come in the second which are immediately followed by a vowel?
half of the alphabetical series are immediately a) Four
followed by a vowel, then the letters are changed b) Two
to the second previous letter as per the c) Three
alphabetical series, else if the letters which come d) One
in the first half of the alphabetical series are e) None
immediately preceded by a vowel, then the
letters are changed to the third succeeding letter 4) Which of the following element is eight from
as per the alphabetical series. the right end in step IV?
Step IV: All the letters that appeared more than a) B
once are dropped from the series and the b) F
remaining letters are written in reverse order. c) P
Step IV is the final output of the given input. d) E
1) Which of the following letter is fifth to the left of e) None of these
the second vowel from the right end in step II?
a) I 5) _____ is fourth to the ____ of the letter which is
b) M _____ from the left end of step IV.
c) B a) Z, left, sixth
d) K b) F, right, Second
e) None of these c)G, Right, Seventh
d) Both a and c
e) Both a and b

Click Here For Bundle PDF Course | support@guidely.in Page 1 of 10


Bank Po Mains PDF Course 2024
Reasoning Day -13 (Eng)

Directions (6-10): Study the following information a) Only III


carefully and answer the given questions. b) All I, II, and III
A school conducts eight matches of four different c) Both II and III
sports on the sports day. The matches were held d) Both I and III
between 8 AM to 6 PM. Two matches for cricket, e) None of these
two matches for volleyball, three matches for
football, and one match for badminton. Each 8) Which of the following match was held
match was held for one hour. There are two immediately before Tea break?
breaks between the matches viz.- 1 PM – 2 PM a) Football
lunch break and tea break at 4 PM-5 PM. b) Cricket
Note: No two consecutive matches were held for c) Badminton
the same sport. d) None
One of the Cricket matches was held after the e) Volleyball
lunch break. Two Football matches were held at
a gap of 120 minutes. None of the football 9) Football is not played at which of the following
matches were held immediately after and time?
immediately before lunch break. Badminton a) 5 PM-6 PM
match was held immediately after one of the b) 11 AM-12 AM
cricket matches but not held after the lunch c) 2 PM-3 PM
break. Only one match was held between both d) 8 AM-9 AM
Volleyball matches. e) None of these
6) Badminton match was held at which of the
following time? 10) How many matches were held before the first
a) 12 PM-1 PM cricket match?
b) 9 AM-10 AM a) Three
c) 3 PM-4 PM b) One
d) 10 AM-11 AM c) Two
e) None of these d)Four
e) None
7) Which of the following match was held
between Lunch and Tea break? Directions (11-15): Study the following
I. Cricket information carefully and answer the questions
II. Football given below.
III. Volleyball

Click Here For Bundle PDF Course | support@guidely.in Page 2 of 10


Bank Po Mains PDF Course 2024
Reasoning Day -13 (Eng)

Certain number of persons from the same family b) Fifteen


of three generations are sitting in a row facing c) Twelve
north. Information about only a few persons are d) Seventeen
known. Not more than three known persons are e) Ten
sitting together. Some of the person’s relations
may or may not be known. 13) How K is related to the one who sits
B, who is the only daughter-in-law of K, sits five immediate right of Y?
places away from R. R sits second to the right of a) Mother-in-law
K. As many persons sit between B and K as b) Grandmother
between K and G, who is the only daughter of K. c) Aunt
G sits third from one of the extreme ends. L, who d) Nephew
is the only son of G, sits third to the right of the e) None of these
one who sits immediate left of G.L sits fourth to
the left of M, who is married to B. The one who 14) Four of the following five are alike in a certain
sits third to the right of M is the father of G. Only way based on the given arrangementand thus
one person sits between M and W, who is the form a group. Which one of the followingdoes not
father of L. Only six persons sit between W and belong to the group?
Q, who is the only daughter of M. Q and Y sit a) PW
together. Only four persons sit between Y and Z, b) UM
who sits at the extreme end of the row. As many c) QB
persons sit between Z and K as between Q and d) WK
U. U sits four places away from P, who is the e) MR
sibling of W’s spouse’s niece.
11) ______ is the _______ of the one who sits 15) If Y is the sister-in-law of R, who has no
fourth to the left of _____. siblings, then how Y is related to the one who
a)W, Uncle, R sits third to the right of U?
b) P, Son, Q a) Maternal Aunt
c) G, Mother, M b) Brother
d) Both A and C c) Sister
e) Both A and B d) Daughter
e) None of these
12) How many persons are sitting to the right of
B’s sister-in-law?
a) Eight

Click Here For Bundle PDF Course | support@guidely.in Page 3 of 10


Bank Po Mains PDF Course 2024
Reasoning Day -13 (Eng)

Directions (16-20):Read the given passage 17) Which of the following can be a probable
carefully and answer the questions based on the ‘Course Of Action’ to pacify the anxious people?
same respectively a) All the protestors should be jailed so that more
In scenes reminiscent of the Arab Spring, (A) people don’t join the protests
citizens stormed the residences of Sri Lanka’s b) The protestors should be promised with free
president and prime minister in Colombo, (B) food and basic facilities for all of them
forcing both leaders to announce that they would c) Present heads of the country should resign
quit their respective offices. President Gotabaya with immediate effect
Rajapaksa, the main target of the protests that d) A new government should take charge and try
erupted in April over food and fuel shortages and to restore the trust of the people
have continued since, promised to resign by e) The protestors should be given employment
Wednesday. The Speaker of Sri Lanka’s which would ensure consistent money flow
parliament, Yapa Abeywardena, is expected to
take over as acting president and prime minister, 18) Which option can be marked as an
and facilitate, possibly, the setting up of a ‘Inference’ as per the contents of the given
national unity government. Gotabaya’s paragraph?
resignation could bring down the temperature a) The people of Sri Lanka have lost trust in their
and help the authorities to persuade the leadership
protestors to return home. Though the marches b) Gotabaya’s presence in the government has
and sit-ins in Galle Face and elsewhere have sparked ire
been largely peaceful, the recent violent events c) People of the island country have become
suggest that the people, battling acute shortage anxious
of essentials, including milk, baby food, petrol, d) Both (a) and (b)
are on the edge. Only a new leadership can win e) None of the given options
back public trust in government and steer the
island nation’s economy out of choppy waters. 19) Which of the given statements can most
16) Which of the given options provides the likely be taken as an ‘assumption’?
correct relationship between sentences A and B? a) The speaker takes over the roles of the
a) A is the cause and B is its effect estranged leaders till the government is formed
b) A is the effect and B is one of the causes b) An island country can only be ruled by a unity
c) Both A and B are causes government
d) Both A and B are effects of independent c) Violent protests were the only way left before
causes the people to show their agony
e) A is the effect and B is its cause

Click Here For Bundle PDF Course | support@guidely.in Page 4 of 10


Bank Po Mains PDF Course 2024
Reasoning Day -13 (Eng)

d) The last resort to save the economy was to b) A new political leadership is needed to steer
oust their leaders out of power Sri Lanka out of present impasse
e) All statements can be assumed c) The violent protests are a result of the
negligence of the government
20) Which of the following can be ‘concluded’ d) The food scarcity is a proof that the country
from the given paragraph? was being mismanaged
a) It is only the citizens who can bring the country e) Graft had taken the centre stage and the
out of the severe trouble economy was in doldrums while the government
enjoyed
Click Here to Get the Detailed Video Solution for the above given Questions
Or Scan the QR Code to Get the Detailed Video Solutions

Answer Key with Explanation


Direction (1-5): For Step II: All the letters are changed to their
1) Answer: C corresponding reverse letter as per the
2) Answer: C alphabetical series.
3) Answer: C Step II: X A L N Q C Y I P M G E J S FH B V K D
4) Answer: D ROUZTW
5) Answer: D For Step III: If the letters which come in the
We have: second half of the alphabetical series are
Input: D G A F L I W P E Y S U H C Z O M J X B immediately followed by a vowel, then the letters
RKNTVQ are changed to the second previous letter as per
For Step I: The first half of the letters are written the alphabetical series, else if the letters which
in reverse order after the second half of the come in the first half of alphabetical series are
letters. immediately preceded by a vowel, then the
Step I: C Z O M J X B R K N T V Q H U S Y E P letters are changed to the third succeeding letter
WILFAGD as per the alphabetical series.

Click Here For Bundle PDF Course | support@guidely.in Page 5 of 10


Bank Po Mains PDF Course 2024
Reasoning Day -13 (Eng)

Step III: V A O N Q C W I P M G E M S F HB V  Two Football matches were held at a gap


KDPMUZTW of 120 minutes.
For Step IV: All the letters that appeared more  None of the football matches were held
than once are removed and the remaining letters immediately after and immediately before
are written in reverse order. lunch break.
Step IV: T Z U D K B H F S E G I C Q N O A That means, in case (1) Cricket match
was held immediately after the lunch
Direction (6-10): break, in case (2) Cricket match was held
6) Answer: D two matches after the lunch break, in
7) Answer: D case (3) Cricket match was held four
8) Answer: E matches after the lunch break.
9) Answer: C Based on the above given information we have:
10) Answer: B

Again, we have:
 Badminton match was held immediately
after one of the cricket matches but not
held after the lunch break.
That means, badminton match must be held at
10 AM.
We have:  Only one match was held between both
 One of the Cricket matches was held after Volleyball matches.
the lunch break. That means, case (2) is not valid.

Click Here For Bundle PDF Course | support@guidely.in Page 6 of 10


Bank Po Mains PDF Course 2024
Reasoning Day -13 (Eng)

Based on the above given information we have: 13) Answer: B


14) Answer: A (Only two persons sit between the
given pair of persons, except option A)
15) Answer: A

We have:
 B, who is the only daughter-in-law of K,
sits five places away from R.
Again, we have:  R sits second to the right of K.
Since, three football matches were held, and That means, in case (1) B sits fifth to the
none of the football match was held immediately left of R, in case (2) B sits fifth to the right
before or after lunch break, thus case (2) and of R.
case (3) is not valid.  As many persons sit between B and K as
Based on the above given information we have: between K and G, who is the only
daughter of K.
 G sits third from one of the extreme ends.
Based on the above given information we have:

Linear Arrangement:

Again, we have:
Direction (11-15):  L, who is the only son of G, sits third to
11) Answer: C the right of the one who sits immediate
12) Answer: B left of G.

Click Here For Bundle PDF Course | support@guidely.in Page 7 of 10


Bank Po Mains PDF Course 2024
Reasoning Day -13 (Eng)

 L sits fourth to the left of M, who is


married to B.
For Blood relation:
That means in case (2) M sits immediate
left of K, case (1) is not valid.
Based on the above given information we have:

Again, we have:
 Only four persons sit between Y and Z,
who sits at the extreme end of the row.
 As many persons sit between Z and K as
between Q and U.
For linear arrangement: That means, in case (2) U sits immediate
right of L, in case (2a) U sits immediate
left of L.
 U sits four places away from P, who is the

Case (1) is not valid as no place for M. sibling of W’s spouse’s niece.

Again, we have: That means, in case (2) P sits immediate

 The one who sits third to the right of M is left of G, case (2a) is not valid.

the father of G. Based on the above given information we have:

 Only one person sits between M and W,


who is the father of L.
Since not more than three known persons
are sitting together, thus W doesn’t sit
adjacent to R.
 Only six persons sit between W and Q,
who is the only daughter of M. Linear arrangement:
 Q and Y sit together.
That means, in case (2) Y sits immediate
right of R, in case (2a) Y sits third to the
right of R.
Based on the above given information we have:

Click Here For Bundle PDF Course | support@guidely.in Page 8 of 10


Bank Po Mains PDF Course 2024
Reasoning Day -13 (Eng)

Case(2a) is not valid as not more than three It has been given in the paragraph that the
known persons are sitting together. people have become anxious as they have no
access to even the basic necessities so as per
16) Answer: E the content above, it has been said that a new
The first sentence ie; A talks about the crowd leadership should quickly take charge. Option (c)
that barged into the residences of the President could be correct had the issue been stopping the
and the PM. If we wonder, what made them do protests. But the issue asked is mentioned at the
the same? We get our answer for this is B which end of the paragraph. (e) becomes superfluous
says that the citizens wanted that these two looking at the situation given in the paragraph.
leaders should leave their offices. So, we can So, the correct answer is option (d).
say that; because of B  A took place…
We can clearly understand that, A is the effect 18) Answer: D
and B is the cause. We have two options, (b) An Inference is something that can be deduced
and (e) to confuse from. on the basis of some known fact. It is not
If we go with (b) which says B is one of the something that is directly given in the paragraph.
causes, the other causes than this one should This becomes the reason that we cancel the
also be mentioned which is not the case. B is the third option as it has been discussed directly at
only immediate cause that has been quoted. multiple places in the paragraph.
So, this makes option (e) the best choice for the Towards the end of the paragraph, it has been
answer. given that a new government can restore the
trust of the people to inspire them to stop their
17) Answer: D violent protests. From this, we can infer that the
A Course of Action should always be inclined first option should be correct.
towards lessening the problem and never an It has been given that Gotabaya’s resignation
extreme step. The option that one chooses would bring down the anger. From this we can
should either solve the issue given in the context infer that his presence in the government has
or at least lessen the intensity of the same. sparked a rage in the country.
The first option is an extreme step which should Thus, of the three options, we can clearly see
be avoided as it won’t make any difference. that the first two can be inferred and the third
Option (b) is not a solution as it would put a lot of can be left out. So, the correct answer is option
pressure on the country and increase the (d).
problem indirectly.
19) Answer: A

Click Here For Bundle PDF Course | support@guidely.in Page 9 of 10


Bank Po Mains PDF Course 2024
Reasoning Day -13 (Eng)

An assumption should be chosen in the reverse


order. We must start with the options and try to 20) Answer: B
find that if the particular option were assumed, A conclusion is logically deduced based on the
would that lead us to any conclusion given in the contents of the given passage. The whole
paragraph or the sentences given. Like for paragraph revolves around one single idea that
example; the issues that have led to protests can be
If we take the first option to be an assumption, addressed only with the change in the
we can very well conclude the point given in the government. The people have lost trust in their
paragraph that the speaker Yapa would be governance as they have been left at the mercy
taking over the charge till the government is of no one.
formed. So, this is correct. The first option is superfluous and can be ruled
Option (b) cannot be taken as an assumption as out. Option (c) is correct as given in the
nothing about the same or related to the same paragraph but it cannot be taken as the
has been clearly mentioned. It has nowhere conclusion. Same goes for option (d).
been mentioned that the violent protests were We have no clue about graft and corruption
the only way out. Even the statement in (d) which makes it wrong for us to mark option (e).
doesn’t count for an assumption as it has directly So, it can be eliminated.
been discussed as an issue in the paragraph. Thus, the correct answer is option (b).
So, the best would be to mark option (a) as the
answer.

Click Here For Bundle PDF Course | support@guidely.in Page 10 of 10


Bank Po Mains PDF Course 2024
Quantitative Aptitude Day -13 (Eng)

Quantitative Aptitude

Directions (01 - 05): Study the following information carefully and answer the questions given below.
There are five companies A, B, C, D and E. Each company has two offices in the city Mumbai and Delhi.
Total number of employees of Mumbai and Delhi office of each company and difference of employees of
Mumbai and Delhi office are given in bar graphs. There are three posts [HR, IT, and Marketing] in each
branch. Ratio of number of in HR, IT and Marketing of each office of each branch is given in table.

Notes – Number of employees in Mumbai office of company A and B is more than that of Delhi office and
Number of employees in Delhi office of company C, D and E is more than that of Mumbai office.

Click Here For Bundle PDF Course | support@guidely.in Page 1 of 11


Bank Po Mains PDF Course 2024
Quantitative Aptitude Day -13 (Eng)

1) Ratio of total number of male and female c) Difference between the number of employees
employees of Mumbai office of company A is 3:2, in marketing post of company E in Mumbai and
3:5 and 3:1 in HR, IT and Marketing posts. Ratio the Delhi office is 18.
of total number of male and female employees of d) Difference of average number of employees in
the Delhi office of company A is 2:3, 1:3 and 3:2 IT post all companies together in Mumbai office
in HR, IT and Marketing posts. Find the and average number of employees in Marketing
difference between the total number of males post all companies together in Mumbai office is
and total number of females of company A? 15 .
a) 16 e) None of these
b) 19
c) 12 4)
d) 18 Quantity I: Total number of employees in
e) 24 Marketing post in Delhi office of companies A, B
and C together is what percent of total number of
2) Find which one is true? employees in IT post of companies B, C and D
I. Difference in total employees in IT posts in together in Mumbai office?
Mumbai of all companies together and total Quantity II: Total number of employees in IT post
employees in IT posts In Delhi of all companies in Delhi office of companies A, B and C together
together is 14. is what percent of total number of employees In
II. Total employees in the Marketing post of all HR post of companies B, C and D together in
companies together are 306. Mumbai?
III. Average number of employees in the Delhi a) Quantity I > Quantity II
office of all companies together is 120. b) Quantity I ≥ Quantity II
a) Only I and II are true c) Quantity II > Quantity I
b) Only I is true d) Quantity II ≥ Quantity I
c) Only I and are III true e) Quantity I = Quantity II or Relation cannot be
d) All are true establish
e) All are false ed

3) Find which one is true? 5) Find the difference between the total number
a) Ratio of number of employees in HR post of of employees in companies A, B and C together
company C in Mumbai and Delhi office is 2:3. in the Mumbai office and the total number of
b) Total number of employees in HR post in employees in companies C, D and E together in
Mumbai office of all companies is 300. the Delhi office?

Click Here For Bundle PDF Course | support@guidely.in Page 2 of 11


Bank Po Mains PDF Course 2024
Quantitative Aptitude Day -13 (Eng)

a) 0 d) 3
b) 1 e) None of these
c) 2

Directions (06 - 10): Study the following information carefully and answer the questions given below.
The line graph shows the percentage of registered voters who cast their vote, the percentage of invalid
votes out of total cast votes, and the percentage of party A's votes out of a valid vote in five cities. The
total number of registered voters in each city is 30000. There is only party A and Party B contest in the
election.

6) Find the sum of the difference of votes got by and the average number of valid votes of all five
parties A and B in all cities together? cities together?
a) 20152 a) 2150
b) 26530 b) 2650
c) 23820 c) 2840
d) 21300 d) 2370
e) 26520 e) 2850

7) Find the difference between the average 8) In city F, the total number of populations is
number of cast votes of all five cities together 35000, and the number of registered voters is
the same as mentioned in the question. The ratio

Click Here For Bundle PDF Course | support@guidely.in Page 3 of 11


Bank Po Mains PDF Course 2024
Quantitative Aptitude Day -13 (Eng)

of valid votes of cities F and R is 4:3 and party A respectively. The speed of boat P, Q, and R in
got 55% of the valid vote then find the number of still water is x km/hr, y km/hr, and z km/hr
votes party B got in city F? respectively. On Monday, boat P goes 66 km
a) 11250 downstream and 56 km upstream in 7 hours and
b) 10800 88 km downstream and 28 km upstream in 6
c) 14200 hours. On Tuesday, boat Q goes 44 km
d) 13000 downstream and 60 km upstream in 8 hours and
e) None of these 88 km downstream and 70 km upstream in 11
hours. On Wednesday, boat R goes 64 km
9) Find the ratio of the total number of people downstream and 32 km upstream in 4 hours and
who did not cast their vote in all cities together 96 km downstream and 64 km upstream in 7
and the number of invalid votes in all cities hours.
together? 11) On Tuesday, boat P goes 5z km downstream
a) 200:79 and boat R goes 8c km upstream. Find the
b) 212:25 difference in time taken by two boats?
c) 241:26 a) 5(4/9)hrs
d) 241:22 b) 3(4/9)hrs
e) None of these c) 1(4/9)hrs
d) 2(4/9)hrs
10) Find the difference between the percentage e) 4(4/9)hrs
of party B's votes out of the total registered
voters in city P and the percentage of party A's 12) Find the ratio of the total distance covered by
votes out of the total registered voters in city Q? boat Q on Monday in b hours downstream and
a) 17.75% the total distance covered by boat R on Monday
b) 15.2% in c hours upstream?
c) 17.8% a) 2:3
d) 16.5% b) 4:5
e) None of these c) 5:6
d) 3:4
Directions (11 - 15): Study the following e) 1:2
information carefully and answer the questions
given below. 13) On Monday,Boat P goes from point M to N
The speed of the stream of river on Monday, downstream of the river. O is the middle point of
Tuesday, and Wednesday is a, b, and c km/hr M and N. From M to O, speed of the stream is

Click Here For Bundle PDF Course | support@guidely.in Page 4 of 11


Bank Po Mains PDF Course 2024
Quantitative Aptitude Day -13 (Eng)

25% less than the original speed. If the distance 16)


between point M and N is 88 km, then find the Quantity I: Total surface area of a sphere is 2464
approximate total time taken by the boat? m2. Find the volume of the sphere?
a) 5 hrs Quantity II: The sum of the height and radius of a
b) 4 hrs cone is 28 m and the ratio of radius and height of
c) 3 hrs the cone is 11:3. Find the volume of the cone?
d) 2 hrs Quantity III: The ratio of the side of the cube and
e) None of these length of the cuboid is 2:5. Volume of the cube is
64 m. Ratio of length and breadth and the ratio of
14) Speed of boat S is 25% more than the speed length and height of the cuboid is 5:3 and 5:7.
of boat of Q in still water. Find the distance cover Find the volume of the cuboid?
by boat S on Wednesday in (a+2) hours a) Quantity I <Quantity II >Quantity III
downstream? b) Quantity I> Quantity II =Quantity III
a) 168 km c) Quantity I< Quantity II< Quantity III
b) 172 km d) Quantity I> Quantity II> Quantity III
c) 180 km e) Quantity I> Quantity II <Quantity III
d) 182 km
e) None of these 17)
Quantity I: The ratio of age of A and B is 2:3.
15) Boat Q and Boat R are travelling in opposite After 5 years age of B is 32 years. Age of C is 5
direction towards each other from a certain years more than A. Find the average age of A, B,
distance on Tuesday. If boat Q goes downstream and C?
and boat R goes upstream and they meet after 7 Quantity II: The sum of age of A and B is 38
hours then find the distance between boat Q and years and age of B is 5 years older than C whose
R initially? age is 13 years. Find the age of A?
a) 280 km Quantity III: The age of the son is 1/3rd of the
b) 220 km age of the father and the age of the mother is
c) 256 km thrice of her daughter. If the sum of the age of
d) 250 km the father and mother is 84 years and both of
e) None of these their ages are equal. Find the sum of the age of
the daughter and son?
Direction (16-20) – Each question has three a) Quantity I >Quantity II >Quantity III
quantities. Calculate the value of each quantity b) Quantity I>Quantity II< Quantity III
and compare it. c) Quantity I =Quantity II< Quantity III

Click Here For Bundle PDF Course | support@guidely.in Page 5 of 11


Bank Po Mains PDF Course 2024
Quantitative Aptitude Day -13 (Eng)

d) Quantity I< Quantity II <Quantity III Quantity II: An item isbought at Rs.1250 then
e) Quantity I >Quantity II= Quantity III marked the price 50% above the buying price
and then gave a 10% discount on the marked
18) price, find the profit percentage?
Quantity I: A person has Rs.12400. He invests Quantity III: The selling price of the item is
40% in the scheme which provides a 10% rate of Rs.1580. The shopkeeper is given a 20%
interest and the rest invests in another scheme discount on the marked price and the marked up
which provides a 5% rate of interest. Find the percentage is 40%. Find the profit percentage?
total interest earned by the person after 5 years if a) Quantity I <Quantity II <Quantity III
both are invested in SI? b) Quantity I= Quantity II >Quantity III
Quantity II: Rs.4852.85 c) Quantity I <Quantity II> Quantity III
Quantity III: A person invests Rs.1200 at 20% d) Quantity I >Quantity II> Quantity III
rate of compound interest for 2 years and e) Quantity I >Quantity II <Quantity III
Rs.6000 invests at 25% rate of compound
interest for 2 years. Find the total interest earned 20) f(x)=x3-3x2+3x-4
by the person? Quantity I: Find the value of f(x) when x=3?
a) Quantity I =Quantity II =Quantity III Quantity II: Find the value of f(x) when x=-2.5?
b) Quantity I >Quantity II >Quantity III Quantity III: Find the value of f(x) when x=1.5?
c) Quantity I <Quantity II< Quantity III a) Quantity I <Quantity II >Quantity III
d) Quantity I >Quantity II =Quantity III b) Quantity I< Quantity II< Quantity III
e) Quantity I < Quantity II >Quantity III c) Quantity I <Quantity II =Quantity III
d) Quantity I >Quantity II< Quantity III
19) e) Quantity I =Quantity II <Quantity III
Quantity I: The ratio of the cost price and selling
price of the item is 7:11. Find the profit
percentage of the item?

Click Here For Bundle PDF Course | support@guidely.in Page 6 of 11


Bank Po Mains PDF Course 2024
Quantitative Aptitude Day -13 (Eng)

Click Here to Get the Detailed Video Solution for the above given Questions
Or Scan the QR Code to Get the Detailed Video Solutions

Answer Key with Explanation


Directions (01 - 05):
Number of employee in Mumbai office of
company A is [220+60]/2=140
Number of employee in Delhi office of company
A is [220-60]/2=80
Number of employees in HR post of Mumbai
Office =140*3/7=60
Number of employees in IT post of Mumbai
office =140*2/7=40
Number of employees in Marketing post of
Mumbai office =140*2/7=40
Number of employees in HR post of Delhi Office
=80*3/8=30
Number of employees in IT post of Delhi office
=80*2/8=20
Number of employees in Marketing post of Delhi
office =80*3/8=30
Similarly, we can calculate other values also.
1) Answer: C
Total male in company A is
=60*3/5+40*3/8+40*3/4+30*2/5+20*1/4+30*3/5=
116
Total female in company A is

Click Here For Bundle PDF Course | support@guidely.in Page 7 of 11


Bank Po Mains PDF Course 2024
Quantitative Aptitude Day -13 (Eng)

=60*2/5+40*5/8+40*1/4+30*3/5+20*3/4+30*2/5= Quantity I
104 Required percentage = [(30+40+20) /
So, difference is = 116-104 = 12 (60+20+40)]*100 = 75%
Quantity II
2) Answer: D Required percentage =
I. Difference of total employees in IT post in [(20+20+60)/(80+60+60)]*100 = 50%
Mumbai of all companies together and total
employees in IT post in Delhi of all companies 5) Answer: A
together is Required difference = [140+160+100] –
= [40+60+20+40+30]-[20+20+60+40+36]=14. [120+160+120] =0
II. Total employees in Marketing post of all
companies together Directions (06 - 10):
=40+20+20+40+20+30+40+20+40+36=306. In city P,
III. Average number of employees in Delhi office The number of total registered voters is 30000.
of all companies together is = The total number of cast votes is
[80+120+120+160+120]/5=120. 30000*80/100=24000
The total number of people who did not cast vote
3) Answer: E is 30000-24000=6000
a) Ratio of number of employees in HR post of The total number of valid votes is
company C in Mumbai and Delhi office is 3:2. 24000*90/100=21600
b) Total number of employees in HR post in the The total number of invalid votes is 24000-
Mumbai office of all companies is 320. 21600=2400
c) Difference between the number of employees The number of votes gets by party A is
in marketing post of company E in Mumbai and 21600*60/100=12960
the Delhi office is 16. The number of votes gets by party B is 21600-
d) Difference of average number of employees in 12960=8640
IT post all companies together in Mumbai office Similarly, we can calculate other cities also.
and average number of employees in Marketing
post all companies together in Mumbai office is
10.
So, all are false.

4) Answer: A

Click Here For Bundle PDF Course | support@guidely.in Page 8 of 11


Bank Po Mains PDF Course 2024
Quantitative Aptitude Day -13 (Eng)

The speed of the stream of river R on Monday,


Tuesday, and Wednesday is a, b, and c km/hr
respectively. The speed of boats P, Q, and R in
still water is x km/hr, y km/hr, and z km/hr
So, we can say,
66/(a+x) + 56/(x-a) =7 --------I
88/(a+x) + 28/(x-a) =6 --------II
By solving I and II we get, x=18 km/hr and a=4
km/hr.
6) Answer: C
44/(y+b) + 60/(y-b) =8 --------III
Required sum
88/(y+b) + 70/(y-b) =11 --------IV
=[12960-8640]+[13965-5985]+[9900-
By solving III and IV we get, y=16 km/hr and b=6
8100]+[13770-9180]+[15390-10260]= 23820
km/hr.
64/(z+c) +32/(z-c) =4 --------V
7) Answer: D
96/(z+c) + 64/(z-c) =7 --------VI
Required difference
By solving V and VI we get, z=24 km/hr and c=8
= [24000+21000+22500+25500+27000]/5 –
km/hr.
[21600+19950+18000+22950+25650]/5
11) Answer: C
=24000-21630=2370
Required difference = 120/(18+6)-64/(24-
6)=1(4/9)hrs
8) Answer: B
B’s party vote is [18000*4/3]*45/100=10800
12) Answer: D
Required ratio =[16+4]*6 : [24-4]*8=120:160=3:4
9) Answer: A
Required ratio=[6000+9000+7500+4500+3000]:
13) Answer: B
[2400+1050+4500+2550+1350]
The speed stream from M to O is 4*75/100=3
=30000:11850=200:79
So, required time is =
[44/(18+3)]+[44/(18+4)]=4.09=4 hours.
10) Answer: A
Required difference =[13965/30000]*100–
14) Answer: A
[8640/30000]*100=17.75%
The speed of boat S is =16*125/100=20 km/hr.
Required distance = [20+8]*[4+2]=28*6=168 km
Directions (11 - 15):

Click Here For Bundle PDF Course | support@guidely.in Page 9 of 11


Bank Po Mains PDF Course 2024
Quantitative Aptitude Day -13 (Eng)

15) Answer: A 18) Answer: E


Total distance of boat Q and R initially is = Quantity I:
[16+6]*7+[24-6]*7=280 km Total interest earned
= [12400*40/100]* 5*10
16) Answer: D /100+[12400*60/100]*5*5/100
Quantity I: =Rs.4340
So, 4*22/7*r*r=2464, r=14m Quantity II: Rs.4852.5
So, the volume is 22/7*14*14*14*4/3=11498.66 Quantity III:
m3 Total interest earned
Quantity II: 1200*[1+20/100]2+6000*[1+25/100]2-
Radius is 28*11/14=22m, height is 28-22=6m 7200=Rs.3903
So, volume is =1/3*22/7*22*22*6=3042.28m3 Quantity I < Quantity II >Quantity III
Quantity III:
The side of the cube is 4m 19) Answer: D
So, the length of the cuboid is 4*5/2=10m Quantity I:
The breadth of the cuboid is 10*3/5=6m Profit percentage = [4/7]*100=57.14%
The height of the cuboid is 10*7/5=14m Quantity II:
So, volume is 10*6*14=840m3 Selling price =
Quantity I> Quantity II> Quantity III 1250*150*90/(100*100)=Rs.1687.5
Profit percentage = [(1687.5-
17) Answer: B 1250)/1250]*100=35%
Quantity I: Quantity III:
Age of B is 32-5=27 years. Cost price of item is
Age of A is 27*2/3=18 years 1580*[100/80]*[100/140]=1410.71
Age of C is 18+5=23 years So, profit percentage [1580-
So, the average age is [27+18+23]/3=22.66 1410]*100/1410=12.05%
years Quantity I >Quantity II> Quantity III
Quantity II:
Age of Ais 38-[13+5]=20 years 20) Answer: D
Quantity III: Quantity I:
The Sum of the age of Son and daughter is So, f(x)=3*3*3-3*(3)2+3*3-4 when x=3,
84/3=28 years Or, f(x)=5
Quantity I>Quantity II< Quantity III Quantity II:

Click Here For Bundle PDF Course | support@guidely.in Page 10 of 11


Bank Po Mains PDF Course 2024
Quantitative Aptitude Day -13 (Eng)

So, f(x)=(-2.5)3-3(-2.5)2+3*(-2.5)-4=-45.875 when So, f(x)=(1.5)3-3*(1.5)2+3*(1.5)-4=-2.875


x=-2.5 So, Quantity I >Quantity II < Quantity III
Quantity III:

Click Here For Bundle PDF Course | support@guidely.in Page 11 of 11


Bank Po Mains PDF Course 2024
English Day - 13

English Language
Directions (1-5) : Rearrange the following five e) E
sentences (A), (B), (C), (D) and (E) in the proper
sequence to form a coherent paragraph and then 2) Choose from the below options the
answer the questions given below. SYNONYM of the word PROMOTE as given in
(A) The alliance operates on the principle of the sentence B.
collective defence/A, stating that an attack a) Stimulate
against one member is/B considered an attack b) demote
against to all, leading to/C joint action to deter c) discourage
and respond to potential threats/D. d) dishonour
(B) Formed in 1949, NATO is a political and e) desist
military alliance that aims to promote collective
defence, preserve peace, and safeguard the 3) Fill in the blank given in sentence C and
freedom of its member nations. complete it.
(C) NATO, the North Atlantic Treaty a) commemoration
Organization, stands as a cornerstone of b) contemplation
international security and __________ . c) connotation
(D) a pivotal role in promoting stability and d) cooperation
peace in /P Europe and beyond, undertaking e) confrontation
missions /Q throughout its history, NATO has
played /R management, and disaster relief 4) Rearrange the parts of the sentence in D to
operations /S such as peacekeeping, crisis /T form a meaningful sentence.
(E) Comprising 30 member countries from North a) TQSPR
America and Europe, NATO fosters close b) QPTRS
diplomatic ties and military cooperation among its c) STRQP
members to address a wide range of security d) PQRST
challenges. e) RPQTS
1) Find which part of sentence ‘A’ has an error in
it and mark that as your answer. Mark option (e) 5) Which of the following is the correct sequence
if no error is found. of rearrangement that forms a meaningful
a) A coherent paragraph ?
b) B
c) C a) CDEAB
d) D b) CBEAD

Click Here For Bundle PDF Course | support@guidely.in Page 1 of 10


Bank Po Mains PDF Course 2024
English Day - 13

c) BCEAD of creating fireworks igniting a sense of


d) AECDB
spread to different childlike wonder in our
e) EDBCA
parts of the world, hearts.
each culture
Directions (6-10): In the following questions two
a) A-F
columns are given containing three
b) A-E
Sentences/phrases each. In the first column,
c) B-E
sentences/phrases are A, B and C and in the
d) A-E and C-D
second column the sentences/phrases are D, E
e) None of these
and F. A sentence/phrase from the first column
may or may not connect with another
7)
sentence/phrase from the second column to
COLUMN 1 COLUMN 2
make a grammatically and contextually correct
sentence. Each question has five options, four of A. While conjunctivitis D. dirty hands, using
which display the sequence(s) in which the is generally not a contaminated eye
sentences/phrases can be joined to form a severe condition, it makeup, or swimming in
grammatically and contextually correct sentence. can be highly water with harmful
If none of the options given forms a correct contagious, bacteria.
sentence after combination, mark option (e), i.e. necessitating
“None of these” as your answer.
B. These infections E. self-medicate eye
6)
can lead to more infections, as improper
COLUMN 1 COLUMN 2
severe symptoms like treatment can
A. From brilliant D. create entertainment severe pain, blurred exacerbate the condition
cascades of sparkles and began incorporating vision, sensitivity or delay proper care.
to thunderous them into religious
C. Eye infections can F. to light, and even
explosions that ceremonies and festive
be caused by various vision loss if left
reverberate in the events.
factors, including untreated.
B. The history of E. they were initially poor hygiene,
fireworks dates back invented by alchemists touching the eyes
to ancient China, searching for the elixir of with
where immortality.

C. Over time, the art F. against the darkness, a) A-D

Click Here For Bundle PDF Course | support@guidely.in Page 2 of 10


Bank Po Mains PDF Course 2024
English Day - 13

b) B-F A. In an age D. simple pleasure of


c) A-F and B-D
dominated by traditional games, the
d) B-F and C-D
screens and great outdoors offers an
e) None of these
technology, the expansive canvas for
allure of outdoor boundless fun and
8) games remains as adventure.
COLUMN 1 COLUMN 2 strong as ever,
A. By being D. technology's benefits
B. From soccer and E. and the thrill of
positive role and ensuring healthy real-
basketball to cricket scoring a point can
models and world interactions is a
and baseball, team create unforgettable
displaying qualities challenge parents face in
sports have a unique memories with friends
such as kindness, the digital age.
way of fostering and family.
B. It is also crucial E. resilience, and honesty, camaraderie,
for parents to parents impart invaluable teaching
remember that no life lessons that extend far
C. Whether it's the F. bring people together,
one is infallible, beyond the classroom.
excitement of team encouraging physical
and making
sports, the thrill of activity and imaginative
mistakes
running freely on play in equal measure.
C. The constant F. ample time to children open fields, or the
juggling act can sometimes leave
a) A-F
between work, parents feeling
b) C-D
household overwhelmed and c) B-D and C-F
responsibilities, exhausted.
d) B-E and C-D
and giving e) None of these
a) A-E and C-F
b) A-D and C-E 10)
c) B-F and C-D COLUMN 1 COLUMN 2
d) B-E and C-F A. Adulteration, a D. affects human health
e) None of these deceptive practice but also tarnishes the
that involves adding reputation of businesses
9) inferior or harmful and industries.
COLUMN 1 COLUMN 2

Click Here For Bundle PDF Course | support@guidely.in Page 3 of 10


Bank Po Mains PDF Course 2024
English Day - 13

substances to food, ensuring a tidy _______ that would attract


customers and make their shopping experience
beverages, or other
more enjoyable.
products, poses a
a) count, place
B. The history of E. adulteration has b) arrange, display
adulteration dates expanded to a wide range c) disturb, present
back centuries, of products, including d) keeping, look
where unscrupulous food items, e) organized, exhibit
individuals sought to pharmaceuticals,
profit at the expense cosmetics, and even 12) Typewriting classes were in high demand
fuels. during the pre-computer era, as they offered
valuable skills for ______office professionals and
C. Contaminated F. serious threat to public
typists, ensuring fast and ______ document
food and beverages health and undermines
creation.
can lead to a host of the integrity of industries
a) demotivated, inexact
health issues, and businesses.
b) beginning, furious
ranging from mild
c) aspiring, accurate
discomfort and
d) disinterested, improper
a) A-F
e) energetic, false
b) B-E
c) B-D and C-F
13) Free meals in the government schools aimed
d) B-E and C-F
to _______ the issue of malnutrition among
e) None of these
underprivileged children, ensuring they received
_______ nutrition for their overall well-being.
Directions (11-15) : Each of the following
a) address, adequate
questions has two blanks, each blank indicating
b) fixed, good
that a word/phrase has been omitted. Choose
c) solve, insufficient
the set of words/phrases that best fits the given
d) complicate, satisfaction
blanks according to the context of the sentence.
e) preclude, fair
Few alphabets are given to help you locate the
blanks and fill them.
14) She worked really hard to manage both her
_______job and her responsibilities as a single
11) The storekeeper was instructed to _______
parent, proving her determination and dedication
the newly arrived merchandise on the shelves,
to ______the best for her child.

Click Here For Bundle PDF Course | support@guidely.in Page 4 of 10


Bank Po Mains PDF Course 2024
English Day - 13

a) demanding, provide 17) The medicines at the store were neatly


b) easy, take arranged (A) on the shelves, with a wide
c) simple, deliver assortment (B) of prescription drugs, over-the-
d) new, lend counter remedies, and health supplements
e) teaching, refuse catering (C) to various healthcare needs (D).
a) A-B, B-assorted
15) The launch of the restaurant was a grand b) A-D, D-equipments
______, with celebrities, food critics, and locals c) B-C, A-arranging
alike attending the event to _____the delectable d) C-D, C-caters
offerings and celebrate the culinary journey. e) No changes required
a) event, tasted
b) launch, enjoy 18) Exams are arriving (A) up, and there's a mix
c) opening, dislike of nervousness and determination among the
d) affair, savour students as they diligently (B) prepare and
e) party, oppose achieve (C) to perform at their best and revise
(D) their academic goals.
Directions (16-20) : The following questions a) A-C, D-revised
given below have four words that have been b) B-C, C-achieving
highlighted as they might’ve been placed at the c) B-D, A-arrived
wrong positions. One of these words might also d) C-D, A-coming
be incorrect and need a replacement. Read the e) No changes required
same carefully and mark the appropriate option
as the answer. 19) Ballpoint pens are better choice (A) for
16) My friend is celebration (A) married this everyday (B) writing tasks due to their smooth ink
month, and the excitement in the air is palpable flow, reliability, and convenience, making them a
(B) as we eagerly anticipate getting (C) their love popular suit (C) among students and
and witnessing (D) the beginning of a beautiful professionals alike (D).
new chapter in their lives. a) A-C, C-suited
a) A-B, D-encountering b) A-D, B-every
b) B-D, C-readily c) B-C, C-suited
c) A-C, A- celebrating d) C-D,B-every
d) C-D, A-celebrating e) No changes required
e) No changes required 20) The OMR sheet given (A) in the examination

Click Here For Bundle PDF Course | support@guidely.in Page 5 of 10


Bank Po Mains PDF Course 2024
English Day - 13

was carefully designed with printed bubbles b) B-C, C-corresponding


allowing (B) to each question, correspond (C) c) B-D, B- allow
candidates to mark their answers precisely and d) C-D, B-allowed
facilitating (D) efficient evaluation. e) No changes required
a) A-D, A-gave
Click Here to Get the Detailed Video Solution for the above given Questions
Or Scan the QR Code to Get the Detailed Video Solutions

Answer Key with Explanation


1) Answer: C The word stimulate is similar in meaning to the
Part C of the sentence has an error in it. word ‘promote’
Replace ‘against to all’ with ‘against all’ to make The other words wither have different meaning
the given sentence grammatically correct and or they are the opposite of ‘promote’
meaningful.
The corrected sentence : The alliance operates Demote - to move somebody to a lower position
on the principle of collective defence, stating that or level
an attack against one member is considered an Dishonour - lack or loss of honour
attack against all, leading to joint action to deter Desist - stop doing something; cease or abstain
and respond to potential threats.
3) Answer: D
2) Answer: A Cooperation is the right word that fits the given
The meaning of promote - to encourage blank correctly.
something Commemoration - something that is done to
Stimulate - to make something active or more remember officially and give respect to a great
active person or event
Contemplation - deeply or seriously thoughtful.

Click Here For Bundle PDF Course | support@guidely.in Page 6 of 10


Bank Po Mains PDF Course 2024
English Day - 13

Connotation - an idea expressed by a word in come after the introduction and before the actual
addition to its main meaning work done by NATO. As, sentence ‘E’ spoke
Confrontation - a fight or an argument about the security challenges ‘A’ is the correct
The sentence: NATO, the North Atlantic Treaty sentence that follows ‘E’. The final sentence or
Organization, stands as a cornerstone of the last sentence is ‘D’ which seems to be a
international security and cooperation. concluding sentence representing the actions of
NATO throughout history.
4) Answer: E
The correct sequence of rearrangement from the 6) Answer: C
given option is option (e) RPQTS. B-E forms a complete sentence which is both
R is the starter sentence and the remaining are a meaningful and correct.
continuation of one another, P follows R joining The other options have a combination of
the sentence. Next comes Q which is a statements/fragments which do not combine to
continuation of P. Among S and T, S is the form a complete sentence.
concluding sentence because it has the last The sentence formed : The history of fireworks
fragment “and…”. So, TS forms the last two dates back to ancient China, where they were
sentences. initially invented by alchemists searching for the
The sentence : Throughout its history, NATO elixir of immortality.
has played a pivotal role in promoting stability
and peace in Europe and beyond, undertaking 7) Answer: D
missions such as peacekeeping, crisis Both B-F and C-D are the correct combinations
management, and disaster relief operations. which combine to form a complete sentence
which is meaningful and correct.
5) Answer: B The other options are incorrect as those
CBEAD is the right sequence of rearrangement combinations of fragments/statements do not
that forms a meaningful paragraph. form a proper sentence.
C is the first sentence, as it has the full form of The sentences formed :
NATO and also the basic/simple definition or B-F : These infections can lead to more severe
purpose of NATO. ‘B’ follows ‘C’ as it talks about symptoms like severe pain, blurred vision,
the formation of NATO and gives more details on sensitivity to light, and even vision loss if left
its purpose. Sentence ‘E’ follows CB and comes untreated.
before both A and D because it talks about the C-D : Eye infections can be caused by various
members and the diplomatic ties which should factors, including poor hygiene, touching the

Click Here For Bundle PDF Course | support@guidely.in Page 7 of 10


Bank Po Mains PDF Course 2024
English Day - 13

eyes with dirty hands, using contaminated eye 10) Answer: A


makeup, or swimming in water with harmful A-F is the correct combination that forms a
bacteria. complete sentence.
The other given options are wrong as they don’t
8) Answer: A contain statements that form a meaningful
A-E forms a complete meaningful sentence sentence.
similarly the combination C-F forms a complete The sentence formed : Adulteration, a deceptive
sentence with meaning. Fragments B and D do practice that involves adding inferior or harmful
not combine with the other statements to substances to food, beverages, or other
generate a sentence and hence are discarded. products, poses a serious threat to public health
The sentences formed : and undermines the integrity of industries and
A-E : By being positive role models and businesses.
displaying qualities such as kindness, resilience, 11) Answer: B
and honesty, parents impart invaluable life Arrange and display are the right words that fit
lessons that extend far beyond the classroom. both the given blanks respectively to complete
C-F : The constant juggling act between work, the meaning of the sentence.
household responsibilities, and giving ample First blank - this blank needs a verb in the first
time to children can sometimes leave parents form - count, arrange and disturb are the verbs
feeling overwhelmed and exhausted. in the first form - so eliminate options d and e.
Disturb completely inappropriate to the given
9) Answer: B context - eliminate option c
The fragment C and D combine to form a Count on the shelves is wrong - so eliminate
meaningful and complete sentence. option a
Even though the other statements also revolve The left out and the correct option is ‘b’.
around the same topic they do not combine with The sentence : The storekeeper was instructed
each other to form a sentence that is meaningful to arrange the newly arrived merchandise on the
and correct. shelves, ensuring a tidy display that would
The sentence formed : Whether it's the attract customers and make their shopping
excitement of team sports, the thrill of running experience more enjoyable.
freely on open fields, or the simple pleasure of
traditional games, the great outdoors offers an 12) Answer: C
expansive canvas for boundless fun and Aspiring and accurate are the two words that fit
adventure. the given blanks respectively.

Click Here For Bundle PDF Course | support@guidely.in Page 8 of 10


Bank Po Mains PDF Course 2024
English Day - 13

First blank - aspiring and energetic are the only The sentence : Free meals in the government
words that fit the context of the sentence, the schools aimed to address the issue of
other words create a negative impact on the malnutrition among underprivileged children,
sentence, so discard options a, b and d. ensuring they received adequate nutrition for
Second blank - again ‘false’ creates a negative their overall well-being.
impact but the sentence is in a positive tone
where professionals learnt typewriting to develop 14) Answer: A
their skills. So, to match the context ‘accurate’ is The correct pair is “demanding and provide”.
the correct word. First blank - All the given words fit the first blank
The sentence : Typewriting classes were in high but ‘easy and simple’ are opposite in context of
demand during the pre-computer era, as they the given sentence and hence options b and c
offered valuable skills for aspiring office are discarded.
professionals and typists, ensuring fast and Among a,d and e - the option which has the
accurate document creation. correct word to fill the second blank is option(a) -
‘provider the best’
13) Answer: A ‘Lend the best’ and ‘refuse the best’ are
Address and adequate are the correct words and incorrect and inappropriate and hence not the
hence option (a) is the right answer for this right pick
question. The sentence : She worked really hard to
The first blank needs a verb in the first form - manage both her demanding job and her
discard option b responsibilities as a single parent, proving her
The government scheme is in place to help the determination and dedication to provide the best
people and children so ‘complicate and preclude’ for her child.
are incorrect words that can be used - eliminate
options d and e 15) Answer: D
Preclude - to prevent something from happening Affair and savour are the right words.
or somebody from doing something; to make Savour - to taste or smell with pleasure
something impossible First blank - launch and opening do not fit the
Between options a and c - look at the second first blank because they are redundant and
words adequate and insufficient - adequate is conveying the same meaning repeatedly - so
the right word that fits the second blank in discard options b and c
accordance with the context of the sentence. Out of options a,d and e - ‘to tasted’ is wrong,
Option a is the correct answer. similarly ‘to oppose’ is inappropriate to the

Click Here For Bundle PDF Course | support@guidely.in Page 9 of 10


Bank Po Mains PDF Course 2024
English Day - 13

context of the sentence, hence ‘savour’ is the The sentence: Exams are coming up, and
correct word. there's a mix of nervousness and determination
The sentence : The launch of the restaurant was among the students as they diligently prepare
a grand affair, with celebrities, food critics, and and revise to perform at their best and achieve
locals alike attending the event to savour the their academic goals.
delectable offerings and celebrate the culinary
journey. 19) Answer: A
‘Are better choice’ is wrong instead it must be
16) Answer: C ‘are a better choice' so definitely word (A) is
First of all the word ‘celebration’ is incorrect, misplaced, swap it with word C to make the
instead it must be celebrating. And the positions sentence correct.
of the words A and C must be interchanged to The word ‘suit’ is still inappropriate, change it to
make the given sentence all correct and ‘suited’ to make the words and the respective
meaningful. sentence correct and meaningful.
The sentence: My friend is getting married this The sentence: Ballpoint pens are better suited
month, and the excitement in the air is palpable for everyday writing tasks due to their smooth ink
as we eagerly anticipate celebrating their love flow, reliability, and convenience, making them a
and witnessing the beginning of a beautiful new popular choice among students and
chapter in their lives. professionals alike.

17) Answer: E 20) Answer: B


The given four words are correctly placed and Swap the words B and C to make the sentence
are appropriate too, hence there is no need to correct and meaningful. The word ‘correspond’ is
swap or replace them. incorrect, instead it must be ‘corresponding’ to
make the sentence more accurate and
18) Answer: D meaningful.
‘Arriving up’ is completely wrong and hence The sentence: The OMR sheet given in the
needs replacement of the word itself. examination was carefully designed with printed
Out of the given options ‘coming’ is the right bubbles corresponding to each question,
word to put in place (A) and replace words C and allowing candidates to mark their answers
D to make the sentence correct. precisely and facilitating efficient evaluation.

Click Here For Bundle PDF Course | support@guidely.in Page 10 of 10


Bank Po Mains PDF Course 2024
Reasoning Day -14 (Eng)

Reasoning Aptitude
Directions (1-5): In each group of questions II. No Pine is Teak
below there are three conclusions followed by III. All Maples are Oak is not a Possibility
five statements. You have to choose the correct Statements:
set of statements that logically satisfy the given I. Only a few Maples are Pine, All Pines are Oak,
conclusions. Only Oak is Teak, No Cedar is Maple
1. Conclusions: II. Only a few Cedars are Pine, No Oak is Pine,
I. Some Emeralds are Sapphire is a possibility Some Oaks are not Teak, All Maples are Teak
II. Some Pearl is not Sapphire III. Some Teaks are Oak, Only a few Oaks are
III. All Diamonds can be Ruby Pine, All Pines are Cedar, No Maple is Teak
Statements: IV. No Maple is Teak, Only Maple is Pine, Some
I. Only a few Pearls are Diamonds. All Emeralds Cedars are Maple, Some Teaks are not Oak
are Diamonds. Some Sapphires are Ruby. No V. All Cedars are Pine, Some Teaks are Oak, All
Diamond is Sapphire Pines are Maples, Only a few Maples are Teak
II. Some Diamonds are Ruby. No Ruby is a) Both I and V follow
Sapphire. Only a few pearls are Sapphire. All b) Only II follows
Emeralds are Diamonds. c) Both I and IV follow
III. Only a few Emeralds are Pearl. All Pearls are d) Only I, IV and V follow
Ruby. No Ruby is Sapphire. Only Sapphire is e) All follows
Diamond.
IV. Some Diamonds are Emerald. All Emeralds 3. Conclusions:
are Pearl. Only a few Pearls are Ruby. No Ruby I. All Quartz can be Marble
is Sapphire. II. All Marbles are Granite is a possibility
V. Some Rubies are Diamond. Only Pearl is III. Some Tiles are Concrete is a possibility
Emerald. No Diamond is Pearl. Only a few Statements:
Sapphires are Ruby. I. Only a few Concretes are Granites, Some
a) All follow Granites are Tiles, No Tile is Marble, Only
b) Both I and III follow Concrete is Quartz
c) Both II and V follow II. Only a few Granites are Marbles, Some
d) Both II and IV follow Marbles are not Tiles, All Tiles are Quartz, Some
e) None follows Concretes are Granites
III. All Marbles are Quartz, All Tiles are Granites,
2. Conclusions: No Quartz is Granite, Only a few Granites are
I. Some Cedars are not Oak is a possibility Concrete

Click Here For Bundle PDF Course | support@guidely.in Page 1 of 12


Bank Po Mains PDF Course 2024
Reasoning Day -14 (Eng)

IV. Only a few Concretes are Marble, Some c) Both II and IV follow
Marbles are Tiles, Some Tiles are not Granite, All d) Both IV and V follow
Quartz are Granite e) None Follows
V. Some Marbles are Tile, Only a few Tiles are
Quartz, All Quartz are Granite, No Granite is 5. Conclusions:
Concrete I. All Monsoons is winter
a) None follows II. All Summer being spring is a possibility
b) Both II and III follow III. Some Winter is not autumn
c) Only II, IV and V follow Statements:
d) All follows I. Only a few Spring is Summer, Some Summer
e) Both II and IV follow is Winter, No Autumn is Winter, All Autumn is
Monsoon
4. Conclusions: II. Some Winter is Autumn, All Autumn is
I. No Tree is Creeper Summer, No Summer is Monsoon, Only a few
II. Some Shrubs are Tree is a Possibility Spring is Monsoon
III. Some Herbs are not Climber III. No Monsoon is Spring, Some Spring is
Statements: Winter, Only a few Winter is Autumn, Only
I. Only a few Shrubs are Climber, All Climbers Autumn is Summer
are Tree, No tree is Herb, Some Creepers are IV. All Monsoon is Winter, Some Winter is
Herb Autumn, Only a few Autumn is Spring, No Spring
II. All Shrubs are Climber, Some Climbers are is Summer
Creeper, Only a few Creepers are Herb, Some V. Some Autumn is Summer, Only a few
Herbs are not Tree Summer is Spring, No Spring is Monsoon, No
III. All Trees are Herb, Only a few Herbs are Winter is Monsoon
Shrub, Only Shrub is Climber, Some Creepers a) Both I and V follow
are Shrub b) Both II and IV follow
IV.Some Trees are Herb, Only a few Herbs are c) Both I and III follow
Shrub, Only Shrub is Creeper, No Climber is d) All follow
Shrub e) None follows
V. Some Climbers are Creepers, Only a few Directions (6-10): Study the following information
Creepers are Shrub, No Shrub is Herb, All Herbs carefully and answer the given questions.
are Tree Ten boxes are placed one above the other in a
a) All Follow stack in an Amazon warehouse where the
b) Only IV follows bottommost box is numbered 1 and the box

Click Here For Bundle PDF Course | support@guidely.in Page 2 of 12


Bank Po Mains PDF Course 2024
Reasoning Day -14 (Eng)

above 1 is numbered 2 and so on till the topmost 6. How many boxes are having an even number
box is numbered 10. Each box contains different of toys and which of the following toy is in the
types of toys viz., P, Q, R, S, T, U, V, W, X and box which has the highest odd number of toys?
Y. Each box has different number of toys. a) 7, R
Note: The number of toys in box 1 is equal to the b) 7, Q
multiple of the box number of the box which is c) 6, P
placed at the topmost position, The number of d) 6, R
toys in box 2 is equal to the multiple of the box e) 7, S
number of the box which is placed at the second
topmost position and so on till the number of toys 7. The number of boxes between the box which
in a box 10 is equal to the multiple of the box has 52 toys and Q is ______ the number of boxes
number of the box which placed at the between W and the box which has 40 toys.
bottommost positions. a) Two less than
Only four boxes are placed between the box b) Three more than
which has 66 toys and V. The box which has 66 c) Three less than
toys is not placed at the topmost position. The d) Two more than
box which has 35 toys which is placed above V e) One more than
and three boxes below W. As many boxes
placed below the box which has 49 toys as 8. The_____ toy is in the box 7 and the _____ toy
above X, which is not placed at the bottommost is in the box 5 respectively.
position. The box which has 50 toys contains Y, a)X, U
which is placed below W. The number of boxes b) X, T
placed between Y and the box which has 52 toys c) P, T
is two more than the number of boxes placed d) P, Q
between the boxes which has 37 toys and T. R is e) Q, W
placed immediately below the box which has 40
toys. More than four boxes are placed between R 9. If the Toys are arranged in alphabetical order
and P, which is not placed at the topmost from top to bottom, then how many boxes remain
position. The box which contains U has 42 toys. unchanged in their position?
The box which has a square number of toys a) One
neither contains neither S nor T. The remaining b) None
boxes are having 28 and 72 toys. c) Two
d) Three
e) More than three

Click Here For Bundle PDF Course | support@guidely.in Page 3 of 12


Bank Po Mains PDF Course 2024
Reasoning Day -14 (Eng)

opposite to the one who sits adjacent to H. More


10. Which of the following statements is/are true than one person sits between H and V, who
according to the given arrangement? faces the same direction as M faces. The
I. Box 7 has 52 toys and it contains X number of persons sitting between V and R is
II. Box 6 has odd number of toys and it is two two more than the number of persons sitting
boxes below the box which has number of toys between S and N. R does not sit adjacent to N.
that is multiple of 11 The one who sits opposite to R sits three places
III. The number of boxes above V is two less away from K. More than four persons sit between
than the number of boxes below P L and P. P sits opposite to E, who sits adjacent
a) Only I and III to K. Only two persons sit between F and T, who
b) All are true faces H. The number of persons sitting to the left
c) Only II and III of E is one less than the number of persons
d) None is true sitting to the right of J, who faces Q. D and W sit
e) Only I and II in the same row but face the opposite direction.
D sits fifth from one of the extreme ends.
Directions (11-15): Study the following 11. Which of the following person sits fourth from
information carefully and answer the given the west end of each row according to the final
questions. arrangement?
Twenty persons from D to W are sitting in three I. The one who faces H
parallel rows. There are three rows i.e., row1, II. The one who sits immediate left of N
row2, and row3 such that row2 is in the north of III. The one who sits diagonally opposite to R
row3 and row1 is in the north of row 2. Five IV. The one who sits fifth to the left of H
persons are sitting in row 1 and facing south a) Only I and II
whereas five persons are sitting in row 3 and b) Only II and III
facing north. Ten persons are sitting in row 2 in c) Only I, II and IV
such a way that five persons from the east in row d) Only IV
2 face north towards the persons in row 1 and e) Only I and IV
five persons from the west in row 2 face south
towards the persons in row 3. 12. How many persons are facing north whose
M sits third from one of the extreme ends. S, who name starts with a vowel?
faces south, sits second to the right of the one a) Two
who faces M. I, who sits diagonally opposite to S, b) Three
sits fourth to the right of O. As many persons sit c) Four
to the left of O as to the left of G, who sits d) None

Click Here For Bundle PDF Course | support@guidely.in Page 4 of 12


Bank Po Mains PDF Course 2024
Reasoning Day -14 (Eng)

e) One “X # Y (45)” means “X is 33m north of Y”


“X $ Y (14)” means “X is 22m south of Y”
13. If U is related to D and T is related to V in a “X & Y (37)” means “X is 24m west of Y”
certain way, then who among the following “X @ Y (21)” means “X is 26m east Y”
person is related to G? Statement: J $ L(12); K @ L(15); I # M(56); M $
a) V K(17); O # Q(32); Q @ K(16); O & P(35); N & J
b) The person who sits extreme right end in Row (30)
2 16. What is the direction of I with respect to O
c) The person who sits second left of the one and what is the shortest distance between K and
who faces T O?
d) D a) West, 29m
e) Q b) Northwest, 28m
c) Southwest, 28m
14. Four of the following five are alike in a certain d) Northwest, 29m
way based on the given arrangement and thus e) Southwest, 29m
form a group. Which one of the following does
not belong to that group? 17. Match the following list I and list II according
a) K, the one who sits extreme left end in Row 1 to the final arrangement:
b) S, the one who sits third to the left of W
c) O, Q
d) F, the one who faces H
e) R, the one who sits second to the right of N

15. As many persons sit to the ____ of Q as to


a) A2, B3, C1, D4
the left of ____.
b) A3, B2, C4, D1
a) Right, E
c) A1, B4, C2, D3
b) Left, the one who faces P
d) A3, B1, C4, D2
c) Right, the one who sits immediate right of D
e) A4, B2, C3, D1
d) Left, K
e) Left, V
18. If R $ K (12), then what is the direction of R
with respect to N and what is the distance
Directions (16-20): Study the following
between R and N?
information carefully and answer the given
a) Southeast, 37m
questions.

Click Here For Bundle PDF Course | support@guidely.in Page 5 of 12


Bank Po Mains PDF Course 2024
Reasoning Day -14 (Eng)

b) Southeast, 20m c) Southeast


c) East, 20m d) Southwest
d) East, 37m e) South
e) Cannot be determined
20. How many points are to the southwest of O?
19. If the letters are changed to the next letter as a) Three
per the English alphabetical series, then what is b) Five
the direction of P with respect to L? c) More than five
a) Northeast d) Four
b) Northwest e) Two
Click Here to Get the Detailed Video Solution for the above given Questions
Or Scan the QR Code to Get the Detailed Video Solutions

Answer Key with Explanation


1. Answer: D 2. Answer: C

Click Here For Bundle PDF Course | support@guidely.in Page 6 of 12


Bank Po Mains PDF Course 2024
Reasoning Day -14 (Eng)

7. Answer: C
3. Answer: C 8. Answer: A
9. Answer: D
10. Answer: E
Final arrangement:

We have,
 Only four boxes are placed between the
box which has 66 toys and V. The box
4. Answer: B
which has 66 toys is not placed at the
topmost position.
Applying the above condition, there are three
possibilities.

5. Answer: E
None Follows

Directions (6-10):
6. Answer: B

Click Here For Bundle PDF Course | support@guidely.in Page 7 of 12


Bank Po Mains PDF Course 2024
Reasoning Day -14 (Eng)

Again we have, Again we have,


 The box which has 35 toys which is  The box which has 50 toys contains Y,
placed above V and three boxes below which is placed below W.
W.  The number of boxes placed between Y
 As many boxes placed below the box and the box which has 52 toys is two
which has 49 toys as above X, which is more than the number of boxes placed
not placed at the bottommost position. between the boxes which has 37 toys and
Applying the above conditions, Case 3 is T.
eliminated because there is no possibility to Applying the above conditions, there is one extra
place the box which has 35 toys have to possibility.
place above V.

Click Here For Bundle PDF Course | support@guidely.in Page 8 of 12


Bank Po Mains PDF Course 2024
Reasoning Day -14 (Eng)

Again we have,
 R is placed immediately below the box
which has 40 toys.
 More than four boxes are placed between
R and P, which is not placed at the
topmost position.
Applying the above conditions, there is one extra
possibility.

Again we have,
 The box which contains U has 42 toys.
 The box which has a square number of
toys neither contains S nor T.
 The remaining boxes are having 28 and
72 toys.

Click Here For Bundle PDF Course | support@guidely.in Page 9 of 12


Bank Po Mains PDF Course 2024
Reasoning Day -14 (Eng)

Applying the above conditions, Case 1, Case Directions (11-15):


2 and Cases 2a(1) are eliminated because 11. Answer: C
there is no possibility to place the box which 12. Answer: B
has 28 and 72 toys. Hence, Case 2a gives a 13. Answer: C
final arrangement. 14. Answer: E
15. Answer: D
Final arrangement:

We have,
 M sits third from one of the extreme ends.
 S, who faces south, sits second to the
right of the one who faces M.
 I, who sits diagonally opposite to S, sits
fourth to the right of O.
 As many persons sit to the left of O as to
the left of G, who sits opposite to the one
who sits adjacent to H.
Applying the above conditions, there are two
possibilities.

Click Here For Bundle PDF Course | support@guidely.in Page 10 of 12


Bank Po Mains PDF Course 2024
Reasoning Day -14 (Eng)

Again we have,
Again we have,
 More than four persons sit between L and
 More than one person sits between H and
P.
V, who faces the same direction as M
 P sits opposite to E, who sits adjacent to
faces.
K.
 The number of persons sitting between V
 Only two persons sit between F and T,
and R is two more than the number of
who faces H.
persons sitting between S and N.
 R does not sit adjacent to N.
 The one who sits opposite to R sits three
places away from K.

Click Here For Bundle PDF Course | support@guidely.in Page 11 of 12


Bank Po Mains PDF Course 2024
Reasoning Day -14 (Eng)

Applying the above conditions, Case 1 is Y (14) + 8 = 22


eliminated because there is no possibility to  “X $ Y (14)” means “X is 22m south of Y”
place P and L. Y (37) -13 = 24
Again we have,  “X & Y (37)” means “X is 24m west of Y”
 The number of persons sitting to the left Y (21) +5 = 26
of E is one less than the number of  “X @ Y (21)” means “X is 26m east Y”
persons sitting to the right of J, who faces From the above statement, we have:
Q.  J $ L (12); J is (12 +8) = 20m south of L
 D and W sit in the same row but face the  K @ L (15); K is (15 +5) = 20m east of L
opposite direction.  I # M (56); I is (56 -12) = 44m north of M
 D sits fifth from one of the extreme ends.  M $ K (17); M is (17 +8) = 25m south of K
Applying the above conditions, Case 2 gives a  K $& O; K is southwest of O
final arrangement.  O # Q (32); O is (32 -12) = 20m north of Q
 Q @ K(16);Q is (16 +5) =21m east of K
 O & P (35); O is (35 –13) = 22m west of P
 N & J (30); N is (30 –13) = 17m west of J

Directions (16-20):
16. Answer: E
17. Answer: B
18. Answer: D
19. Answer: A
20. Answer: C
Y (45) – 12 = 33
 “X # Y (45)” means “X is 33m north of Y”

Click Here For Bundle PDF Course | support@guidely.in Page 12 of 12


Bank Po Mains PDF Course 2024
Quantitative Aptitude Day -14 (Eng)

Quantitative Aptitude

Directions (01 - 04): Study the following information carefully and answer the questions given below.
The given table shows the total surface area and curved surface area of
cylinder, cone, sphere, cube and cuboid. [Take approximate value always]

1) The ratio of the radius of given cylinder and 3) The radius of the cone is ______ more than
another cylinder is 4:3 and the ratio of the height the radius of the sphere and the volume of the
of the given cylinder and another cylinder is 3:5. sphere is ____ more than the volume of the cone.
Find the difference between the volume of the Find which of the following value satisfy the
two cylinder? blank?
a) 3582 m3 a) 8 m, 2540 m3
b) 3642 m3 b) 7 m, 2259 m3
c) 3234 m3 c) 9 m, 2520 m3
d) 3842 m3 d) 11 m, 2549 m3
e) 3684 m3 e) None of these

2) Side of a square is 20% more than the side of 4) The Ratio of the radius of the sphere and the
a cube and the length of a rectangle is the radius of the circle is 2:3. The Area of the circle
average of the length and height of a cuboid and is L m2more than the total surface area of the
the width of the rectangle is 25% more than the cube. The Base of the triangle is √L m and the
breadth of the cuboid. Find the sum of the area height of the triangle is same as the radius of the
of the square and rectangle? [In cuboid , length > sphere. Find the area of the triangle?
height > breadth and length – breadth is 20] a) 42 m2
a) 445 m2 b) 58 m2
b) 645 m2 c) 64 m2
c) 654 m2 d) 48 m2
d) 554 m2 e) None the of these
e) 664 m2

Click Here For Bundle PDF Course | support@guidely.in Page 1 of 11


Bank Po Mains PDF Course 2024
Quantitative Aptitude Day -14 (Eng)

5) Total number of populations in village A is II.4800,35,4230


_______. The Ratio of the male and female III.6400,24,8776
population in village A is 5:3. The number of Find which of the is satisfy the blank?
male populations increased by 20% and the a) both I and II
number of female populations increased by b) only I
____% so that the total population of the village c) only II
now is _____. d) only III
I.3200,25,3900 e) None of these

Directions (06 - 09): Study the following information carefully and answer the questions given below.
The given table shows the speed (km/hr), length (m), time taken to cross a post and time taken to cross
platform P of five different trains [A, B, C, D and E].

6) Length of platform P is 25% less than the cross each other in 43.5 sec when moving in the
length of platform Q. Time taken by train C to same direction. Find which of the following is
cross platform Q is t sec. In (t+21) sec train A true?
cross platform S. Find the length of platform S? a) Speed of train F = Speed of train D
a) 445 b) Speed of train F/2 > speed of train D
b) 475 c) Speed of train F > 1.5*speed of train D
c) 460 d) Speed of train F< speed of train D
d) 485 e) Speed of train F> speed of train D
e) 560
8) Train A crosses platform R in ______ sec and
7) The Ratio of length of train D and F is 9:7. train B crosses platform T in ______ sec. Sum of
Train D and F cross each other when moving in length of platform R and platform T is _____.
same direction is 80 sec. Train F and train E Find which of the following is true?

Click Here For Bundle PDF Course | support@guidely.in Page 2 of 11


Bank Po Mains PDF Course 2024
Quantitative Aptitude Day -14 (Eng)

I.20,16, 310 c) only II


II. 25, 18, 425 d) only III
III. 32, 30, 485 e) None of these
a) only I and II
b) only I 10) The ratio of the speed of boat A and boat B
c) only II and III in still water is 5:6. Downstream speed of boat A
d) only III in river P is 30 km/hr and the upstream speed of
e) None of these boat A in the same river is ______. Boat B covers
_____ km in 10 hours in downstream and _____
9) Train B cross platform P in m sec and train C km in 7 hours in upstream.
crosses double the length of platform P in n sec. I. 20, 350, 175
Find the possible time taken by train E and D to II. 10,170, 125
cross each other when moving in same III. 18, 180, 120
direction? Find which of the following is satisfy?
I.5n+7 a) only I
II.2[m+n]+13 b) only II
III.5m+11 c) only III
a) both I and III d) all true
b) only I and II e) None of these

Directions (11 - 14): Study the following information carefully and answer the questions given below.
The pie chart shows the percentage distribution of the number of seminars [software + hardware]
organized by five different colleges [A, B, C, D and E] in 2022.

Click Here For Bundle PDF Course | support@guidely.in Page 3 of 11


Bank Po Mains PDF Course 2024
Quantitative Aptitude Day -14 (Eng)

Note – I. 2P+3Q+9R=58, II. 5P+4Q+7R=57, III.8P+9Q+4R=63 IV. S=2P. Total number of seminars
organized in five colleges is 300. The Ratio of software and hardware seminars organized in each college
is 2:1.
11) Total number of software seminars organized number of seminars organized by males and the
by colleges A, B and C is ______ and the total total number of seminars organized by female
number of hardware seminars organized by employees.
colleges C, D and E is ______. The Sum of two a) Q+R
blanks is _______. b) R+S
Find which of the following is true? c) P+Q
a) 14S, 30S, 190 d) P+S
b) 11R, 13Q, 200 e) P+R
c) 24R, 35P, 190
d) 28P, 38Q, 200 13) The ratio of the number of software seminars
e) none of these organized by E and F is 2:3, and the ratio of the
number of hardware seminars organized by D
12) Out of the total software seminars organized and F is 5:7. Find the possible value of the total
by College C, 40% were hosted by males and number of seminars organized by F?
the rest were hosted by females. Out of the total I.30R
hardware seminars organized by College C, 60% II.50S
were hosted by males and the rest were hosted III.75P
by females. Find the difference between the total a) all false

Click Here For Bundle PDF Course | support@guidely.in Page 4 of 11


Bank Po Mains PDF Course 2024
Quantitative Aptitude Day -14 (Eng)

b) all true The cost price of item A is Rs. 6X. The ratio of
c) only I cost price of items A and B is 3:4. The selling
d) only II price of item A is Rs. 72 more than the selling
e) None of these price of item E when both items are sold at 20%
profit. The ratio of the selling price of items A and
14) In 2023, college A wasincreased software C when item A is sold at 25% profit and item C is
seminars by 5.55% and decrease hardware sold at 35% profit is 25:42. Cost price of item C
seminars by 5.55%. In the same year, college B is Rs. 7Y. The cost price of item D is [6Z+20].
increased both hardware and software seminars The cost price of item E is 120. The cost price of
by 8.33%. Find the total number of seminars item D is Rs. 40 more than the cost price of item
organised by colleges A and B in 2023. C.
a) 94 16) Find which one is true?
b) 92 I. When item A is sold at Rs.6Y then the profit
c) 93 percentage is 20%.
d) 91 II. When item B is sold at 9X then the profit
e) None of these percentage is 12.5%.
III. when item C is sold at 6Z then the profit
15) Sum of the present age of A and B is _____ percentage is 10%.
years. The Ratio of age A after 3 years and age a) only III
of B after 2 years is ______ . The Age of C is b) only I
20% more than the A and the age of D is 20% c) only II
more than B. The Sum of Age of C and D is d) all three
______. e) only II and III
Find which of the following is satisfy the blank?
a) 85, 4:5, 92 17) When item C is sold at 25% profit. Find
b) 75, 7:8, 89 possible selling price of item C?
c) 45, 5:4, 64 I.7Z
d) 55, 7:8, 66 II.10Y-50
e) None of these III.12X-10
a) only I true
Directions (16 - 17): Study the following b) only II true
information carefully and answer the questions c) all false
given below. d) all true
e) only III

Click Here For Bundle PDF Course | support@guidely.in Page 5 of 11


Bank Po Mains PDF Course 2024
Quantitative Aptitude Day -14 (Eng)

Directions (18-20): Study the following e) None of these


information carefully and answer the questions 19) The difference between the number of
given below. laptops and phones sold in A is m; the difference
The ratio of the number of laptops sold in shops between the number of phones sold in B and D
A and B is 7:8. The ratio of the number of laptops is n; and the difference between the number of
and phones sold in Shop C is 7:8. The number of laptops sold in B and D is p. Find the value of
phones sold in shop B is double the number of mn+mp.
phones sold in A. The number of phones sold in a) 159
A is 20% less than the number of laptops sold in b) 147
B. The number of laptops sold in D is 28.57% c) 158
more than the number of laptops sold in C. The d) 185
ratio of the number of laptops sold in B to the e) None of these
number of phones sold in D is 8:5. The total
number of items sold in D is 79, and the total 20) The ratio of 5G and 4G phones sold in A is
number of phones sold in B and C together is 5:3. The ratio of 5G and 4G phones sold in B is
112. 3:5. The ratio of 5G and 4G phones sold in C is
18) The ratio of the number of laptops sold in D 2:1. The total number of 5G phones sold in A, B,
and E is 9:11, and the ratio of the number of and C together is (approx.)what percent of the
phones sold in C and E is 8:9. Find the total number of 4G phones sold in A, B, and C
difference between the total number of items together?
sold in E and B. a) 108%
a) 7 b) 112%
b) 5 c) 115%
c) 6 d) 117%
d) 9 e) None of these

Click Here For Bundle PDF Course | support@guidely.in Page 6 of 11


Bank Po Mains PDF Course 2024
Quantitative Aptitude Day -14 (Eng)

Click Here to Get the Detailed Video Solution for the above given Questions
Or Scan the QR Code to Get the Detailed Video Solutions

Answer Key with Explanation


1) Answer: C = [22/7] *28*28*21 – [22/7] *35*21*21
The radius of the cylinder and height are r and h = 51744-48510=3234 m3
respectively.
So, 2*[22/7]*r*h=3696 –(I) and 2) Answer: D
2*[22/7]*r*[r+h]=8624 ----(II) The side of the cube is a.
By divide (II) from I The total surface area of the cube is 6a2= 1350.
So, [r+h]/h=8624/3696 So, a2=1350/6=225
Or, r+h=2.33h So, a=15
Or, r=1.33h The total surface area of a cuboid is 2*[lb+bh+lh]
So, from I = 1744.
1.33h2=3696*7/(22*2)=588 And the curved surface area of a cuboid is
Or, h2=588/1.33=441 2h*[l+b]=1296.
Or, h=+21,-21 Where l, b, and h are the length, breadth, and
So, h=21 height of the cuboid, respectively. [l>h>b]
So, r=21*1.33=27.9=28 2lb=1744-1296=448, lb=224.
Another method: And l=20+b
2πr(h+r) - 2πrh = 8624 – 3696 Or, b*[b+20]=224
2πr2 = 4928 Or, b2+20b-224=0
r = 28 Or, b2+28b-8b-224=0
The radius of another cylinder is = 28*3/4=21 Or, (b+28)(b-8)=0, b=-28, b=8.
Height of another cylinder is = 21*5/3=35 So, the breadth is 8.
So, the difference in volume of two cylinder Length is 20+ 8 = 28.

Click Here For Bundle PDF Course | support@guidely.in Page 7 of 11


Bank Po Mains PDF Course 2024
Quantitative Aptitude Day -14 (Eng)

Height is = [1296/(36*2)] = 18. The total surface area of the sphere is =


So, the side of the square is = 15*120/100=18. 4*22/7*r2=2464
Rectangle length = [28+18]/2=46/2=23 Or, r2= [2464*7/(22*4)]=196, r=14
The width of the rectangle is equal to So, The radius of the circle is = 14*3/2=21
8*125/100=10. Area of circle = 22/7* 21*21=1386
So, the sum of the area is So, L=1386-1350=36
18*18+23*10=324+230=554 sq m. So, The base of the triangle is √36 =6
The Height of the triangle is = 14
3) Answer: B So, area = ½ * 6 *14=42 sq m
For sphere
The total surface area of the sphere is = 5) Answer: B
4*22/7*r2=2464 For value I,
Or, r2= [2464*7/(22*4)]=196, r=14 Male in village A is = 3200*5/8=2000
For cone, Female in village A is = 3200-2000=1200
Total surface area is = 22/7 * r *[r+l]=3300 Total population now = 2000*120/100 +
Curved surface area is = 22/7 * r *l=1914 1200*125/100=3900
So, (l+r)/l=3300/1914 Similarly, we can check others value also and
Or, l+r=1.7241l we see only I is satisfy.
Or, r=0.7241l
So, r*l=1914*7/22=609 Directions (06 - 09):
Or, l*[0.7241l]=609 The speed of train E is 72 km/hr = 72* 5/18 = 20
Or, l2=609/0.7241=841, l=29 m/sec.
So, r= [1914*7/[22*29]=21 Let the length of platform P be l.
So, h2=29*29-21*21=400, h=20 So, [l+260]/20=22
So, the radius of cone is 21-14= 7 more than Or, l=440-260=180 m.
radius of sphere Now, the speed of train A is 72 km/hr = 72* 5/18
Volume of sphere is = = 20 m/sec.
22/7*[14*14*14]*4/3=11498.66=11499 Let the length of train A be m.
Volume of cone is = 1/3 * 22/7* 21*21*20=9240 So, (m+180)/20=21
So, difference = 11499-9240=2259 Or, m=420-180=240 m.
The speed of train B is 250/10=25 m/sec.
4) Answer: A The speed of train C is 108 * 5/18 = 30 m/sec.
For sphere Length of train C is = 30*6=180 m

Click Here For Bundle PDF Course | support@guidely.in Page 8 of 11


Bank Po Mains PDF Course 2024
Quantitative Aptitude Day -14 (Eng)

The speed of train D is = [225+ 180]/27 = 15 =97=5n+7=5m+11


m/sec.
10) Answer: A
6) Answer: C For value of I,
Length of platform Q is = 180*100/75=240 The speed of boat A is [30+20]/2=25km/hr
So, t= [240+180]/30=420/30 =14 sec So, the speed of boat B is = 25*6/5=30km/hr
So, In [14+21]=35 sec train A cross = 35*20=700 The speed of the stream is 30-25=5km/hr
m So, boat B cover [30+5]*10=350 km in 10 hours
So, the length of platform S is = 700-240=460 m in downstream
Boat B cover [30-5]*7=175 km in 7 hours in
7) Answer: D upstream.
Length of train F is = 225*7/9=175 Similarly, we can check others' values also.
Let speed of train F is s. We can see only I is satisfying.
So, [175+225]/[15 ~ s]=80
So, [15~s]=400/80=5 Directions (11 - 14):
So, s= 20 or 10 I. 2P+3Q+9R=58, II. 5P+4Q+7R=57,
Now, [260+175]/(20-20) III.8P+9Q+4R=63
Or, [260+175]/(20-10)=43.50 By solving above equation, we get, P=2, Q=3,
So, speed of train F is 10 m/sec. R=5.
So, S=2*2=4.
8) Answer: B The Percentage distribution of number of
Let check option I, seminars in A, B, C, D and E is 6*3=18%,
Length of platform R, 20*20-240=160 = 12% , (2*3*5)=30%,
Length of platform T, 16*25-250=150 (2+3)*5=25%, 3*5=15% respectively.
So, sum is 160+150=310 The Number of seminars organized by A is =
Similarly, we can check other values also. 300*18/100=54
So, only I is satisfy. The Number of software seminars organized by
A is = 54*2/3=36
9) Answer: A THe Number of hardware seminars organized by
So, m = [180+250]/25=17.2 A is = 54-36=18
So, n=[180*2 +180]/30=18
So, train E and D cross each other in
[225+260]/[20-15]

Click Here For Bundle PDF Course | support@guidely.in Page 9 of 11


Bank Po Mains PDF Course 2024
Quantitative Aptitude Day -14 (Eng)

=24*108.33/100 + 12*108.33/100=39
So, the required sum is = 55+39=94

15) Answer: D
Let's check value of d,
Let the age of A after 3 years and age of B after
11) Answer: C 2 years is 7x and 8x.
The total number of software seminars So, 7x-3+8x-2=55
organized by colleges A, B and C is Or, 15x=55+5=60
= [36+24+60] =120 Or, x=60/15=4
The Total number of hardware seminars Age of A is =28-3 = 25 years
organized by colleges C, D and E is Age of B is = 32-2=30 years
= [30+25+15] =70 So, sum of C and D is = 25*120/100 +
So, the sum is 120+70=190 30*120/100=30+36=66
So, it is satisfied.
12) Answer: D
The total number of seminars organized by Directions (16 - 17):
males is The Cost price of item E is Rs.120.
= 60*40/100+30*60/100=24+18=42 The Selling price of item E when sold at 20%
The total number of seminars organized by profit is = 120*120/100=144
females is The Selling price of item A when sold at 20%
= 60*60/100+30*40/100=36+12=48 profit is = 144+72=216
So, required difference = [48-42]=6 So, the cost price of item A is =
216*100/120=180=6X, X=30
13) Answer: A So, the cost price of item B is = 180*4/3=240
The number of seminars organized by F is So, the selling price of item C when sold at 35%
= [30*3/2]+[25*7/5]=45+35=80is not equal to profit is
30R, 50S, 75P = [180*125/100]*42/25 =225*42/25=378
So, the cost price of item C is =
14) Answer: A 378*100/135=280=7Y, Y=40
Total number of seminars organised by A Cost price of D = 280+40=320=6Z+20,
=36*105.55/100 + 18*[100-5.55]/100=38+17=55 Z=300/6=50
Total number of seminars organized by B 16) Answer: C

Click Here For Bundle PDF Course | support@guidely.in Page 10 of 11


Bank Po Mains PDF Course 2024
Quantitative Aptitude Day -14 (Eng)

I. When item A sold at Rs.6Y then the profit The number of laptops sold in A and B is 7*5=35
percentage is 20%. and 8*5=40.
So, [240-180]*100/180=33.33% The number of phones sold in A is =6.4*5=32.
II. When item B sold at 9X then the profit The number of phones sold in B is =12.8*5=64.
percentage is 12.5%. Let the number of laptops and phones sold in C
[270-240]*100/240=12.5% be 7*6 = 42 and 8*6 = 48.
III. when item C is sold at 6Z then the profit The number of laptops sold in D is 9*6=54.
percentage is 10%. The number of phones sold in D is 5*5=25.
[300-280]*100/280=7.14% 18) Answer: E
The total number of items sold in E is 54*11/9 +
17) Answer: D 48*9/8 = 66+54 = 120.
So, required selling price = 280*125/100=350 So, the difference is = 120 – 40 – 64 = 16.
=12X-10=10Y-50=7Z
19) Answer: A
Directions (18-20): So, m=35-32=3.
Let the number of laptops sold in A and B be 7x So, n=64-25=39
and 8x. So, p=54-40=14
The number of phones sold in A is So, m*n + m*p = 3*39 + 3*14 = 159.
8x*80/100=6.4x.
The number of phones sold in B is 6.4x*2=12.8x. 20) Answer: B
The number of laptops and phones sold in C is The total number of 5G phones sold is 32*5/8 +
7y and 8y. 64*3/8 + 48*2/3.
The number of laptops sold in D is 7y*9/7=9y. = 20+24+32=76
The number of phones sold in D is 8x*5/8=5x. The total number of 4G phones sold is =
So, 9y+5x=79 and 12.8x+8y=112. [32+64+48] – 76 = 68.
By solving the above equation, we get x = 5 and So, the required percentage is [76/68]*100 =
y = 6. 112%.

Click Here For Bundle PDF Course | support@guidely.in Page 11 of 11


Bank Po Mains PDF Course 2024
English Day - 14

English Language
Directions (1-5): The given questions carry two 3.
sentences in which three words each have been Experts (A) noted that the dam, about 70
highlighted. These words might not be in their kilometres (44 miles) to the east of the city of
correct positions. You must choose an option Kherson, was (B) believed to be in disrepair and
which carries the correct pairs of words that (C) vulnerable to collapse
when interchanged would form correct and Besides a vigilant (D) approach like being
meaningful sentences. mindful of unusual odours, discolouration, or (E)
1. changes in food texture, cutting-edge technology
High global oil prices have (A) measure can play a pivotal role in (F) ensuring food
consequences for India that (B) imports 85% of safety.
its (C)transactions. A. A-D, B-F
The current (D) account—which is the B. B-E, A-D, C-F
broadest (E) seriousof India’s goods and C. A-E, C-D, B-F
services (F) requirements—slipped into a deficit D. B-D, A-C
A. A-E, B-D E. No swapping needed
B. B-D, C-F
C. A-E, C-F 4.
D. A-E, B-D, C-F Govt can’t do the (A) heavy lifting on recovery
E. No swapping needed alone, but the (B)transactionsfor business (C)
needs to get easier.
2. The current (D) account—which is the
The external affairs minister also (A)dangerthat broadest (E) measure of India’s goods and
India is making significant economic (B) impact services (F) environment.
that has been recognised (C)rang. A. A-D
Amid the disaster response, artillery shelling B. C-E
(D) globallyout as people scrambled to get out of C. B-F
the (E) saidzone, climbing onto military (F) trucks D. B-E
or rafts. E. No swapping needed
A. B-D, A-E, C-F
B. A-F, C-E 5.
C. C-D, B-F Punjab and Haryana farmers are (A) worsento
D. A-E, C-D paddy and a (B) importscycle that makes stubble
E. No swapping needed (C) hookedan imperative.

Click Here For Bundle PDF Course | support@guidely.in Page 1 of 12


Bank Po Mains PDF Course 2024
English Day - 14

High global oil prices have serious B. IV, III, I, I, II.


consequences for India that (D) cultivation85% of C. III, V, IV, II, I.
its requirements and will (E)burningthe current D. I, II, III, IV, V.
account (F) situation in FY22. E. None of the combination is correct.
A. A-E, B-D, C-F
B. Only A-E 7. One of the achievements of the NarendraModi
C. B-D, C-E government is the network of excellent highways
D. A-F, B-D, C-E across the country, which____________ matching
E. No swapping needed driving skills, especially as car manufacturersare
developing more powerful engines and foreign
Directions (6-10): In the following questions, a car makersare bringing their best wares. That
passage has been given with multiple blanks. can make for a deadly combination.And drivers
You are required to choose the appropriate word must _____________ that with great speedcomes
for each blank from the given options. great responsibility. The ______________ with
6. Science has been a powerful ____________for strapping onseatbelts is of recent origin. While
humanity’s progress. It has become those in the front seats do itwilly-nilly, backseat
the____________ of human follies and also passengers ______________ this safety measure.
theremedial tool that rescues us from Untilthe accident which claimed billionaire Cyrus
theirconsequences. Anti-microbial resistancehas Mistry in September 2022 on the Mumbai
become a major _____________ to human Ahmedabad Expressway, the seatbeltdid not get
survivaland well-being, as several multi- its due importance. Neither was it widely
drugresistant microbes have ____________ knownthat airbags do not ___________ if the
throughantibiotic misuse. The prospect of seatbelt is not strapped on.
findingpotent novel antibiotics ____________ dim I. Compliance
tillit was announced recently that II. Deploy
artificialintelligence (AI) helped to identify a III. Require
newantibiotic (again) that is highly IV. Remember
effectiveagainst the pathogenic bacterium. V. Ignore
I. Appeared A. III, IV, I, V, II.
II. Emerged B. V, II, I, IV, III.
III. Propellant C. IV, III, I, II, V.
IV. Threat D. II, III, V, IV, I.
V. Instrument E. None of the combination is correct.
A. V, IV, I, I, II.

Click Here For Bundle PDF Course | support@guidely.in Page 2 of 12


Bank Po Mains PDF Course 2024
English Day - 14

8. Indian education system_____________ the but also_______________ to the overallwell-being


required flexibilityover the years, which is of communities.By being _______________ in
verymuch required to take it tothe next level and foodsafety practices, one can significantlyreduce
make it truly global in approach andpracticality. the _________________of foodborne illnessesand
In Budget 23,the government has focusedon the their associated economicand social burdens.
_______________ of educationand skills I. Fatal
according tothe aptitude of the youthand the II. Paramount
demands of thefuture.It is for the first time thatan III. Contribute
equal ______________ is beinggiven to both IV. Proactive
education andskilling as part of the V. Incidence
neweducational policy, whichunburdens the A. IV, I, III, V, II.
students fromthe ______________ regulations B. II, I, V, III, IV.
andprocesses of the past. Thenew technological C. I, IV, II, V, III.
revolutionin recent years is helpinga lot in the D. III, V, II, I, IV.
process by creatingnew types of classroomswith E. None of the combination is correct.
unrestricted access to all,_______________ the
considerationslike region and distance. 10. It has been known for nearly a decade now
I. Obsolete that to_____________ the cops, drug traffickers
II. Lacks are ______________ to tradingon marketplaces
III. Blurring on the darknet — an Internet platformthat uses
IV. Emphasis secret alleys on The Onion Router (ToR),
V. Reorientation provides________________ to the traffickers and
A. IV, II, III, V, I. their customersand is, thus, difficult to detect and
B. II, V, III, I, IV. ____________. Theirmodus operandi involves
C. III, IV, V, III, I. clandestinely _____________ customerson social
D. II, V, IV, I, III. media, dealing in cryptocurrency andusing
E. None of the combination is correct. courier services for delivery.
I. Luring
9. Contaminated foodscan cause a__________ II. Anonymity
outcome.From posing a risk to life and III. Resorting
causingsevere permanent damage tothe body, it IV. Evade
is of _____________importance to identify V. Penetrate
foodhazards and ensure 360-degree food safety. A. V, III, II, IV, I.
Suchmeasures not only protectindividual health B. IV, III, II, V, I.

Click Here For Bundle PDF Course | support@guidely.in Page 3 of 12


Bank Po Mains PDF Course 2024
English Day - 14

C. IV, I, II, IV, III. could be transformative for energy-intensive


D. II, V, III, I, IV. sectors in India, starting with renewable power
E. None of the combination is correct. expansion to 450 GW. (I) With open source
technologies, India (1)/could incorporate
Directions (11-17): Read the following passage innovative materials and (2)/processes to
and answer the following questions. Some words decarbonise industry, transport and buildings,
are given in bold to help you answer some of the (3)/the biggest emitters, apart to power.(4)
questions. Many developed countries tend to view India’s
The U.S.-India Climate and Clean Energy reluctance to commit to a net zero emissions
Agenda 2030 Partnership raises expectations target as recalcitrance, but the climate change
that the coming decade will see sustained crisis originated not here but in the industrialised
financial and technological cooperation between world, which has used up much of the world’s
the two countries to cut greenhouse gas carbon space. A forward-looking policy should,
emissions. At the Leaders Summit on Climate therefore, envision green development anew,
organised by U.S. President Joe Biden, the providing funding and green technologies as
world’s attention was focused on countries compensation for the emissions space lost by
responsible for the highest carbon emissions. poorer countries.
India ranks third, behind the U.S. and China, This is a win-win game, since it would aid
although its per capita CO2 emissions are less sustainable development, boost employment,
than 60% of the global average, as Prime clean up the environment and, crucially, help all
Minister Narendra Modi pointed out. There is little countries emerge ……………. (II) from the
confidence in a pandemic-stricken world, pandemic. British Prime Minister Boris Johnson,
however, that future growth pathways will be who announced enhanced ambition at the
aligned away from fossil fuels. summit for Britain to cut carbon emissions by
The International Energy Agency, in fact, expects 78% by 2035 over 1990 levels, advanced the
a dramatic rise in emissions as countries race to agenda by calling for climate funding by rich
shake off the impact of the coronavirus, as they nations to exceed the decade-old goal of $100
did after the 2008 financial crisis. Yet, the years billion. For the India-U.S. agreement to yield
to 2030, as President Biden put it, are part of a results, Mr. Biden would have to persuade
“decisive decade”, and action to scale up funding industry and research institutions at home to
and innovation can help all countries move closer share knowledge and subsidise transfer of
to keeping global warming well below 2°C or technologies. He has won commendations for
even 1.5°C, as the Paris Agreement envisages. steering America around from the science-
There are many aspects to the bilateral pact that deprived Trump years and announcing enhanced

Click Here For Bundle PDF Course | support@guidely.in Page 4 of 12


Bank Po Mains PDF Course 2024
English Day - 14

ambition: cuts in emissions by 50% to 52% by 13. Which of the following statements correctly
2030 over 2005 levels. But much of his climate mention the conflict of ideas between Internal
effort will rely on executive authority, rather than Energy Agency and President Biden?
bipartisan support. With political will on both (I) Drastic rise in emissions is expected in
sides, the engagement with India can become a countries amid shaking off the pandemic impact
model. (II) The Paris Agreement envisages climate
11. According to the author, the U.S.-India actions taken by the countries at individual level
Climate and Clean Energy Agenda 2030 (III) The years till 2030 will be a decisive decade
Partnership arose what expectations? with bringing down the global warming through
(I) The partnership will witness financial innovation and scaled-up funding
cooperation between the two nations A. Only I
(II) It will boost the technological collaboration B. Only III
between US and India C. Both I and II
(III) The partnership will lead to collaborative D. Both I and III
climate actions between both the countries E. All of I, II, and III
A. Only I
B. Only III 14. What steps should be taken to yield the
C. Both I and II results from the India-US agreement?
D. Both II and III (I) Persuading industry and research facilities to
E. All of I, II, and III share knowledge
(II) Political will from both the countries
12. Which of the following statements can be (III) Climate effort of Biden will rely on executive
inferred from the first paragraph? authority and not on bipartisan support
A. The Leaders Summit on Climate aimed at the (IV) Subsidization of technologies’ transfer will
nations having the highest carbon emissions have to be encouraged
B. The rank of India is 4th followed by China and A. Both I and II
US B. Both II and III
C. The per capita CO2 emission of India is less C. Both I and IV
than 60% D. Both II and IV
D. It’s unlikely to align away the growth from E. All of the above
fossil fuels 15. Which of the following is the most opposite
E. Both A and C in meaning to the word ‘RECALCITRANCE’ as
highlighted in the given passage?
A. Resistant

Click Here For Bundle PDF Course | support@guidely.in Page 5 of 12


Bank Po Mains PDF Course 2024
English Day - 14

B. Obstinate B. Meticulous, preferring


C. Intransigent C. Careful, excluding
D. Cooperative D. Lax, including
E. Non-Compliant E. Careless, counted as

16. Which of the following words can fill in the 19. What ______ nearly 4,000 years ago in the
blank II to make the sentence grammatically court of the Kauravas is still happening because
correct and contextually meaningful? basic human nature has ________ changed.
A. Peddle A. Happened, hardly
B. Hulking B. Came about, not much
C. Healthier C. Materialised, bare
D. Incendiary D. Hit upon, slightly
E. None of these E. None of these

17. The italicized sentence I is divided into four 20. While Assam is the most _________ example,
parts. Find the part which contains an error. If several states in the Northeast have witnessed a
there is no error in any of the parts; mark your significant ________ from across the border.
answer as E. A. Serious, migrants
A. 3 B. Obvious, influx
B. 2 C. Evident, exodus
C. 4 D. Apparent, arrival
D. 1 E. Both (c) and (d)
E. No Error
21. This fact is _________ since about 96% of
Directions (18-22): The questions given below China’s container trade with Europe went
carry sentences in which words from the options ________ sea routes.
given below them can be filled to complete the A. Importance, across
same grammatically and meaningfully. You must B. Venerable, haywire
choose the most appropriate option for each C. Crucial, via
sentence as your answer. D. Paramount, though
18. Our regulators have had a record of ______ E. None of these
and poor oversight over unlisted companies’
_________ startups.
A. Negligible, of

Click Here For Bundle PDF Course | support@guidely.in Page 6 of 12


Bank Po Mains PDF Course 2024
English Day - 14

22. A _________ search operation was set in B. Massive, imploded


motion and hours later the vessel was found C. Tremendous, splintered
_________ with deceased individuals. D. Vast, burnt
A. Evacuation, mended E. Immense, sunk
Click Here to Get the Detailed Video Solution for the above given Questions
Or Scan the QR Code to Get the Detailed Video Solutions

Answer Key with Explanation


1. Answer: C said should come after the external affairs
The word serious should come before minister which makes A-E one correct pair to be
consequences as it acts as a perfect adjective interchanged.
that defines the nature of the consequences. The wordglobally should come after recognised.
This makes A-E one good pair to be swapped. The phrase rang out means to be heard loudly
The options carrying the same are (a), (c) and and clearly… This means that the word in C
(d). should come in D.
The wordrequirements should come in C as 85% This makes option (d) the most logical answer
of the required oil is imports is being mentioned choice.
in the line. This meaning can only be understood
when the words in C and F are interchanged as 3. Answer: E
well. The given words are in their correct positions
This makes option (c) the most logical answer and need no interchange.
choice. This makes option (e) the most logical answer
choice.
2. Answer: D
The words that should be swapped between 4. Answer: C
themselves are given in option (d) as, the word

Click Here For Bundle PDF Course | support@guidely.in Page 7 of 12


Bank Po Mains PDF Course 2024
English Day - 14

The word environment should come in C the other words are grammatically incorrect.
because the environment for business is being Because of the misuse, microbes have emerged.
talked about. Then, the remaining word “appeared” should be
This tells us that the words in B and F need to be filled in the fifth blank.
interchanged. Therefore, option C is the correct answer.
Therefore, the correct answer is option (c).
7. Answer: A
5. Answer: A Here, the passage is about driver’s safety and
The hints: the first sentence is expressed in a positive note.
 Stubble – burning(is a method of So, we cannot use the word “ignore” and the
removing paddy crop residues from the word “compliance” which is a noun, so “require”
field to sow more). will be placed here. The second sentence says
 Hooked to - very interested and that drivers should go in speed with great
enthusiastic about something responsibility, they have to keep this in mind, for
 Cultivation – cycle (one crop being sown that, the word “remember” should be used. Here,
after the other) the word “the” is present which will precede the
These hints tell us that the pairs given in the first noun usually so it will take “compliance” in the
option are correct and (a) should be marked the blank. The fourth statement says backseat
answer. passengers should also use this safety measure.
From this, we can infer “ignore” should be placed
6. Answer: C here. The fifth blank will take “deploy” as the
Propellant – a fuel force or something that answer.
propels. Therefore, option A is the correct answer.
Here, the blank requires a word that describes
the word “science”, because “science” is helping 8. Answer: D
humans to get progress. So, the word The second sentence says something is still
“propellant” should be used. The second needed to move to the next level, so we can
statement indicates that it works as a remedial infer that flexibility is missing in the Indian
tool from overcoming the consequences, so the Education System, so the word “lacks” should be
word “instrument” should be used. The third used in the first blank. In 2023 Budget,
statement says anti-microbial resistance is government has focused to reshape this, so the
becoming something to human survival and well- word “reorientation” should be used. The third
being, here the word “threat” should be filled, as statement says that this is the first time

Click Here For Bundle PDF Course | support@guidely.in Page 8 of 12


Bank Po Mains PDF Course 2024
English Day - 14

government is considering both, so it has given blank as the word “detect” is used. The word
importance, for that the word “emphasis” should “anonymity” should be used in the third blank as
be used. Here, the fourth blank requires a word the word “provides” require a noun to complete
that should describe the word “regulation” an the sentence. How the customers get involved
adjective is needed so the word “obsolete” into it, they are “lured” by social media.
should be used. The word “blurring” should be Therefore, option B is the correct answer.
used in the fifth blank.
Therefore, option D is the correct answer. 11. Answer: C
Refer to the introductory lines of the first
9. Answer: E paragraph, you can draw the hint from the lines,
Here, the blanks should be filled with the The U.S.-India Climate and Clean Energy
answers in the sequence I, II, III, IV and V. Agenda 2030 Partnership raises expectations
Because, eating contaminated food will result to that the coming decade will see sustained
deadliest consequence. For that, the word “fatal” financial and technological cooperation between
would come. To avoid this, we need to give the two countries to cut greenhouse gas
importance to food safety, for that the word emissions.
“paramount” should be used. If the importance is Hence, C is the right answer choice where I and
given to food, it will not only help individuals but II are the only correct statements.
also communities, for that “contribute” would be
correct. The fifth blank requires a noun so 12. Answer: E
“incidence” should be placed and “proactive” Read the lines of first paragraph, you can extract
should be placed at fourth blank. the hint from the below quoted lines,
Therefore, option E is the correct answer. At the Leaders Summit on Climate organised by
U.S. President Joe Biden, the world’s attention
10. Answer: B was focused on countries responsible for the
Luring – tempt. highest carbon emissions. India ranks third,
Anonymity – the condition of being unknown. behind the U.S. and China, although its per
Resorting – The act of turning to a person or capita CO2 emissions are less than 60% of the
thing for help or as a means of achieving global average, as Prime Minister Narendra
something. Modi pointed out. There is little confidence in a
Here, only the first and the fourth blank will have pandemic-stricken world, however, that future
base form verbs, as the word “to” precede. The growth pathways will be aligned away from fossil
word “penetrate” should be placed in the fourth fuels.

Click Here For Bundle PDF Course | support@guidely.in Page 9 of 12


Bank Po Mains PDF Course 2024
English Day - 14

Hence, E is the right answer choice in which A climate effort will rely on executive authority,
and C are the only correct statements and can rather than bipartisan support. With political will
be inferred from the first paragraph of the given on both sides, the engagement with India can
passage. become a model.
Hence, E is the right answer choice where all the
13. Answer: D given statements stand true as per the given
Read the lines of second paragraph, you can information in the passage.
take the hint from the below quoted lines,
The International Energy Agency, in fact, 15. Answer: D
expects a dramatic rise in emissions as Recalcitrance: having an obstinately
countries race to shake off the impact of the uncooperative attitude towards authority or
coronavirus, as they did after the 2008 financial discipline.
crisis. Yet, the years to 2030, as President Biden Antonym: Apologetic, Cooperative
put it, are part of a “decisive decade”, and action Resistant: offering resistance to something or
to scale up funding and innovation can help all someone.
countries move closer to keeping global warming Obstinate: stubbornly refusing to change one's
well below 2°C or even 1.5°C, as the Paris opinion or chosen course of action, despite
Agreement envisages. attempts to persuade one to do so.
Hence, D is the right answer choice. Intransigent: unwilling or refusing to change
one's views or to agree about something.
14. Answer: E Non-Compliant: failing to act in accordance with
For the right answer, read the lines of thelast a wish or command.
paragraph, you can draw the hint from the below Hence, D is the right answer choice where
quoted lines, ‘Cooperative’ is the antonym of the given word
. For the India-U.S. agreement to yield results, ‘Recalcitrance’.
Mr. Biden would have to persuade industry and
research institutions at home to share 16. Answer: C
knowledge and subsidise transfer of ‘Healthier’ can fill in the blank II to make the
technologies. He has won commendations for sentence grammatically correct and contextually
steering America around from the science- meaningful.
deprived Trump years and announcing Peddle: promote (an idea or view) persistently or
enhanced ambition: cuts in emissions by 50% to widely
52% by 2030 over 2005 levels. But much of his Hulking: large, heavy, or clumsy

Click Here For Bundle PDF Course | support@guidely.in Page 10 of 12


Bank Po Mains PDF Course 2024
English Day - 14

Incendiary: tending to excite Materialised means to become real; to happen…


Hence, option C is the most viable answer bare means uncovered. Both these words make
choice. no sense in the given sentence.
Hit upon means discover or think of something,
17. Answer: C especially by chance… This word cancels the
Here, the error is in the4th part of the sentence. fourth option as well.
‘From’ should be used instead of ‘to’ after ‘apart’ The only option carrying both the correct words
to make the sentence grammatically correct and is (a).
contextually meaningful. Hence, C is the right
answer choice where 4th part of the italicized 20. Answer: B
sentence is erroneous. The first option can be ruled out as the word
migrants does not fit in grammatically.
18. Answer: D The word obvious means easily seen or
The word negligible means very small and understood; clear. Influx means large numbers
therefore not important. This can be taken as of people or things arriving suddenly… (this fits
correct but the second word of makes no sense as it describes people coming from across the
in the context. The word meticulous is same as border). Exodus is the opposite of influx as it
careful but both cannot fill the first blank as they means the mass departure of people. Apparent
will go contextually incorrect. The choice for the means the same as obvious or evident but
second blank in the last option makes it arrival is not fit here. It is best used with respect
incorrect. to railway stations or airports etc.
Lax means not having high standards; not strict. The only pair of words that fits correctly can be
The words given in option (d) fill the sentence found in option (b).
correctly.
21. Answer: C
19. Answer: A The first blank can take the words in the third
The first blank can take the word happened as and the fourth options only and not the others.
the sentence itself gives a hint by using the The word venerable means highly respectable…
phrase ‘still happening’ in the later part of the The word haywire means erratic; out of control…
sentence. important could have been a better choice
The phrase came about also means happened grammatically for choosing the first option. The
but the choice for the second blank does not fit fourth option gets eliminated for the usage of the
in. word though…

Click Here For Bundle PDF Course | support@guidely.in Page 11 of 12


Bank Po Mains PDF Course 2024
English Day - 14

The best combination of words that can be found collapsed. This word describes the state of the
in the options hence, is (c). vessel mentioned in the sentence. Tremendous
is very large or great; very good but this carries a
22. Answer: B positive tone making it unfit for the sentence
Evacuation means the removal of persons or given above. Splintered means to split or rend
things from an endangered area. The sentence into long thin pieces.
carries the word search after the first blank Of all the given options, the pair of words that
which would make the given word redundant. make the sentence grammatically and
Mended means to repair something that is meaningfully correct is (b).
damaged or broken… imploded means

Click Here For Bundle PDF Course | support@guidely.in Page 12 of 12


Bank Po Mains PDF Course 2024
Reasoning Day -15 (Eng)

Reasoning Aptitude
Directions (1-5): Study the following information Gujarat. Both Baseball and the kabaddi matches
carefully and answer the given below. were held in the same state but not held in the
Eight trophies of different games viz. Cricket, same state as chess match. Final match of the
Chess, Kabaddi, Kho-Kho, Carrom board, game which trophy was kept second from the
Volleyball, Baseball and Hockey were kept in a bottom of the stack in arrangement 1 was held in
single stack in the college office room one above neither Karnataka nor Gujarat. Hockey match
another, but not necessarily in the same order. was not held in the same state as Kabaddi
The final of each match was held in different match. Both Volleyball and hockey matches were
states viz. Haryana, Karnataka and Gujarat. At held in the same state.
least two and not more than three games were Arrangement 3:
held in each state. These games were held in Final match of the game which trophy was kept
different years 1988, 1996, 2001, 2005, 2010, at the bottom of the stack in arrangement 1 was
2013, 2017 and 2021. held in an even numbered year. The difference
Arrangement 1: between the years in which chess and baseball
Only three trophies were kept between the matches were held is a multiple of 11. Cricket
trophies of Cricket and Carrom board. As many match was held immediately before baseball
trophies kept above cricket trophy as below match. The difference between the years of the
baseball trophy. Hockey trophy was kept matches held in Haryana other than cricket in
immediately below the cricket trophy. Baseball arrangement 2 is 12 years. Only three matches
trophy was not kept adjacent to Cricket trophy. were held between Carrom board and hockey,
Only two trophies were kept between the which was not held immediately after chess
trophies of hockey and Kabaddi. Neither kabaddi match. The difference between the years in
nor hockey trophy was kept adjacent to chess which Baseball and Kabaddi matches were held
trophy. Volleyball trophy was kept three trophies is a multiple of 7.
above Kho-Kho trophy, which was not kept at the 1. Volleyball match was held in which of the
bottom of the stack. following year as per the arrangement 3?
Arrangement 2: a) 1988
Cricket match was held in Haryana but not with b) 2005
chess. Final match of the game which trophy was c) 1996
kept immediately above baseball trophy in d) 2021
arrangement 1 was not held in Karnataka. Final e) 2013
match of the game which trophy was kept at the
top of the stack in arrangement 1 was not held in

Click Here For Bundle PDF Course | support@guidely.in Page 1 of 14


Bank Po Mains PDF Course 2024
Reasoning Day -15 (Eng)

2. As per arrangement2, in which state only two c) The match which was held immediately after
matches were held and name those two matches Carromboard match
respectively? d) Chess match
a) Haryana, Kho-Kho and Volley ball e) None of these
b) Karnataka, Baseball and Kabaddi
c) Haryana, Cricket and Hockey Directions (6-10): Study the following information
d) Gujarat, Carromboard and Kabaddi carefully and answer the given below.
e) Gujarat, baseball and cricket &@©≤*+!$#%=~^
Step I: Consider the first four prime numbers in
3. Which of the following game trophy was kept the number series, add “2” to each of these
three stacks below Kho-Kho trophy as per numbers and write all numbers one by one after
arrangement 1? every third special character from left to right in
a) Volleyball trophy the same order.
b) The trophy which is kept immediately above Step II: Pick out the letters from the English
Volleyball trophy alphabetical series which have its corresponding
c) Baseball trophy place value is a multiple of “6” and write all such
d) The trophy which is kept at the bottom of the letters in reverse alphabetical order (one by one)
stack from left to right after every second element in
e) Carromboard trophy the series thus formed after step I.
Step III: Pick out the vowels except I from the
4. Which of the following match was held in English alphabetical series, and write all such
Gujarat as per arrangement 2? vowels(one by one) in alphabetical order from
a) Kho-Kho right to left after fifth, twelve, fifteen and
b) Baseball nineteenth element from the right end in the
c) Chess series thus formed after step II.
d) Both a and b Step IV: The letters which have its place value of
e) Both b and c even number in the alphabetical series are
replaced by second succeeding letter and the
5. If Kabaddi match is related to chess match letters which have its place value of odd numbers
and baseball match is related to cricket match in are replaced by preceding letter, in the series
a certain way in arrangement 3, then which of thus formed after step III.
the following match is related to hockey match? Thus step IV is the final step and answer the
a) The match which was held in the year 2001 below questions according to the final step.
b) Volleyball match

Click Here For Bundle PDF Course | support@guidely.in Page 2 of 14


Bank Po Mains PDF Course 2024
Reasoning Day -15 (Eng)

6. Which of the following element is the ninth c) !


element to the left of the third consonant from the d) 4
right end? e) Z
a) @
b) 1 10. How many symbols are there in the series
c) Z which is/are immediately proceeded by a
d) T consonant and immediately followed by a
e) * number?
a) One
7. If first 16 elements are reversed, then how b) Two
many such letters are there in the series which is c) Three
either immediately preceded by or immediately d) More than three
followed by a symbol (but not both)? e) None
a) Six
b) Seven Directions (11-15): Study the following
c) Eight information carefully and answer the given
d) Five below.
e) Nine Eighteen persons from D to U are sitting in three
parallel rows viz. row-1, row-2, and row-3 such
8. If 9 is related to = and D is related to * in a that row-2 is to the north of row-3 and row-1 is to
certain way, then 7 is related to which of the the north of row-2. 5 persons are sitting in row-1
following element? where 9 persons are sitting in row-2 and 4
a) 4 persons are sitting in the row-3. Persons sitting in
b) ^ row-3 faces north. Persons sitting in row-1 face
c) 5 south. The first 5 persons sitting from west to
d) * east in row-2 face north and the last four persons
e) $ sitting from west to east in row-2 face south.
Note: All the persons sitting in row1 and row 3
9. If all the letters which are immediately followed are facing the persons sitting in row 2 each
by a symbol are dropped, then which of the other.
following element is sixth to the right of the fifth K sits opposite to the one who sits second to the
element from the left end? right of U, who neither sits at the end of the row
a) 5 nor facing north. Only three persons sit between
b) $ K and G. As many persons sit to the left of U as

Click Here For Bundle PDF Course | support@guidely.in Page 3 of 14


Bank Po Mains PDF Course 2024
Reasoning Day -15 (Eng)

to the right of M, where both of them facing e) All I, II and III


opposite directions. Only two persons sit
between L and the one who sits opposite to M. J 13. Who among the following person sits north-
sits north-west of L but doesn’t sit at the end of east of the one who sits immediate right of P?
the row. The number of persons sitting to the left a) O
of J is one less than the number of persons b) The one who sits second to the right of I
sitting to the right of S. E sits opposite to the one c) H
who sits second to the right of S. K doesn’t sit d) The one who sits second to the left of S
adjacent to E.Only three persons sit between P e) The one who sits immediate left of U
and T, who sits second to the right of I. The
number of persons sitting between T and L is 14. What is the position of O with respect to K?
one less than the number of persons sitting a) Fourth to the right
between F and O, who sits opposite to Q. N sits b) Immediate left
immediate left of Q. The number of persons c) Second to the right
sitting between N and D is one more than the d) Third to the left
number of persons sitting between H and R, who e) Fifth to the left
doesn’t sit at the end.
11. Who among the following person sits exactly 15. Four of the following five are alike in a certain
between I and the one who sits opposite to J? way as per the given arrangement and thus form
a) O a group. Find the one that does not belong to the
b) The one who sits second to the right of K group.
c) L a) QN
d) The one who sits immediate right of L b) LF
e) N c) PI
d) RM
12. Who among the following persons are sitting e) TE
opposite to each other?
I) MT Direction (16-20): Study the following information
II) LQ carefully and answer the questions given below.
III) UF There are eight units placed one above the other
a) Only I and II in a single stack. Each unit contains a book and
b) Only III some boxes. The lowermost unit is numbered as
c) Only II and III 1 and the unit immediately above it is numbered
d) Only I as 2 and so on. Eight different books viz. L, M, N,

Click Here For Bundle PDF Course | support@guidely.in Page 4 of 14


Bank Po Mains PDF Course 2024
Reasoning Day -15 (Eng)

O, P, Q, R, and S are placed in each unit, but not units between S and the unit with a height of
necessarily in the same order. There is a certain 45cm. Book N is placed immediately above the
number of boxes which are placed one above the unit which has half of the boxes of book P. Book
other in each unit and the total height of each P is placed above Book R which is placed
unit is between 20cm and 75cm with all the immediately above the unit with a height of
boxes in each unit having the same height. No 72cm. Book O is placed three units below the
unit has the same height and the same number unit which has 3 boxes. Book Q does not have
of boxes. the lowest number of boxes. The height of the
Note: unit with book S is 2cm more than the unit which
i) If the total height of the unit is 12cm and the has 4 boxes. One of the units is 24cm in height.
number of boxes in that unit is 3, then the height The height of the Unit with book S is less than
of each box will be 4cm. the height of the Unit with book M.
ii) The total height of each unit will be the multiple 16. Which of the following book has maximum
of the number of boxes in that unit. At least two boxes along with it?
boxes are there in each unit. a) Q
iii) The unit which has an odd number of boxes b) R
kept on an even number unit and vice versa. The c) The book which is kept immediately below S
total number of boxes in each unit is not more d) M
than 9. e) The book which is kept immediately above O
Book L is placed on an even-numbered unit but
below unit number five. Only two books are 17. What is the total number of boxes of the unit
placed between L and the unit with a height of with books O, M, and S?
32cm. Book M is placed two units below the unit a) 9
with five boxes which is kept above the unit with b) 14
a height of 32cm. The lowermost unit doesn’t c) 12
have a height of 32 cm. The number of units d) 10
above book M is the same as the number of units e) 8
below the unit with a height of 52cm. Book S is
placed three units above the unit which has 6 18. How many books are kept above the unit
boxes. The unit with a height of 35cm is placed with a height of 24cm?
two units below the unit which has 9 boxes. The a) One
height of the unit with book L is an even number. b) Two
The number of units between L and the unit c) Three
which has 7 boxes is the same as the number of d) Four

Click Here For Bundle PDF Course | support@guidely.in Page 5 of 14


Bank Po Mains PDF Course 2024
Reasoning Day -15 (Eng)

e) More than four 20. Which of the following statement is true?


a) Book S kept on an even-numbered unit
19. What is the height of unit 6? b) No unit is there between L and N
a) 30cm c) Book R has 2 boxes
b) 45cm d) The height of the unit which has book S is
c) 72cm 32cm
d) 35cm e) All statements are true
e) None of these
Click Here to Get the Detailed Video Solution for the above given Questions
Or Scan the QR Code to Get the Detailed Video Solutions

Answer Key with Explanation


Directions (1-5):
1. Answer: D
2. Answer: B
3. Answer: D
4. Answer: C
5. Answer: B
Final arrangement
Arrangement 1:

Arrangement 2:

Click Here For Bundle PDF Course | support@guidely.in Page 6 of 14


Bank Po Mains PDF Course 2024
Reasoning Day -15 (Eng)

Arrangement 3:

Again we have,
 Only two trophies were kept between the
trophies of hockey and Kabaddi.
 Neither kabaddi nor hockey trophy was
kept adjacent to chess trophy.
Arrangement 1:
Here Case2 and 4 get eliminated
We have,
 Only three trophies were kept between
the trophies of Cricket and Carrom board.
 As many trophies kept above cricket
trophy as below baseball trophy.
 Hockey trophy was kept immediately
below the cricket trophy.
 Baseball trophy was not kept adjacent to
Cricket trophy.
From the above condition, there are five
possibilities
Again we have,
 Volleyball trophy was kept three trophies
above Kho-Kho trophy, which was not
kept at the bottom of the stack.
So Case1 and 5 get eliminated, hence Case-3
shows the final arrangement

Click Here For Bundle PDF Course | support@guidely.in Page 7 of 14


Bank Po Mains PDF Course 2024
Reasoning Day -15 (Eng)

 Both Baseball and the kabaddi matches


were held in the same state but not held
in the same state as chess match.
 Final match of the game which trophy was
kept second from the bottom of the stack
in arrangement 1 was held in neither
Karnataka nor Gujarat.
Here Case1 and 1a get eliminated

Arrangement 2:
We have,
 Cricket match was held in Haryana but
not with chess.
 Final match of the game which trophy was
kept immediately above baseball trophy in
arrangement 1 was not held in Karnataka.
From the above condition, there are four
possibilities Again we have,
 Hockey match was not held in the same
state as Kabaddi match.
 Both Volleyball and hockey matches were
held in the same state.
So Case2a gets eliminated, hence Case-2
shows the final arrangement

Again we have,
 Final match of the game which trophy was
kept at the top of the stack in
arrangement 1 was not held in Gujarat.

Click Here For Bundle PDF Course | support@guidely.in Page 8 of 14


Bank Po Mains PDF Course 2024
Reasoning Day -15 (Eng)

Arrangement 3:
 Final match of the game which trophy was
kept at the bottom of the stack in
arrangement 1 was held in an even
numbered year.
 The difference between the years in
which chess and baseball matches were
held is a multiple of 11.
 Cricket match was held immediately
before baseball match.
From the above condition, there are three Again we have,

possibilities  The difference between the years in


which Baseball and Kabaddi matches
were held is a multiple of 7.
So Case-3 gets eliminated, hence Case-1 shows
the final arrangement

Again we have,
 The difference between the years of the
matches held in Haryana other than
cricket in arrangement 2 is 12 years.
 Only three matches were held between
Carromboard and hockey, which was not
held immediately after chess match.
Here Case-2 gets eliminated Direction (6-10):
We have:
&@©≤*+!$#%=~^
Step I: Consider the first four prime numbers in
the number series, add “2” from each of these

Click Here For Bundle PDF Course | support@guidely.in Page 9 of 14


Bank Po Mains PDF Course 2024
Reasoning Day -15 (Eng)

numbers and write all numbers one by one after Thus we have the following result:
every third special character from left to right in &@TZ©4TN≤*ND+5H!$#7Z%=~9
the same order. ^
For Example: 2+2=4, 3+2=5, 5+2=7, 7+2=9 6. Answer: D
Thus we have the following result: Final series
&@©4≤*+5!$#7%=~9^ &@TZ©4TN≤*ND+5H!$#7Z%=~9
Step II: Pick out the letters from the English ^
alphabetical series which have its corresponding -> Third consonant from the right end is D and
place value is a multiple of “6” and write all such ninth to the left of D is T
letters in reverse alphabetical order (one by one)
from left to right after every second element in 7. Answer: B
the series thus formed after step I. Final series
Then, we have the letters: F, L, R, X & @ T Z ©4T N ≤ * N D + 5H ! $ # 7 Z % = ~ 9 ^
Thus we have the following result: After reversing first 16 elements
&@X©4R≤*L+5F!$#7%=~9^ ! H 5 + DN *≤ N T 4 © ZT @& $ # 7 Z % = ~ 9 ^
Step III: Pick out the vowels except I from the
English alphabetical series, and write all such 8. Answer: E
letters(one by one)in alphabetical order from Final series
right to left after fifth, twelve, fifteen and & @ T Z © 4 T N ≤ * N D + 5 H !$ # 7 Z % = ~ 9
nineteenth element from the right end in the ^
series thus formed after step II. 9 is second to the right of = and D is second to
The vowels from the English alphabetical series the right of * in the same way 7 is related to $.
are: A, E O and U
Thus we have the following result: 9. Answer: A
&@UX©4RO≤*LE+5F!$#7A%=~9 Final series
^ & @ TZ © 4 T N ≤ * N D + 5 H ! $ # 7 Z % = ~ 9
Step IV: The letters which have its place value of ^
even number in the alphabetical series are After letters are dropped
replaced by second succeeding letter and the &@T©4T≤*N+5!$#7%=~9^
letters which have its place value of odd 4 is the fifth element from the left end and sixth
numbers are replaced by preceding letter, in the to the right of 4 is 5
series thus formed after step III.
U-21; X-24; R-18; O-15; L-12; E-5; F-6; A-1 10. Answer: B

Click Here For Bundle PDF Course | support@guidely.in Page 10 of 14


Bank Po Mains PDF Course 2024
Reasoning Day -15 (Eng)

Final series
& @ TZ © 4 T N ≤ * N D + 5H !$ # 7 Z % = ~ 9 ^
Z©4 and D+5

Direction (11-15):
11. Answer: D
12. Answer: B
13. Answer: E
14. Answer: B
15. Answer: C
Final arrangement

Again we have,
 As many persons sit to the left of U as to
the right of M, where both of them facing
opposite directions.
 Only two persons sit between L and the
one who sits opposite to M.

We have,
 K sits opposite to the one who sits second
to the right of U, who neither sits at the
end of the row nor facing north.
 Only three persons sit between K and G.
From the above condition, there are three
possibilities

Click Here For Bundle PDF Course | support@guidely.in Page 11 of 14


Bank Po Mains PDF Course 2024
Reasoning Day -15 (Eng)

persons sitting between F and O, who sits


opposite to Q.
 N sits immediate left of Q.
 The number of persons sitting between N
and D is one more than the number of
persons sitting between H and R, who
doesn’t sit at the end.
Again we have,
So Case-1 and 2 get eliminated, hence Case-3
 J sits north-west of L but doesn’t sit at the
shows the final arrangement
end of the row.
 The number of persons sitting to the left
of J is one less than the number of
persons sitting to the right of S.
 E sits opposite to the one who sits second
to the right of S.
 K doesn’t sit adjacent to E.

Directions (16-20):
16. Answer: E
17. Answer: D
18. Answer: C
19. Answer: B
20. Answer: A
Again we have,
Final Arrangement
 Only three persons sit between P and T,
who sits second to the right of I.
 The number of persons sitting between T
and L is one less than the number of

Click Here For Bundle PDF Course | support@guidely.in Page 12 of 14


Bank Po Mains PDF Course 2024
Reasoning Day -15 (Eng)

Again we have,
 The number of units above book M is the
same as the number of units below the
unit with a height of 52cm.
 Book S is placed three units above the
unit which has 6 boxes.
From the above condition, case-2a gets
eliminated.

We have,
 Book L is placed on an even-numbered
unit but below unit number five. Only two
books are placed between L and the unit
with a height of 32cm.
 Book M is placed two units below the unit
with five boxes which is kept above the
Again we have,
unit with a height of 32cm.
 The unit with a height of 35cm is placed
 The lowermost unit doesn’t have a height
two units below the unit which has 9
of 32 cm
boxes. The height of the unit with book L
From the above condition, there are three
is an even number.
possibilities.
 The number of units between L and the
unit which has 7 boxes is the same as the
number of units between S and the unit
with a height of 45cm

Click Here For Bundle PDF Course | support@guidely.in Page 13 of 14


Bank Po Mains PDF Course 2024
Reasoning Day -15 (Eng)

Again we have,
 Book N is placed immediately Again we have,
above the unit which has half of the  One of the units is 24cm in height.
boxes of book P.  The height of the Unit with book S is less
 Book P is placed above Book R than the height of the Unit with book M.
which is placed immediately above From the above condition, case1 shows the final
the unit with a height of 72cm. arrangement.
 Book O is placed three units below
the unit which has 3 boxes.
 Book Q does not have the lowest
number of boxes. The height of the
unit with book S is 2cm more than
the unit which has 4 boxes.
From the above condition, case2 gets
eliminated.

Click Here For Bundle PDF Course | support@guidely.in Page 14 of 14


Bank Po Mains PDF Course 2024
Quantitative Aptitude Day -15 (Eng)

Quantitative Aptitude

Directions (1-4): Study the following data 3) Series that follows a certain logic given below
carefully and answer the questions: and contains one wrong term. There is another
1) Given below is a number series that follows series that follows the same logic as in first
certain logic. If ‘P’ is the nth term and ‘Q’ is ‘n + series and starts with the wrong term of first
1’th term, then find the correct relationship series. Find the 3rd term of second series?
between ‘P’ and ‘Q’. Series I: 2, 14, 112, 672, 3360, 13440, 40320
Series: 8, 25, 76, 229, 688 A.784
A.Q = 3P – 1 B.6272
B.Q = 2P + 3 C.5376
C.Q = 3P + 1 D.896
D.Q = 2P – 1 E.4704
E.Q = 2P – 3
4) Two series I and II with different logics are
2) There are two series I and II. Logic in both the given below. Series I contains one wrong
series are different. element termed as A and series II contains one
Series I: 3, 12, 28, 53, 89, ……. missing element termed as B. Which of the
Series II: 259, 270, 283, 300, 319. following is not TRUE regarding A and B?
Which of the following will be the first term which Series I: 18480, 1680, 240, 48, 18, 8
is common in both the series? Series II: 200, 79, ?, 5, -4, -8
A.504 A.HCF of A and B is 6.
B.342 B.LCM of A and B is 90.
C.383 C. Total number of factors of A are 6.
D.283 D. Total number of factors of B are 8.
E.371 E.A > B

Directions (5-8): Study the following data carefully and answer the questions:
There are 6 shopkeepers A, B, C, D, E and F. Each shopkeeper purchased two types of bread i,e;
Banana bread and Garlic bread and sold only some of the breads.
The Pie chart given below shows the percentage distribution of the number of banana bread sold by each
shopkeeper on a particular day.

Click Here For Bundle PDF Course | support@guidely.in Page 1 of 12


Bank Po Mains PDF Course 2024
Quantitative Aptitude Day -15 (Eng)

The table given below is the average number of banana bread and that of garlic bread sold by
shopkeepers B, C, D, E and F.

Note:
1: Number of garlic bread sold by shopkeeper A is 40 more than that of banana bread sold by him.
2: Average number of banana bread and garlic bread sold by shopkeeper A is 90% of the number of
garlic bread sold by him.
5) Number of banana bread sold by C is 60% of total unsold bread (banana + garlic) of C are
the total banana bread purchased by him and the rotten, then find the number of unsold bread
number of garlic bread sold by C is 66(2/3) % of (banana + garlic) of C, which are not rotten?
total garlic bread purchased by him. If 20% of A.164

Click Here For Bundle PDF Course | support@guidely.in Page 2 of 12


Bank Po Mains PDF Course 2024
Quantitative Aptitude Day -15 (Eng)

B.120 C.50
C.180 D.25
D.148 E.45
E.152
9) In container A, contains the mixture of milk
6) Number of banana bread sold by C and F and water. The ratio of milk and water in
together is what percent less than the number of container A is 8:5. 39L mixture is taken out and
garlic bread sold by C and F together? put it on container B. when 4L milk and 5L water
A.34% is added in container A,then the ratio of milk to
B.42% water in the container A becomes 3:2.Find the
C.40% difference between the total milk in container A
D.32% and B together and total water in container A and
E.36% B together?
A.29 L
7) If 66(2/3) % of total bread (banana + garlic) B.20 L
sold by A are sold to males, out of which 40% C.22 L
are banana bread and 50% of total bread D.31 L
(banana + garlic) sold by D are sold to males, out E. None of these
of which 40% are banana bread, then find the
number of garlic bread sold by A and D together 10) If A started the business with initial
to males. investment of Rs. 6000 and after X months B
A.204 joined the business with initial investment of Rs.
B.216 9000. 3 months after B joining the business A
C.184 withdraws 1/4th of his initial investment. At the
D.198 end of the year A’s share is Rs. 3850 out of total
E.210 profit Rs. 8800. Find the value of X?
A.7 months
8) Shopkeeper B sold 82% of the total number of B.4 months
bread (banana + garlic) purchased by him and C.6 months
sold 80% of banana bread purchased by him. D.3 months
Find the number of unsold garlic bread of E.5 months
shopkeeper B.
A.40 11) Ratio of the radius of the cylinder to the
B.30 length of the rectangle is 1:2 and the breadth of

Click Here For Bundle PDF Course | support@guidely.in Page 3 of 12


Bank Po Mains PDF Course 2024
Quantitative Aptitude Day -15 (Eng)

the rectangle is 15% of the perimeter of the and speed ratio between Train B and Train A is
rectangle. If the height of the cylinder is half of 5:3. If the two Trains are travelling in opposite
the breadth of the rectangle and the curved direction they will meet after 10 seconds. Find
surface area of the cylinder is 528 cm2, then find the ratio between time taken by Train A and
the radius of the cylinder? Train B to cross the platform has length of
A.7 cm 220m?
B.14 cm A.7:8
C.3.5 cm B.9:7
D.21 cm C.9:5
E. Cannot be determined D.8:9
E.7:9
12) If the ratio between length of two Trains A
and B is 2:3. The speed of Train B is 180 kmph

Directions (13-16): Study the following information carefully and answer the questions given below.
The given table shows the average number of people who visit park A and B and the percentage of the
number of people who visit park B is more than that of A and the ratio of the number of people who visit
park A and C in two different seasons in five different states.

Click Here For Bundle PDF Course | support@guidely.in Page 4 of 12


Bank Po Mains PDF Course 2024
Quantitative Aptitude Day -15 (Eng)

13) What is the ratio of the total number of 15) The difference between the total number of
people who visit park C in summer season in people who visit park B and C in winter season in
Tamilnadu and Bihar together to the total number Bihar is what percent of the difference between
of people who visit park B in winter season in the number of people who visit park A and C in
Tamilnadu and Kerala together? summer season in Assam?
A.21:16 A.37.78%
B.22:17 B.39.10%
C.7:5 C.41.67%
D.11:8 D.43.56%
E. None of these E.45.89%

14) What is the difference between the number 16) What is the difference between the number
of people who visit park B and C together in of people who visit park B and C together in
winter and summer season in Assam? winter season in Manipur and the number of
A.2460 people who visit park A and C in summer season
B.2465 in Kerala?
C.2470 A.9000
D.2475 B.9200
E.2480 C.9100
D.9300
E.9400

Directions (17-20): Study the following data carefully and answer the questions:
There are 5 car manufacturing companies P, Q, R, S and T. Each company manufactures three types of
cars i,e; TUV, SUV and KUV.
The bar graph given below shows the number of SUVs manufactured by a company as percent of that of
TUVs manufactured by the respective company and also shows the average number of TUVs, SUVs and
KUVs manufactured by a company in a month.

Click Here For Bundle PDF Course | support@guidely.in Page 5 of 12


Bank Po Mains PDF Course 2024
Quantitative Aptitude Day -15 (Eng)

Note:
1: Ratio of the number of TUVs manufactured by P and R is 4: 5 respectively and the difference between
them is 37.5% of the number of TUVs manufactured by Q.
2: Ratio of the number of TUVs manufactured by R and S is 3: 2 respectively and the number of TUVs
manufactured by Q is 40.
3: Average number of TUVs manufactured by S and T is 65.
17) Cost of manufacturing one SUV in of the month and the ratio of the number of SUVs
companies P, R and S is Rs.5 Lakh, Rs.6 Lakh to that of KUVs manufactured by company Q in
and Rs.4 Lakh respectively, then find the the last 5 days is 5: 2, then find the ratio of
average total cost of manufacturing SUVs of number of SUVs to that of KUVs manufactured
these three companies in a month. by company Q in first 25 days. (Assume 30 days
A.Rs.30 million in the month)
B.Rs.36 million A.4: 7
C.Rs.24 million B.2: 3
D.Rs.40 million C.5: 9
E.Rs.32 million D.1: 2
E.3: 5
18) If 16(2/3) % of total KUVs manufactured by
company Q are manufactured in the last 5 days

Click Here For Bundle PDF Course | support@guidely.in Page 6 of 12


Bank Po Mains PDF Course 2024
Quantitative Aptitude Day -13 (Eng)

19) Total number of KUVs manufactured by 20) Total number of TUVs produced by
companies R and S together is what percent companies P and T together is what percent of
more than that of TUVs manufactured by these that of KUVs produced by these two companies
two companies together? together?
A.83% A.87.33%
B.76% B.91.33%
C.80% C.93.33%
D.85% D.85.33%
E.78% E.89.33%
Click Here to Get the Detailed Video Solution for the above given Questions
Or Scan the QR Code to Get the Detailed Video Solutions

Answer Key with Explanation


1) Answer: C 53 + 62 = 89
Logic in the series is: 89 + 72 = 138
(8 * 3) + 1 = 25 138 + 82 = 202
(25 * 3) + 1 = 76 202 + 92 = 283
(76 * 3) + 1 = 229 283 + 102 = 383
(229 * 3) + 1 = 688 Logic in series II:
Hence, 259 + 11 = 270
Q = 3P + 1 270 + 13 = 283
283 + 17 = 300
2) Answer: D 300 + 19 = 319
Logic in series I: 319 + 23 = 342
3 + 32 = 12 Hence, we can see that the first term which is
12 + 42 = 28 common in both the series is 283.
28 + 52 = 53

Click Here For Bundle PDF Course | support@guidely.in Page 7 of 12


Bank Po Mains PDF Course 2024
Quantitative Aptitude Day -15 (Eng)

3) Answer: A (d): Total number of factors of B = 8


Logic in the series is: (e): A < B
2 * 8 = 16 Hence, option (e) is not TRUE.
16 * 7 = 112
112 * 6 = 672 Directions (5-8):
672 * 5 = 3360 Let the number of banana bread sold by
3360 * 4 = 13440 shopkeepers A, B, C, D, E and F are ‘20x’, ‘12x’,
13440 * 3 = 40320 ‘15x’, ‘10x’, ‘25x’ and ‘18x’ respectively.
Wrong term = 14 So, number of garlic bread sold by shopkeeper A
New series: = 20x + 40
14 * 8 = 112 And average of number of banana bread and
112 * 7 = 784 garlic bread sold by shopkeeper A = (20x + 40) *
14, 112, 784 (90/100)
Hence, 3rd term of second series = 784 Now,
(20x + 20x + 40)/2 = (20x + 40) * (90/100)
4) Answer: E 40x + 40 = (20x + 40) * (9/5)
Logic in series I: 200x + 200 = 180x + 360
18480 ÷ 11 = 1680 x=8
1680 ÷ 7 = 240 Average of number of banana bread and garlic
240 ÷ 5 = 48 bread sold by shopkeeper A = (20 * 8 + 40) *
48 ÷ 3 = 16 (Not 18) (90/100) = 180
16 ÷ 2 = 8
A = 18 = 21 * 32
Logic in series II:
200 – 112 = 79
79 – 72 = 30
30 – 52 = 5
5 – 32 = -4
-4 – 22 = -8
B = 30 = 21 * 31 * 51
(a): HCF of 18 and 30 = 6
(b): LCM of 18 and 30 = 90 5) Answer: E
(c): Total number of factors of A = 6 Number of banana bread sold by C = 120

Click Here For Bundle PDF Course | support@guidely.in Page 8 of 12


Bank Po Mains PDF Course 2024
Quantitative Aptitude Day -15 (Eng)

So, the total number of banana bread, Total number of breads (banana + garlic) sold by
purchased by C: A = 160 + 200 = 360
120 * (100/60) = 200 Number of breads (banana + garlic) sold by A to
Number of garlic bread sold by C = 220 males = 66(2/3) % of 360 = 240
So, the total number of garlic bread, purchased Number of garlic breads sold by A to male =
by C: 60% of 240 = 144
220 * (300/200) = 330 Total number of breads (banana + garlic) sold by
Total number of breads (banana + garlic), D = 80 + 160 = 240
purchased by C: Number of breads (banana + garlic) sold by D to
200 + 330 = 530 males = 50% of 240 = 120
Total number of unsold bread (banana + garlic) Number of garlic breads sold by D to male =
of C: 60% of 120 = 72
530 – 120 – 220 = 190 Number of garlic breads sold by A and D
Since 20% of total unsold bread (banana + together to males = 144 + 72 = 216
garlic) of C are rotten.
So, the number of unsold bread (banana + 8) Answer: B
garlic) of C, which are not rotten: Total number of breads (banana + garlic) sold by
80% of 190 = 152 B = 96 + 150 = 246
So, total number of breads (banana + garlic)
6) Answer: A purchased by B:
Number of banana bread sold by C = 120 246 * (100/82) = 300
Number of banana bread sold by F = 144 Number of banana breads sold by B = 96
Total number of banana bread sold by C and F So, number of banana breads purchased by B =
together = 120 + 144 = 264 96 * (100/80) = 120
Number of garlic bread sold by C = 220 And number of garlic breads purchased by B =
Number of garlic bread sold by F = 180 300 – 120 = 180
Total number of garlic breads sold by C and F Number of garlic breads sold by B = 150
together = 220 + 180 = 400 So, number of unsold garlic breads of B = 180 –
Required percentage = [(400 – 264)/400] * 100 = 150 = 30
34%
9) Answer: A
7) Answer: B Let milk and water in container A is 8x and 5x
respectively.

Click Here For Bundle PDF Course | support@guidely.in Page 9 of 12


Bank Po Mains PDF Course 2024
Quantitative Aptitude Day -15 (Eng)

In 39L milk and water is 39*(8/13)=24 and Radius of the cylinder = 7 * 4/2 = 14 cm
39*(5/13)=15L respectively.
So, we can say, (8x-24+4)/(5x-15+5) = 3/2 12) Answer: B
Or, 16x-40=15x-30 The speed of Train B = 180 kmph = 180 * 5 / 18
Or, x=10 L = 50 m/s
So, in container A amount of milk is = 80- The speed of Train A = 50 * 3 / 5 = 30 m/s
24+4=60 L Length of two trains = 10 * (50 + 30) = 800
In container A amount of water is = 50-15+5=40 Length of Train A = 800 * 2 / 5 = 320m
L Length of Train B = 800 * 3 / 5 = 480m
Required difference = (60+24) – (40+15) = 29 L Time taken by Train A to cross the platform has
10) Answer: D length of 220m = (320 + 220) / 30 = 18s
At the end of the year, Time taken by Train B to cross the platform has
A’s share =Rs. 3850 length of 220m = 700 / 50 = 14s
B’s share = Rs. 4950 Required ratio = 18:14 = 9:7
Profit ratio of A to B = 7:9
(6000*X + 6000*3 + 4500*(9-X)) / (9000*3 + Directions (13-16):
9000(9-X)) = 7/9 Tamilnadu in winter season:
(1500x + 58500) / (10800 – 9000x) = 7/9 Number of people visit park A and B = 12900 * 2
765x = 2295 = 25800
X=3 A + A * 115/100 = 25800
A = 12000
11) Answer: B B = 25800 – 12000 = 13800
Ratio of the breadth and perimeter of the C = 5/4 * 12000 = 15000
rectangle =15:100 = 3:20 Similarly we can find the remaining values.
Breadth of the rectangle = 3x
Perimeter of the rectangle = 20x
Radius of the cylinder = y
Length of the rectangle = 2y
2 * (3x + 2y) = 20x
2y = 7x
y = 7x/2 13) Answer: A
2 * 22/7 * 7x/2 * 3x/2 = 528 Required ratio = (22400 + 19600):(13800 +
x=4 18200)

Click Here For Bundle PDF Course | support@guidely.in Page 10 of 12


Bank Po Mains PDF Course 2024
Quantitative Aptitude Day -15 (Eng)

= 42000:32000 Since, the average number of TUVs


= 21:16 manufactured by S and T is 65.
So, number of TUVs manufactured by T = (2 *
14) Answer: D 65) – 50 = 80
Required difference = (15000 + 16875) – (13800
+ 15600)
= 2475

15) Answer: C
Required % = (14000 – 12500)/(15600 – 12000)
* 100
=1500/3600*100
= 41.67%

17) Answer: A
16) Answer: B
Number of SUVs manufactured by company P in
Difference = (18000 + 28800) - (17600 + 20000)
a month = 72
= 9200
So, total cost of manufacturing SUVs of
company P in a month = 72 * 500000 = Rs.36
Directions (17-20):
million
Since, number of TUVs manufactured by Q = 40
Number of SUVs manufactured by company R in
So, difference between number of TUVs
a month = 60
manufactured by P and R = 37.5% of 40 = 15
So, total cost of manufacturing SUVs of
Since, the ratio of number of TUVs
company R = 60 * 600000 = Rs.36 million
manufactured by P and R is 4: 5 respectively.
Number of SUVs manufactured by company S in
So, number of TUVs manufactured by P = 15 *
a month = 45
4/ (5 – 4) = 60
So, total cost of manufacturing SUVs of
And number of TUVs manufactured by R = 60 *
company S = 45 * 400000 = Rs.18 million
(5/4) = 75
Required average = (36 + 36 + 18)/3 = Rs.30
Since, the ratio of number of TUVs
million
manufactured by R and S is 3: 2 respectively.
So, number of TUVs manufactured by S = 75 *
18) Answer: D
(2/3) = 50
Total number of KUVs manufactured by
company Q in the month = 60

Click Here For Bundle PDF Course | support@guidely.in Page 11 of 12


Bank Po Mains PDF Course 2024
Quantitative Aptitude Day -15 (Eng)

So, number of KUVs manufactured by company 105 + 115 = 220


Q in first 25 days of the month = 83(1/3) % of 60 Total number of TUVs manufactured by
= 50 companies R and S together:
Number of SUVs manufactured by company Q in 75 + 50 = 125
last 5 days of the month = 16(2/3) % of 60 * (5/2) Required percentage = [(220 – 125)/125] * 100 =
= 25 76%
Total number of SUVs manufactured by
company Q in the month = 50 20) Answer: C
So, number of SUVs manufactured by company Total number of TUVs produced by P and T
Q in the first 25 days of the month = 50 – 25 = 25 together = 60 + 80 = 140
Required ratio = 25: 50 = 1: 2 Total number of KUVs produced by P and T
together = 48 + 102 = 150
19) Answer: B Required percentage = (140/150) * 100 =
Total number of KUVs manufactured by 93.33%
companies R and S together:

Click Here For Bundle PDF Course | support@guidely.in Page 12 of 12


Bank Po Mains PDF Course 2024
English Day - 15

English Language
Directions (1-7): Given below are a few rubbing it in. It can cause your eyes then become
questions based on the passage. You have to red, irritated and watery," said Dr Gopal Pillai. To
read the passage carefully and answer the given prevent dry eyes, wash your eyes frequently,
questions accordingly. If none of the options are suggested Dr Pillai. "After about every few
correct then choose option E as your answer. minutes, voluntarily, close your eyes tightly and
Summer has its own set of infections that are then open them. Watch your eyes more often as
spread rampantly if a proper lifestyle is not it will help lubricate the eyeballs," he said. To
followed. Our eyes go through a great deal of combat dryness, use tear substitutes like eye
stress and pressure, especially due to the drops as well. Dr Sarang Goel, Medical Director,
summer sun that is bright and scorching. Cases Ayu Health Network of Hospitals, told
of ocular diseases like conjunctivitis are IndiaToday.in that rehydration is important for
reportedly heavily in the hot months and taking eye health.
necessary precautions is of utmost importance. "Rehydration is extremely important in summer.
Eyes are quite sensitive to the ultraviolet (UV) Avoid any chance of dehydration as it can affect
rays of the sun. In summer, prolonged exposure the eyes. While applying moisturiser, rub it
to UV modifies the lens proteins, leading to around your eyes. Wear a hat when out in the
various problems like cataract formation and sun or use an umbrella. Most importantly do not
worsening eyesight. UV can also damage the rub your eyes," said Dr Sarang Goel.
retina with a high risk of cancer like basal cell Another major problem in summer is ocular
carcinoma and squamous cell carcinoma. Dr diseases like conjunctivitis, keratitis,
Gopal S Pillai, Clinical Professor, Head of endophthalmitis, cellulitis and stye, among
Opthalmology, Amrita Hospital, Kochi, told others. Dr Pillai said that conjunctivitis is one of
IndiaToday.in people should wear UV the most common eye problems in the hot
sunglasses to protect their eyes. This will also months.
prevent dryness in the eyes, caused by the sun "Conjunctivitis spreads via human touch.
and more screen exposure. Someone could use an infected towel or even a
"Dryness is a major problem in eye health these door knob that has been touched by an infected
days, especially among children who study for a person. In a classroom, one child could infect
long time and those who are in front of the another child if they have conjunctivitis. It's a viral
screens. Many people look at their screens for 8- infection, so it spreads rampantly," the expert
14 hours a day which can cause extreme said.
dryness. In summer, dryness is a feeling in which Ideally, a healthy well balanced diet is essential
a person puts mud in their eyes and starts for eye health along with that we must have

Click Here For Bundle PDF Course | support@guidely.in Page 1 of 9


Bank Po Mains PDF Course 2024
English Day - 15

antioxidants like beta carotene, vitamins C and 2. Which of the following option is incorrect as a
E, Omega 3 and zinc. These help in preventing result of prolonged exposure to UV in the
the development of serious eye conditions. Avoid summer?
unprocessed foods on a daily basis," Dietician A. UV can also damage hair follicles and causes
Jyoti Khaniojh, Dietetics, Nutrition And Dietetics, hairfall.
Max Super Speciality Hospital, Patparganj, New B. UV can also damage the retina with a high risk
Delhi, told IndiaToday.in. of cancer like basal cell carcinoma and
When we eat a meal plate, which is colourful like squamous cell carcinoma
a rainbow, we get all sorts of antioxidants in our C. prolonged exposure to UV modifies the lens
diet. proteins, leading to various problems like
Since our childhood, we have been taught that cataract formation and worsening eyesight
an apple a day keeps the doctor away. Likewise, D. Both C&D
carrots are known to be best for the eyes. It is E. None of the above
rich in beta-carotene and vitamin, which protects
the eye against infections," Jyoti Khaniojh said. 3. Which of the following option is the synonym
Besides this, add lemon and citrus fruits to your of “VOLUNTARILY”?
diet for vitamin C intake. Nuts and seeds that A. willingly
contain vitamin E prevent cataracts and age- B. reluctantly
related macular degeneration. C. unwillingly
Salmon fish is a rich source of omega-3 and this D. unintentionally
healthy fat helps in prevent eye dryness as it is E. None of the above
great for retina health.
Egg yolks contain vitamin A, zinc, lutein and 4. Which of the following is not one of the
zeaxanthin and zinc, which help in age-related reasons given by Dr. Pillai to prevent dry eyes?
degeneration of eyes. Zinc helps in night A. You should wash your eyes frequently
improvement of night vision. B. After about every few minutes, voluntarily,
close your eyes tightly and then open them
1. Which of the following option is the antonym of C. Watch your eyes more often as it will help
RAMPANTLY? lubricate the eyeballs
A. unbridled D. Wash your eyes with hygienic soap frequently
B. unhindered E. None of the above
C. unhampered 5. Which of the following is not an example of
D. restrained ocular diseases?
E. None of the above A. conjunctivitis

Click Here For Bundle PDF Course | support@guidely.in Page 2 of 9


Bank Po Mains PDF Course 2024
English Day - 15

B. cellulitis A. a
C. endophthalmitis B. b
D. orthopthalmitis C. c
E. None of the above D. d
E. e
6. Why does an infected towel or even a door
knob is susceptible to conjunctivitis? 9. While the rapidly growing Indian aviation
A. Because it spreads only via human touch sector offers ample opportunities (a)/ for private
B. Because it spreads only via clothes carriers to launch operations (b)/, he remains an
C. Because it spreads only via saliva unforgiving space (c)/, considering the number of
D. Because it is a viral infection and it spreads airlines that have been forced to shut shop due
rampantly to financial troubles (d). No error (e).
E. None of the above A. a
B. b
7. Which of the food item is not mentioned by Dr. C. c
jyoti khanioj for a healthier eye? D. d
A. carrot E. e
B. lemon juice
C. selmon fish 10. In November 2019 (a)/, the bankruptcy court
D. zinga fish ordered a corporate insolvency resolution
E. None of the above process for the grounded regional low-cost air
carrier, Air Costa (b)/, aviation regulator
Directions (8-12): Given below are a few Directorate General of Civil Aviation (DGCA) in
questions divided into five parts which may or June 2017 suspended the flying license for Air
may not contain errors in them. You have to find Costa (c)/, making it permanently stop operations
the part containing errors in them. If none of the (d). No error (e).
parts contains error then choose option E as A. a
your answer. B. b
8. Banga, 63, was nominated for the post by US C. c
President Joe Biden (a)/ in late February and D. d
was the sole contender (b)/ to replace departing E. e
World Bank chief David Malpass (c)/, an 11. Sahara Airlines was first established on Sept
economist and former US Treasury official during 20, 1991 (a)/, and later begun its operation after
the Trump administration (d). No error (e). 2 years on December 3, 1993 (b)/. Two Boeing

Click Here For Bundle PDF Course | support@guidely.in Page 3 of 9


Bank Po Mains PDF Course 2024
English Day - 15

737-200 aircraft served as Sahara Airlines (c)/. company (b) indicated that it could lose more
On October 2, 2000, the airline was rebranded to subscribers in the current quarter.
Air Sahara, it controlled 12 per cent of India's One of the reasons behind growing losses could
domestic flight market at its peak (d). No error be the sharp fall in subscribers in the South Asia-
(e). focused Disney+ Hotstar after it lost the (c)
A. a streaming rights to the popular Indian Premier
B. b League (IPL) cricket matches.
C. c A. Only a
D. d B. Only b
E. e C. Only c
D. Both a&b
12. The World Bank has been led by an E. None of the above
American (a)/ since its founding at the end of
World War Two (b)/, while the International 14. While Go First had (a) blamed P&W for its
Monetary Fund has been led by an European financial troubles and bankruptcy filing, the US
(c)/. Banga, who was born in India and spent his engine maker said earlier that the airline’s (b)
early career there, has been a US citizen since claim was “unfounded” as it had not paid for the
2007 (d). No error (e). maintenance and lease charges for years, which
A. a (c) penultimately led to the necessary
B. b suspension of services.
C. c A. Only a
D. d B. Only b
E. e C. Only c
D. Both a&c
Directions (13-16):Given below are a few E. None of the above
questions with three words highlighted which
may or may not used appropriately as per the 15. While a (a) surge in traffic in India, the
context of the sentence. You have to find the world’s third-biggest aviation market in the world,
word which is used out of the context of the led to record jet orders, the (b) failure of two
sentence. If all words are appropriately used major airlines – Kingfisher Airlines in 2012 and
then choose option E as your answer. Jet Airways in 2019 – have made lessors (c)
13. Disney+ lost a (a) whopping 4 million comprehensive about the market.
subscribers in just the second quarter. The A. Only a

Click Here For Bundle PDF Course | support@guidely.in Page 4 of 9


Bank Po Mains PDF Course 2024
English Day - 15

B. Only b D. who expect lower food prices to aid the


C. Only c decline
D. Both a&b E. No improvement is required
E. None of the above
18. Taiwanese electronics giant Foxconn has
16. The Go First bankruptcy decision has also bought a huge tract of land on the outskirts of
made lessors nervous about another financially- Indian tech hub Bengaluru, the key Apple
troubled airline, SpiceJet. While SpiceJet said it supplier said in a filing as it looks to diversify
has no plans of filing for insolvency, lessors are production towards China.
worried that a similar move by the airline could A. he looks to diversify production away from
(a) spoil their (b) respect of (c) repossessing China
planes. B. she looks to diversify production away from
A. Only a China
B. Only b C. them looks to diversify production toward
C. Only c China
D. Both b&c D. it looks to diversify production away from
E. None of the above China
E. No improvement is required
Directions (17-20): Given below are few question
with a highlighted phrase which may or may not 19. Apple have making its own push into India
need improvement. You have to find the correct and chief executive Tim Cook last month opened
replacement of the phrase if required. If no its first two retail stores in the world's most
improvement is required then choose option E as populous country.
your answer. The California-based firm is betting big on the
17. India's headline retail inflation in May is likely nation of 1.4 billion people -- home to the
to fall further towards 4% — the midpoint of the second-highest number of smartphone users in
central bank's target and a level last seen in the world, after China.
January 2021, according to a few economists, A. Apple have been making its own push into
which expect lower food prices to add the India
decline. B. Apple have been made its own push into India
A. who expect lower food prices to add the C. Apple has been making its own push into
decline India
B. who expect lower food prices to aid the rise D. Apple has been made its own push into India
C. who expect lower food prices to add the rise E. No improvement is required

Click Here For Bundle PDF Course | support@guidely.in Page 5 of 9


Bank Po Mains PDF Course 2024
English Day - 15

A. Adani Ports has been floated a tender of up to


20. Adani Ports had floated a tender of up to B. Adani Ports had been floated a tender of up to
$130 million of 3.375% 2024 maturity dollar- C. Adani Ports was floated a tender of up to
denominated bonds late last month, as it seeks D. Adani Ports were floated a tender of up to
to boost investor confidence after the group's E. No improvement is required
shares were pummelled earlier this year by a US
short-seller's report.
Click Here to Get the Detailed Video Solution for the above given Questions
Or Scan the QR Code to Get the Detailed Video Solutions

Answer Key with Explanation


1. Answer: D Head of Opthalmology, Amrita Hospital, Kochi,
Here, rampant means showing no sign of being told IndiaToday.in people should wear UV
under control and restrained means just the sunglasses to protect their eyes. This will also
opposite to it whereas all other options are prevent dryness in the eyes, caused by the sun
synonyms of rampant. So, option D is the correct and more screen exposure. We can say from the
answer. options that hair fall is nowhere mentioned in the
passage. So, option A is the correct answer.
2. Answer: A
Here, from the lines, In summer, prolonged 3. Answer: A
exposure to UV modifies the lens proteins, Here, voluntarily means of one’s own free will
leading to various problems like cataract and here willingly is the only synonym whereas
formation and worsening eyesight. UV can also all other options are antonyms. So, option A is
damage the retina with a high risk of cancer like the correct answer.
basal cell carcinoma and squamous cell
carcinoma. Dr Gopal S Pillai, Clinical Professor, 4. Answer: D

Click Here For Bundle PDF Course | support@guidely.in Page 6 of 9


Bank Po Mains PDF Course 2024
English Day - 15

Here, from the lines, Dryness is a major problem rampantly," the expert said. We can say that
in eye health these days, especially among option D is the correct answer.
children who study for a long time and those who
are in front of the screens. Many people look at 7. Answer: D
their screens for 8-14 hours a day which can Here, as per the lines, When we eat a meal
cause extreme dryness. In summer, dryness is a plate, which is colourful like a rainbow, we get all
feeling in which a person puts mud in their eyes sorts of antioxidants in our diet. Since our
and starts rubbing it in. It can cause your eyes childhood, we have been taught that an apple a
then become red, irritated and watery," said Dr day keeps the doctor away. Likewise, carrots are
Gopal Pillai. To prevent dry eyes, wash your known to be best for the eyes. It is rich in beta-
eyes frequently, suggested Dr Pillai. "After about carotene and vitamin, which protects the eye
every few minutes, voluntarily, close your eyes against infections," Jyoti Khaniojh said. Besides
tightly and then open them. Watch your eyes this, add lemon and citrus fruits to your diet for
more often as it will help lubricate the eyeballs," vitamin C intake. Nuts and seeds that contain
he said. We can say that option D is the correct vitamin E prevent cataracts and age-related
answer. macular degeneration. Salmon fish is a rich
source of omega-3 and this healthy fat helps in
5. Answer: D prevent eye dryness as it is great for retina
Here, as per the lines, Another major problem in health. Egg yolks contain vitamin A, zinc, lutein
summer is ocular diseases like conjunctivitis, and zeaxanthin and zinc, which help in age-
keratitis, endophthalmitis, cellulitis and stye, related degeneration of the eyes. Zinc helps in
among others. Dr Pillai said that conjunctivitis is the night improvement of night vision. We can
one of the most common eye problems in the hot say that option D is the correct answer.
months. We can say that option D is the correct
answer. 8. Answer: E
Here, in this question, there is no error. So,
6. Answer: D option E is the correct answer.
Here, as per the lines, Conjunctivitis spreads via
human touch. Someone could use an infected 9. Answer: C
towel or even a door knob that has been touched Here, the error lies in part c as the use of he is
by an infected person. In a classroom, one child inappropriate as here in this question there is no
could infect another child if they have gender-specific thing so he or she cannot be
conjunctivitis. It's a viral infection, so it spreads

Click Here For Bundle PDF Course | support@guidely.in Page 7 of 9


Bank Po Mains PDF Course 2024
English Day - 15

used as per the context, instead it should have Here, ultimately should have been used in place
been used. So, option C is the correct answer. of penultimately as ultimately means in the end,
and here ultimately fits best whereas
10. Answer: C penultimately means next to the last which is
Here, the error lies in part c as the use of for is inappropriate as per the context of the sentence.
inappropriate before Air Costa as of should have So, option C is the correct answer.
been used instead of for. Of shows possession
and here license was possessed by the 15. Answer: C
company so it should have been used. So, Here, apprehensive should have been used here
option C is the correct answer. in place of comprehensive as comprehensive
means including everything or nearly everything
11. Answer: B that is connected to a particular subject whereas
Here, the error lies in part b as the use of begun apprehensive means worried or afraid that
is inappropriate, and began should have been something bad may happen and here failure of
used here because the sentence is in past airlines is the main focus of the sentence. So,
simple tense and here the second form of the option C is the correct answer.
verb should have been used. So, option B is the
correct answer. 16. Answer: B
Here, prospects should have been used in place
12. Answer: C of respects which is contextless here as per the
Here, the error lies in part c as the use of an context of the sentence. Prospects mean a
before European is inappropriate as an is used possibility that something might happen whereas
before words which begins with a vowel sound, respect means polite behavior or care towards
and European sounds like European and somebody and here former fits as per the
therefore an cannot be used before it, instead a context of the sentence. So, option B is the
should have been used. So, option C is the correct answer.
correct answer.
17. Answer: D
13. Answer: E Here, which should be replaced with who as
Here, none of the highlighted words are here economists word is used which is a living
inappropriately used. So, option E is the correct being and for them who should have been used
answer. there and add should have been replaced with
14. Answer: C aid as add means to put something together and

Click Here For Bundle PDF Course | support@guidely.in Page 8 of 9


Bank Po Mains PDF Course 2024
English Day - 15

aid means help and here latter fits best. So, Here, have should be replaced with has been as
option D is the correct answer. the sentence is in present perfect continuous
tense and Apple is a singular subject and
18. Answer: D therefore we will use a singular helping verb
Here, towards should have been replaced with which is has been. So, option C is the correct
away from as land has been bough in India answer.
which means the company has bought land
away from China and therefore option D fits 20. Answer: E
best. So, option D is the correct answer. Here, there is no need for any improvement. So,
option E is the correct answer.
19. Answer: C

Click Here For Bundle PDF Course | support@guidely.in Page 9 of 9


Bank Po Mains PDF Course 2024
Reasoning Day -16 (Eng)

Reasoning Aptitude
Directions (1-5): Study the following information a) T
carefully and answer the given questions. b) The one who joined on June 7th
Nine persons – O, P, Q, R, S, T, U, V, and, W c) The one who joined two persons after S
joined the bank in three different months viz.- d) O
March, June, and August on three different dates e) V
– 4th, 7th, and 9th. Only one person joined on one
date of each month. All persons like different 2) Who among the following person joined on 7th
shoe brands viz.- Mochi, Puma, Reebok, Bata, August?
Relaxo, Nike, Red Chief, Liberty, and Adidas. a) The one who likes Bata
The one who likes Bata joined on an even b) The one who joined two persons after U
numbered date in a month having an odd c) The one who likes Puma
number of days. Only three persons joined d) V
between the one who likes Bata and W, who e) P
joined immediately after the one who likes Nike.
R joined four persons before the one who likes 3) Four of the following five are alike in a certain
Red Chief. At least one but not more than three way based on the given arrangement and thus
persons joined between the one who likes Bata form a group. Which one of the following does
and R. Both R and O like neither Nike nor not belong to the group?
Adidas. S joined three persons before the one a) T
who likes Adidas. Both O and S like neither Bata b) Q
nor Nike. As many persons joined before the one c) W
who likes Adidas as after O. As many persons d) R
joined between the one who likes Red Chief and e) S
O as between P and the one who likes Relaxo.
The one who likes Relaxo joined immediately 4) How many persons joined between U and the
after U, who joined on an even numbered date. P one who likes Reebok?
and the one who likes Mochi joined in the same a) As many persons joined before the one who
month. Only three persons joined between T and likes Nike
the one who likes Mochi. More than two persons b) Three
joined between the one who likes Liberty and Q, c) As many persons joined between V and the
who likes neither Red Chief nor Reebok. one who likes Reebok
1) Who among the following person likes d) Four
Relaxo? e) None

Click Here For Bundle PDF Course | support@guidely.in Page 1 of 10


Bank Po Mains PDF Course 2024
Reasoning Day -16 (Eng)

digital sum of the outcome from top to bottom


5) Which of the following statements is/are not respectively.
true as per the given arrangement? Arrangement 1:
a) T joined in August D, I, R, Z, T, E, O, F, K, U, G, P, and V are to be
b) The one who likes Puma joined immediately filled against the outcomes. Only one letter is
after V placed along with each outcome.
c) Three persons joined before U I. All vowels are placed in reverse alphabetical
d) Either B or C order along with each outcome whose digital
e) All the above statements are true sum is a prime number from top to bottom
respectively.

Directions (6-10): Study the following information II. All consonants which come before M are

carefully and answer the given questions. placed in reverse alphabetical order along with

There are nine levels marked one to nine from each outcome whose digital sum is an even

bottom to top respectively. There are thirteen number from top to bottom respectively.

outcomes which are placed in these nine levels. III. All the remaining letters are placed with the

At least one and at most two outcomes are remaining outcomes from bottom to top in

placed on each level. No two levels have the alphabetical order. (If more than two outcomes

same outcome. Two dice are thrown and their are at the same level, then the digital sum of the

outcomes are given below: outcome is highest given first preference)

(6, 4), (6, 3), (4, 5), (1, 2), (1, 5), (1, 1), (3, 2), (5, Arrangement 2:

3), (3, 3), (5, 2), (4, 4), (6, 6), and (5, 5). All the letters are placed in a row from the left

Condition: end based on the descending order of their

I) If the digital product of the outcome is digital sum of the outcome and facing north. (If

even(more than 16), then the respective the outcome of letters are same, then give

outcomes are arranged in descending order and preference in alphabetical order).

placed at an even numbered level from top to 6) Which of the following letter is placed second

bottom respectively. to the right of the letter which is fifth to the left of

II) If the digital product of the outcome is odd, G in arrangement 2?

then the respective outcomes are arranged in a) Z

ascending order and placed at an odd numbered b) The letter which is placed immediate left of E

level from top to bottom respectively. c) The letter which is placed fourth from the left

III) The remaining outcomes are placed on an end

even numbered level in ascending order of the d) O


e) F

Click Here For Bundle PDF Course | support@guidely.in Page 2 of 10


Bank Po Mains PDF Course 2024
Reasoning Day -16 (Eng)

a) U, 8, K
7) ______ is the outcome of the letter which is b) F, 6, D
_______ to the right of V in arrangement 2. c) G, 7, I
a) (1, 1), Fourth d) Both B and C
b) (4, 5), Second e) I, 7, O
c) (1, 5), Third
d) (6, 4), Fifth Directions (11-15): Study the following
e) None of these information carefully and answer the given
questions.
8) Which of the following letters are placed at Sixteen persons from A to P are sitting around
even numbered levels? the two concentric square tables in such a way
I. TRD that eight persons are sitting at the inner table
II. RUF facing away from the centre whereas eight
III. FRT persons are sitting at the outer table facing
IV. OTK towards the centre. One person sits at each
a) Only III and IV corner whereas one person sits in the middle of
b) Only III each side of both tables.
c) Only I, III, and IV Note: I. Consecutive alphabetically named
d) Only I, II and IV persons are neither sitting at the same table nor
e) None facing each other.
II. If A is facing B, then both are facing each
9) Four of the following five are alike in a certain other at different tables, if A sits opposite to B,
way based on the given arrangement 2 and thus then both are sitting at the same table.
form a group. Which one of the following does H sits third to the right of the one who is facing C,
not belong to the group? who sits at the corner of the inner table. One
a) The letter which is placed third to the right of G person sits between C and E, who sits opposite
b) F to G. The one who is facing G sits third to the left
c) The letter which is placed second to the right of N. Only three persons sit between F and D,
of P who sits adjacent to H. As many persons sit
d) E between F and J as between K and M, when
e) O counted from the left of both F and K. K sits
opposite to O, who neither sits adjacent to C nor
10) _____ letter is placed at level _____ and E. Both M and I don’t sit adjacent to E. Only
immediately above ______. three persons sit between A and I. The number

Click Here For Bundle PDF Course | support@guidely.in Page 3 of 10


Bank Po Mains PDF Course 2024
Reasoning Day -16 (Eng)

of persons sitting between I and O is one less a) N


than the number of persons sitting between B b) The one who sits third to the left of D
and J, when counted from the right of both B and c) The one who faces I
I. Two persons sit between B and P. d) Both A and B
11) What is the position of C with respect to the e) Either B or C
one who faces J?
a) Immediate right 15) Who among the following persons are sitting
b) Second to the left between the one who faces I and B, when
c) Third to the left counted from the left of B?
d) Second to the right I. O
e) Immediate left II. The one who sits second to the left of L
III. The one who faces C
12) How many persons are sitting between A a) Only II
and the one who faces N, when counted from the b) Only I and III
right of A? c) Only III
a) Two d) All I, II, and III
b) One e) Only II and III
c) Three
d) More than three Directions (16-20): Study the following
e) None information carefully and answer the given
questions.
13) Four of the following five are alike in a certain In a certain code language following statements
way based on the given arrangement and thus are coded as follows:
form a group. Which one of the following does “Burn Are More Empty Close” is coded as “#%+
not belong to the group? @%¥ #$¥ @^* @^¥”.
a) L “Went All File Above Equal” is coded as “@^¥
b) The one who sits second to the right of A @^© #$£ #%£ #%¥”.
c) N “Blue Copy Age Down Today” is coded as “@%+
d) The one who sits third to the left of B #$¥ @^+ @^* @^¥”.
e) O Note: (All the given codes are three symbols
codes only)
14) If K is related to F, similarly D is related to O, 16) How “went” is coded in the given code
then who among the following person is related language?
to E? a)#$¥

Click Here For Bundle PDF Course | support@guidely.in Page 4 of 10


Bank Po Mains PDF Course 2024
Reasoning Day -16 (Eng)

b) @^© d) Either A or B
c)@^¥ e) Either B or C
d)@%¥
e) @^* 19) What may be the code of “Run Today
Away”?
17) Which of the following phrase is coded as a) @%* @%+ #^+
“#$¥ #%£”? b)@$* #%* #^+
a) Age Equal c)@$* @%+ #^+
b) Down Empty d) Either A or B
c) Are Equal e) None of these
d) Both A and C
e) None of these 20) What is the code of “Empty File”?
a) #*^ @^£
18) How “More Copy” is coded in the given code b) #%+ @^¥
language? c) #*^ @^+
a) @^¥ @^+ d) @%^ #^¥
b) @^* #$¥ e) None of these
c) @^¥ #$¥
Click Here to Get the Detailed Video Solution for the above given Questions
Or Scan the QR Code to Get the Detailed Video Solutions

Answer Key with Explanation


Direction (1-5): 4. Answer: A
1. Answer: B 5. Answer: B
2. Answer: C
3. Answer: D(All the persons joined in the month
having 31 days except option D)

Click Here For Bundle PDF Course | support@guidely.in Page 5 of 10


Bank Po Mains PDF Course 2024
Reasoning Day -16 (Eng)

Again, we have:
 R joined four persons before the one who
likes Red Chief.
 At least one and not more than three
persons joined between the one who likes
Bata and R.
 Both R and O like neither Nike nor
Adidas.
That means, in case (1) R joined on 9
March, in case (2) R joined on 7 June, in
case (2a) R joined on 4 June.
Based on the above given information we have:

We have:
 The one who likes Bata joined on an even
numbered date in a month having an odd
number of days.
 Only three persons joined between the
one who likes Bata and W, who joined
immediately after the one who likes Nike.
That means, in case (1) the one who likes
Bata joined on 4 March, in case (2) the
one who likes Bata joined on 4 August. Again, we have:
Based on the above given information we have:  S joined three persons before the one
who likes Adidas.
 Both O and S like neither Bata nor Nike.
 As many persons joined before the one
who likes Adidas as after O.
Since, R doesn’t like Adidas.
That means, in case (2) S joined on 4
March, in case (2a) R joined on 9 June,
case (1) is not valid.
Based on the above given information we have:

Click Here For Bundle PDF Course | support@guidely.in Page 6 of 10


Bank Po Mains PDF Course 2024
Reasoning Day -16 (Eng)

Case (1) is not valid as O doesn’t like Bata.


Again, we have:
 As many persons joined between the one
who likes Red Chief and O as between P
and the one who likes Relaxo. Case (2a) is not valid as the one who likes
 The one who likes Relaxo joined Relaxo joined immediate after U.
immediately after U, who joined on an Again, we have:
even numbered date.  More than two persons joined between
That means, in case (2) U joined on 4 the one who likes Liberty and Q, who likes
June, case (2a) is not valid. neither Red Chief nor Reebok.
 P and the one who likes Mochi joined in Thus, S likes Liberty and O likes Reebok.
the same month. Based on the above given information we have:
 Only three persons joined between T and
the one who likes Mochi.
That means, W likes Mochi and T likes
Bata.
Based on the above given information we have:

Click Here For Bundle PDF Course | support@guidely.in Page 7 of 10


Bank Po Mains PDF Course 2024
Reasoning Day -16 (Eng)

Direction (6-10):
6. Answer: B
7. Answer: C
8. Answer: A
9. Answer: A(All the letters are placed in the
middle of the row except option A)
10. Answer: D
Arrangement 1:

Arrangement 1:
D, I, R, Z, T, E, O, F, K, U, G, P, and V are to be
filled against the outcomes. Only one letter is
placed along with each outcome.
I. All vowels are placed in reverse alphabetical
order along with each outcome whose digital
sum is a prime number from top to bottom
respectively.
Thus, U, O, I, and E are placed.
Arrangement 2: II. All consonants which come before M are
placed in reverse alphabetical order along with
each outcome whose digital sum is an even
number from top to bottom respectively.
We have:
Thus, K, G, F, and D are placed.
Outcome: (6, 4), (6, 3), (4, 5), (1, 2), (1, 5), (1, 1),
III. All the remaining letters are placed with the
(3, 2), (5, 3), (3, 3), (5, 2), (4, 4), (6, 6), and (5,
remaining outcomes from bottom to top in
5).
alphabetical order. (If more than two outcomes
Even digital products: 36, 24, 20, 18,
are at the same level then the digital sum of the
Odd Digital products: 1, 5, 9, 15, 25
outcome is highest given first preference)
Digital sum: 3, 5, 7, 8
Thus, the remaining letters: P, R, T, V, and Z are
placed
Based on the above conditions we have:

Click Here For Bundle PDF Course | support@guidely.in Page 8 of 10


Bank Po Mains PDF Course 2024
Reasoning Day -16 (Eng)

We have:
 H sits third to the right of the one who is
facing C, who sits at the corner of the
inner table.
Arrangement 2:
 One person sits between C and E, who
All the letters are placed in a row from the left
sits opposite to G.
end based on the descending order of their
That means, in case (1) E sits second to
digital sum of the outcome and facing north. (If
the left of C, in case (2) E sits second to
the outcome of letters are same, then give
the right of C.
preference in alphabetical order).
 The one who is facing G sits third to the
Based on the above given condition we have:
left of N.
Based on the above given information we have:

Direction (11-15):
11. Answer: C
12. Answer: B
13. Answer: A(All the persons are sitting in the
middle of the table except option a) Again, we have:

14. Answer: D  Only three persons sit between F and D,

15. Answer: C who sits adjacent to H.


Since, the persons in alphabetical order
are neither sitting at the same table nor
sits facing each other.
That means, in case (1) & case (2) D sits
immediate right of H.

Click Here For Bundle PDF Course | support@guidely.in Page 9 of 10


Bank Po Mains PDF Course 2024
Reasoning Day -16 (Eng)

 As many persons sit between F and J as Case (1) & case (2) are not valid as no place
between K and M, when counted from the available for I and A.
left of both F and K. Again, we have:
 K sits opposite to O, who neither sits  Two persons sit between B and P.
adjacent to C nor E. That means, P sits immediate left of F.
 Both M and I don’t sit adjacent to E. Based on the above given information we have:
Since, we have to consider the number of
persons between F and J, thus J and K
can’t sit at the same table.
That means, in case (1a) M is facing D, in
case (1) M sits immediate right of C, in
case (2) M sits immediate left of C.
Based on the above given information we have:

Direction (16-20):
16. Answer: B
17. Answer: D
18. Answer: A
Again, we have:
19. Answer: C
 Only three persons sit between A and I.
20. Answer: B
 The number of persons sitting between I
We have:
and O is one less than the number of
For the first symbol: If the word starts with a
persons sitting between B and J, when consonant then it is coded as ‘@’, else it is
counted from the right of both B and I.
coded as ‘#’.
That means, in case (1a) I sits immediate
For the second symbol: the second symbol
right of C, case (1) & case (2) are not
represents the number of letters in the word.
valid.
For 3  $, 4  ^, and 5  %
Based on the above given information we have:
For the third symbol: the third symbol represents
the last letter of the word.
Y  +, N  *, E  ¥, and L  £

Click Here For Bundle PDF Course | support@guidely.in Page 10 of 10


Bank Po Mains PDF Course 2024
Quantitative Aptitude Day -16 (Eng)

Quantitative Aptitude

Directions (01 - 04): Study the following information carefully and answer the questions given below.
There are five showrooms [A, B, C, D, and E]. In each showroom different numbers of diesel, petrol, CNG
and electric cars sold. The given charts show the difference between the number of diesel and electric,
diesel and CNG, petrol and electric, petrol and electric, and CNG and electric cars sold in showrooms A,
B, C, D, and E respectively. The number of diesel cars sold in each shop is the maximum and the
number of electric cars sold in each shop is the minimum.

NOTE – I. The total number of cars sold in A is 420. The number of CNG cars sold in A is double the
number of electric cars sold in A. The number of petrol cars sold in A is 50% more than the number of
CNG cars sold in A.
II. The number of Petrol cars sold in A is 20% more than the number of petrol cars sold in B. The number
of diesel cars sold in B is 22.22% less than the number of diesel cars sold in A. The ratio of the number of
CNG and Electric cars sold in B is 7:5.
III. The total number of petrol and electric cars sold in C is 200. The number of CNG cars sold in C is 3/2
of the number of electric cars sold in C. The number of Diesel cars sold in C is 166.66% more than the
number of electric cars sold in C.

Click Here For Bundle PDF Course | support@guidely.in Page 1 of 13


Bank Po Mains PDF Course 2024
Quantitative Aptitude Day -16 (Eng)

IV. Difference between the number of diesel and CNG cars sold in D is 120, the sum of CNG and electric
cars sold in D is 120. The ratio of petrol and diesel cars sold in D is 4:5 respectively.
1) The number of diesel cars sold in E is 10% 3) Difference between the number of petrol cars
less than the same in D. The sum of Diesel and and CNG cars sold in A is _________ % of the
CNG cars sold in E is 240 and the difference difference between the number of petrol cars and
between the number of diesel and petrol cars electric cars sold in D and the Difference
sold in E is 60. The ratio of the Top model and between the number of petrol cars and CNG cars
the base model of all type's cars sold in E is 5:3. sold in B is ________ % of the difference between
The average price of the Top model car and the number of petrol cars and electric cars sold
Base model car sold in E is 15 lakh and 12 lakhs in C.
respectively. Find the total revenue (in lakhs) Find which of the following satisfies the blank?
earn by E? a) 37.5%, 33.33%
a) 2540 b) 33.33%, 37.5%
b) 8520 c) 62.5%, 37.5%
c) 5550 d) 66.66%, 33.33%
d) 6500 e) None of these
e) 7500
4) The number of XUV sold in C and D is more
2) Find which of the following is/are true? than the number of sedan cars sold in C and D
I. k is the difference between the number of out of the total number of diesel and petrol cars
diesel and petrol cars sold in C and l is the sold in C and D respectively. The difference
difference between the number of CNG and between the number of XUV and sedan petrol
Electric cars sold in C. So k2+k*l+l2=2900. cars and diesel cars out of total diesel and petrol
II. Sum of electric cars sold in A, B and C cars sold in C is 40 respectively and Difference
together is 150. between the number of XUV and sedan petrol
III. Difference between the total number of cars cars and diesel cars out of total diesel and petrol
sold in C and D is 40. cars sold in D is 40 respectively. The number of
a) all false XUVs sold in C and D together is what percent of
b) all true the number of sedan cars sold in C and D
c) only II and III together?
d) only II a) 164%
e) only III b) 154%
c) 157%
d) 132%

Click Here For Bundle PDF Course | support@guidely.in Page 2 of 13


Bank Po Mains PDF Course 2024
Quantitative Aptitude Day -16 (Eng)

e) None of these less than the number of students who liked both
math and physics but not chemistry. The number
5) A shopkeeper marked up an article by _______ of students who like only physics and math but
% above the cost price and give discount of not chemistry is 15. The number of students who
______% and make profit of 20%. The cost price like only physics is 30. The number of students
of the article is ________. who passed only Math is three times the number
Find which of the following satisfies the blank? of students who like both math and physics but
I.60%, 25% ,240 not chemistry. The number of students who like
II.50%, 20%,360 both chemistry and math but not physics is three
III. 75%, 15%, 420 times the number of students who passed all
a) Only I and II three exams, which is equal to 6 and the number
b) Only II and III of students like all three subject is 5. The number
c) only I and III of students who passed both math and chemistry
d) both I and II but not physics is 7. The ratio of the number of
e) None of these students like only math to the number of students
passing only Math, the number of students who
Directions (06 - 09): Study the following like only Math to the number of students who
information carefully and answer the questions pass only chemistry, the number of students who
given below. like only chemistry and the number of students
There is a survey taken among the students. who passed only chemistry is 7:9, 7:5 and 8:5
Each student likes at least one subject respectively.
chemistry, physics, and math. Each student 6) Ratio of the number of male and female
passed at least one subject. The number of students who like only chemistry is 3:5
students who passed both physics and chemistry respectively and the ratio of the number of male
but not math is 33.33% less than the number of and female students who passed only chemistry
students who like both chemistry and physics but is 3:2 respectively . Find the difference between
not Math. The number of students who passed the number of female students who like only
only physics is 37.5% more than the number of chemistry and the number of female students
students who like only chemistry. The number of who passed only chemistry?
students who like both chemistry and math but a) 10
not physics is 50% more than the number of b) 19
students who like both chemistry and physics but c) 15
not math. The number of students who passed d) 13
both physics and math but not chemistry is 40% e) 11

Click Here For Bundle PDF Course | support@guidely.in Page 3 of 13


Bank Po Mains PDF Course 2024
Quantitative Aptitude Day -16 (Eng)

7) Quantity: I The total number of students who 9) x is the percentage of the number of students
like physics is what percent of the number of who like all three subjects out of the total number
students who passed in Physics? of students who passed all three subjects and y
Quantity: II The total number of students who like is the percentage of the number of students who
math is what percent of the number of students passed only physics out of the number of
who passed in Math? students who passed only math. Find the value
A. Quantity: I < Quantity: II of 2x+3y?
B. Quantity: I ≥ Quantity: II a) 533.32
C. Quantity: II ≥ Quantity: I b) 543.2
D. Quantity: I > Quantity: II c) 582.33
E. Quantity I = Quantity II or relation can't be d) 592.37
established e) None of these

8) Difference between the number of students 10) A can complete a work alone in 40 days and
who like both chemistry and physics but not math B can complete the same work alone in 30 days.
and the number of students who passed both When A work alone but takes a break one day
math and physics but not chemistry is _______ after every 5 days of work, then the total work
and the difference between the number of completed in n days. When B works alone but
students who like both chemistry and math but takes a break one day after every 5 days of
not physics and the number of students who work, total work is completed in m days. D
passed both chemistry and physics but not math completes the work in m+25 days and C
is _______. ;Which is 1/3rd of the Multiply of completes n-23 days. Find the time taken by C
values of two blanks is ______. and D together to complete the work?
Find the possible value to satisfy the blank? a) 120/7
a) 12, 10 , 40 b) 145/7
b) 3, 10, 12 c) 120/9
c) 12, 4, 16 d) 120/11
d) 10, 3, 10 e) None of these
e) None of these

Directions (11 - 14): Study the following information carefully and answer the questions given below.
The given pie chart shows the percentage distribution of the speed of five different boats [A, B, C, D and
E] in still water.

Click Here For Bundle PDF Course | support@guidely.in Page 4 of 13


Bank Po Mains PDF Course 2024
Quantitative Aptitude Day -16 (Eng)

The given pie charts show the percentage distribution of the speed of streams of five different rivers [R,
S, T, U and V].

Note – The speed of boat B in still water is 18 km/hr and the downstream speed of boat D in river V is 57
km/hr.
11) Boat A covers d km distance downstream in the following given boats in upstream in river U
river R and then the same distance upstream in can cover a distance 2d km within n hours?
n hours. The same boat covers d+60 km I. Boat D
downstream in river T in 5 hours. Find which of II. Boat E
III. Boat A

Click Here For Bundle PDF Course | support@guidely.in Page 5 of 13


Bank Po Mains PDF Course 2024
Quantitative Aptitude Day -16 (Eng)

a) only I and II e) None of these


b) both I and III
c) only I 14) Find which of the following is/are true?
d) all three I. Boat B covers 48 km downstream in river U
e) none and 64 km downstream in river S in 3.32 hours.
II. Boat C covers 88 km downstream in river U
12) River S and River T meet at point M and and 54 km downstream in river T in 4.3 hours.
move towards N WHICH IS considered as the III. Boat E covers 84 km downstream in river U
name of river U. O and P are two points of river and 104 km downstream in river U in 3.41 hours.
S and T respectively. Distance from M to O and a) Both I and III
P is 70 km and 80 km respectively. The distance b) Both I and II
from M to N is 75 km. Boat C and D start the c) All three
same journey towards M from O and P d) only I
respectively and then reached N. Find the e) None of these
difference in time taken by boats C and D to
reach N from their starting point? 15) Jar A contains _______ litres of mixture of
[ All the distance covered downstream] milk and water, the percentage of Milk is 60%.
a) 1.78 Jar B contains _____ litres of a mixture of milk
b) 2.23 and water, in a ratio of 5:3. 40% mixture of
c) 2.01 mixture A and 60% of mixture B is mixed in C.
d) 1.42 Difference of milk and water in C is ______.
e) 1.98 Find which of the following satisfies the blank?
a) 150, 120, 22.6
13) In river T, boat D goes a certain distance b) 100, 96 , 12.6
downstream and then returns and covers some c) 124, 88, 17.6
part of the distance in upstream in total 9 hours. d) 120, 80 , 21.6
The same distance that boat D covers in e) None of these
upstream, boat B can cover in 17.5 hours in
upstream in river V. Find the time taken by boat Directions (16 - 19): Study the following
D to return to the starting point? information carefully and answer the questions
a) 16.42 hours given below.
b) 17.14 hours 16) Find the profit share of B in business 1? [ A
c) 17.35 hours and B invest in each business at different times]
d) 21.33 hours

Click Here For Bundle PDF Course | support@guidely.in Page 6 of 13


Bank Po Mains PDF Course 2024
Quantitative Aptitude Day -16 (Eng)

Statement I: The profit share of A in Business 2 Statement I: Sum of the age of A and B after 5
is 20% more than the profit share of A in years is 85 years. The ratio of age of C and E is
Business 1. 5:3. Sum of the age of C and D is 54 years.
Statement II: Total share of profit in business 1 is Statement II: The sum of age of A and E is 58
6800. B invest 10 months in each business. years and the average age of A and D is 32
Statement III: B invests Rs.14000 in business2. years.
The total profit share of A in business1 and 2 is Statement III: D is 6 years older than E. Ratio of
Rs.5500. age between C and D is 5:4. Difference in age
a) All III together necessary between B and C is 5 years.
b) only I a) only I
c) only I and III b) all three are necessary to answer
d) I and II together c) only III
e) only III d) only I and II
e) None of these
17) Find the volume of the cuboid? [ take pi = 3]
Statement I: The length of the cuboid is equal to 19) Find the compound interest after 2 years?
the height of the cylinder whose volume is Statement I: The rate of interest is 30%.
25137. The curved surface area of the cylinder is Statement II: If the same principle invests at
2394. simple interest for 3 years at the same interest
Statement II: The breadth of the cuboid is equal rate, then the interest amount is Rs.10800.
to the radius of the cone whose volume is 486. Statement III: For the same principle invests the
The breadth of the cuboid is 4 more than the amount of compound interest at R% for 2 years
radius of the sphere. The ratio of the length to is Rs.5280.
the breadth of the cuboid is 3:2 a) both I and II
Statement III: The height of the cuboid is equal to b) Only II
the radius of the sphere whose volume is 10976 c) only I and III
m. d) only III
a) only II e) None of these
b) only I
c) only I and III 20) Length of train A is ________ m and crosses
d) II and III together a platform in ________ sec. The length of train B
e) only III is double the length of the platform and the
speeds of train A and B is 90 km/hr and 72 km
18) Find the sum of the age of B and C?

Click Here For Bundle PDF Course | support@guidely.in Page 7 of 13


Bank Po Mains PDF Course 2024
Quantitative Aptitude Day -16 (Eng)

per hour respectively. Train B crosses the b) 480, 14, 40


platform in ________ sec. c) 240, 16, 36
Find which of the following satisfies the blank? d) 320, 18, 48
a) 320, 24, 42 e) None of these
Click Here to Get the Detailed Video Solution for the above given Questions
Or Scan the QR Code to Get the Detailed Video Solutions

Answer Key with Explanation


Directions (01 - 04): The number of petrol cars sold in A is
Let the number of petrol and diesel cars sold in 2x*150/100 = 3x.
D be 4y and 5y respectively. Or, x+x+140+3x+2x=420
The number of CNG cars sold in D is 5Y-120. Or, 7x = 420-140 = 280, and x = 40.
The number of Electric cars sold in D is 120- So, the number of electric cars sold in A is 40,
5y+120=240-5y. and the number of CNG cars sold in A is 2* 40 =
The number of electric cars sold in D is 4Y-120. 80.
So, 240-5y=4y-120 The number of diesel cars sold in A is x + 140 =
Or, 9y = 360, y = 40. 40 + 140 = 180.
The number of petrol and diesel cars sold in D is The number of petrol cars sold in A = 3x = 3*40
4* 40 = 160 and 5* 40 = 200, respectively. = 120.
The number of CNG cars sold in D is 5y-120 = The number of petrol cars sold in B is 120 *
200-120 = 80. 100/120 = 100.
The number of electric cars sold in D is 4y- The number of diesel cars sold in B is 180 *
120=160-120=40. [100-22.22]/100 = 180* [100-22.22]/100 = 140.
Let the number of electric cars sold in A be x and The Number of CNG cars sold in B is 140-
the number of CNG cars sold in A be 2x. 70=70.
The number of diesel cars sold in A is x+140.

Click Here For Bundle PDF Course | support@guidely.in Page 8 of 13


Bank Po Mains PDF Course 2024
Quantitative Aptitude Day -16 (Eng)

The number of electric cars sold in B is 70*5/7 = So, II is true.


50. III. Difference between the total cars sold in C
The number of petrol cars sold in C = and D is
[200+80]/2 = 140. = [200+160+80+40] – [160+140+90+60]=30.
The Number of electric cars sold in C = 140- So, III is false
80=60
The Number of CNG cars sold in C is 60*3/2 = 3) Answer: B
90. Value for first blank = [40/120] *100=33.33%
The Number of Diesel cars sold in C is 60 * Value of for second blank = [30/80]*100=37.5%
266.66 / 100=160
1) Answer: C 4) Answer: A
The Number of diesel cars sold in E is Number of XUV sold in C = [160+40]/2 +
200*90/100=180 [140+40]/2 =100+90=190
The Number of CNG cars sold in E is = 240- Number of XUV sold in D = [200+40]/2 +
180=60 [160+40]/2 = 120+100=220
The Number of Petrol cars sold in E is = 180- Total number of sedan cars sold in C and D =
60=120 [300-190]+ [360-220] =110+140=250
The Number of electric cars sold in E is = 60- So, required percentage = [410/250] *100=164%
20=40
Total cars sold in E is = 180+120+60+40=400 5) Answer: A
So, total revenue is For I, the cost price is 240, marked price =
= [400*5/8]*15 + [ 240*160/100=384
400*3/8]*12=250*15+150*12=5550 lakh Selling price = 384*75/100=288
So, profit percentage = [288-240]*100/240=20%
2) Answer: D So, it's satisfying.
k is the difference between the number of diesel Similarly, we can check others' values also,
and petrol sold in C, so k= 160-140=20, Only I and II satisfy.
l is the difference between the number of CNG
and Electric cars sold in C, so I= 90-60=30 Directions (06 - 09):
So, 202+20*30+302=1900. The number of students who passed in all three
So, I is false subjects is 6.
II. Sum of the number of electric cars sold in A, B Number of students who like all three subjects is
and C together is =40+50+60= 150. 5.

Click Here For Bundle PDF Course | support@guidely.in Page 9 of 13


Bank Po Mains PDF Course 2024
Quantitative Aptitude Day -16 (Eng)

Number of students who like only physics and


math but not chemistry is 15.
Number of students who passed only math =
15*3 = 45.
The Number of students who like only math is
45*7/9 = 35.
Number of students who passed only chemistry
= 35*5/7 = 25
Number of students who like only chemistry =
Venn diagram of the number of students who
25*8/5 = 40
like the subjects.
Number of students who passed only physics =
40* 137.5/100 = 55
Number of students who like only physics = 30.
Number of students who passed both math and
physics but not chemistry is = 15*60/100=9
Number of students who like both chemistry and
Math but not physics is 6* 3 = 18.
Number of students who like both physics and
chemistry but not math
=18*100/150=12
Number of students who passed both chemistry
and math but not physics is 7. 6) Answer: C
The number of students who passed both Required difference = 40*5/8 – 25 *2/5= 25-
chemistry and physics but not math is = 10=15
12*66.66/100=8
VENN DIAGRAM of the number of students who 7) Answer: A
passed Quantity I
Required percentage =
[(30+12+15+5)/(8+55+6+9)]*100=79.48%
Quantity II
Required percentage =
[(35+18+15+5)/(7+45+6+9)]*100=108.95
Quantity I < Quantity II

Click Here For Bundle PDF Course | support@guidely.in Page 10 of 13


Bank Po Mains PDF Course 2024
Quantitative Aptitude Day -16 (Eng)

Directions (11 - 14):


8) Answer: E The speed of the boat B in still water is 18 km/hr.
Possible value for 1st blank is = [12-9]=3 So, total value of 1st pie chart is =
Possible value for 2nd blank is =[18-8]=10 [18/12]*100=150
So, possible value for 3rd blank is = [10*3]/3=10 So, the speed of boat D is = 150*30/100=45
So, the speed of a stream of river V is 57-45=12
9) Answer: A km/hr
So, x =[5/6]*100=83.33% So, total value of 2nd pie chart is =
So, y=[55/45]*100=122.22% [12/15]*100=80
So, 2*83.33 + 122.22*3=533.32

10) Answer: A
A completed in 40 days and B was completed in
30 days.
So, total work = LCM of 40 and 30 =120
11) Answer: E
A’s one days work is = 120/40=3 and B’s one
So, we can say, d/39 + d/ 15 = n
days work is = 120/30=4
2nd case,
So, when A takes a break after 5 days, means in
[d+60]/[27+24] = 5
6 days A completes 5*3=15 units of work, so,
Or, d=255-60=195
total work will complete
So, 195/39 +195 /15 = 5 +13 =18 =n
= [6*8-1]=47 days
Time taken by boat D to cover 2d distance in
So, when B takes a break after 5 days, means in
upstream in river U is = [2*195/(45-17.6]=14.23
6 days B completes 5*4=20 units of work, so, the
Time taken by boat E to cover 2d distance in
total work will complete
upstream in river U is = [2*195/(37.5-17.6]=19.59
= [6*6-1]=35 days
Time taken by boat A to cover 2d distance in
So, D takes 35+25=60 days to complete the
upstream in river U is = [2*195/(27-17.6]=41.49
work and C takes 47-23=24 days to complete
So, none of the given boats can cover the
the work.
distance in n hour.
The efficiency of C and D is 120/24=5 and
120/60=2 respectively.
12) Answer: D
So, C and D together can complete the work in =
Time taken by boat C = [70/[22.5+14.4]]
120/7 days
+[75/[22.5+17.6]
=1.89+1.87=3.76

Click Here For Bundle PDF Course | support@guidely.in Page 11 of 13


Bank Po Mains PDF Course 2024
Quantitative Aptitude Day -16 (Eng)

Time taken by boat D = [80/[45+24]] So, Quantity of Milk in C =72*40/100


+[75/[45+17.6] +50*60/100=58.8
=1.15+1.19=2.34 Quantity of Water in C = 48*40/100 +
So, difference = 1.42 30*60/100=37.2
So, the difference = 58.8-37.2=21.6
13) Answer: B
Distance that boat D covers in Upstream is = 16) Answer: A
17.5*[18-12] =105 From statement I, if the profit amount of A in
So, 105 km distance covered by boat D in business 1 is 5x then the profit share of business
upstream in river T in =105/[45-24]=5 hours 2 of A is 6x.
So, in 4 hours downstream boat cover = From statement II, the total profit share in
[45+24]*4=276 business 1 is 6800.
So, the total time taken by boat D to return to the From III, the total profit share of A is 5500
starting point From I and III, the profit share of A in business 1
=4+276/[45-24]=17.14 hours is = 5500*5/11=2500
From III, we can say the profit share of B in
14) Answer: A business 1 is 6800-2500=4300
I. Boat B covers 48 km downstream in river U All three statements together are necessary.
and 64 km downstream in river S in
=[48/(18+17.6)] +[64/(18+14.4)]= 3.32 hours. 17) Answer: D
II. Boat C covers 88 km downstream in river U From a single statement, we cannot get the
and 54 km downstream in river T in 9 hours answer.
=[88/(22.5+17.6)]+[54/(22.5+24]=3.35. From III we get the radius of the sphere from the
III. Boat E covers 84 km downstream in river U volume of the sphere.
and 104 km downstream in river U in 4/3*3*r*r*r=10976
=[84/(37.5+17.6)+104/[37.5+17.6]= 3.41 hours. So, r=14 m
And the height of the cuboid is equal to the
15) Answer: D radius of the sphere.
Let's check options D, From II we know that the breadth is 4 more than
Quantity of Milk in Jar A, 120*60/100=72 the height cuboid and the ratio of length and
Quantity of Water in Jar A 120-72=48 breadth is also given in II.
Quantity of Milk in Jar B = 80*5/8=50 Breadth is 14+4=18 m
Quantity of Water in Jar B = 80-50=30 Length is 14*3/2=21

Click Here For Bundle PDF Course | support@guidely.in Page 12 of 13


Bank Po Mains PDF Course 2024
Quantitative Aptitude Day -16 (Eng)

So, volume = 18*21*14 So, all the statements together are necessary
So, from II and III together we can calculate the 19) Answer: A
answer. From each single statement, we cannot calculate
the answer.
18) Answer: B But from I and II we get the answer.
From I, we get the sum of the age of A and B If the principal amount is p.
and the sum of the age of C and D. So, p*30*3/100=10800
Sum of the present ages of A and B = 85-10=75 Or, p=10800*100/90=12000
The sum of ages of C and D is = 54
From II, the Sum of the age of A and E is 58 and 20) Answer: A
the sum of the age of A and D is 32*2=64 Let's check option A,
From I and III, present age of C is 54*5/9=30, The length of train A is 320 m
present age of D is 24. The speed of train A is 90*5/18=25 m/sec
present age of E = 24-6=18 So, the length of the platform = 25*24-320=280
From II, we HAVE sum OF the ages A and E. So, the length of train B is 560
So, the present age of A is 58-18=40 The speed of train B is 72*5/18=20 m/sec
From I we get the sum of the present age of B So, required time = 3*280/20=42 sec
and A. So, satisfied.
So, the age of B is 75-40=35 Similarly, we can check others' values also.

Click Here For Bundle PDF Course | support@guidely.in Page 13 of 13


Bank Po Mains PDF Course 2024
English Day - 16

English Language
Directions (1-5): In the following questions three issued in Chawl land
columns are given containing three
several places scam case.
Sentences/phrases and three connectors. A
across the
sentence/phrase from the first column may or country
may not connect with another sentence/phrase
A. a-II-f
from the second column by using the given
B. c-III-d
connectors to make a grammatically and
C. b-I-e
contextually correct sentence. Each question has
D. a-II-f and c-III-d
five options, each of which display the
E. a-II-f, b-I-e and c-III-d
sequence(s) in which the sentences/phrases can
be joined to form a grammatically and
2.
contextually correct sentence.
1. Column I Column II Column III

Column I Column II Column III a) Indian I. or d) they want to


Railways carry goods
a) The I. despite d) heavy rain
announced new weighing more
Enforcement and the release
luggage rules, than the
Directorate of dam water
as a result of standard fixed
conducted into rivers.
which train by the railway.
searches at
passengers will
some locations
now have to
in Mumbai
either limit their
b) Tamil Nadu's II. in e) the luggage,
government connection independence
b) Railways II. such that e) will have to
school's with day event and
have fixed a pay more for
headmistress stated that her
limit on luggage carrying extra
refused to hoist religion
according to baggage while
the national disallowed her
each coach, travelling.
flag to do so.
c) Passengers III. if f) heavy goods
will now have to ranging from
c) Flood alerts III. due to f) the Rs 1,034
pay extra 40 kg to 70 kg
have been crore Patra

Click Here For Bundle PDF Course | support@guidely.in Page 1 of 10


Bank Po Mains PDF Course 2024
English Day - 16

can be kept in averages.


the train A. b-I-f and c-III-e
compartment. B. b-I-f, a-II-d and c-III-e
A. a-I-e and b-II-f C. b-I-f
B. c-III-d D. a-II-d
C. b-II-f and c-III-d E. c-III-e
D. a-I-e and c-III-d
E. a-I-e, b-II-f and c-III-d 4.

Column I Column II Column III


3.
a) India on I. Indeed d)the number
Column I Column II Column III
Wednesday of active cases
a) Of 7,239 I. and d)with the most recorded 9,062 came down to
cities, India is severe fresh COVID-19 1,05,058.
home to 18 of increase in infections that
the 20 cities PM2.5 pollution took its tally to
from 2010 to 4,42,86,256,
2019
b) In case a II. while e) the airlines
b) In the 20 II. yet e) China has passenger does have been
cities with seen the not comply with advised again
highest PM2.5 greatest the directions, on August 16
exposures, improvements. to strictly
residents in comply with
cities from India, COVID-19
Nigeria, Peru, protocol inside
the aircraft.
c) India and III. whereas f) Bangladesh
Indonesia have are exposed to c) The rise in the III. f) strict action
seen the most PM 2.5 levels number of subsequently will be taken
severe increase that are COVID-19 by airlines
in PM2.5 several-fold cases against the
pollution, higher than the passenger
global
A. b-III-f

Click Here For Bundle PDF Course | support@guidely.in Page 2 of 10


Bank Po Mains PDF Course 2024
English Day - 16

B. a-II-d and c-I-e D. a-I-e, b-III-f and c-II-d


C. a-II-d, b-III-f and c-I-e E. a-I-e
D. a-II-d
E. c-I-e Directions (6-10): Below are given sentences
that follow a word highlighted in bold above
5. them. These sentences have been made from

Column I Column II Column III words that are directly related to the word
highlighted. One of the sentences uses a word
a) India’s I. though d)would last for that may be an antonym or unrelated to the given
ambitious a month. word. Find that sentence and mark the same as
project to your answer:
translocate 6. Snag:
African cheetahs A. The biggest impediments to trade are no
has missed an longer tariffs but non-tariff barriers, such as
unofficial regulations or licences.
deadline of B. They must overcome a number of obstacles
August 15, before the restaurant can be opened.
C. Life itself, without the assistance of colleges
b) In February, II. which e) sources say
and universities, is becoming an advanced
India’s High the animals are
institution of learning.
Commissioner likely to arrive
D. My youngest sister is always more of a
to Namibia, within this year.
hindrance than a help when she tries to assist
Prashant
me with a task.
Agrawal,
E. Difficulty of communication was still a barrier,
c) The cheetahs III. however f) had stated and technology was still highly limited.
from Namibia that the
were currently in cheetahs were 7. Affluence:
medical expected to A. Because the movie star made over 10 million
quarantine arrive in India dollars a film, she was able to maintain an
by “mid-2022.” opulent lifestyle

A. b-III-f B. Hiding the lucre in many different accounts,


B. c-II-d the mobsters kept a watchful eye on their funds

C. a-I-e and c-II-d

Click Here For Bundle PDF Course | support@guidely.in Page 3 of 10


Bank Po Mains PDF Course 2024
English Day - 16

C. Many things in the physical world that we think 10. Cacophony:


of as scarce are not really scarce and just A. General Greene spoke finally, his voice jarring
presently beyond our ability to capture her out of her thoughts.
D. If you wish to play for the Indian team, you B. The heavy door made him jump with a grating
should make them wonder that you have such an sound.
elite class. C. Owls soar through the night sky, screeching
E. We have got plenty of wood today from the out their presence.
forest. D. The space inside the wagons was a din of
mules and man.
8. Incorrigible: E. The performers were elated by euphony of
A. Most of our family members are habitual of applause.
reading newspaper in the morning.
B. He was feeling guilty and depressed, Directions (11-15): In each of the following
repentant and scared. questions a sentence has been given with some
C. Tracey is a chronic worrier who gets upset highlighted group of words/phrases which may or
with everything with the wink of an eye may not be grammatically correct. Sentence is
D. We need to remember that God’s wrath does then followed by three alternatives. You are
burn against impenitent sinners required to choose the grammatically correct
E. These range from practice and mock drills to option that best replaces the incorrect phrase.
hardcore leaders who were keeping them in 11. He tried pulling the shawl over our eyes by
discipline. hiding the profit in a separate account, but we
were quick to catch onto his scheme.
9. Malign: a) Pulling the shroud over our eyes
A. She is always upfront and direct when it b) Pulling the wool over our eyes
comes to correcting me as she knows it will be c) To deceive
beneficial for me to listen to her. A. Only (c)
B. The evil queen was portrayed by the author as B. Both (a) and (b)
malignant and powerful C. Only (b)
C. You say she has become very spiteful since D. Both (b) and (c)
the last few weeks E. Only (a)
D. His hostile gaze met hers
E. The former vamp’s baleful look remained but 12. Prohibition will not curb an alcohol craving for
his eyes were blue, not red. alcohol; it will only make an addict seek out
clandestine ways to consume it.

Click Here For Bundle PDF Course | support@guidely.in Page 4 of 10


Bank Po Mains PDF Course 2024
English Day - 16

a) Alcoholic craving for alcohol 15. Instead of abating this sordid situation
b) Alcohol craving for alcoholic through additional fund allocation and
c) Alcoholic’s craving for alcohol appropriate policy intervention, the government is
A. Both (a) and (c) moving in the opposite direction, promoting and
B. Both (b) and (c) enriching rich oligarchy.
C. Only (a) a) Enrich richer oligarchs
D. Only (b) b) Enriching rich oligarchs
E. Only (c) c) Enrich rich oligarchy
A. Only (a)
13. Without the necessary infrastructure for B. Both (a) and (c)
rehabilitation medication and therapy, no ban, C. Both (b) and (c)
however, stringent will keep the teetotalers away D. Only (c)
from the bottle. E. Only (b)
a) Tippler away from the bottle
b) Boozer away from the bottle Directions (16-20): In the following questions,
c) Abstainer away from the bottle three sentences are given, which may or may not
A. Both (b) and (c) be free from error, you have to identify which
B. Only (b) sentence is free from error and mark it as your
C. Only (c) answer
D. Both (a) and (b) 16.
E. Only (a) 1. Economic growth and rapid urbanization are
major cause of increasing waste generation.
14. In most countries, diplomacy has become 2. He said that people were so obsessed with
hostage to domestic populist agendas with taking mobile phone footage that they miss the
diplomats left to play the organ grinder monkey. actual experience of the moment.
a) Organ grinder’s monkey 3. Freedom of expression and the freedom to
b) Grinder organ’s monkey practice and propagate a person’s own religion
c) Organ’s grinder monkey are constitutional endowed rights in this multi-
A. Only (b) religious country.
B. Both (b) and (c) A. Only 1
C. Only (a) B. Only 3
D. Only (c) C. Both 1 and 2
E. Both (a) and (b) D. Both 2 and 3
E. All are incorrect

Click Here For Bundle PDF Course | support@guidely.in Page 5 of 10


Bank Po Mains PDF Course 2024
English Day - 16

17. 19.
1. Not believing in the deity’s existence is as 1. If these numbers were updated, it would have
much an opinion as believes in it; the latter is enable the states to issue new ration cards over
religion while the former is, perhaps, philosophy. time
2. In an ideal arrangement, neither side should 2. Beside that, the food ministry has, in eight
threaten the other, and neither side should feels years, annulled some 47 million ghost or
insulted by the other’s presence. duplicate ration cards (170 million people) across
3. Any hostility between the two sides can laid the states under the PDS
bare the inclinations of society at that point of 3. Newly-enrolled beneficiaries, who have not
time. yet seeded their cards to the online portal, don’t
A. Only 1 receive their quotas.
B. Only 2 A. Only 2
C. Only 3 B. Only 3
D. Both 1 and 3 C. Both 1 and 2
E. All are incorrect D. Both 1 and 3
E. All are incorrect
18.
1. The bilateral meetings between leaders in 20.
Samarkand could shape the course of the region 1. One-third of the country’s land mass are
and the world far more than the UN described as flooded, with Sindh and Balochistan
2. Secularism functions more as tolerance and provinces the worst affected.
peaceful coexistence than as a strict separation 2. The larger point is that this is the third major
of the State from religion natural catastrophe that have impacted the
3. There is little chances, then, of any coherent, country in less than two decades
common position emerging from the SCO 3. If international responses to each of these
summit. three visitations are to be ranked, the 2005
A. Only 1 earthquake had evoked the strong reactions.
B. Only 2 A. Only 1
C. Both 1 and 3 B. Only 2
D. Both 2 and 3 C. Only 3
E. All are incorrect D. Both 1 and 2
E. All are incorrect

Click Here For Bundle PDF Course | support@guidely.in Page 6 of 10


Bank Po Mains PDF Course 2024
English Day - 16

Click Here to Get the Detailed Video Solution for the above given Questions
Or Scan the QR Code to Get the Detailed Video Solutions

Answer Key with Explanation


1. Answer: D We use “or” to connect two or more possibilities
The correct match is- a-II-f and c-III-d or alternatives.
a-II-f- The Enforcement Directorate conducted b-II-f- Railways have fixed a limit on luggage
searches at some locations in Mumbai in according to each coach, such that heavy goods
connection with the Rs 1,034 crore Patra Chawl ranging from 40 kg to 70 kg can be kept in the
land scam case. train compartment.
We use “in connection with” for expressing a We use “such that” to give reason.
relationship of one thing with another. c-III-d Passengers will now have to pay extra if
c-III-d- Flood alerts have been issued in several they want to carry goods weighing more than the
places across the country due to heavy rain and standard fixed by the railway.
the release of dam water into rivers. We use “if”to introduce a possible or unreal
We use “due to” to show cause-and-effect situation or condition.
relationships and function in the same way. Hence option E is the correct answer.
Hence option D is the correct answer.
3. Answer: A
2. Answer: E The correct match is- b-I-f and c-III-e
The correct match is- a-I-e, b-II-f and c-III-d b-I-f- In the 20 cities with highest PM2.5
a-I-e- Indian Railways announced new luggage exposures, residents in cities from India, Nigeria,
rules, as a result of which train passengers will Peru, and Bangladesh are exposed to PM 2.5
now have to either limit their luggage, or will levels that are several-fold higher than the global
have to pay more for carrying extra baggage averages.
while travelling.

Click Here For Bundle PDF Course | support@guidely.in Page 7 of 10


Bank Po Mains PDF Course 2024
English Day - 16

We use “and” conjunction between words, We use “which” conjunction to connect two
phrases etc in order to connect them together. unequal parts, e.g., dependent and independent
c-III-e- India and Indonesia have seen the most clauses.
severe increase in PM2.5 pollution, whereas Hence option C is the correct answer.
China has seen the greatest improvements.
We use “whereas” to indicate a contrast between 6. Answer: C
two facts or ideas. The word snag is used to define a small difficulty
Hence option A is the correct answer. or disadvantage that is often unexpected or
hidden. The given sentences except the one in
4. Answer: D (c), use the synonyms of the word snag.
The correct match is- a-II-d The word assistance is a positive word that
a-II-d India on Wednesday recorded 9,062 fresh means to help in making something happen. So,
COVID-19 infections that took its tally to this sentence uses an antonym to the word given
4,42,86,256, while the number of active cases in bold.
came down to 1,05,058. Therefore, the best is to mark option (c) as the
We use “while” to show “during the time that; at answer.
the same time as” and as a “whereas (indicating
a contrast).” 7. Answer: C
Hence option D is the correct answer. Affluent means prosperous, extremely rich and
also abundant. Of the given sentences above,
5. Answer: C only (c) ฀ scarce is the antonym of affluent as it
The correct match is- a-I-e and c-II-d means scanty or available in less quantity. Let’s
India’s ambitious project to translocate African understand the usages of some related words:
cheetahs has missed an unofficial deadline of Opulent means luxurious
August 15, though sources say the animals are Elite is used to denote aristocracy or superiority
likely to arrive within this year. Lucre hints at money earned dishonestly
“Though/although” are subordinating
conjunctions used to connect a subordinate 8. Answer: B
clause to a main clause. Incorrigible is something that is unable to be
The cheetahs from Namibia were currently in changed or corrected.
medical quarantine which would last for a month. The above statements are quite easy to
understand as (a) talks about being habitual

Click Here For Bundle PDF Course | support@guidely.in Page 8 of 10


Bank Po Mains PDF Course 2024
English Day - 16

which is similar to incorrigible. Also, hardcore is 11. Answer: D


similar in meaning as well. The right phrase is “pull the wool over our eyes”
(d) talks about impenitent sinners means the which means to trick or deceive someone, to
ones who are not remorseful for their acts or are hide the truth.
shamelessly audacious. This is also in a way a
synonym to the highlighted word. The word 12. Answer: E
chronic means severe and this is rule out as The right structure is “alcoholic’s craving for
well. alcohol”. Alcoholic is a person suffering from
So, (b) happens to be the best as repentant is alcoholism. And to show the possession of an
one who feels shameful or is feeling sincere alcoholic person on alcohol (‘s) is used.
regret for his actions.
13. Answer: D
9. Answer: A “Teetotaler” or “Abstainer” are persons who
‘Mal’ is a negative prefix which denotes that never drink alcohol. But the context of the
something unfriendly, harmful or bad is being sentence is for those who consume alcohol; we
described. So, malign means evil or ill-willed. use “tippler” or “boozer” for a habitual drinker of
Clearly we know that a positive word will be an alcohol.
antonym to the above so option (a) beneficial is 14. Answer: C
the obvious answer for us. The right saying is “organ grinder’s monkey”
Malignant (dangerous, harmful), Spiteful (mean, which means someone who is closely
unfriendly, hurtful), Baleful (unfriendly, harmful), associated with a powerful person, and acts on
Hostile (malicious and confrontational) are all their behalf but has no real power themselves.
similar words that carry a negative tone.
15. Answer: E
10. Answer: E The right structure is “enriching rich oligarchs”
Cacophony is a harsh and loud sound or Rule of parallelism will be applied here, so
screaming voice that is irritating to the ears. The “enrich” is not grammatically right.
words grating, screeching, jarring and din are “Oligarchy” is a form of government
related as they all carry a similar meaning. characterized by a rule of a few people or
Euphony which means pleasant to the ears or a families. But we need a plural form of “oligarch”
likeable sound is the obvious unrelated word in which is “oligarchs” which means rulers in an
the above so that’s the answer. oligarch.

Click Here For Bundle PDF Course | support@guidely.in Page 9 of 10


Bank Po Mains PDF Course 2024
English Day - 16

16. Answer: E (3) – ‘chance’ will be replaced by ‘chances’, ‘little’


(1) – ‘cause’ will be replaced by ‘causes’, the is an uncountable noun, and it is used with
subject of the noun is ‘economic growth and singular form. So, ‘chance’ is incorrect here
rapid urbanization’ which is plural, so the noun
should be used in plural form 19. Answer: B
(2) – ‘miss’ will be replaced by ‘missed’, the (1) – according to the rule of conditional
structure of the sentence is in past tense, so the sentence (if +simple past, subject + would +V1)
verb should be according to its tense. So, ‘miss’ is used, so ‘would have’ is incorrect here, only
is incorrect here ‘would’ will be used.
(3) – ‘constitutional’ will be replaced by (2) – ‘beside’ will be replaced by ‘besides’,
‘constitutionally’, we need an adverb to qualify ‘beside’ means – at the side of; next of, whereas
the verb (endowed), ‘constitutional’ is an ‘besides’ means – in addition to; apart from. The
adjective. So, ‘constitutional’ will be replaced context of the sentence talks about the additional
information, so, ‘besides’ will be used here
17. Answer: A
(2) – ‘feels’ will be replaced by ‘feel’, model verb 20. Answer: E
‘should’ doesn’t follow the verb using ‘e/es’ with (1) – ‘are’ will be replaced by ‘is’; the subject of
it. So, ‘feels’ is incorrect here the verb is ‘one-third of the country’s land’ which
(3) – ‘laid’ will be replaced by ‘lay’, the structure is a single unite, so the verb should be singular.
of the sentence is in present tense, so the verb So, ‘are’ is incorrect here
will be used according to the tense (2) – ‘have’ will be replaced by ‘has’, the subject
(Lay (V1) – Laid (V2) – Laid (V3)) of the verb is ‘natural catastrophe’ which is
singular, so the verb will be used according to
18. Answer: B the subject. So, ‘has’ will be used here
(1) – ‘can’ will be used after ‘the UN’, here the (3) – the context of the sentence talks about the
comparison is between the efforts of the leaders occurrence of three natural calamities, in which
who can shape the course of the region, which is 2005’s earthquake evoked the strongest
the leaders in Samarkand could shape it far reaction. Superlative degree ‘strongest’ will be
more the UN can. So, ‘can’ will be used after ‘the used here.
UN’

Click Here For Bundle PDF Course | support@guidely.in Page 10 of 10


Bank Po Mains PDF Course 2024
Reasoning Day -17 (Eng)

Reasoning Aptitude
Directions (1-5): Study the following statements Plane ^ Ship
and then decide which of the given conclusions Bus % Plane
logically does not follow from the given Bus $ Auto
statements disregarding the commonly known Conclusions:
facts. I) All ships can never be auto
“A@B” means “Some A is not B” II) All buses being tempo is not a possibility
“A&B” means “Only a few B is A” III) Some cars are not auto is a possibility
“A%B” means “At least some B is A” IV) Some vans cannot be plane
“A^B” means “All B is A” a) Both conclusions I and II don’t follow
“A+B” means “Only A is B” b) Only conclusion I doesn’t follow
“A$B” means “No A is B” c) Only conclusion II doesn’t follow
1. Statements: d) Only conclusions I, II and IV don’t follow
Milk @ Almond e) None follows
Almond & Nuts
Milk $ Water 3. Statements:
Almond ^ Cookie Thread % Rope
Nuts & Acid Jute ^ Thread
Acid $ Alkaline Cloth & Jute
Conclusions: Cloth $ Fibre
I) All nuts cannot be alkaline Fibre + Ribbon
II) No cookie being milk is a possibility Rope $ Cotton
III) All nuts can never be cookie Conclusions:
IV) Some water is definitely not almond I) All fibres cannot be thread
a) Both conclusions II and IV don’t follow II) Some jute can be cotton
b) Only conclusion III doesn’t follow III) All clothes can never be ribbon
c) Both conclusions I and III don’t follow IV) Some ropes are definitely not cloth
d) Only conclusion IV doesn’t follow a) Both conclusions I and IV don’t follow
e) Both conclusions III and IV don’t follow b) Only conclusion III doesn’t follow
c) Only conclusions I, II and IV don’t follow
2. Statements: d) Only conclusion IV doesn’t follow
Tempo + Van e) Both conclusions III and IV don’t follow
Tempo & Car
Car @ Ship 4. Statements:

Click Here For Bundle PDF Course | support@guidely.in Page 1 of 11


Bank Po Mains PDF Course 2024
Reasoning Day -17 (Eng)

Layer & Part Directions (6-10): Study the following information


Slice % Layer carefully and answer the given questions.
Portion ^ Slice A certain number of persons from the same
Piece & Portion family of three generations sit in a linear row
Piece $ Total facing the north directions. Information about
Lump % Total only a few of them is known.
Conclusions: Only four persons sit between N and O, who is
I) All portions can be total the child of J and is not married. N sits second
II) No slice being total is a possibility from one of the extreme ends. I sits adjacent to
III) Some layers cannot be piece N, who is the only son of B. The one who is the
IV) All lump is part is not a possibility wife of B is the only daughter of J, who has only
a) Both conclusions I and III don’t follow two children. Only three persons sit between H
b) Only conclusion II doesn’t follow and O. The number of persons sitting between
c) Only conclusions I, III and IV don’t follow the one who is the brother-in-law of O and H is
d) Only conclusion I doesn’t follow one more than the number of persons sitting
e) All conclusions I, II, III and IV don’t follow between I and L, who sits immediate right of H. B
is the child of L and doesn’t have any siblings.
5. Statements: The one who is the husband of L sits fourth to
Clock + Alarm the left of the one who is the son-in-law of J and
Watch % Clock sits at one of the extreme ends. A sits sixth to the
Time $ Watch left of T and is the brother of H. Only one person
Seconds & Time sits between T and the one who is the father of
Minutes ^ Seconds N. T is the daughter-in-law of B. A is the father-
Minute $ Hour in-law of I. As many persons sit to the left of A as
Conclusions: to the right of J.
I) Some alarms can never be minutes. 6. If V is the wife of J, then the relationship
II) Some second is watch is not a possibility between the one who sits third to the right of L
III) All time cannot be hour and V is same as the relationship between __
IV) No hour is clock and __.
a) Both conclusions I and II don’t follow a) H, T
b) Both conclusions II and IV don’t follow b) N, H
c) Only conclusion IV doesn’t follow c) L, B
d) Only conclusions I, II and IV don’t follow d) A,T
e) None follows e) None of these

Click Here For Bundle PDF Course | support@guidely.in Page 2 of 11


Bank Po Mains PDF Course 2024
Reasoning Day -17 (Eng)

7. What is the sum of the number of persons b) N is the father of the one who sits immediate
sitting in the row and the number of persons right of J.
sitting between O and J? c) H is the sister-in-law of L
a) 36 d) Only five persons sit to the left of the one who
b) 28 is the son-in-law of B
c) 41 e) O is the maternal uncle of the one who is the
d) 33 spouse of T.
e) 56
Directions (11-15): Study the following
8. If X sits sixth to the left of N, then what is the information carefully and answer the given
position of X with respect to the one who is the questions.
paternal grandmother of N? Seven persons - L, M, N, O, P, Q and R live on
a) Immediate left seven different floors of a seven-storey building
b) Third to the right where the lowermost floor is numbered one and
c) Sixth to the left the floor immediately above it is numbered two
d) Second to the right and so on. Each of them joined a company in
e) Eighth to the left different years. No two persons joined in the
same year and no one joined after 2004.
9. Which among the following pair of persons are The one who joined the company in 1989 lives
nephew and uncle respectively? three floors above M. Only one person lives
I) HB between M and L, who doesn’t live above the
II) NO fourth floor. The number of floors above L is one
III) LB more than the number of floors below R. The
a) Only II difference between the years in which the one
b) Only I and III who lives on the topmost floor and R joined the
c) Only II and III company is 16 years. Only four floors are
d) Only I between the one who joined in 1995 and Q, who
e) All I, II and III didn’t join in 1989. As many floors above Q as
below N, who joined three years after P. The
10. Which of the following statement(s) is/are difference between the years in which the one
true as per the given arrangement? who lives immediately below N and N joined the
a) Only one person sits between B and the one company is 6 years. There is a gap of seven
who is the son of I. years between the years in which P and L joined
the company. The difference between the years

Click Here For Bundle PDF Course | support@guidely.in Page 3 of 11


Bank Po Mains PDF Course 2024
Reasoning Day -17 (Eng)

in which O and M joined the company is 22 a) Three more


years. One of the persons joined the company in b) Two more
1982. c) One less
11. If the experience of each person is calculated d) Two less
from the base year 2023, then what is the sum of e) Three less
the experience of the one who lives on the
topmost floor and N? 15. In which of the following option the first
a) 38 person joined three years after the second
b) 80 person?
c) 72 a) LR
d) 52 b) RQ
e) 65 c) LO
d) ML
12. What is the difference between the years in e) PM
which the one who lives immediately below O
and M joined the company? Directions (16-20): Study the following
a) 5 years information carefully and answer the given
b) 10 years questions.
c) 8 years Raj, Dev, Hari, Sam, James and Dino went for
d) 17 years the mountain trucking to take the photo of the
e) 13 years mountain range. A different team of two persons
took the photo of six mountain range viz.
13. What is the sum of the floor number of P and Himalayas, Aravalli, Vindhya, Western Ghats,
the difference between the years in which R and Hindu Kush and Garhwal. No two teams take the
Q joined the company? photo of the same mountain range.
a) 10  Each person takes a photo of only two
b) 30 mountain ranges
c) 16  James, Sam and Dev are not combined as a
d) 12 team while taking photo.
e) 22  James did not take a photo of Himalayas.
 Raj was not the member of the team who took
14. The number of floors above the one who the photo of Garhwal and Vindhya.
joined in 1995 is ___ than the number of floors  Hari is one person in the team who took a
below P. photo of Aravalli.

Click Here For Bundle PDF Course | support@guidely.in Page 4 of 11


Bank Po Mains PDF Course 2024
Reasoning Day -17 (Eng)

 One of the persons, who took a photo of e) None of these


Vindhya, also a part of taking a photo of another
mountain range when pairing with James 18) Which of the following combination is true?
 Raj and Sam, Dev and Dino were not a) Dino – Dev – Garhwal
combined as a team while taking a photo b) Hari – Dino – Vindhya
 Neither Dino nor Raj takes the photo of c) James – Dev – Aravalli
Western Ghats. d) Dino – Sam – Vindhya
 No same person was member of the team who e) More than one combination is true.
took a photo of both the Western Ghats and
Himalayas. 19) Which of the following person is one of the
 Dino and James successfully photographed as team members of the one who took the photo of
a team. Garhwal?
16) Which among the following pair of person a) Dev
took the photo of Hindu Kush? b) Sam
a) James – Hari c) Hari
b) Dino – Sam d) Raj
c) Raj – James e) James
d) Sam – James
e) None of these 20) Which of the following pair of persons form a
successful team?
17) Sam took the photo of which of the following a) Dino – Hari
mountain Ranges? b) Dev – Hari
a) Vindhya & Aravalli c) Hari – James
b) Aravalli & Western Ghats d) James – Sam
c) Himalayas & Garhwal e) None of these
d) Western Ghats & Vindhya

Click Here For Bundle PDF Course | support@guidely.in Page 5 of 11


Bank Po Mains PDF Course 2024
Reasoning Day -17 (Eng)

Click Here to Get the Detailed Video Solution for the above given Questions
Or Scan the QR Code to Get the Detailed Video Solutions

Answer Key with Explanation


Directions (1-5):
1) Answer: D

4) Answer: C
2) Answer: A

5) Answer: D

3) Answer: D

Directions (6-10):
6) Answer: C

Click Here For Bundle PDF Course | support@guidely.in Page 6 of 11


Bank Po Mains PDF Course 2024
Reasoning Day -17 (Eng)

7) Answer: B Again, we have


8) Answer: D  Only three persons sit between H and O.
9) Answer: A  The number of persons sitting between
10) Answer: E the one who is the brother-in-law of O and
Final arrangement H is one more than the number of
persons sitting between I and L, who sits
immediate right of H.
 B is the child of L and doesn’t have any
siblings.
 The one who is the husband of L sits
We have,
fourth to the left of the one who is the son-
 Only four persons sit between N and O,
in-law of J and sits at one of the extreme
who is the child of J and is not married.
ends.
 N sits second from one of the extreme
After applying the above conditions case-2 and
ends.
case-2a get eliminated because L’s husband
 I sit adjacent to N, who is the only son of
doesn’t sit at one of the extreme ends.
B.
 The one who is the wife of B is the only
daughter of J, who has only two children.
From the above conditions, there are four
possibilities

Again, we have
 A sits sixth to the left of T and is the
brother of H.
 Only one person sits between T and the
one who is the father of N.
 T is the daughter-in-law of B. A is the
father-in-law of I.
 As many persons sit to the left of A as to
the right of J.

Click Here For Bundle PDF Course | support@guidely.in Page 7 of 11


Bank Po Mains PDF Course 2024
Reasoning Day -17 (Eng)

After applying the above conditions case-1 gets From the above conditions, there are four
eliminated because there is no possibility to possibilities
place J, hence case-1a shows the final
arrangement.

Again, we have
Directions (11-15):
 The difference between the years in
11) Answer: C
which the one who lives on the topmost
12) Answer: E
floor and R joined the company is 16
13) Answer: D
years.
14) Answer: B
 Only four floors are between the one who
15) Answer: E
joined in 1995 and Q, who didn’t join in
Final arrangement
1989.
 As many floors above Q as below N, who
joined three years after P.
 The difference between the years in
which the one who lives immediately
below N and N joined the company is 6
years.
After applying the above conditions, case-2 and
case-3 get eliminated because there is no
possibility to place N in case 3 and no possibility
We have,
to place P in case 2.
 The one who joined the company in 1989
lives three floors above M.
 Only one person lives between M and L,
who doesn’t live above the fourth floor.
 The number of floors above L is one more
than the number of floors below R.

Click Here For Bundle PDF Course | support@guidely.in Page 8 of 11


Bank Po Mains PDF Course 2024
Reasoning Day -17 (Eng)

20. Answer: B

Again, we have
 There is a gap of seven years between
the years in which P and L joined the We have:

company.  Each person takes a photo of only two

 The difference between the year in which mountain ranges

O and M joined the company is 22 years.  James, Sam and Dev are not combined

 One of the persons joined the company in as a team while taking photo.

1982.  Raj and Sam, Dev and Dino were not

After applying the above conditions, case-4 gets combined as a team while taking photo

eliminated because the difference between the  Dino and James successfully

year in which O and M joined the company is 22 photographed as a team.

years is not satisfied. Hence case-1 shows the From the above given conditions, the possible

final arrangement. pairs are,


(Raj, Dev), (Raj, Hari), (Raj, James), (Raj, Dino),
(Dev, Hari), (Hari, Sam), (Hari, James), (Hari,
Dino), (Sam, Dino) and (Dino and James)
I) Dino and James is one of the pair
Given that, each person takes the photo of only
two mountain ranges. Thus, Sam is paired with
only two persons, (i.e.) Hari and Sam, Sam and
Dino is another pair.
II) Hari and Sam is one of the pair
Directions (16-20): III) Sam and Dino is one of the pair
16. Answer: C Again, Dino is already paired with two persons,
17. Answer: D which is Sam and James. So, (Raj and Dino),
18. Answer: D (Hari and Dino) pair gets eliminated
19. Answer: E

Click Here For Bundle PDF Course | support@guidely.in Page 9 of 11


Bank Po Mains PDF Course 2024
Reasoning Day -17 (Eng)

(Raj, Dev), (Raj, Hari), (Raj, James), (Raj, Dino),


(Dev, Hari), (Hari, Sam), (Hari, James),
(Hari,Dino), (Sam, Dino) and (James, Dino).
Dev is paired with only two persons. So, another
two pair is (Raj, Dev) and (Dev, Hari)
IV) Raj and Dev is one of the pair
V) Dev and Hari is one of the pair
Now, Hari is already paired with two persons, Again we have,
which is Dev and Sam. So (Hari and Raj), (Hari  Raj was not the member of the team who
and James) pair get eliminated took the photo of Garhwal and Vindhya.
(Raj, Dev), (Raj, Hari), (Raj, James), (Dev, Hari),  One of the persons, who took a photo of
(Hari, Sam), (Hari, James), (Sam, Dino) and Vindhya, also a part of taking a photo of
(James, Dino). another mountain range when pairing with
(Raj, Dev), (Raj, James), (Dev, Hari), (Hari, James
Sam), (Sam, Dino) and (James, Dino). Thus, the person paired with James is Dino and
Remaining pair is (Raj and James) Raj, but raj does not take the photo of Vindhya
VI) Raj and James is one of the pair Range. So Dino is the one who took the photo of
Again, we have Vindhya.
 Hari is one person in the team who took a
photo of Aravalli.
From the above conditions, there are two
possibilities

Again, we have
 James did not take a photo of Himalayas.
 No same person was member of the team
who took a photo of both the Western
Ghats and Himalayas.
Again, we have
No common person should be between the pair
 Neither Dino nor Raj takes the photo of
of persons who took the photo of Western Ghats
Western Ghats.
and Himalayas, So case 1 gets eliminated. Thus,
case 2 gives the final arrangement.

Click Here For Bundle PDF Course | support@guidely.in Page 10 of 11


Bank Po Mains PDF Course 2024
Reasoning Day -17 (Eng)

Click Here For Bundle PDF Course | support@guidely.in Page 11 of 11


Bank Po Mains PDF Course 2024
Quantitative Aptitude Day -17 (Eng)

Quantitative Aptitude

Directions (1-4): Study the following data b) 265 cubic feet


carefully and answer the questions: c) 245 cubic feet
There are two cubical boxes A and B and two d) 235 cubic feet
cuboidal boxes C and D. The side (in feet) of e) 275 cubic feet
each cubical box is different and the length,
width, and height (in feet) of each cuboidal box 3) If the cost of painting the walls of boxes is ₹15
are different. per square feet, then find the average of total
The length of box C is equal to the side of box A, cost of painting box B (except the floor), the total
the width of box D is equal to the side of box B cost of painting box C (except the floor) and the
and the height of box C is half of its length. The total cost of painting box D (except the floor)?
width of boxes C and D are equal. The height of a) ₹2420
box D is 4 feet less than its width and the length b) ₹2220
of box D is 10 feet. The volume of box D is 120 c) ₹2320
cubic feet, and the volume of box B is 24 cubic d) ₹2620
feet more than the volume of box C. e) ₹2520
1) If the ratio of the volume of box A to the
volume of box B is M: N and the ratio of the 4) If the length, width and height of cuboidal box
volume of box C to the volume of box D is P: Q, E are respectively 125%, and 50% of
then find the value of (NQ – M – P) is divisible by that of box C, then find that which of the following
which of the following? statements is/are true?
a) 5 X: The difference between the volumes of boxes
b) 9 D and E is 40 cubic feet.
c) 7 Y: The ratio of the side of box B to the length of
d) Both (a) and (b) box E is 2: 5.
e) Both (b) and (c) Z: The average of the side of box A and the
height of box E is 5 feet.
2) If box A is filled with water up to the height of 5 a) Only Y
feet and box D is filled with water up to the height b) Only X and Z
of 1.25 feet, then find the difference between the c) Only Y and Z
volume of water filled in box A and the volume of d) None is true
water filled in box D? e) Only X
a) 255 cubic feet

Click Here For Bundle PDF Course | support@guidely.in Page 1 of 18


Bank Po Mains PDF Course 2024
Quantitative Aptitude Day -17 (Eng)

Directions (5-8): Study the following data carefully and answer the questions:
The data given below shows the number of students in two schools A and B in different age groups i,e;
(5-8) years, (9-12) years, (13-16) years and (17-20) years.
The bar graph given below shows the following data:

Note:
1: In school A, the number of students in the age group (5 – 8) years, is 10 less than those in the age
group (13 – 16) years.
2: In school B, the number of students in the age group (9 – 12) years, is equal to those in the age group
(17 – 20) years.
5) If the ratio of the total number of 5 to 20 years c) Only P
old students in school A to those in school B is d) Only R
M: N, then which of the following can’t be the e) Only Q and R
average of the number of students in school B, in
age group (9 – 12) years, those in age group (13 6) If the average of the number of students in
– 16) years and those in age group (17 – 20) age group (5 – 8) years in schools A, B and C is
years? 48 and the average of the number of students in
P: age group (9 – 12) years in schools A, B and C is
Q: 25(N – M) 70, then find that the number of students in
R: (M – 2)% of 10(N + 3) school C in age group (5 – 8) years is what per
a) Only P and Q cent of those in age group (9 – 12) years?
b) Only Q a) 51%

Click Here For Bundle PDF Course | support@guidely.in Page 2 of 18


Bank Po Mains PDF Course 2024
Quantitative Aptitude Day -17 (Eng)

b) 85% 8) Which of the following statements is/are true?


c) 34% P: The ratio of the total number of 5 to 12 years
d) 102% old students in school A to those in school B is 5:
e) 68% 4.
Q: If the average of the number of students in
7) In school B, if the ratio of boys to girls in age age group (13 – 16) years in school B and those
group (5 – 8) years is 3: 2, the ratio of boys to in school C is 90, then the average of the number
girls in age group (9 – 12) years is 5: 9, the ratio of students in age group (13 – 16) years in
of boys to girls in age group (13 – 16) years is 9: school A and those in school C will be 80.
11 and the ratio of boys to girls in age group (17 R: The difference between the total number of 9
– 20) years is 4: 3, then find the total number of 5 to 16 years old students in school A and those in
to 20 years old girls in school B? school B is 20.
a) 150 a) Only Q
b) 130 b) Only P and Q
c) 145 c) Only Q and R
d) 125 d) Only P
e) 140 e) Only R

Directions (9-12): Study the following data carefully and answer the questions:
Seven workers A, B, C, D, E, F and G are hired to complete a work and the efficiency of each worker is
different.
The hexagonal graph given below shows the following data:

Click Here For Bundle PDF Course | support@guidely.in Page 3 of 18


Bank Po Mains PDF Course 2024
Quantitative Aptitude Day -17 (Eng)

Note: If A works with 125% of its original efficiency and G works with of its original efficiency,
then A and G together will finish the work in 9.6 hours.
9) If B and C work with 50% of their respective Q: The average of time, in which A alone, C
original efficiencies and D and E work with 200% alone and D alone can complete the work, is 13
of their respective original efficiencies, then find hours.
the time, in which B, C, D and E together will R: The ratio of C’s efficiency to F’s efficiency is 4:
complete the work with their new efficiencies? 1.
a) a) Only Q
b) b) Only P and Q

c) c) Only Q and R

d) d) Only P
e) Only R
e)

11) Each work received wages in the ratio of


10) Which of the following statements is/are
their efficiencies. If the difference between B’s
true?
wages and D’s wages is ₹5400, then what is the
P: D and G together can complete the work in
difference between A’s wages and G’s wages?
hours.
a) ₹1018

Click Here For Bundle PDF Course | support@guidely.in Page 4 of 18


Bank Po Mains PDF Course 2024
Quantitative Aptitude Day -17 (Eng)

b) ₹3675 I: 4, X, 24, XY, 840


c) ₹9187.5 II: (X – 5), 6, (Y + 3), 90, 630
d) ₹3187 Find that which of the following statements is/are
e) ₹11187.5 true?
P: The 1st term of series II is 32.5% of the 2nd
12) Find the ratio of between the time, in which A term of series I.
and B together can complete the work, the time, Q: The value of X is of the value of Y.
in which C and D together can complete the work R: The average of the 4th term of series I and the
and the time, in which F and G together can 3rd term of series II is 79.
complete the work respectively? a) Only Q
a) 3: 2: 7 b) Only P and Q
b) 15: 8: 28 c) Only Q and R
c) 9: 5: 21 d) None is true
d) 5: 2: 11 e) Only R
e) 45: 28: 112
15) A series is given below:
Direction (13-15): Study the following data 16, 30, 58, 114, 226, 5X, (10X – Y)
carefully and answer the questions: If a new series is formed, whose 1st term is
13) A series is given below, in which the next and the other terms are obtained by adding 1 on
term is obtained by adding 1 on double of the the double of the previous term, then find the 3rd
previous term. term of the new series?
4, (x + y), 19, (8x – 1) a)
If a new series is formed, whose 1st term is (4y – b)
x) and follows the same logic as in the given
c)
series, then find the average of 5th term of the
d) Both (a) and (b)
given series and the 5th term of the new series?
e) Both (b) and (c)
a) 125
b) 140
Directions (16-19): Study the following data
c) 135
carefully and answer the questions:
d) 130
There are five mixtures A, B, C, D, and E of two
e) 120
types of tea P and Q. The table given below
shows the percent of quantity of each mixture out
14) Two series I and II are given below and both
of total quantity in all the five mixtures together.
the series follow the same pattern:

Click Here For Bundle PDF Course | support@guidely.in Page 5 of 18


Bank Po Mains PDF Course 2024
Quantitative Aptitude Day -17 (Eng)

Note: The cost of type P and type Q tea per kg is b) 7.5%


₹40 and ₹60 respectively and all the quantities c) 15%
are in kg. d) 12.5%
e) 20%

17) If the whole quantity of mixture D is mixed


with ‘x’ kg of mixture G to form a new mixture
such that per kg cost of new mixture becomes
₹ . If √(y2 – x2) = 60, then find the value of ‘x
+ y’.
a) 180
b) 160
c) 200
d) 240
e) 120

18) Two mixtures A and C are mixed in a certain

Some other information is also given: ratio to form a new mixture whose selling price

Profit percent earned when mixture E is sold at lies between ₹46.53 and ₹48.4 when sold at 10%

the cost of type Q tea is 31.25%. Mixture F is profit. Which of the following ratio(s) given below

formed after mixing mixtures A and D in the ratio can be the possible ratio in which mixtures A and

of 2: 3 while mixture G is formed after mixing C must be mixed?

mixtures B and C such that after selling mixture I: 1: 4

G at the cost of type P tea, loss percent incurred II: 2: 9

is . III: 3: 7
a) Only I
16) If in mixture F, 15 kg of type P is removed
b) Only I and III
and 15 kg of type Q tea is added, then the
c) Only II
quantity of type P tea in the final mixture
d) All I, II, and III
becomes 15 kg more than the quantity of type Q
e) Only II and III
tea in that mixture F. Find the profit percent
earned when the final mixture F is sold at the
19) The whole quantity of two mixtures X and Y
cost of ₹54 per kg.
are mixed together to form another mixture M (‘X’
a) 10%

Click Here For Bundle PDF Course | support@guidely.in Page 6 of 18


Bank Po Mains PDF Course 2024
Quantitative Aptitude Day -17 (Eng)

and ‘Y’ are the mixtures of the same per kg cost and B are quadratic in ‘y’. Choose the pair of
price among the mixtures A, B, C, D, and E). If statements in which ‘x > y’.
55 kg of another mixture N (ratio of type P to
type Q tea is 3: 8) is mixed with the whole
quantity of mixture M to form a final mixture, then
find the difference between type P and type Q
I: PA
tea in the final mixture.
II: PB
a) 45 kg
III: QA
b) 30 kg
IV: QB
c) 25 kg
a) I and II
d) 40 kg
b) II and IV
e) 35 kg
c) II and III
d) III and IV
20) Below given is a table that contains four
e) I and IV
quadratic equations numbered P, Q, A, and B
respectively. P and Q are quadratic in ‘x’ while A
Click Here to Get the Detailed Video Solution for the above given Questions
Or Scan the QR Code to Get the Detailed Video Solutions

Answer Key with Explanation


Directions (1-4): The width of box C = ‘y’ feet
Let the side of box A be ‘2x’ feet. And the height of box D = (y – 4) feet
So, the length of box C = 2x feet Since, the length of box D is 10 feet and its
And the height of box C = (2x/2) = ‘x’ feet volume is 120 cubic feet.
Let the side of box B be ‘y’ feet. So,
So, the width of box D = ‘y’ feet 10 * y * (y – 4) = 120

Click Here For Bundle PDF Course | support@guidely.in Page 7 of 18


Bank Po Mains PDF Course 2024
Quantitative Aptitude Day -17 (Eng)

y2 – 4y – 12 = 0 So, the volume of water filled in box A = 8 * 8 * 5


y2 – 6y + 2y – 12 = 0 = 320 cubic feet
y(y – 6) + 2(y – 6) = 0 Since, box D is filled with water up to the height
y=6 of 1.25 feet.
Since, the volume of box B is 24 cubic feet more So, the volume of water filled in box D = 10 * 6 *
than the volume of box C. 1.25 = 75 cubic feet
So, Required difference = 320 – 75 = 245 cubic feet
y3 – 2x2y = 24
216 – 12x2 = 24 3) Answer: C
x=4 Since, the side of box B = 6 feet
The side of box A = 2 * 4 = 8 feet So, the total surface area of box B to be painted
The side of box B = 6 feet = 5 * 62 = 180 square feet
The length of box C = 2 * 4 = 8 feet And the total cost of painting box B = 180 * 15 =
The width of box C = 6 feet ₹2700
The height of box C = 4 feet Since, the length, width and height of box C are
The length of box D = 10 feet 8 feet, 6 feet and 4 feet respectively.
The width of box D = 6 feet So, the total surface area of box C to be painted:
The height of box D = 6 – 4 = 2 feet (8 * 6) + 2(8 * 4 + 6 * 4) = 48 + 112 = 160 square
1) Answer: E feet
The volume of box A = 8 * 8 * 8 = 512 cubic feet And the total cost of painting box C (except the
The volume of box B = 6 * 6 * 6 = 216 cubic feet floor) = 160 * 15 = ₹2400
So, M: N = 512: 216 = 64: 27. Since, the length, width and height of box D are
The volume of box C = 8 * 6 * 4 = 192 cubic feet 10 feet, 6 feet and 2 feet respectively.
The volume of box D = 10 * 6 * 2 = 120 cubic So, the total surface area of box D to be painted:
feet (10 * 6) + 2(10 * 2 + 6 * 2) = 60 + 64 = 124
So, P: Q = 192: 120 = 8: 5 square feet
Now, the value of (NQ – M – P) = (135 – 64 – 8) And the total cost of painting box D (except the
= 63 floor) = 124 * 15 = ₹1860
And 63 are divisible by both 9 and 7. Required average = = ₹2320

2) Answer: C 4) Answer: B
Since, box A is filled with water up to the height Since, the length of box C = 8 feet
of 5 feet. So, the length of box E = 125% of 8 = 10 feet

Click Here For Bundle PDF Course | support@guidely.in Page 8 of 18


Bank Po Mains PDF Course 2024
Quantitative Aptitude Day -17 (Eng)

Since, the width of box C = 6 feet So,


SO, the width of box E = of 6 = 8 feet (5Y – 15) – (5X + 5) = 10
Since, the height of box C = 4 feet Y – X = 6 -----------(1)
So, the height of box E = 50% of 4 = 2 feet The number of students in school B in age group
From X: (9 – 12) years:
The volume of box D = 10 * 6 * 2 = 120 cubic
feet
The number of students in school B in age group
The volume of box E = 10 * 8 * 2 = 160 cubic
(17 – 20) years:
feet
Required difference = 160 – 120 = 40 cubic feet
Since, in school B, the number of students in
So, X is true,
From Y: age group (9 – 12) years, is equal to those in

Ratio of the side of box B to the length of box E age group (17 – 20) years.
So,
= 6: 10 = 3: 5
3(X + Y) – 14 = 10(Y – X) + 10
So, Y is not true.
From Z: From equation (1):

Average of the side of box A and the height of 3(X + 6 + X) – 14 = 10 * 6 + 10

box E: X = 11

So, Z is true.
Hence, only X and Z are true.

Directions (5-8):
The number of students in school A in age group
(5 – 8) years:

The number of students in school A in age group


(13 – 16) years:

Since, in school A, the number of students in


age group (5 – 8) years, is 10 less than those in
age group (13 – 16) years. 5) Answer: B

Click Here For Bundle PDF Course | support@guidely.in Page 9 of 18


Bank Po Mains PDF Course 2024
Quantitative Aptitude Day -17 (Eng)

The total number of 5 to 20 years old students in (3 * 70) – (90 + 70) = 50


school A: Required average = = 68%
60 + 90 + 70 + 50 = 270
The total number of 5 to 20 years old students in 7) Answer: A
school B: In school B,
50 + 70 + 100 + 70 = 290 The number of girls in age group (5 – 8) years =
So, M: N = 270: 290 = 27: 29 = 20
Now, the required average: The number of girls in age group (9 – 12) years =
= 45

From P: The number of girls in age group (13 – 16) years


= = 55
The number of girls in age group (17 – 20) years
So, P can be the required average.
= = 30
From Q:
25(N – M) = 25(29 – 27) = 50 So, the total number of 5 to 20 years old girls in

So, Q can’t be the required average. school B = 20 + 45 + 55 + 30 = 150

From R:
(M – 2)% of 10(N +3) = (27 – 2)% of 10(29 + 3) = 8) Answer: D

80 From P:

So, R can be the required average. Ratio of the total number of 5 to 12 years old

Hence, only Q can’t be the required average. students in school A to those in school B = (60 +
90): (50 + 70) = 150: 120 = 5: 4

6) Answer: E So, P is true.

Since, the average of the number of students in From Q:

age group (5 – 8) years in schools A, B and C is Since, the average of the number of students in

48. age group (13 – 16) years in school B and those

So, the number of students in age group (5 – 8) in school C is 90.

years in school C: So, the number of students in age group (13 –

(3 * 48) – (60 + 50) = 34 16) years in school C:

Since, the average of the number of students in (2 * 90) – 100 = 80

age group (9 – 12) years in schools A, B and C And the average of the number of students in

is 70. age group (13 – 16) years in school A and those

So, the number of students in age group (9 – 12) in school C:

years in school C:

Click Here For Bundle PDF Course | support@guidely.in Page 10 of 18


Bank Po Mains PDF Course 2024
Quantitative Aptitude Day -17 (Eng)

So, Q is not true. So, the time, in which A alone can finish the
From R: work with 125% of its original efficiency =
The difference between the total number of 9 to = 12t hours
16 years old students in school A and those in And the time, in which G alone can finish the
school B = (70 + 100) – (90 + 70) = 10 work with of its original efficiency =
So, R is not true. = 48t hours
Hence, only P is true. So,

Directions (9-12):
The ratio of A’s efficiency to B’s efficiency = A: B
= 1: 0.75 = 4: 3 t=1
Since, the ratio of B’s efficiency to C’s efficiency The time, in which A alone can complete the
= B: C = 1: 1.25 = 4: 5 work = 15 hours
So, the ratio of A’s efficiency to C’s efficiency = The time, in which B alone can complete the
(4 * 4): (3 * 5) = 16: 15 work = = 20 hours
Since, the ratio of C’s efficiency to D’s efficiency The time, in which C alone can complete the
= C: D = 1: 2 work = = 16 hours
So, the ratio of A’s efficiency to D’s efficiency = The time, in which D alone can complete the
(16 * 1): (15 * 2) = 16: 30 = 8: 15 work = = 8 hours
Since, the ratio of D’s efficiency to E’s efficiency The time, in which E alone can complete the
= D: E = 1: 0.25 = 4: 1 work = = 32 hours
So, the ratio of A’s efficiency to E’s efficiency =
The time, in which F alone can complete the
(8 * 4): (15 * 1) = 32: 15
work = = 32 hours
Since, the ratio of E’s efficiency to F’s efficiency
The time, in which G alone can complete the
= E: F = 1: 1
work = = 64 hours
So, the ratio of A’s efficiency to F’s efficiency =
9) Answer: C
32: 15
The time, in which B alone can complete the
Since, the ratio of F’s efficiency to G’s efficiency
work with 50% of its original efficiency =
= F: G = 1: 0.5 = 2: 1
= 40 hours
So, the ratio of A’s efficiency G’s efficiency = (32
The time, in which C alone can complete the
* 2): (15 * 1) = 64: 15
work with 50% of its original efficiency =
Let the time, in which A alone and G alone can
= 32 hours
complete the work be ‘15t’ hours and ‘64t’ hours.

Click Here For Bundle PDF Course | support@guidely.in Page 11 of 18


Bank Po Mains PDF Course 2024
Quantitative Aptitude Day -17 (Eng)

The time, in which D alone can complete the Hence, only Q is true.
work with 200% of its original efficiency =
= 4 hours 11) Answer: B
The time, in which E alone can complete the The ratio of B’s efficiency to D’s efficiency = 8 :
work with 200% of its original efficiency = 20 = 2 : 5
= 16 hours So, the wages received by B = =

Part of completed by B, C, D and E together in 1 ₹3600


hour: Since, the ratio of B’s efficiency to A’s efficiency
= 15: 20 = 3: 4
So, the wages received by A = =
So, the time, in which B, C, D and E together will
₹4800
complete the work with their new efficiencies:
Since, the ratio of B’s efficiency to G’s efficiency
= 64: 20 = 16: 5
So, the wages received by G = =
10) Answer: A
₹1125
From P:
The required difference = 4800 – 1125 = ₹3675
Part of work, which can be completed by D and
G together in 1 hour:
12) Answer: E
Part of work, which can be completed by A and
So, the time, in which D and G together can B together in 1 hour:
complete the work:

So, the time, in which A and B together can


So, P is not true. complete the work = hours
From Q: Part of work, which can be completed by C and
Average of time, in which A alone, C alone and D together in 1 hour:
D alone can complete the work:

So, the time, in which C and D together can


So, Q is true. complete the work = hours
From R: Part of work, which can be completed by F and
Ratio of C’s efficiency to F’s efficiency = 32: 16 = G together in 1 hour:
2: 1
So, R is not true.

Click Here For Bundle PDF Course | support@guidely.in Page 12 of 18


Bank Po Mains PDF Course 2024
Quantitative Aptitude Day -17 (Eng)

So, the time, in which F and G together can 120 ÷ 5 = 24


complete the work = hours In series II:
Required ratio: (Y + 3) = 90 ÷ 5 = 18
Y = 15
From equation (1):
X=8
13) Answer: C
Now, the series I: 4, 8, 24, 120, 840
The given series:
And the series II: 3, 6, 18, 90, 630
4, (x + y), 19, (8x – 1)
From P:
Since,
The 1st term of series II = 3
(x + y) = 2 * 4 + 1
The 2nd term of series I = 8
x + y = 9 ----------(1)
So, the required percentage = = 37.5%
And,
(8x – 1) = 2 * 19 + 1 So, P is not true.

x=5 From Q:

From equation (1): X = 8, Y = 15

y=4 So, X is of Y.

The given series = 4, 9, 19, 39 So, Q is true.

And the 5th term of the series = 2 * 39 + 1 = 79 From R:

Now, the 1st term of the new series = 4 * 4 – 5 = The 4th term of series I = 120

11 The 3rd term of series II = 18

The 2nd term of the new series = 2 * 11 + 1 = 23 The required average = = 69

The 3rd term of the new series = 2 * 23 + 1 = 47 So, R is not true.


The 4th term of the new series = 2 * 47 + 1 = 95 Hence, only Q is true.
And the 5th term of the new series = 2 * 95 + 1 =
191 15) Answer: D
So, the required average = = 135 Logic in the given series:
16 + 14 = 30

14) Answer: A 30 + 28 = 58

Since, both the series follow the same pattern: 58 + 56 = 114

So, in series II: 114 + 112 = 226

630 ÷ 7 = 90 226 + 224 = 450

And, in series I: 450 + 448 = 898

XY = 840 ÷ 7 = 120 ------------(1) Since, 5X = 450

Click Here For Bundle PDF Course | support@guidely.in Page 13 of 18


Bank Po Mains PDF Course 2024
Quantitative Aptitude Day -17 (Eng)

X = 90 Total cost of mixture C = 6500 – 4400 = ₹2100


And, (10X – Y) = 898 Total cost of mixture D = 7800 – 6500 = ₹1300
Y=2 Total cost of mixture E = 11000 – 7800 = ₹3200
So, the 1st term of the new series = = 11 Let quantity of type P and type Q tea in mixture
The 2nd term of the new series = 11 * 2 + 1 = 23 E is ‘e’ and ‘28x – e’ respectively.
The 3rd term of the new series = 23 * 2 + 1 = 47 According to the question:
From option (a): e * 40 + (28x – e) * 60 = 3200
2e + 84x – 3e = 160
e = 84x – 160 ……….. (1)
So, option (a) is correct.
131.25% of [e * 40 + (28x – e) * 60] = 28x * 60
From option (b):
21 * [e * 40 + (28x – e) * 60] = 16 * 28x * 60
2e + 84x – 3e = 64x
So, option (b) is correct. e = 20x …… (2)
From option (c): From (1) and (2):
20x = 84x – 160
So, option (c) is not correct. 64x = 160
Hence, both options (a) and (b) are correct. x = 2.5
e = 20x = 50
Directions (16-19):
Percent of total quantity of mixture A out of total
quantity = 16%
Percent of total quantity of mixture B out of total
quantity = 40 – 16 = 24%
Percent of total quantity of mixture C out of total
quantity = 60 – 40 = 20%
Percent of total quantity of mixture D out of total
quantity = 72 – 60= 12%
Let quantity of type P tea in mixtures A, B, C,
Percent of total quantity of mixture E out of total
and D is ‘a’, ‘b’, ‘c’, and ‘d’ respectively.
quantity = 100 – 72 = 28%
Total cost of mixture A = a * 40 + (40 – a) * 60 =
Let total quantity of mixture A, B, C, D, and E is
1800
‘16x’, ‘24x’, ‘20x’, ‘12x’, and ‘28x’ respectively.
a = 30
Total cost of mixture A = ₹1800
Total cost of mixture B = b * 40 + (60 – b) * 60 =
Total cost of mixture B = 4400 – 1800 = ₹2600
2600

Click Here For Bundle PDF Course | support@guidely.in Page 14 of 18


Bank Po Mains PDF Course 2024
Quantitative Aptitude Day -17 (Eng)

b = 50 Ratio of type P to type Q tea in mixture F = 4: 1


Total cost of mixture C = c * 40 + (50 – c) * 60 = Since loss percent incurred after selling mixture
2100 G is 5.88%.
c = 45 Let ratio of type P to type Q tea in mixture G is g:
Total cost of mixture D = d * 40 + (30 – d) * 60 = 1.
1300 According to the question:
d = 25
Total cost of mixture E = e * 40 + (70 – e) * 60 =
16 * [g * 40 + 1 * 60] = 17 * (g + 1) * 40
3200
2 * [g * 40 + 1 * 60] = 17 * (g + 1) * 5
80g + 120 = 85g + 85
5g = 35
g=7
Ratio of type P to type Q tea in mixture G = g: 1
= 7: 1
16) Answer: D
Ratio of type P to type Q in the mixture F = 4: 1
Let ratio of type P to type Q tea in mixture F is f: Let initial quantity of type P and type Q tea in
1. mixture F are ‘4f’ and ‘f’ respectively.
Part of type P tea in mixture A = 3/4 According to the question:
Part of type P tea in mixture D = 5/6 (4f – 15) – (f + 15) = 15
By the rule of alligation: 3f – 30 = 15
3f = 45
f = 15
Final quantity of type P tea in mixture F = 4f – 15
= 45 kg
Final quantity of type Q tea in mixture F = f + 15
= 30 kg
Cost price of final quantity of mixture F = 45 * 40
+ 30 * 60 = 1800 + 1800 = ₹3600
Selling price of final quantity of mixture F when
sold at ₹54 per kg = (45 + 30) * 54 = 75 * 54 =
4f + 4 = 5f ₹4050
f=4 Profit amount = 4050 – 3600 = ₹450

Click Here For Bundle PDF Course | support@guidely.in Page 15 of 18


Bank Po Mains PDF Course 2024
Quantitative Aptitude Day -17 (Eng)

Profit percent (450/3600) * 100 Hence, value of x + y


= 12.5% = 80 + 100
= 180
17) Answer: A
Total cost price of mixture D = 25 * 40 + 5 * 60 = 18) Answer: D
1000 + 300 = ₹1300 Per kg cost price of mixture A = (30 * 40 + 10 *
Per kg cost of mixture D = 1300/(25 + 5) = 60)/(30 + 10) = 1800/40 = ₹45
1300/30 = ₹130/3 Per kg cost price of mixture C = (45 * 40 + 5 *
Total cost of mixture G = 7x * 40 + x * 60 = 280x 60)/(45 + 5) = 2100/50 = ₹42
+ 60x = 340x Per kg cost price of final mixture when sold at
Per kg cost of mixture G = 340x/(7x + x) = ₹46.53 at 10% profit = 46.53 * (100/110) = ₹42.3
340x/8x = ₹85/2 Per kg cost price of final mixture when sold at
Per kg cost of final mixture = ₹42(8/11) = ₹48.4 at 10% profit = 48.4 * (100/110) = ₹44
₹470/11 Case 1: When cost price of final mixture is
By the rule of alligation: ₹42.3.
130/3 85/2 By the rule of alligation:
470/11 Mixture A Mixture C
Ratio in which mixtures D and G are mixed = 45 42
42.3
Ratio in which mixtures A and C are mixed = (42
– 42.3): (42.3 – 45) = 0.3: 2.7 = 1: 9
Percent of mixture A in the final mixture = (1/10)
So, the ratio in which mixture D and G are mixed * 100 = 10%
= 3: 8 Case 1: When cost price of final mixture is ₹44.
Hence, quantity of mixture G mixed in the By the rule of alligation:
mixture = (25 + 5) * (8/3) Mixture A Mixture C
x kg = 80 kg 45 42
x = 80 44
Now, Ratio in which mixtures A and C are mixed = (42
√(y2 – x2) = 60 – 44): (44 – 45) = 2: 1
√(y2 – 802) = 60 Percent of mixture A in the final mixture = (2/3) *
y2 – 6400 = 3600 100 = 66.67%
y2 = 10000
y = 100

Click Here For Bundle PDF Course | support@guidely.in Page 16 of 18


Bank Po Mains PDF Course 2024
Quantitative Aptitude Day -17 (Eng)

So, in order to make a mixture whose selling = 35 kg


price lies between ₹46.53 and ₹48.4, the
quantity of mixture A must lie between 10% and 20) Answer: B
66.67% of total quantity. P: x2 – 12x + 35 = 0
In statement I:Percent of mixture A in the final x2 – 7x – 5x + 35 = 0
mixture = (1/5) * 100 = 20% x(x – 7) – 5(x – 7) = 0
In statement II:Percent of mixture A in the final (x – 7)(x – 5) = 0
mixture = (2/11) * 100 = 18(2/11)% x = 7 and 5
In statement III:Percent of mixture A in the final Q: x2 – 19x + 88 = 0
mixture = (3/10) * 100 = 30% x2 – 8x – 11x + 88 = 0
Hence, all the 3 statements are TRUE. x(x – 8) – 11(x – 8) = 0
(x – 8)(x – 11) = 0
19) Answer: E x = 8 and 11
Per kg cost price of mixture A = ₹45 A: y2 – 14y + 33 = 0
Per kg cost price of mixture B = (50 * 40 + 10 * y2 – 3y – 11y + 33 = 0
60)/(50 + 10) = 2600/60 = ₹43.33 y(y – 3) – 11(y – 3) 0
Per kg cost price of mixture C = ₹42 (y – 3) (y – 11) = 0
Per kg cost price of mixture D = (25 * 40 + 5 * y = 3 and 11
60)/(25 + 5) = 1300/30 = ₹43.33 B: y2 + 9y – 52 = 0
Per kg cost price of mixture E = (50 * 40 + 20 * y2+ 13y – 4y – 52 = 0
60)/(50 + 20) = 3200/70 = ₹45.71 y(y + 13) – 4(y + 13) 0
So, mixtures X and Y will be mixtures B and D. (y + 13) (y – 4) = 0
Total quantity of type P tea in mixture M = 50 + y = 4 and -13
25 = 75 kg
Total quantity of type Q tea in mixture M = 10 + 5
= 15 kg
Total quantity of type P tea when 55 kg of
mixture N is mixed with mixture M = 75 + 55 *
(3/11) = 75 + 15 = 90 kg
Total quantity of type Q tea when 55 kg of
mixture N is mixed with mixture M = 15 + 55 *
(8/11) = 15 + 40 = 55 kg
Required difference = 90 – 55

Click Here For Bundle PDF Course | support@guidely.in Page 17 of 18


Bank Po Mains PDF Course 2024
Quantitative Aptitude Day -17 (Eng)

Hence, the statements PB and QB are TRUE.

Click Here For Bundle PDF Course | support@guidely.in Page 18 of 18


Bank Po Mains PDF Course 2024
English Day - 17

English Language
Directions (1-5) : Swap the words given in bold (if I. The GDP has seen a improve (A) upward
necessary) to make the given sentences trend, reflecting a steady growth in economic
meaningful and correct. activity and indicating (B) positive momentum for
1) the overall economy.
I. The furniture (A) was so soft and luxurious, it II. The country has to gradual (I) on exports by
felt like a gentle appeal (B) against my skin. implementing strategic trade policies, diversifying
II. The fabric (I) cushion is selected based on its (II) its export markets, and enhancing
durability, comfort, and aesthetic embrace (II) to competitiveness.
create the perfect balance of style and a) A-I
functionality. b) B-II
a) A-I c) Both a and b
b) B-II d) A-II
c) Both a and b e) No changes required
d) A-II
e) No changes required 4)
I. The baby was asleep, cradled (A) in the
2) exhaustion (B) of the crib, breathing softly and
I. Worshipping God is like a deeply (A) spiritual peacefully as dreams danced through its
journey, a connection to the divine that fills the innocent slumber.
heart with reverence and gratitude (B). II. The online session was so tiring, leaving me
II. The professor asked the students to be drained (I) and longing for a break from the
actively (I) engaged in the discussion, fostering screen-induced warmth (II).
(II) an environment of critical thinking and a) A-I
collaborative learning. b) B-II
a) A-I c) A-II
b) B-II d) Both a and b
c) Both a and b e) No changes required
d) B-I
e) No changes required 5)
I. The book has 15 chapters, each expressed (A)
3) into a different aspect of the protagonist's
transformative journey, creating a rich (B) and
multifaceted narrative.

Click Here For Bundle PDF Course | support@guidely.in Page 1 of 10


Bank Po Mains PDF Course 2024
English Day - 17

II.The author has delving (I) her profound c) Bihar, a state in eastern India, holds a
insights on human nature and the complexities of significant place in the country's history and
relationships through eloquent (II) prose that heritage also known for its ancient roots
resonates deeply with readers' emotions and d) While many factors contribute to the dynamics
experiences. of the tigers present in a region
a) A-I e) None of these
b) B-I
c) B-II 7) The Digital Data Protection Bill, 2023, was
d) Both a and c passed in the Lok Sabha on Monday and will
e) No changes required now have to be cleared by the Rajya Sabha. The
fresh iteration, which has undergone a few
Directions (6-10) : In each of these questions, a drafts, seems to have incorporated suggestions
paragraph is given that has a blank in it. Out of made to its 2022 version. Although it is not clear
the given options, only one sentence fits in with what the submissions were as the consultation
the context of the paragraph. Select that as your process was not brought to light by the
answer. government. __________.
6) Madhya Pradesh, for the second time in eight a) The highlight of the Bill is the provision that
years, reported 785 tigers, or about a fifth of the personal data of an individual may be processed
national count. The State reported a 50% rise in without the consent of the data principal
tigers since the last census, a figure bettered b) Data protection and privacy are essential
only by Bihar which has less than 10% of M.P’s rights in the digital age, ensuring that individuals
tigers______________________. M.P. over the have control over their personal information
years has perfected the approach of actively c) The Lok Sabha is the lower house of the
moving both tigers, as well as their prey, within Parliament of India, consisting of elected
the State to balance predator and prey representatives from various constituencies
population. across the country
a) Tigers, majestic and powerful predators, roam d) The highlight of the Bill is the provision that
the dense jungles with their distinctive orange personal data of an individual may be processed
coats and dark stripes only after the consent of the data principal
b) Madhya Pradesh, located in the heart of India, e) None of these
boasts a rich tapestry of history, culture, and
natural beauty 8) For the second time in three months,
Pakistan’s former Prime Minister Imran Khan has
been arrested.This arrest was after a conviction

Click Here For Bundle PDF Course | support@guidely.in Page 2 of 10


Bank Po Mains PDF Course 2024
English Day - 17

in a case involving the misappropriation of official c) The spacecraft entered lunar orbit on 5 August
gifts meant for the ‘Toshakhana’. He was last 2023
arrested on May 9 in the ‘Al Qadir’ trust case d) The primary issue was the five engines which
charge sheet.__________________. It means that were used to reduce the velocity.
he is now disqualified from Parliament for the e) None of these
next five years, and will not be able to stand for
election in polls due by November. 10) The Supreme Court on August 10
a) The latest conviction, which carries a sentence announced the launch of portal 'Suswagatam'.
of three years This would enable advocates, visitors, interns
b) Established in 1974, the Tosha­khana is a and others to register themselves online and get
department which stores precious gifts given to e-passes to enter the apex court. Suswagatam'
rulers is a web-based and mobile-friendly application
c) He transitioned into politics and became Prime that allows users to register themselves online
Minister after his party won the general elections and request for e-passes for various purposes
in 2018 such as attending court hearings, meeting
d) Conviction refers to a formal declaration by a advocates. ____________________
court of law that someone has been found guilty a) The portal issues electronic passes to the
of a criminal offence after a legal trial or people
proceeding b) The Supreme Court of India is the country's
e) None of these highest judicial court
c) The 'Suswagatam' portal was tested as a pilot
9) Indian Space Research Organisation (ISRO) project from July 25, 2023
has released two images taken by the d) Many such portals are available that help the
Chandrayaan-3 spacecraft._________________. functioning of courts easier
The photo was taken on July 14, when the e) None of these
Chandrayaan-3 mission was launched from the
Satish Dhawan Space Centre in Sriharikota.The Directions (11-15) : Rearrange the following five
second image is of the Moon captured by the sentences (A), (B), (C), (D) and (E) in the proper
Lander Horizontal Velocity Camera (LHVC) on sequence to form a coherent paragraph and then
board the spacecraft on August 6. answer the questions given below.
a) Chandrayaan-3 was launched on 14 July 2023 (A) And while monetary authorities are right in
from Satish Dhawan Space Centre laying the onus on the government for ensuring
b) The first image is of the Earth as viewed by timely supply side interventions to ‘limit the
the Lander Imager Camera severity and duration of such shocks’, a broader

Click Here For Bundle PDF Course | support@guidely.in Page 3 of 10


Bank Po Mains PDF Course 2024
English Day - 17

unmooring of price stability will undermine c) C


macroeconomic stability and growth. d) D
(B) RBI Governor Shaktikanta Das e) No error
acknowledged that while/A the likely short-term
nature of these shocks allowed for policymakers 13) Fill the given blank in sentence D.
to/B look a couple past of high inflation prints, a) remained
frequently/C recurring food price shocks risk b) unchanged
destabilising inflation expectations/D. c) altered
(C) Barely two months after projecting d) changed
Consumer Price Index-based inflation to average e) unnoticed
5.1% over the current fiscal year ending in March
2024, the MPC has raised its forecast for the 14) Replace the bold word in sentence E to
annual average by 30 basis points to 5.4%. make it correct and meaningful.
(D) The latest decision by the Reserve Bank of a) expansion
India’s Monetary Policy Committee (MPC) to b) climb
leave its policy rate _______ is a calculated risk c) growth
taken by the rate setting panel, especially when d) spike
viewed in light of the upward revision of its e) addition
inflation forecast.
(E) Conceding that a improvement in tomato 15) Which of the following is the correct
prices had contributed to a shock that had forced sequence of rearrangement ?
a revision in the headline inflation projection for a) eadcb
the July-September quarter by a ‘substantial’ 100 b) ceadb
basis points to 6.2%. c) bceda
11) Meaning of the word ‘Onus’ in sentence A. d) abdec
a) something that is one's duty or responsibility. e) dceba
b) onus is when all people come together
c) an official or person in high position Directions (16-20) : In the statements given
d) praising someone for his/her act below, a blank has been given. This needs to be
e) one’s unwanted commitment filled up using a correct word from the options
mentioned below. The statement so formed must
12) Spot the erroneous part in sentence B. be contextually and grammatically sound.
a) A 16) The curbs were to kick in with immediate
b) B effect, rattling the entire supply chain and ______

Click Here For Bundle PDF Course | support@guidely.in Page 4 of 10


Bank Po Mains PDF Course 2024
English Day - 17

fears of shortages and price surges, especially b) conquer


ahead of the festive season. c) construct
a) enhancing d) consensus
b) illuminating e) convince
c) igniting
d) deciding 19) All the claims of scientists having found a
e) running room-temperature superconductor so far have
failed to _______ independent scrutiny.
17) The government sought to suggest this was a) withstand
primarily driven by security concerns and b) standby
imported devices could be used for__________, c) stick with
just as mobile phones could have spyware. d) give up
a) survival e) stand up
b) surveillance
c) surpassing 20) A Supreme Court judgement in March said
d) surveying the panel should _______ the Prime Minister, the
e) surrendering Leader of the Opposition in Lok Sabha and the
Chief Justice of India.
18) The main goal of Ukraine and its western a) compromise
partners was to build ________ among major b) apprise
powers, especially in the Global South, on c) reprise
working towards a fair and durable peace. d) supervise
a) consult e) comprise

Click Here For Bundle PDF Course | support@guidely.in Page 5 of 10


Bank Po Mains PDF Course 2024
English Day - 17

Click Here to Get the Detailed Video Solution for the above given Questions
Or Scan the QR Code to Get the Detailed Video Solutions

Answer Key with Explanation


1) Answer: C The furniture cushion is selected based on its
Both A-I and B-II need to be swapped to make durability, comfort, and aesthetic appeal to
the sentences I and II correct and meaningful. create the perfect balance of style and
Swap fabric and furniture, similarly swap appeal functionality.
and embrace.
2) Answer: E
Furniture is an object that cannot be worn, All the labelled words are correct and
whereas fabric refers to the material and can be appropriate. Hence, no changes required.
felt on the skin - hence replacing fabric and
furniture. 3) Answer: A
Only words A and I need swapping. Improve and
Embrace - to put your arms around somebody gradual and placed inappropriately and must be
as a sign of love, happiness interchanged to make the given two sentences
Appeal - to be attractive or interesting to correct and meaningful. The other two words
somebody indicating and diversifying are correct.
In the sentence I -’ felt like gentle embrace’ In sentence I - 'gradual upward trend’ would be
would be the correct phrase correct - gradual(slow)
In the sentence II - ‘aesthetic appeal’ would be In sentence II - ‘the country has to improve on
the correct phrase exports’ would be correct
The sentences: The sentences:
The fabric was so soft and luxurious, it felt like a
gentle embrace against my skin.

Click Here For Bundle PDF Course | support@guidely.in Page 6 of 10


Bank Po Mains PDF Course 2024
English Day - 17

The GDP has seen a gradual upward trend, The sentences:


reflecting a steady growth in economic activity The book has 15 chapters, each delving into a
and indicating positive momentum for the overall different aspect of the protagonist's
economy. transformative journey, creating a rich and
The country has to improve on exports by multifaceted narrative.
implementing strategic trade policies, The author has expressed her profound insights
diversifying its export markets, and enhancing on human nature and the complexities of
competitiveness. relationships through eloquent prose that
resonates deeply with readers' emotions and
4) Answer: B experiences.
The words exhaustion and warmth must be
interchanged to make the sentences I and II 6) Answer: D
correct and meaningful. Sentence in option d fits the blank perfectly as
Exhaustion - a state of being incredibly tired the passage deals with the conservation of
Warmth - a fairly high temperature or the effect tigers, talking about tigers in general or about the
created by this, especially when it is pleasant involved states is not necessary here.
In sentence I - baby fell asleep by the warmth of D is more appropriate and suitable here.
the crib A - generic about tigers (not relevant)
In sentence II - the session was tiring and B - about Madhya Pradesh (not relevant)
exhausting C - about Bihar (not relevant)
The sentences: The only relevant sentence is D.
The baby was asleep, cradled in the warmth of
the crib, breathing softly and peacefully as 7) Answer: D
dreams danced through its innocent slumber. D is the correct sentence that is talking about the
The online session was so tiring, leaving me data protection bill and privacy.
drained and longing for a break from the screen- A is incorrect because it is well known that
induced exhaustion. usage of personal information without consent is
illegal.
5) Answer: A B is a general statement about data protection
Interchange ‘expressed’ and ‘delving’ to make and privacy
the sentence correct and meaningful. C conveys general information about the Lok
Delve - to search inside something sabha which is also irrelevant to the context of
the passage.

Click Here For Bundle PDF Course | support@guidely.in Page 7 of 10


Bank Po Mains PDF Course 2024
English Day - 17

8) Answer: A B - generic sentence about supreme court


Among the given statements the sentence which D - does not fit the context
completes the given passage is ‘a’ as it talks
about the recent conviction, the sentence which 11) Answer: A
follows the blank also talks about the conviction Onus means - one’s duty or responsibility
and punishment.
B - Toshakhanna is somehow related to the 12) Answer: C
context of the passage but the blank cannot be The error is in part c of the given sentence.
filled with this sentence as there is no continuity. The phrase ‘look a couple past of’ is completely
C - the sentence is about Imran Khan but in the wrong instead it must be ‘look past a couple of’.
passage given it is irrelevant to talk about his Look past - To see potential beyond obvious
D - definition of conviction is not really needed in flaws; to consider something more than
this context something else.
The sentence : RBI Governor Shaktikanta Das
9) Answer: B acknowledged that while the likely short-term
B is the right statement/sentence to fill the given nature of these shocks allowed for policymakers
blank because the sentence following the blank to look past a couple of high inflation prints,
talks about a photo/image which must be a frequently recurring food price shocks risk
continuation. So, B is the correct sentence when destabilising inflation expectations.
compared to the other given options.
A - launch date of chandrayaan 3 is 13) Answer: B
comparatively less relevant than sentence b ‘Unchanged’ is the correct word that fits the
C - also irrelevant because the passage talks given blank.
about images that ISRO released ‘To leave something’ conveys a meaning - to
D - talks about some issues related to the leave something undisturbed/to leave it the
spacecraft launch same way it is
Changed and altered are wrong, unnoticed and
10) Answer: C remained are also incorrect and does not convey
C is the correct sentence that completes the a proper meaning
passage as it revolves around the main idea of The sentence:
the passage - Suswagatam portal. The latest decision by the Reserve Bank of
A - redundant information as the passage India’s Monetary Policy Committee (MPC) to
already spoke about the same leave its policy rate unchanged is a calculated

Click Here For Bundle PDF Course | support@guidely.in Page 8 of 10


Bank Po Mains PDF Course 2024
English Day - 17

risk taken by the rate setting panel, especially 16) Answer: C


when viewed in light of the upward revision of its Out of the given options, igniting is the best word
inflation forecast. that fits the blank without changing the actual
meaning of the sentence.
14) Answer: D Igniting - to start burning or to make something
Improvement is an incorrect word and does not start burning
fit the given sentence. Enhancing - to improve something or to make
Among the given words ‘spike’ is the correct something look better (enhance fear would
word that fits the sentence without altering the sound wrong)
original meaning. Illuminating - helping to explain something or
Spike - A spike is a sudden and large move in make something clear (irrelevant)
the price of an asset Deciding and running are also inappropriate
The sentence : Conceding that a spike in tomato words and hence discarded
prices had contributed to a shock that had forced The sentence: The curbs were to kick in with
a revision in the headline inflation projection for immediate effect, rattling the entire supply chain
the July-September quarter by a ‘substantial’ and igniting fears of shortages and price surges,
100 basis points to 6.2%. especially ahead of the festive season.

15) Answer: E 17) Answer: B


The correct sequence of rearrangement is According to the given context ‘surveillance’ is
DCEBA. the correct word that fits the blank correctly.
D is the introductory sentence as it has got the As the context is related to security, surveillance
information on the latest decision of RBI. D is would be the best fit.
followed by C which conveys information The other given words have a different meaning
regarding the RBI’s forecast in the midst of and hence are not suitable.
which the RBI has left the policy rate unchanged. Survival - the act or fact of living
After DC, comes E which cites the reason for the Surveillance - the careful watching of somebody
inflation project - price of tomatoes. E is followed who may have done something wrong
by B which talks about the price shocks in detail Surpass - to do something better than
and what it leads to. Last sentence would be A, somebody/something else or better than
which adds more to the price shocks and expected
inflation projection.

Click Here For Bundle PDF Course | support@guidely.in Page 9 of 10


Bank Po Mains PDF Course 2024
English Day - 17

Survey - the process of collecting, analysing and Withstand is the correct word that fits the blank
interpreting data from many individuals making the sentence complete and meaningful.
Surrender - to stop fighting and admit that you Withstand - to be strong enough not to break,
have lost/to give somebody/something to give up
somebody else Standby - a thing or person that can be used if
The sentence: needed
The government sought to suggest this was Stick with - to continue using or doing
primarily driven by security concerns and Give up - to cease doing
imported devices could be used for surveillance, Stand up - to be or become vertical
just as mobile phones could have spyware. The sentence:
All the claims of scientists having found a room-
18) Answer: D temperature superconductor so far have failed to
Consensus is the right word that fits the blank. withstand independent scrutiny.
Consensus - agreement among a group of
people (build agreement would be the correct 20) Answer: E
phrase as it conveys a proper meaning) Comprise is the correct word that fits the given
Conquer - to take control (irrelevant) blank.
Consult - to ask somebody for some information Other words are discarded because of their
or advice (does not fit because different meaning as they don’t fit the context of the
meaning) sentence.
Construct - to build (not relevant) Compromise - an agreement
Convince - to succeed in making someone Apprise - inform or tell
believe something (does not fit the blank Reprise - repeat
properly) Supervise - to watch somebody/something to
The sentence: make sure that work is being done properly
The main goal of Ukraine and its western Comprise - to consist of
partners was to build consensus among major The sentence:
powers, especially in the Global South, on A Supreme Court judgement in March said the
working towards a fair and durable peace. panel should comprise the Prime Minister, the
Leader of the Opposition in Lok Sabha and the
19) Answer: A Chief Justice of India.

Click Here For Bundle PDF Course | support@guidely.in Page 10 of 10


Bank Po Mains PDF Course 2024
Reasoning Day -18 (Eng)

Reasoning Aptitude
Directions (1-5): Study the following information
carefully and answer the given questions.
Nine different mobile shops – A, B, D, G, I, J, M,
N and O run on different floors of a nine storey
building where the lowermost floor is numbered
one and the floor immediately above it is
a) I-A, II-D, III-B, IV-C
numbered two and so on. No two shops were run
b) I-B, II-C, III-A, IV-D
on the same floor.
c) I-B, II-A, III-C, IV-D
Note 1: The shop names which start with a vowel
d) I-A, II-B, III-D, IV-C
were not run on an odd-numbered floor of the
e) I-B, II-D, III-A, IV-C
building.
Note 2: The consecutive alphabetically named
3. Which among the following shop runs on the
shops were not run on the adjacent floors of the
prime numbered floor as per the given
building.
arrangement?
G runs below fourth floor. Only three shops run
I. The shop which runs two floors below I.
between G and D. I runs either three floors below
II. The shop which runs immediately above A.
or three floors above D. The number of floors
III. The shop which runs on the adjacent floor of
above I is two less than the number of floors
N.
below M. N neither runs adjacent floor of D nor
a) Only III
runs below third floor. At most two shops run
b) Only I and II
between N and O. A runs below J. The number
c) Only I and III
of floors between J and A is three less than the
d) Only II
number of floors between D and B.
e) Only II and III
1. N runs __ floors below D and __ floors above
O respectively?
4. The number of floors between B and N is
a) One; Three
same as the number of floors between __ and __
b) Three; Three
a) M and I
c) Three; Four
b) A and O
d) Two; Three
c) Both a and b
e) One; Two
d) D and I
e) Both a and d
2. Match the list 1 with list 2

Click Here For Bundle PDF Course | support@guidely.in Page 1 of 10


Bank Po Mains PDF Course 2024
Reasoning Day -18 (Eng)

5. The difference between the floor numbers of G c) Third to the left; Daughter-in-law
and A is equal to the difference between the floor d) Fourth to the right; Daughter-in-law
numbers of __ and __. e) Third to the left; Mother
a) M and N
b) D and G 7. The number of persons sitting between __ and
c) B and N __ is two less than the number of persons sitting
d) J and I between __ and __, when counted from the left of
e) None of these the first persons in the given pairs respectively.
a) H and Husband of N; Son-in-law of P and N
Directions (6-10): Study the following information b) Mother-in-law of A and W; N and Son of H
carefully and answer the given questions. c) Husband of H and A; K and P
Eight persons from the same family – A, C, F, H, d) Both a and b
K, N, P, and W are sitting around a circular table e) Both a and c
facing the centre with equal distance between
adjacent persons. Four married couples are 8.__ sits second to the left of the mother of W,
there in the family. who sits ___ to the left of the Mother of C?
Note: Neither the couples nor the persons of the a) A; Second
same gender are sitting adjacent to each other. b) K; Fifth
K sits third to the right of W’s father. C is the only c) P; Fourth
child of K’s brother. Only one person sits d) A; Third
between K and C’s grandmother(either from left e) K; Fourth
or right). A is the brother-in-law of W and sits
adjacent to C’s grandmother. N sits immediate 9. Which of the following pair of persons are
left of the one who sits opposite to A, who has no married couples?
siblings. The number of persons sitting between
H and C’s father is one more than the number of
persons sitting between C’s father and C, when
counted from the right of H and left of C. H is the
parent of C and daughter-in-law of F. P sits
second to the left of K’s brother.
6. What is the position of W with respect to F and
How F is related to H’s sister-in-law?
a) Third to the right; Mother-in-law
b) Second to the left; Mother

Click Here For Bundle PDF Course | support@guidely.in Page 2 of 10


Bank Po Mains PDF Course 2024
Reasoning Day -18 (Eng)

Rickshaw A travels for 6m towards the south


direction from the rickshaw stand to reach
stoppage 1. Then turns towards the left and
travels for 8m to reach stoppage 2. Then, from
stoppage 2 again turns towards the left and
travels for 4m to reach stoppage 3, from there it
turns towards the right and travels for 9m to
reach stoppage 4.
Rickshaw B travels for 4m towards the east
direction from the rickshaw stand to reach
stoppage 5, from there it takes a left turn and
a) I-C, II-D, III-B, IV-C travels for 13m to reach stoppage 6. Then turns
b) I-C, II-A, III-A, IV-D towards the right and travels for 6m to reach
c) I-B, II-A, III-C, IV-D stoppage 7. Stoppage 8 is 7m south of stoppage
d) I-D, II-B, III-D, IV-C 7.
e) I-C, II-A, III-D, IV-B Rickshaw C travels for 9m towards the north
direction from the rickshaw stand to reach
10. In which among the following pair of persons stoppage 9. Stoppage 10 is east of stoppage 9
the first person sits second to the left of the and south of stoppage 7. Stoppage 11 is
second person? northeast of stoppage 10 and 7m east of
a) The one who faces C, A stoppage 7.
b) F, The one who sits immediate right of N These stoppages are assigned different names
c) The one who is the sister-in-law of K, N according to the below given conditions:
d) The one who is the mother of W, P 1). If the distance between the consecutive
e) None of these stoppages reached by the rickshaw is a multiple
of three, then “P” is named to the second
Directions (11-15): Study the following stoppage reached by the rickshaw (Consider
information carefully and answer the given rickshaw stand as one stoppage).
questions. 2). If the distance between the stoppage and the
There are three rickshaws i.e. rickshaw A, rickshaw stand is an even number (calculate as
rickshaw B, and rickshaw C starts travelling from total distance), then stoppage is named as “Q”,
a common rickshaw stand. Each rickshaw travel else the stoppage is named as “K”.
to some stoppages. 3). If the distance between the consecutive
stoppages reached by rickshaw is an odd prime

Click Here For Bundle PDF Course | support@guidely.in Page 3 of 10


Bank Po Mains PDF Course 2024
Reasoning Day -18 (Eng)

number, then “R” is named to the first stoppage a) 36m


reached by rickshaw. b) 25m
4). If the stoppage is in northeast or south c) 49m
direction from the rickshaw stand, then the d) 16m
stoppage is named as “M”. e) 64m
5). If the stoppage is in southeast or north
direction from the rickshaw stand, then the 14. Find the shortest distance between the
stoppage is named as “N”. stoppages S10 and S11.
6). If the distance between the consecutive a) √65m
stoppages is a root value, then only either b) √63m
conditions 4 or 5 follows. c) 2√31m
11. The total distance between which of the d) 2√27m
following two stoppages is greater than 15 with e) 2√11m
respect to the final arrangement? (If the rickshaw
stand is named as Y) 15. Which of the following pairs of stoppages,
a) Stoppages 1 and 4 the first stoppage is in northeast of the second
b) Stoppages 6 and Y stoppage with respect to the final arrangement?
c) Stoppages 5 and 11 a)Stoppages 7 and 5
d) Stoppages Y and 10 b) Stoppages 11 and 9
e) All of the above c) Stoppages 6 and 10
d) Both a and b
12. What is the shortest distance between the e) Both a and c
starting point to its final destination of rickshaw
B? Directions (16-20): Each of the questions below
a) √135 m consists of statements below it. You have to
b) 6√34 m decide whether the data provided in the
c) 2√34 m statements are sufficient to answer the question
d) 6√37 m and mark the appropriate option as answer.
e) 4√37 m 16. Seven vegetables viz. A, B, C, D, E, F, and G
were cooked on different days from Monday to
13. Find the square value of the difference Sunday of the same week. G was cooked after
between the sum of the total distance between Thursday. Only two vegetables were cooked
S5 and S8 and the sum of the total distance between G and C. Which of the following
between S5 and S4.

Click Here For Bundle PDF Course | support@guidely.in Page 4 of 10


Bank Po Mains PDF Course 2024
Reasoning Day -18 (Eng)

vegetables was cooked on the second day of the d) One person sits between B and G. A sits
week? adjacent to G. C sits third to the right of F.
a) B was cooked immediately after C. The e) H sits opposite to B. Two persons between H
number of vegetables cooked before B is one and F. G sits second to the right of F.
less than the number of vegetables cooked after
E. 18. Eight persons – P, Q, R, S, T, U, V and W
b) D was cooked two days before F. Only one live on eight different floors of an eight storey
vegetable was cooked between C and D. Only building where the bottommost floor is numbered
two vegetables were cooked between F and A. B 1 and the topmost floor is numbered 8. R lives on
was cooked neither before E nor immediately one of the even numbered floors. Only three
after E. floors between R and T. Who lives two floors
c) As many vegetables cooked before C as after above T?
E. F was cooked three days before E. A was a) As many floors above T as below P. Only two
cooked adjacent to either E or F. floors between P and W. Q lives four floors
d) A was cooked two days after C. Only three above W. S lives adjacent floor of neither T nor
vegetables were cooked between A and E. W.
Atmost one vegetable was cooked between E b) Q lives two floors below T. As many floors
and F. above Q as below W. Only one floor between W
e) D was cooked adjacent to neither C nor G. and P.
Only three vegetables were cooked between D c) The number of floors between R and T is one
and E. F was cooked five days after A. more than the number of floors between R and
W. Q lives two floors above W. Not less than
17. Eight persons - A, B, C, D, E, F, G and H are three floors between W and S.
sitting around a circular table facing the centre. E d) Only two floors between R and W. Q lives four
sits third to the left of D. B sits second to the left floors below W. As many floors above W as
of E. Who sits second to the left of F? below P. U lives adjacent to P.
a) Two persons sit between B and F. C sits e) Q lives three floors below R. The number of
adjacent to B. More than two persons sit floors between R and Q is one less than Q and
between C and G, when counted from the right of W. P lives immediately above W. Only two floors
C. between P and S.
b) As many persons sit between D and B as E
and H. G sits second to the right of A. 19. Six persons – E, G, J, K, T and W have
c) I sits adjacent to B. G sits two places away different number of Toys. Atmost one person has
from H. less number of toys than W. Who among the

Click Here For Bundle PDF Course | support@guidely.in Page 5 of 10


Bank Po Mains PDF Course 2024
Reasoning Day -18 (Eng)

following person has the second highest number 20. There are six members in the family. Two
of Toys? married members and each married couple have
a) J has more toys than W but less than T. K has at least one child. How Q is related to R?
more toys than J and T but less than E. G has a) R is the mother of Q. U is the son of R’s only
more toys than T but less than E. son.
b) T has less toys than J but more than K. G has b) U is the son of Q’s brother. M is the only son
more toys than K but does not have the second of E, who is the father-in-law of T.
highest number of toys. c) T is the only daughter-in-law of E. M is the
c) T has more toys than J but less than K. G has father of U, who is the nephew of Q.
more toys than K. d) U is the daughter of Q’s brother. M is the only
d) Either a or b son of E, who is married to R.
e) None of these e) None of these

Click Here to Get the Detailed Video Solution for the above given Questions
Or Scan the QR Code to Get the Detailed Video Solutions

Answer Key with Explanation


Directions (1-5):
1. Answer: D
2. Answer: E
3. Answer: B
4. Answer: C
5. Answer: A
Final arrangement

Click Here For Bundle PDF Course | support@guidely.in Page 6 of 10


Bank Po Mains PDF Course 2024
Reasoning Day -18 (Eng)

We have,
 G runs below fourth floor.
 Only three shops run between G and D.
 I runs either three floors below or three
floors above D.
From the above conditions, there are three
possibilities

Again we have,
 A runs below J.
 The number of floors between J and A is
three less than the number of floors
between D and B.
From the above condition Case-2 gets
eliminated because the number of floors
Again we have, between J and A is three less than the number
 The number of floors above I is two less of floors between D and B. Hence Case-1 shows
than the number of floors below M. the final answer.
 N neither runs adjacent floor of D nor runs
below third floor.
 At most two shops run between N and O.
From the above condition Case-3 gets
eliminated because the number of floors above I
is two less than the number of floors below M.

Directions (6-10):
6. Answer: E

Click Here For Bundle PDF Course | support@guidely.in Page 7 of 10


Bank Po Mains PDF Course 2024
Reasoning Day -18 (Eng)

7. Answer: D From the above conditions, there are three


8. Answer: B possibilities
9. Answer: E
10. Answer: C
Final arrangement

Again we have,
 The number of persons sitting between H
and C’s father is one more than the
number of persons sitting between C’s
father and C, when counted from the right
of H and left of C.
 H is the parent of C and daughter-in-law
We have,
of F.
 K sits third to the right of W’s father.
 P sits second to the left of K’s brother.
 C is the only child of K’s brother.
Applying the above conditions, Case-2 gets
 Only one person sits between K and C’s
eliminated because C is not the father of W.
grandmother(either from left or right).
Case-1(a) gets eliminated because we cannot
 A is the brother-in-law of W and sits
place P. Hence, Case-1 shows the final answer.
adjacent to C’s grandmother.
 N sits to the immediate left of one who
sits opposite to A, who has no siblings.

Click Here For Bundle PDF Course | support@guidely.in Page 8 of 10


Bank Po Mains PDF Course 2024
Reasoning Day -18 (Eng)

Directions (16-20):
16. Answer: B

Directions (11-15):
11. Answer: E
12. Answer: C
13. Answer: B 17. Answer: D
14. Answer: A
15. Answer: D
Final arrangement

Click Here For Bundle PDF Course | support@guidely.in Page 9 of 10


Bank Po Mains PDF Course 2024
Reasoning Day -18 (Eng)

19. Answer: E

20. Answer: E
Clearly, none of the statements gives the relation
18. Answer: A
between Q and R.
Hence, option E is the correct choice.

Click Here For Bundle PDF Course | support@guidely.in Page 10 of 10


Bank Po Mains PDF Course 2024
Quantitative Aptitude Day -18 (Eng)

Quantitative Aptitude

Directions (01 - 05): Study the following b) 29.65%


information carefully and answer the questions c) 27.50%
given below. d) 15.35%
The table given below shows the number of e) 10.33%
mobiles [4G+5G] sold in shop A in five months
and also given the Percentage of the number of 2) Total number of mobile phones sold in shop B
4G mobiles sold in each month in shop A. in March is [(2M-N+O)/2] % more than that of
same month in shop A. If the ratio of the number
of 4G and 5G mobiles sold in shop B in March is
3:2 then find the total number of 5G mobiles sold
in shop A and shop B together in March?
a) 1452
b) 2952
c) 2150
d) 1664
e) 1894

3)
I. Number of 5G mobiles sold in April is ______
more than the number of 4G mobiles sold in
May.
Note: II. Number of 4G mobiles sold in February is ____
1. M+N+120=220 more than the number of 5G mobiles sold in
2. O% of 120 + 60% of 240=204 January.
3. 2M+O=50% of 260 Find which of the given option satisfy the above
1) If the number of 4G mobiles sold in June is blank?
increased by (N/4) % than the previous month a) 360,150
and the number of total mobiles sold in June is b) 360,120.
increased by (M/2) % than the previous month. c) 120,360
Find the percentage of increase in 5G mobiles d) 150,360.
sold in June than the previous month? e) None of these
a) 24.35%

Click Here For Bundle PDF Course | support@guidely.in Page 1 of 9


Bank Po Mains PDF Course 2024
Quantitative Aptitude Day -18 (Eng)

4) If the total number of 4G and 5G mobiles sold covered by car A in 6 hours. Car C can cover a
in all months is represented in the pie chart then distance of 255 km in 8.5 hours.
find the central angle made by the total number 6) Car A covers L km in (t+2) hours and car B
of 4G and 5G mobiles sold in May? covers M km in (t+1) hours. The value of L is 40
a) 600 km less than the distance between M and N.
b) 960 Then the distance covered by car C in 5t hours is
c) 1080 how much more than M km?
d) 720 a) 220km
e) None of these b) 260km
c) 250km
5) Number of 4G and 5G mobiles sold in June is d) 210km
840 and 680 respectively and the Number of 4G e) 280km
and 5G mobiles sold in July is 940 and 780
respectively. Find the difference between the 7) Car B covers (D+120) km in 5 hours and Car
total number of 4G mobiles sold in the first six E covers (2D+140) km in 15 hours. Find the
months and the total number of 5G mobiles sold average speed of car B and car E?
in the last six months till July? a) 145/3 km/hr
a) 20 b) 401/3 km/hr
b) 30 c) 332/3 km/hr
c) 50 d) 115/3km/hr
d) 80 e) 135/3 km/hr
e) None of these
8) Speed of car F is 20% more than the speed of
Directions (06 - 10): Study the following car D. Car F and Car C together covered some
information carefully and answer the questions distance in 5 hours. The total distance covered
given below. by these two cars is covered by car A in t hours.
The speed of car A is 33.33% more than the Find the value of t?
speed of car C. Speed of car C is 40% less than a) 11.75 hours
the speed of car B. A certain distance is covered b) 12.75 hours
by car D in 2a hours and same distance is c) 19.35 hours
covered by car A in 3a hours. Distance between d) 21.35 hours
two points P and Q is covered by car D in 3 e) None of these
hours. Distance between two points M and N is

Click Here For Bundle PDF Course | support@guidely.in Page 2 of 9


Bank Po Mains PDF Course 2024
Quantitative Aptitude Day -18 (Eng)

9) Car A start travelling at point P and car B at ____(n)____ days. They together complete 50%
point Q at 9 am towards Q and P respectively. of work in ___(o)___ days.
After some hours, they meet each other then car Find in which of the given options value of m, n,
B reached P and car A reached Q but car A and o satisfy the above condition.
returns to P. Find the difference between the I. m=1/4, n=8 and o=24/5
total time (in minutes) when both cars reached II. m=1/5, n=10 and o= 12
P? III. m=1/3, n=15 and o=10
a) 324 min a) Only I
b) 321 min b) Only III
c) 334 min c) Only I and III
d) 384 min d) Only II and III
e) None of these e) Only I and II

10) Point O and N is in the East and south 12) Cost price of the item is Rs.240. Shop
direction from point M. Distance between MN keeper marked up the item 60% above the cost
and MO is the same. Car A and D start their price then gave a discount D% and earn P%
journey with their original speed at the same time profit. Find the correct combination of values D
from M and they reached N and O respectively. and P in the given Column.
After reached their respective points, again they
start their journey from N and O and reached M
but both of them reached at same time. Find the
increased percent speed of car A when coming
back to M?
a) II-A
a) 50%
b) III-A
b) 100%
c) II-B
c) 200%
d) I-C
d) 75%
e) More than one option is correct
e) None of these

13) Speed of the boat in still water is m km/hr


Directions (11 - 15): Study the following
and the Speed of the stream is 4 km/hr. Boat can
information carefully and answer the questions
cover 48 km in 3 hours in upstream.
given below.
Find which of the given option is true from the
11) A can complete ___(m)____ part of work in 4
above data?
days and B can complete 1/3rd of work in

Click Here For Bundle PDF Course | support@guidely.in Page 3 of 9


Bank Po Mains PDF Course 2024
Quantitative Aptitude Day -18 (Eng)

I. The Speed of the car is (m+28) km/hr and car d) 40, 25


cover 144 km in 3 hours. e) More than one option is true.
II. The boat covers 116 km in 4.5 hours
downstream. Directions (16-20): Study the following
III. The distance covered by boat in 5 hours information carefully and answer the questions
downstream is the same as the distance covered given below. There are three rows, calculate the
by bus with the speed of 32 km/hour in 3 hours. roots of the equation and compare them with
a) Only II other columns. Check which one matches.
b) Only I 16)
c) Only III Equations Statements
d) Only I and II i) 5x2-18x+9=0 a) Sum of the roots is
e) None of these positive.
ii) 2x2-12x+18=0 b) Sum of roots is
14) Ratio of milk and water in a mixture is 3:2. negative.
Total quantity of the mixture is _____ liters and iii) 2x2+15x+13=0 c) Both roots are
40% mixture is taken out from the mixture then integers.
amount of milk in the remaining mixture is _____ a) ii)-b)
litres. b) i)-b)
Find which of the option is true which fill the c) i)-a) & iii) -b)
blank in same sequence? d) iii)-c)
a) 1050,378 e) None of these
b) 1430,512.8
c) 1200,430 17)
d) 1800,640 Equations Statements
e) None of these i) x2+16x+39=0 a) Sum of the roots is
positive.
15) Average age of A, B and C is ____ years. ii) x2+15x+56=0 b) Sum of roots is
Ratio of age of B and C is 3:2. Age of A after 5 negative.
years is 29 years. The difference in age between iii) x2-9x+14=0 c) Both the roots are
B and C is ______. positive.
Find which given option satisfies the blank? a) ii)-a)
a) 30, 15 b) i)-c)
b) 25, 18 c) i)-a) & ii) -c)
c) 28, 12

Click Here For Bundle PDF Course | support@guidely.in Page 4 of 9


Bank Po Mains PDF Course 2024
Quantitative Aptitude Day -18 (Eng)

d) iii)-c) negative.
e) None of these iii) x2+20x+75=0 c) Both the roots are
positive.
18) a) ii)-a)
Equations Statements b) i)-b)
i) x2 -56x+784=0 a) Sum of the roots is c) i)-b) & ii) -b)
positive. d) i)-c)
ii) x2+10x-171=0 b) Sum of roots is e) None of these
negative.
iii) 2x2+12x+16=0 c) Highest root is a 20)
perfect square Equations Statements
number. i) 4x2+2x-20=0 a) Sum of the roots is
a) ii)-a) positive.
b) i)-b) ii) 3x2-10x-13=0 b) Sum of roots is
c) ii)-c) & ii) -b) negative.
d) iii)-c) iii) 10x2-8x-2=0 c) Both the roots are
e) None of these positive prime
numbers.
19) a) ii)-a)
Equations Statements b) i)-a)
i) 2x2-19x+17=0 a) Sum of the roots is c) i)-a) & ii) -b)
positive. d) iii)-c)
ii) x2+16x+60=0 b) Both the roots are e) None of these

Click Here For Bundle PDF Course | support@guidely.in Page 5 of 9


Bank Po Mains PDF Course 2024
Quantitative Aptitude Day -18 (Eng)

Click Here to Get the Detailed Video Solution for the above given Questions
Or Scan the QR Code to Get the Detailed Video Solutions

Answer Key with Explanation


Directions (01 - 05):
2. O% of 120 + 60% of 240=204
Or, O=[204-144]*100/120=50
3. 2M+O=50% of 260
So, 2M=130-50=80, M=40
1. M+N+120=220
So, N=220-120-40=60
The Number of mobile phones sold in January is 1) Answer: C

1200. The total number of mobiles sold in June is

The Number of 4G mobile phones sold in 1200*120/100=1440

January is 1200*40/100=480 Number of 4G mobiles sold in June is

The number of 5G mobile phones sold in 720*[100+(N/4)]/100=828

January is 1200-480=720 The Number of 5G mobiles sold in June is 1440-

The Number of mobile phones sold in February 828=612

is 2600-1200=1400 So, increase percentage = [(612-

The Number of 4G mobiles sold in February is 480)/480]*100=27.5%

1400*60/100=840
The Number of 5G mobiles sold in February is 2) Answer: D

1400-840=560 Number of mobile phone sold in shop B in March

Similarly, we can calculate forother months also. is 1600*[100+(80-


60+50)/2]=1600*135/100=2160
The number of 5G mobiles sold in shop B in
March is 2160*2/5=864

Click Here For Bundle PDF Course | support@guidely.in Page 6 of 9


Bank Po Mains PDF Course 2024
Quantitative Aptitude Day -18 (Eng)

The total number of 5g Mobile sold in March in So, The ratio of the speed of cars B and C is 5:3.
shop A and B together is 864+800=1664 So, The speed of cars A, B, C, and D is 4:5:3:6.
The Speed of car C is 255/8.5=30 km/hr.
3) Answer: B So, The Speed of car A, B, and D is 4*10=40
I. Number of 5G mobiles sold in April is __(1080- km/hr, 5*10=50 km/hr, 6*10=60 km/hr.
720)=360___ more than the number of 4G Distance betweenP and Q is 60*3= 180 km
mobiles sold in May. Distance between M and N is 40*6=240 km
II. Number of 4G mobiles sold in February is 6) Answer: C
__(840-720)=120_ more than the number of 5G 40*(t+2)=L=240-40=200km
mobiles sold in January. Or, 40t=120, t=3
So, the answer is 360,120. M=50*4=200km
C cover distance in 5*3=15 hours is
4) Answer: A =15*30=450km
The central angle made by total mobiles sold in So, the difference is 450-200=250 km
May
= 7) Answer: D
[1200/(1200+1400+1600+1800+1200)]*360=600 So, D+120=5*50=250
Or, D=250-120=130
5) Answer: A So, the speed of car E is
Required difference [130*2+140]/15=400/15km/hr
= [480+840+800+720+720+840] – So, average speed is = (400/15 + 50)/2 =230/6 =
[560+800+1080+480+680+780] 115/3km/hr
= 20
8) Answer: B
Directions (06 - 10): The speed of car F is 60*120/100=72 km/hr.
The speed of car A is 33.33% more than the Total distance covered by car F and Car C is
speed of car of C. 5*(72+30)=510km
So, the speed ratio of car A and car C is 4:3. So, value of t= 510/40=12.75 hours
The Speed ratio of cars A and D is 2:3.
The Speed of car C is 40% less than the speed 9) Answer: A
of car B. So, car A cover 180*2=360 Km distance.
So, if the speed of car B is 100 then the speed of Total time taken by car A is = 360/40=9
car C is 60. hours=540 min

Click Here For Bundle PDF Course | support@guidely.in Page 7 of 9


Bank Po Mains PDF Course 2024
Quantitative Aptitude Day -18 (Eng)

Car B cover only 180 km. 13) Answer: B


So, time taken by car B is 180/50=3.6 hours=216 The Speed of the boat upstream is =48/3=16
min km/hr
The difference in time is 540-216=324 min The Speed of the boat in still water is
16+4=20km/hr=m.
10) Answer: C I. The Speed of the car is (m+28) km/hr and the
D cover full journey in (240/60) * 2 =8 hrs car cover 144 km in 3 hours.
A cover first half journey in (240/40) =6 hrs So, [20+28]*3=144km
Remaining hrs = 2 So, it is true.
Increase in speed of car A = 240/2 = 120km/hr II. The boat covers 116 km in 4.5 hours
Required increase in speed = (120-40)/40 * 100 downstream.
= 200% [20+4]*4.5=108 km, so it is false.
III. The distance covered by boat in 5 hours in
11) Answer: A downstream is the same as the distance covered
Now check the value of I, by bus with the speed of 32 km/hour in 3 hours.
So, m=1/4, A can complete in 4*4=16 days, Boat covers in 5 hours in downstream is
B can complete in 8*3=24 days 5*24=120 km
So, the total work is LCM of 16, and 24 is 48 Bus covers in 3 hours is 32*3=96 km
units. So, it is false,
The efficiency of A and B is 3 and 2 units/day Only I is true.
respectively.
A and B together complete 50% of work in 24/5 14) Answer: A
days. Let's check option a,
Options I is true. Similarly, we can check others The total quantity of the mixture is 1050 liters,
options so other two options are false. The amount of milk is 1050*3/5=630 liters
Left milk is 630*60/100=378 liters
12) Answer: E So, it is true. Similarly, we check other options
From the given options E is true. also. So, only A is true.
Marked price is 240*160/100=Rs.384
Selling price 384*75/100=288 15) Answer: C
Profit percentage = [(288-240)/240] *100=20% Let's check the option C,
Similarly, we can check for other values also. II- The average age of A,B, and C is 28years, total
A also satisfies the given condition. age is 28*3=84years

Click Here For Bundle PDF Course | support@guidely.in Page 8 of 9


Bank Po Mains PDF Course 2024
Quantitative Aptitude Day -18 (Eng)

The Sum of the age of B and C is 84-[29- or, (x-28)2=0, x=28, 28


5]=60years x2+10x-171=0
So, difference in age of B and C is 60/5=12years or, x2+19x-9x-171=0
So, it is true. or, (x+19)(x-9)=0, x=-19,9
2x2+12x+16=0
16) Answer: C 2x2+8x+4x+16=0
5x2-18x+9=0 Or, (x+4)(2x+4)=0, x=-4,-2
Or, 5x2-15x-3x+9=0 So, option C is true.
Or, (x-3)(5x-3)=0, x=3,5/3
2x2-12x+18=0 19) Answer: D
Or, 2x2-6x-6x+18=0 2x2-19x+17=0
Or, (x-3)(2x-6)=0, x=3,3 Or, 2x2-17x-2x+17=0
2x2+15x+13=0 Or, (x-1)(2x-17)=0, x=1,17/2
Or, 2x2+13x+2x+13=0 x2+16x+60=0
Or, (2x+13)(x+1)=0, x=-1,-13/2 or, x2+10x+6x+60=0
So, option c is true. or, (x+10)(x+6)=0, x=-10,-6
x2+20x+75=0
17) Answer: D or, x2+15x+5x+75=0
x2+16x+39=0 or, (x+15)(x+5)=0, x=-15,-5
or, x2+13x+3x+39=0 so, option D is true.
or, (x+13)(x+3)=0, x=-13,-3
x2+15x+56=0 20) Answer: A
or, x2+7x+8x+56=0 4x2+2x-20=0
or, (x+7)(x+8)=0, x=-7,-8 Or, 4x2+10x-8x-20=0
x2-9x+14=0 Or, (2x+5)(2x-4)=0, x=-5/2,2
or, x2-7x-2x+14=0 3x2-10x-13=0
or, (x-7)(x-2)=0, x=7,2 Or,3x2-13x+3x-13=0
so, option D is true. Or, (3x-13)(x+1)=0, x=13/3,-1
10x2-8x-2=0
18) Answer: C Or, 10x2-10x+2x-2=0
x2-56x+784=0 Or, (x-1)(10x+2)=0, x=1,-1/5
or, x2-2*28x+282=0 So, option A is true.

Click Here For Bundle PDF Course | support@guidely.in Page 9 of 9


Bank Po Mains PDF Course 2024
English Day - 18

English Language
Directions (1-5): Rearrange the following five 1) Which of the following is the FOURTH
sentences (A), (B), (C), (D) and (E) in the proper sentence after the rearrangement ?
sequence to form a coherent paragraph and then a) A
answer the questions given below. b) B
(A) Prior to this announcement, earlier in 2020, c) C
MEiTY notified rules stating that the Central d) D
government may allow Aadhaar authentication e) E
by requesting entities in the interest of good
governance, prevention of leakage of public 2) Which of the following is the FIRST sentence
funds and to promote ease of living. after the rearrangement ?
(B) The Registrar General of India (RGI) a) A
appointed under the Registration of Birth and b) B
Death Act, 1969 can now perform the yes or no c) C
Aadhaar authentication during registration of d) D
births and deaths in the country. e) E
(C) The State governments and Union Territory
administrations shall adhere to the guidelines 3) Which of the following sentences immediately
with respect to the use of Aadhaar authentication follows ‘sentence B’ in the rearranged sequence
as laid down by MEiTY. ?
(D) A gazette notification published in June said a) A
the Ministry of Electronics and Information b) B
Technology (MEiTY) has allowed the RGI office c) C
to use the Aadhaar database for authenticating d) D
the identity details provided during registration of e) E
births and deaths.
(E) The authentication may be on a voluntary 4) Which of the following is the SECOND
basis, for verification of Aadhaar number being sentence after the rearrangement ?
collected along with other details, as the case a) A
may be, for the purpose of establishing the b) B
identity of child, parent and the informant in case c) C
of deaths during registration of births or deaths”. d) D
e) E

Click Here For Bundle PDF Course | support@guidely.in Page 1 of 11


Bank Po Mains PDF Course 2024
English Day - 18

5) Which of the following is the LAST sentence 8. From June 1, India has made tests
after the rearrangement ? ___________ for cough syrups before they are
a) A exported. This step is the result of some reports
b) B regarding the contamination of cough syrups.
c) C A. propel
d) D B. contradictory
e) E C. remorse
D. mandatory
Directions (6-10): Given below are a few E. None of the above
questions with a blank in each of them. You have
to fill the blanks with appropriate word in order to 9. Mangoes priced at a _____________ Rs 2.5
make sentence contextually correct. If none of lakh per kg in the global market were stolen from
the options are correct then choose option E as a farm in Odisha’s Nuapada district shortly after
your answer. the farm owner shared photos of the fruit on
6. The Uttar Pradesh Madrassa Education Board social media.
has issued orders to celebrate Yoga Day in all A. stunning
aided and non-aided ___________ madrassas of B. staggering
the state on the occasion of International Yoga C. disgusting
Day. D. palpitating
A. recommended E. None of the above
B. primitive
C. inductive 10. Overall, India is the largest provider of
D. recognised generic drugs globally, ____________ over 50 per
E. None of the above cent of global demand for various vaccines,
about 40 per cent of generic demand in the US
7. Modi said the International Day of Yoga was and about 25 per cent of all medicines in the UK.
special this year as researchers at India's A. importing
research stations in the Arctic and Antarctica too B. mending
were _____________ in the celebrations. C. distracting
A. participating D. supplying
B. escalating E. None of the above
C. precipitating
D. mandating
E. None of the above

Click Here For Bundle PDF Course | support@guidely.in Page 2 of 11


Bank Po Mains PDF Course 2024
English Day - 18

Directions (11-15): Read each question to find e) E


out whether there is any grammatical error in it.
The error, if any, will be in one part of the 14. Students should always stay motivated and
sentence. Choose the option with that part as stop seeking/A motivation from outside instead
your answer. If there is no error, mark(e) as your one must/B always ask themselves one question
answer. time and again,/C “Did I take a forward step
11. Families who were caught together/A at the today that leads to my success ?”/D.No error/E
US-Mexico border and separated/B are now a) A
being offered an opportunity/C to reunite and b) B
stay together/D.No error/E c) C
a) A d) D
b) B e) E
c) C
d) D 15. During the pandemic the/A companies were
e) E providing more and more benefits to their/B
employees and the employees fell for the/C
12. He was right pretty much /A when he said furniture and gift hampers not knowing what was
almost all the people/B fail to change their coming for them/D.No error/E
behaviour and continue to/C blame the system a) A
for all their problems/D.No error/E b) B
a) A c) C
b) B d) D
c) C e) E
d) D
e) E Directions (16-17): A sentence is divided into five
parts. Two of these parts may have an error in it.
13. The human eardrum is able to receive/A and Choose the parts which have error in it. If there is
transmit signals across a radio/B frequency no error in the given statement, choose ‘No error’
range more greater than/C what we can hear as your answer.
with the human ear/D.No error/E 16. The commission amount was evenly
a) A distributed (A)/ among the two agents, following
b) B which (B)/ they were expected to report on their
c) C superior (C)/ to elaborate on the progress of their
d) D ongoing projects (D).

Click Here For Bundle PDF Course | support@guidely.in Page 3 of 11


Bank Po Mains PDF Course 2024
English Day - 18

a) A and D a) What we need to do is to focus on them, and


b) B and C to hone their skills in every possible way to get
c) B and D them acquainted with how they could
d) C and A revolutionize the society for good.
e) No error b) Education is the main ground of each student,
where they can develop their skills in their
17. One of the two administrative positions has chosen field.
(A)/ been opened for quite some time (B)/, and c) Not only does education train minds into a
you have made the right decision (C)/ by variety of subjects that will later on help them in
choosing to apply for the most efficient one (D)/. their preferred career, it also provides a safe
a) A and B place for students that they can consider as a
b) C and D second home.
c) B and D d) Moreover, the concept of learning is to
d) C and A prepare these young people for the world that
e) No error would not always cater to their comfort.
e) All the above
Directions (18-20): Directions: In each of the
following questions a short passage is given with 19. Since long time, many trains including metros
one of the lines in the passage missing and have been running on electricity. Indian Railways
represented by a blank. Select the best out of the trains are now almost running on electricity.
five answer choices given, to make the passage Electric bike, electronic car, electronic rikshaw
complete and coherent. are already in market. Now people should use
18. Unbeknown to others, the effectiveness of more electric vehicles in place of traditional
creating a progressive change in the society can petrol and diesel vehicles. __________________.
only be achieved if we start to believe in the Government has started campaigns to promote
capabilities of these young people. Empowering use of electric vehicles. Some rebate on taxes
the youth by means of training their leadership and subsidy on purchasing the electric vehicles,
and critical thinking skills approach many are also provided by the Government. Recently
beneficial factors to our main aim; to create a Delhi Government has launched ‘Switch Delhi’
step-forward action for the betterment of the campaign to promote the use of electric vehicles.
country. Young people can also become agents a) Dedicated social media handles were
of social change as much as the current leaders assigned to share updates on policy
of our generation. ________________________. implementation, provide information about
incentives, resolve grievances, and answer

Click Here For Bundle PDF Course | support@guidely.in Page 4 of 11


Bank Po Mains PDF Course 2024
English Day - 18

queries pertaining to EVs as part of the welcome respite. Other students may feel relief
campaign. from not having the pressure to look good or
b) As electric vehicle is cheaper in long run and meet certain social expectations, and shy or
also environment friendly, Government is trying anxious students may find it easier to reach out
to acquaint the people with the advantages of for teacher assistance. A lack of social activities
using electric vehicles using serious measures. such as clubs or sports teams can also allow
c) With the subsidy granted by the government students to focus more intensely on their
being reduced to 15 per cent, it is clear that the schoolwork and improve their study habits.
electric vehicle ecosystem in India is growing a) Children who have anxiety are often quiet and
rapidly and there is demand. obedient and do not talk about their fears or
d) HOP Electric Mobility Co-Founder & Chief worries explicitly.
Operating Officer Nikhil Bhatia supported the b) Additionally, classroom learning can be
government's move saying It was time for the EV expensive since the educational institutions
industry to stand on its own require expenses for resources, and that
e) All the above increases the cost of courses.
c) Many children have flourished during online
20. Students may welcome the lack of social learning because they aren’t feeling the social
pressure that comes with online learning. The pressure that comes with being on campus.
social aspect of the classroom learning can be a d) In the case of classroom learning, trainers
distraction for some students and anxiety- have the opportunity to observe and interact with
inducing for others. ____________. In particular, children, which enables the trainers to identify
children who may be the victims of bullying at the strengths and learning style of the children.
school have found online learning to be a e) All the above
Click Here to Get the Detailed Video Solution for the above given Questions
Or Scan the QR Code to Get the Detailed Video Solutions

Answer Key with Explanation

Click Here For Bundle PDF Course | support@guidely.in Page 5 of 11


Bank Po Mains PDF Course 2024
English Day - 18

1. Answer: C noticed well, ‘D’ is more of a gist of the whole


‘C’ is the fourth sentence after rearrangement. paragraph that is to be formed whereas ‘B’
The correct sequence is DBECA. contains specific information on the
The first or the introductory sentence is ‘D’ authentication mechanism. A paragraph always
because sentences A,C and E do not contain with generic information and moves to specific
any introduction, instead they are a continuation content. With this we conclude that ‘D’ is our first
of one another. So the options to pick the statement of the paragraph. Immediate to ‘D’
introductory sentence are two - B and D. If follows ‘B’ which holds the full form of RGI which
noticed well, ‘D’ is more of a gist of the whole is mentioned in the first sentence. After ‘B’
paragraph that is to be formed whereas ‘B’ comes ‘E’ which talks about the purpose of this
contains specific information on the authentication and the need. Fourth sentence
authentication mechanism. A paragraph always would be ‘C’ as it talks about who should adhere
with generic information and moves to specific to the announcement that has been made by the
content. With this we conclude that ‘D’ is our first ministry. Lastly, it is ‘A’ which goes back to 2020
statement of the paragraph. Immediate to ‘D’ when the ministry had an idea to allow Aadhar
follows ‘B’ which holds the full form of RGI which authentication to make life easier.
is mentioned in the first sentence. After ‘B’
comes ‘E’ which talks about the purpose of this 3. Answer: E
authentication and the need. Fourth sentence The correct sequence is DBECA.
would be ‘C’ as it talks about who should adhere The sentence which immediately follows ‘B’ is
to the announcement that has been made by the ‘E’.
ministry. Lastly, it is ‘A’ which goes back to 2020 The first or the introductory sentence is ‘D’
when the ministry had an idea to allow Aadhar because sentences A,C and E do not contain
authentication to make life easier. any introduction, instead they are a continuation
of one another. So the options to pick the
2. Answer: D introductory sentence are two - B and D. If
‘D’ is the first sentence after the rearrangement. noticed well, ‘D’ is more of a gist of the whole
The correct sequence is DBECA. paragraph that is to be formed whereas ‘B’
The first or the introductory sentence is ‘D’ contains specific information on the
because sentences A,C and E do not contain authentication mechanism. A paragraph always
any introduction, instead they are a continuation with generic information and moves to specific
of one another. So the options to pick the content. With this we conclude that ‘D’ is our first
introductory sentence are two - B and D. If statement of the paragraph. Immediate to ‘D’

Click Here For Bundle PDF Course | support@guidely.in Page 6 of 11


Bank Po Mains PDF Course 2024
English Day - 18

follows ‘B’ which holds the full form of RGI which when the ministry had an idea to allow Aadhar
is mentioned in the first sentence. After ‘B’ authentication to make life easier.
comes ‘E’ which talks about the purpose of this
authentication and the need. Fourth sentence 5. Answer: A
would be ‘C’ as it talks about who should adhere The last sentence after the rearrangement is ‘A’.
to the announcement that has been made by the The correct sequence is DBECA.
ministry. Lastly, it is ‘A’ which goes back to 2020 The first or the introductory sentence is ‘D’
when the ministry had an idea to allow Aadhar because sentences A,C and E do not contain
authentication to make life easier. any introduction, instead they are a continuation
of one another. So the options to pick the
4. Answer: B introductory sentence are two - B and D. If
‘B’ is the second sentence after rearrangement. noticed well, ‘D’ is more of a gist of the whole
The correct sequence is DBECA. paragraph that is to be formed whereas ‘B’
The first or the introductory sentence is ‘D’ contains specific information on the
because sentences A,C and E do not contain authentication mechanism. A paragraph always
any introduction, instead they are a continuation with generic information and moves to specific
of one another. So the options to pick the content. With this we conclude that ‘D’ is our first
introductory sentence are two - B and D. If statement of the paragraph. Immediate to ‘D’
noticed well, ‘D’ is more of a gist of the whole follows ‘B’ which holds the full form of RGI which
paragraph that is to be formed whereas ‘B’ is mentioned in the first sentence. After ‘B’
contains specific information on the comes ‘E’ which talks about the purpose of this
authentication mechanism. A paragraph always authentication and the need. Fourth sentence
with generic information and moves to specific would be ‘C’ as it talks about who should adhere
content. With this we conclude that ‘D’ is our first to the announcement that has been made by the
statement of the paragraph. Immediate to ‘D’ ministry. Lastly, it is ‘A’ which goes back to 2020
follows ‘B’ which holds the full form of RGI which when the ministry had an idea to allow Aadhar
is mentioned in the first sentence. After ‘B’ authentication to make life easier.
comes ‘E’ which talks about the purpose of this
authentication and the need. Fourth sentence 6. Answer: A
would be ‘C’ as it talks about who should adhere Here, recommended fits best as per the context
to the announcement that has been made by the of the sentence as it means to suggest that you
ministry. Lastly, it is ‘A’ which goes back to 2020 would find that thing or person good or useful

Click Here For Bundle PDF Course | support@guidely.in Page 7 of 11


Bank Po Mains PDF Course 2024
English Day - 18

and here it is said in the context of Yoga. So, something and here it is said for generic drug.
option A is the correct answer. So, option D is the correct answer.
Primitive means very simple and not developed. Mending means to repair something that is
Inductive means using particular facts and damaged or broken.
examples to form general rules and principles.
11. Answer: A
7. Answer: A The error is in part A of the sentence.
Here, participating fits best as per the context of Replace ‘who’ with ‘which’(even though family
the sentence as it means to take part or become consist of member/people it is an entity and not
involved in something and here it is said for a member by itself so it should be represented
Yoga day. So, option A is the correct answer. by ‘which’)
The corrected sentence: Families which were
8. Answer: D caught together at the US-Mexico border and
Here, mandatory fits best as per the context of separated are now being offered an opportunity
the sentence as it means compulsory and here it to reunite and stay together.
is said for cough syrup. So, option D is the
correct answer. 12. Answer: A
Propel means to push or drive usually forward or The error is in part A of the sentence.
onward. He was right - is the main idea.
Remorse means a feeling or sadness because Pretty much - nearly right(to agree that he was
you have done something wrong. right)
He was pretty much right - is the correct phrase.
9. Answer: B The corrected sentence : He was pretty much
Here, staggering fits best as per the context of right when he said almost all the people fail to
the sentence as it means something that you change their behaviour and continue to blame
find difficult to believe and here it is said for such the system for all their problems.
an unbelievably high prices of mangoes. So,
option B is the correct answer. 13. Answer: C
Part C is erroneous.
10. Answer: D More greater(redundancy in the degree of
Here, supplying fits best as per the context of the comparison) - it is wrong
sentence as it means to give or provide The corrected sentences:

Click Here For Bundle PDF Course | support@guidely.in Page 8 of 11


Bank Po Mains PDF Course 2024
English Day - 18

Replace ‘more’ with ‘far’ - The human eardrum is something that has happened and ‘report to’
able to receive and transmit signals across a means to give people information about what
radio frequency range far greater than what we you have seen, heard, done, etc. Since the two
can hear with the human ear. agents are conveying information about their
Simply remove more - The human eardrum is projects to their superior here, it is right to
able to receive and transmit signals across a replace ‘report on’ by ‘report to’.
radio frequency range greater than what we can Parts A and D are correct.
hear with the human ear. The right sentence is:
The commission amount was evenly distributed
14. Answer: E between the two agents, following which they
The given sentence is error free as it is were expected to report to their superior to
grammatically correct and meaningful. elaborate on the progress of their ongoing
projects.
15. Answer: B
Error is in part B of the sentence. 17. Answer: C
Replace ‘their’ with ‘its’. Company is an Part B is incorrect. ‘Has been open’ means that
entity(not a living thing) and usage of ‘their’ is the subject of this phrase was opened some time
inappropriate. ago and is still open. ‘Has been opened’ means
The corrected sentence: During the pandemic that the action of "being opened" is completed.
the companies were providing more and more Thus, the position is still open because the
benefits to their employees and the employees speaker has just applied for the job now. Thus,
fell for the furniture and gift hampers not knowing ‘has been opened’ should be replaced by ‘has
what was coming for them. been open’.
Part D is incorrect. When referring to two
16. Answer: B persons, places or things use the comparative
The word ‘among’ is incorrect here. ‘Among’ is form; when referring to more than two, use the
usually used with more than two subjects; superlative form. Thus, ‘most’ should be
‘between’ is used with exactly two subjects. replaced by ‘more.
Since it is clearly mentioned that there are only Parts A and C are right.
two people involved here, it is right to replace The right sentence is:
‘among’ by ‘between’ in part B. One of the two administrative positions has been
Part C of the sentence is incorrect because open for quite some time, and you have made
‘report on’ means to write or speak about

Click Here For Bundle PDF Course | support@guidely.in Page 9 of 11


Bank Po Mains PDF Course 2024
English Day - 18

the right decision by choosing to apply for the possible way to get them acquainted with how
more efficient one. they could revolutionize the society for good.

18. Answer: A 19. Answer: B


The sentence before the blank talks about how The right answer is option b. Because the
students could be agents of social change. sentences after the blank talks about how the
Option a bets fits the blank because the government is trying to bring in awareness
sentence explains how the skills of the youth among the public about electric vehicles. The
should be honed (made perfect) and how they sentence in option B introduces the same the
should be made acquainted (familiar) with how involvement of government for the first time.
they could revolutionize the society for good. Thus, option B is the right answer.
Here, revolutionize means to change radically. Option a talks about a particular campaign since
Thus, option a speaks about how the youth the word is preceded by ‘the’ and not ‘a’. ‘The
should be prepared to be the agents of social campaign’ denotes a campaign that has already
change that is mentioned in the sentence before been discussed and thus, option a is incorrect.
the blank. Thus, option a best suits that blank. Option c and d are incorrect because the
Options b, c, and d talk about education and the involvement of government since the passage
passage given in the passage does not talk before the blank has not mentioned anything
anywhere about the education of the youth. about the involvement of the government.
Thus, options b, c, and are incorrect. The right paragraph is:
The right passage is: Since long time, many trains including metros
Unbeknown to others, the effectiveness of have been running on electricity. Indian Railways
creating a progressive change in the society can trains are now almost running on electricity.
only be achieved if we start to believe in the Electric bike, electronic car, electronic rikshaw
capabilities of these young people. Empowering are already in market. Now people should use
the youth by means of training their leadership more electric vehicles in place of traditional
and critical thinking skills approach many petrol and diesel vehicles. As electric vehicle is
beneficial factors to our main aim; to create a cheaper in long run and also environment
step-forward action for the betterment of the friendly, Government is trying to acquaint the
country. Young people can also become agents people with the advantages of using electric
of social change as much as the current leaders vehicles using serious measures. Government
of our generation. What we need to do is to has started campaigns to promote use of electric
focus on them, and to hone their skills in every vehicles. Some rebate on taxes and subsidy on

Click Here For Bundle PDF Course | support@guidely.in Page 10 of 11


Bank Po Mains PDF Course 2024
English Day - 18

purchasing the electric vehicles, are also of classroom learning and do not focus on
provided by the Government. Recently Delhi elaborating on where children stand when it
Government has launched ‘Switch Delhi’ comes to online learning.
campaign to promote the use of electric vehicles. The right passage is:
Students may welcome the lack of social
20. Answer: C pressure that comes with online learning. The
The sentence after the blank begins with ‘in social aspect of the classroom learning can be a
particular’ which means the sentence is just distraction for some students and anxiety-
about to render information that is an addition to inducing for others. Many children have
the information stated in the previous sentence. flourished during online learning because they
The sentence after the blank states how children aren’t feeling the social pressure that comes with
who are victims of bullying at school, in being on campus. In particular, children who
particular, find online learning welcoming. Thus, may be the victims of bullying at school have
the sentence in the blank should speak about found online learning to be a welcome respite.
how children, in general, have grown to like Other students may feel relief from not having
online learning. Thus, option c is the right the pressure to look good or meet certain social
answer. expectations, and shy or anxious students may
Options a is incorrect it talks about children with find it easier to reach out for teacher assistance.
anxiety in general and states no direct A lack of social activities such as clubs or sports
information of the children’s opinion on online teams can also allow students to focus more
learning. intensely on their schoolwork and improve their
Option b and d are incorrect since they proceed study habits.
to describe the disadvantages and advantages

Click Here For Bundle PDF Course | support@guidely.in Page 11 of 11


Bank Po Mains PDF Course 2024
Reasoning Day -19 (Eng)

Reasoning Aptitude
Directions (1-5): Study the following information 1) Who among the following persons is not
carefully and answer the given questions. senior to Aden?
Nine persons – Adam, Aden, Arlo, Ezra, Evan, a) The one works two position junior to Arlo
Jack, Lucy, Luna, and Nora are working in the b) The one who works at immediately senior
real Estate company at different designations position of Vacant designation
such as Director, Chief Executive Officer(CEO), c) Evan
Chief Operating Officer(COO), Managing d) Nora
Director, Project Director, Deputy Director, e) The one who works three positions senior to
Regional Director, Admin Manager, Sales Lucy
Manager, and Sales Executive. The designations
are given in decreasing order such as Director is 2) Which of the following statements is/are true
the seniormost designation and Sales Executive as per the given arrangement?
is the juniormost designation. One of the I) Adam works as Sales Manager
positions is left Vacant. II) Only one person is designated between Arlo
Aden works as either Deputy director or Admin and the one who works as Chief Operating
Manager. Only two persons are designated Officer.
between Aden and Nora. At least two persons III) Lucy and Aden work as senior and junior
are junior to Nora. Ezra works three positions position of Vacant position respectively
senior to the one who works two positions junior IV) No one works junior to the one who works
to Lucy. At least two persons are senior to Ezra. two positions junior to Evan
Lucy neither works as Regional Director nor a) Only (III) and (IV)
Project Director. Jack works two positions Junior b) Only (I) and (II)
to the one who works immediately senior to c) Only (II) and (III)
Adam. The number of persons designated d) Only (II)
between Jack and Lucy is two less than the e) All (I), (II) and (III)
number of persons designated between Luna
and Ezra. Jack is designated one of the positions 3) How many persons are senior to the one who
junior to vacant post. The number of persons works two positions senior to Adam?
senior to the one who is Regional Director is a) None
same as the number of persons designated b) One
between Arlo and Adam. Evan is not designated c) Two
as Sales Manager. d) Three
e) Four

Click Here For Bundle PDF Course | support@guidely.in Page 1 of 10


Bank Po Mains PDF Course 2024
Reasoning Day -19 (Eng)

4) If Aden gets a promotion to one position, then Note-IV: Atmost one person lives in each flat and
which of the following designation does he work? atmost two persons live on each floor.
a) Director Note-V: Adjacent flats mean flat which is
b) Chief Executive Officer immediately above or immediately below and
c) Chief Operating Officer immediately left or right only considered.
d) Project Director Ira lives three floors above the one who lives
e) Managing Director immediately below the floor of Oma. Oma and Ira
are not living in the same type of flat. Leo lives
5) Four of the following five are alike in a certain one of the floors above Ira but does not live in
way based on the given arrangement and thus the same type of flat. The number of floors above
form a group. Which one of the following doesn’t Leo is one less than the number of floors below
belong to the group? Nat. Nat and Leo are living in the same type of
a) Director-Nora flat. Ema lives southeast flat of the one who lives
b) CEO-Ezra south-west flat of Kim. Kim does not live on the
c) Admin Manager-Luna odd numbered floor. The number of floors below
d) Project Director-Aden Ema is one less than the number of floors above
e) Deputy Director-Evan Joy. Joy and Oma are living in different type of
flats and on different floors. Only one floor gap
Directions (6-10): Study the following information between Kim and Lex, who lives below Kim. No
carefully and answer the given questions. one lives on the adjacent flat of Ira. Lex lives
Eleven persons - Ema, Gia, Ira, Joy, Kim, Leo, either on 4th or 2nd floor in flat A. As many floors
Lex, May, Nat, Nim and Oma live on six different between Nim and Ira as between vacant flat and
floors of a six storeyed building where the Gia. May lives in the same type of flat as vacant
lowermost floor is numbered 1, the floor above it flat but not live in the adjacent flat of vacant flat.
is numbered 2 and so on. One of the flats is Nim and Ema are not living on the same floor.
vacant. 6) Who among the following person lives west of
Note-I: Each floor has two flats viz., Flat A and the one who lives two floors above Lex?
Flat B where Flat A is to the west of Flat B. a) May
Note-II: Flat A of Floor 2 is immediately above b) The one who lives in Flat A on Floor four
Flat A of Floor 1 and immediately below Flat A of c) The one who lives in Flat A on Floor three
Floor 3 and so on. Similarly, Flat B of floor 2 is d) Joy
immediately above Flat B of Floor 1 and e) Gia
immediately below Flat B of Floor 3 and so on.
Note-III: Area of each flat on each floor is same. 7) As many floors above May as below_____

Click Here For Bundle PDF Course | support@guidely.in Page 2 of 10


Bank Po Mains PDF Course 2024
Reasoning Day -19 (Eng)

a) Ira Ten persons - L, M, N, O, P, Q, R, S, T and U


b) Nat belong to the same family. They bought a new
c) The one who lives on Floor 3 of flat B house in different years - 2001, 2003, 2005,
d) The one who lives on the same floor of Lex 2006, 2008, 2011, 2012, 2014, 2019, and 2022.
e) The one who lives on the same floor of Ema Only one person bought a house in each year.
N’s father bought three years before T’s only
8) Which of the following statements is/are true daughter. At least one person bought after T’s
as per the given arrangement? daughter. The number of persons bought before
a) May lives on the odd number floor N’s father is one more than the number of
b) Ira lives in a northeast flat of Nat persons bought after M, who is not N’s father. L
c) Nat and Vacant flat on the same floor is the only son of the one who is the father-in-law
d) Only one floor is between Oma and Leo of O’s spouse. N’s spouse bought three years
e) All the statements are true before the one who bought two years after R’s
son. Only one person bought between N’s
9) In which of the following floor and flat does the spouse and O. R’s only son is the brother-in-law
one lives who is immediate southwest of Leo? of N and vice versa. Neither O’s spouse nor T’s
a) Floor 1, Flat A spouse has a sibling. The brother-in-law of L is
b) Floor 2, Flat A the son-in-law of R. T’s daughter is the niece of
c) Floor 4, Flat A the one who is the daughter of R’s husband. Q is
d) Floor 5, Flat A the nephew of L, who is the only brother of T. L is
e) Floor 3, Flat A unmarried. The number of years difference
between T’s brother and O’s sister’s father-in-law
10) Which among the following pair of the person is one less than the number of years difference
live on the same floor? between P’s spouse and U’s father. U’s father
a) Ira, Leo did not buy in an odd numbered year. T’s spouse
b) Joy, Nim did not have a male kid. M’s brother-in-law is
c) May, Leo unmarried. The number of persons bought before
d) Lex, Oma L’s nephew is one more than the number of
e) Gia, Nat persons bought after S’s son. P did not buy in an
odd numbered year.
Directions (11-15): Study the following 11) Which of the following statement(s) is/are
information carefully and answer the given true as per the given arrangement?
questions. a) Q and U are siblings.
b) R’s daughter is the spouse of Q

Click Here For Bundle PDF Course | support@guidely.in Page 3 of 10


Bank Po Mains PDF Course 2024
Reasoning Day -19 (Eng)

c) All the statements are true 15) How many years difference between the one
d) Only two persons bought between O’s spouse who bought the house three years before T’s
and R father-in-law and the one who bought
e) T is the spouse of U’s father immediately after O’s spouse?
a) Fourteen
12) As many persons bought between N’s b) Twelve
spouse and P’s son as between O’s brother-in- c) Eight
law and _____ d) Nine
a) R’s spouse e) Ten
b) P’s spouse
c) N’s father Directions (16-20): Study the following
d) M’s daughter information carefully and answer the given
e) O’s spouse questions.
D&4%G27#E^38>LM@5A£6O9¥
13) Four of the following five are alike in a certain Step I: All such symbols which are immediately
way based on the given arrangement and thus preceded by a vowel and immediately followed
form a group. Which one of the following doesn’t by a number, are to be placed immediate right of
belong to the group? ‘L’ in the same order from left to right.
a) Q-N’s spouse Step II: All such letters which are immediately
b) L-T’s father-in-law preceded by a symbol and immediately followed
c) O- M’s son by a number are to be placed in alphabetical
d) R-U’s father order to the immediate right of ‘2’.
e) M-P Step III: All such numbers which are immediately
preceded and immediately followed by the same
14) If U and M interchange their positions and element type are to be placed immediate right of
R’s spouse and O’s mother did the same, with ‘^’ in ascending order.
respect to the new positions, who among the Note: Step 1 is followed by step II and step II is
following person bought the house three persons followed by step III
before O’s brother-in-law? Step III is the final step of the given input and
a) O answer the questions below based on the final
b) The one who bought immediately after Q step.
c) The one who is the son of O 16) How many such letters is/are there either
d) The one who is the daughter of N immediately preceded or immediately followed
e) M by a number?

Click Here For Bundle PDF Course | support@guidely.in Page 4 of 10


Bank Po Mains PDF Course 2024
Reasoning Day -19 (Eng)

a) Five c) One
b) Four d) More than three
c) Three e) None
d) Six
e) None of these 19) If all the letters immediately followed by
symbols are dropped, then which of the following
17) Which of the following element is tenth from element is twelfth from the left end?
the right end, if all the digits are dropped? a) M
a) L b) #
b) # c) 4
c) M d) 8
d) E e) None of these
e) None of these.
20) Find the odd one out.
18) How many such symbols are there in the a) E3#
final arrangement which is/are immediately b) 65@
preceded by a number and immediately followed c) %7G
by a consonant? d) M@O
a) Two e) 84^
b) Three
Click Here to Get the Detailed Video Solution for the above given Questions
Or Scan the QR Code to Get the Detailed Video Solutions

Answer Key with Explanation


Directions (1-5): 3) Answer: E
1) Answer: C 4) Answer: D
2) Answer: C

Click Here For Bundle PDF Course | support@guidely.in Page 5 of 10


Bank Po Mains PDF Course 2024
Reasoning Day -19 (Eng)

5) Answer: C(the designation is immediately  Ezra works three positions senior


higher position of the person, except option c) to the one who works two positions
Final Arrangement junior to Lucy.
 At least two persons are senior to
Ezra.
 Lucy neither works as Regional
Director nor Project Director.
 Jack works two positions Junior to
the one who is immediately senior
to Adam.
 The number of persons designated
between Jack and Lucy is two less
than the number of persons
designated between Luna and
We have,
Ezra.
 Aden works as either Deputy
 Jack is designated one of the
director or Admin Manager.
positions junior to vacant post.
 Only two persons are designated
From the above condition, Case-2, and Case-2a
between Aden and Nora.
get eliminated. Case1a has one more possibility.
 At least two persons are junior to
Nora.
From the above condition, there are four
possibilities.

Again we have,
 The number of persons senior to
the one who is Regional Director is
same as the number of persons

Again we have, designated between Arlo and


Adam.

Click Here For Bundle PDF Course | support@guidely.in Page 6 of 10


Bank Po Mains PDF Course 2024
Reasoning Day -19 (Eng)

 Evan is not designated as a Sales  Oma and Ira are not living in the same
manager. type of flat.
From the above condition, case 1 and Case-1a  Leo lives one of the floors above Ira but
get eliminated. Case-1a(1) shows the final does not live in the same type of flat.
arrangement.  The number of floors above Leo is one
less than the number of floors below Nat.
 Nat and Leo are living in the same type of
flat.
From the above condition, there are four
possibilities.

Directions (6-10):
6) Answer: B
Again we have,
7) Answer: E
 Ema lives southeast flat of the one who
8) Answer: C
lives southwest flat of Kim.
9) Answer: C
 Kim does not live on the odd numbered
10) Answer: D
floor.
Final Arrangement
 The number of floors below Ema is one
less than the number of floors above Joy.
 Joy and Oma are living in different type of
flats and on different floors.
 Only one floor gap between Kim and Lex,
who lives below Kim.
 No one lives on the adjacent flat of Ira.
 Lex lives either on 4th or 2nd floor in flat A.
 May lives in the same type of flat as
We have,
vacant flat but not live in the adjacent flat
 Ira lives three floors above the one who
of vacant flat.
lives immediately below the floor of Oma.

Click Here For Bundle PDF Course | support@guidely.in Page 7 of 10


Bank Po Mains PDF Course 2024
Reasoning Day -19 (Eng)

From the above condition, case-1 and case 4 get 14) Answer: D
eliminated. 15) Answer: D
Final Arrangement

Again we have,
 As many floors between Nim and Ira as
between vacant flat and Gia.
 Nim and Ema are not living on the same
floor.
From the above condition, case-3 gets
eliminated. Case-2 shows the final arrangement.

We have,
 N’s father bought three years before T’s
only daughter.
 At least one person bought after T’s
daughter.
 The number of persons bought before N’s
father is one more than the number of
Directions (11-15): persons bought after M, who is not N’s
11) Answer: E father.
12) Answer: B From the above condition, there are three
13) Answer: D (Both persons bought house possibilities.
either in odd or in even numbered year except
option d)

Click Here For Bundle PDF Course | support@guidely.in Page 8 of 10


Bank Po Mains PDF Course 2024
Reasoning Day -19 (Eng)

Again we have,
 L is the only son of the one who is the
father-in-law of O’s spouse.
 N’s spouse bought three years before the
one who bought two years after R’s son. Again we have,
 Only one person bought between N’s  The number of years difference between
spouse and O. T’s brother and O’s sister’s father-in-law is
 R’s only son is the brother-in-law of N and one less than the number of years
vice versa. difference between P’s spouse and U’s
 Neither O’s spouse nor T’s spouse has a father.
sibling.  .U’s father did not buy in an odd
 The brother-in-law of L is the son-in-law of numbered year.
R.  T’s spouse did not have a male kid.
 T’s daughter is the niece of the one who  M’s brother-in-law is unmarried.
is the daughter of R’s husband.  The number of persons bought before L’s
 Q is the nephew of L, who is the only nephew is one more than the number of
brother of T. persons bought after S’s son.
 L is unmarried.  P did not buy in an odd numbered year.
From the above condition, there are four From the above condition, Case-1, Case-2, and
possibility Case-2a get eliminated. Case-3 shows the final
arrangement.

Click Here For Bundle PDF Course | support@guidely.in Page 9 of 10


Bank Po Mains PDF Course 2024
Reasoning Day -19 (Eng)

D &4 % 2 E G 7 # 3 8 > L ^ £M @ 5 A6 O 9 ¥
For step III: All such numbers which are
immediately preceded and immediately followed
by the same element type are to be placed
immediate right of ‘^’ in ascending order.
D & % 2 E G 7 # 3 8 > L ^ 4 6 £M @ 5 A O 9 ¥
16. Answer: B
D & % 2 EG 7 # 3 8 > L ^ 4 6 £M @ 5 AO 9 ¥
Thus, four such letters are possible  E, G, A,
and O

17. Answer: B
D & % E G # > L ^ £M @ A O ¥
Thus, tenth element from the right end  #
Directions (16-20):
We have:
18. Answer: A
D&4%G27#E^38>LM@5A£6O9¥
D & % 2 E G 7 # 3 8 > L ^ 4 6 £M @ 5 A O 9 ¥
For step I: All such symbols which are
Thus, only two such symbols are possible >
immediately preceded by a vowel and
and £
immediately followed by a number, are to be
placed immediate right of ‘L’ in the same order
19. Answer: C
from left to right.
&%2EG7#38>^46£@5AO9¥
D & 4 % G 2 7 # E 3 8 > L ^ £M @ 5 A 6 O 9 ¥
Clearly, the twelfth element from the left end  4
For step II: All such letters which are
immediately preceded by a symbol and
20. Answer: D
immediately followed by a number are to be
placed in alphabetical order to the immediate
right of ‘2’.

Click Here For Bundle PDF Course | support@guidely.in Page 10 of 10


Bank Po Mains PDF Course 2024
Quantitative Aptitude Day -19 (Eng)

Quantitative Aptitude

Direction (1 – 5): Study the given information carefully and answer the given questions.
There are five students – Rajat, Anshul, Kanak, Sachin and Bhanu. All are attending examination X,which
consist of two papers (Paper 1 and Paper 2). Paper 1 for 100 marks and Paper 2 for 200 marks. Both
papers have different weightage in the final score. Paper 1 has 40% weightage and Paper 2 has 60%
weightage in the total weightage mark. The table given below shows the marks and weightage obtained
by each student in two papers and the total weightage Mark.

Note:
I. Total weightage mark= Weightage of obtained marks in Paper 1 + Weightage of obtained marks in
Paper 2
1. Naina’s final score is 20% more than the final b. 18.24%
score of Kanak. Naina obtained 180 marks on c. 10.66%
Paper 2. Find her obtained marks in Paper 1. d. 22.36%
a. 75 e. None of these
b. 80
c. 60 3. For the final score of the student, govt of India
d. 90 (GoI) announces Rs 500 for each percent above
e. None of these 60% to below 71%, Rs. 1000 for each percent
above 70% to below 76%, and Rs 2000 for each
2. If Anshul obtained 20 more marks in Paper 1 percent above 75%. Find the amount Sachin got
and the marks obtained by Anshul in Paper 2 from GoI.
remain the same. Find the percentage increase a. Rs.15000
in the final score of Anshul. b. Rs.12000
a. 12.36% c. Rs.18000

Click Here For Bundle PDF Course | support@guidely.in Page 1 of 14


Bank Po Mains PDF Course 2024
Quantitative Aptitude Day -19 (Eng)

d. Rs.21000 e. None of these


e. None of these
5. The sum of the total weightage marks of
4. If the marks obtained by Kanak in Paper 2are Sachin and Bhanu together is what percent more
increased by 20% and the total weightage mark than the sum of the total weightage marks of
of Kanak is 78, then find the percentage change Anshul and Kanak together.
in the marks obtained by Kanak in Paper 1. a. 15%
a. 30% b. 6%
b. 15% c. 12%
c. 25% d. 9%
d. 20% e. None of these

Direction (6 – 10): Study the given information carefully and answer the given questions.
There are five shopkeepers – A, B, C, D, and E who sells cold drink of three types – Pepsi, Thumbs Up,
and Coca-Cola. The bar graph given below shows the sum of Pepsi bottles and Thumbs Up bottles sold
by every shopkeeper. Also shows the difference between Thumbs Up bottles sold and Coca-Cola bottles
sold by each shopkeeper. The table given below shows the ratio of the Pepsi bottles sold to Coca-Cola
bottles sold by each shopkeeper. The number of Thumbs Up bottles sold by each shopkeeper is always
greater than the number of Coca-Cola bottles sold by each shopkeeper.

Click Here For Bundle PDF Course | support@guidely.in Page 2 of 14


Bank Po Mains PDF Course 2024
Quantitative Aptitude Day -19 (Eng)

6. If the sum of the total number of Coca-Cola container such that the ratio of Pepsi, Thumbs
bottles sold by A and C together is increased by Up, and Soda in the container is 12 : 9 : 13
d% then it will become 5 less than the sum of the respectively. Find the quantity of mixture in
total number of Thumbs Up bottles sold by D and vessel B is what percent of the quantity of
E together. Find the value of d. mixture in vessel A.
a. 12 a. 70%
b. 22 b. 50%
c. 15 c. 60%
d. 18 d. 75%
e. None of these e. None of these

7. There are 3 different quantity of Pepsi bottles 9. Shopkeeper B sold 52% of Pepsi, 58% of
sold by B such as 600 ml, 1 liter, and 2 liter and Thumbs Up, and 65% of Coca-Cola bottles he
its ratio is 5 : 13 : 8 respectively. Find the total has. Find the total number of all cold drink bottles
quantity of Pepsi sold by B in liters? together that B has.
a. 520 liter a. 4300
b. 480 liter b. 3700
c. 560 liter c. 6100
d. 640 liter d. 3500
e. None of these e. None of these

8. Tanu makes a mixture of Pepsi and Soda in 10. If all the bottles of cold drinks sold by E are 1
vessel A in the ratio 3 : 2 and another mixture of liter in quantity. The selling price of a 1-liter bottle
Thumbs Up and Soda in the ratio 9 : 5 in vessel of Pepsi ,Thumbs Up and Coca-Cola is Rs 55,
B. She mixes the mixture of both the vessel in a

Click Here For Bundle PDF Course | support@guidely.in Page 3 of 14


Bank Po Mains PDF Course 2024
Quantitative Aptitude Day -19 (Eng)

Rs 62 and Rs 60 respectively, Find the average c. Rs. 60.12


selling price of 1 bottle of cold drink sold by E. d. Rs. 54.68
a. Rs. 58.22 e. None of these
b. Rs. 64.12

Direction (11 – 15): Study the given information carefully and answer the given questions.
There are five villages- A, B, C, D, and E in which elections are held. The line graph given below shows
the difference between the winning candidate and runner-up candidate of elections in each village.

11. In village C, 80% of the population cast their number of voters in village D, if 15% of casted
votes, and 15% of the casted votes were votes are invalid, If all the people in that village
declared invalid. The winning candidate gets are eligible and all are casted their votes?
48% of the total casted votes. Find the a. 18300
population of village C. b. 17200
a. 32567 c. 15400
b. 33400 d. 16800
c. 36700 e. None of these
d. 39500
e. None of these 13. The winning candidate of village B gets
15900 more votes than the winning candidate of
12.In village D, the winning candidate gets 52% village E. The ratio of the number of votes got by
of total casted votes. Then Find the total the winning candidate of village E to the runner-

Click Here For Bundle PDF Course | support@guidely.in Page 4 of 14


Bank Po Mains PDF Course 2024
Quantitative Aptitude Day -19 (Eng)

up candidate of village B is 2:3. Find the number c. 2331


of votes got by the runner-up candidate of village d. 2731
E. e. None of these
a. 20142
b. 24012 Direction (16 – 18): Study the given information
c. 28412 carefully and answer the given questions.
d. 26114 In a township there are four blocks – A, B, C, and
e. None of these D. The dimension of all room/house in a block is
same but different from other blocks.
14. In village F, there are three candidates – P, The length of the rooms of block A is equal to the
Q, and R in the election. The ratio of votes P, Q, height of rooms of block C which is 25% more
and R get in the election is 6 : 4 : b. Q gets 1892 than the height of the rooms of block D. Ratio of
votes more than R. Find the value of b. the length of the rooms of blocks B, C, and D is 5
(Note: P is the winner of the election in village F : 6 :4. Height of rooms of block A is 50% more
and Q is the runner-up of the election in village than the breadth of rooms of block A. Breadth of
F.) rooms of block B is 3 meters more than the
a. 8 breadth of rooms of block A and 3 meters less
b. 4 than the breadth of rooms of block C. Height of
c. 7 rooms of block B is 9 meter and is same as the
d. 3 breadth of rooms of block D and is 3/4th to the
e. None of these height of the rooms of block A. Length of rooms
of block D is equal to the height of the rooms of
15. In village A, 70% of casted voters are literate block A. Length of the rooms of block B is 50%
and the rest are illiterate. The winning candidate more than the length of the rooms of block A.
gets 60% votes out of the total votes casted, of 16. If the height of the rooms of block C is
which 65% votes are literate voters. Find the increased by d% to fit a solid spherical ball of
number of illiterate voters who cast their votes to volume 4312/3 m3. Find the new volume of 15
runner-up candidate. rooms of block C.
(Note: There are only two candidates in the a. 58462 m3
election in village A and all the eligible voters b. 52920 m3
cast their vote and all votes are valid.) c. 51290 m3
a. 2557 d. 56460 m3
b. 2134 e. None of these

Click Here For Bundle PDF Course | support@guidely.in Page 5 of 14


Bank Po Mains PDF Course 2024
Quantitative Aptitude Day -19 (Eng)

oil of shopkeeper A is Rs. 60 more than the


17. A painter charges Rs. 12/m2 for painting a selling price of the hair oil of shopkeeper B. Find
house. Find the total amount painter gets for the selling price of the hair oil at which
painting the inner walls and ceiling of 42 rooms shopkeeper A sold it.
of the houses of block A. a. Rs. 212
a. Rs.258048 b. Rs. 204
b. Rs.248468 c. Rs. 180
c. Rs.254064 d. Rs. 192
d. Rs.263348 e. None of these
e. None of these
20. Station P is 900 km away from Station Q.
18. There are small wooden cubes of side 0.5 m Train A with a speed of M km/h and Train B with
which are fitted in the 13 rooms of block B such a speed of N km/h start at the same time from
that all 13 rooms are fully packed. Find the total Station P and Station Q respectively move
number of small wooden cubes which are fitted towards each other. After 4 hours from their
in 13 rooms of block B. starting journey when they are 252 km away
a. 136440 from each other, a technical glitch occurs in the
b. 145440 engine of Train A by which its speed becomes
c. 145880 4/5th of its original speed. After 1 more hour, both
d. 154440 trains are 18 km away from each other. Find the
e. 158480 speed of Train B.
a. 60 km/h
19. The cost price of hair oil is Rs. a. b. 80 km/h
Shopkeeper A marked it 50% above the cost c. 72 km/h
price and sell it after giving a discount of 15% on d. 90 km/h
the marked price. Shopkeeper B sold the article e. None of these
at a 12.5% profit. If the marked price of the hair

Click Here For Bundle PDF Course | support@guidely.in Page 6 of 14


Bank Po Mains PDF Course 2024
Quantitative Aptitude Day -19 (Eng)

Click Here to Get the Detailed Video Solution for the above given Questions
Or Scan the QR Code to Get the Detailed Video Solutions

Answer Key with Explanation


Direction (1 – 5): Marks obtained in Paper 1 by Marks obtained in Paper 1 by Kanak = 75
Rajat = 70 Weightage of obtained marks in Paper 1 by
Weightage of obtained marks in Paper 1 by Kanak = 75/100 *40
Rajat = 70/100 * 40 = 30
= 28 Marks obtained in Paper 2 by Kanak = 150
Weightage of obtained marks in Paper 2 by Weightage of marks obtained in Paper 2 by
Rajat = 48 Kanak = 150/200*60 = 45
Marks obtained in Paper 2 by Rajat = 48/60 * Final score of Kanak = 30 + 45
200 = 75
= 160 Weightage of marks obtained in Paper 2 by
Final score of Rajat = 28 + 48 = 76 Sachin = 55.5
Weightage of obtained marks in Paper 1 by Marks obtained in Paper 2 by Sachin = 55.5/60
Anshul = 24 *200 = 185
Marks obtained in Paper 1 by Anshul = 24/40 Weightage of marks obtained in Paper 1 by
*100 Sachin = 77.5 – 55.5
= 60 = 22
Weightage of marks obtained in Paper 2 by Marks obtained in Paper 1 by Sachin =
Anshul = 75 – 24= 51 22/40*100
Marks obtained in Paper 2 by Anshul = 51/60 = 55
*200 Final score of Sachin = 22 + 55.5 = 77.5
= 170 Weightage of marks obtained in Paper 1 by
The final score of Anshul = 75 Bhanu = 32

Click Here For Bundle PDF Course | support@guidely.in Page 7 of 14


Bank Po Mains PDF Course 2024
Quantitative Aptitude Day -19 (Eng)

Marks obtained in Paper 1 by Bhanu = Percentage increase in final score = ((83 –


32/40*100 = 80 75)/75) * 100
Marks obtained in Paper 2 by Bhanu = 165 = (8/75) * 100
Weightage of marks obtained in Paper 2 by = 10.66%
Bhanu = 165/200*60
= 49.5 3. Answer: A
Final score of Bhanu = 32 + 49.5 = 81.5 We know that sachin’s Total weightage Mark is
77.5
So,
Amount get by Sachin for 61% to 70 % = 500 *
10
= 5000
Amount get by Sachin for 71% to 75% = 1000 *
5
1. Answer: D = 5000
Naina’s final score = 75 *120/100 Amount got by Sachin for 76% to 77.5% = 2000
= 90 * 2.5
Marks obtained in Paper 2 by Naina = 180 = 5000
Weightage of marks obtained in Paper 2 by Total amount gets by Sachin = 5000 + 5000 +
Naina = 180/200*60 5000
= 54 = Rs.15000
Weightage of marks obtained in Paper 1 by
Naina = 90 – 54 4. Answer: D
= 36 New marks obtained in Paper 2 by Kanak = 150
Marks obtained in Paper 1 by Naina = 36/40*100 * (120/100)
= 90 = 180
2. Answer: C New weightage of obtained marks in Paper 2 by
New marks obtained in Paper 1 by Anshul = 60 Kanak = 180 * (3/10)
+ 20= 80 54
Weightage of marks obtained in Paper 1 by New weightage of obtained marks in Paper 1 by
Anshul = 80/100*40 = 32 Kanak = 78 – 54
Old final score of Anshul = 75 = 24
New final score of Anshul = 32 + 51= 83 Marks obtained in Paper 1 by Kanak = 24 * (5/2)

Click Here For Bundle PDF Course | support@guidely.in Page 8 of 14


Bank Po Mains PDF Course 2024
Quantitative Aptitude Day -19 (Eng)

= 60 Number of Coca-Cola bottles sold by A = 3 * 144


Percentage change in marks obtained in Paper 1 = 432
by Kanak = ((75 – 60)/75) * 100 Number of Thumbs Up bottles sold by A = 1340
= (15/75) * 100 – 720 = 620
= 20% Let the number of Pepsi bottles sold by B be 2b
and the number of Coca-Cola bottles sold by B
5. Answer: B be 3b.
Total weightage marks Sachin = 77.5 We get, (Pepsi + Thumbs Up) – (Thumbs Up –
Total weightage marks of Bhanu = 81.5 Coca-Cola)
The sum of the total weightage marks of Sachin = Pepsi + Coca Cola
and Bhanu together = 77.5 + 81.5 So, 2b + 3b = 1390 – 90
= 159 5b = 1300
Total weightage marks of Anshul = 75 b = 260
Total weightage marks of Kanak = 75 Number of Pepsi bottles sold by B = 2 * 260 =
The sum of the total weightage marks of Anshul 520
and Kanak together = 75 + 75 Number of Coca-Cola bottles sold by B = 3 * 260
= 150 =780
Reqd. percentage = ((159 – 150)/150) * 100 Number of Thumbs Up bottles sold by B = 1390
= (9/150) * 100 – 520 = 870
= 6% Let the number of Pepsi bottles sold by C be 3c
and the number of Coca-Cola bottles sold by C
Direction (6 – 10): Let the number of Pepsi be 4c.
bottles sold by A be 5a and the number of Coca- We get, (Pepsi + Thumbs Up) – (Thumbs Up –
Cola bottles sold by A be 3a. Coca-Cola)
We get, (Pepsi + Thumbs Up) – (Thumbs Up – = Pepsi + Coca Cola
Coca-Cola) 3c + 4c = 1591 - 72
= Pepsi + Coca Cola 7c = 1519
So, 5a + 3a = 1340 – 188 c = 217
= 8a = 1152 Number of Pepsi bottles sold by C = 3 * 217 =
a = 144 651
Number of Pepsi bottles sold by A = 5 * 144 = Number of Coca-Cola bottles sold by C = 4 * 217
720 = 868

Click Here For Bundle PDF Course | support@guidely.in Page 9 of 14


Bank Po Mains PDF Course 2024
Quantitative Aptitude Day -19 (Eng)

Number of Thumbs Up bottles sold by C = 1591


– 651 = 940
Let the number of Pepsi bottles sold by D be 5d
and the number of Coca-Cola bottles sold by D
be 2d.
We get, (Pepsi + Thumbs Up) – (Thumbs Up – 6. Answer: C
Coca-Cola) The sum of the number of Coca-Cola bottles
= Pepsi + Coca Cola sold by A and C together = 432 + 868
5d + 2d = 1475 – 124 = 1300
7d = 1351 The sum of the number of Thumbs Up bottles
d = 193 sold by D and E together = 510 + 990
Number of Pepsi bottles sold by D = 5 * 193 = = 1500
965 Then, 1300 * ((100 + d)/100) + 5 = 1500
Number of Coca-Cola bottles sold by D = 2 * 193 1300 * ((100 + d)/100) = 1495
= 386 ((100 + d)/100) = 1495/1300
Number of Thumbs Up bottles sold by D = 1475 100 + d = 115
– 965 = 510 d = 15
Let the number of Pepsi bottles sold by E be 2e
and the number of Coca-Cola bottles sold by E 7) Answer: D
be 5e. The number of 600 ml Pepsi bottles sold by B =
We get, (Pepsi + Thumbs Up) – (Thumbs Up – 520 * (5/26
Coca-Cola) = 100
= Pepsi + Coca Cola The number of 1 liter Pepsi bottles sold by B =
2e + 5e = 1332 – 135 520 * (13/26)
7e = 1197 = 260
e = 171 The number of 2-liter Pepsi bottles sold by B =
Number of Pepsi bottles sold by E = 2* 171 = 520 * (8/26)
342 = 160
Number of Coca-Cola bottles sold by E = 5 * 171 Total quantity of Pepsi sold by B = 100 * 0.6 +
= 855 260 * 1 + 160 * 2
Number of Thumbs Up bottles sold by E = 1332 = 60 + 260 + 320
– 342 = 990 = 640 liter

8) Answer: A

Click Here For Bundle PDF Course | support@guidely.in Page 10 of 14


Bank Po Mains PDF Course 2024
Quantitative Aptitude Day -19 (Eng)

Let the quantity of Soda in vessel A be 2x and Total amount E gets after selling Thumbs Up
the quantity of Soda in vessel B be 5y. bottles = 62 * 990
Then, the ratio of Pepsi, Thumbs Up, and Soda = 61380
in the container = 12 : 9 : 13 Total amount E gets after selling Coca-Cola
3x : 9y : 2x + 5y = 12 : 9 : 13 bottles = 60 * 855
From here we can see, 3x/9y = 12/9 = 51300
x/y = 4 Total amount E gets after selling all cold drinks =
Or, x = 4y 131490
Quantity of mixture in vessel A = 3x + 2x = 5x = Total cold drinks sold by E = 342 + 990 + 855
20y = 2187
Quantity of mixture in vessel B = 9y + 5y = 14 y The average selling price of 1 bottle of cold drink
Reqd. percentage = (14y/20y) * 100 sold by E = 131490/2187
= 70% = 60.12

9. Answer: B 11) Answer: D


Total number of Pepsi bottles B has = 520 * Let total votes casted be 100%
(100/52) Total valid casted votes = 100% – 15% = 85%
= 1000 Total casted votes runner up candidate get =
Total number of Thumbs Up bottles B has = 870 85% – 48% = 37%
* (100/58) The difference of votes between the winning
= 1500 candidate and runner-up candidate gets = 3476
Total number of Coca Cola bottles B has = 780 * 48% - 37% = 3476
(100/65) 11% = 3476
= 1200 1% = 316
Total number of all cold drinks bottles B has = 100% = 31600
1000 + 1500 + 1200 Total number of casted votes = 31600
= 3700 Population of village C = 31600 * (100/80)
= 39500
10. Answer: C
Total amount E gets after selling Pepsi bottles = 12) Answer: B
55 * 342 Total valid votes = 100% – 15%
= 18810 = 85%

Click Here For Bundle PDF Course | support@guidely.in Page 11 of 14


Bank Po Mains PDF Course 2024
Quantitative Aptitude Day -19 (Eng)

Valid votes runner-up candidate gets = 85% – The difference between the number of votes
52% winner candidate gets and the number of votes
= 33% runner up candidate gets = 3784
The difference between the number of votes 6x – 4x = 3784
winner candidate gets and the number of votes 2x = 3784
runner up candidate gets = 3268 x = 1892
52% - 33% = 3268 The number of votes P gets = 6 * 1892
19% = 3268 = 11352
1% = 172 The number of vote Q get = 4 * 1892
100% = 17200 = 7568
The total number of voters in village D is 17200. The number of vote R get = 7568 – 1892
= 5676
13) Answer: A The ratio of the number of votes get by P, Q, and
Let the number of votes the winner candidate of R = 11352 : 7568 : 5676
village E gets be x. =6:4:3
Number of votes got by winner candidate of The value of b is 3.
village B = x + 15900
Number of votes got by runner-up candidate of 15) Answer: C
village B = x + 15900 – 4392 Let total votes cast be 100%
= x + 11508 Percentage of votes winner candidate get = 60%
The ratio of the number of votes got by winner Percentage of votes runner-up candidate gets =
candidate of village E to the number of votes got 40%
by runner up candidate of village b = 2 : 3 The difference between the number of votes
x/(x + 11508) = 2 /3 winner candidate gets and the number of votes
3x = 2x + 23016 runner up candidate gets = 5180
x = 23016 60% - 40% = 5180
The number of votes got by runner up candidate 20% = 5180
of village E = 23016 – 2874 1% = 259
= 20142 100% = 25900
Total number of casted votes = 25900
14) Answer: D Number of literate voters = 25900 * (70/100)
Let the number of votes P gets be 6x and the = 18130
number of votes Q gets be 4x respectively. Number of illiterate voters = 25900-18130=7770

Click Here For Bundle PDF Course | support@guidely.in Page 12 of 14


Bank Po Mains PDF Course 2024
Quantitative Aptitude Day -19 (Eng)

Number of votes winner candidate get = 25900 *


(60/100)= 15540
Number of literate voters voting to winner
candidate = 15540*(65/100) =10101
Number of illiterate voters who votes to winner
candidate = 15540 – 10101= 5439
16) Answer: B
Number of illiterate voters voting for runner-up
The volume of the spherical ball = 4312/3 m2
candidate = 7770 – 5439
(4/3) ∏r3 = 4312/3
= 2331
(4/3) (22/7) r3 = 4312/3
r3 = 49 * 7
Direction (16 – 18): Height of rooms of block B =
r = 7 meter
9 meter
The diameter of the sphere is the minimum
The breadth of rooms of block D = 9 meter
dimension of the length of the room.
Height of rooms of block A = 9 * (4/3)
Diameter = 2 * 7
= 12 meter
= 14 m
Breadth of rooms of block A = 12 * (2/3)
The new height of the room = 14 m
= 8 meter
Volume of 15 rooms of block C = 18 * 14 * 14 *
Length of rooms of block D = 12 meter
15
Length of the rooms of block B = 9 * (5/3)
= 52920 m3
= 15 meter
The breadth of rooms of block B = 8 + 3
17) Answer: A
= 11 meter
Surface area of four walls of the room block A =
Length of rooms of block C = 15 * (6/5)
2 (10 * 12) + 2 (8 * 12)
= 18 meter
= 240 + 192
The breadth of rooms of block C = 11 + 3
= 432 m2
= 14 meter
The surface area of the ceiling of the room of
Length of the rooms of block A = 15 * (2/3)
block A = 10 * 8
= 10 meter
= 80 m2
Height of the rooms of block C = 10 meter
Total surface area to be painted of the room of
Height of the rooms of block D = 10 * (4/5)
block A = 512 m2
= 8 meter
Total cost to paint a room of block A = 512 * 12
6144

Click Here For Bundle PDF Course | support@guidely.in Page 13 of 14


Bank Po Mains PDF Course 2024
Quantitative Aptitude Day -19 (Eng)

Total cost to paint 42 rooms of block A = 6144 * 1.5a – 1.125a = 60


42 0.375a = 60
= Rs.258048 a = 160
The selling price of hair oil at which shopkeeper
18) Answer: D A sells = 1.275a
Volume of a room of block B = 15 * 11 * 9 = 1.275 * 160
= 1485 m3 = 204
Volume of a small wooden cube = 0.5 * 0.5 * 0.5
= 0.125 m3 20. Answer: C
Number of small wooden cubes fitted in a room Speed of Train A = M km/h
of block B = 1485/0.125 Speed of Train B = N km/h
= 11880 According to the question,
Number of small wooden cubes fitted in 13 (M + N) * 4 = 900 – 252
rooms of block B = 11880 * 13 (M + N) * 4 = 648
= 154440 Or, (M + N) = 162 …. (i)
And, ((4/5) M + N) * 1 = 144
19) Answer: B Or, 4M+5N = 720 …. (ii)
The cost price of Hair oil = a Subtracting eqn. ii from eqn. i
The marked price of hair oil marked by Multiplying eqn (i) with 4 both side, in order
shopkeeper A = 1.5a eliminate M.
Selling price of hair oil at which shopkeeper A 4M + 4N = 648
sell = 1.5a ((100 – 15)/100) (-)4M+(-)5N = (-)720
= 1.275a ------------------------
The selling price of hair oil at which shopkeeper -N = -72
B sells = a (112.5/100) N = 72 km/h
= 1.125a The speed of Train B is 72 km/h.
According to the question,

Click Here For Bundle PDF Course | support@guidely.in Page 14 of 14


Bank Po Mains PDF Course 2024
English Day - 19

English Language
Directions (1-5): Given below are a few 2. Which of the following option fits best in the
questions based on the paragraph given with a second blank?
few blanks to be filled from the appropriate A. convinced
options in order to make a meaningful sentence. B. perceived
If none of the options are correct then choose C. conceived
option E as your answer. D. treated
The Good Book is being (a)_____________ like a E. None of the above
bad book in Utah after a parent frustrated by
efforts to ban materials from schools 3. Which of the following option fits best in the
(b)___________ a suburban district that some third blank?
Bible verses were too vulgar or violent for A. secondary
younger children. And the Book of Mormon could B. senior secondary
be next. C. elementary
The 72,000-student Davis School District north of D. college
Salt Lake City removed the Bible from its E. None of the above
(c)___________ and middle schools while keeping
it in high schools after a committee reviewed the 4. Which of the following option fits best in the
(d)_____________ in response to a parental fourth blank?
complaint. The district has removed other titles, A. scripture
including Sherman Alexie’s “The Absolutely True B. sculpture
Diary of a Part-Time Indian” and John Green’s C. caricature
“Looking for Alaska,” following a 2022 state law D. despotic
requiring districts to include parents in decisions E. None of the above
over what (e)______________ “sensitive material.”
1. Which of the following option fits best in the 5. Which of the following option fits best in the
first blank? fifth blank?
A. fractioned A. meddled
B. termed B. constitutes
C. pasted C. dismantle
D. treated D. desecrate
E. None of the above E. None of the above

Click Here For Bundle PDF Course | support@guidely.in Page 1 of 7


Bank Po Mains PDF Course 2024
English Day - 19

Directions (6-10): Given below are a few White House nomination (e)/, papers filed with
questions with jumbled parts in order to the US Federal Election Commission (f).
rearrange them and make a contextually correct A. abcdf
sentence. One of the part is fixed and rest to be B. acdfb
rearranged if needed. If no rearrangement is C. bcdfa
required then choose option E as your answer. D. fadcb
6. The ties between India and Nepal (a)/ came E. No rearrangement is required
under severe strain after Kathmandu published
(b)/ a new political map in 2020 (c)/ Indian 9. Years of financial mismanagement (a)/ have
territories Limpiyadhura (d) that showed three pushed Pakistan's economy to the limit (b)/,
(e), Kalapani and Lipulekh as part of Nepal (f). exacerbated by a global energy crisis (c)/ that
A. abced submerged a third of the country (d)/ and
B. acdeb devastating floods (e)/ in 2022 (f).
C. bcdae A. abced
D. bedca B. acdeb
E. No rearrangement is required C. adebc
D. acedb
7. Under international law (a)/, ships are allowed E. No rearrangement is required
to sail through foreign EEZs (b)/, but
unauthorised surveys are not permitted and 10. Pakistan needs billions of dollars (a)/ in
China's operations (c)/ in the South China Sea financing to service (b)/ staggering levels of
have long been problematic (d)/ for countries in external debt (C)/, and foreign exchange
the region, as Beijing claims most of the energy- reserves (d)/ barely enough for a month of
rich sea (e)/, including foreign EEZs (f). imports (e)/ have dwindled to just $4.2 billion (f).
A. bcedf A. acdfe
B. cdefb B. adfec
C. bdcef C. cedfa
D. edcab D. cdeaf
E. No rearrangement is required E. No rearrangement is required

8. showed Monday (a)/ Republican former vice Directions (11-15): Given below are a few
president Mike Pence (b)/ has launched his questions with three highlighted words in each of
hotly-anticipated challenge (c)/ to his one-time them. You have to find the synonyms of the
boss Donald Trump (d)/ for the party's 2024 words highlighted in the same sequence as they

Click Here For Bundle PDF Course | support@guidely.in Page 2 of 7


Bank Po Mains PDF Course 2024
English Day - 19

are given in the question. If none of the options A. accord, compeer, bodacious
are correct then choose option E as your answer. B. compeer, accord, bodacious
11. If we mark spots around the globe where the C. compeer, bodacious, accord
militaries of the world’s superpowers could run D. bodacious, accord, compeer
into each other, both the South China Sea and E. None of the above
the Black Sea would top the chart. Earlier it was
rare that the world's most powerful nations would 14. In response to Russia's invasion of Ukraine,
engage in assertive manoeuvring over Western powers imposed a price cap on Russian
international waters, but now that is a common crude of $60 a barrel.
sight. While non-EU countries can import seaborne
A. involve, conspire, enterprising Russian crude, Western shipowners and insurers
B. enterprising, conspire, involve are prohibited from handling such cargoes
C. conspire, involve, enterprising unless they are sold at or below that price.
D. involve, enterprising, conspire A. count, levy, interdicted
E. None of the above B. interdicted, levy, count
C. levy, count, interdicted
12. While US-China tensions may have been D. interdicted, count, levy
given new impetus under the Trump E. None of the above
administration which levied tariffs broadly and
imposed sanctions on Huawei, the friction has 15. The aggressive push by Reliance to increase
continued unabated under President Joe Biden popular content on its streaming platform, in
as both countries vie for global tech pre- addition to premium sports content, could lead to
eminence. a massive disruption in the OTT streaming
A. goad, incessant, paramountcy space, which has been dominated mostly by
B. goad, paramountcy, incessant foreign players.
C. paramountcy, incessant, goad A. overpower, fierce, disrangement
D. incessant, goad, paramountcy B. overpower, disrangement, fierce
E. None of the above C. fierce, overpower, disrangement
D. fierce, disrangement, overpower
13. Reliance’s JioCinema has signed another E. None of the above
major streaming deal to take on rivals Netflix,
Disney+ Hotstar and Amazon in India after Directions (16-20): Given below are a few
striking a partnership with Warner Bros questions with blank in each of them in the last to
Discovery in April. be filled with an appropriate option in order to

Click Here For Bundle PDF Course | support@guidely.in Page 3 of 7


Bank Po Mains PDF Course 2024
English Day - 19

make the sentence meaningfully correct. If none India, fintech incubator Rainmatter, and
of the options are correct then choose option E Rainmatter Foundation,
as your answer. _________________________.
16. The BRICS brings together five of the largest A. which is still the smallest curated thing.
developing countries of the world, representing B. which is known for its sports adventure.
41 per cent of the global population, C. which supports climate-related non-profits
____________________________. D. which is known for its mouth-watering taste.
A. pertaining to the risk involved they just denied E. None of the above
doing that.
B. the following problem is the main cause of the 19. Musk's net worth dropped below $200 billion
rift. last year as investors dumped Tesla's shares on
C. also the BRICS is the one of the toughest worries the top executive and largest
ruled organisations. shareholder of the world's most valuable electric-
D. 24 per cent of the global GDP, and 16 per vehicle maker is more preoccupied with Twitter,
cent of the global trade. ____________________.
E. None of the above A. which he dumped at some billion dollars.
B. which he bought for $44 billion.
17. Zerodha co-founder Nikhil Kamath became C. which he took as a loan from Jake Dorsey.
the youngest and fourth Indian to join 'The Giving D. which is one of the most important factor in
Pledge', where the wealthiest families and deciding the future of the company.
individuals _________________________________. E. None of the above
A. doesn’t provide information to the needy and
use the cyber data to manipulate election of the 20. While the RBI acknowledged that the
developed nations. domestic economy faces some challenges due
B. doesn’t provide shelter to the needy. to the sluggish global economic outlook, it said
C. is country’s largest broking company and still strong macroeconomic factors, financial
bootstrapped. conditions and expected dividends
D. globally commit to giving the majority of their from______________________.
fortune to charitable causes. A. past reforms put the country in an
E. None of the above advantageous position
B. that the repo and reverse repo rates are rising
18. Kamath founded Zerodha in 2010, as well as from the last five monetary policy meetings.
Gruhas for private investments, hedge fund True C. that the last meeting was not fruitful due to
Beacon that manages wealth for ultra HNIs in less convincing environment.

Click Here For Bundle PDF Course | support@guidely.in Page 4 of 7


Bank Po Mains PDF Course 2024
English Day - 19

D. future prediction of inflation cannot be done as E. None of the above


situation is changing everyday.
Click Here to Get the Detailed Video Solution for the above given Questions
Or Scan the QR Code to Get the Detailed Video Solutions

Answer Key with Explanation


1. Answer: D 4. Answer: A
Here, treated fits best in the first blank as per the Here, scripture fits best in the fourth blanks as it
context of the sentence because here book is means the sacred writings of Christianity
being treated as bad and therefore it fits best. contained in the Bible and here the same is
So, option D is the correct answer. being talked about. So, option A is the correct
answer.
2. Answer: A Sculpture means a statue made out of hands.
Here, convinced fits best in the second blank as Caricature means a picture or description of
it means to be completely sure about something somebody that makes his or her appearance or
and here it fits perfectly as per the context of the behaviour funnier and more extreme than it
sentence. So, option A is the correct answer. really is.
Perceived means to notice or realize something.
Conceived means create by fertilizing an egg. 5. Answer: B
Here, constitutes fits best in the fifth blank as it
3. Answer: C means to contain and here it fits best as the
Here, elementary fits best as it is said in the content of book is being talked with respect to
context of a type of school and elementary the sensitivity or sensitive content. So, option B
schools are the first school for a child. So, option is the correct answer.
C is the correct answer. Desecrate means to damage a place of religious
importance or treat it without respect.

Click Here For Bundle PDF Course | support@guidely.in Page 5 of 7


Bank Po Mains PDF Course 2024
English Day - 19

6. Answer: A 10. Answer: A


Here, the first part after rearrangement will be a Here, the first part after rearrangement will be a
which is introductory part starting with the ties which gives introductory part as need of billions
between India and Nepal followed by b which of dollars for Pakistan followed by b which is
gives the reason for the stress mentioned fixed followed by c which gives reason for need
followed by c and then d and finally ends with e of money followed by d then f and finally ends
and f in which f is already fixed. So, option A is with e which shows how much amount is left with
the correct answer. them. So, option A is the correct answer.

7. Answer: E 11. Answer: D


Here, no rearrangement is required and Here, engage means to provide occupation for
therefore, option E is the correct answer. and here involve is the synonym, assertive
means disposed to or characterized by bold or
8. Answer: C confident statements and behaviour and here
Here, the first part after rearrangement will be b enterprising means same, manoeuvring means
as it gives introductory part as former vice to move to a different position using skill and
president followed by c which gives information here conspire means the same. So, option D is
about a challenge launched by him followed by d the correct answer.
and then e which is fix followed by f which gives
information about details submitted by him and 12. Answer: C
finally ends with a which is ending part. So, Here, impetus means a driving force and here
option C is the correct answer. goad is the synonym, unabated means being at
full strength or force and here incessant means
9. Answer: A the same, eminence means a position of
Here, the first part after rearrangement will be a prominence or superiority and here paramountcy
which is introductory part as it gives information means the same. So, option C is the correct
about some mismanagement from years answer.
followed by b which gives information about
whom it is referring followed by c then e which 13. Answer: A
gives information about the floods and then Here, deal means to take action with regard to
followed by d and finally ends with f which is someone or something and here accord, rivals
fixed. So, option A is the correct answer. means one of two or more striving to reach or
obtain something that only one can possess,

Click Here For Bundle PDF Course | support@guidely.in Page 6 of 7


Bank Po Mains PDF Course 2024
English Day - 19

striking means attracting attention or notice Here, the context of the sentence is about
through unusual or conspicuous qualities and BRICS and its importance and here option D
here bodacious means the same. So, option A is which is giving information about the contribution
the correct answer. of BRICS in global economy is the correct
option. So, option D is the correct answer.
14. Answer: E
Here, imposed means to establish or apply by 17. Answer: D
authority and here levy means the same, import Here, the context of the sentence is about the
means to bring from a foreign or external source giving pledge and option D explains the use of
and here count is somewhat near to the meaning fund donated under this pledge. So, option D is
but not exact synonym as per the context of the the correct answer.
sentence, prohibited means not permitted or
forbidden by the authority and here interdicted 18. Answer: C
means the same. So, option E is the correct Here, the blank should be filled with the
answer. information about rainmatter foundation and
option C gives the information correctly as per
15. Answer: D the context. So, option C is the correct answer.
Here, aggressive means marked by combative
readiness or tending toward or exhibiting and 19. Answer: B
here fierce means the same, disruption means a Here, the context of the sentence is about twitter
break or interruption in the normal course or and in the end as per the context it should end
continuation of some activity and here with the price at which it is bought and option B
derangement means the same, dominated explains it correctly. So, option B is the correct
means to exert the supreme determining or answer.
guiding influence on something and here
overpower means the same. So, option D is the 20. Answer: A
correct answer. Here, the context of the sentence is about
domestic economic situations as per the RBI and
16. Answer: D here option A is contextually fitting in the blank.
So, option A is the correct answer.

Click Here For Bundle PDF Course | support@guidely.in Page 7 of 7


Bank Po Mains PDF Course 2024
Reasoning Day -20 (Eng)

Reasoning Aptitude
Directions (1-5): Answer the questions based on are arranged in such a way to form a third
the information given below. meaningful word.
A nine-letter word (may or may not be 1) Which of the following letter is placed
meaningful) with no repeated letters is arranged immediate left of W in arrangement 2?
in three arrangements to form the series. The a) K
same letters are used in each of the b) The letter which is second to the right of F
arrangements. c) No letter
Arrangement 1: d) E
R is placed fourth to the left of W but none of the e) A
letters is placed at the end. Only two letters are
placed between R and I. The number of letters 2) What will be the first three letters meaningful
placed to the left of I is the same as to the right of word formed?
F.E is placed third to the left of T.T is placed a) RAT
adjacent to neither W nor R. As many letters b) ART
placed between F and T as between W and D.A c) TRA
is placed to the left of K.The number of letters d) TAR
placed between A and R is one more than the e) None of the above
number of letters placed between K and D.
Arrangement 2: 3) What is the position of I from the right end in
All the given letters from arrangement 1 are Arrangement 2?
arranged in alphabetical order from right to left. a) Fourth
Thus, forms the new arrangement of the series. b) Fifth
Arrangement 3: c) Second
The first letter and third letter from the left end in d) Third
arrangement 1 are taken along with the exact e) None of these
middle letter in arrangement 1 to form a first
meaningful word. R is placed to the right of A. 4) Which of the following combination of the
The exact middle letter of arrangement 1 is not letters are arranged adjacent to each other in
the first letter of any meaningful words. The arrangement 3?
fourth letter from the left end in arrangement 1 I. WK
and the third letter from the right end of both II. TF
arrangements 1 and 2 are taken to form a III. AT
second meaningful word. The remaining letters a) Only I

Click Here For Bundle PDF Course | support@guidely.in Page 1 of 10


Bank Po Mains PDF Course 2024
Reasoning Day -20 (Eng)

b) Only III immediately before S. R is not married to Q. At


c) Both I and II least two persons attend the event between S’s
d) All I, II, and III daughter and Q.
e) None of these 6) How V is related to P?
a) Mother-in-law
5) How many letters are there between T and K b) Son
in Arrangement 3? c) Brother
a) Four d) Niece
b) Same as between E and I in Arrangement 1 e) None of these
c) More than Five
d) Same as between R and F in Arrangement 2 7) Who among the following persons attend the
e) Five event between V and S’s son?
I. The one who attends the event on Friday
Directions (6-10) Study the following information II. The one who attends the event three persons
carefully and answer the below questions. after S’s husband
Seven persons from a three generations family – III. S
P, Q, R, S, T, U, and V attend the event during a a) Only II
week from Monday to Sunday. Two married b) Both I and III
couples are in the family. No single parent has a c) Both II and III
child. d) All I, II, and III
T’s daughter-in-law attends three persons after e) None
T. T neither attends on Monday nor Wednesday.
U is the only son of P’s brother. The number of 8) Who among the following person attends the
persons attending before U is one more than the event on Tuesday?
number of persons attending after V. Both U and a) R
V attend the event on neither Tuesday nor b) U’s grandfather
Thursday. S is the sister-in-law of P and vice- c) S
versa. Only two persons attend the events d) Q
between P and Q, who is the father of V. The e) None of these
number of persons attending the event between
U and Q is the same as the number of persons 9) Four of the following five are alike in a certain
attending the event between V and P. R, who is way as per the given arrangement and hence
the mother of P and attends the event three form a group. Find the one that doesn’t belong to
persons after the one who attends the event that group.

Click Here For Bundle PDF Course | support@guidely.in Page 2 of 10


Bank Po Mains PDF Course 2024
Reasoning Day -20 (Eng)

a) The one who attends the event on Monday Q, who sits to the right of P. Neither P nor S is
b) R facing F. D sits second to the right of B but none
c) The one who attends the event two persons of them neither sits adjacent chair of C nor at the
before R end. S sits third to the left of U. Only two persons
d) V sit between P and T. E sits adjacent to the one
e) The one who attends the event on Thursday who is facing T.
11) What is the position of D with respect to the
10) Which of the following information is not one who sits opposite to U?
true? a) Third to the left
a) P is the sister of the one who attends the b) Fourth to the right
event on Monday c) Immediate left
b) More than three persons attend the event d) Second to the right
before U’s sister e) None of these
c) R attends the event immediately after P’s
daughter 12) Four of the following five are alike in a certain
d) S attends the event on Friday way as per the given arrangement and thus form
e) All the above statements are true a group. Find the one that does not belong to
that group.
Directions (11-15) Study the following a) The one who sits immediate right of D
information carefully and answer the below b) C
questions. c) The one who sits third to the right of U
Fourteen chairs are placed in two parallel rows in d) S
such a way that in row 1 – persons A, B, C, D, E, e) The one who sits opposite to R
and F are sitting and all of them face south
whereas, in row2 – persons P, Q, R, S, T, and U 13) How many persons are sitting between F and
are sitting and all of them face north. Row 1 is E?
north of row 2. One chair in each row is vacant. a) Three
Neither the chairs at the end nor the chairs facing b) As many persons sitting to the left of U
each other are vacant. c) As many persons sitting between S and P
F sits third from the extreme right end. Only one d) One
person sits between F and C. The one who is e) Either B or C
facing C sits immediate right of Q. Only three
persons sit between P and R, and either of them 14) Who among the following person sits
sits at the end. Neither P nor R sits adjacent to adjacent chair of C?

Click Here For Bundle PDF Course | support@guidely.in Page 3 of 10


Bank Po Mains PDF Course 2024
Reasoning Day -20 (Eng)

a) Only B neither holds the first rank nor ranks before X. Y


b) Both A and E holds the rank after A but before E. Z holds the
c) Only E rank after B and is four ranks after C. Y is not
d) Both A and F secured the first rank.
e) None of these 16) In which direction is C standing with respect to
A?
15) Which of the following statement is not true? a) North-east
a) One person sits between B and E b) South-east
b) D is facing the one who sits immediate right of c) North-west
S d) South-west
c) T is facing C e) Cannot be determined
d) Two persons are sitting between Q and S
e) All the above statements are true 17) How many persons rank after E in the final
arrangement?
Directions (16-20): Study the following information a) 1
carefully and answer the questions given below. b) 2
Nine students viz., A, B, C, D, E, W, X, Y and Z c) 3
have cleared the class test and are ranked from 1 d) 4
to 9. They are standing in the ground at a certain e) None
distance. No two persons secured the same rank.
Note: The distance between any two students is 18) What is the approximate shortest distance
the twice of the lowest rank among them. For between D and A and who ranks according to that
example, if P is the first rank and Q is the second value?
rank, the distance between them is 4m. a) 5, Y
X is to the west of A and is the immediate previous b) 7, E
rank of D who is to the north of X. The number of c) 5, B
persons ranked between D and W is an odd prime d) 7, B
number where no one ranks after W who is to the e) 5, E
east of D. E ranks exactly between D and W and
is to the south of Y. C stands to the south of W 19) Find the odd one out according to their
and ranks before E. Z stands to the west of C and standing position in the ground?
ranks after Y. The direction of Y from E is same as a) D-Y
the direction of B from Z. B holds the immediate b) X-B
next rank to E and stands to the west of E. C c) A-E

Click Here For Bundle PDF Course | support@guidely.in Page 4 of 10


Bank Po Mains PDF Course 2024
Reasoning Day -20 (Eng)

d) Z-W a) A,B,C,E
e) C-D b) X,A,C,W
c) A,Y,E,B
20) Who among the following group of persons d) Z,W,A,B
hold the prime numbered ranks? e) X,D,Y,B

Click Here to Get the Detailed Video Solution for the above given Questions
Or Scan the QR Code to Get the Detailed Video Solutions

Answer Key with Explanation


Directions (1-5):
1) Answer: C
2) Answer: B
3) Answer: B
4) Answer: C Again we have,

5) Answer: B  The number of letters placed to the left of

Final Arrangement: I is the same as to the right of F.


 E is placed third to the left of T.
 T is placed adjacent to neither W nor R.
From the above condition, Case-1 and case 3
get eliminated.

We have,
 R is placed fourth to the left of W but none
of the letters is placed at the end.
 Only two letters are placed between R
Again we have,
and I.
 As many letters placed between F and T
From the above condition, there are four
as between W and D.
possibilities.

Click Here For Bundle PDF Course | support@guidely.in Page 5 of 10


Bank Po Mains PDF Course 2024
Reasoning Day -20 (Eng)

 A is placed to the left of K.  The fourth letter from the left end in
 The number of letters placed between A arrangement 1 and the third letter from
and R is one more than the number of the right end of both arrangements 1 and
letters placed between K and D. 2 are taken to form a second meaningful
From the above condition, Case-4 get word.
eliminated. Case-2 shows the final arrangement.

Again we have,
 The remaining letters are arranged in
such a way to form a third meaningful
word.

Arrangement 2:
Again we have,
 All the given letters from arrangement 1 Therefore the final arrangements of the given

are arranged in alphabetical order from series are as follows.

right to left. Thus, forms the new


arrangement of the series.

Arrangement 3: Directions (6-10):


Again we have, 6) Answer: D
 The first letter and third letter from the left 7) Answer: D
end in arrangement 1 are taken along 8) Answer: B
with the exact middle letter in 9) Answer: A(All the persons are of the same
arrangement 1 to form a first meaningful gender except option A)
word. 10) Answer: C
 R is placed to the right of A.
 The exact middle letter of arrangement 1
is not the first letter of any meaningful
words.

Again we have,

Click Here For Bundle PDF Course | support@guidely.in Page 6 of 10


Bank Po Mains PDF Course 2024
Reasoning Day -20 (Eng)

We have:
 T’s daughter-in-law attends three persons
after T. Again, we have:
 T neither attends on Monday nor  S is the sister-in-law of P and vice-versa.
Wednesday. That means, S must be the mother of U.
That means, in case (1) T visited on  Only two persons attend the event
Tuesday, in case (2) T visited on between P and Q, who is the father of V.
Thursday.  The number of persons attending the
 U is the only son of P’s brother. event between U and Q is the same as
 The number of persons attending before the number of persons attending the
U is one more than the number of event between V and P.
persons attending after V.  R, who is the mother of P and attends the
 Both U and V attend the event on neither event three persons after the one who
Tuesday nor Thursday. attends the event immediately before S.
That means, in case (1) & case (2) U  R is not married to Q.
attends the event on Wednesday, in case Since, only two married couples are in the
(1a) & case (2a) U attended the event on family, and Q is not married to R, thus Q
Saturday. must be married to S, and T must be
Based on the above given information we have: married to R.
That means, in case (1) Q attends the
event on Monday, in case (1a) Q attends
the event on Thursday, case (2) & case
(2a) are not valid.
Now, since, U is the only son of P’s
brother, and Q is the father of V, thus, V
must be the sister of U.

Click Here For Bundle PDF Course | support@guidely.in Page 7 of 10


Bank Po Mains PDF Course 2024
Reasoning Day -20 (Eng)

 At least two persons attend the event


between S’s daughter and Q.
That means, case (1a) is not valid.
Based on the above given information we have:

We have:
 F sits third from the extreme right end.
 Only one person sits between F and C.

Case (2) & case (2a) are not valid as R attends  The one who is facing C sits immediate

the event two persons after S, case (1a) At least right of Q.

two persons attend the event between V and Q. That means, in case (1) C sits third from

For Blood Relation: the left end of row1, in case (2) C sits
second from the left end of row1.
Based on the above given information we have:

Direction (11-15):
11) Answer: D Again, we have:
12) Answer: B  Only three persons sit between P and R,
13) Answer: B and either of them sits at the end.
14) Answer: C  Neither P nor R sits adjacent to Q, who
15) Answer: E sits to the right of P.
 Neither P nor S is facing F.
That means, in case (1) P sits at the left
end of row2, in case (1a) & case (2) P sits
second from the left end of row 2.
Based on the above given information we have:

Click Here For Bundle PDF Course | support@guidely.in Page 8 of 10


Bank Po Mains PDF Course 2024
Reasoning Day -20 (Eng)

Case (1) & case (1a) are not valid as D sits


second to the right of B.
Again, we have:
 S sits third to the left of U.
Since, S doesn’t sit facing F.
That means, S sits at the left end of row2.
 Only two persons sit between P and T.
 E sits adjacent to the one who is facing T.
That means, E sits immediate left of C.
Based on the above given information we have:

Again, we have:
 D sits second to the right of B but none of
them neither sits adjacent chair of C nor
at the end.
That means, in case (2) D sits second
from the right end of row1, case (1) &
case (1a) are not valid. Directions (16-20):
Based on the above given information we have: 16) Answer: B
17) Answer: C
18) Answer: D
19) Answer: E
20) Answer: E
Following the statements given above, the rank,
distance and the directions of each student are
obtained as follows.

Click Here For Bundle PDF Course | support@guidely.in Page 9 of 10


Bank Po Mains PDF Course 2024
Reasoning Day -20 (Eng)

Click Here For Bundle PDF Course | support@guidely.in Page 10 of 10


Bank Po Mains PDF Course 2024
Quantitative Aptitude Day -20 (Eng)

Quantitative Aptitude

Directions (1-4): Read the following information carefully and answer the questions based on it.
A shopkeeper sold five articles – P, Q, R, S and T. Further partial information in given in table below.

Note:
a) Value of ‘n’ is 4 times of larger root of given equation.
K2 – 32P + 252 = 0
b) Article T is marked up Rs. 1280 more than its cost price, which is 28% more than profit earned on
article S.
c) Discount given on article Q is 16.66%, and selling price of S is Rs. (5m + 7n + 4a + 296).
d) Value of ‘m’ is twice of missing value in the given sequence.
140, 136, 161, (?), 329, - 200
1) If cost price of P is 87.5% more than that of T, A.100/7
and P is sold after discount of (2L – 210) %, then B.40
find which of the following is/are is possible value C.250
of L? D.2/5
I. 1.5m E. None of these
II. (b – n – 8)
III. 2.4(b – a) 3) If Profit % given on article Q is thrice as that of
IV. 240 Discount%, then find difference between cost
A.I and III only price of Q and S.
B.II and IV only A.Rs. 160
C.I, II, and III only B.Rs. 80
D.I, II, III, and IV C.Rs. 120
E. None of these D.Rs. 240
E. None of these
2) If marked price of T is Rs. 80 less than cost
price of R, which is sold after discount of Rs. 400 4) If the discount given on article T is (b – a) %
or (5L/14)%. Find the value of L. and sold at profit of L%, while marked price of S

Click Here For Bundle PDF Course | support@guidely.in Page 1 of 14


Bank Po Mains PDF Course 2024
Quantitative Aptitude Day -20 (Eng)

is Rs. 400 less than that of P, and S sold after Statement I: Speed of train B is 72 km/hr and
giving discount of D%. Find (L% - D %). length of train A is 225m.
A.30% Statement II: Ratio of speed of train A and train B
B.20% is 5:4 and length of train B is 240m.
C.25% Statement III: Train A crosses 125 m long bridge
D.40% in 14 sec and cross a pole in 9 sec.
E. None of these A. Any one
B. Only I
Directions (5-7): Question consists of three C. Only II
statements, I, II and III. You’re required to decide D. Either III or I and II together
that data in which statement(s) is sufficient. E. Only II and III
5) 20% of total scores obtained in the first 4
matchesby a batsmanis 88. Find the average of 7) Find the time taken by B alone to complete the
all the scores obtained by him in five matches (I, work?
II, III, IV and V)? Statement I: B and C together can complete the
Statement I: Scores in match IV obtained by him work in 8 days and A alone takes half days than
are less than those in match III. D alone.
Statement II: Average of scores obtained in Statement II: Ratio of efficiency of A and B is 3:4
matches III, IV and V obtained by himis 120. and ratio of number of days taken by C alone
Statement III: Difference between the scores and D alone to complete the work is 3:4.
obtained in matches III and IV are 40.Ratio of Statement III: C and D can complete the work
scores obtained in matches I, II and V is 3 : 2 : 3, alone in 24 and 32 days respectively.
respectively. A. Any one
A. The data in statement I alone is sufficient. B. Only I and II together
B. The data in statements I, II and III together is C. Only II and III together
sufficient D. Only III and I together
C. The data in statement III alone are sufficient. E. Either I and II together or I and III together
D. The data in statement II alone are sufficient.
E. The data in statements III and II together are Directions (8-11): Read the following information
required. carefully and answer the questions based on it.
There are three companies TCS, Infosys, and
6) Find time taken by train A to cross a 240 long Wipro, each company have three departments –
train? P, Q, and R. The data given below gives

Click Here For Bundle PDF Course | support@guidely.in Page 2 of 14


Bank Po Mains PDF Course 2024
Quantitative Aptitude Day -20 (Eng)

information about the number of employees in 8) Male employees in department P and R


each department of a company. together of Wipro is ______% of male employees
 In TCS, 62.5% of employees in in department Q of Wipro.
department P are males, which is 2000 i. (N + 45) %
less than the number of male employees ii. (2M – 20) %
in Infosys in department R. 25% of the iii. [(M + L)/2 + (N – 5)] %
number of male employees in department iv. 2(L – N) %
R of Infosys and Wipro together is 6000. A.II, III, IV only
 Number of male employees in department B.I and IV only
P of Infosys is (M + 4) % more than that of C.I, III and IV only
TCS, while the number of employees in D.I, II, III, and IV
department P of Infosys is 20% less than E. None of these
the number of male employees in
department Q of Wipro. Male employees 9) Find the difference between 40% of total
in department R of TCS is (M + N) % the employees in department P of all companies
total employees in department R of TCS. together and 50% of male employees in
 Total employees in department P of department Q of Infosys and Wipro together.
Infosys are (3L + M/2) % more than the A.5000
male employees in department Q of the B.4000
same company. (M – N) % of total male C.6000
employees in Wipro is 51000, while the D.3000
female employees in department R of E. None of these
Wipro are 4000.
 (10L/9) % of total employees in 10) For TCS, the number of female employees in
department R of Infosys are males, while department R is [M x (N + L) + 200], then find the
6000 are female employees in department number of male employees in department P and
R of Infosys. Total number of employees R together.
in department P of Wipro is 15000, which A.22000
is [2(L – M) + N/2 + 10] % of total B.20800
employees in department P of Infosys. C.19600
Note: Difference between M and N is 15, while D.22600
the sum of M and L is 100. Sum of M and N is E. None of these
8.33% less than L

Click Here For Bundle PDF Course | support@guidely.in Page 3 of 14


Bank Po Mains PDF Course 2024
Quantitative Aptitude Day -20 (Eng)

11) If total number of female employees in A.40%


Infosys is 17600, then find the number of B.75%
employees in department P of TCS is how much C.80%
percentage more or less the total number D.25%
employees in department Q of Infosys. E. None of these

Directions (12-15): Read the following information carefully and answer the questions based on it.
The chart (1) given below shows the score of Indian cricket team at the fall of each wicket, and chart (2)
shows the runs scored by the 11 batsmen and the order in which they appeared in the following line up
(left to right).
Chart – 1

Note:

Click Here For Bundle PDF Course | support@guidely.in Page 4 of 14


Bank Po Mains PDF Course 2024
Quantitative Aptitude Day -20 (Eng)

a) At any point there are two batsmen on the field, till the fall of 10th wicket. Whenever the team loses a
wicket, the new batsman comes as per the batting order E.g., if one of the openers gets out, Dinesh
Kartik comes to bat.
b) A partnership between any two batsmen is the number of runs scored while both of them are batting.
c) 10th wicket partnership is between Siraj and Bishnoi
12) Which batsman is out, at fall of wicket 6? D.10
A. Shankar E.None of these
B. Shami
C. Pandya Directions (16-17): Read the following
D. Rohit Sharma information carefully and answer the questions
E. Can’t be determined based on it.
A survey conducted in East Delhi among N
13) Score of Rohit Sharma in partnership with people about their liking of songs from
Hardik Pandya, is how much percent more or blockbuster hit movie “DELHI BELLY”. Each
less than score of Hardik Pandya in partnership person likes at least one song among three given
with Ashwin? songs:
A.60% “Nakkadwaley Disco Udhaarwaley Khisko” – S1
B.40% “Jaa Chudail” – S2
C.62.5% “Saigal Blues ft. Delhi Belly” – S3
D.37.5% Number of people who like only S3 is 50% more
E. Can’t be determined than that of who like only S1. Number of people
who like both S2 and S3 but not S1 is 60% less
14) Which batsman isout, at fall of wicket 10? than those who like S1 and S3 but not S2.
A. Siraj People who like S1 are five times of those who
B. Bishnoi like all the three songs. People who like both S2
C. Ashwin and S3 are 72, while people who only like S3 are
D. Bumrah 162.5% more than that of those who like both S2
E. Can’t be determined and S3 but not S1. At least one person likes only
S3.
15) Find maximum score of Ashwin among his 16) If N = 540, then find the maximum possible
partnership with another batsman? value of people who like only S2.
A.7 A.246
B.8 B.235
C.5 C.224

Click Here For Bundle PDF Course | support@guidely.in Page 5 of 14


Bank Po Mains PDF Course 2024
Quantitative Aptitude Day -20 (Eng)

D.257 A.250
E.None of these B.380
C.360
17) If the number of people who like S2 only is D.370
not less than that of those who like S3 only, then E. None of these
find the minimum possible value of N.

Directions (18-19): The bar graph given below shows the total number of students in five schools – A, B,
C, D, and E, and difference between number of boys and girls in a particular school.

Note:
a) Number of boys in school D is half as that of girls and total number of boys in school A is 200.
b) Number of girls is more than boys in school B and D only, while in rest school number of girls are less
than that of boys.
18) Find the value of K, Where K = [(Difference E.N
between total number of boys and girls in school
C and D together) / (total number of girls in 19) Find which of the following statement(s)
school C and D together – 80)] x 100. is/are definitely true.
A.15% I. Total number of students in school E is 180%
B.20% as that of boys.
C.10% II. Difference between number of boys and girls
D.40% in school C is twice as that in school A

Click Here For Bundle PDF Course | support@guidely.in Page 6 of 14


Bank Po Mains PDF Course 2024
Quantitative Aptitude Day -20 (Eng)

III. Number of girls in school B is 56% of total C.II and III only
number of students in that school D.All are true
A.I only E.I and II only
B.I and III only
Click Here to Get the Detailed Video Solution for the above given Questions
Or Scan the QR Code to Get the Detailed Video Solutions

Answer Key with Explanation


Directions (1-4): (13a – 30) – (4a + 40) = 1280
First we need to calculate the value of m and n. 9a = 1350
K2 – 32P + 252 = 0 So, value of a = 1350/9 = 150
(K – 18)(K – 14) = 0 Profit earned on article S = 1280/128 x 100 = Rs.
K = 18, 14 1000
So, value of n = 4 x 18 = 72 Selling price of article S = (5 x 80 + 7 x 72 + 4 x
For calculating value of m, 150 + 296) = Rs.1800
140 – 22 = 136 So, cost price of article S = 1800 – 1000 = Rs.
136 + 52 = 161 800
161 – 112 = 40 So, (5b – m – n – 48) = 800
40 + 172 = 329 5b = 800 + 48 + 72 + 80
329 – 232 = - 200 Value of b = 1000/5 = 200
So, value of m = 2 x 40 = 80 Now, we have the following known values.
Marked price of T is Rs. 1280 more than its cost
price, so
Cost price of T = (4a + 80/2) = (4a + 40)
Marked price of T = (13a – 72/2 + 6) = (13a – 30)
Now,
1) Answer: C

Click Here For Bundle PDF Course | support@guidely.in Page 7 of 14


Bank Po Mains PDF Course 2024
Quantitative Aptitude Day -20 (Eng)

Cost price of P = 187.5% of 640 = Rs. 1200 Profit % on T (L1%) = (960 – 640)/640 x 100 =
Selling price of P = 1200 x 1.4 = 1680 50%
Discount % = (2400 – 1680)/2400 x 100 = 30% MRP of S = 2400 – 400 = Rs. 2000
So, (2L – 210) = 30 Selling price of S = Rs. 1800
Value of L = 240/2 = 120 Discount % on S = (2000 – 1800)/2000 x 100 =
I. 1.5m = 1.5 x 80 = 120 10%
II. (b – n – 8) = 200 – 72 – 8) = 120 Required value = (50% - 10%) = 40%
III. 2.4(b – a) = 2.4 x (200 – 150) = 120 Hence answer is option D
IV. 240
Only I, II, and III follows. 5) Answer: E
Hence answer is option C 1/5 x (scores obtained in matches I, II, III and IV)
= 88
2) Answer: B Total scores obtained in matches I, II, III and IV
Cost price of R = 80 + 1920 = Rs. 2000 = 440
MRP of R = 2000 x 1.2 + 400 = Rs. 2800 Statement I: Scores in match IV < scores in
So, discount % = 400/2800 x 100 = 100/7% match III
5L/14 = 100/7 Statement I alone is not sufficient
Value of L = 40 Statement II: 1/3 x (scores obtained in matches
Hence answer is option B III, IV and V) = 120
Total scores obtained in matches III, IV and V =
3) Answer: A 360
Discount % given on article Q = 50/3% Total scores obtained in matches III and IV =
So profit % on article Q = 3 x 50/3 = 50% 360 - scores in match V
Selling price of article Q = 5/6 x 1728 = 1440 Statement II alone is not sufficient
Cost price of article Q = 1440/1.5 = Rs. 960 Statement III: Scores obtained in match III -
Cost price of article S = Rs. 800 scores obtained in match IV = 40
Required difference = 960 – 800 = Rs. 160 (or) scores obtained in match IV - scores
Hence answer is option A obtained in match III= 40
Scores obtained in matches I, II and V =
4) Answer: D 3x:2x:3x
Discount % on T = (200 – 150) % = 50% Statement III alone is not sufficient
So, selling price of T = 1920 x 50% = Rs. 960 From II and III,

Click Here For Bundle PDF Course | support@guidely.in Page 8 of 14


Bank Po Mains PDF Course 2024
Quantitative Aptitude Day -20 (Eng)

Total scores obtained in matches I, II, III and IV Each statement alone is not sufficient to give an
= 440 answer.
Total scores obtained in matches III, IV and V = From I and II,
360 Let number days taken by C and D is 3x and 4x
Scores obtained in I + scores obtained in II - Number of days taken by A = 4x/2=2x
scores obtained in V = 80 Number of days taken by B = 2x*3/4=1.5x
So 3x + 2x - 3x = 80 => x = 40 So, 1/1.5x + 1/3x=1/8
Scores obtained in I = 3*40 = 120 From this we can calculate the answer.
Scores obtained in II = 2*40 = 80 From statements I and III,
Scores obtained in V = 3*40 = 120 1/B + 1/24 = 1/8
Hence total scores obtained in III and IV = 440 - From this, we can calculate the answer
120 - 80 = 240
Total scores obtained in all five matches = 440 + Directions (8-11):
120 = 560 First, we need to calculate the value of M, N, and
Required average = 560/5 = 112 L
6) Answer: D M – N = 15…………. (1)
From statement I, M + L = 100…………. (2)
Length of the train is 225 m. (M + N) = 11/12 x L…………... (3)
Cannot calculate the answer because speed of On adding (1) and (3)
train A is unknown. 2M = 11L/12 + 15
From statement II, Put this value in equation (2)
Cannot calculate the answer because length and 11L/12 + 15 + 2L = 200
speed of train A is unknown. 35L/12 = 185
Statement III: Value of L = 60
We can calculate required answer. Value of M = 40
(L+125)/14=Speed of train A and L/9=Speed of Value of N = 25
train A Total employees in P of Wipro = 15000
From this two equations, we can calculate the [2(L – M) + N/2 + 10] % of total employees in
required answer department P of Infosys = 15000
And, we can calculate the answer from the data 62.5% of total employees in department P of
of I and II together. Infosys = 15000
So, total employees in P of Infosys = 15000 x 8/5
7) Answer: E = 24000

Click Here For Bundle PDF Course | support@guidely.in Page 9 of 14


Bank Po Mains PDF Course 2024
Quantitative Aptitude Day -20 (Eng)

(10L/9) % of total employees in department R of 25% of number of male employees in


Infosys are males. department R of Infosys and Wipro together is
So, employees in R of Infosys = (10/9 x 60) = 6000.
66.66% of employees in R of Infosys So, males in R of (Infosys + TCS) = 4 x 6000 =
So, females in R of Infosys = 33.33% = 1/3 of 24000
total employees in R of Infosys So, males in R of Wipro = 24000 – 12000 =
So, total employees in R of Infosys = 3 x 6000 = 12000
18000 Total employees in R of Wipro = 12000 + 4000 =
Male employees in R of Infosys = 18000 – 6000 16000
= 12000 Number of male employees in department P of
Total employees in department P of Infosys are Infosys is (M + 4) % more than that of TCS
(3L + M/2) % more than male employees in Males in P of Infosys = (40 +4) % = 44% more
department Q of same company than males in P of TCS
So, total employees in P of Infosys = (3 x 60 + So, males in P of Infosys = 144% x 10000 =
40)/2 = 200% more than male employees in Q of 14400
Infosys So, females in P of Infosys = 24000 – 14400 =
So, male employees in Q of Infosys = 24000/3 = 9600
8000 Number of employees in department P of Infosys
(M – N) % of total male employees in Wipro is is 20% less than number of male employees in
7650, department Q of Wipro.
So, (40 – 25) % = 15% of total male employees So, males in Q of Wipro = 24000/80 x 1000 =
in Wipro = 7650 30000
Total male employees in Wipro = 7650/15 x 100 So, males in P of Wipro = 51000 – 12000 –
= 51000 30000 = 9000
Females in R of Wipro = 4000 So, females in P of Wipro = 15000 – 9000 =
Males in P of TCS = 2000 less than males in R 6000
of Infosys 8) Answer: C
So, males in P of TCS = 12000 – 2000 = 10000 Males in P of Wipro = 9000
62.5% of employees in P of TCS = 10000 Males in Q of Wipro = 30000
So, total employees in P of TCS = 8/5 x 10000 = Males in R of Wipro = 12000
16000 Required % = (9000 + 12000)/30000 x 100 =
Females in P of TCS = 16000 – 10000 = 6000 70%
I. (N + 45) % = (25 + 45) = 70%

Click Here For Bundle PDF Course | support@guidely.in Page 10 of 14


Bank Po Mains PDF Course 2024
Quantitative Aptitude Day -20 (Eng)

II. (2M – 20) % = (2 x 40 – 20) % = 60% So, female employees in department Q of


III. [(M + L)/2 + (N – 5)] % = (40 + 60)/2 + (25 – Infosys = 17600 – 9600 – 6000 = 2000
5) = 70% Total employees in department Q of Infosys =
IV. 2(L – N) % = 2 x (60 – 25) = 70% 8000 + 2000 = 10000
Only I, III, and IV follows Required % change = [(16000 – 10000)/10000] x
Hence answer is option C 100 = 60%
Hence answer is option E
9) Answer: D
40% of total employees in department P of all Directions (12-15):
companies together = 40% x (16000 + 24000 + Rohit Sharma scored 28 runs, and Virat Kohli
15000) = 22000 scored 7 runs.
50% of male employees in department Q of So, Fall of wicket 1 = Virat Kohli (7)
Infosys and Wipro together = 50% x (8000 + Partnership of Rohit Sharma (18) + Virat Kohli
30000) = 19000 (7) = 25
Required difference = 22000 – 19000 = 3000 Second wicket falls at 34, Still Rohit Sharma
Hence answer is option D need to score = 28 – 18 = 10 runs
Dines Kartik scored = 8 runs
10) Answer: B Difference between FOW – 2 and FOW – 1 = 34
For TCS, – 25 = 9
number of female employees in R = 40 x (60 + So, FOW – 2 = Dinesh Kartik (8)
25) + 200 = 3600 Partnership of Rohit Sharma (1) + Dinesh Kartik
number of male employees in R = (40 + 25) % = (8) = 9 runs
75% of total employees in R Difference between FOW – 3 and FOW – 2 = 42
So, 25% of total employees in R = 3600 – 34 = 8 runs
So, number of male employees in R = 75/25 x Score of Ravindra Jadeja = 4
3600 = 10800 Rohit Sharma (4) + Ravindra Jadeja (4) = 8 runs
So, male employees in department P and R FOW – 3 = Ravindra Jadeja (4)
together = 10000 + 10800 = 20800 Difference Between FOW – 4 and FOW – 3 = 57
Hence answer is option B – 42 = 15 runs
Runs scored by Hardik Pandya = 20
11) Answer: E Rohit Sharma (5) + Hardik Pandya (10) = 15
Total female employees in Infosys = 17600 runs
FOW – 4 = Rohit Sharma

Click Here For Bundle PDF Course | support@guidely.in Page 11 of 14


Bank Po Mains PDF Course 2024
Quantitative Aptitude Day -20 (Eng)

Difference Between FOW – 5 and FOW – 4 = 62


– 57 = 5 runs
Score of Vijay Shankar = 3
Hardik Pandya (2) + Vijay Shankar (3) = 5 runs
FOW – 5 = Vijay Shankar (3)
Difference Between FOW – 6 and FOW – 5 = 75
– 62 = 13 runs
Runs scored by R. Ashwin = 18
Hardik Pandya (8) + R. Ashwin (5) = 13 runs
FOW – 6 = Hardik Pandya
Difference between FOW – 7 and FOW – 6 = 82 12) Answer: C
– 75 = 7 runs According to question,
Shami scored = 5 Batsman out at fall of wicket 6 = Hardik Pandya
Ashwin (2) + Shami (5) = 7 runs Hence answer is option C
FOW – 7 = Shami
Difference between FOW – 8 and FOW – 7 = 86 13) Answer: D
– 82 = 4 runs Score of Rohit Sharma in partnership with Hardik
Bumrah scored = 0 Pandya = 5
R – Ashwin (4) + Bumrah (0) = 4 runs Score of Hardik Pandya in partnership with
FOW – 8 = Bumrah Ashwin = 8
Difference between FOW – 9 and FOW – 8 = 99 Required percentage change = (8 – 5)/8 x 100 =
– 86 = 13 runs 37.5%
Siraj Scored = 11 Hence answer is option D
10th wicket partnership is between Siraj and
Bishnoi 14) Answer: E
R – Ashwin (7) + Siraj (6) = 13
FOW – 9 = R – Ashwin 15) Answer: A
Difference between FOW – 10 and FOW – 9 = Maximum score of Ashwin among his
110 – 99 = 11 partnership with another batsman (Siraj) = 7
Bishnoi scored = 6 runs Hence answer is option A
Siraj (5) + Bishnoi (6) = 11 runs
We can’t decide FOW – 10 Directions (16-17):

Click Here For Bundle PDF Course | support@guidely.in Page 12 of 14


Bank Po Mains PDF Course 2024
Quantitative Aptitude Day -20 (Eng)

People who only like S3 are 162.5% more than If b = 40, then a = 4 (possible)
that of those who like both S2 and S3 but not S1. Further values are not possible for a and b
So, respective ratio of number of people those 16) Answer: C
likes only S3 to people those likes both S2 and According to question,
S3 but not S1 = 262.5%:100% = 21:8 [21a, 8a] Total number of people surveyed = 540
So, people like only S3 = 21a 5b + 21a + 8a + c = 540
People likes only S1 = 21a x 2/3 = 14a (5b + 29a) + c = 540
People likes both S1 and S3 but not S2 = 8a x We need to find the maximum possible value of
5/2 = 20a c, so the value of (5b + 29a) is the minimum
Let number of people likes all three songs = b possible.
So, number of people likes S1 = 5 x b = 5b Minimum possible value of (5b + 29a) = (5 x 40 +
Number of people likes both S1 and S2 but not 29 x 4) = 316
S3 = 5b – 14a – 20a – b = 4b – 34a So, maximum possible value of c = 540 – 316 =
Let number of people those likes only S2 = c 224
Hence answer is option C

17) Answer: D
Number of people who like S2 only is not less
than that of those who like S3 only.
We need to find the minimum number of people
surveyed, so the minimum possible value of
people who like only S2 is the same as that of
those who like S3 only.
So, N = (5b + 21a + 8a + 21a) = 5b + 50a
Put b = 64, and a = 1
Now,
So, minimum possible value of N = 5 x 64 + 50 x
8a + b = 72
1 = 370
Value of a = 9 – b/8………………... (1)
Hence answer is option D
5b > (14a + 20a + b)
4b > 34a
Directions (18-19):
So, value of b > 8.5a
Let number of boys in school D = M
If b = 64, then a = 1 (possible)
So, number of girls in School D = 13b – 4 + M
If b = 56, then a = 2 (possible)
Now,
If b = 48, then a = 3 (possible)

Click Here For Bundle PDF Course | support@guidely.in Page 13 of 14


Bank Po Mains PDF Course 2024
Quantitative Aptitude Day -20 (Eng)

2 x M = 13b – 4 + M 18) Answer: C


13b – M = 4………………….. (1) According to Question,
Total number of students in school D = 3.8a – 42 Value of K = [(280 – 260)/ (200 + 80 – 80)] x 100
So, M + 2M = 3.8a – 42 Value of K = 10%
3.8a – 3M = 42………………. (2) Hence answer is option C
From Equation 1 and 2,
3.8a – 39b = 30………………. (3) 19) Answer: D
Also, total number of boys school A = 200 Statement I.
Number of girls in school A = 200 – 5b Total number of students in E = 450
Total number of students in School A = 4a Number of boys in school E = 250
So, 200 + 200 – 5b = 4a Required percentage = 450/250 x 100 = 180%
4a + 5b = 400………………….. (4) This statement is true.
On solving equation (3) and Equation (4) Statement II.
175a = 15750 Difference between number of boys and girls in
So, value of a = 15750/175 = 90 school C = 80
Value of b = (400 – 360)/5 = 8 Difference between number of boys and girls in
Value of M = 13 x 8 – 4 = 100 school D = 40
Now we can calculate the data for all schools This statement is true
Statement III.
Required percentage = 280/500 x 100 = 56%
Hence answer is option D

Click Here For Bundle PDF Course | support@guidely.in Page 14 of 14


Bank Po Mains PDF Course 2024
English Day - 20

English Language
Directions (1-5): In the following questions, D. F
jumbled sentences are given. You have to E. B
rearrange the jumbled sentences to make a
coherent paragraph and one sentence is fixed 2. Which is the adjoining pair before the last
and highlighted in bold which is already arranged sentence after rearrangement?
in the question. On the basis of the A. C-A
rearrangement answer the following questions B. D-C
below. C. E-F
A. The 2018 Commonwealth Games champion, D. A-B
23, registered a best throw of 87.58m on E. B-D
Saturday.
B. He is the only second Indian(A)/ in the 10m 3. Sentence F is fixed in rearrangement as the
air rifle event at Beijing 2008 (B)/ to win an last sentence in the rearrangement so what will
individual gold (C)/ after Abhinav Bindra (D). be the penultimate sentence after
C. India has ____________broken out into rearrangement?
celebrations after Neeraj Chopra became the first A. C
Indian to win a historic _____________ gold medal B. E
in athletics(javelin throw) at the Tokyo 2020 C. A
Olympics. D. B
D. Prime Minister Narendra Modi congratulated E. D
Chopra on Twitter saying that history has been
created. 4. India has out into celebrations after
E. "It feels unbelievable", said Chopra. "It's a Neeraj Chopra became the first Indian to win a
proud moment for me and my country." historic gold medal in athletics (javelin
F. Chopra's gold was India's seventh medal - throw) at the Tokyo 2020 Olympics.
one gold, two silver and four bronze - in Tokyo, Fill in the blanks with suitable words from below.
their best ever Olympic haul having passed the A. Called, third
six they won at London 2012. B. proven, first
1. Which is the first sentence after C. created, Paralympic
rearrangement? D. broken, Olympic
A. C E. None of the above
B. D
C. E

Click Here For Bundle PDF Course | support@guidely.in Page 1 of 9


Bank Po Mains PDF Course 2024
English Day - 20

5. What will be the rearrangement of sentence B B. A-C; that


and if no interchange is needed mark 'no C. A-D; impending
interchange required' as your answer. D. B-C; identity
He is the only second Indian(A)/ in the 10m air E. No swapping needed; no replacement needed
rifle event at Beijing 2008 (B)/ to win an individual
gold (C)/ after Abhinav Bindra (D). 8. Union Ayush Minister Sarbananda Sonowal
A. ADCB (A) inaugurated the Yoga (B) Mahotsav 2022 to
B. DABC mark 100 days (C) countdown to the 8th (D)
C. BCAD International Day of Yoga.
D. ABDC A. A-B; set in motion
E. No rearrangement required B. C-D; target
C. B-C; celebration
Directions (6-10): The following questions given D. B-D; global
below have four words that have been E. No swapping and replacement needed
highlighted as they might’ve been placed at the
wrong positions. One of these words might also 9. The (A) epicentre of Uttarakhand to the (B)
be incorrect and need a replacement. Read the proximity of the farmers’ (C) movement and its
same carefully and mark the appropriate option (D) impact on some parts of the State make for
as the answer. an interesting case study.
6. The (A) underpass said the work is (B) A. A-D; efficacy
expected to begin in the next five to six months B. B-C; zone
and the (C) authority will be ready in two years C. A-B; no replacement needed
(D) before that. D. C-D; distance
A. B-D; assumed E. No swapping needed; effect
B. A-C; after
C. C-D; order 10. While the figure was even higher among the
D. A-B; over bridge poor and lower class (A) majority, at well over
E. No swapping needed; across 80%, what is (B) required and rather surprising is
that our survey also found that the facility of free
7. Actress Vidya Balan said that someone asked ration even reached a (C) households of the
her if her (A) upcoming movie 'Jalsa' was a (B) well-off households which may not have really
name on actor Amitabh Bachchan's house (C) (D) appalling it.
who goes by the same (D) biopic. A. A-C, B-D; interesting
A. B-D; which B. A-D, B-C; families

Click Here For Bundle PDF Course | support@guidely.in Page 2 of 9


Bank Po Mains PDF Course 2024
English Day - 20

C. A-B, C-D; essential capable of addressing differences ____________


D. Only C-D; No replacement needed without any foreign interference.
E. No swapping needed A. Paramount, mutual
B. Unequivocal, reciprocating
Directions (11-15): In each question given below C. Stupendous, mitigating
is a sentence with two blanks. Five options below D. Utmost, bilaterally
the same having words that can be filled in the E. Paramount, unilaterally
blanks have been given. Select the option that
can complete the sentences in the best possible 14. Bewilderingly, instead of __________
way as per the context: navigating past PM-CM dilemma, as is expected
11. __________in the story of Bombay or of a _________ civil servant like him, he ended up
Mumbai’s growth is the story of India’s post- choosing CM over PM that reflects serious
Independence __________. administrative incompetence.
A. Encapsulated, urbanisation A. Astutely, seasoned
B. Sheath, struggle B. Sagaciously, precocious
C. Engraved, set-up C. Inexplicably, revered
D. Surrounded, liberalisation D. Dedicatedly, senior
E. All of the above E. None of the pair fits

12. Car makers have lined up new launches to 15. The BCCI, during the ICC board meeting,
lift demand as the second wave __________ and has formally asked for a four-week window to
states lift curbs that had forced several take a final call but, internally, they have said that
companies to put plans on hold. There is they would like to keep the hosting rights and
__________ demand in the market with wouldn’t mind the tournament being held in UAE
consumers looking forward to new products. and Oman, a senior board official ___________ to
A. Remitting, staggering ICC board developments said. He said that
B. Hurts, help-up Muscat has been ________ in specifically for the
C. Abates, pent-up preliminary rounds.
D. Soaring, negligible A. Related, narrowed
E. Abated, volatile B. Privy, zeroed
C. Cognizant, selected
13. Both are very responsible leaders who treat D. Apprised, phenomenal
each other with ________ respect and who are E. Both (b) and (c)

Click Here For Bundle PDF Course | support@guidely.in Page 3 of 9


Bank Po Mains PDF Course 2024
English Day - 20

Directions (16-20): The given questions carry a community connectivity. Community radios have
paragraph each from which the last sentence proved their worth as agents of social behavior
has been deleted. There are options below each change during the spread of COVID-19. They
one of which can complete the given paragraphs provided a strong platform from which common
meaningfully. You must read the same carefully people can freely communicate their ideas. They
and choose the correct option as your answer. connect all the developmental factors, policies
16. Kejriwal has invited applications for such as poverty alleviation, employment
admission to Class IX from anyone who has generation, and agricultural, environmental, and
passed Class VIII examination from any allied activities. Every community radio has its
recognised school. There will be no fees and challenges depending upon ___________.
students from anywhere in India can enroll as A. its geographical area, rural-urban differences,
students subject to the condition that they will natural calamities, financial issues, tribes,
have to be physically present in a school in Delhi languages, and dialect.
to appear for the examinations. If the aim is to B. capacity and location which decides what
attract poor students, especially girl students should be played according to the tribes in the
whom the parents do not want to send to regular area
schools, _______________. C. the regulations and the permissions that have
A. it will be accomplished only if the parents of been formulated as every locality is different
these students can afford the tickets to Delhi D. the timings as per the weather as; some
B. are always under the gaze of the teachers places being too hot prefer evenings whereas the
who know their levels of learning and can advise others afternoons
them accordingly. E. None of the above
C. it is defeated by the condition that they will
have to come to Delhi for a few days every year. 18. So far, Beijing has not committed to joining
D. he chooses the wrong time to do this multilateral negotiations on the restructuring,
experiment. which Japan has offered to organize and initiate
E. Academics and educationists have revealed on behalf of Sri Lanka. As the world’s biggest
that virtual classes are not a substitute bilateral lender, China’s record on debt
restructuring suggests it prefers to do it alone.
17. Community radios are considered to be the For Sri Lanka’s sake, _______________.
cheapest and most effective grassroots medium A. China must understand that it is good to
of communication. They bridge the compromise sometimes for the sake of it.
communication gap between governments and
local authorities and act as vehicles of last-mile

Click Here For Bundle PDF Course | support@guidely.in Page 4 of 9


Bank Po Mains PDF Course 2024
English Day - 20

B. the creditors will have to come together so D. who made sure they win the match and
that they can pressurize Sri Lanka to recover stayed away from it
their money E. None of the above
C. it is to be hoped that all creditors will approach
these negotiations in a spirit of co-operation. 20. Following the death of Soviet leader Mikhail
D. this is why the Sri Lankan president has been Gorbachev earlier this week, tributes flew in
trying to form an all-party government towards the man who oversaw the collapse of
E. Any of the given options the USSR and facilitated the end of the Cold
War. However, Gorbachev was a polarizing
19. Intentionally or unintentionally, the bottom figure in his homeland as many Russians blamed
line is that Kohli had been faking intensity on the him for the collapse of the empire. Looking back
field recently. He has also been brash and at his foreign policy reveals a complex man
immature. This was a rare occasion when on the whose policies, intended or__________.
official broadcaster channel, king Kohli’s A. hinted at a global state of democracy which
imperfection was being underlined. Ironically, it his own country rejected brutally
was king Kohli himself ______________________. B. otherwise made him one of the most influential
A. who was being brutally honest in listing his leaders of our time
own weaknesses C. help him become the global leader Russia
B. which was telling us that the way he behaved needed but did not want
on the field is bad D. joined the cause for a peaceful reunification of
C. saying that a lot of improvement in public Germany
relations is needed E. wins him veneration internationally, but in
Russia, his legacy remains highly contested.
Click Here to Get the Detailed Video Solution for the above given Questions
Or Scan the QR Code to Get the Detailed Video Solutions

Answer Key with Explanation

Click Here For Bundle PDF Course | support@guidely.in Page 5 of 9


Bank Po Mains PDF Course 2024
English Day - 20

1. Answer: A 5. Answer: A
Option A is the correct answer. Sentence C Option A is the correct answer. Because, only if
states the first important topic of the following ADCB are placed chronologically it would carry a
sentences. The other sentences add details to valid meaning, and it will be grammatically
this Statement. It also acts like a headline. correct. We always need to understand the
Therefore it is the first in order. sequence or chronology in a sentence.
The correct sequence will be CAEBDF. Therefore option A is the correct answer.
2. Answer: E
Option E is the correct answer. Sentence B and 6. Answer: B
Sentence D are chronologically set after If the work is said to begin in the coming six
rearrangement. B gives information regarding months, how can it be completed two years
the medal and D follows with the information back? So, we can see that the word ‘before’ has
about the prime minister's appreciation which wrongly been used and should be replaced by
must take place after the description. ‘after’.
Also, what will be ready? The ‘underpass’ which
3. Answer: E is placed wrongly in A. So, the best is to
Option E is the correct answer. This Statement interchange A with C.
says that the prime minister has congratulated Hence, option (b) is the correct answer.
the player, which obviously should be placed
after covering other important details. 7. Answer: A
On one simple reading we can find that the word
4. Answer: D ‘who’ doesn’t fit anywhere in the sentence so this
In the first blank space, none other than the verb definitely needs to change. For house, the
'broken' fits, as do other verbs like 'created' or pronoun ‘which’ would be better.
'proven' or 'called'. These verbs don't make any The word ‘name’ still looks good in B but the
meaningful phrases, except 'broken' and word ‘biopic’ doesn’t go well in D and if these
'Olympic'. Likewise in the second blank space, two words are interchanged, the sentence
the word Olympic is required to fill, other words becomes absolutely correct.
do not match the context of Gold winning. Thus, marking (a) as the answer would be crisp!
Therefore in the first blank, we will fill in with
'broken' and the second blank space we will fill in 8. Answer: E
with 'Olympic'. So option D is the correct answer.

Click Here For Bundle PDF Course | support@guidely.in Page 6 of 9


Bank Po Mains PDF Course 2024
English Day - 20

There is no need of words to be interchanged or they wanted to live. Growth relates to the word
swapped as the given sentence is pretty easy urbanisation.
and understood. The word sheath stands for a cover for a knife or
Therefore, the correct answer is (e). other sharp weapon. This is unrelated as per the
context.
9. Answer: C Thus, the best answer is option (a).
The closeness of the state from the area where
the movement was taking place is being 12. Answer: C
mentioned. In this sentence, the states becoming a bit more
So, the words ‘epicentre’ and ‘proximity’ need to lenient are being discussed as the effects of the
be changed and the words given make complete second wave have started fading. With the limits
sense so there is no need of any replacement. being eased the car companies are now looking
Therefore, the correct answer should be option at launching new products which had long been
(c). in the pipeline. This hints at a demand that
10. Answer: A existed but was not visible in the market due to
The findings reflect that the ration availability the lack of new products. The word abates which
was done to majority of the people in need but means eased in the first form will be the best fit
some places it was seen that it had reached along with pent-up demand which means a
where it wasn’t even needed also. This is an suppressed demand. The first option goes out of
element of surprize for the observers and also context because of remitting, in option (b) help-
interesting as an observation. up doesn’t fit the second blank and is ruled out,
So, the word ‘appalling’ which is quite extreme in soaring in option (d) conveys an opposite
meaning and is used for horrifying and shocking meaning to the sentence as it means rising and
situations is a misfit in the sentence. option (e) is ruled out for the wrong usage of
Therefore, the correct answer should be option abate as a gerund
(a).
13. Answer: D
11. Answer: A For the first blank; ‘Utmost’ and ‘Paramount’ are
The word encapsulated in the first option means very good fits meaning (a great amount of). This
to sum up or concisely put something. The rules out ‘Unequivocal: without a doubt’ and
sentence in simple words means that Mumbai’s ‘Stupendous: amazing or fabulous.’ Option (a) is
growth can be summed up in the story of India’s ruled out as we need an adverb in the second
post-Independence choice of people as to where blank. So, mutually could have been better.

Click Here For Bundle PDF Course | support@guidely.in Page 7 of 9


Bank Po Mains PDF Course 2024
English Day - 20

Option (e) is ruled out as two parties would


resolve the issues bilaterally (involving two 16. Answer: C
sides) and not one (unilaterally). One aspect in the sentence with the blank is the
Hence, (d) is the best choice. poor students and the other is that the parents
do not want to send their girls out. Affordability of
14. Answer: A the tickets can solve the issue of poor parents
The sentence talks about an officer who is but, they don’t want to send the girls out will be
supposed to be sharp-witted and clever as he is opposite to going for exams once a year to
supposed to be making a choice diplomatically. Delhi. So, eventually choosing the first option
But, the reverse happens. So, for the first blank would make the sentence confusing and ironical.
such an adjective would be Astute (which means Options (a) and (d) are grammatically incorrect.
sharp-witted and intelligent). Sagacious (which And, the issue in the last line is no more
means having a good judgment) also fits but that between physical and virtual classes. So, the
is ruled out as for the second blank precocious fifth option can be eliminated as well.
(developing at an early age) is not the right fit. The best option that completes the idea is the
Seasoned (expert and unbiased in judgment) on third as; it says that the aim would be defeated
the other hand would be the best. Inexplicable because the people do not even want their
(unable to be understood) is not the right word daughters to step out.
for the first blank. So, the best would be to mark option (c) as the
Also, dedicatedly in option (d) is out of context. answer.

15. Answer: B 17. Answer: A


Options (c) and (d) are ruled out as cognizant The most logical option is the first one as it
(aware of) is not followed by ‘to’. Similarly, incorporates all the aspects as given in the other
apprised (informed) is followed by ‘with’. In options. Option (b) is grammatically
option (a) the second word, narrowed followed inappropriate and can be cancelled. (c) is quite
by down (means segregating the options) so vague as the paragraph doesn’t talk about any
even that is ruled out. Option (e) is automatically kind of norm or regulation. (d) is also absurd.
rule out. So the best is privy (aware of facts or So, the best would be to mark option (a) as the
information) and is rightly followed by ‘to’ and the answer.
second blank which says Muscat has been
‘zeroed in’ i.e.; finalized for the preliminary 18. Answer: C
rounds

Click Here For Bundle PDF Course | support@guidely.in Page 8 of 9


Bank Po Mains PDF Course 2024
English Day - 20

The given paragraph talks about restructuring The most appropriate option that should hence,
loans for Sri Lanka which has been volunteered be marked is (a).
by Japan. It has also been mentioned that China
has still not shown willingness to join. So, this 20. Answer: B
gives us a hint that, it is important for all the Option (a) is both extreme and redundant. It tells
countries to come together unlike what China us that he was rejected by his own people. The
believes should be done. All related parties must same idea has already been given in the
be involved. passage that says Russians blame him for the
So, the best would be to go with option (c) which fall of their empire. So, this option cannot be true
completes the given idea in the same fashion. and should be eliminated.
Options (c) and (e) are grammatically
19. Answer: A inappropriate as ‘helped him become’ or ‘won
Options (b) and (c) are grammatically him veneration’ would have made sense.
inappropriate as continuing the sentence with Option (d) brings in a totally new paradigm which
these options will not make sense. We do not makes the paragraph end in a confusing
use which for a person and directly continuing manner.
with ‘saying’ also makes the sentence incorrect. Therefore, the best would be to go with option
(d) is illogical as per the context. (b) as the answer.

Click Here For Bundle PDF Course | support@guidely.in Page 9 of 9


Bank Po Mains PDF Course 2024
Reasoning Day -21 (Eng)

Reasoning Aptitude
Directions (1-5): Study the following information a) As many persons attend a marriage between
carefully and answer the following questions. S and W
In a three generations family of eight members – b) Three
P, Q, R, S, T, U, V, and W attend a marriage c) As many persons attend a marriage after T
event in four different months of the same year d) Both a) and c)
viz.- January, April, August, and November on e) None of these
two different dates viz.- 9 and 12 but not
necessarily in the same order. Only one person 3) Who among the following person attends a
attended the marriage on each date. Three marriage immediately after V’s uncle?
married couples are there in the family. One a) P’s wife
married couple has no child. b) The one who attends three persons after S
Q’s father-in-law attends a marriage event on an c) V
even date of the month which has 30 days. S d) W’s father
attends a marriage event on August 12. Q is the e) None of these
mother of V. W is the only daughter of Q’s father-
in-law and attends a marriage event three 4) Four of the following five are alike in a certain
persons before S. P is the father of T, who is way based on the given arrangement and thus
married to U. U is the only sister-in-law of S. form a group. Which one of the following doesn’t
Three persons attend a marriage event between belong to the group?
U and S’s son. U attends a marriage on an odd a) V’s mother
date and immediately before Q. As many b) The one who attends immediately before T
persons attend a marriage between P and V as c) W’s mother
after P’s wife. d) S's brother
1) How W is related to the one who attends a e) R’s husband
marriage on January 9?
a) Mother 5) Which of the following statement is/are not
b) Sister-in-law true?
c) Aunt a) S is the father of V
d) Brother b) Three persons attend between T and R’s
e) None of these husband
c) V attends on August 9
2) How many persons attend a marriage event d) W is the aunt of V
between Q and S’s father? e) None of these

Click Here For Bundle PDF Course | support@guidely.in Page 1 of 12


Bank Po Mains PDF Course 2024
Reasoning Day -21 (Eng)

7) Six persons P, Q, R, S, T and U are of


Directions (6-10): Each of the questions below different heights. Who is the tallest person
consists of statements below it. You must decide among them?
whether the data provided in the statements are a) Q is shorter than S who is shorter than P. U is
sufficient to answer the question and mark the shorter than T who is not the second shortest
appropriate option as answer. person. At least two persons are taller than P.Q
6) Six different vegetables viz., Carrot, Potato, is not the shortest person.
Tomato, Onion, Brinjal and Pumpkin were b) Exactly three persons are shorter than U but
purchased on different days from Monday to taller than R. S is taller than Q but shorter than P
Saturday, then which of the vegetable was as well as T.
purchased on Wednesday? c) P is taller than S. P is shorter than at least two
a) Tomato was purchased after Thursday. Only persons. Only one person is shorter than Q who
two vegetables were purchased between Tomato is shorter than S. U is neither the tallest nor the
and Onion. Brinjal was purchased before shortest person.
Pumpkin. d) P is taller than T who is taller than only two
b) Pumpkin was purchased on Thursday. Only persons. Q is taller than S who is the shortest
one vegetable was purchased between Pumpkin person. R who is not the tallest person, is taller
and Onion. Two vegetables were purchased than U who is not the second shortest person.
between Onion and Tomato. Potato was e) P is taller than S and T. R is taller than S. Q is
purchased before Carrot. not the shortest person.
c) Only one vegetable was purchased before
Potato. Onion was purchased two days after 8) Seven persons J, K, L, M, N, O and P are in a
Potato. As many vegetables were purchased family of three generation. A single person
after Tomato is same as before Brinjal. doesn’t have a child. There are three married
d) Only two vegetables were purchased between couples. How is N related to L?
Brinjal and Carrot. Onion was purchased four a) M is the father-in-law of K. L is the mother of
days after Carrot. Pumpkin was purchased N. J is not in the third generation. P is the sister
before Tomato. in law of K.
e) Carrot was purchased before Wednesday. b) L is the mother of N. M is the grandfather of N
More than two vegetables were purchased who is married to O. K is not married to M and is
between Onion and Pumpkin. Brinjal was not the son of M. K doesn’t have a sibling.
purchased before Pumpkin but after Potato. c) N and J are married couples. L is the child of
Pumpkin was not purchased on Saturday. M. P is the brother in law of K.

Click Here For Bundle PDF Course | support@guidely.in Page 2 of 12


Bank Po Mains PDF Course 2024
Reasoning Day -21 (Eng)

d) N is the grandchild of M. K is the daughter of 10) Eight persons G, H, I, J, K, L, M and N sit


L. O is not married to N. around a circular table facing towards the centre.
e) J is the mother of L who is the mother of N. K Who sits to the immediate right of H?
is not married to J. O is a female member and a) M sits second to the right of K who sits third to
doesn’t have siblings. P is the maternal aunt of the right of J. L sits second to the right of G who
N. sits adjacent to K. H sits adjacent to J.
b) J and I are the neighbors of L.Only one person
9) Seven persons A, B, C, D, E, F and G live on sits between J and M. K sits second to the right
different floors of a seven storey building. The of M. N sits adjacent to K.
bottommost floor is numbered as 1 and the c) N sits second to the right of L. M sits fourth to
topmost floor is numbered as 7. Who lives the right of N.I sits second to the left of G. I sits
immediately above B? adjacent to N.K sits opposite to L.
a) A lives on an odd numbered floor and d) J sits sixth to the left of L. I sits second to the
immediately above C. There are two floors right of J. Only two persons sit between M and I.
between A and D. F lives two floors below D. G sits to the immediate right of K. N doesn’t sit
b) D lives two floors above F. The number of adjacent to I. M faces H.
floors above D is the same as the number of e) H sits fourth to the right of the person, who sits
floors below C. G lives above E who lives above to the immediate left of M. L sits second to the
B. right of P. K sits adjacent to I.
c)F lives two floors above C. There are three
floors between C and D. B lives two floors below Directions (11-15): Study the following
E. G lives two floors above E who doesn’t live on information carefully and answer the below
even numbered floor. A doesn’t live above E. questions
d) D lives on even numbered floor. E lives three Certain number of persons are sitting around a
floors below D. G lives above B but neither on circular table of diameter 273cm. All the persons
fourth floor nor fifth floor. F lives three floors are facing outside the table. The distance
below G. between the adjacent persons is different and is
e) The number floors above C is one more than a consecutive multiple of eleven in a clockwise
the number of floors below F. F doesn’t live direction. The minimum distance between each
below C. D lives three floors above F. person is 11cm. The number of persons sitting
A lives immediately below C. around the table is divisible by six also not
exceeding fifteen.
A sits 165cm to the left of G. The number of
persons sitting between A and G is a prime

Click Here For Bundle PDF Course | support@guidely.in Page 3 of 12


Bank Po Mains PDF Course 2024
Reasoning Day -21 (Eng)

number. D sits third to the left of B. One person 14) If Z sits fourth to the right of B, then what is
sits between D and E. As many persons sit the distance between Z and C when measuring
between B and E when counted from the left of B from the left of Z?
as between G and D when counted from the right a) 210cm
of G. The distance between E and D is same as b) 280cm
the distance between F and C when counted c) 297cm
from the right of both D and C. Neither F nor C is d) 209cm
sitting adjacent to E. e) None of these
11) Who among the following sits third to the left
of A? 15) How many persons are sitting at the table?
a) G a) Eleven
b)The one who sits to the immediate left of F b) Fourteen
c) B c) Twelve
d) The one who sits sixth to the right of D d) Thirteen
e) None of these e) None of these

12) What is the distance between G and E when Directions (16-20): Read the given passage
measuring from the right of G? carefully and answer the questions based on the
a) 165cm same respectively
b) 143cm In scenes reminiscent of the Arab Spring, (A)
c) 121cm citizens stormed the residences of Sri Lanka’s
d) 132cm president and prime minister in Colombo, (B)
e) None of these forcing both leaders to announce that they would
quit their respective offices. President Gotabaya
13) Which of the following statements is/are true Rajapaksa, the main target of the protests that
with respect to the final arrangement? erupted in April over food and fuel shortages and
a) A sits second to the right of F have continued since, promised to resign by
b) B sits third to the right of G Wednesday. The Speaker of Sri Lanka’s
c) E sits adjacent to A parliament, Yapa Abeywardena, is expected to
d) All are true take over as acting president and prime minister,
e) None of these and facilitate, possibly, the setting up of a
national unity government. Gotabaya’s
resignation could bring down the temperature
and help the authorities to persuade the

Click Here For Bundle PDF Course | support@guidely.in Page 4 of 12


Bank Po Mains PDF Course 2024
Reasoning Day -21 (Eng)

protestors to return home. Though the marches a) The people of Sri Lanka have lost trust in their
and sit-ins in Galle Face and elsewhere have leadership
been largely peaceful, the recent violent events b) Gotabaya’s presence in the government has
suggest that the people, battling acute shortage sparked ire
of essentials, including milk, baby food, petrol, c) People of the island country have become
are on the edge. Only a new leadership can win anxious
back public trust in government and steer the d) Both (a) and (b)
island nation’s economy out of choppy waters. e) None of the given options
16) Which of the given options provides the
correct relationship between sentences A and B? 19) Which of the given statements can most
a) A is the cause and B is its effect likely be taken as an ‘assumption’?
b) A is the effect and B is one of the causes a) The speaker takes over the roles of the
c) Both A and B are causes estranged leaders till the government is formed
d) Both A and B are effects of independent b) An island country can only be ruled by a unity
causes government
e) A is the effect and B is its cause c) Violent protests were the only way left before
the people to show their agony
17) Which of the following can be a probable d) The last resort to save the economy was to
‘Course Of Action’ to pacify the anxious people? oust their leaders out of power
a) All the protestors should be jailed so that more e) All statements can be assumed
people don’t join the protests
b) The protestors should be promised with free 20) Which of the following can be ‘concluded’
food and basic facilities for all of them from the given paragraph?
c) Present heads of the country should resign a) It is only the citizens who can bring the country
with immediate effect out of the severe trouble
d) A new government should take charge and try b) A new political leadership is needed to steer
to restore the trust of the people Sri Lanka out of present impasse
e) The protestors should be given employment c) The violent protests are a result of the
which would ensure consistent money flow negligence of the government
d) The food scarcity is a proof that the country
18) Which option can be marked as an was being mismanaged
‘Inference’ as per the contents of the given e) Graft had taken the centre stage and the
paragraph? economy was in doldrums while the government
enjoyed

Click Here For Bundle PDF Course | support@guidely.in Page 5 of 12


Bank Po Mains PDF Course 2024
Reasoning Day -21 (Eng)

Click Here to Get the Detailed Video Solution for the above given Questions
Or Scan the QR Code to Get the Detailed Video Solutions

Answer Key with Explanation


Directions (1-5): in case (2) Q’s father-in-law attends a
1) Answer: B marriage event on 12 November.
2) Answer: C  W is the only daughter of Q’s father-in-law
3) Answer: A and attends a marriage event three
4) Answer: D persons before S.
5) Answer: B  S attends a marriage event on August 12.
Based on the above given information we have:

We have:
 Q’s father-in-law attends a marriage event
For Blood Relation:
on an even date of the month which has
30 days.
 Q is the mother of V.
That means, in case (1) Q’s father-in-law
attended the marriage event on 12 April,

Click Here For Bundle PDF Course | support@guidely.in Page 6 of 12


Bank Po Mains PDF Course 2024
Reasoning Day -21 (Eng)

For Blood Relation:

Case (1a) is not valid as no place left for S’s son.


Again, we have:
 Since, one married couple has no child
Again, we have: and three married couples are there in the
 P is the father of T, who is married to U. family.
 U is the only sister-in-law of S. Thus, Q must be married to S and V is the
That means, U must be married to S’s son of S.
brother.  As many persons attend a marriage
 Three persons attended the marriage between P and V as after P’s wife.
event between U and S’s son. That means, in case (1) R attended the
 U attends a marriage on an odd date and marriage event on 12 November, case (2)
immediately before Q. is not valid.
That means, in case (1) & case (2) U Based on the above given information we have:
attended marriage on January 9, case
(1a) is not valid
Based on the above given information we have:

Case (2) is not valid as many persons attend a


marriage between P and V as after P’s wife.
For Blood Relation:

Click Here For Bundle PDF Course | support@guidely.in Page 7 of 12


Bank Po Mains PDF Course 2024
Reasoning Day -21 (Eng)

 Q is taller than S who is the shortest


person.
 R who is not the tallest person, is taller
than U who is not the second shortest
person.
P is the tallest person.

6) Answer: E
We have,
 Carrot was purchased before
Wednesday.
 More than two vegetables were
purchased between Onion and Pumpkin.
 Brinjal was purchased before Pumpkin
but after Potato.
 Pumpkin is not purchased on Saturday. 8) Answer: E
From the above conditions Case1 and Case2a We have,
gets eliminated. Hence Case2 shows the final  J is the mother of L who is the mother of
arrangement. N.
Potato was purchased on Wednesday.  K is not married to J.
 O is a female member and doesn’t have
siblings.
 P is the maternal aunt of N.
From the above conditions N is the son of L.

7) Answer: D
9) Answer: C
We have,
We have,
 P is taller than T who is taller than only
 F lives two floors above C.
two persons.

Click Here For Bundle PDF Course | support@guidely.in Page 8 of 12


Bank Po Mains PDF Course 2024
Reasoning Day -21 (Eng)

 There are three floors between C and D.  N doesn’t sit adjacent to I.


 B lives two floors below E.  M faces H.
 G lives two floors above E who doesn’t From the above conditions Case2 gets
live on even numbered floor. eliminated. Hence Case1 shows the final
 A doesn’t live above E. arrangement.
From the above conditions Case2 gets I sits to the immediate right of H.
eliminated. Hence Case1 shows the final
arrangement.
F lives immediately above B.

Directions (11-15):
11) Answer: B
12) Answer: D
13) Answer: A
14) Answer: D
15) Answer: C
Final arrangement:
10) Answer: D
We have,
 J sits sixth to the left of L.
 I sits second to the right of J.
 Only two persons sit between M and I.

We have,
Since, diameter of the table = 273cm.
Thus, circumference = 2 x 22/7 x 273/2 =
Again we have,
858cm.
 G sits to the immediate right of K.

Click Here For Bundle PDF Course | support@guidely.in Page 9 of 12


Bank Po Mains PDF Course 2024
Reasoning Day -21 (Eng)

 A sits 165cm to the left of G.  The distance between E and D is same


 The number of persons sitting between A as the distance between F and C when
and G is a prime number. counted from the right of both D and C.
From the above conditions, there are two  Neither F nor C is sitting adjacent to E.
possibilities So Case 1, Case 2 and Case 2a get eliminated,
hence Case 1a shows the final arrangement.

Again we have,
 D sits third to the left of B.
 One person sits between D and E.
 As many persons sit between B and E
when counted from the left of B as
between G and D when counted from the
right of G.
16) Answer: E
The first sentence ie; A talks about the crowd
that barged into the residences of the President
and the PM. If we wonder, what made them do
the same? We get our answer for this is B which
says that the citizens wanted that these two
leaders should leave their offices. So, we can
say that; because of B  A took place…
We can clearly understand that, A is the effect
and B is the cause. We have two options, (b)
and (e) to confuse from.
If we go with (b) which says B is one of the
causes, the other causes than this one should
also be mentioned which is not the case. B is the
Again we have,
only immediate cause that has been quoted.

Click Here For Bundle PDF Course | support@guidely.in Page 10 of 12


Bank Po Mains PDF Course 2024
Reasoning Day -21 (Eng)

So, this makes option (e) the best choice for the Towards the end of the paragraph, it has been
answer. given that a new government can restore the
trust of the people to inspire them to stop their
17) Answer: D violent protests. From this, we can infer that the
A Course of Action should always be inclined first option should be correct.
towards lessening the problem and never an It has been given that Gotabaya’s resignation
extreme step. The option that one chooses would bring down the anger. From this we can
should either solve the issue given in the context infer that his presence in the government has
or at least lessen the intensity of the same. sparked a rage in the country.
The first option is an extreme step which should Thus, of the three options, we can clearly see
be avoided as it won’t make any difference. that the first two can be inferred and the third
Option (b) is not a solution as it would put a lot of can be left out. So, the correct answer is option
pressure on the country and increase the (d).
problem indirectly.
It has been given in the paragraph that the 19) Answer: A
people have become anxious as they have no An assumption should be chosen in the reverse
access to even the basic necessities so as per order. We must start with the options and try to
the content above, it has been said that a new find that if the particular option were assumed,
leadership should quickly take charge. Option (c) would that lead us to any conclusion given in the
could be correct had the issue been stopping the paragraph or the sentences given. Like for
protests. But the issue asked is mentioned at the example;
end of the paragraph. (e) becomes superfluous If we take the first option to be an assumption,
looking at the situation given in the paragraph. we can very well conclude the point given in the
So, the correct answer is option (d). paragraph that the speaker Yapa would be
taking over the charge till the government is
18) Answer: D formed. So, this is correct.
An Inference is something that can be deduced Option (b) cannot be taken as an assumption as
on the basis of some known fact. It is not nothing about the same or related to the same
something that is directly given in the paragraph. has been clearly mentioned. It has nowhere
This becomes the reason that we cancel the been mentioned that the violent protests were
third option as it has been discussed directly at the only way out. Even the statement in (d)
multiple places in the paragraph. doesn’t count for an assumption as it has directly
been discussed as an issue in the paragraph.

Click Here For Bundle PDF Course | support@guidely.in Page 11 of 12


Bank Po Mains PDF Course 2024
Reasoning Day -21 (Eng)

So, the best would be to mark option (a) as the governance as they have been left at the mercy
answer. of no one.
The first option is superfluous and can be ruled
20) Answer: B out. Option (c) is correct as given in the
A conclusion is logically deduced based on the paragraph but it cannot be taken as the
contents of the given passage. The whole conclusion. Same goes for option (d).
paragraph revolves around one single idea that We have no clue about graft and corruption
the issues that have led to protests can be which makes it wrong for us to mark option (e).
addressed only with the change in the So, it can be eliminated.
government. The people have lost trust in their Thus, the correct answer is option (b).

Click Here For Bundle PDF Course | support@guidely.in Page 12 of 12


Bank Po Mains PDF Course 2024
Quantitative Aptitude Day -21 (Eng)

Quantitative Aptitude

Directions (01 - 04): Study the following information carefully and answer the questions given below.
There are five families [A, B C, D and E]. There are six members [father, mother, daughter, son,
grandfather and grandmother] in each family. The given table charts show the age of the grandfather, the
ratio of age of grandfather and grandmother, the ratio of age of the father and mother, the age of the
daughter and the age of the son. Some values are missing.

1) The age of the daughter of family A is 10% 2) The ratio of the age of son and daughter of the
less than the age of the son of that family. The family B is 11:12. The sum of the ages of the
difference between the age of the father in family daughter and grandfather in family B is (25*4+8)
A and the age of his mother is equal to (40% of years. The age of a mother is 3 years older than
50 years). The age of the mother is double the 56.25% of the age of a grandmother. The age of
age of the son after 2 years, and the grandfather the father is 2 years less than the age of the
is 4 years older than the grandmother. Find the mother. If the difference between the ages of
possible average age of family A where y is father and grandfather is x, what is the possible
equal to 7.5. difference between the ages of son and
I.8y-15 daughter?
II.7y-21 I.(x/19)
III.3y+22.5 II.0.4x-13.2
a) Only I and II III.0.9x-32.4
b) only III a) Only II and III
c) only I and III b) only I
d) only II and III c) Only II
e) only II d) only I and II

Click Here For Bundle PDF Course | support@guidely.in Page 1 of 12


Bank Po Mains PDF Course 2024
Quantitative Aptitude Day -21 (Eng)

e) only I and III and grandfather of D is 123. The age of the


daughter of family E is half of the father's age 5
3) The age of the father of family C is years ago.
(45*2.5*2/(7.5*15)) times the age of the son. The Statement II: The age of the grandfather of family
age of the mother of the same family is 140% E and the age of the grandmother of family D is
more than the age of the daughter. The average the same. The present Age of the daughter of
age of the father and grandfather and the family D is half of the age of the mother of E after
average age of the grandmother and mother of 10 years. The age of the mother of the families D
family C are the same, i.e., ___ years. The and E is the same.
difference between grandfather's and A. The data in statement I alone is sufficient to
grandmother's ages is ___ years. answer the question, while the data in statement
Determine the possible values that will fill in the II alone is not sufficient to answer the question
given blanks respectively. B. The data in statement II alone is sufficient to
a) 76, 2 answer the question, while the data in statement
b) 70, 2 I alone is not sufficient to answer the question
c) 78, 4 C. The data either in statement I alone or in
d) 82, 5 statement II alone is sufficient to answer the
e) More than one option question
D. The data given in both statements I and II
4) Find the sum of the ages of the daughters of together are not sufficient to answer the question
families D and E. E. The data given in both statements I and II
Statement I: The ratio of the age of sons in family together are necessary to answer the question.
D and E is 8:9. The sum of the ages of the father

Directions (05 - 08): Study the following information carefully and answer the questions given below.
The given pie chart shows the percentage distribution of total expenditure of five different companies in
the year 2022.

Click Here For Bundle PDF Course | support@guidely.in Page 2 of 12


Bank Po Mains PDF Course 2024
Quantitative Aptitude Day -21 (Eng)

Note- Each company makes profit of 25% in the year 2022. The income of Company D in the year 2022
is 135 lakhs. The ratio of the total income of each company in 2022 and 2023 is 5:6 and every company
makes profit of 25% in 2022 and 2023. Profit = [Income – expenditure] *100/expenditure
5) Find which of the following is true? b) 67.2%
I. The average expenditure of A in 2022 and c) 12.5%
2023 is 88 lakhs. d) 66.66%
II. The average expenditure of C in 2022 and e) 19.32%
2023 is 103.25 lakhs.
III. The average expenditure of B in 2022 and 7) The income of Company B in 2022 is _______
2023 is 96.2 lakhs. % less than the income of Company E in 2022
a) only II and the income of Company E in 2023 is
b) only I and II ______% more than the income of Company C in
c) only I 2023.
d) only II and III Find which of the following satisfies the blank?
e) only I and III a) 22.5%, 12.5%
b) 25%, 16.66%
6) The ratio of income of Company C in 2022 c) 16.66%, 25%
and 2021 is 5:6. The ratio of expenditure of d) 12.5%, 22.5%
Company C in 2022 and 2021 is 4:3. Find the e) None of these
decrease in profit percentage of Company C
from 2021 to 2022? 8) The ratio of income of Company E and F in
a) 62.55% 2022 is 7:8 and the ratio of income of Company

Click Here For Bundle PDF Course | support@guidely.in Page 3 of 12


Bank Po Mains PDF Course 2024
Quantitative Aptitude Day -21 (Eng)

E and F in 2023 is 4:7. The profit percentage of b) 92


Company F in 2022 and 2023 is 25% and 40% c) 88
respectively. Find the difference between the d) 94
expenditure of F (in lakhs) in 2022 and 2023? e) None of these
a) 82

Directions (09 - 12): Study the following information carefully and answer the questions given below.
The given chart shows the number of fruits (in kg) sold by four different shops [A, B, C and D].

Note – The quantity of papaya sold in shop A lies between 80 and 90. Quantity of apples sold in shop A
lies between 85-95. Yellow, red, blue and green colours represent shops A, B, C and D respectively.
The quantity of fruits sold by all the shops is in multiples of five
9) The quantity of grapes sold in A is 20% more c) 33
than the quantity of apples sold in the same d) 39
shop. The ratio of the quantity of grapes sold in A e) 40
and B is 2:3. The average quantity of grapes sold
in B and C is 102 kg. Find the difference 10) The difference between the quantity of
between apples and grapes sold in C? papayas and oranges sold in B is x and the
a) 32 difference between the quantity of apples sold in
b) 37 A and D is 2y. Find which of the following is true?

Click Here For Bundle PDF Course | support@guidely.in Page 4 of 12


Bank Po Mains PDF Course 2024
Quantitative Aptitude Day -21 (Eng)

I. The total quantity of fruits sold in B is 8x. A. Quantity: I < Quantity: II


II. The total quantity of fruits sold in C is B. Quantity: I ≥ Quantity: II
7.5x+13.5. C. Quantity: II ≥ Quantity: I
III. The total quantity of oranges sold in all shops D. Quantity: I > Quantity: II
is 10y+25. E. Quantity I = Quantity II or relation can't be
a) II and III true established
b) I and II true
c) only I true 12) The average number of papayas sold in
d) I and III true shops A and E, apples in shops C and E and
e) only II true oranges in shops D and E is 92 kg, 84 kg and 81
kg respectively. Find the difference between the
11) Quantity: I The total quantity of papayas sold total quantity of fruits sold in B and E?
in A and B together is what percent of the total a) 79
quantity of oranges sold in C and D together? b) 69
Quantity: II The total quantity of apples sold in B c) 99
and C together is what percent of the total d) 74
quantity of papayas sold in C and D together? e) None of these

Directions (13- 16): Study the following information carefully and answer the questions given below.
The given bar graph shows the number of Math books (ssc+bank) and English books (ssc+bank) sold in
a shop for first five months respectively.

Click Here For Bundle PDF Course | support@guidely.in Page 5 of 12


Bank Po Mains PDF Course 2024
Quantitative Aptitude Day -21 (Eng)

Note – In Each month, math books for SSC and Bank sold in the ratio of 3:2 respectively and English
books for SSC and Bank sold in the ratio of 3:1 respectively.
13) The number of reasoning books sold in the total math books sold for bank in April and
February for SSC is 25% more than the number May together is 28.
of math books sold in February for SSC. The II. The difference between the total number of
number of reasoning books sold in February for math and English books sold in April is 5.
the Bank is 25% less than the total number of III. The total number of English books sold for
English books sold in February. Find the total Bank in all months together is 118.
number of reasoning books sold in February? a) only I true
a) 128 b) only I and II true
b) 119 c) only II and III true
c) 123 d) none is true
d) 113 e) only II true
e) 133
15) The price of a Math book for SSC sold in
14) Find which of the following is true? March is Rs. 320 and the price of a math book
I. The difference between the total English books for bank sold in March is Rs.280. The price of an
sold for the bank in April and May together and English book for SSC sold in March is Rs. 260

Click Here For Bundle PDF Course | support@guidely.in Page 6 of 12


Bank Po Mains PDF Course 2024
Quantitative Aptitude Day -21 (Eng)

and the price of an English book for bank sold in 18) x2-px+48=0 and x2-qx+130=0. q is 17 more
March is Rs.240. Find the total revenue earned than p. The smaller root of the first equation is 1
by the shop in March? more than the smaller root of the second
a) 44520 equation, larger root of the second equation is
b) 43680 26. Find the equation whose smaller root is equal
c) 49620 to larger root of the first equation and larger root
d) 47620 is equal to the value of p?
e) None of these a) x2-22x+120=0
b) x2-18x+112=0
16) The total number of Math books sold in June c) x2-20x+112=0
is 20% more than the number of math books sold d) x2-22x+112=0
in May and the total number of English books e) x2-22x+102=0
sold in June is 25% more than the number of
English books sold in May. If the total number of 19) x2-px+42=0 and x2-38x+q=0. The sum of the
books sold for the Bank (in both months) is the roots of the first equation is 23 and the
same then find the ratio of the number of math multiplication of the roots of the second equation
and English books sold for SSC in June? is 325. Find the equation whose one root is p
a) 1:1 and another root is q/13.
b) 1:2 a) x2-(2q/5)x + 575 = 0
c) 2:1 b) x2-(q/5 -2)x + 575 = 0
d) 5:6 c) x2-(2q/5 -2)x + 575 = 0
e) can’t be determined d) x2-(2p/5 -2)x + 575 = 0
e) None of these
17) I. x2-16x+63=0 and II. x2-Px+221=0
The larger root of the first equation is 4 less than 20) The sum of roots of equation x2-px+132=0 is
the smaller root of the second equation. Find the 28. The larger root of equation x2-qx+r=0 is 9
multiplication of the value of p and larger root of less than the larger root of the first equation and
the second equation? the smaller root is 2 more than the smaller root of
a) 720 the first equation. Find the value of q+r=?
b) 630 a) 125
c) 510 b) 129
d) 910 c) 120
e) 240 d) 123
e) None of these

Click Here For Bundle PDF Course | support@guidely.in Page 7 of 12


Bank Po Mains PDF Course 2024
Quantitative Aptitude Day -21 (Eng)

Click Here to Get the Detailed Video Solution for the above given Questions
Or Scan the QR Code to Get the Detailed Video Solutions

Answer Key with Explanation


1) Answer: C II.0.4x-13.2=0.4*38-13.2=2
The age of the son of family A is 18*100/90 = 20
years. 3) Answer: B
The mother's age is [20+2]*2 = 44 years. The age of the father of family C is
The age of the grandmother is 72-4=68. =25*[45*2.5*2/(7.5*15)] =25*2=50 years
Age of father is = 68 - [40*50/100] =68-20=48 The age of the mother of family C is =
So, the average age is = 20*240/100 = 48 years.
[72+68+48+44+20+18]/6 = 270/6 = 45. If we use the values given in option (b)
=8y-15=3y+22.5 The age of the grandfather is = 70*2-50 = 90.
The age of the grandmother is = 70*2-48 = 92.
2) Answer: D So, the difference is 92-90=2.
The age of the daughter of family B is = So, it's satisfying.
22*12/11 = 24.
Age of grandfather is = 25*4+8-24=108-24=84 4) Answer: E
The age of the grandmother is = 84*20/21 = 80. From single statements, we cannot calculate the
The mother's age is equal to 80 * 56.25/100 + 3 answer.
= 45 + 3 = 48 years. From both statements,
The age of the father is 48-2=46. The age of the grandmother of family D is 80.
So, x=84-46=38 The age of the grandfather of D is 80*15/16=75.
So, the difference between the ages of son and The age of the father of family D is 123-75=48.
daughter is 24-22, or 2 years. The age of the mother of family D is
So, I. x/19 = 38/19 = 2. 48*14/16=42.

Click Here For Bundle PDF Course | support@guidely.in Page 8 of 12


Bank Po Mains PDF Course 2024
Quantitative Aptitude Day -21 (Eng)

The age of the mother of family E is 42 years. 6) Answer: D


The age of the father of family E is 42*15/14=45. Profit in 2021 is = [120*6/5] – [96*3/4]=144-
The age of the daughter of D is = [42+10]/2=26. 72=72
The daughter of family E is [45-5]/2 = 20 years Profit in 2022 is = 120-96=24
old. So, decrease percentage = [(72-
So, the required sum is 26+ 20 = 46. 24)]*100/72=66.66%

Directions (05 - 08): 7) Answer: B


The income of company D in 2022 is 135 lakh. The income of company B in 2022 is [140-
Expenditure of company D in 2022 is = 105)*100/140]=25% less than the income of
135*100/125=108 lakh. company E in 2022
So, the total expenditure of all company is = The income of company E in 2023 is [(168-
108*100/22.5=480 lakh 144)/144]*100=16.66% more than the income of
So, expenditure of A in 2022 is = company C in 2023.
480*16.67/100=80 lakh
Total income of A in 2022 is = 80*125/100=100 8) Answer: A
lakh. Expenditure of F in
The income of A in 2023 is = 100 *6/5=120 lakh. 2022=[140*8/7]*100/125=128
Expenditure in A in 2023 is = 120*100/125=96 Expenditure of F in 2023 =[168*7/4]*100/140=
lakh. 210
Similarly, we can calculate others' values also. So, the difference is = 210-128=82 (in lakhs)

Directions (09 - 12):

5) Answer: C
I. The average expenditure of A in 2022 and
2023 is =[80+96]/2=88 lakhs.
II. The average expenditure of C in 2022 and
2023 is =[96+115.2]/2=105.6 lakhs. 9) Answer: C

III. The average expenditure of B in 2022 and The quantity of grapes sold in A is =

2023 is [84+100.8]/2=92.4 lakhs 90*120/100=108

Click Here For Bundle PDF Course | support@guidely.in Page 9 of 12


Bank Po Mains PDF Course 2024
Quantitative Aptitude Day -21 (Eng)

The Quantity of grapes sold in B is = So, required difference = 279-[75+75+50]=279-


108*3/2=162 200=79
The Quantity of grapes sold in C is = 102*2-
162=204-162=42 Directions (13- 16):
So, The required difference = 75-42=33 The number of Math books sold in January,
February, March, April and May is
10) Answer: D =120, [200-120] =80, [290-200] =90, [400-290]
The difference between the quantity of papayas =110, 520-400=120 respectively.
and oranges sold in B is x = 75-50=25 Number of English books sold in January,
The difference between the quantity of apples February, March, April and May is =96, [180-96]
sold in A and D is 2y = 90-50=40, y=40/2=20. =84, [244-180] =64, [364-244] =120, 460-364=96
I. Total quantity of fruits sold in B is respectively.
=75+75+50=200=8*25=8x. The number of Math books sold for SSC in
II. Total quantity of fruits sold in C is = January is = 120*3/5=72
50+75+75=200 The Number of math books sold for Bank in
= 7.5*25+13.5=201=7.5x+13.5. January is = 120-72=48
III. The total quantity of oranges sold in all shops The Number of English books sold for SSC in
is = 25+50+75+75=225=10y+25. January is = 96*3/4=72
The Number of English books sold for Bank in
11) Answer: A January is = 96-72=24
Quantity I Similarly, we can calculate the others values
Required percentage = [(85+75)/(75+75)]*100 = also.
106.66%
Quantity II
Required percentage = [(75+75)/(50+75)]*100 =
120%
Quantity I < Quantity II

12) Answer: A
The total quantity of fruits sold in shop E is
= [92*2-85] + [84*2-75] + [81*2-75]
13) Answer: C
= 99+93+87=279
The total number of reasoning books sold in
February is

Click Here For Bundle PDF Course | support@guidely.in Page 10 of 12


Bank Po Mains PDF Course 2024
Quantitative Aptitude Day -21 (Eng)

=48*125/100 + 84*75/100= 123 The smaller root of the second equation is


9+4=13
14) Answer: D Let the larger roots of the second equation be y.
I. The Difference between the total English So, y+13=p
books sold for bank in April and May together And 13y=221
and the total math books sold for bank in April Or, y=17
and May together is =[44+48-30-24]=38. So, p=17+13=30
II. The Difference between the total number of So, the required value = 30*17=510
math and English books sold in April is =90+30-
66-44=120-110=10. 18) Answer: D
III. Total number of English books sold for Bank So, q-p=17
in all months together is 24+21+16+30+24=115. Let the roots of first equation be I and J and the
15) Answer: B roots of the second equation be m and n. and
The total revenue earned by the shop in March also consider I>J and m>n
is So, I+J=p and I*J=48, m+n=q, m*n=130
= 320*54+280*36+260*48+240*16=43680 Or, J-n=1, m=26, so, n=130/26=5
So, j=5+1=6
16) Answer: E So, q=26+5=31, p=31-17=14
The total number of Math books sold in june So, I=14-6=8
= 120*120/100 = 144 The sum of roots of the required equation is
The total number of english books sold in june 8+14=22
96*5/4 =120 The multiplication of roots is 8*14=112
Total number of books sold for bank = 72 So, the equation is x2-22x+112=0
But , we cannot find the total number of math
and English books for ssc in june from the given 19) Answer: E
information so answer is E So ATQ, p=23 and q=325
So, the sum of roots of the required equation is
17) Answer: C 23+325/13=23+25=48
x2-16x+63=0 Multiplication of roots of the required equation is
or, x2-9x-7x+63=0 = 23*25=575
or, (x-9)(x-7)=0 So, the required equation is = x2-48x+575=0
so, x=9,7. 20) Answer: A
II. x2-Px+221=0 The sum of roots of equation x2-px+132=0 is 28.

Click Here For Bundle PDF Course | support@guidely.in Page 11 of 12


Bank Po Mains PDF Course 2024
Quantitative Aptitude Day -21 (Eng)

So, p=28 The larger root of the second equation is 22-


So, x2-28x+132=0 9=13 and the smaller root is 6+2=8.
Or, x2-22x-6x+132=0 So, q=13+8=21
Or, (x-22)(x-6)=0, x=22, 6 And, r=13*8=104
So, q+r=104+21=125

Click Here For Bundle PDF Course | support@guidely.in Page 12 of 12


Bank Po Mains PDF Course 2024
English Day - 21

English Language
Directions (1-5): In the following passage, some observed. The tech workforce has found new
of the words have been left out, each of which is love with remote working, giving them freedom
indicated by a number. Find the suitable word and productivity that requires HR to be more
from the options given against each number and trusting, it said adding that unengaged or
fill up the blanks with appropriate words to make (4)___________ employees will find alternatives
the paragraph meaningful. faster and exit faster than pre-pandemic times."It
With the COVID-19 pandemic bringing is interesting to observe how the fast
unprecedented changes in work life, a study has (5)__________ 'Future of Work' is getting past the
revealed that 82% of (1)__________ admitted that world of conventional HR and transforming into
they prefer working from home to going back to an ecosystem that now involves people beyond
the office.The remote work trend was initially the confines of company payroll and office
forced on employees due to the pandemic, boundaries.
however, after two years remote working has 1. Fill in the blank (1)
become a new normal and as things (2)________ a) Responses
down new habits have formed.The Talent Tech b) Respondents
Outlook 2022 study is an analysis of inputs from c) Senators
100 plus C-suite and human capital leaders d) Employee
across four continents, done by surveys, social e) Spectators
media inputs, interviews and panel
discussions.The study further (3)_________ that 2. Fill in the blank (2)
64% employees said they are more productive a) Settled
working from home and feel less stressed.The b) Drowned
new normal for HR shifted from engaging face to c) Fell
face to engaging with remote employees whom d) Sorted
they meet only virtually most of the time, it e) Resolved
stated.More than 80% of the HR managers
admitted that hiring employees for full-time office 3. Fill in the blank (3)
presence is getting more difficult, the study a) Said
said.Working from home is no longer a choice b) Left out
but a new normal that every tech talent expects c) Revealed
from their employer and the employers not willing d) Concealed
to adapt will face challenges in hiring good talent e) Wrote
and retaining those already employed, it

Click Here For Bundle PDF Course | support@guidely.in Page 1 of 9


Bank Po Mains PDF Course 2024
English Day - 21

4. Fill in the blank (4) I. George Federick is the present incumbent and
a) Impolite he has three more years to serve.
b) Undeserving II. She felt it incumbent on her to be there for her
c) Inappropriate cousin during the surgery.
d) Involved III.The incumbent has been awarded a new
e) Underutilised contract despite the heavy competition.
a) Only I
5. Fill in the blank (5 ) b) Only II
a) Running c) Only III
b) Forwarding d) Both II and III
c) Moving e) All are correct
d) Evolving
e) Proceeding 8. AMBIVALENT
I. She remained ambivalent on her promotion as
Directions (6-10): In each of the following she loves both her career and family.
questions, a highlighted word is given followed II. Ambivalent decisions always lead to disaster
by three sentences. Choose the sentence(s) that or failure because of the lack of confidence.
has/have the wrong usage of the highlighted III.There is ambivalency among men when a
word. woman or girl rides a gear bike.
6. DISPARATE a) Only I
I. The people in the community are so disparate b) Only II
that their ideologies are very distinct and difficult c) Only III
for any other common man to understand. d) Both I and III
II. Ravi became disparate after the thief stole his e) All are correct
wallet and ran away.
III.She has always got disparate ideas for which 9. LIGHTNING
she has recently been awarded by the corporate. I. It was a lightning strike by Sir Ravindra Jadeja
a) Only I with the last two balls.
b) Only II II. The thunder lightning was so heavy and bright
c) Only III that the huge neighbourhood tree was struck and
d) Both I and II burst into flames.
e) All are correct III.E-commerce site Amazon offers lightning
7. INCUMBENT deals that have a pre-set time limit to place the
order.

Click Here For Bundle PDF Course | support@guidely.in Page 2 of 9


Bank Po Mains PDF Course 2024
English Day - 21

a) Only I defending champion Gujarat Titans with


b) Only II Ravindra Jadeja thumping the winning runs in a
c) Only I and II final that was spread over three days.
d) Both II and III (D) The weather may have been fickle with a
e) All are correct summer night yielding rain but there was no
mistaking the inevitable air around Chennai
10. CALLED Super Kings (CSK) lifting the Indian Premier
I. The marriage was called off after the dowry League (IPL) Trophy on a Monday night that
issue. spilled over to Tuesday.
II. I called her around 9:30 AM in the morning but (E) The target was revised to 171 following a
she was busy and told me to send a text rain-break but CSK proved equal to the task
message. (even though Dhoni fell for a duck) as Jadeja’s
III.I was called down by my mother for not putting long-handle did the trick.
the clothes to wash. 11. Which of the following is the
a) Both I and II INTRODUCTORY sentence after the
b) Both II and III rearrangement ?
c) Both I and III a) A
d) All are wrong b) B
e) All are correct c) C
d) D
Directions (11-15): Rearrange the following five e) E
sentences (A), (B), (C), (D) and (E) in the proper
sequence to form a coherent paragraph and then 12. Which of the following is the FOURTH
answer the questions given below. sentence after the rearrangement ?
(A) Sunday was a washout and on Monday, the a) A
rains intervened and delayed the climax b) B
into the early hours of Tuesday. c) C
(B) CSK prised out Shubman Gill for 39 with d) D
Dhoni’s lightning stumping delivering the fatal e) E
blow, but once Sai Sudharsan hammered a 96 —
the player hails from Chennai — the Titans 13. Which of the following is the SECOND
finished with 214 for four before the intervention. sentence after the rearrangement ?
(C) At Ahmedabad’s Narendra Modi Stadium, a) A
CSK pulled off a last-over heist against b) B

Click Here For Bundle PDF Course | support@guidely.in Page 3 of 9


Bank Po Mains PDF Course 2024
English Day - 21

c) C e) No error
d) D
e) E 17. Sita told Ram that/A she felt ignored as she
was counting on him/B to be there at her 25th
14. Which of the following is the CONCLUDING birthday party/C that was held last thursday/D.
sentence after the rearrangement ? a) A
a) A b) B
b) B c) C
c) C d) D
d) D e) No error
e) E
18. A large number of sim cards have/A never
15. Which of the following is the THIRD sentence being used and people stopped recharging
after the rearrangement ? them/B, so, the company has decided to call
a) A off/C its business as a service provider/D.
b) B a) A
c) C b) B
d) D c) C
e) E d) D
e) No error
Directions (16-20): Read the sentence to find out
whether there is an error in it. The error, if any, 19. Because there is no support by/A the society
will be in one part of the sentence. The letter to eliminate the most inhuman of professions/B,
corresponding to that part will be your answer. If manual scavenging still persist/C which is really
the given sentence is correct as it is, mark the saddening/D.
answer as ‘No Error’. a) A
16. The Indian cricket player was injured b) B
heavily/A while the bowler threw the ball/B c) C
straight on his face/C but he claimed it to be an d) D
accident/D. e) No error
a) A 20. I have recently used/A the services of his
b) B travel agency to/B book a cruise ship for a family
c) C trip to Atlanta/C, which was started two decades
d) D before/D.

Click Here For Bundle PDF Course | support@guidely.in Page 4 of 9


Bank Po Mains PDF Course 2024
English Day - 21

a) A d) D
b) B e) No error
c) C
Click Here to Get the Detailed Video Solution for the above given Questions
Or Scan the QR Code to Get the Detailed Video Solutions

Answer Key with Explanation


1. Answer: B Settled down is a phrasal verb with meaning ‘to
The blank needs a collective noun. get comfortable’ or ‘to get used to’
With the COVID-19 pandemic bringing The remote work trend was initially forced on
unprecedented changes in work life, a study has employees due to the pandemic, however, after
revealed that 82% of respondents admitted that two years remote working has become a new
they prefer working from home to going back to normal and as things settled down new habits
the office. have formed.
Respondent - a person who answers a question Other options don’t fit the blank for the following
or provides a response for a survey. reasons :
Other options don’t fit the blank for the following Drowned - to die in water/to sink
reasons : Fell down - to collapse
Responses - is not the right pick because the Sorted - organised/dealt correctly
sentence has ‘they’ referring to a set of people. Resolved - to fix something
Senator - person working for the government All these words have a meaning that is different
Employee - is in singular form(singular noun) from the context of the sentence which has the
and hence doesn’t fit the blank. blank.
Spectator - a person who is watching an event
3. Answer: C
2. Answer: A Revealed fits the blank correctly.

Click Here For Bundle PDF Course | support@guidely.in Page 5 of 9


Bank Po Mains PDF Course 2024
English Day - 21

The study further revealed that 64% employees Deserving, inappropriate, involved all these three
said they are more productive working from words will change the original meaning of the
home and feel less stressed. sentence.
Revealed - is to let out some information, here it
means the study further provided the information 5. Answer: D
that 64% of employees said they are comfortable Fast-evolving is the right way to put.
with wfh concept. It is interesting to observe how the fast evolving
Other options don’t fit the blank for the following 'Future of Work' is getting past the world of
reasons : conventional HR and transforming into an
Using said will cause redundancy in the ecosystem that now involves people beyond the
sentence and it is not the best option too. confines of company payroll and office
Left out - to leave something/fail to include boundaries.
Concealed - to hide something Running, moving, proceeding and forwarding are
Wrote is an inappropriate word and makes the actions of motion but fast evolving fits the blank
sentence meaningless. perfectly.
Evolving - to develop/to grow
4. Answer: E
The blank (4) needs a negative word because it 6. Answer: B
is preceded by a negative word unengaged. Disparate means ‘to be different’. The word is
The tech workforce has found new love with correctly used in sentence I and III.But usage of
remote working, giving them freedom and the word ‘disparate’ is completely wrong in
productivity that requires HR to be more trusting, sentence II because the disparate should be
it said adding that unengaged or underutilised replaced with desperate. ‘Ravi became
employees will find alternatives faster and exit desperate’ would be the correct way instead the
faster than pre-pandemic times. word disparate is used - which does not add
Underutilised - to not use something/someone meaning to the sentence.
up to the level
Other options don’t fit the blank for the following 7. Answer: E
reasons : All the sentences are correct and have the
Impolite cannot fit the blank because the context correct usage of the word incumbent.
here is about the employees contribution and not Incumbent - an official/a person who holds a
about their qualities/character. position/a business

Click Here For Bundle PDF Course | support@guidely.in Page 6 of 9


Bank Po Mains PDF Course 2024
English Day - 21

Be incumbent - to feel that it is necessary to be took place providing the opponent team
with someone/to do something at the right time. information and it also adds the message that
the final was delayed for a period of three
8. Answer: C days.The third sentence is ‘A’ as it is revisiting
The sentence III is absolutely wrong because the context of the introductory statement and
‘ambivalency’ must be replaced with explains the reason for delay.This is followed by
‘ambivalence’. sentence ‘B’ as it has a clue ‘intervention’ - from
Sentence I and II are correct as it is.The that we may conclude that the Gujarat Titans
meaning of the word ambivalent/ambivalence - was the first to bat and set a target for CSK.The
to have a mixed feeling or emotion towards last sentence is ‘E’ which talks about the target
something or someone. and the rain break(which is a continuation of
sentence ‘B’) and how the team still managed to
9. Answer: E lift the trophy.
All the given sentences are right as it is.The
word lightning is used in a correct way and the 12. Answer: B
sentences are contextually and grammatically The correct sequence after rearrangement is
correct and meaningful. DCABE.
The first or the introductory sentence should be
10. Answer: E ‘D’ which provides the first information of CSK
All the given sentences are correct and lifting the IPL trophy, the sentence gives more
meaningful. information on the weather and also mentions
Called off - to cancel the details of the days.The second sentence is
Called down - to be angry or be severe ‘C’ because it elaborates the cricket match that
took place providing the opponent team
11. Answer: D information and it also adds the message that
The correct sequence after rearrangement is the final was delayed for a period of three
DCABE. days.The third sentence is ‘A’ as it is revisiting
The first or the introductory sentence should be the context of the introductory statement and
‘D’ which provides the first information of CSK explains the reason for delay.This is followed by
lifting the IPL trophy, the sentence gives more sentence ‘B’ as it has a clue ‘intervention’ - from
information on the weather and also mentions that we may conclude that the Gujarat Titans
the details of the days.The second sentence is was the first to bat and set a target for CSK.The
‘C’ because it elaborates the cricket match that last sentence is ‘E’ which talks about the target

Click Here For Bundle PDF Course | support@guidely.in Page 7 of 9


Bank Po Mains PDF Course 2024
English Day - 21

and the rain break(which is a continuation of information on the weather and also mentions
sentence ‘B’) and how the team still managed to the details of the days.The second sentence is
lift the trophy. ‘C’ because it elaborates the cricket match that
took place providing the opponent team
13. Answer: C information and it also adds the message that
The correct sequence after rearrangement is the final was delayed for a period of three
DCABE. days.The third sentence is ‘A’ as it is revisiting
The first or the introductory sentence should be the context of the introductory statement and
‘D’ which provides the first information of CSK explains the reason for delay.This is followed by
lifting the IPL trophy, the sentence gives more sentence ‘B’ as it has a clue ‘intervention’ - from
information on the weather and also mentions that we may conclude that the Gujarat Titans
the details of the days.The second sentence is was the first to bat and set a target for CSK.The
‘C’ because it elaborates the cricket match that last sentence is ‘E’ which talks about the target
took place providing the opponent team and the rain break(which is a continuation of
information and it also adds the message that sentence ‘B’) and how the team still managed to
the final was delayed for a period of three lift the trophy.
days.The third sentence is ‘A’ as it is revisiting
the context of the introductory statement and 15. Answer: A
explains the reason for delay.This is followed by The correct sequence after rearrangement is
sentence ‘B’ as it has a clue ‘intervention’ - from DCABE.
that we may conclude that the Gujarat Titans The first or the introductory sentence should be
was the first to bat and set a target for CSK.The ‘D’ which provides the first information of CSK
last sentence is ‘E’ which talks about the target lifting the IPL trophy, the sentence gives more
and the rain break(which is a continuation of information on the weather and also mentions
sentence ‘B’) and how the team still managed to the details of the days.The second sentence is
lift the trophy. ‘C’ because it elaborates the cricket match that
took place providing the opponent team
14. Answer: E information and it also adds the message that
The correct sequence after rearrangement is the final was delayed for a period of three
DCABE. days.The third sentence is ‘A’ as it is revisiting
The first or the introductory sentence should be the context of the introductory statement and
‘D’ which provides the first information of CSK explains the reason for delay.This is followed by
lifting the IPL trophy, the sentence gives more sentence ‘B’ as it has a clue ‘intervention’ - from

Click Here For Bundle PDF Course | support@guidely.in Page 8 of 9


Bank Po Mains PDF Course 2024
English Day - 21

that we may conclude that the Gujarat Titans The word ‘being’ must be replaced with
was the first to bat and set a target for CSK.The ‘been’.The sentence is in passive voice whereas
last sentence is ‘E’ which talks about the target ‘being’ is used only for continuous tense.
and the rain break(which is a continuation of The correct sentence is : A large number of sim
sentence ‘B’) and how the team still managed to cards have never been used and people stopped
lift the trophy. recharging them, so, the company has decided
to call off its business as a service provider.
16. Answer: B 19. Answer: C
While must be replaced with when. Replace persist with persists.
The correct sentence is : The Indian cricket The correct sentence is : Because there is no
player was injured heavily when the bowler support by the society to eliminate the most
threw the ball straight on his face but he claimed inhuman of professions, manual scavenging still
it to be an accident. persists which is really saddening.
When - at what time
While - during the time 20. Answer: D
The error is in part ‘D’ - he started the agency
17. Answer: E two decades before is wrong, replace ‘before’
The given sentence is correct as it is.There is no with ‘ago’.
error. The correct sentence is : I have recently used
the services of his travel agency to book a cruise
18. Answer: B ship for a family trip to Atlanta, which was started
two decades ago.

Click Here For Bundle PDF Course | support@guidely.in Page 9 of 9


Bank Po Mains PDF Course 2024
Reasoning Day -22 (Eng)

Reasoning Aptitude
Directions (1-5): Study the following information b) The one who sits opposite to R
carefully and answer the c) The one who belongs to Uttar Pradesh
below questions. d) Both c and e
Eight persons – A, B, C, D, E, F, G and H are e) The one who sits second to the right of S
sitting around a square table in such a way that
four of them are sitting at the corners while four 2. Who among the following person sits at the
of them are sitting in the middle of the sides, all corner of the table?
facing towards the centre. All of them belong to I) The one who sits opposite to T
different states i.e. Himachal Pradesh, Andhra II) P
Pradesh, Uttar Pradesh, Madhya Pradesh, III) The one who sits second to the right of Q
Uttarakhand, Arunachal Pradesh, Mizoram and IV) R and V
Manipur. All the information is not necessarily in a) Only I and II
the same order. b) Only III and IV
V sits third to the left of one who belongs to c) Only I and III
Himachal Pradesh. Only one person sits d) Only II and IV
between the one who belongs to Himachal e) Only I and IV
Pradesh and the one who belongs to Arunachal
Pradesh. U, who belongs to Arunachal Pradesh 3. ____ sits immediate left of _____ who sits
is an immediate neighbor of P and sits opposite opposite to _____.
of Q. As many persons sit between U and Q as a) U, the one who sits opposite to T, R
between the one who belongs to Mizoram and S, b) U, the one who belongs to Mizoram, S
who belongs to Manipur. Either P or Q belongs to c) S, W, Q
Mizoram. The one who belongs to Uttarakhand d) W, the one who sits second to the right of Q, U
sits second to the right of T. R sits opposite to e) R, T, S
the one who sits immediate right of T. W is sitting
adjacent to one who belongs to Uttar Pradesh. 4. If all the persons interchange their positions
The one who belongs to Madhya Pradesh sits with the persons sitting opposite to them, then
opposite to the one who sits immediate right of who is an immediate neighbor of one who
W, who doesn’t sit in the middle of the table. belongs to Himachal Pradesh?
1. Who among the following person sits fourth to a) Both c and d
the right of one, who sits opposite to the one who b) U
belongs to Uttar Pradesh? c) S
a) S d) R

Click Here For Bundle PDF Course | support@guidely.in Page 1 of 11


Bank Po Mains PDF Course 2024
Reasoning Day -22 (Eng)

e) None of these then which of the number yields the resultant


which is an even number?
5. How many persons sit between the one who a) 2343545
belongs to Uttar Pradesh and one who belongs b) 4322324
to Madhya Pradesh? c) 2346548
a) One d) 6532429
b) Two e) 7656987
c) Three a) Only I and II
d) More than three b) Only III and V
e) None of these c) Only I and III
d) Only II and IV
6. If 2 is subtracted from all the odd digits and 1 e) Only I and V
is added to all the even digits of a number, then
add all the digits within the number , which of the 9. In the following words, the vowels are
following number yields composite number? changed to next letters and consonants are
a) 23435 changed to the previous letter in the alphabetical
b) 43566 order and all the letters are arranged in the
c) 32534 alphabetical order from the left (within the word)
d) 32545 what is the third letter from the left end of the
e) 45366 newly formed words respectively?
I. Dream
7. If all the digits are added within the number, II. Reality
then in which of the following resultant obtained III. Focus
will be a prime number after adding the highest a) fqp
and lowest numbers thus formed in each option? b) fjp
a) 234, 354, 325 c) jpf
b) 432, 566, 854 d) pjf
c) 332, 245, 384 e) qfp
d) 326, 735, 645
e) 234, 345, 364 10. If all the vowels are arranged first in an
alphabetical order followed by all consonants are
8. If in the given numbers, the digits at odd written in an alphabetical order, then which of the
positions from the right are dropped and the following letters remain at their original position
remaining digits are added within the numbers, in all the words respectively?

Click Here For Bundle PDF Course | support@guidely.in Page 2 of 11


Bank Po Mains PDF Course 2024
Reasoning Day -22 (Eng)

I. Patience 45K#A. The number of persons sitting to the right


II. Perseverance of one who earns 45k is one less than the
III. Dedication persons sitting to the left of one who earns 48k.
a) IN, EE, EI H@F. The one who earns 4k less than 50k sits
b) P, EE,IE to the right of one who earns 33k more than 46 k
c) I, EE, IE but not immediate right. The one who earns 79k
d) E, EE, II doesn’t sit at the extreme ends. F%D. The one
e) A, EE, EE who earns 66k sits immediate left of B. The
number of persons sitting to the right of G is one
Directions (11-15): Study the following more than the number of persons sitting to the
information carefully and answer the questions left of the one who earns twice of A.
given below. 11) How many persons are sitting between E
Eight persons A, B, C, D, E, F, G and H are and the one whose earning is
sitting in a straight line and all are facing north 96k?
but not necessarily in the same order. Each of a) More than four
them earn different amounts which are in the b) Same as between H and E
multiples of 1000s. Their earning amount is c) As many persons sit to the right of A
mentioned in the coded format as follows. d) Two
A@B- Means the earning amount of B is half of e) None
the prize amount of A.
A*B -Means the earning amount of B is 55,000 12) If all the persons’ earning are arranged in the
more than that of A. ascending order from left,
A%B- Means the difference between the earning then how many persons sit to the right of one
amounts of A and B is 20,000 where B>A who earns79k?
A#B- Means the sum of the earning amounts of a) One
A and B is 93,000 b) As many persons sit between D and C
Only four persons sit between B and E, who sits c) Four
second from the extreme ends. As many persons d) As many persons sit to the right of G
sit to the left of B as to the right of the one whose e) None of the above
earning is 37000. C sits second to the right of H,
where E*H. Neither H nor C sits at the extreme 13) G sits ____ of one whose earning is 4k less
ends of the row. C doesn’t sit adjacent to B. As than 70k.
many persons sit to the left of one whose earning a) Immediate right
is 92k as between the who earns 45k and H. b) Seventh to the right

Click Here For Bundle PDF Course | support@guidely.in Page 3 of 11


Bank Po Mains PDF Course 2024
Reasoning Day -22 (Eng)

c) Fifth to the right two floors below W. S, who is the only daughter
d) Second to the right of W lives immediately below Q. U is the wife of
e) None of the above P and lives four floors below the mother of S. U
is the grand–daughter of R, who don’t have
14) If all the persons are facing south direction, siblings. Q belongs to the same generation as
then what is the position of F that of W but unmarried person. T, who is the
with respect to the one whose earning is half of daughter-in-law of W, doesn’t have any son and
B? lives two floors above her husband. V, who is not
a) Immediate right a male member of the family, lives adjacent floor
b) Second to the left to X. U is not the daughter of W’s daughter. V is
c) Third to the left the sister-in-law of P. Y, who belongs to the
d) Immediate left same generation as T, is not the nephew of Q
e) Fourth to the left and has no kids. Only three persons live between
U’s husband and S’ husband, who lives above
15) _____ sits second to the right of one whose his wife’s sister-in-law but below V’s brother-in-
earning is _____. law.
a) E, 92K 16) Four of the following five are alike in a certain
b) C, 96K way based on the given arrangement and thus
c) A, 66K form a group. Which one of the following does
d) H, 66k not belong to the group?
e) F, 79k a) U and Y’s wife
b) V and P’s wife
Directions (16-20): Study the following c) T and The one who lives immediately above P
information carefully and answer the below d) U’s husband and the one who lives on 1st floor
questions. e) The one who lives on 10th floor and T
P, Q, R, S, T, U, V, W, X, and Y are living on
different floors of the ten storey building where 17) If T is related to S and R is related to Q in a
lowermost floor is numbered one and the floor certain way, then in the same way person ___ is
immediately above it is numbered two and so on. related to _____ respectively.
No two persons live on the same floor. They are a) U and the one who lives immediately above
related to each other through some blood U’s husband
relations. b) T and the one who lives on 2nd floor
Only five floors are between W and R who is the c) P and W
husband of W. Q, who is the male member, lives d) T’s husband and W’s husband

Click Here For Bundle PDF Course | support@guidely.in Page 4 of 11


Bank Po Mains PDF Course 2024
Reasoning Day -22 (Eng)

e) None of the above a) T’s son-in-law


b) X’ brother-in-law
18) The number of floors are between the Q and c) None
R’s wife is two less than the d) T’s sister-in-law
number of persons live below ____ . e) The one who lives on 8th floor
a) The one who lives immediately below T
b) Q 20) Find the next pair.
c) The one who lives on the 4th floor WU PS’ husband W’ sibling R __
d) W a) TV
e) None of the above b) S X’s wife
c) U U’s husband
19) If all persons are made to sit in alphabetical d) V the one who lives on 5th floor
order from top to bottom, then who among the e) QX
following remain unchanged in their position?
Click Here to Get the Detailed Video Solution for the above given Questions
Or Scan the QR Code to Get the Detailed Video Solutions

Answer Key with Explanation


Directions (1-5):
1) Answer: D
2) Answer: C
3) Answer: B
4) Answer: A
5) Answer: C
Final arrangement is:
We have,

Click Here For Bundle PDF Course | support@guidely.in Page 5 of 11


Bank Po Mains PDF Course 2024
Reasoning Day -22 (Eng)

 V sits third to the left of one who belongs to  The one who belongs to Uttarakhand sits
Himachal Pradesh. second to the right of T.
 Only one person sits between the one who  R sits opposite to the one who sits immediate
belongs to Himachal Pradesh and the one who right of T.
belongs to Arunachal Pradesh.
From above conditions, we have four
possibilities.

Again we have
 W is sitting adjacent to one who belongs to
Uttar Pradesh.
 The one who belongs to Madhya Pradesh sits

Again we have opposite to the one who sits immediate right of

 U, who belongs to Arunachal Pradesh is an W, who doesn’t sit in the middle of the table.

immediate neighbor of P and sits opposite of Q.


 As many persons sit between U and Q as
between the one who belongs to Mizoram and S,
who belongs to Manipur.
 Either P or Q belongs to Mizoram.

6. Answer: D
a) 23435
31513 = 3+1+5+1+3 = 13
b) 43566
51377 = 5+1+3+7+7 = 23
c) 32534
13315 = 1+3+3+1+5 = 13
d) 32545
Again we have 13353 = 1+3+3+5+3 = 15

Click Here For Bundle PDF Course | support@guidely.in Page 6 of 11


Bank Po Mains PDF Course 2024
Reasoning Day -22 (Eng)

e) 45366 I. Dream – cqfbl


53177 = 5+3+1+7+7 = 23 bcflq
II. Reality – qfbkjsx
7. Answer: C Bfjkqsx
a) 234, 354, 325 III. Focus – epbvr
9, 12, 10 Beprv
9+12 = 21
b) 432, 566, 854 10. Answer: A
9, 17, 17 I. Patience =aeeicnpt
9+17 = 26 II. Perseverance = aeeeecnprrsv
c) 332, 245, 384 III. Dedication = aeiiocddnt
8, 11, 15
8+15 = 23 Directions (11-15):
d) 326, 735, 645 11) Answer: C
11, 15, 15 12) Answer: D
11+15 = 26 13) Answer: C
e) 234, 345, 364 14) Answer: E
9, 12, 13 15) Answer: D
9+13 =22 Final arrangement is:

8. Answer: E
a) 2343545
334= sum is 10 We have
b) 4322324  Only four persons sit between B and E, who
223= sum is 7 sits second from the extreme ends.
c) 2346548  As many persons sit to the left of B as to the
463= sum is 13 right of the one whose earning is 37000.
d) 6532429  C sits second to the right of H, where E*H.
225= sum is 9  Neither H nor C sits at the extreme ends of the
e) 7656987 row.
866= sum is 20  C doesn’t sit adjacent to B.
From above conditions, we have three
9. Answer: B possibilities.

Click Here For Bundle PDF Course | support@guidely.in Page 7 of 11


Bank Po Mains PDF Course 2024
Reasoning Day -22 (Eng)

Again, we have
 As many persons sit to the left of one whose Again, we have
earning is 92k as between the who earns 45k  F%D- Means the difference between the prize
and H. amounts of F and D is 20,000 where D>F. As
 45K#A i.e. A= 48k. F=46k therefore D=66k.
 The number of persons sitting to the right of  The one who earns 66k sits immediate left of
one who earns 45k is one less than the persons B.
sitting to the left of one who earns 48k.

Again, we have
Again, we have
 The number of persons sitting to the right of G
 H@F- Means the earning amount of F is half of
is one more than the number of persons sitting
the earning amount of H i.e. 46k
to the left of the one who earns twice of A.
 The one who earns 4k less than 50k sits to the
After applying the above conditions case-2a(1)
right of one who earns 33k more than 46 k but
gets eliminated because there is no place left for
not immediate right.
G and the one who earns twice of A as per the
 The one who earns 79k doesn’t sit at the
given data. Hence, case-2a shows the final
extreme ends.
arrangement.

Click Here For Bundle PDF Course | support@guidely.in Page 8 of 11


Bank Po Mains PDF Course 2024
Reasoning Day -22 (Eng)

 Only five floors are between W and R who is


the husband of W.
 Q, who is the male member, lives two floors
below W.
 S, who is the only daughter of W lives
immediately below Q.
From above conditions, we have five
Directions (16-20): possibilities.
16) Answer: D
17) Answer: A
18) Answer: C
19) Answer: D
20) Answer: B
Final arrangement:

Again we have
 U is the wife of P and lives four floors below
the mother of S.
 U is the grand–daughter of R, who don’t have
siblings.
 Q belongs to the same generation as that of W
but unmarried person.

We have

Click Here For Bundle PDF Course | support@guidely.in Page 9 of 11


Bank Po Mains PDF Course 2024
Reasoning Day -22 (Eng)

 Y, who belongs to the same generation as T, is


not the nephew of Q and has no kids.

Again we have
 T, who is the daughter-in-law of W, doesn’t
have any son and lives two floors above her
husband.
 V, who is not a male member of the family,
lives adjacent floor to X.

Again we have
 Only three persons live between U’s
husband and S’ husband, who lives
above his wife’s sister-in-law but below
V’s brother-in-law.
After applying the above conditions, case-3
and case-2a get eliminated because there is
no place left for P and Y with three persons
gap. Also, case-5 gets eliminated because Y
lives below T. Hence, case-2 shows the final
arrangement.

Again we have
 U is not the daughter of W’s daughter.
 V is the sister-in-law of P.

Click Here For Bundle PDF Course | support@guidely.in Page 10 of 11


Bank Po Mains PDF Course 2024
Reasoning Day -22 (Eng)

Click Here For Bundle PDF Course | support@guidely.in Page 11 of 11


Bank Po Mains PDF Course 2024
Quantitative Aptitude Day -22 (Eng)

Quantitative Aptitude

Directions (1-4): Study the following information carefully and answer the questions given below.
There are five colleges [A, B, C, D and E]. There are three streams [science, Arts and commerce] in each
college. The given below table shows total number of students in the college, Percentage of students in
science stream, Number of students in Arts stream, Ratio of boys and girls in commerce stream, Number
of boys in science stream and Number of girls in Arts stream.

1) Number of girls in the science stream of A.66


college A is half of the number of students in the B.70
Arts stream of the same college. Number of boys C.75
in Arts stream of college A is 1/4th of the number D.80
of boys of commerce stream of the same E.50
college. Find the number of girls in the Arts
stream of college A. 3) Number of boys in science stream of college
A.80 C is 20% less than the number of girls in Arts
B.85 stream of college B. Number of girls in
C.70 commerce stream of college C is 25% of the
D.60 number of boys in science stream of college C.
E.65 Then find the percentage of girls in commerce
stream of college C out of total number of
2) Ratio of the number of students of Arts stream students in commerce stream of college C?
to commerce streams of college B is 15:13. A.25.32%
Number of girls in commerce stream of college B B.16.66%
is 78. If 33.33% of the total number of students of C.20%
science stream of college B is boys, then find the D.15%
number of girls in science stream of college B. E. None of these

Click Here For Bundle PDF Course | support@guidely.in Page 1 of 12


Bank Po Mains PDF Course 2024
Quantitative Aptitude Day -22 (Eng)

4) Number of boys in commerce stream of A. The data in statement I alone are sufficient to
college D is 5 more than the number of boys in answer the question, while the data in statement
science stream. Number of Arts students in II alone are not sufficient to answer the question.
college D is 20% less than the total number of B. The data in statement II alone are sufficient to
commerce stream students in college D. Then answer the question, while the data in statement
find the number of students in the science I alone are not sufficient to answer the question.
stream of college D,if 60% of the total students C. The data either in statement I alone or in
are from Arts and commerce streams. statement II alone are sufficient to answer the
A.240 question.
B.252 D. The data given in both statements I and II
C.236 together are not sufficient to answer the
D.221 question.
E. None of these E. The data in both statements I and II together
are necessary to answer the question.
Directions (5-8): Following questions contain two
statements the statement I and statement II. You 6) There are three different mixtures of milk and
have to determine which statement/s is/are water P, Q and R. Find the ratio in which all three
necessary to answer the question and give an mixtures must be mixed together so that average
answer as, price of the resultant mixture is Rs. 62 per litres.
5) A certain number of students gave at least Statement I: Price of the mixture P and Q is Rs.
one of the two exams i.e., Physics and 40 per litres and Rs. 75 per litres, respectively.
Chemistry. Find total number of students who Statement II: Price of 15 litres of mixture R is Rs.
gave the exam if all the students who gave the 900.
exam passed it. A. The data in statement I alone are sufficient to
Statement I: 73(1/3)% of the students passed in answer the question, while the data in statement
Chemistry and 59(2/3)% of students passed in II alone are not sufficient to answer the question.
Physics and number of students who passed in B. The data in statement II alone are sufficient to
both the exam is 396. answer the question, while the data in statement
Statement II: Number of students who passed I alone are not sufficient to answer the question.
only in Physics and only in Chemistry are 76 less C. The data either in statement I alone or in
and 88 more, respectively, than the number of statement II alone are sufficient to answer the
students who passed in both the exams. Total question.
number of students who passed in Physics is
716.

Click Here For Bundle PDF Course | support@guidely.in Page 2 of 12


Bank Po Mains PDF Course 2024
Quantitative Aptitude Day -22 (Eng)

D. The data given in both statements I and II Statement II: B and C together complete work in
together are not sufficient to answer the 20/3 days. A alone complete the work in 10 days.
question. D is 25% more efficient than A.
E. The data in both statements I and II together A. The data in statement I alone is sufficient to
are necessary to answer the question. answer the question, while the data in statement
II alone is not sufficient to answer the question
7) What is the difference between the length of B. The data in statement II alone is sufficient to
the platform and the train itself? answer the question, while the data in statement
Statement I: The train running at a speed of 54 I alone is not sufficient to answer the question
kmph can cross a bus going in the same C. The data either in statement I alone or in
direction at a speed of 36 kmph in 20 seconds. statement II alone is sufficient to answer the
Statement II: Time taken by the bus to cross the question
platform is 30 seconds more than the time taken D. The data given in both statements I and II
by the train to do the same. together are not sufficient to answer the question
A. The data in statement I alone are sufficient to E. The data given in both statements I and II
answer the question, while the data in statement together are necessary to answer the question.
II alone are not sufficient to answer the question.
B. The data in statement II alone are sufficient to Directions (9-12): Study the following information
answer the question, while the data in statement carefully and answers the questions based on it.
I alone are not sufficient to answer the question. In a Car Expo in the New Delhi, there are a total
C. The data either in statement I alone or in of 67,200 visitors who visited the car expo. The
statement II alone are sufficient to answer the ratio of number of males to number of females
question. who visited in the car expo is 9: 7 respectively.
D. The data given in both statements I and II All the visitors who visited the car expo, visits the
together are not sufficient to answer the six different car company pavilion namely Ford,
question. Maruti, Volkswagen, Toyota, Honda and Nissan.
E. The data in both statements I and II together Total number of visitors who visited Ford pavilion
are necessary to answer the question. is 16,128 which is (__P%__) of the total number
of visitors. Number of female visitors who visited
8) Find the number of days taken by A and C Maruti pavilion is (__Q%__) of the total number of
together to complete the work? female visitors who visited car expo and 10,983
Statement I: Efficiency of A is 20% more than the less than the total number of visitors who visited
efficiency of B. D completes the work in 8 days. Ford pavilion.
Ratio of efficiency of D and C is 15:8.

Click Here For Bundle PDF Course | support@guidely.in Page 3 of 12


Bank Po Mains PDF Course 2024
Quantitative Aptitude Day -22 (Eng)

Number of male visitors who visited Honda C.49


pavilion is 15%of the total male visitors. Number D.18
of male visitors who visited Volkswagen pavilion E. None of these
is 1890 more than male visitors who visited
Honda pavilion and is(__R%__)of the total 10) Find the value of Q?
number of male visitors who visited car expo. A.17.5
Number of male visitors who visited Maruti B.10.25
pavilion is 1/3rd of the number of female visitors C.21
who visited Maruti pavilion. D.33.33
Number of female visitors who visited Toyota E. None of these
pavilion is 7³ more than the number of male
visitors who visited Maruti pavilion and is 11) Find the value of S and T respectively?
(__S%__) of the female visitors who visited A.17 and 20
Maruti pavilion. Number of female visitors who B.51 and 27
visited Nissan pavilion is 4410 which is (__T%__) C.32 and 22
of the total number of female visitors who visited D.40 and 15
car expo. E. None of these
Number of male visitors who visited Toyota
pavilion is 3 times of the number of female 12) Find the value of [(Q × S × T) – (P × R ×
visitors who visited same pavilion. Total number T)]/100?
of visitors who visited Honda pavilion is 22% of A.23
total visitors. The ratio of male to female visitors B.17 (5/7)
who visited Nissan pavilion is 9: 5 respectively. C.33
9) Find the value of P? D.16 (2/3)
A.37 E. None of these
B.24

Directions (13-16): Study the table given below carefully and answer the following questions.
The given pie chart shows the percentage distribution of students who paid fees for semester exams in 5
different years. The given table shows the percentage of students who paid the fees by online mode in
five different years.
Note: Fees are paid through online and offline modes.
Total number of students who paid fees = 6,00,000.

Click Here For Bundle PDF Course | support@guidely.in Page 4 of 12


Bank Po Mains PDF Course 2024
Quantitative Aptitude Day -22 (Eng)

13) Number of students who paid their fees 14) Number of students who paid fees through
through offline in 2016 and 2017 together is how online in 2015 is equal to the average of students
much more/less than the number of students who paid through online in 2019 and 2020
who paid fees through online in the years 2018 together and the number of students who paid
and 2019 together? fees through online is 3⁄4th of the total number of
A.54750 students in 2015. Then find the total students in
B.57450 2015.
C.54450 A.43000
D.52389 B.44000
E.54455 C.40500
D.45500
E.44500

Click Here For Bundle PDF Course | support@guidely.in Page 5 of 12


Bank Po Mains PDF Course 2024
Quantitative Aptitude Day -22 (Eng)

15) Students who paid fees through offline in 16) Find the average number of students who
2020 is what percent of the students who paid paid their fees through online in the years 2016,
fees through online in 2018? 2017 and 2019 together?
A.140% A.36000
B.145% B.34560
C.150% C.36850
D.160% D.36750
E.135% E.37750

Directions (17-20): Study the following information carefully and answer the questions given below.
There are five cities. In each, there are three types of cars are registered, i.e., Petrol, Diesel, and Electric.
Sum of the number of petrol and diesel cars and difference of the number of petrol and diesel cars
registered in five cities are given in the below graph.

Note: -
a) Number of petrol cars registered in cities A, C, and E is more than the number of diesel cars registered
in the same cities, in other cities the number of diesel cars is more than the number of petrol cars.
b) Percentage of electric cars out of the total cars registered in cities A, B, C, D, and E is 20%, 20%,
30%, 25%, and 20% respectively.
17) Find the difference between the 20% of A.50
petrol cars registered in cities D and E together B.55
and 10% of electric cars registered in cities A C.75
and C together. D.45

Click Here For Bundle PDF Course | support@guidely.in Page 6 of 12


Bank Po Mains PDF Course 2024
Quantitative Aptitude Day -22 (Eng)

E.26 number of Tata company’s registered cars in city


A?
18) If the number of registered diesel and electric A.94.94%
cars of all cities is represented individually in a B.84.84%
pie chart then find the difference of central angle C.74.74%
formed by the number of diesel car register in C D.64.64%
and central angle formed by the number of E. None of these
electric car register in city B?
A.20̊ 20) Find which one is true.
B.85̊ I) 75% of the number of diesel cars registered in
C.65̊ city A is equal to the number of electric cars
D.15̊ registered in city B.
E.45̊ II) 20% of the total number of electric cars
registered in city D and city E together is equal to
19) Number of all registered petrol and diesel 20% less than the total number of electric cars
cars are from only three companies, i.e., Maruti, registered in city E.
Hyundai, and Tata in city A. No electric cars in A. Only I is true
city A are from these companies. Ratio of petrol B. Only II is true
cars and diesel cars from companies Maruti, C. Both are true
Hyundai and Tata are 4:3:5 and 2:1:2 in city A. D. None is True
Find the total number of Maruti company's E. Can’t be determined
registered cars is what percent of the total
Click Here to Get the Detailed Video Solution for the above given Questions
Or Scan the QR Code to Get the Detailed Video Solutions

Answer Key with Explanation


1) Answer: C

Click Here For Bundle PDF Course | support@guidely.in Page 7 of 12


Bank Po Mains PDF Course 2024
Quantitative Aptitude Day -22 (Eng)

Total number of students in science stream of Number of boys in commerce stream of college
college A =150+(100/2)=200 D =120+5=125
Total number of students in commerce stream of Total number of students in commerce
college A = 500-100-200=200 department is =125*8/5=200
Number of boys in commerce stream of college Total number of student in science stream is =
A =200*3/5=120 {[200+160]/60} *40=240
Number of boys in Arts stream of college A
=120/4=30 5) Answer: C
Number of girls in Arts stream of college A =100- Statement I,
30=70 Let total number of students who gave exam be
2) Answer: D 100x
Let the number of Arts and commerce students Number of students who passed only in Physics
of college B be 150x and 130x. = 26(2/3)% of 100x = 80x/3
So, 130x*3/5=78 Number of students who passed only in
Or, 78x=78, x=1 chemistry = 40(1/3)% of 100x = 121x/3
The number of Arts and commerce students of So according to question,
college B is 150 and 130. 80x/3 + 121x/3 + 396 = 100x
So, number of student in science stream is = 300x – 80x + 121x = 2412
[280/70] *30= 120 99x = 1188
Number of boys in science stream is = 120* (1/3) x = 12
=40 Required number of students = 12 × 100 = 1200
So, number of girls in science stream of college Data in statement I alone is sufficient to answer
B is = 120-40=80 the question.
Statement II,
3) Answer: E Let number of students who passed in both
Number of boys in science stream of college C is exams be x
=100*80/100=80 Number of students who passed only in Physics
Number of girls in commerce stream of college C = x – 76
is =80*25/100=20 Number of students who passed only in
Total number of students in commerce stream is Chemistry = x + 88
= 300-150-100=50 x + x – 76 = 716
So, required percentage = [20/50]*100=40% 2x = 792
4) Answer: A x = 396

Click Here For Bundle PDF Course | support@guidely.in Page 8 of 12


Bank Po Mains PDF Course 2024
Quantitative Aptitude Day -22 (Eng)

Required number of students = x – 76 + x + 88 + If L is the length of the train, then,


x = 3x + 12 = 3 × 396 + 12 = 1200 L = 5 × 20 = 100 meter.
Data in statement II alone is sufficient to answer Data in statement I alone is not sufficient to
the question. answer the question.
Therefore, the data either in statement I alone or Statement II,
in statement II alone are sufficient to answer the Let the length of the platform be xmeter.
question. Time taken by the bus to cross the platform =
{x/(36 × 5/18)} = x/10 sec
6) Answer: D Time taken by the train to cross the platform =
Statement I, {(L + x)/(54 × 5/18)} = (L + x)/18 sec
Price of the mixture P and Q is Rs. 40 per litres Now,
and Rs. 75 per litres, respectively x/10 – (L + x)/15 = 30
Data in statement I alone is not sufficient to Data in statement II alone is not sufficient to
answer the question. answer the question.
Statement II: Combining statement, I and II,
Price of one liter of mixture R = 900/15 = Rs. 60 L = 100
Data in statement II alone is not sufficient to x/10 - (L + x)/15 = 30
answer the question. x/10 – (100 + x)/15 = 30
Combining statement, I and II we get (3x – 200 – 2x)/30 = 30
Let ratio in which mixture P, Q and R are mixed x – 200 = 900
is a:b:c x = 1100meter
62 = {(40 × a + 75 × b + 60 × c)/(a + b + c)} Required difference = 1100 – 100 = 1000 meter
62a + 62b + 62c = 40a + 75b + 60c So, data in statement I and II together are
22a + 2c = 13b sufficient to answer the question.
Since, we cannot find the value of a, b and c.
So, the data given in both statements I and II 8) Answer: E
together are not sufficient to answer the From statement I,
question. C completes the work in 8*15/8= 15 days.
But we cannot calculate the number of days
7) Answer: E taken by A to complete the work.
Statement I, From statement II,
Relative speed between the train and the bus =
(54 – 36) = 18 kmph = 18 × 5/18 = 5 m/s

Click Here For Bundle PDF Course | support@guidely.in Page 9 of 12


Bank Po Mains PDF Course 2024
Quantitative Aptitude Day -22 (Eng)

A alone complete the work in 10 days. But we Number of male visitors who visited Volkswagen
cannot calculate the number of days taken by C pavilion = 5670 + 1890 = 7560
to complete the work. So, the percentage of male visitors who visited
By combining I and II, Volkswagen pavilion is,
A alone can complete the work in 10 days = 7560/37,800 × 100 =>20 (Value of R)
C alone can complete the work in 15 days Number of male visitors who visited Maruti
= 1/10 + 1/15 = 6 days pavilion = 5145 × 1/3 = 1715
From both statements we can get the number of Number of female visitors who visited Toyota
days taken by A and C together to complete pavilion is = 1715 + 7³ = 1715 + 343 => 2058
work. So, the percentage of female visitors who visited
Hence, the data given in both statements I and II Toyota pavilion is,
together are necessary to answer the question. = 2058/5145 × 100 =>40 (Value of S)
Number of female visitors who visited Nissan
Directions (9-12): pavilion = 4410
Total number of visitors who visited the car expo So, the percentage of female visitors who visited
is = 67,200 Nissan pavilion is,
Total number of male visitors who visited car = 4410/29,400 × 100 =>15 (Value of T)
expo = 67,200 × 9/16 => 37,800 Number of male visitors who visited Toyota
Total number of female visitors who visited car pavilion = 3 × 2058 =>6174
expo = 67,200 × 7/16 => 29,400 Total number of visitors who visited Honda
Total number of visitors who visited Ford pavilion pavilion = 67,200 × 22/100 => 14,784
= 16,128 Number of male visitors who visited Nissan
So, the percentage of visitors who visited Ford pavilion = 4410 × 9/5 =>7938
Pavilion is, Number of female visitors who visited Honda
= 16,128/67,200 × 100 =>24 (Value of P) pavilion = 14,784 – 5670 => 9114
Number of female visitors who visited Maruti Number of male visitors who visited Ford
pavilion = 16,128 – 10,983 => 5145 pavilion is,
So, the percentage of female visitors who visited = 37,800 – (5670 + 1715 + 7560 + 7938 + 6174)
Maruti pavilion is, = 37,800 – 29,057 =>8743
= 5145/29,400 × 100 =>17.5 (Value of Q) Number of female visitors who visited Ford
Number of male visitors who visited Honda pavilion = 16,128 – 8743 => 7385
pavilion = 37,800 × 15/100 = 5670 Number of female visitors who visited
Volkswagen pavilion is,

Click Here For Bundle PDF Course | support@guidely.in Page 10 of 12


Bank Po Mains PDF Course 2024
Quantitative Aptitude Day -22 (Eng)

= 29,400 – (9114 + 5145 + 4410 + 7385 + 2058) = 168000


= 29,400 – 28,112 => 1288 Number of students who paid fees through
For question 11: online in 2018 & 2019 together = 600000 *
9) Answer: B (20/100 * 62.5/100 + 15/100 * 42.5/100)
From the above common explanation it is clear = 60 * (1250+637.50)
that, = 113250
The value of P is = 24 Required difference = 168000-113250=54750
Hence, the required answer is = 24.
14) Answer: D
10) Answer: A Number of students who paid through online in
From the above common explanation it is clear 2015 = 1/2 * 600000 * (15/100 * 42.5/100 +
that, 25/100 * 20/100)
The value of Q is = 17.5 = 30 * (637.5 + 500)
Hence, the required answer is = 17.5 = 34125
Therefore Total number of students in 2015 =
11) Answer: D 34125 * 4/3 = 45500
From the above common explanation it is clear
that, 15) Answer: D
The value of S and T is = 40 and 15 Required percentage = (600000 * 25/100 *
Hence, the required answer is = 40 and 15 80/100)/(600000 * 20/100 * 62.5/100) * 100 =
160%
12) Answer: C
Value of [(Q × S × T) – (P × R × T)]/100 16) Answer: D
= [(17.5 × 40 × 15) – (24 × 20 × 15)]/100 Required average = 1/3 * 600000 (16/100 *
= (10,500 – 7200)/100 37.5/100 + 24/100 * 25/100 + 15/100 * 42.5/100)
= 3300/100 => 33 = 36750

13) Answer: A Directions (17-20):


Number of students who paid fees through The total number of petrol cars and diesel cars
offline in 2016 & 2017 together registered in city A is 400 and the difference
= 600000 * (16/100 * 62.5/100 + 24/100 * between petrol and diesel cars registered in city
75/100) A is 200.
= 60 * (1000+1800)

Click Here For Bundle PDF Course | support@guidely.in Page 11 of 12


Bank Po Mains PDF Course 2024
Quantitative Aptitude Day -22 (Eng)

We know the number of petrol cars register is Central angle of diesel cars in C = [360/900]
more than number of diesel cars registered in *150 = 60̊
city A. so we can say the number of petrol cars Total number of electric cars registered = 100 +
register = [400 + 200]/2= 300 75 + 150 + 150 + 125 = 600
Number of diesel cars registered = [400 - 200]/2 Central angle of electric cars in B = [360/600] *
= 100 75 = 45̊
Percentage of electric cars registered in city A is So, the required difference = 15̊
20%, so the number of electric cars registered in
city A = [400/80] * 20 = 100 19) Answer: B
Similarly, we can also calculate the value of For City A:
other cities. Total number of Maruti cars registered in the city
= [300 * 4/12] + [100 * 2/5] = 100 + 40 = 140
Total number of Tata cars registered in the city =
[300 * 5/12] + [100 * 2/5] = 125 + 40 = 165
Required percentage = [140/165] *100 = 84.84%

20) Answer: A
75% of the number of diesel cars registered in
city A = 100 * 75/100 = 75
17) Answer: C
Number of electric cars registered in city B = 75
Required difference = [(150 + 350) * 20/100] –
So, this statement is true.
[(100 + 150) * 10/100] = 75
20% of the total number of electric cars
registered in city D and E together = [150 + 125]
18) Answer: D
* 20/100 = 55
Total number of diesel cars registered = 100 +
20% less than the total number of electric cars
200 + 150 + 300 + 150 = 900
registered in city E = 125 * 80/100 = 100
So, this statement is false.

Click Here For Bundle PDF Course | support@guidely.in Page 12 of 12


Bank Po Mains PDF Course 2024
English Day - 22

English Language
Directions (1-5) : In each of the following d) deployed, drives
questions given below, there is a sentence in e) No changes required
which two words are highlighted. They may or
may not be grammatically and contextually 4) When burnt, CNG produces fewer greenhouse
correct. Choose the best alternatives among the gas emissions, particulate matter, and other
four given options to replace those words. If no pollutants that addition to air pollution and
replacement is required choose option (e) as climate change.
your answer. a) combusted, contributed
1) The Article had gave special rights and b) fired, add
advantages to the people of Jammu and Kashmir c) blazed, governed
since 1954 in accordance with the Instrument of d) ignited, increased
Accession. e) No changes required
a) give, opportunities
b) accorded, privileges 5) Some drones also have automatic capabilities
c) grant, freedom that allow them to perform pre-programmed
d) awarded, rights tasks, follow flight paths, or use advanced
e) No changes required algorithms to navigation and make decisions
based on sensor inputs.
2) Many private banks have an international a) different, decide
presence, allowing clients to assess services and b) advancing , fly
investment opportunities in many jurisdictions. c) free, sail
a) access, multiple d) autonomous, navigate
b) availing, few e) No changes required
c) use, numerous
d) enable, most Directions (6-10): Seven sentences are given
e) No changes required below out of which one sentence is either
irrelevant or inappropriate (odd one out).
3) Blood pressure is the force exerted by the Rearrange the remaining six sentences in a
blood against the walls of the arteries as the proper sequence so as to form a coherent
heart pumps it throughout the body. paragraph and then answer the questions given
a) released, pushes below.
b) on, pulls
c) applied, sends

Click Here For Bundle PDF Course | support@guidely.in Page 1 of 12


Bank Po Mains PDF Course 2024
English Day - 22

(A) Tightening compliance and the post- a) FG


pandemic rebound in economic activity have b) FA
helped improve revenues from the GST. c) GF
(B) Marking the occasion, Finance Minister d) CA
Nirmala Sitharaman emphasised that the GST e) EB
has moved the country towards a unified market
from a situation where each State mandated 7) Fill in the blanks based on the sequence of
different indirect tax structures and procedures. rearrangement. The following represents the first
(C) GST Compensation cess levies have been five sentences.
extended till at least March 2026, instead of the F_D_ _
initial five-year tenure, due to the transitory shock a) AEB
of COVID-19 lockdowns on revenues. b) EBG
(D) Introduced soon after the demonetisation c) BEG
shock/A, the GST was viewed as another d) EAG
disruptor/B for the informal economy and its e) ABG
initial technical, structural/C and procedural
challenges took a while to sorting out/D. No 8) Which is the odd one out sentence among the
error/E given options ?
(E) starting this August, and that there has a) A
been(P) now that all businesses with annual b) B
turnover of ₹ 5 crore(Q) indicates that firms have c) C
gradually embraced the change(R) no ostensible d) E
pushback from smaller businesses over this,(S) e) G
will have to generate e-invoices(T)
(F) India’s tryst with the Goods and Services Tax 9) Choose the erroneous part in sentence D as
(GST), launched at a special midnight Parliament your answer.
session with unusual fanfare, completed six a) A
years this month. b) B
(G) The Revenue Department’s crackdown on c) C
fake invoicing and other techniques deployed by d) D
tax evaders may compel the few outliers to fall in e) No error
line too.
6) Which of the following options represents the 10) Sentence E is divided into five parts namely
First and the last sentence after rearrangement ? P, Q, R, S and T which may or may not be in

Click Here For Bundle PDF Course | support@guidely.in Page 2 of 12


Bank Po Mains PDF Course 2024
English Day - 22

correct order, rearrange them if necessary to the ideal population size of a family, a
form a proper meaningful sentence. Choose the community, a country and even the world ought
correct sequence of rearrangement as your to be. It is important to understand that there are
answer. no ideal numbers or figures.
a) PQRST Population stability comes when reproductive
b) RPQST and sexual health decisions are free of
c) TRPQS discrimination, coercion(B) and violence, that
d) STRPQ reproductive and sexual health services are
e) QTPSR affordable, acceptable, accessible and of high
quality, and that women and couples are
Directions (11-17): Read the given passage and supported to have the number of children they
answer the following questions based on the want, when they want them. On World
passage. Some words are highlighted to help Population Day (July 11), India deserves to be
you locate while answering the questions. commended for its family planning initiatives,
The theme of this year’s World Population Day, where despite the many challenges, the aim is to
i.e., ‘Unleashing the power of gender equality: provide an increasingly comprehensive package
Uplifting the voices of women and girls to unlock of reproductive health services to every potential
our world’s infinite possibilities’, could not be beneficiary — with a focus on the provision of
more apt for India. When we unlock the full modern short and long-acting reversible
potential of women and girls, encouraging and contraceptives, permanent methods, information,
nurturing their desires for their families and counselling, and services, including emergency
themselves, we galvanise(A) half the leadership, contraception.India’s commitment towards the
ideas, innovation, and creativity available to Family Planning 2030 partnership includes
societies. In India, the world’s most populous expanding its contraceptive basket. The inclusion
nation, the template for women-led development, of new contraceptive options advances women’s
be it in science, technology, agriculture, rights and autonomy, leading to a spike in
education or health care, must also include modern contraceptive prevalence. Access to
reproductive autonomy at its core.For far too long timely, quality and affordable family planning
the world has been obsessed with population services is crucial because unspaced
numbers and targets. Instead of ensuring pregnancies may have a detrimental influence on
reproductive autonomy for each woman, we are the new-born’s health as well as major effects on
obsessed with total fertility rates; instead of maternal mortality, morbidity, and health-care
ensuring that family planning services reach all expenditure. The Indian government’s health,
those who want it, we are obsessed with what population and development programmes have

Click Here For Bundle PDF Course | support@guidely.in Page 3 of 12


Bank Po Mains PDF Course 2024
English Day - 22

shown steady progress over the years. Life importantly, the focus on gender equality helps
expectancy at birth has significantly increased in shift the focus away from the notion of
the country over the years. Compared to the ‘population stabilisation’ to ‘population dynamics’
1990s, Indians are currently living a decade based on reproductive choices people make.
longer. In terms of maternal health, India has India has a significant opportunity to advance
made impressive strides. The current rate of gender equality and grow its economy. In fact,
maternal mortality is 97 (per 100,000 live births in raising the women’s labour force participation by
a year), down from 254 in 2004. Another triumph 10 percentage points might account for more
of these programmes is gender empowerment. than 70% of the potential GDP growth
Since the beginning of 2000, India has cut the opportunity ($770 billion in additional GDP by
number of child marriages by half. Teen 2025). The path to such a bright future is clear.
pregnancies, too, have dramatically decreased. Focusing on gender equality-centred growth,
Access to vital services, including health, rights, and choices promises to help all achieve
education, and nutrition, has also improved. their aspirations. Gender equality can be ensured
However, this progress has a fine print too. Many by making investments in a woman’s life at every
women continue to lack physical autonomy. stage, from childbirth to adolescence to maturity.
According to the most recent National Family Engaging with women, girls and other
Health Survey (NFHS-5), just 10% of women in marginalised people and formulating legislation
India are independently able to make decisions and policies that empower them to asset their
about their own health, and 11% of women rights and to taking life changing personal
believe that marital violence is acceptable if a decisions(X) are the first steps in this
woman refuses to have sex with her husband. direction.World Population Day this year is an
Nearly half of all pregnancies in India are opportunity to reaffirm our commitment to putting
unplanned, as they are globally. Advancing individual rights, particularly women’s rights and
gender equality is not just about women but also well-being, at the centre of the population and
about populations as a whole. In ageing societies development discourse(C). Gender-just
that worry about labour productivity, achieving approaches and solutions are the fundamental
gender parity in the workforce is the most building blocks of a more prosperous India, and
effective way to improve output and income indeed the world.
growth. And in countries experiencing rapid 11) According to the given passage, which of the
population growth, women’s empowerment following contributes to the growth of the
through education and family planning can bring economy ?
enormous benefits by way of human capital and
inclusive economic development. More

Click Here For Bundle PDF Course | support@guidely.in Page 4 of 12


Bank Po Mains PDF Course 2024
English Day - 22

I. Empowering women through education and 13) Which of the following best conveys the
creating awareness to them may help in meaning of the sentence “Life expectancy at
achieving good economic growth. birth has significantly increased in the country
II. By achieving equal contribution of women and over the years” in the passage ?
men in the workforce which is considered as the a) Expected life period of the infant during birth
most effective way to improve the economy. has increased over the years.
III. Focusing more on gender equality and equal b) Infant mortality rate has increased over the
rights may also contribute to the growth of the years.
nation in terms of economy. c) Expected life period of the mother during birth
a) Only I has increased over the years.
b) Only II d) Both a and c
c) Both I and III e) None of these
d) Both II and III
e) All I, II and III 14) Which of the following questions cannot be
answered after reading the given passage ?
12) Which of the following given statements a) What is the main motive or agenda behind
is/are true and correct as per the context of the celebrating world population day every year ?
passage ? b) What should India as a nation must focus on
I. Every year India celebrates Population day on other than population numbers and targets ?
July 11th and this year the theme is Unleashing c) In what way men are superior to women when
the power of gender equality. it comes to work ?
II. Most of the women in India find it difficult to d) What are the steps to achieve inclusive
make independent decisions because they are economic development ?
dependent on others and accept everything e) None of these
including violence against them.
III. For every 100,000 childbirths 97 women die 15) Which of the following best replaces the
but this is an improved figure as India has highlighted phrase (X) in the above passage to
focused on population, health and development make the sentence grammatically and
programmes for the past years. contextually correct and meaningful ?
a) Only I a) take life changing decisions on the assets and
b) Only II rights
c) Only III b) assert there rights and take personal changes
d) Both I and III that will change life
e) Both II and III

Click Here For Bundle PDF Course | support@guidely.in Page 5 of 12


Bank Po Mains PDF Course 2024
English Day - 22

c) asset their right and takes life changing COLUMN 1 COLUMN 2


personnel decisions (A) galvanise (D) dialogue
d) assert their rights and take life changing
personal decisions (B) coercion (E) energise

e) to take their assets through rights and making


(C) discourse (F) oppression
life changing decisions
a) A-E, B-D, C-F
b) A-D, B-E, C-F
16) Three statements are given below, you are
c) A-E, B-F, C-D
required to mark the statements as TRUE OR
d) A-F, B-D, C-E
FALSE in accordance with the information given
e) None of the above
in the passage.
I. Child marriages In India have been reduced by
Directions (18-22) : Read each question to find
half in numbers with the base year as 2000.
out whether they are erroneous , errors if any,
II. Increasing the men’s labour force participation
will be in two parts of the sentence. Choose the
by 25% percentage points might account for
option that represents the incorrect parts as your
more than 70% of the potential GDP growth
answer. If there is no error, mark(e) as your
opportunity
answer.
III. India’s Family Planning 2030 partnership
18) NASA’s Hubble Space Telescope has
includes new contraceptive options.
discovered/A the furthest individual star/B ever
a) True, False, False
seen, an enormous big blue stellar/C body
b) True, false, True
nicknamed Icarus located over halfway across
c) False, False, True
the universe/D. No error/E
d) False, True, False
a) AB
e) True, True, False
b) BC
c) BD
17) Three words have been highlighted in the
d) CD
given passage namely (A), (B) and (C). Two
e) No error
columns are given below with column 1 having
the highlighted words and column 2 having the
19) The report sites research that shows/A that
synonyms of the words. You are required to
gender gaps cause an average income loss/B of
match the words with their appropriate
15 percent in the OECD economies,/C 40
synonyms.
percent of which is since entrepreneurship
gaps/D. No error/E

Click Here For Bundle PDF Course | support@guidely.in Page 6 of 12


Bank Po Mains PDF Course 2024
English Day - 22

a) AC companies dealing in crypto currencies/D.No


b) AD error/E
c) BC a) AC
d) CD b) BC
e) No error c) BD
d) CD
20) Manipuri ponies which are central/A to the e) No error
game are facing a very precarious/B situation of
possible extinction/C sooner rather than 22) In most countries, a combustible mixture/A of
later/D.No error/E authoritarianism, unemployment and youth/B has
a) AD given rise to disaffection of strongmen/C rulers
b) BC which has in turn spill over into uprising/D.No
c) BD error/E
d) CD a) AB
e) No error b) AC
c) BD
21) The central bank issues a circular/A three d) CD
months ago, banning all financial institutions/B e) No error
under its control by providing service to/C
Click Here to Get the Detailed Video Solution for the above given Questions
Or Scan the QR Code to Get the Detailed Video Solutions

Answer Key with Explanation


1. Answer: B Replace ‘gave’ with ‘accorded’ and ‘advantages’
with ‘privileges’ to make the sentence correct
and meaningful.

Click Here For Bundle PDF Course | support@guidely.in Page 7 of 12


Bank Po Mains PDF Course 2024
English Day - 22

‘Had gave’ is wrong - hence it needs Many private banks have an international
replacement, similarly the word ‘privileges’ is presence, allowing clients to access services
more suitable to the context of the sentence than and investment opportunities in multiple
‘advantages’. jurisdictions.
Accorded - to give something to somebody
Privileges - a right or advantage 3. Answer: E
Option a - give is wrong, opportunities sound No replacement required, the highlighted words
inappropriate are correct and are used appropriately in the
Option c - grant wrong form of usage(not a past sentence.
tense) Exerted - to make a big effort
Option d - rights is wrong because it is
redundant(special rights and rights - wrong) 4. Answer: A
Hence, Option b is the best answer. Replace ‘burnt’ with ‘combusted’ and ‘addition’
The sentence after replacement : with ‘contributed’ to make the given sentence
The Article had accorded special rights and meaningful and correct.
privileges to the people of Jammu and Kashmir Combusted - consumed/destroyed by fire
since 1954 in accordance with the Instrument of CNG - compressed natural gas can be
Accession. consumed(combusted), other options like fired,
blazed and ignited seem inappropriate and
2. Answer: A hence we discard them.
Replace assess with access(because assess is Option a - combusted and contributed are the
to judge or test), similarly replace many with right words that can be used to replace the
multiple. highlighted word in the sentence.
Access - to use The sentence after replacement :
Multiple - more in number When combusted, CNG produces fewer
Option b - to availing is incorrect, few is greenhouse gas emissions, particulate matter,
inappropriate because the sentence talk and other pollutants that contribute to air
Option c - numerous in incorrect to the context of pollution and climate change.
the sentence
Option d - clients don't enable services(client will 5. Answer: D
access the service), most is also inappropriate Replace ‘automatic’ with ‘autonomous’ and
Hence, option a is the right answer. ‘navigation’ with ‘navigate’ to make the sentence
The sentence after replacement : correct.

Click Here For Bundle PDF Course | support@guidely.in Page 8 of 12


Bank Po Mains PDF Course 2024
English Day - 22

Automatic and navigation are either incorrect or first because A seems to be a concluding
inappropriate in the sentence. sentence because it has taken a generic form.
Autonomous - one’s own So, the sequence of the coherent passage
Navigate - to use map and travel formed is FBDEGA.
Option a - different is not relevant, to decide and C is the odd one out because it is irrelevant to
make decisions(makes no sense) the passage as it talks about the compensation
Option b - advancing wrong form of word, cess which is not the main idea of the passage.
navigate is a better word than fly in this context Although it comes under the vast topic ‘GST’,
Option c - free and sail(both are inappropriate in having a look at the other sentences, sentence C
the sentence) seems to be deviating and hence is the odd one
The sentence after replacement : out.
Some drones also have autonomous capabilities 6. Answer: B
that allow them to perform pre-programmed The first sentence after rearrangement is ‘F’ and
tasks, follow flight paths, or use advanced the last sentence is ‘A’.
algorithms to navigate and make decisions
based on sensor inputs. 7. Answer: C
Directions (6-10): BEG is the correct answer because B is the
The correct sequence of rearrangement is second sentence, E is the fourth sentence and G
FBDEGA (C is the odd one out) is the fifth sentence.
Sentence ‘F’ is the first sentence as it gives
proper introduction to the passage with 8. Answer: C
information that it’s been 6 months since the Sentence C is the odd one out sentence.
launch of GST. The other sentences do not
seem to be a starter of the passage. The second 9. Answer: D
sentence would be ‘B’ because ‘marking the The error is in part d of the given sentence D.
occasion the Finance Minister has said…’ this Replace the word ‘sorting’ with ‘sort’ to make the
phrase makes it clear that sentence B succeeds sentence correct.
sentence F. The third sentence is ‘D’ as it talks ‘To sorting’(wrong) instead it must be ‘to sort’.
about the initial phase of GST introduction, how Introduced soon after the demonetisation shock,
it was a shock then and its challenges. The the GST was viewed as another disruptor for the
fourth sentence is ‘E’ which explains that the informal economy and its initial technical,
firms/companies have adapted to change(GST). structural and procedural challenges took a while
Between the sentences G and A, G will come to sort out.

Click Here For Bundle PDF Course | support@guidely.in Page 9 of 12


Bank Po Mains PDF Course 2024
English Day - 22

10. Answer: E II - Attaining gender parity meaning equal


QTPSR is the correct sequence of workforce of men and women will lead to good
rearrangement. economic growth, it is also proven that women
P - cannot be the first fragment because ‘starting workforce are more productive in nature.
this august and’ cannot be the first part of the III - Women are deprived of basic human rights
sentence and equality which acts as a hindrance for their
Q- Good starter starting with ‘now’, it can be a growth, by providing equal rights to all, everyone
continuation to the previous sentence in the can contribute to the nation and for its growth.
coherent passage. This is followed by T, now the So, all the three statements hold true and hence
firms have to generate invoices.Now comes ‘P’ option (e) is the right answer.
as the third fragment. Following P comes S
‘there has been no ostensible pushback’(joining 12. Answer: E
P and S). Lastly it is R to end the sentence in a Both the statements II and III are true according
correct way. to the information given in the passage.
Now that all businesses with annual turnover of Statement I is wrong because July 11th is World
₹ 5 crore will have to generate e-invoices population day and it is not specific to India.
starting this August, and that there has been no II and III are information that are inferred from
ostensible pushback from smaller businesses the given passage and are true and valid.
over this, indicates that firms have gradually Hence, option (e) is the correct answer for this
embraced the change. question.

11. Answer: E 13. Answer: A


According to the given passage, all the three Life expectancy at birth means the average
statements I , II and III contribute to the number of years a newborn infant is expected to
economic growth of a nation or country. live based on the prevailing patterns of mortality
I - Women empowerment plays a major role in rate.
the economic development of a nation, all So, only option (a) conveys the exact meaning of
women must be given good education and the highlighted phrase as given in the passage.
awareness about every difficulty they face and
how they can face and voice out their 14. Answer: C
opinions.According to the passage, empowered All the questions a,b and d can be answered
women can contribute really well to the society. with the information given in the passage above
as the passage contains relevant and

Click Here For Bundle PDF Course | support@guidely.in Page 10 of 12


Bank Po Mains PDF Course 2024
English Day - 22

appropriate information regarding all these topics Option c is the correct answer.
of discussion. Synonym of galvanise is energise
Question in option (c) cannot be answered Synonyms of coercion is oppression
because there is no such discussion made in the Synonym of discourse is dialogue
passage and the data/information is inadequate Galvanise - to stimulate or excite
to answer the question. Coercion - the use of force to persuade
Hence, option(c) is the correct answer for this someone to do something
question. Discourse - long serious discussion

15. Answer: D 18. Answer: B


Only phrase/fragment d is correct and can Part B and C has errors.
possibly replace the highlighted part (X) to make B - Replace ‘furthest’ with ‘farthest’(furthest -
the respective sentence correct and meaningful. higher degree, farthest - actual physical
Assert - to be clear/confident/bold distance)
Options a,c and e are wrong because they talk C - enormous big is incorrect as both the words
about assets which is not even a part of the have the same meaning
discussion in the given passage. The correct sentence : NASA’s Hubble Space
Option b is wrong because ‘there’ is incorrect Telescope has discovered the farthest individual
and the statement is inappropriate and incorrect star ever seen, an enormous blue stellar body
by itself. nicknamed Icarus located over halfway across
the universe.
16. Answer: B
Statement I is true, II is false and III is true 19. Answer: B
according to the given passage. The errors are in part A and D of the given
II is false because the passage has got the sentence.
information only regarding the women,i.e, by Replaces ‘sites’ with ‘cites’ in part A (site - a
increasing the women’s labour force participation piece of land, cite - to mention)
by 10% percentage points might account for Change ‘since’ to ‘due to’ in part D (since in
more than 70% of the potential GDP growth incorrect here)
opportunity. The correct sentence : The report cites research
Hence, the correct answer is Option (b). that shows that gender gaps cause an average
income loss of 15 percent in the OECD
17. Answer: C

Click Here For Bundle PDF Course | support@guidely.in Page 11 of 12


Bank Po Mains PDF Course 2024
English Day - 22

economies, 40 percent of which is due to service to companies dealing in crypto


entrepreneurship gaps. currencies.

20. Answer: E 22. Answer: D


The given sentence is correct and error free, Errors are in part C and D of the given sentence.
hence we go for option (e) as our answer. C - ‘disaffection of’ is incorrect instead it must be
‘disaffection with’, one gets disappointed with
21. Answer: A something/someone not ‘of someone/something’
The erroneous parts are A and C. (change the preposition used to make the part
The circular had been issued three months ago - correct)
so replace ‘issues’ with ‘issued’. D - has in turn spilled over(replace spill with
Banning institutions from providing services is spilled to make it correct, use past tense)
the correct way to put the phrase(banning by is The correct sentence : In most countries, a
wrong, it should be banning from) combustible mixture of authoritarianism,
The correct sentence : The central bank issued unemployment and youth has given rise to
a circular three months ago, banning all financial disaffection with strongmen rulers which has in
institutions under its control from providing turn spilled over into uprising.

Click Here For Bundle PDF Course | support@guidely.in Page 12 of 12


Bank Po Mains PDF Course 2024
Reasoning Day -23 (Eng)

Reasoning Aptitude
Direction (1-5): Study the following information number of shoes. No one lives to the east of K,
carefully and answer the given questions who has 4 shoes less than L. The difference
Eight persons - I, J, K, L, M, N, O and P are living between the number of shoes with I and J is 3. J
on four different floors of a four storey building has the number of shoes which is reverse of the
where the lowermost floor is numbered one and number of shoes with M. The difference between
the floor immediately above it is numbered two the number of shoes with O and K is 15. No one
and so on. lives to the east of the one who has the number
Note-I: Each floor has two type of flats viz., Flat- of shoes which is a multiple of 5.
A and Flat-B, where Flat A is to the west of Flat 1. What is the sum of the number of shoes with
B. L, N and P?
Note-II: Flat B of floor 2 is immediately above a) 110
Flat B of floor 1 and immediately below Flat B of b) 96
floor 3 and so on. Similarly, Flat A of Floor 2 is c) 133
immediately above Flat A of floor 1 and d) 109
immediately below Flat A of floor 3 and so on. e) 113
Note-III: Area of each flat on each floor is equal.
Note-IV: Only two persons live on each floor and 2. Which of the following statements is/are not
only one person lives in each flat. false as per the given arrangement?
Note V: Each person has different number of a) The difference between the number of shoes
shoes with them, which is a two digit number. No with J and O is a prime number
one has more than 50 shoes. b) The one who lives immediately above the flat
The one who has 27 shoes lives two floors above of I has an odd number of shoes
the flat of N, who lives on an odd numbered floor, c) M has 20 shoes less than K
has 8 shoes less than L. The number of floors d) All the given statement is true
below N is one less than the number of floors e) All the given statement is false
above I, who has the number of shoes which is a
multiple of seven. Only one floor is between I and 3. Who among the following person lives two
the one who has 17 shoes less than P, and both floors above J?
are living in the same type of flat. M, who has a a) The one who lives immediately below the flat
prime number of shoes, lives two floors above of M
the flat of L. Both M and P neither live on the b) P
same floor nor in the same type of flat. Only one c) I
floor is between K and P, who has a square d) The one who has 46 shoes

Click Here For Bundle PDF Course | support@guidely.in Page 1 of 10


Bank Po Mains PDF Course 2024
Reasoning Day -23 (Eng)

e) Both a and c I. H$T


II. R@L
4. Which of the following combination is true? III. Q$P
a) Floor number 4, Flat B - M a) Only conclusion I is true
b) Floor number 2, Flat A - P b) Only conclusion II is true
c) Floor number 3, Flat A - I c) Both conclusions I and II are true
d) Floor number 1, Flat B - J d) Only conclusion III is true
e) Floor number 4, Flat A - K e) Both conclusions II and III are true

5. If the rate of each shoe bought by the person 7. Statements:


who lives in flat A is Rs.1000 and Flat B is A@B&C%D$E; G%F$D#H$I; L$H&J&K%M
Rs.500, then who among the following person Conclusions:
spent the highest amount to buy shoes? I. K@B
a) Both N and K II. L$G
b) O III. E#M
c) Both P and M a) Only conclusion I is true
d) L b) Only conclusion II is true
e) Both J and I c) Both conclusions I and II are true
d) Only conclusion III is true
Directions (6-10): In the given questions, the e) None is true
relationship between different elements is shown
in the statements followed by some conclusions. 8. Statements:
Find the conclusion which is definitely true. K@L%J&H#G; A&S%Y$D@F; E#L$T%Y@O
A#B means B is neither greater than nor equal to Conclusions:
A I. F&K
A$B means B is neither smaller than nor equal to II. H$A
A III. J#O
A%B means B is neither greater than nor less a) Only conclusion I is true
than A b) Only conclusion II is true
A&B means B is not greater than A c) Both conclusions I and II are true
A@B means B is not smaller than A d) Only conclusion III is true
6. Statements: e) Both conclusions II and III are true
Q@W$E%R&T; U&E@I%O$P; H#O@G%L#A
Conclusions: 9. Statements:

Click Here For Bundle PDF Course | support@guidely.in Page 2 of 10


Bank Po Mains PDF Course 2024
Reasoning Day -23 (Eng)

Z$X%K&O#V; S&H%K$L@C; A#S@D#F$G Row 2 contains numbers which is a consecutive


Conclusions: multiple of 19 from left to right starting from 57
I. X$G Row 3 contains numbers which is a consecutive
II. A#V even numbers from left to right starting from 28
III. K&G Row 4 contains numbers which is a consecutive
a) Only conclusion I is true multiple of 13 from left to right starting from 13
b) Only conclusion II is true Row 5 contains numbers which is a consecutive
c) Both conclusions I and II are true prime numbers from left to right starting from 41
d) Either conclusion I or III is true Conditions:
e) Both b and d are true I. If the outcome is less than 500, we get Rs.90
as cashback
10. Statements: II. If the outcome is between 501 and 1300, we
P@L@M%N&B; S@W#A%E@R; A%V$N#C$X get Rs.200 as cashback
Conclusions: III. If the outcome is between 1301 and 2200, we
I. P@R get Rs.450 as cashback
II. W#C IV. If the outcome is between 2201 and 3500, we
III. E$X get Rs.700 as cashback
a) Only conclusion I is true V. If the outcome is between 3501 and 5000, we
b) Both conclusions I and II are true get Rs.1500 as cashback
c) None is true Note: The amount of money we receive will differ
d) Only conclusion II is true depending on the outcome we receive from the
e) Both conclusions I and III are true string
For the outcome of the string:
Direction (11-15): Study the following information A. If an odd number is followed by a non-prime
carefully and answer the given questions. number, then the sum of both numbers is
There is a 5x5 matrix that is used to get cash multiplied by 3
back amount while using credit card in POS. The B. If an even number is followed by a prime
row of the matrix is denoted as 1 to 5 from top to number, then find the difference between both
bottom and the column of the matrix is denoted numbers and cube the sum of the digits of the
as @, #, $, % and & from left to right resultant.
respectively. C. If an odd number is followed by a prime
Row 1 contains numbers which is a consecutive number, then the sum of the unit digits of both
multiple of 17 from left to right starting from 34. numbers is multiplied by 7

Click Here For Bundle PDF Course | support@guidely.in Page 3 of 10


Bank Po Mains PDF Course 2024
Reasoning Day -23 (Eng)

D. If an even number is followed by a non-prime c) 4


odd number, then the difference between the unit d) 8
digits of both numbers is multiplied by 9. e) 9
E. If an even number is followed by an even
number, then multiply the unit digit of both 14. How much cash back will get from the
numbers and square the resultant. outcome of the given string?
11. How much cashback will get from the sum of 1% 4@ 3& 5$
the outcome of both strings? a) Rs.90
I. 2# 5% 1& b) Rs.200
II. 3@ 2$ 4% c) Rs.450
a) Rs.90 d) Rs.700
b) Rs.200 e)Rs.1500
c) Rs.450
d) Rs.700 15. Find the difference between the cash back
e) Rs.1500 we get from the outcome of each string given
below.
12. What is the product of the outcome of the I. 1%5#2&
below strings? II. 2@3#4$
I. 2% 3& 1# a) Rs.90
II. 4@ 3% 5$ b) Rs.210
a) 2420 c) Rs.110
b) 2320 d) Rs.150
c) 2220 e) Rs.100
d) 2520
e) 2120 Direction (16-20): Study the following information
carefully and answer the given questions.
13. If the rate of one chocolate is Rs.40, then Ten persons – A, B, C, D, E, F, G, H, I and J are
how many chocolates can buy using the cash sitting around a circular table, where some of
back will get from the sum of the outcome of both them are facing towards the centre while some of
strings? them are facing away from the centre.
I. 1@ 3$ 4# Note: Consecutive alphabetically named persons
II. 5& 1$ 5@ are not sitting opposite to each other.
a) 3 G sits fourth to the right of B, where both are
b) 5 facing opposite directions. Immediate neighbours

Click Here For Bundle PDF Course | support@guidely.in Page 4 of 10


Bank Po Mains PDF Course 2024
Reasoning Day -23 (Eng)

of B are facing the opposite direction to him.


Two persons sit between G and H, where both 18. What is the position of E with respect to I in
are facing the same direction. J and H are sitting the given arrangement?
immediate right of each other. The number of a) Third to the left
persons sitting between J and A is one less than b) Seventh to the right
the number of persons sitting between A and C, c) Fourth to the left
when counted from the left of both J and A d) Third to the right
respectively. Three persons sit between I and D. e) Fifth to the left
E sits to the immediate right of D. Both I and F
are facing the opposite direction of E. The one 19. If J and B interchanged their position,
who sits to the immediate right of F is facing similarly D and H interchanged their position,
away from the centre. then who among the following person sits third to
16. Who among the following pair of persons the left of D?
facing the same direction? a) F
a) GJ b) C
b) HI c) B
c) CE d) J
d) DJ e) G
e) FH
20. As many persons sit between G and __ as
17. Who among the following person sits fourth between C and __, when counted left of both G
to the right of F? and C.
a) E a) J, H
b) The one who sits opposite to H b) I, A
c) A c) A, E
d) I d) H, F
e) The one who sits second to the right of E e) D, J

Click Here For Bundle PDF Course | support@guidely.in Page 5 of 10


Bank Po Mains PDF Course 2024
Reasoning Day -23 (Eng)

Click Here to Get the Detailed Video Solution for the above given Questions
Or Scan the QR Code to Get the Detailed Video Solutions

Answer Key with Explanation


Directions (1-5): From the above conditions, we have two
1. Answer: C possibilities:
2. Answer: A
3. Answer: E
4. Answer: D
5. Answer: D
Final arrangement:

Again we have,
 Only one floor is between I and the one
who has 17 shoes less than P, and both
are living in the same type of flat.
 M, who has prime number of shoes, lives
We have, two floors above the flat of L.
 The one who has 27 shoes lives two  Both M and P neither live on same floor
floors above the flat of N, who lives on an nor in the same type of flat.
odd numbered floor, has 8 shoes less  Only one floor is between K and P, who
than L. has a square number of shoes.
 The number of floors below N is one less  No one lives to the east of K, who has 4
than the number of floors above I, who shoes less than L.
has the number of shoes which is a
multiple of seven.

Click Here For Bundle PDF Course | support@guidely.in Page 6 of 10


Bank Po Mains PDF Course 2024
Reasoning Day -23 (Eng)

 If K has 12 shoes, then L may have 16


shoes and if K has 42 shoes then L may
have 46 shoes (L=K+4)
 If L has 46 shoes then N may have 38
shoes and if L has 16 shoes, then N may
have 8 shoes, which is not possible (Only
Again we have, two digit number)
 The difference between the number of  Thus, we may conclude, O=27, K=42,
shoes with I and J is 3. L=46, N=38
 J has the number of shoes which is While applying above conditions, case 2 gets
reverse of the number of shoes with M. eliminated, because no one should live to the
 The difference between the number of east of the one who has the number of shoes
shoes with O and K is 15. which is a multiple of 5. Thus, case 1 gives the
 No one lives to the east of the one who final arrangement.
has the number of shoes which is a
multiple of 5.
Calculations:
 M (Prime) – Taking all the prime numbers
from 11 to 50, we can find the number of
shoes had by J, who has the reverse of
the shoes had by M.
 Then we can find the number of shoes
6. Answer: E
had by P by using J (P=J+17, P has
I. H$T -> T>H (T≤R=E≤I=O<H) -> False
square number of shoes)
II. R@L -> L≥R (R=E≤I=O≤G=L) -> True
 By this, we can conclude: M=23, J=32
III. Q$P -> Q<P (P>O=I≥E>W≥Q) -> True
and P=49.
 Next, I-J is 3 (i.e. I=35/29), but I has
7. Answer: D
number of shoes which is multiple of 7, so
I. K@B -> B≥K (K≤J≤H<D=C≤B) -> False
I=35.
II. L$G -> G>L (G=F<D>H>L) -> False
 We know O has 27 shoes, so K may have
III. E#M -> M<E ( E>D>H≥J≥K=M) -> True
12 or 42 shoes (K-O is 15)

8. Answer: B
I. F&K -> F≥K (K≤L<T=Y<D≤F) -> False

Click Here For Bundle PDF Course | support@guidely.in Page 7 of 10


Bank Po Mains PDF Course 2024
Reasoning Day -23 (Eng)

II. H$A -> A>H (H≤J=L<T=Y=S≤A) ->True 13. Answer: B


III. J#O -> O<J (J=L<T=Y≤O) -> False I. 1@3$4# -> 34 32 26
By condition E: square of (4*2) = 64
9. Answer: E By condition E: square of (4*6) = 576
I. X$G -> X<G (X=K=H≤S≤D>F<G) -> False II. 5&1$5@ -> 59 68 41
II. A#V -> V<A (V<O≤K=H≤S<A) -> True By condition A: (59+68)*3 = 381
III. K&G -> K≥G (X=K=H≤S≤D>F<G) -> False By condition C: (1+1)*7 = 14
Combining I and III, we can say either Sum of both the strings= 590. Thus, cashback
conclusion I or III is true we get is Rs.200, so we can buy 5 chocolates.

10. Answer: C 14. Answer: E


I. P@R -> P≤R (P≤L≤M=N>V=A=E≤R) -> False 1%4@3&5$ -> 85 13 36 47
II. W#C -> W>C (C<N>V=A<W) -> False By condition C: (5+3)*7 = 56
III. E$X -> E<X (X>C<N>V=A=E) -> False By condition E: square of (6*6) = 1296
By condition B: (1296-47) = 1249 =Cube of
11. Answer: B (1+2+4+9) = 4096
I. 2#5%1& -> 76 53 102
By condition B: 76-53=23 -> 2+3 -> cube of 5 = 15. Answer: C
125 I. 1%5#2& -> 85 43 133
By condition A: (125+102)*3 = 681 By condition C: (5+3)*7 = 56
II. 3@2$4% -> 28 95 52 By condition D: (6-3)*9 = 27
By condition D: (8-5)*9 =27 We get Rs.90 as cashback.
By condition A: (27+52)*3 =237 II. 2@3#4$ -> 57 30 39
Sum of both strings=681+237 = 918 By condition A: (57+30)*3 = 261
By condition A: (261+39)*3 = 900
12. Answer: D We get Rs.200 as cashback.
I. 2%3&1# -> 114 36 51 Hence, difference between the two cashback is
By condition E: square of (6*4) = 576 Rs.110
By condition D: (6-1)*9 = 45
II. 4@3%5$ -> 13 34 47
By condition A: (13+34)*3 = 141
By condition C: (1+7)*7 = 56
Product of both strings: 45*56=2520

Click Here For Bundle PDF Course | support@guidely.in Page 8 of 10


Bank Po Mains PDF Course 2024
Reasoning Day -23 (Eng)

Again we have,
 The number of persons sitting between J
and A is one less than the number of
Directions (16-20): persons sitting between A and C, when
16. Answer: C counted from the left of both J and A
17. Answer: E respectively.
18. Answer: D  Three persons sit between I and D.
19. Answer: D  E sits to the immediate right of D.
20. Answer: B
Final arrangement:

Again we have,
 Both I and F are facing the opposite
direction of E.
 The one who sits to the immediate right of
F is facing away from the centre.
We have, While applying the above condition, case 1 gets
 G sits fourth to the right of B, where both eliminated, because the one who sits to the
are facing opposite directions. immediate right of F should face away from the
 Immediate neighbours of B are facing the centre. Thus, case 2 gives the final arrangement.
opposite direction to him.
 Two persons sit between G and H, where
both are facing the same direction.
 J and H are sitting immediate right of
each other.
From the above conditions, we have two
possibilities:

Click Here For Bundle PDF Course | support@guidely.in Page 9 of 10


Bank Po Mains PDF Course 2024
Reasoning Day -23 (Eng)

Click Here For Bundle PDF Course | support@guidely.in Page 10 of 10


Bank Po Mains PDF Course 2024
Quantitative Aptitude Day -23 (Eng)

Quantitative Aptitude

Directions (1 – 4): Study the given information a. 30


carefully and answer the given questions. b. 60
The table given below shows the number of runs c. 36
scored by three different players – P, Q, and R d. 48
against five different teams. Some data is e. None of these
missing you have to find the missing data and
answer the questions given below. 3. If Q scored 25% more runs against New
Zealand then find the percentage increase in the
total runs scored by P, Q, and R together against
New Zealand.
a. 7%
b. 12%
c. 5%
Note: Runs scored by Q against England are d. 15%
less than the runs scored by R against England. e. None of these
1. Runs scored by Q against England are only by
hitting 6s and 4s. If the number of 4s hit by Q is 2 4. The strike rate of P against Australia and
less than the double number of 6s hit by Q. Find Pakistan is 80 and 50 respectively. Find the
the runs scored by Q by hitting 4s. difference between the number of balls faced by
a. 96 P against Australia and Pakistan.
b. 120 Note: Strike Rate = ((Number of runs scored) /
c. 112 (Number of balls faced)) * 100
d. 80 a. 30
e. None of these b. 12
c. 16
2. Q played 4 matches against Pakistan and 6 d. 24
matches against England. Find the average runs e. None of these
scored by Q against England is what percentage
of his average runs scored against Pakistan.

Directions (5 – 8): Study the given information carefully and answer the given questions.

Click Here For Bundle PDF Course | support@guidely.in Page 1 of 16


Bank Po Mains PDF Course 2024
Quantitative Aptitude Day -23 (Eng)

In school X different numbers of students are studying in class – 6th, 7th, 8th, 9th, and 10th in 2022. The first
pie chart given below shows the percentage distribution of students in given classes. The second pie
chart given below shows the percentage distribution of the number of girls in given classes.

Click Here For Bundle PDF Course | support@guidely.in Page 2 of 16


Bank Po Mains PDF Course 2024
Quantitative Aptitude Day -23 (Eng)

Note: i. Total number of students in 9th class is 50% more than the total number of students in 7 th class.
ii. The ratio of the total number of students in the 6th class to the 7th class is 5 : 3.
5. Rs.62 has been divided among students of 8. Number of girls in class 6th and 9th together
class 10 in such a way that each boy gets 50% are what percent of the number of students in
more amount than each girl gets. Find the class 8th and 10th together.
amount got by each girl. a. 25%
a. Rs.0.75 b. 60%
b. Rs.0.25 c. 50%
c. Rs.0.60 d. 40%
d. Rs.0.40 e. None of these
e. None of these
Direction (9 – 12): Study the given information
6. 60% of students from class 6th have carefully and answer the given questions.
participated in a dance competition. 3/5th of the Four students – P, Q, R, and S have scored
boys from class 6th have participated in the different marks in five subjects – Mathematics,
dance competition. Find the number of girls who Science, English, Hindi, and Computer inthe
participated in the dance competition. board exam 2022. The table given below shows
a. 50 the average marks scored by P, Q, R, and S,
b. 75 Marks scored by P as a percentage of marks
c. 60 scored by Q, the Difference between average
d. 80 marks scored by P and S together, and average
e. None of these marks scored by R and S together, marks scored
by S in five subjects.
7. 15% of students from class 8th and 25% of
students from class 9 failed in 2022. Find the
number of students in class 9thin 2023 if no new
admission takes place in 2023.
a. 274
b. 244
c. 296
d. 288 Note: Only in Mathematics R scored more marks
e. None of these than P. In other subjects, P scored more marks
than R

Click Here For Bundle PDF Course | support@guidely.in Page 3 of 16


Bank Po Mains PDF Course 2024
Quantitative Aptitude Day -23 (Eng)

9. The average marks scored by P, Q, and S 11. Average marks scored by P in Hindi, R in
together in English is what percent more than the Computer, and S in Mathematics together are
average marks scored by P, Q, R, and S what percent of the sum of marks scored by Q in
together in Computer? Mathematics and S in Computer together?
a.15% a. 44.44%
b.12.5% b. 36.36%
c.10% c. 37.5%
d.17.5% d. 50%
e. None of these e. None of these

10. In Computer, R fails by 12 marks. If the 12. Find the difference between the sum of
passing mark is 40% of the maximum marks marks scored by P, R, and S in Hindi and the
then find the maximum marks in Computer. average marks scored by P, Q, and S in English.
a. 120 a. 134
b. 180 b. 96
c. 130 c. 112
d. 150 d. 146
e. None of these e. None of these

Direction (13 – 14): Study the given information carefully and answer the given questions.
In a laptop store, laptops of three brands – Dell, HP, and Lenovo have been sold in five months –
January, February, March, April, and May. The bar graph given below shows the total number of laptops
sold. The table given below shows HP laptops sold as a percentage of Lenovo laptops sold, the
percentage of Lenovo laptops sold out of the total number of laptops sold in the given month.

Click Here For Bundle PDF Course | support@guidely.in Page 4 of 16


Bank Po Mains PDF Course 2024
Quantitative Aptitude Day -23 (Eng)

13. Find the ratio of the sum of the number of b. 75%


Dell laptops sold in January and HP laptops sold c. 83.33%
in April together to the sum of HP laptops sold in d. 88.88%
March and Lenovo laptops sold in February e. None of these
together.
a. 5 : 7 Direction (15 – 17): Study the given information
b. 7 : 9 carefully and answer the given questions.
c. 9 : 8 Three friends – P, Q, and R have invested
d. 8 : 11 different sums in four businesses – A, B, C, and
e. None of these D.
The sum invested by R in business D is Rs.’b’.
14. The average number of Dell laptops sold in Sum invested by Q in business D is 75% of the
February and March together is what percent of sum invested by R in business D which is 1/9th
the sum of Dell laptops sold in January and HP less than the sum invested by P in business D.
laptops sold in April together? Sum invested by Q in business C is Rs.’a’. Sum
a. 87.5% invested by P in business B is 20% more than

Click Here For Bundle PDF Course | support@guidely.in Page 5 of 16


Bank Po Mains PDF Course 2024
Quantitative Aptitude Day -23 (Eng)

the sum invested by Q in business C. The ratio of d. 8137


the sum invested by Pin Business B to the sum e. None of these
invested by R in Business A is 6 : 7. The Sum
invested by P in Business A is 1/3rd more than 17. In business D, 25% of the total profit is
the sum invested by R in Business D. The ratio donated to charity and the rest of the profit is
of the sum invested by P, Q, and R in business A distributed among P, Q, and R as per their
is 8 : 6 : 7 respectively. R invested 50% more investment. P received Rs.3348 as his profit
sum in business C than Q invested in business share. Find the total profit at the end of 1 year.
C. The ratio of the sum invested by P and the a. 11408
sum invested by R in business C is 5 : 6 b. 9454
respectively. R invested 33.33% less sum in c. 12658
business B than he invested in business C. Q d. 10852
invested Rs.6000 more in business B than R e. None of these
invested in business B. P, Q, and R together
invested Rs.75000 in business C. Direction (18 – 20): Study the given information
15. In business C, R withdraws his sum after 8 carefully and answer the given questions.
months from the start of business. At the end of 18. Find the value of P, Q, and R in the given
1 year, the total profit is Rs.5681. Find the profit series and choose the correct relation among
share of Q. them.
a. 1576 I. 10, 10, 17, 43, 106, P, 445
b. 1748 II. 36, 71, Q, 1059, 7412, 81531
c. 1864 III. 22, 30, 46, 78, R, 270, 526
d. 1654 a. P > Q > R
e. None of these b. P > Q < R
c. P < Q < R
16. In business A, Q increases his investment by d. P < Q > R
25% after 6 months from the start of the e. None of these
business and after 3 more months, P withdraws
his investment. At the end of 1 year, P got 19. Find the value of P, Q, and R in the given
Rs.2472 as his profit share. Find the total profit series and choose the correct relation among
at the end of 1 year. them.
a. 6472 I. 81, 85, P, 137, 201, 301
b. 6792 II. 47, 48, 50, 56, Q, 200, 920
c. 7537 III. 508, 339, 218, 137, R, 63, 54

Click Here For Bundle PDF Course | support@guidely.in Page 6 of 16


Bank Po Mains PDF Course 2024
Quantitative Aptitude Day -23 (Eng)

a. P > Q > R I. 38, 42, P, 76, 125, 246, 415


b. P > Q < R II. 18, 35, Q, 409, 2041, 12241
c. P < Q < R III. 24, 26, 32, 44, 64, 94, R
d. P < Q > R a. P > Q > R
e. None of these b. P > Q < R
c. P < Q < R
20. Find the value of P, Q, and R in the given d. P < Q > R
series and choose the correct relation among e. None of these
them.
Click Here to Get the Detailed Video Solution for the above given Questions
Or Scan the QR Code to Get the Detailed Video Solutions

Answer Key with Explanation


Directions (1 – 4): Runs scored by P against Sri Lanka = 540 –
Runs scored by P, Q, and R together against Sri (162 + 270)
Lanka = 540 = 108
The ratio of runs scored by Q and R against Sri The ratio of runs scored by Q and R against
Lanka = 60 : 100 Australia = 75 : 100
=3:5 =3:4
Difference between runs scored by Q and R Difference between runs scored by Q and R
against Sri Lanka = 108 against Australia = 48
Runs scored by Q against Sri Lanka = (108/(5 – Runs scored by Q against Australia = (48/(4 – 3))
3)) * 3 *3
= 162 = 144
Runs scored by R against Sri Lanka = (108/(5 – Runs scored by R against Australia = (48/(4 – 3))
3)) * 5 *4
= 270 = 192

Click Here For Bundle PDF Course | support@guidely.in Page 7 of 16


Bank Po Mains PDF Course 2024
Quantitative Aptitude Day -23 (Eng)

Runs scored by P against Australia = ((144 + The ratio of runs scored by Q and R against New
192)/84) * 16 Zealand = 70 : 100
= 64 = 7 : 10
Runs scored by P against Pakistan = 400 * Total runs scored by Q and R together against
(12/100) New Zealand = 800 – 256
= 48 = 544
The ratio of runs scored by Q and R against Runs scored by Q against New Zealand = 544 *
Pakistan = 120 : 100 (7/17)
=6:5 = 224
Total runs scored by Q and R together against Runs scored by R against New Zealand = 544 *
Pakistan = 400 – 48 (10/17)
= 352 = 320
Runs scored by Q against Pakistan = 352 *
(6/11)
= 192
Runs scored by R against Pakistan = 352 *
(5/11)
= 160
Runs scored by P against England = 600 * 1. Answer: B

(19/100) Runs scored by Q against England = 216

= 114 Let the number of 6s hit by Q against England

Total runs scored by Q and R together against be x.

England = 600 – 114 Then, number of 4s hit by Q against England =

= 486 2x – 2

Runs scored by Q against England = (486 – According to the question,

54)/2 6 * x + 4 * (2x – 2) = 216

= 216 6x + 8x – 8 = 216

Runs scored by R against England = 216 + 54 14x = 224

= 270 x = 16

Runs scored by P against New Zealand = 800 * Number of 4s hit by Q against England = 2 * 16 –

(32/100) 2

= 256 = 30
Runs scored by Q against England by 4s = 30 *
4

Click Here For Bundle PDF Course | support@guidely.in Page 8 of 16


Bank Po Mains PDF Course 2024
Quantitative Aptitude Day -23 (Eng)

= 120 Number of balls faced by P against Pakistan =


(48/50) * 100
2. Answer: E = 96
Runs scored by Q against Pakistan = 192 Difference between the number of balls faced by
Average of Q against Pakistan = 192/4 P against Australia and Pakistan = 96 – 80
= 48 = 16
Runs scored by Q against England = 216
Average of Q against England = 216/6 Directions (5 – 8):
= 36 From i,
Required percentage = (36/48)*100 = 75% Total number of students in 9th class = ab2
Total number of students in 7th class = a2b
3. Answer: A Then, (ab2)/(a2b) = 3/2
Runs scored by P against New Zealand = 256 b/a = 3/2
Runs scored by Q against New Zealand = 224 From ii,
Runs scored by R against New Zealand = 320 Total number of students in 6th class = a2c
Total runs scored by P, Q, and R together Total number of students in 7th class = a2b
against New Zealand = 256 + 224 + 320 Then, (a2c) / (a2b) = 5/3
= 800 c/b = 5/3
New runs scored by Q against New Zealand = a = 2, b = 3, c = 5
224 * (5/4) Total number of students in class 6th = 800 *
= 280 (20/100) = 160
New total runs scored by P, Q, and R together Total number of students in class 7th = 800 *
against New Zealand = 256 + 280 + 320 (12/100) = 96
= 856 Total number of students in class 8th = 800 *
Percentage increase = ((856 – 800)/800) * 100 (35/100) = 280
= 7% Total number of students in class 9th = 800 *
(18/100) = 144
4. Answer: C Total number of students in class 10th = 800 *
Runs scored by P against Australia = 64 (15/100) = 120
Runs scored by P against Pakistan = 48 Number of girls in class 6th = 500 * (25/100) =
Number of balls faced by P against Australia = 125
(64/80) * 100 Number of girls in class 7th = 500 * (18/100) = 90
= 80

Click Here For Bundle PDF Course | support@guidely.in Page 9 of 16


Bank Po Mains PDF Course 2024
Quantitative Aptitude Day -23 (Eng)

Number of girls in class 8th = 500 * (32/100) = Number of students from class 6th who has
160 participated in dance competition = 160 *
Number of girls in class 9th = 500 * (15/100) = 75 (60/100)
Number of girls in class 10th = 500 * (10/100) = = 96
50 Number of boys from class 6th who has
Number of boys in class 6th = 160 – 125 = 35 participated in dance competition = 35 * (3/5)
Number of boys in class 7th = 96 – 90 = 6 = 21
Number of boys in class 8th = 280 – 160 = 120 Number of girls from class 6th who has
Number of boys in class 9th = 144 – 75 = 69 participated in dance competition = 96 – 21
Number of boys in class 10th = 120 – 50 = 70 = 75
7. Answer: A
Number of students in class 8th = 280
Number of students in class 9th = 144
Number of students fail in class 8th = 280 *
(15/100)
= 42
Number of students fail in class 9th = 144 *
5. Answer: D
(25/100)
Number of boys in class 10th = 70
= 36
Number of girls in class 10th = 50
Number of students in class 9th in 2023 = (280 –
Let each girl in class 10 get Rs.x.
42) + 36
Each boy in class 10 gets Rs.1.5x.
= 274
Then, 70 * 1.5x + 50 * x = 62
105x + 50x = 62
8. Answer: C
155x = 62
Number of girls in class 6th = 125
x = 62/155
Number of girls in class 9th = 75
x = 0.40
Number of students in class 8th = 280
The amount got by each girl is Rs.0.40.
Number of students in class 10th = 120
Reqd. percentage = ((125 + 75)/(280 + 120)) *
6. Answer: B
100
Number of students in class 6th = 160
= (200/400) * 100
Number of boys in class 6th = 35
= 50%
Number of girls in class 6th = 125

Click Here For Bundle PDF Course | support@guidely.in Page 10 of 16


Bank Po Mains PDF Course 2024
Quantitative Aptitude Day -23 (Eng)

Direction (9 – 12): =4:5


Total marks scored by P, Q, R, and S together in Difference between marks scored by P and R =
Mathematics = 71.5 * 4 3*2=6
= 286 Let the marks scored by P be 4b and the marks
Total marks scored by P, Q, R, and S together in scored by Q be 5b.
Science = 72.75 * 4 Marks scored by R = 4b – 6
= 291 Then, 4b + 5b + 4b – 6 + 76 = 291
Total marks scored by P, Q, R, and S together in b = 17
English = 73 * 4 Marks scored by P in Science = 4 * 17 = 68
= 292 Marks scored by Q in Science = 5 * 17 = 85
Total marks scored by P, Q, R, and S together in Marks scored by R in Science = 68 – 6 = 62
Hindi = 64.75 * 4 The ratio of marks scored by P to marks scored
= 259 by Q in English = 140 : 100
Total marks scored by P, Q, R, and S together in =7:5
Computer = 64 * 4 Difference between marks scored by P and R =
= 256 4*2=8
The ratio of marks scored by P to marks scored Let the marks scored by P be 7c and marks
by Q in Mathematics = 70 : 100 scored by Q be 5c.
= 7 : 10 Marks scored by R = 7c – 8
Difference between marks scored by P and R = Then, 7c + 5c + 7c – 8 + 72 = 292
5 * 2 = 10 c = 12
Let the marks scored by P be 7a and the marks Marks scored by P in English = 7 * 12 = 84
scored by Q be 10a. Marks scored by Q in English = 5 * 12 = 60
Marks scored by R = 7a + 10 Marks scored by R in English = 84 – 8 = 76
Then, 7a + 10a + 7a + 10 + 84 = 286 The ratio of marks scored by P to marks scored
24a = 192 by Q in Hindi = 80 : 100
a=8 =4:5
Marks scored by P in Mathematics = 7 * 8 = 56 Difference between marks scored by P and R =
Marks scored by Q in Mathematics = 10 * 8 = 80 2*2=4
Marks scored by R in Mathematics = 7 * 8 + 10 = Let the marks scored by P be 4d and the marks
66 scored by Q be 5d.
The ratio of marks scored by P to marks scored Marks scored by R = 4d – 4
by Q in Science = 80 : 100 Then, 4d + 5d + 4d – 4 + 68 = 259

Click Here For Bundle PDF Course | support@guidely.in Page 11 of 16


Bank Po Mains PDF Course 2024
Quantitative Aptitude Day -23 (Eng)

13d = 195 = 72
d = 15 Average of marks scored by P, Q, R, and S
Marks scored by P in Hindi = 4 * 15 = 60 together in Computer = (54 + 90 + 48 + 64)/4
Marks scored by Q in Hindi = 5 * 15 = 75 = 64
Marks scored by R in Hindi = 60 – 4 = 56 Reqd. percentage = ((72 – 64)/64) * 100
The ratio of marks scored by P to marks scored = 12.5%
by Q in Computer = 60 : 100
=3:5 10. Answer: D
Difference between marks scored by P and R = Marks scored by R in Computer = 48
3*2=6 Passing marks in Computer = 48+12 = 60
Let the marks scored by P be 3e and the marks Maximum marks of Computer exam = 60 *
scored by Q be 5e. (100/40)
Marks scored by R = 3e – 6 = 150
Then, 3e + 5e + 3e – 6 + 64 = 256
11e = 198 11. Answer: A
e = 18 Marks scored by P in Hindi = 60
Marks scored by P in Computer = 3 * 18 = 54 Marks scored by R in Computer = 48
Marks scored by Q in Computer = 5 * 18 = 90 Marks scored by S in Mathematics = 84
Marks scored by R in Computer = 54 – 6 = 48 Marks scored by Q in Mathematics = 80
Marks scored by S in Computer = 64
Average of mark scored by P in Hindi, R in
Computer, and S in Mathematics = (60 + 48 +
84)/3
= 64
9. Answer: B
Reqd. percentage = (64/144) * 100
Marks scored by P in English = 84
= 44.44%
Marks scored by Q in English = 60
Marks scored by S in English = 72
12. Answer: C
Marks scored by P in Computer = 54
Marks scored by P in Hindi = 60
Marks scored by Q in Computer = 90
Marks scored by R in Hindi = 56
Marks scored by R in Computer = 48
Marks scored by S in Hindi = 68
Marks scored by S in Computer = 64
Marks Scored by P in English =84
Average of marks scored by P, Q, and S
Marks Scored by Q in English = 60
together in English = (84 + 60 + 72)/3

Click Here For Bundle PDF Course | support@guidely.in Page 12 of 16


Bank Po Mains PDF Course 2024
Quantitative Aptitude Day -23 (Eng)

Marks Scored by S in English = 72 Dell laptops sold in April = 819 – (234 + 195) =
The sum of marks scored by P, R, and S in Hindi 390
= 60 + 56 + 68 Lenovo laptops sold in May = 700 * (35/100)
= 184 = 245
Average of marks scored by P, Q, and S in HP laptops sold in May = 245 * (4/5)
English = (84 + 60 + 72)/3 = 196
= 72 Dell laptops sold in May = 700 – (245 + 196) =
Reqd. difference = 184 – 72 259
= 112

Direction (13 – 14):


Lenovo laptops sold in January = 660 * (40/100)
= 264
HP laptops sold in January = 264 * (87.5/100) 13. Answer: C
= 231 Number of Dell laptops sold in January = 165
Dell laptops sold in January = 660 – (264 + 231) Number of HP laptops sold in April = 195
= 165 Number of HP laptops sold in March = 128
Lenovo laptops sold in February = 576 * (1/3) Number of Lenovo laptops sold in February =
= 192 192
HP laptops sold in February = 192 * (3/4) Reqd. ratio = (165 + 195) : (128 + 192)
= 144 = 360 : 320
Dell laptops sold in February = 576 – (192 + 144) =9:8
= 240
Lenovo laptops sold in March = 704 * (1/4) 14. Answer: D
= 176 Number of Dell laptops sold in February = 240
HP laptops sold in March = 176 * (8/11) Number of Dell laptops sold in March = 400
= 128 Number of Dell laptops sold in January = 165
Dell laptops sold in March = 704 – (176 + 128) = Number of HP laptops sold in April = 195
400 Average of number of Dell laptops sold in
Lenovo laptops sold in April = 819 * (2/7) February and March together = (240 + 400)/2
= 234 = 320
HP laptops sold in April = 234 * (5/6)
= 195

Click Here For Bundle PDF Course | support@guidely.in Page 13 of 16


Bank Po Mains PDF Course 2024
Quantitative Aptitude Day -23 (Eng)

The sum of the number of Dell laptops sold in The ratio of investment P, Q, and R in business
January and HP laptops sold in April together = C = 25000 * 12 : 20000 * 12 : 30000 * 8
165 + 195 =5:4:4
= 360 Profit share of Q = 5681 * (4/13)
Reqd. percentage = (320/360) * 100 = 1748
= 88.88%
16. Answer: D
Direction (15 – 17): The ratio of investment P, Q, and R in business
The sum invested by R in business D = b A = 32000 * 9 : 24000 * 6 + 30000 * 6 : 28000 *
The sum invested by Q in business D = 3b/4 12
The sum invested by P in business D = 9b/8 = 24 : 27 : 28
The sum invested by Q in business C = a Total profit at the end of 1 year = 2472 * (79/24)
The sum invested by P in business B = 6a/5 = 8137
The sum invested by R in business A = 7a/5
The sum invested by P in business A = 4b/3 17. Answer: A
The sum invested by P in business A = 8a/5 The ratio of investment of P, Q, and R in
Then, 4b/3 = 8a/5 business D = 27000 * 12 : 18000 * 12 : 24000 *
a/b = 5/6 12
The sum invested by Q in business A = 6a/5 =9:6:8
The sum invested by R in business C = 3a/2 Profit received by P, Q, and R in business D
The sum invested by P in business C = 5a/4 after charity = 3348 * (23/9)
The sum invested by R in business B = a = 8556
The sum invested by Q in business B = a + 6000 Total profit occur in business D after 1 year =
According to the question, 8556 * (4/3)
5a/4 + a + 3a/2 = 75000 = 11408
a = 20000
b = 20000 * (6/5) = 24000 18. Answer: A
From I,
10 +(1)3 – 1 = 10
10 + (2)3 – 1 = 17
17 + (3)3 – 1 = 43
15. Answer: B 43 + (4)3 – 1 = 106
106 + (5)3 – 1 = 230

Click Here For Bundle PDF Course | support@guidely.in Page 14 of 16


Bank Po Mains PDF Course 2024
Quantitative Aptitude Day -23 (Eng)

230 + (6)3 – 1 = 445 200 + 6! = 920


The value of P is 230. The value of Q is 80.
From II, From III,
36 * 2 – 1 = 71 508 – (13)2 = 339
71 * 3 – 1 = 212 339 – (11)2 = 218
212 * 5 – 1 = 1059 218 – (9)2 = 137
1059 * 7 – 1 = 7412 137 – (7)2 = 88
7412 * 11 – 1 = 81531 88 – (5)2= 63
The value of Q is 212. 63 - (3)2 = 54
From III, The value of R is 88.
22 + 8 = 30 Hence, P > Q < R
30 + 16 = 46
46 + 32 = 78 20. Answer: C
78 + 64 = 142 From I,
142 + 128 = 270 38 + (2)2 = 42
270 + 256 = 526 42 + (3)2 = 51
The value of R is 142. 51 + (5)2 = 76
Hence, P > Q > R 76 + (7)2 = 125
125 + (11)2 = 246
19. Answer: B 246 + (13)2 = 415
From I, The value of P is 51.
81 + (2)2 = 85 From II,
85 + (4)2 = 101 18 * 1 – 0 = 18
101 + (6)2 = 137 18 * 2 – 1 = 35
137 + (8)2 = 201 35 * 3 – 2 = 103
201 + (10)2 = 301 103 * 4 – 3 = 409
The value of P is 101. 409 * 5 – 4 = 2041
From II, 2041 * 6 – 5 = 12241
47 + 1! = 48 The value of Q is 103.
48 + 2! = 50 From III,
50 + 3! = 56 24 + (1 * 2) = 26
56 + 4! = 80 26 + (2 * 3) = 32
80 + 5! = 200 32 + (3 * 4) = 44

Click Here For Bundle PDF Course | support@guidely.in Page 15 of 16


Bank Po Mains PDF Course 2024
Quantitative Aptitude Day -23 (Eng)

44 + (4 * 5) = 64 The value of R is 136.


64 + (5 * 6) = 94 Hence, P < Q < R
94 + (6 * 7) = 136

Click Here For Bundle PDF Course | support@guidely.in Page 16 of 16


Bank Po Mains PDF Course 2024
English Day - 23

English Language
Directions (1-3): In the following questions, two C. Only (A-D)
sentences are given in which some highlighted D. Only (B-D)
words are given, you have to identify if these E. No replacement required
words are used correctly or not. If not, then swap
word with another to make the sentence 3)
meaningful. I. The summit is being held largely (A) because
1) some member states, including India, were coup
I. The Dalai Lama, now 87, is recognized globally (B) to share a platform with Myanmar’s junta,
as a man of peace who has faced religious and which has shown no signs of moving the country
political-cultural conflict with great guidance(B), towards democracy.
patience, and non-violence. II. The summit on March 30 will be a virtual affair,
II. The Dalai Lama, who describes himself with virtually (C) because of concerns among member
pride as a “son of India”, has ensured the states about the situation in Myanmar following
intellectual(C) and emotional lineage of Lord last year’s military reluctant (D).
Buddha is flourishing under his poise (D). A. Both (A-C) and (B-D)
A. Both (A-D) and (B-C) B. Both (A-B) and (C-D)
B. Both (A-C) and (B-D) C. Only (A-C)
C. Only (B-C) D. Only (B-D)
D. Only (B-D) E. No replacement required
E. No replacement required
Directions (4-10): The passage given below
2) carries information based on which questions
I. Adolescents and young adults are particularly have been framed. You must read the same
vulnerable (A) to feeling complex (B) about their carefully and Answer: the questions that follow:
lives the more they use social media, according One consequence of lockdowns imposed across
to a study the world to cope with Covid was the quickening
II. The link between social media use and mental pace of economic transactions shifting from
well-being is clearly very worse(C); changes physical to digital marketplaces. Eighteen
within our bodies, such as brain development, months since the pandemic hit societies have
and in our social circumstances appear to make irrevocably moved towards more engagements
us susceptible (D) at particular times of our lives. online. However, regulatory architecture hasn’t
A. Both (A-C) and (B-D) kept pace. It has large gaps when it comes to
B. Both (A-D) and (B-C) dealing with digital markets. This isn’t new. The

Click Here For Bundle PDF Course | support@guidely.in Page 1 of 11


Bank Po Mains PDF Course 2024
English Day - 23

regulatory architecture always bagged There’s one key piece of the architecture that’s in
technological advances. But now we have limbo. The Personal Data Protection Bill was
reached a stage where the slow pace of introduced in Parliament in December 2019 and
regulatory retooling may have an adverse impact referred to a joint committee of both Houses.
on the nature of digital markets. After 66 sittings, a report still hasn’t come in. The
Digital markets have a set of unique features that inaction in regulatory space means that early-
make the need for a new regulatory architecture mover advantages accruing to some firms may
essential. They offer hitherto unavailable weaken the competitive nature of the market. Ad
economies of scale where following a high initial hoc regulations covering platforms in standalone
cost; incremental customers can be added at areas such as e-commerce may create new
practically no cost. This makes for the so-called distortions. A sector-specific approach is a bad
network effect: Increase in the number of idea. What India needs is a comprehensive
participants concurrently enhances the value of a umbrella legislation to cover digital platforms. A
service. Also, the ability to accumulate huge delay could lead to irreversible distortions.
amounts of data on users offers economies of 4) Which of the following has been mentioned as
scope inconceivable for a dominant firm in a the problem associated with major digital
traditional industry like steel or cement. To advancements in general?
illustrate, Amazon started as an online bookstore A. They have resulted as a major consequence
less than three decades ago and is now among of the lockdown imposed due to covid restrictions
the world’s top five firms by sales. B. They have unique requirements and hence
If unique features of digital markets allow for a the speed with which the digital marketplace
remarkable pace of growth, they also confer a grows is not proportional to the laws regulating
set of advantages to first movers that can them
potentially choke competition. In this context, the C. They come as advancements but create
danger comes from large digital platforms that problems in the beginning because of the not so
start off as mere intermediaries but later also tech-savvy citizens
compete against businesses using their platform. D. All digital platforms have unique features and
There’s an inherent conflict of interest in hence they take more time than needed for
simultaneously being player and referee. These people to adopt them
platforms or digital gatekeepers as they are E. Advancements are always good and they
referred to, have been the focus of standalone must be welcomed with open arms
laws. It’s an area where India’s regulatory
architecture is non-existent. 5) What has been described as a ‘conflict of
interest’ as per the passage?

Click Here For Bundle PDF Course | support@guidely.in Page 2 of 11


Bank Po Mains PDF Course 2024
English Day - 23

A. Large digital platforms that start off as mere III. Amazon is the world’s top six companies by
intermediaries later also compete against sale
businesses using their platform A. Only (ii)
B. A common law that will regulate all the digital B. Only (i)
platforms will be more of a compulsion than just C. Only (iii)
being regulators D. Both (i) and (iii)
C. The followers of the regulations becoming the E. None is correct
creators or the rules
D. Both (a) and (c) 9) What is the central idea being discussed by
E. None of the above the author?
A. There is a serious need for the government to
6) Which of the following is the best synonym for come up with data protection laws
the word ‘hitherto’ as used in the passage? B. New customers who are willing to come to
A. Subsequently digital platforms should be charged for the
B. Later services
C. Previously C. We need an umbrella law for platforms as
D. Unprecedented digital markets are fast expanding.
E. None of the above D. The gatekeepers should not convert their core
business into that of the payment gateways
7) Which word from the options can correctly E. None of the above
replace the word ‘bagged’ in the passage above?
A. Amazed 10) What do we understand by the network effect
B. Speculated that is being discussed by the author?
C. Sacked I. The first-movers have an advantage and that
D. Lagged makes it difficult for the companies to compete
E. Other than the given options II. The large amount of data helps the companies
to grow because of the huge customer base they
8) Which of the statements given below is correct already have developed.
as per the passage? III. Increase in the number of participants
I. Lockdowns have given rise to digitization concurrently enhances the value of a service.
II. After 65 sittings the report on the Bill has still A. Only (i)
not come into force B. Both (i) and (iii)
C. Only (iii)
D. None of the three

Click Here For Bundle PDF Course | support@guidely.in Page 3 of 11


Bank Po Mains PDF Course 2024
English Day - 23

E. All (i), (ii) and (iii) (I) Indian farming conditions are characteristically
______ by droughts and floods.
Direction (11-13): Two sentences I and (II) The book is overly long, and is ________ by a
sentences II are given with a blank in it. Choose considerable amount of repetition of the
a word from the given options that would fill both theoretical argument.
the blanks in the given sentences. A. nourished
11) B. deployed
(I) Increasing ____________ of groundwater is C.marred
lowering groundwater levels in most regions D. devoid
besides creating soil salinity. E. fragile
(II) The whole world has been mapped out into
oil areas and spheres of influence, and no corner Direction (14-16): In the following questions, a
of the earth can escape their exploration and sentence has been given in which a part of the
___________. sentence has been eliminated. From the given
A. demand options, choose the most appropriate alternative
B. elevation that fits correctly in the given sentence
C.exploitation grammatically and contextually. If none of the
D. appraisal alternatives is correct, choose option E as your
E. depreciation answer.
14) A woman suffered a panic attack after Air
12) India staff allegedly denied her entry at the
(I) The Jal Shakti Ministry focuses on the revival boarding gate of her flight.
and conservation of wetlands in the river basins ____________________ out of the airport instead
and _________ the alarming levels of river of calling for medical assistance.
pollution. A. However, as a responsible airline, we have to
(II) Technology is a contributory factor in adhere to rules laid down by the regulatory
_________ many problems. authorities
A. tackling B. But she began feeling better and declined any
B. alienating medical or wheelchair assistance.
C. measuring C. Further, her nephew alleged that the staff
D. rectify called security and forced the heart and diabetes
E. threatening patient

13)

Click Here For Bundle PDF Course | support@guidely.in Page 4 of 11


Bank Po Mains PDF Course 2024
English Day - 23

D. According to Air India, the woman and her two C. Naidu said Modi is impatient to slow progress
accompanying passengers reported to the and wants everything to be accelerated
boarding gate D. Modi was able to win polls due to his
E. None of these communication skills as he communicated with
the people and they had trust in him
15) Banned from leaving India by the E. None of these
Enforcement Directorate (ED), Bollywood actor
Jacqueline Fernandez has sought permission Directions (17-18): In the questions given below,
from a Delhi court _______________________. five sentences have been given which use words
A. was stopped by the immigration authorities at that have been highlighted. One of these
the Mumbai International Airport from going sentences uses a word that is unrelated to the
outside the country. rest. You are required to choose the same as
B. the Sri Lanka-born actor has also sought your answer.
permission to visit France and Nepal during the 17)
trip. A. And judging by the repeated glances the
C. who is being investigated by the ED in the Rs woman gave the sky, she too was cognizant of
200-crore extortion case involving. the failing light.
D. to travel to Abu Dhabi to attend the B. The new reporter was apprised of
International Indian Film Academy (IIFA) awards. developments in this story.
E. None of these C. Good reporters are keenly observant of
everything around them.
16) Asserting that even detractors of Prime D. I wasn't even conscious of what was
Minister Narendra Modi agree that he is a happening.
“phenomenon”, Vice President M Venkaiah E. Authors are famously ignorant about the
Naidu on Wednesday said that while Mahatma realities of publishing.
Gandhi took the freedom struggle out of the
hands of the elite to turn it into a mass 18)
movement, __________________. A. Other companies are in an even worse
A. Modi is fully conversant with the struggles of predicament than ourselves.
the poor and India had them in good numbers B. Opposition leaders this week implored the
when he was young president to break the deadlock.
B. Modi had converted developmental initiatives C. The police have announced a breakthrough
into mass movements in the murder case.

Click Here For Bundle PDF Course | support@guidely.in Page 5 of 11


Bank Po Mains PDF Course 2024
English Day - 23

D. Kate found herself in a dilemma when her two A. Only (i)


best friends were fighting. B. Only (ii)
E. The imbroglio seemed to be on the floor, on C. Both (a) and (b)
the other side of the table, near the wall. D. Only (iii)
E. Both (ii) and (iii)
Directions (19-20): In the questions given below,
two sentences have been given. These might be 20)
separate sentences or parts of the same  young students at the prestigious Institute of
sentence. Three starters below the same have International Relations here on Saturday
also been given. You are required to use those  President Kovind said India’s approach is
starters so that the given two sentences can be based on cooperation and collaboration and is
connected into one and choose the option with elaborated through the vision of SAGAR
the same as your answer. I. Interacting with…
19) II. Addressing…
 highest standards and commitment to ethical III. Speaking at…
conduct of anti-militancy operations, A. Only (i)
 the court martial proceedings were then B. Both (iii) and (ii)
initiated against Captain Bhoopendra Singh C. Only (ii)
I. According to… D. All (i), (ii) and (iii)
II. Maintaining its… E. Both (i) and (ii)
III. However…
Click Here to Get the Detailed Video Solution for the above given Questions
Or Scan the QR Code to Get the Detailed Video Solutions

Answer Key with Explanation


1) Answer: D

Click Here For Bundle PDF Course | support@guidely.in Page 6 of 11


Bank Po Mains PDF Course 2024
English Day - 23

(B-D) – ‘Poise’ means – marked by balance or 3) Answer: A


equilibrium, according to the context of sentence (A-D) – ‘The summit is being held’, the context
(I), Dalai Lama, faced conflicts with great explains where the meeting held, ‘largely’ is
balance. And in sentence (II), the right context is incorrect, instead of ‘largely’, ‘virtually’ will be
‘Buddha is flourishing under his guidance’ used
The right sentence is - (I) - The Dalai Lama, now (B-D) – ‘Military coup’ is the right context, which
87, is recognized globally as a man of peace means – the sudden, violent overthrow of an
who has faced religious and political-cultural existing government by a small group
conflict with great poise, patience, and non- The right sentence is - (I) - The summit is being
violence. held virtually because some member states,
(II) - The Dalai Lama, who describes himself with including India, were reluctant to share a
pride as a “son of India”, has ensured the platform with Myanmar’s junta, which has shown
intellectual and emotional lineage of Lord no signs of moving the country towards
Buddha is flourishing under his guidance. democracy.
(II) - The summit on March 30 will be a virtual
2) Answer: B affair, largely because of concerns among
(A-D) – According to the context of sentence (I), member states about the situation in Myanmar
adolescents and young adults are susceptible to following last year’s military coup.
the use of social media.
Meaning of ‘susceptible’ – likely to be influenced 4) Answer: B
(B-C) – ‘Very worse’ is grammatically incorrect, it Read the lines give below to get a closer
should be ‘very complex’ understanding of the correct Answer::
The right sentence is - (I) - Adolescents and However, regulatory architecture hasn’t kept
young adults are particularly susceptible to pace. It has large gaps when it comes to dealing
feeling worse about their lives the more they use with digital markets. This isn’t new. The
social media, according to a study regulatory architecture always lagged
(II) - The link between social media use and technological advances.
mental well-being is clearly very complex; Option (b) explains the lines highlighted above to
changes within our bodies, such as brain explain the same as the correct Answer: option.
development and puberty, and in our social
circumstances appear to make us vulnerable at 5) Answer: A
particular times of our lives Large digital platforms that start off as mere
intermediaries but later also compete against

Click Here For Bundle PDF Course | support@guidely.in Page 7 of 11


Bank Po Mains PDF Course 2024
English Day - 23

businesses using their platform. There’s an (iii) Amazon is world’s top five should be correct
inherent conflict of interest in simultaneously So, option (b) is the correct choice.
being player and referee.
Option (a) encapsulates the meaning of the 9) Answer: C
highlighted line as the reason for the required Points (a) and (d) are given in the passage but
question. they cannot be called as the complete central
Hence, option (a) should be correct. idea in themselves. They are simply aspects that
are covered under the central idea.
6) Answer: C The actual issue around which almost all the
The line where the word has been used talks paragraphs are revolving is that one common
about the ‘economies of scale’ in the digital law that would regulate all the digital platforms in
spectrum that were previously unavailable or the country before it is too late.
missing until the point in time under discussion. Option (b) is not directly being discussed so can
Of the given options (a) and (b) will be the be ruled out.
antonyms of the meaning explained. Hence, option (c) is the correct Answer:
So, option (c) fits the best…
10) Answer: E
7) Answer: D The link of the context goes like this:
Bagged  succeed in securing something The digital businesses have an advantage
(which is not the case here) because;
Amazed and sacked are both out of context so  The number of users/customers
can be eliminated. increases
The line where the word is used means that the  The companies gain a large database
regulatory advancements are not able to match  The existing companies have a huge
their pace with the technological advancements. database already which makes it difficult
So, ‘lagged: slowed down, will be the correct for the new companies to compete as the
usage and hence, option (d) will be correct. existing ones already have a good
number of customers.
8) Answer: B Looking at all the points above, we can conclude
Of the three statements just the first statement that, all the given points are correct.
goes well as it is given at various places in the So, option (e) is the correct Answer:.
passage.
(ii) After 66 sittings should be correct 11) Answer: C

Click Here For Bundle PDF Course | support@guidely.in Page 8 of 11


Bank Po Mains PDF Course 2024
English Day - 23

● Exploitation means “the action of making ● Rectify means “put right”. Rectify cannot
use of and benefiting from resources.” be the answer, as a verb of continuous
● In sentence I, exploitation is used to refer tense is required.
to the usage of groundwater. ● Threatening means “having a hostile or
● In sentence II, exploitation is used to refer deliberately frightening quality or manner”.
to the usage of oil areas. ● Alienating, measuring and threatening are
● Demand means “require; need” contextually incorrect.
● Elevation means “the action or fact of
raising or being raised to a higher or more 13) Answer: C
important level, state, or position.” ● Marred means “to ruin or diminish the
● Appraisal means “an act of assessing perfection or wholeness of; spoil”
something or someone.” ● Sentence I says Indian farming conditions
● Depreciation means “the reduction or are spoiled.
underestimation of the worth or ● Sentence II says that the theoretical
importance of something.” argument spoiled the book.
● Except option C other options are ● Nourished means “provide with the food
contextually incorrect. or other substances necessary for growth,
health, and good condition.”
12) Answer: A ● Deployed means “move (troops or
● Tackling means “make determined efforts equipment) into position for military
to deal with (a problem or difficult task)”. action.”
● In sentence I, ‘tackling’ is used to refer to ● Devoid means “entirely lacking or free
the efforts to deal with the alarming levels from.”
of river pollution. ● Fragile means “(of an object) easily
● Sentence II says, technology is used to broken or damaged.”
deal with many problems.
● Alienating means “cause (someone) to 14) Answer: C
feel isolated or estranged”. ● The above given paragraph is about a
● Measuring means “estimate or assess the woman who had a panic attack at Delhi
extent, quality, value, or effect of airport after she was allegedly denied
(something).” entry at the boarding gate of her flight by
Air India staff. She was asked to leave the
airport instead of being offered medical

Click Here For Bundle PDF Course | support@guidely.in Page 9 of 11


Bank Po Mains PDF Course 2024
English Day - 23

assistance, as per the nephew's account. informed about things. ‘Observant’ and
Only option C carries the theme of the ‘Conscious’ are synonyms of ‘cognizant’ and
paragraph aptly, hence option C is the ‘apprised’. The word ‘ignorant’ however is the
correct answer. opposite of the given words as it means being
unaware of things.
15) Answer: D Therefore, the best answer is option (e).
● The above given paragraph is about a
bollywood actor Jacqueline Fernandez 18) Answer: C
who sought permission from a Delhi court Predicament is a difficult, unpleasant, or
to travel to Abu Dhabi to attend the embarrassing situation.
International Indian Film Academy A deadlock means to come to a point where no
awards. So only option D carries the progress can be made because of fundamental
theme of the paragraph aptly, hence disagreement.
option D is the correct answer. A dilemma is a difficult situation or problem.
An imbroglio an extremely confused,
16) Answer: B complicated, or embarrassing situation.
● The above given paragraph is about our A breakthrough is a sudden discovery that
Vice President M Venkaiah Naidu solves a problem. This word is different from the
releasing the book “Modi@20 Dreams others as all the other words are synonyms of
Meet Delivery” on Wednesday in New each other.
Delhi. Where he compares two great
leaders. He said that Modi’s dreams and 19) Answer: B
Mission India have been shaped by his The sentence is of the tone that the trend of
extensive travels and insightful, court martial continues with this Bhoopendra
experiential journey. So option B carries Singh issue. The connected sentence looks like;
the theme of the paragraph aptly, Maintaining its highest standards and
remaining options have no direct commitment to ethical conduct of anti-militancy
connection with the given paragraph operations, the court martial proceedings were
hence option B is the correct answer. then initiated against Captain Bhoopendra
Singh.
17) Answer: E Therefore, the best is to go with option (b).
Being aware or knowing things is what is meant
by being ‘cognizant’. ‘Apprised’ means being 20) Answer: E

Click Here For Bundle PDF Course | support@guidely.in Page 10 of 11


Bank Po Mains PDF Course 2024
English Day - 23

The sentences can best be connected with the Addressing young students at the prestigious
help of the first two starters only and not the third Institute of International Relations here on
as the third is grammatically incorrect. Saturday, President Kovind said India’s
The sentences after joining will look like: approach is based on cooperation and
Interacting with young students at the prestigious collaboration and is elaborated through the
Institute of International Relations here on vision of SAGAR.
Saturday, President Kovind said India’s Therefore, the best answer that should be
approach is based on cooperation and marked is option (e).
collaboration and is elaborated through the
vision of SAGAR.

Click Here For Bundle PDF Course | support@guidely.in Page 11 of 11


Bank Po Mains PDF Course 2024
Reasoning Day -24 (Eng)

Reasoning Aptitude
Directions (1-5): Study the following information a) 27
carefully and answer the below questions b) 25
Eleven chocolate boxes- Lindor, Cadbury, c) 24
Nestle, Galaxy, Lindt, Snickers, Toblerone, Twix, d) 26
Mars, Kinder and Guylian are kept in twelve e) 22
different shelves but not necessarily in the same
order. The lowermost shelf is numbered as 1, 2) Which of the following statement is true
shelf just above it is numbered as 2 and so on till according to the Vacant shelf?
the topmost shelf isnumbered as 12. One shelf is I) No boxes kept between Vacant and Twix box
Vacant. II) Vacant shelf is three shelves below the box
Twix is kept two shelves above the shelf which is which is kept two boxes above Mars
immediately above Cadbury which is kept at odd III) The vacant shelf is kept three shelves above
number shelf. The number of shelves kept above Kinder box
Twix is one more than the number of shelves IV) Even number shelf is vacant
kept below Snickers which is kept at an even a) Only (III) and (IV)
number shelf. The Galaxy box is kept two b) Only (I) and (II)
shelves above the shelf which is three shelves c) Only (II) and (III)
below Lindor. The number of boxes kept d) Only (II)
between Lindor and Twix is one more than the e) All (I), (II) and (III)
number of boxes kept between Cadbury and
Snickers. The Galaxy box is neither kept 3) As many boxes kept between Lindor and
adjacent to Twix box nor the Snickers box. Snickers is one more than the number of boxes
Nestle box is kept three shelves below the shelf kept between Toblerone and _____?
which is immediately above Vacant shelf. The a) Cadbury
number of boxes kept below Toblerone is one b) The box which is kept on the fourth shelf
less than the number of boxes kept above c) The box which is kept on the seventh shelf
Guylian. Guylian box does not kept adjacent to d) The box which is kept immediately below
Snickers. Guylian box is kept above Toblerone Galaxy box
box. The sum of the shelf number of Nestle and e) The box which is kept two boxes below
Kinder is equal to the shelf number of Guylian. Snickers box
Mars does not kept at odd number shelf.
1) What is the sum of the shelf number of the 4) Which of the following pair of chocolate boxes
Mars, Kinder, and Galaxy boxes? are kept on an adjacent shelf of Vacant shelf?

Click Here For Bundle PDF Course | support@guidely.in Page 1 of 10


Bank Po Mains PDF Course 2024
Reasoning Day -24 (Eng)

a) Lindt, Guylian three months before Q’s only male child. P does
b) Kinder, Mars not have more than three kids. O is neither
c) Lindt, Snickers spouse of T nor parent of N. P is not spouse of Q
d) Mars, Twix both O. At least one person renewals license
e) Nestle, Toblerone after the third generation persons. S is the only
daughter of the one who is renewal immediately
5) How many boxes are kept between the box before S who is not child of Q. The number of
which is kept immediately above Kinder and the persons renewing before Q’s father-in-law is one
box which is kept on the second shelf? less than the number of persons renewing after
a) More than four R’s daughter. The number of months between
b) Three R’s spouse and M’s nephew is three less than
c) Four the number of months between N’s father and
d) One S’s Cousin
e) Two 6) How many months between the one who
renewals immediately before R’s spouse and the
Directions (6-10): Study the following information one who is T’s sister?
carefully and answer the given questions. a) Two
Nine persons- L, M, N, O, P, Q, R, S and T b) One
belong to the same family of three generations c) Three
and they renewal their license in different d) Four
months- January, March, April, May, June, e) Five
August, September, November and December of
the same year but not necessarily in the same 7) As many person renewals before P’s spouse
order. is the same as the number of person renewals
T is the daughter of O’s father-in-law and after ____?
renewals in the month which has 31 days. R a) M
renewals three persons after the one who b) The one who renewals in March
renewals two months before T. P is the mother- c) The one who renewals in November
in-law of the one who is the father of N. M is not d) T’s spouse
a married person. The number of persons e) The one who renewals immediately before R
renewals before R is one more than the number
of person renewals after the only brother of T. T 8) How S is related to the one who renewals in
is the Sister-in-law of R’s spouse. M is the only April and which month does M’s niece renew the
brother of N’s mother and T’s sister. L renewals license?

Click Here For Bundle PDF Course | support@guidely.in Page 2 of 10


Bank Po Mains PDF Course 2024
Reasoning Day -24 (Eng)

a) Daughter, September 11) What is the conclusion of the statement


b) Niece, January N<L=S≤O; E>M≥S; E<V=K≤J; R>L?
c) Cousin, November
d) Daughter, June
e) Niece, September

a) Both I and VI follow


9) Which of the following statement is true?
b) Both II and VI follow
a) More than three-persons renewal license
c) Both III and IV follow
before N
d) Both III and V follow
b) No one renewal between O’s spouse and L’s
e) Both I and IV follow
spouse
c) P renewals in December
12) What is the conclusion of statement
d) T is the daughter of the one who renewed in
J>E≥B≥M; N>X<T; F>N≤W≤M; X>Y=A?
March
e) All the statement is true

10) How many female members renewed their


licenses in a month which has an even number a) Both I and VI follow

of days? b) Both II and VI follow

a) Two c) Both III and IV follow

b) One d) Both III and V follow

c) None e) Both I and IV follow

d) Three
e) More than three 13) What is the conclusion of statement
M>O≥B≥Z; G<E≤S ; B>N≤D≤J; S<F=O?

Directions (11-15): Each of the questions below


consists of two columns and three statements
are given you have to decide which of the
following combination are to be the conclusion of a) Both I and VI follow
given statements. The statements given below in b) Both II and VI follow
column 1 can be used along with the statement c) Both III and IV follow
given below in column 2. No statement is d) Both III and V follow
combined together in the same column. e) Both I and IV follow

Click Here For Bundle PDF Course | support@guidely.in Page 3 of 10


Bank Po Mains PDF Course 2024
Reasoning Day -24 (Eng)

14) What is the conclusion of statement c) Both I and II


N=O≤B<Z; A<R≤M ; B<Q≤P≤W; A>L≥O? d) Both II and III
e) None

17) If all the letters in the given words are


arranged in alphabetical order from the left end,
a) Both I and VI follow then which of the following first letter of I from the
b) Both II and VI follow left end, the second letter of II from the left end,
c) Both III and IV follow and the third letter of III from the left end are
d) Both III and V follow remaining unchanged in its original position?
e) Both I and IV follow I. K I Y L G F
II. D N H T A Z
15) What is the conclusion of statement III. L C I R T D
X<Y≥S>W; I<A≤S; R<F<O≤Z; Y≤K≥Z? a) Only I
b) Both II and III
c) Only III
d) All I, II, and III
e) None
a) Both I and VI follow
b) Both II and VI follow
18) If the middle letter of all the words is taken
c) Both III and IV follow
and forms a meaningful word, then which of the
d) Both III and V follow
following does not form a meaningful word?
e) Both I and IV follow
a) Cache, Carry, Modem, Alarm
b) Beach, Micro, Refer, Alter
16) If in the given numbers all the digits are
c) Delay, Rapid, Rifle, Defer
arranged in ascending order with in the number
d) Apart, Alpha, Align, Candy
from the left end, then which of the following
e) Amend, Doubt, Early, False
number has more than two digits are unchanged
in its position?
19)If all the vowel letters in the word “DEPTH”
I. 28695423
are changed to the next letter in the alphabetical
II. 69826481
series and all the consonants are changed to the
III. 42862983
previous letter in the alphabet series. In the next
a) Only III
step, all the letters are changed to the next letter
b) Only II
in the alphabet series until each letter is not the

Click Here For Bundle PDF Course | support@guidely.in Page 4 of 10


Bank Po Mains PDF Course 2024
Reasoning Day -24 (Eng)

same as the original letters in the word. How number, then which of the following group of
many steps to complete the process? number has more than one even number?
a) Four I. 2624, 3647, 1654
b) Six II. 5263, 6843, 2576
c) Three III. 7365, 3153, 5438
d) Five a) Only I
e) Two b) Both I and II
c) Only II
20) If ‘1’ is added to the digit at the even position d) All I, II, and III
in the given group of number from the left end of e) None
each number, then added all the digits within the
Click Here to Get the Detailed Video Solution for the above given Questions
Or Scan the QR Code to Get the Detailed Video Solutions

Answer Key with Explanation


Directions (1-5):
1) Answer: C
2) Answer: B
3) Answer: E
4) Answer: D
5) Answer: C
Final Arrangement

Click Here For Bundle PDF Course | support@guidely.in Page 5 of 10


Bank Po Mains PDF Course 2024
Reasoning Day -24 (Eng)

 The Galaxy box is kept two shelves above the


shelf which is three shelves below Lindor.
 The number of boxes kept between Lindor and
Twix is one more than the number of boxes kept
between Cadbury and Snickers.
 The Galaxy box is neither kept adjacent to
Twix box nor the Snickers box.
From the above condition, there are four
possibilities.

We have,
 Twix is kept two shelves above the shelf which
is immediately above Cadbury which is kept at
odd number shelf.
 The number of shelves kept above Twix is one
more than the number of shelves kept below
Snickers which is kept at an even number shelf.
From the above condition, there are three Again we have,
possibilities.  Nestle box is kept three shelves below the
shelf which is immediately above Vacant shelf.
 The number of boxes kept below Toblerone is
one less than the number of boxes kept above
Guylian.
 Guylian box does not kept adjacent to
Snickers.
 Guylian box is kept above Toblerone box.
From the above condition, case 3, Case-4, and
Case-1a get eliminated.
Again we have,

Click Here For Bundle PDF Course | support@guidely.in Page 6 of 10


Bank Po Mains PDF Course 2024
Reasoning Day -24 (Eng)

9) Answer: D
10) Answer: A
Final Arrangement

Again we have,
 The sum of the shelf number of Nestle and
Kinder is equal to the shelf number of Guylian.
 Mars does not kept at odd number shelf.
From the above condition, case-1b gets
eliminated. Case-1 shows the final arrangement.

We have,
 T is the daughter of O’s father-in-law and
renewals in the month which has 31 days.
 R renewals three persons after the one who
renewals two months before T.
 P is the mother-in-law of the one who is the
father of N.
 The number of persons renewals before R is
one more than the number of persons renewals
after the only brother of T.
From the above condition, there are three
possibilities.
Directions (6-10):
6) Answer: B
7) Answer: D
8) Answer: C

Click Here For Bundle PDF Course | support@guidely.in Page 7 of 10


Bank Po Mains PDF Course 2024
Reasoning Day -24 (Eng)

Again we have,
Again we have,  S is the only daughter of the one who
 T is the Sister-in-law of R’s spouse. renewals immediately before S who is not
 M is not a married person. child of Q.
 M is the only brother of N’s mother and  The number of persons renewing before
T’s sister. Q’s father-in-law is one less than the
 L renewals three months before Q’s only number of persons renewing after R’s
male child. daughter.
 P does not have more than three kids. Since, only three generations are in the
 O is neither spouse of T nor parent of N. family, and S is the daughter of O, thus L
 At least one person renewal after the third must be married to P.
generation persons.  The number of months between R’s
 P is not spouse of Q both O. spouse and M’s nephew is three less than
From the above condition, there are five the number of months between N’s father
possibilities. and S’s Cousin.
From the above condition, Case-1, Case-3,
Case-1a and Case-3a get eliminated. Case-2
shows the final arrangement.

Click Here For Bundle PDF Course | support@guidely.in Page 8 of 10


Bank Po Mains PDF Course 2024
Reasoning Day -24 (Eng)

F≥B(F=O≥B)
F≥Z(F=O≥B≥Z)
O>N(O≥B>N)

14) Answer: C
Both III and IV follow
O<P (O≤B<Q≤P)
R>N (R>A>L≥O=N)
L≥N(L≥O=N)
O<A (A>L≥ O)

15) Answer: D
Both III and V follow
Y≥A(Y≥S≥A)
S≤K(K≥Y≥S)
K>R(K≥Z≥O>F>R)
11) Answer: A I<Y(Y≥S≥A>I)
Both I and VI follow
N<E (N<L=S≤M<E) 16) Answer: E
L≤M (L=S≤M) I. 2869542322345689
R>N (N<L<R) II. 6982648112466889
S<J (S≤M<E<V=K≤J) III. 4286298322346889

12) Answer: B 17) Answer: C


Both II and VI follow I. K I Y L G FF G I K L Y
E≥N(N≤W≤M≤B≤E) II. D N H T A ZA D H N T Z
F>Y(F>N> X>Y) III. L C I R T D C D I L R T
W>X(X<N≤W)
T>A(T>X>Y=A) 18) Answer: C
a) Cache, Carry, Modem, AlarmCRDACard
13) Answer: C b) Beach, Micro, Refer, AlterACFTFact
Both III and IV follow c) Delay, Rapid, Rifle, DeferLPFF
M>S(M>O=F>S) d) Apart, Alpha, Align, CandyAPINPain

Click Here For Bundle PDF Course | support@guidely.in Page 9 of 10


Bank Po Mains PDF Course 2024
Reasoning Day -24 (Eng)

e) Amend, Doubt, Early, FalseEURLRule 20) Answer: D


I. 2624, 3647, 1654 2725, 3748, 175516,
19) Answer: A 22, 18
DEPTH II. 5263, 6843, 25765364, 6944, 267718, 23,
1. CFOSG 22
2. DGPTH III. 7365, 3153, 54387466, 3254, 5539 23,
3. EHQUI 14, 22
4. FIRVJ

Click Here For Bundle PDF Course | support@guidely.in Page 10 of 10


Bank Po Mains PDF Course 2024
Quantitative Aptitude Day -24 (Eng)

Quantitative Aptitude

Direction (1-4): Study the following data carefully and answer the questions:
Three persons A, B and C have to type a book in five days Mon, Tue, Wed, Thu and Fri of a certain
week. Each of them typed a different number of pages on each of the given days.
Table given below shows the data related to number of pages typed by A, B and C in the given days.

Note:
1: Some data in the table is missing, which needs to be calculated in the questions.
2: The missing data can be different for each question.
1) If number of pages typed by B on Thu is one- Q: Total number of pages typed by C on Tue and
third of that typed by C on Thu and number of Wed together.
pages typed by B on Mon is 140% more than R: Ratio of the number of pages typed by A on
that typed by B on Thu, then find the number of Wed to that typed by C on Wed.
pages typed by C on Mon. a) Only P and Q
a) 16 b) Only Q and R
b) 21 c) None can be determined
c)18 d) Only P and R
d) 15 e) All P, Q and R
e) 12
3) If number of pages typed by B till Thu is 60%
2) If number of pages typed by B till Tue is 18, of that typed by A till Thu and ratio of number of
then find that which of the following can be pages typed by B on Fri to that typed by C on Fri
determined? is 2: 1, then find the average of number of pages
P: Difference between the number of pages typed by A and C till Fri?
typed by A on Wed and Thu. a) 84

Click Here For Bundle PDF Course | support@guidely.in Page 1 of 16


Bank Po Mains PDF Course 2024
Quantitative Aptitude Day -24 (Eng)

b) 80 of classes A and B are 52 kg and 48 kg, then find


c)82 the average weight of all the boys of classes A
d) 86 and B?
e) 88 a) 51.5 kg
b) 55.5 kg
4) If number of pages typed by A on Mon is ___ c)53.5 kg
and number of pages typed by A till Tue is 30, d) 58.5 kg
then number of pages typed by C on Tue is 1 e) 52.5 kg
more than that typed by C on Thu.
Find the value, which can be filled in the blank? 6) Find that which of following statements is /are
a) 26 true?
b) 18 P: Average weight of all the students of classes
c) 30 B and C is more than 50 kg.
d) 20 Q: Difference between number of students in
e) 24 classes C and D is 15.
R: Ratio of average weight of class B to that of
Direction (5-8): Study the following data carefully class C is 10: 11.
and answer the questions: a) Only P and R
Data given below is related to the number of b) Only Q
students and their average weight of 4 different c)None is true
classes A, B, C and D in a school. d) All are true
Ratio of number of students in class A to those in e) Only Q
class D is 3: 5 and ratio of number of students in
class B to those in class C is 4: 3. Total weight of 7) If ratio of boys to girls in class D is 8: 7 and
class A is 2430 kg, which is 60.75% of total average weight of girls in class D is 50 kg, then
weight of class B. Ratio of total weight of class C find the difference between average weight of
to total weight of class D is 11: 12 and difference boys and that of girls in class D?
between total weight of classed C and D is 300 a) 3.75 kg
kg. Ratio of average weight of class A to average b) 2.75 kg
weight of class B is 27: 25 and average weight of c) 6.75 kg
class C is 7 kg more than the average weight of d) 5.75 kg
class D. e) 4.75 kg
5) If number of girls in classes A and B are 25
and 40 respectively and average weight of girls

Click Here For Bundle PDF Course | support@guidely.in Page 2 of 16


Bank Po Mains PDF Course 2024
Quantitative Aptitude Day -24 (Eng)

8) In class C, when a student of weight 45 kg left a) 6


the class and another student of weight ‘M’ kg b) 8
joined the class, average weight of the class c)2
increased by 0.5 kg. Find the value of [20% of M d) 5
+ 16(2/3)% of (M – 15)]1/2? e) 4

Direction (9-12): Study the following data carefully and answer the questions:
There are 5 mixtures of milk and water A, B, C, D and E and each mixture contains different quantities of
milk and water.
Table given below shows the following data.

Note:
1: Some data in the table is missing, which needs to be calculated in the questions.
2: Ratio of quantity of water in mixture B to that in mixture D is 5: 4.
9) If percentage of milk in mixture B is M% and what per cent more/less than the cost of mixture
percentage of milk in mixture D is N%, then find A?
the ratio of M to N? a) 16.67%
a) 9: 8 b) 15%
b) 21: 20 c)20%
c)18: 17 d) 12.5%
d) 16: 15 e) 25%
e) 12: 11
11) When mixtures B and C are mixed together,
10) If total quantity of mixture B is 90% of that of the cost of new mixture becomes ₹ 43.2 per L.
mixture E, then find that the cost of mixture E is Find the total quantity of mixture C.
a) 36 L

Click Here For Bundle PDF Course | support@guidely.in Page 3 of 16


Bank Po Mains PDF Course 2024
Quantitative Aptitude Day -24 (Eng)

b) 30 L a) ₹ 37.9 per L
c) Data inadequate b) ₹ 41.9 per L
d) 24 L c)₹ 33.9 per L
e) 42 L d) ₹ 45.9 per L
e) ₹ 47.9 per L
12) When mixtures A, B and D are mixed
together, then what will be the cost of the new
mixture?

Directions (13–16): Study the following information carefully and answer the questions given below:
Graph given below shows the number of students who cleared the exam, number of students who failed
the exam, and number of students who are in the waitlist of the exam in five different years. The students
which are in the waitlist will either clear the exam or will fail in the exam later on.
Note:
1. Total number of students = Number of students who cleared (fresh) the exam + Number of students
who failed (fresh) the exam + Number of students who are in the waitlist in the exam
2. Number of students who are in waitlist in the exam = Number of students who cleared (from waitlist)
the exam + Number of students who failed (from waitlist) in the exam.
3. Total number of students who cleared the exam = Number of students who cleared (fresh) the exam +
Number of students who cleared (from waitlist) the exam
4. Total number of students who failed the exam = Number of students who failed (fresh) the exam +
Number of students who failed (from waitlist) the exam
5. All the values (except the percent) in the below graph are in ’00.

Click Here For Bundle PDF Course | support@guidely.in Page 4 of 16


Bank Po Mains PDF Course 2024
Quantitative Aptitude Day -24 (Eng)

Some pointers regarding above graph:


 Ceiling of the box shows the number of students who cleared (fresh) the exam.
 Floor of the box shows the number of students who are in the waitlist of the exam.
 Ceiling of the line shows the percent number of students who cleared the exam (from waitlist) out
of total students who cleared (fresh) the exam.
 Floor of the line shows the number of students who failed (fresh) the exam.
13) What is the ratio of total number of students B: Square of P is a perfect cube of an integer.
who cleared from (both fresh and waitlist) the C: Total factors of P are 6.
exam in 2014 to the number of students who a) Only B
cleared (fresh) the exam in 2017? b) All A, B, and C
a) 19: 15 c) Only A and B
b) 29: 25 d) Only C
c) 39: 35 e) Only B and C
d) 21: 17
e) 16: 11 15) If out total students appeared for the exam in
2015 and 2017, A% cleared (from fresh) the
14) If total number of students who failed (from exam and B% cleared (from waitlist) the exam
both fresh and waitlist) in the exam in 2015 is respectively, find the value of A + B.
P% of total students who appeared for the exam a)
in that year, find which of the following b)
statement(s) regarding ‘P’ is/are not TRUE? c)
A: P is a multiple of both 4 and 5. d)

Click Here For Bundle PDF Course | support@guidely.in Page 5 of 16


Bank Po Mains PDF Course 2024
Quantitative Aptitude Day -24 (Eng)

e) cleared (from both fresh and waitlist) the exam in


that year what percent are boys?
16) If ratio of boys to girls who cleared (from a)
fresh) the exam in 2016 is 8: 7 and ratio of boys b) 36%
to girls who cleared (from waitlist) the exam in c)
that year is 2: 7, then out of total students who d) 30%
e) 25%

Directions (17–20): Study the following information carefully and answer the questions given below:
There are five phones which have different number of apps in that. Apps can be either system or non-
system apps. Out of non-system apps, apps can be either gaming or payment or social media. Pie chart
given below shows the degree distribution of number of system apps in all the five phones.

Table given below shows the ratio of number of gaming to social media to the payment apps in all the five
phones. It also shows the total number of apps in all the five phones.

Click Here For Bundle PDF Course | support@guidely.in Page 6 of 16


Bank Po Mains PDF Course 2024
Quantitative Aptitude Day -24 (Eng)

Note:
1. Total apps = System Apps + Non-system Apps
2. Non-system apps = Gaming apps + Social media apps + Payment apps
3. Some data in the above table is missing that need to be calculated.
4. Number of gaming apps in phone A and number of payment apps in phone D is same which is half of
the number of payment apps in phone B.
17)What percent of total apps in phones B, D, 19) Sum of total gaming apps and payment apps
and E together are system apps? together in phone B is what percent of total non-
a) 30% system apps in that phone?
b) 20% a)
c) 40% b)
d) 25% c)
e) 35% d)
e) None of these
18) If average of total apps in all the five phones
is 54, then how many integer values the variable
20) If total number of social media apps in all the
‘y’ can take and which of the following can never
five phones together is 65, then find the average
be the value of ‘y’?
number of payment apps in all the five phones
a) 6 and 14
together.
b) 7 and 14
a) 15
c) 6 and 77
b) 16
d) 7 and 77
c) 17
e) 6 and 49
d) 18
e) 19

Click Here For Bundle PDF Course | support@guidely.in Page 7 of 16


Bank Po Mains PDF Course 2024
Quantitative Aptitude Day -24 (Eng)

Click Here to Get the Detailed Video Solution for the above given Questions
Or Scan the QR Code to Get the Detailed Video Solutions

Answer Key with Explanation


1) Answer: A Number of pages typed by A on Wed = 50 – 30 =
Number of pages typed by C till Wed = 2 × 48 – 20
50 = 46 Number of pages typed by A on Thu = 75 – 50 =
Number of pages typed by C till Thu = 2 × 68 – 25
75 = 61 Required difference = 25 – 20 = 5
Number of pages typed by C on Thu = 61 – 46 = From Q:
15 Since, we can’t determine the number of pages
Number of pages typed by B on Thu = 15/3 = 5 typed by C on Tue.
Number of pages typed by B on Mon = 240% of So, we can’t determine the total number of
5 = 12 pages typed by C on Tue and Wed together.
Number of pages typed by A on Mon = 12 × From R:
(300/200) = 18 Number of pages typed by A on Wed = 50 – 30 =
Number of pages typed by C on Mon = 2 × 17 – 20
18 = 16 Number of pages typed by C till Wed = 2 × 48 –
50 = 46
2) Answer: D Number of pages typed by C till Tue = 26
Number of pages typed by B till Tue = 18 Number of pages typed by C on Wed = 46 – 26
Number of pages typed by A till Tue = 18 × = 20
(100/60) = 30 Required ratio = 20: 20 = 1: 1
Number of pages typed by C till Tue = 2 × 28 – Hence, only P and R can be determined.
30 = 26
From P: 3) Answer: B

Click Here For Bundle PDF Course | support@guidely.in Page 8 of 16


Bank Po Mains PDF Course 2024
Quantitative Aptitude Day -24 (Eng)

Number of pages typed by A till Thu = 75 Let the number of students in classes A and D
Number of pages typed by B till Thu = 60% of 75 are 3x and 5x respectively.
= 45 Also let the number of students in classes B and
Number of pages typed by B till Fri = 70% of 90 C are 4y and 3y respectively.
= 63 Total weight of class A = 2430 kg
Number of pages typed by B on Fri = 63 – 45 = Total weight of class B = 2430 × (100/60.75) =
18 4000 kg
Number of pages typed by C on Fri = 18 × (1/2) Total weight of class C = 300 × [11/(12 – 11)] =
=9 3300 kg
Number of pages typed by A till Fri = 90 Total weight of class D = 3300 + 300 = 3600 kg
Number of pages typed by C till Thu = 2 × 68 – Average weight of class A = 2430/3x = (810/x)
75 = 61 kg
Number of pages typed by C till Fri = 61 + 9 = 70 Average weight of class B = 4000/4y = (1000/y)
Required average = (90 + 70)/2 = 80 kg
Average weight of class C = 3300/3y = (1100/y)
4) Answer: E kg
Number of pages typed by C till Wed = 2 × 48 – Average weight of class D = 3600/5x = (720/x)
50 = 46 kg
Number of pages typed by C till Thu = 2 × 68 – Since, ratio of average weight of class A to the
75 = 61 average weight of class B is 27: 25
Number of pages typed by C on Thu = 61 – 46 = So,
15 [(810/x)/(1000/y)] = 27/25
Number of pages typed by C on Tue = 15 + 1 = (810 × y)/(1000 × x) = 27/25
16 3y = 4x -------------(1)
Number of pages typed by A till Tue = 30 Since, average weight of class C is 7 kg more
Number of pages typed by C till Tue = 2 × 28 – than the average weight of class D.
30 = 26 (1100/y) – (720/x) = 7
Number of pages typed by C on Mon = 26 – 16 = 1100x – 720y = 7xy -------------(2)
10 From equation (1) and (2):
Number of pages typed by A on Mon = 2 × 17 – 1100x – 960x = 7x × (4x/3)
10 = 24 140x = 7x × (4x/3)
x = 15
Direction (5-8): From equation (1):

Click Here For Bundle PDF Course | support@guidely.in Page 9 of 16


Bank Po Mains PDF Course 2024
Quantitative Aptitude Day -24 (Eng)

y = 20 Hence, all are true.

7) Answer: A
Total number of students in class D = 75
Number of boys in class D = 75 × [8/(8 + 7)] = 40
kg
Number of girls in class D = 75 – 40 = 35 kg
5) Answer: C
Total weight of girls = 35 × 50 = 1750 kg
Number of girls in class A = 25
Average weight of boys = (3600 – 1750)/40 =
Number of boys in class A = 45 – 25 = 20
46.25 kg
And total weight of boys in class A = 2430 – (25
Required difference = 50 – 46.25 = 3.75 kg
× 52) = 1130 kg
Number of girls in class B = 40
8) Answer: D
Number of boys in class B = 80 – 40 = 40
Total number of students in class C = 60
And total weight of boys in class B = 4000 – (40
Initial total weight of class C = 3300 kg
× 48) = 2080 kg
Initial average weight of class C = 55 kg
So, the average weight of all the boys of classes
So,
A and B:
(3300 – 45 + M)/60 = 55.5
(1130 + 2080)/(20 + 40) = 53.5 kg
M = 75
Now, the value of [20% of M + 16(2/3)% of (M –
6) Answer: D
15)]1/2
From P:
[20% of 75 + 16(2/3)% of 60]1/2 = [15 + 10]1/2 = 5
Average weight of all the students of classes B
and C:
Direction (9-12):
(4000 + 3300)/(80 + 60) = 52 kg (approx.)
Let the quantities of water in mixtures B and D
So, A is true.
are ‘5x’ L and ‘4x’ L respectively.
From Q:
So, quantity of milk in mixture B = (45 – 5x) L
Difference between number of students in
And quantity of milk in mixture D = (32 – 4x) L
classes C and D = 75 – 60 = 15
Let the cost of pure milk = ₹ y per L
So, B is true.
Since, the cost of mixture B = ₹ 40 per L
From R:
So,
Ratio of average weight of class B to that of
[(45 – 5x) × y]/45 = 40
class C = 50: 55 = 10: 11
y = 360/(9 – x) ------------(1)
So, C is true.

Click Here For Bundle PDF Course | support@guidely.in Page 10 of 16


Bank Po Mains PDF Course 2024
Quantitative Aptitude Day -24 (Eng)

Since, the cost of mixture D = ₹ 37.5 per L Quantity of milk in mixture A = 60% of 40 = 24 L
So, So, the cost of mixture A = (24 × 60)/40 = ₹ 36
[(32 – 4x) × y]/32 = 37.5 per L
y = 300/(8 – x) ------------(2) Required percentage = [(42 – 36)/36] × 100 =
From equations (1) and (2): 16.67%
360/(9 – x) = 300/(8 – x)
48 – 6x = 45 – 5x 11) Answer: B
x=3 Let the total quantity of mixture C = ‘5c’ L
From equation (1): So, the quantity of milk in mixture C = 80% of
y = 360/(9 – 3) ‘5c’ = ‘4c’ L
y = 60 Total quantity of mixture B = 45 L
Cost of pure milk = ₹ 60 per Lit Quantity of milk in mixture B = 30 L
Quantity of water in mixture B = 5 × 3 = 15 L So,
Quantity of milk in mixture B = 45 – 15 = 30 L [(30 + 4c) × 60]/(45 + 5c) = 43.2
Quantity of water in mixture D = 4 × 3 = 12 L 1800 + 240c = 1944 + 216c
Quantity of milk in mixture D = 32 – 12 = 20 L c=6
9) Answer: D So, the total quantity of mixture C = 30 L
Percentage of milk in mixture B = M = (30/45) ×
100 = 66(2/3)% 12) Answer: A
Percentage of milk in mixture D = N = (20/32) × Total quantity of mixture A = 40 L
100 = 62.5% Quantity of milk in mixture A = 60% of 40 = 24 L
Required ratio = M: N = (200/3): (625/10) = Total quantity of mixture B = 45 L
(200/3): (125/2) = 16: 15 Quantity of milk in mixture B = 30 L
Total quantity of mixture D = 32 L
10) Answer: A Quantity of milk in mixture D = 20 L
Total quantity of mixture B = 45 L When mixtures A, B and D are missed together,
Total quantity of mixture E = 45 × (100/90) = 50 then the cost of the new mixture:
L [(24 + 30 + 20) × 60]/(40 + 45 + 32) = ₹ 37.9 per
Quantity of milk in mixture E = 70% of 50 = 35 L L
Since, the cost of pure milk = ₹ 60 per L
So, the cost of mixture E = (35 × 60)/50 = ₹ 42 Directions (13–16):
per L Number of students who cleared (fresh) the
Total quantity of mixture A = 40 L exam in 2014 = 4000

Click Here For Bundle PDF Course | support@guidely.in Page 11 of 16


Bank Po Mains PDF Course 2024
Quantitative Aptitude Day -24 (Eng)

Number of students who failed (fresh) the exam Number of students who are in the waitlist in the
in 2014 = 1000 exam in 2017 = 4500
Number of students who are in the waitlist in the Number of students who cleared (from waitlist)
exam in 2014 = 2000 the exam in 2017 = 70% of 5000 = 3500
Number of students who cleared (from waitlist) Number of students who failed (from waitlist) the
the exam in 2014 = 45% of 4000 = 1800 exam in 2017 = 4500 – 3500 = 1000
Number of students who failed (from waitlist) the Number of students who cleared (fresh) the
exam in 2014 = 2000 – 1800 = 200 exam in 2018 = 3000
Number of students who cleared (fresh) the Number of students who failed (fresh) the exam
exam in 2015 = 4500 in 2018 = 1000
Number of students who failed (fresh) the exam Number of students who are in the waitlist in the
in 2015 = 1500 exam in 2018 = 2000
Number of students who are in the waitlist in the Number of students who cleared (from waitlist)
exam in 2015 = 3000 the exam in 2018 = 50% of 3000 = 1500
Number of students who cleared (from waitlist) Number of students who failed (from waitlist) the
the exam in 2015 = 60% of 4500 = 2700 exam in 2018 = 2000 – 1500 = 500
Number of students who failed (from waitlist) the
exam in 2015 = 3000 – 2700 = 300
Number of students who cleared (fresh) the
exam in 2016 = 3000
Number of students who failed (fresh) the exam
in 2016 = 1500
Number of students who are in the waitlist in the
exam in 2016 = 2500
Number of students who cleared (from waitlist)
the exam in 2016 = 60% of 3000 = 1800 13) Answer: B
Number of students who failed (from waitlist) the Total number of students who cleared from (both
exam in 2016 = 2500 – 1800 = 700 fresh and waitlist) the exam in 2014 = 4000 +
Number of students who cleared (fresh) the 1800 = 5800
exam in 2017 = 5000 Total number of students who cleared (fresh) the
Number of students who failed (fresh) the exam exam in 2017 = 5000
in 2017 = 2500 Required ratio = 5800: 5000
= 29: 25

Click Here For Bundle PDF Course | support@guidely.in Page 12 of 16


Bank Po Mains PDF Course 2024
Quantitative Aptitude Day -24 (Eng)

Required sum = A + B
14) Answer: A
Total number of students who failed (from both
fresh and waitlist) in the exam in 2015 = 1500 +
300 = 1800
Total students appeared for the exam in 2015 = 16) Answer: C

9000 Number of boys who cleared (fresh) the exam in

According to the question: 2016 = = 1600


Number of girls who cleared (fresh) the exam in
2016 = = 1400
P = 20
Number of boys who cleared (from waitlist) the
P is multiple of both 4 and 5.
exam in 2016 = = 400
Total factors of P = 1, 2, 4, 5, 10, and 20 = 6
Number of girls who cleared (from waitlist) the
factors
exam in 2016 = = 1400
Square of P is 400 which is not a perfect cube of
any integer. Total number of boys who cleared the exam in

Hence, statement B is not TRUE. 2016 = 1600 + 400 = 2000


Total number of students who cleared the exam

15) Answer: E in 2016 = 3000 + 1800 = 4800


Required percent =
Total students appeared for the exam in 2015 =
9000
Total students who cleared (fresh) the exam in
2015 = 4500 Directions (17–20):
According to the question: System apps in phone A = = 20
System apps in phone B = = 15
A = 50 System apps in phone C = = 10
Total students appeared for the exam in 2017 = System apps in phone D = = 25
12000 System apps in phone E = =5
Total students who cleared (from waitlist) the Total apps in phone B = 60
exam in 2017 = 3500 Total non-system apps in phone B = 60 – 15 =
According to the question: 45
Gaming apps in phone B = = 15
Social media apps in phone B = = 20

Click Here For Bundle PDF Course | support@guidely.in Page 13 of 16


Bank Po Mains PDF Course 2024
Quantitative Aptitude Day -24 (Eng)

Payment apps in phone B = = 10


Number of payment apps in phone D = =5
Number of gaming apps in phone D = = 25
Number of social media apps in phone D =
= 20
Total apps in phone D = 25 + (5 + 25 + 20) = 75
75 = 15x
x=5
Number of gaming apps in phone A = =5
17) Answer: D
Number of payment apps in phone A = =
Total apps in phones B, D, and E together = 60
15 + 75 + 45 = 180
Total apps in phone A = 10x = 50 Total system apps in phones B, D, and E
Non-system apps in phone A = 50 – 20 = 30 together = 15 + 25 + 5 = 45
Number of social media apps in phone A = 30 – Required percent =
5 – 15 = 10
= 25%
Total apps in phone E = 9x = 45
Non-system apps in phone E = 45 – 5 = 40
18) Answer: E
Number of gaming apps in phone E =
Total apps in all the five phones = 5 * 54 = 270
= 10 Total apps in phone C = 270 – (50 + 60 + 75 +
Number of social media apps in phone E = 45) = 40
=5 Non-system apps in phone C = 40 – 10 = 30
Number of payment apps in phone E = Ratio of social media apps to payment apps in
= 25 phone C = 7: 21 = 1: 3
We can calculate the exact number of apps for Case 1: When number of social media apps and
phone B. payment apps in phone C are 1 and 3
respectively.
Number of gaming apps in phone C = 30 – 1 – 3
= 26
According to the question:
26: 1: 3 = y: 7: 21
y = 182

Click Here For Bundle PDF Course | support@guidely.in Page 14 of 16


Bank Po Mains PDF Course 2024
Quantitative Aptitude Day -24 (Eng)

Case 2: When number of social media apps and Case 6: When number of social media apps and
payment apps in phone C are 2 and 6 payment apps in phone C are 6 and 18
respectively. respectively.
Number of gaming apps in phone C = 30 – 2 – 6 Number of gaming apps in phone C = 30 – 6 –
= 22 18 = 6
According to the question: According to the question:
22: 2: 6 = y: 7: 21 6: 6: 18 = y: 7: 21
y = 77 y=7
Case 3: When number of social media apps and Case 7: When number of social media apps and
payment apps in phone C are 3 and 9 payment apps in phone C are 7 and 21
respectively. respectively.
Number of gaming apps in phone C = 30 – 3 – 9 Number of gaming apps in phone C = 30 – 7 –
= 18 21 = 2
According to the question: According to the question:
18: 3: 9 = y: 7: 21 2: 7: 21 = y: 7: 21
y = 42 y=2
Case 4: When number of social media apps and Hence, possible values of ‘y’ = 182, 77, 42, 14,
payment apps in phone C are 4 and 12 7, 2
respectively. Total values of ‘y’ = 6
Number of gaming apps in phone C = 30 – 4 – Hence, 49 cannot be the possible value of ‘y’.
12 = 14
According to the question: 19) Answer: B
14: 4: 12 = y: 7: 21 Sum of total gaming apps and payment apps
y = 24.5 [Invalid] together in phone B = 15 + 10 = 25
Case 5: When number of social media apps and Total non-system apps in phone B = 45
payment apps in phone C are 5 and 15 Required percent =
respectively.
Number of gaming apps in phone C = 30 – 5 –
15 = 10
20) Answer: C
According to the question:
Total number of social media apps in all the five
10: 5: 15 = y: 7: 21
phones together = 65
y = 14
Number of social media apps in phone C = 65 –
(10 + 20 + 20 + 5) = 10

Click Here For Bundle PDF Course | support@guidely.in Page 15 of 16


Bank Po Mains PDF Course 2024
Quantitative Aptitude Day -24 (Eng)

Number of payment apps in phone C = = Required average =


30 = 17
Total number of payment apps in all the five
phones together = 15 + 10 + 30 + 5 + 25 = 85

Click Here For Bundle PDF Course | support@guidely.in Page 16 of 16


Bank Po Mains PDF Course 2024
English Day - 24

English Language
Directions (1-5): The following paragraph given policy responses to inflationary pressures. India’s
below has five blanks in which words from recent curbs on export of rice have _________ (D)
options given in each question would be filled in some alarm, with the prices of the regional staple
order to complete the paragraph meaningfully. reportedly nearing a 15-year high. The onset of
You must read the same carefully and choose an El Niño, which is historically associated with
the correct answer for each question. __________ (E) weather events, queers the
Prime Minister Narendra Modi’s whistle-stop ground further, and ASEAN leaders are justifiably
summit sojourn to the Indonesian capital of wary.
Jakarta earlier this week was __________ (A) 1) Which of the following words should fill the
aimed at deepening India’s engagement with the blank A in the given passage ?
economically significant grouping of 10 a) primarily
Southeast Asian nations. Coming on the eve of b) main
India’s hosting of the G-20 summit in New Delhi c) focused
as the current holder of the bloc’s presidency, d) thorough
Mr. Modi’s presence at the annual ASEAN-India e) abundantly
summit was an _________ (B) to cement
traditional ties with the neighbouring Asian 2) Which of the following words should fill the
economies at a time of heightened global trade blank B in the given passage ?
uncertainty. As the trade facilitation body a) coincidence
UNCTAD noted in its June 21 ‘Global Trade b) situation
Update’, the ‘outlook for global trade in the c) inviting
second half of 2023 is pessimistic as negative d) opportunity
factors including downgraded world economic e) incharge
forecasts, persistent inflation, financial
vulnerabilities and geopolitical tensions 3) Which of the following words should fill the
dominate. Against this backdrop, the joint blank C in the given passage ?
leaders’ statement on ‘Strengthening Food a) developing
Security and Nutrition in Response to Crises’ at b) emerging
the ASEAN-India summit underscores the shared c) happening
vulnerability the region perceives in the face of d) becoming
the ________ (C) heightened global food e) ongoing
insecurity, which has been exacerbated by the 4) Which of the following words should fill the
war in Ukraine, climate change and national blank D in the given passage ?

Click Here For Bundle PDF Course | support@guidely.in Page 1 of 11


Bank Po Mains PDF Course 2024
English Day - 24

a) enabled I. causing some delays in our travel plans


b) triggered II. as there were no flights available to the
c) turned on country
d) made III. making it difficult for the parents who were
e) produced hungry
a) Only I
5) Which of the following words should fill the b) Only II
blank E in the given passage ? c) Only III
a) pleasant d) Both I and III
b) awesome e) All I, II and III
c) disruptive
d) badly 8) The maid made so many mistakes while
e) good cleaning the house _____________.
I. and the house was neat and clean later
Directions (6-10) : In each question a sentence is II. that we had to spend extra time fixing things
given with a blank. You are provided with three afterward
fragments I, II and III. You have to identify which III. despite giving her many suggestions
statement/statements can carry forward the a) Only II
given sentence in the most logical way so as to b) Only III
make the sentence coherent and contextually c) Both I and II
correct. d) Both II and III
6) The family left for _____________ spend quality e) All I, II and III
time together.
I. Australia and has planned to 9) ___________________ fantastic time riding
II. a relaxing vacation by the beach to unwind thrilling roller coasters and enjoying all the
and entertaining shows and attractions.
III. purchase at the supermarket and
a) Only I I. I went to Germany for a presentation and had a
b) Only III II. I went to the zoo yesterday and spent
c) Both I and II III. I went to the theme park recently and had a
d) Both II and III a) Only II
e) All I, II and III b) Only III
7) The visa process took longer than expected, c) Both I and III
_____________________________. d) Both II and III

Click Here For Bundle PDF Course | support@guidely.in Page 2 of 11


Bank Po Mains PDF Course 2024
English Day - 24

e) All I, II and III


B. India’s recent 2. as negative factors
curbs on export of including downgraded
10) The dancers were incredibly talented and
rice have triggered world economic forecasts,
___________________.
some alarm, persistent inflation,
I. put on a mesmerizing performance financial vulnerabilities
II. managed to pull off a terrific show and geopolitical tensions
III. travelled to many countries to perform dominate.
a) Only I
b) Only II C. The Prime 3. which has no territorial

c) Only III Minister’s not-so- ambitions that could

d) Both II and III veiled message to discomfit them. India also

e) All I, II and III the ASEAN sought to position itself as


members is that a voice to amplify the

Directions (11-15): Given below are a few India is a more concerns of the Global

questions with a table given with two columns reliable long-term South, stressing that it

with highlighted connectors to be matched in strategic and would be mutually

order to make sentences contextually correct as economic partner, beneficial for all.

per the rules of connectors. If none of the options a) A-1, B-2, C-3
are correct then choose option E as your answer. b) A-2, B-3, C-1
11) c) A-2, B-1, C-3
d) A-3, B-1, C-2
e) None of these

A. As the trade 1. with the prices of the


facilitation body regional staple reportedly 12.

UNCTAD noted in its nearing a 15-year high. A. The report also 1. however, it does not
June 21 ‘Global dwells on the need to single out individual
Trade Update’, the reverse deforestation countries or provide a
‘outlook for global and the adoption of more granular analysis
trade in the second electric vehicles as of where the existing
half of 2023 is vital prongs to a clean shortcomings are in the
pessimistic energy economy; approach adopted by
countries to curtail
emissions.

Click Here For Bundle PDF Course | support@guidely.in Page 3 of 11


Bank Po Mains PDF Course 2024
English Day - 24

B. The United 2. that awaited the C. Now, with the 3. with a military wing
Nations’ Global major economies of the ASEAN move, it is that has joined hands
Stocktake, a report world even as it evident that the with some ethnic
that was released just presented little beyond regime stands separatist groups, thus
ahead of the G-20 what is already known. isolated, posing a challenge to the
meet, set out the junta.
scope of challenges
a) A-1, B-3, C-2

C. These are gaps 3. and an eschewal of b) A-3, B-2, C-1

unlikely to be filled fossil fuel sources such c) A-2, B-3, C-1

without a rapid as coal, oil and natural d) A-2, B-1, C-3

upscaling of gas. e) None of these

renewable energy
resources 14)

a) A-1, B-2, C-3 A. The government’s 1. but the days ahead

b) A-1, B-3, C-2 decision to hold the G- will be crucial in

c) A-2, B-1, C-3 20 summit two months cementing its legacy.

d) A-3, B-2, C-1 early,

e) None of these B. India’s G-20 2. instead of in


moment is already November, the practice,
13) memorable, has given officials less
A. Myanmar has seen 1. while the domestic time.
a security and situation remains
economic decline untenable.

B. The political 2. ever since the military


opposition has formed ousted the
a National Unity democratically elected
Government government of Aung San
Suu Kyi in February
2021.

Click Here For Bundle PDF Course | support@guidely.in Page 4 of 11


Bank Po Mains PDF Course 2024
English Day - 24

C. The global 3. such as climate C. The Rashtriya 3. but these labels of


economic headwinds, finance, and investment Swayamsevak Sangh pride have now been
exacerbated by the in health and poverty has been asking for weaponised for narrow
COVID-19 pandemic, alleviation. privileging the use of political ends.
meant a challenging Bharat over India for
environment for New long,
Delhi in ensuring that
a) A–1, B-3, C-2
all countries were b) A–2, B-3, C-1
willing to contribute c) A–3, B-1, C-2
resources for d) A–2, B-1, C-3
imperatives e) None of these
a) A-2, B-1, C-3
b) A-2, B-1, C-3 Directions (16-20) : In each of the questions
c) A-2, B-1, C-3 given below a phrase in the sentence has been
d) A-2, B-1, C-3 highlighted. It may or may not need replacement.
e) None of these Read the question carefully and choose an
option that would best replace the highlighted
15) part to make the sentence grammatically and

A. The Bharatiya 1. but the Opposition contextually correct. In case, the given sentence

Janata Party bloc’s decision to label is correct in its current form and there is no

government at the itself INDIA as an improvement needed, mark (e) as your answer.
16) The grocery stores on the country has a wide
Centre has decided to acronym also might
use Bharat instead of have influenced the variety of fresh produce and a great selection of
international foods.
India in some official BJP’s hurry in the
a) the grocery stores in the country have a wide
communication and naming exercise.
variety of
documents, a practice
b) the groceries stores in the country has a wide
B. India and Bharat 2. that its variety
have both evoked the representatives say will c) the grocery stores at the country have a wide
same emotions now expand. variety
among patriots for d) the grocery stores to the country have a wide
decades, variety
e) No changes required

Click Here For Bundle PDF Course | support@guidely.in Page 5 of 11


Bank Po Mains PDF Course 2024
English Day - 24

17) The president visited the hurricane- leaving them determined to come back stronger
devastated region to assess the damage and next season.
offer support to the affected communities. a) the team has performed well and feeling short
a) for assess the damage and to support to the b) the team has performed short and felt
b) to assessing the damage and supporting the perfromed
c) for assess damage and offering support c) the team had performed well and short and fell
d) by assessing and offering support d) the team have performed short and felt well
e) No changes required e) No changes required

18) On Teachers' Day, the students organized a 20) Rekha was in a hurry, so she quickly grab
special assembly to expressing there gratitude her belongings and dashed to the door, hoping to
and appreciation of their dedicated educators. catch her train before it departed.
a) by expressing their gratitude and appreciation a) so she quick grabbed her belongings and
of their dashed to
b) to express their gratitude and appreciation for b) so she quickly grabbed her belongings and
their dashed by
c) for expressing there gratitude and appreciation c) thus she quickly grabbed her belongings by
of there dashed out
d) to express the gratitude and appreciate their d) so she quickly grabbed her belongings and
e) No changes required dashed out
e) No changes required
19) The team had performed well but fell short of
winning the championship by a narrow margin,

Click Here For Bundle PDF Course | support@guidely.in Page 6 of 11


Bank Po Mains PDF Course 2024
English Day - 24

Click Here to Get the Detailed Video Solution for the above given Questions
Or Scan the QR Code to Get the Detailed Video Solutions

Answer Key with Explanation


1) Answer: A Ongoing is the correct word that can fit in the
Primarily is the right word to fill the given blank A given blank.
of the passage. Option a - developing (no proper meaning
Option b - was main aimed (incorrect) conveyed and has positive meaning which seem
Option c - was focused aimed (incorrect) irrelavant)
Option d - was thorough aimed (incorrect and no Option b - emerging (positive meaning)
proper meaning) Option c - happening (irrelevant)
Option e - was abundantly aimed (incorrect and Option d - becoming (irrelevant)
no proper meaning)
4) Answer: B
2) Answer: D Triggered is the correct word that fits in the given
Opportunity is the correct word that fills the given blank making the respective sentence
blank in the most appropriate way. meaningful and correct.
Option a - an coincidence (wrong, ‘an’ must be Out of the given options triggered is the most
followed by a word starting with a vowel or ‘h’) appropriate word that conveys a proper
Option b - an situation (wrong, ‘an’ must be meaning.
followed by a word starting with a vowel or ‘h’)
Option c - was an inviting (no proper meaning) 5) Answer: C
Option e - was an incharge (no proper meaning) Disruptive is the right word that can fill the given
blank.
3) Answer: E Pleasant, awesome and good - positive tone
(irrelevant to the context)

Click Here For Bundle PDF Course | support@guidely.in Page 7 of 11


Bank Po Mains PDF Course 2024
English Day - 24

9) Answer: B
Badly is incorrect and hence discarded The statement or fragment III completes the
given sentence making it meaningful and
6) Answer: C correct.
Both the given fragments I and II complete the The sentence: I went to the theme park recently
given sentence in a meaningful way. and had a fantastic time riding thrilling roller
III - incorrect and causes grammatical error too coasters and enjoying all the entertaining shows
(hence discarded) and attractions.
The sentences:
The family left for Australia and has planned to 10) Answer: E
spend quality time together. All the given three fragments complete the given
The family left for a relaxing vacation by the sentence.
beach to unwind and spend quality time The sentences:
together. The dancers were incredibly talented and put on
a mesmerizing performance.
7) Answer: A The dancers were incredibly talented and
Only I complete the sentence in the most managed to pull off a terrific show.
meaningful way. The dancers were incredibly talented and
The sentence: The visa process took longer than travelled to many countries to perform.
expected, causing some delays in our travel
plans. 11) Answer: C
The correct option is ‘c’.
8) Answer: D A-2, B-1 and C-3 are the correct combinations
Both the fragments II and III complete the given that form meaningful sentences.
sentence. The sentences:
The sentences: As the trade facilitation body UNCTAD noted in
The maid made so many mistakes while its June 21 ‘Global Trade Update’, the ‘outlook
cleaning the house that we had to spend extra for global trade in the second half of 2023 is
time fixing things afterward. pessimistic as negative factors including
The maid made so many mistakes while downgraded world economic forecasts,
cleaning the house despite giving her many persistent inflation, financial vulnerabilities and
suggestions. geopolitical tensions dominate.

Click Here For Bundle PDF Course | support@guidely.in Page 8 of 11


Bank Po Mains PDF Course 2024
English Day - 24

India’s recent curbs on export of rice have These are gaps unlikely to be filled without a
triggered some alarm, with the prices of the rapid upscaling of renewable energy resources
regional staple reportedly nearing a 15-year and an eschewal of fossil fuel sources such as
high. coal, oil and natural gas.
The Prime Minister’s not-so-veiled message to
the ASEAN members is that India is a more 13) Answer: C
reliable long-term strategic and economic Option c is the right answer for this question
partner, which has no territorial ambitions that forming three meaningful sentences from the
could discomfit them. India also sought to combinations given.
position itself as a voice to amplify the concerns The sentences:
of the Global South, stressing that it would be Myanmar has seen a security and economic
mutually beneficial for all. decline ever since the military ousted the
democratically elected government of Aung San
12) Answer: A Suu Kyi in February 2021.
The right set of combinations that forms proper The political opposition has formed a National
meaningful sentences is A-1, B-2 and C-3. Unity Government with a military wing that has
The other combinations are discarded as they do joined hands with some ethnic separatist groups,
not form proper sentences. thus posing a challenge to the junta.
The sentences: Now, with the ASEAN move, it is evident that the
The report also dwells on the need to reverse regime stands isolated, while the domestic
deforestation and the adoption of electric situation remains untenable.
vehicles as vital prongs to a clean energy
economy; however, it does not single out 14) Answer: D
individual countries or provide a more granular Option d is the right answer for this question
analysis of where the existing shortcomings are forming three meaningful sentences from the
in the approach adopted by countries to curtail combinations given.
emissions. The sentences:
The United Nations’ Global Stocktake, a report The government’s decision to hold the G-20
that was released just ahead of the G-20 meet, summit two months early, instead of in
set out the scope of challenges that awaited the November, has given officials less time.
major economies of the world even as it India’s G-20 moment is already memorable, but
presented little beyond what is already known. the days ahead will be crucial in cementing its
legacy.

Click Here For Bundle PDF Course | support@guidely.in Page 9 of 11


Bank Po Mains PDF Course 2024
English Day - 24

The global economic headwinds, exacerbated by Option b - groceries stores and has - incorrect
the COVID-19 pandemic, meant a challenging Option c - at the country (incorrect)
environment for New Delhi in ensuring that all Option d - to the country (incorrect)
countries were willing to contribute resources for The sentence: The grocery stores in the country
imperatives such as climate finance, and have a wide variety of fresh produce and a great
investment in health and poverty alleviation. selection of international foods.

15) Answer: B 17) Answer: E


Option b is the right option forming three The highlighted phrase is correct and
meaningful and correct sentences. meaningful. No changes required.
A-2, B-3 and C-1 are the right combinations.
The sentences: 18) Answer: B
The Bharatiya Janata Party government at the The highlighted phrase is incorrect and needs
Centre has decided to use Bharat instead of replacement.
India in some official communication and The correct replacement phrase is given in
documents, a practice that its representatives Option b.
say will now expand. Option a - the given phrase in incorrect
India and Bharat have both evoked the same Option c - the phrase has grammatical error in it
emotions among patriots for decades, but these Option d - the phrase has grammatical error in it
labels of pride have now been weaponised for The sentence: On Teachers' Day, the students
narrow political ends. organized a special assembly to express their
The Rashtriya Swayamsevak Sangh has been gratitude and appreciation of their dedicated
asking for privileging the use of Bharat over India educators.
for long, but the Opposition bloc’s decision to
label itself INDIA as an acronym also might have 19) Answer: E
influenced the BJP’s hurry in the naming The highlighted phrase in the given sentence is
exercise. correct and meaningful. No replacement is
required.
16) Answer: A
The highlighted part of the sentence is 20) Answer: D
grammatically incorrect and needs replacement. The highlighted phrase is incorrect and needs
replacement.
The correct phrase is in option a.

Click Here For Bundle PDF Course | support@guidely.in Page 10 of 11


Bank Po Mains PDF Course 2024
English Day - 24

The phrase in option d is the right phrase for The sentence: Rekha was in a hurry, so she
replacement. quickly grabbed her belongings and dashed out
Option a - quick is incorrect the door, hoping to catch her train before it
Option b - dashed by - incorrect departed.
Option c - by dashed out - incorrect

Click Here For Bundle PDF Course | support@guidely.in Page 11 of 11


Bank Po Mains PDF Course 2024
Reasoning Day -25 (Eng)

Reasoning Aptitude
Directions (1-5): Study the following information c) 8
carefully and answer the given questions. d) 6
Step 1: Take the alphabets whose place value is e) None of these
an even number as per the English alphabetical
series and they are written in the reverse 2. If the last 15 elements are reversed, then how
alphabetical order from the left end. Then write many such consonants are there in the series
the vowels from the right end in alphabetical which is immediately preceded by a symbol and
order. not immediately followed by a vowel?
Step 2: After completing step 1, take the a) One
consecutive three digit odd numbers up to the b) Two
number 117 and add the digits within the number c) Three
till it becomes a single digit, then place the d) Four
resultant number one by one in the same order e) None
after every two letters from the left end.
Step 3: After completing step 2, the first 12 3. What is the sum of the numbers between the
elements from the left end are reversed and the third vowel from the right end and the fourth
following symbols “$, &, %, #, @, ©, *, ®, ∞”, are consonant from the left end?
placed (in the same order) in the series after a) 10
every three elements from the left end. b) 17
Step 4: After completing step 3, the letters which c) 12
are immediately preceded by a symbol and d) 14
immediately followed by a vowel are changed to e) 8
the immediate next letter as per the alphabetical
series. 4. In the given series, all the letters in the first
Step IV is the final step and answer the given sixteen elements are replaced by the
questions according to the final step. complementary paired letter in the English
1. If all the symbols are dropped from the series alphabetical series, then which among the
and all the numbers are arranged in ascending following letters are repeated more than once?
order from the right end then which of the a) E, D, M, U
following element will be 11th to the right of the 9th b) N, P, T, E
element from the left end? c) F, D, P, T
a) A d) Z, X, G, A
b) 9 e) I, C, A, F

Click Here For Bundle PDF Course | support@guidely.in Page 1 of 11


Bank Po Mains PDF Course 2024
Reasoning Day -25 (Eng)

5. In the given series, all the consonants which below input and answer the questions given
come before G and after T in the alphabetical below.
series are dropped, then how many symbols are
immediately preceded by a consonant and
immediately followed by a number? 6. What will be the resultant if the fourth digit

a) Two from the left end in step 1 is multiplied with the

b) One first digit from the right end in step 3 of the given

c) Four input?

d) Three a) 14

e) None b) 18
c) 16

Directions (6-10): Study the following information d) 24

carefully and answer the given questions. e) 20

The number arrangement machine, when given


an input, rearranges them by following a 7. What is the difference between the square

particular rule in each step. The following is the values of the two numbers in step 3 of the given

illustration of an input and rearrangement. input?


a) 200
b) 384
c) 250
d) 376
e) 380

8. What will be the output, if the second digit


from the left end in step 2 is doubled?
a) 6
b) 4
c) 10
d) 12
e) 8

9. If all the digits in step 1 are arranged in


Step 4 is the last and final step of the given input. ascending order from left to right, then how many
By following the same rules, find the steps for the digits remain unchanged in their position?

Click Here For Bundle PDF Course | support@guidely.in Page 2 of 11


Bank Po Mains PDF Course 2024
Reasoning Day -25 (Eng)

a) Three that A and B sit opposite to each other, then both


b) Two are sitting at the same table.
c) One F sits third to the right of B, who is facing the
d) Four centre of the table. One person sits between F
e) None and K. Two persons sit between J and the one
who faces K. H sits opposite to J. L sits second
10. What is the difference between the digits of to the right of the one who faces J. The number
the number in the penultimate step of the given of persons sitting between L and F is two less
input? than the number of persons sitting between J
a) Five and O, who does not face K. C is an immediate
b) Seven neighbour of both O and P. N sits second to the
c) Six left of the one who faces P. I sits to the
d) Eight immediate left of E. The number of persons
e) Four sitting between E and C is two more than the
number of persons sitting between B and A. M
Directions (11-15): Study the following does not face the centre of the table.
information carefully and answer the given 11. If all the persons sitting at the outer circular
questions table are rearranged in alphabetical order in a
Sixteen persons - A, B, C, D, E, F, G, H, I, J, K, clockwise direction starting from A, then how
L, M, N, O, and P are sitting around the two many persons remain unchanged except A?
concentric circular tables as one is inscribed in a) One
another one. Eight persons are sitting on the b) Two
dinner table facing away from the centre. Eight c) Three
persons are sitting on the outer table facing the d) Four
centre. Each person sitting on the outer table e) None
faces the person sitting on the dinner table and
vice versa. 12. If all the persons interchanged their positions
Note: with the person who is facing them, then who
i) The persons whose name’s place value is a among the following sits third to the right of L?
prime number as per alphabetical order are not a) The one who faces I
sitting adjacent to each other. b) N
ii) If it is given that A and B face each other, then c) The one who sits second to the left of D
both are sitting at different tables and if it is given d) A
e) G

Click Here For Bundle PDF Course | support@guidely.in Page 3 of 11


Bank Po Mains PDF Course 2024
Reasoning Day -25 (Eng)

13. Four of the following five are alike in a certain lowermost floor is numbered one and the floor
way based on the given arrangement and thus immediately above it is numbered two and so on.
form a group. Which one of the following does There are three lifts Lift I, Lift II, and Lift III from
not belong to the group? west to east in the basement of the building.
a) L Each delivery boy takes lifts to deliver the food.
b) D Not more than two persons delivered food on the
c) E same floor. Also, persons delivering on the same
d) B floor take different lifts.
e) A Note:
i) The person whose name’s place value is an
14. Which among the following pair of persons odd number as per the alphabetical series
are facing each other? delivered in an even numbered floor and the
I. KM person whose name’s place value is an even
II. AP number as per the alphabetical series delivered
III. GJ in an odd numbered floor.
a) Only I ii) No two persons delivered the food on the
b) Only II adjacent floors take the same lift.
c) Only I and II F delivered the food three floors above I, where
d) Only II and III neither of them uses the same lift nor uses lift II.
e) All I, II and III F did not deliver above floor number 6. Both N
and H delivered below I. J does not use lift II and
15. How many persons sit between P and the delivered two floors above H, where both of them
one who faces N when counted from the right of use the same lift. J and D delivered on the same
P? floor. The number of persons delivered the food
a) Two below D is three more than the number of
b) One persons delivered the food above C, who did not
c) Three use the same lift as N and J. Less than three
d) Four floors are between the floors on which C and G
e) None delivered the food. L delivered immediately
above the floor on which G delivered the food. G
Direction (16-20): Study the following information uses lift II. More than two floors are between the
carefully and answer the given questions. floors on which L and K delivered the food. The
Fourteen Zomato delivery boys – A to N number of floors between the floors on which K
delivered food in a ten storey building where the and I delivered the food is one less than the

Click Here For Bundle PDF Course | support@guidely.in Page 4 of 11


Bank Po Mains PDF Course 2024
Reasoning Day -25 (Eng)

number of floors between the floors on which F a) One


and M delivered the food. E delivered below A b) Two
but above B. E does not use lift III. c) Three
16. How many persons are using lift II? d) Four
a) Three e) Five
b) Four
c) Six 19. As many floors between B and N as between
d) Five _____ and _____.
e) Seven a) I and H
b) L and F
17. Who among the following person(s) delivered c) A and G
food to floor number 5? d) M and B
I. B e) C and G
II. D
III. F 20. Four of the following five are alike in a certain
a) Only I way based on the given arrangement and thus
b) Only II form a group. Which one of the following does
c) Only I and III not belong to the group?
d) Only II and III a) L
e) All I, II and III b) C
c) M
18. How many persons delivered the food above d) E
E’s Floor? e) I
Click Here to Get the Detailed Video Solution for the above given Questions
Or Scan the QR Code to Get the Detailed Video Solutions

Answer Key with Explanation

Click Here For Bundle PDF Course | support@guidely.in Page 5 of 11


Bank Po Mains PDF Course 2024
Reasoning Day -25 (Eng)

Directions (1-5): 8LN$6PR&4TV%2XZ#JH1@FD3


Step 1: Take the alphabets whose place value is ©CU5*PI7®FA9∞
an even number as per the English alphabetical 8LN6PR4TV2XZJH1FD3CU5PI7
series and they are written in the reverse FA9
alphabetical order from the left end. Then write LNPRTVXZJHFDCUPIFA987654
the vowels from the right end in alphabetical 321
order.
ZXVTRPNLJHFDBUOIEA 2. Answer: B
Step 2: After completing step 1, take the Given series:
consecutive three digit odd numbers up to the 8LN$6PR&4TV%2XZ#JH1@FD3
number 117 and add the digits within the number ©CU5*PI7®FA9∞
till it becomes a single digit, then place the 8LN$6PR&4TV%2XZ#JH1@F∞9A
resultant number one by one in the same order F®7IP*5UC©3D
after every two letters from the left end.
ZX2VT4RP6NL8JH1FD3BU5OI7 3. Answer: A
EA9 Given series:
Step 3: After completing step 2, the first 12 8LN$6PR&4TV%2XZ#JH1@FD3
elements from the left end are reversed and the ©CU5*PI7®FA9∞
following symbols “$, &, %, #, @, ©, *, ®, ∞”, are 4+2+1+3= 10
placed (in the same order) in the series after
every three elements from the left end. 4. Answer: E
8LN$6PR&4TV%2XZ#JH1@FD3 Given series:
©BU5*OI7®EA9∞ 8LN$6PR&4TV%2XZ#JH1@FD3
Step 4: After completing step 3, the letters which ©CU5*PI7®FA9∞
are immediately preceded by a symbol and 8OM$6KI&4GE%2CA#JH1@FD3
immediately followed by a vowel are changed to ©CU5*PI7®FA9∞
the immediate next letter as per the alphabetical
series. 5. Answer: D
8LN$6PR&4TV%2XZ#JH1@FD3 Given series:
©CU5*PI7®FA9∞ 8LN$6PR&4TV%2XZ#JH1@FD3
1. Answer: C ©CU5*PI7®FA9∞
Given series: 8LN$6PR&4T%2#JH1@3©5*PI7
®9∞

Click Here For Bundle PDF Course | support@guidely.in Page 6 of 11


Bank Po Mains PDF Course 2024
Reasoning Day -25 (Eng)

Directions (6-10): 12. Answer: D


6. Answer: D 13. Answer: C (All the persons face the centre of
7. Answer: B the table except option c)
8. Answer: E 14. Answer: E
9. Answer: E 15. Answer: B
10. Answer: A Final arrangement:
Step 1: The product of the first digit of the two
boxes and the product of the second digit of the
two boxes.
Step 2: The difference between the product of
the two digits of the first and second boxes and
the difference between the product of the two
digits of the second and third boxes.
Step 3: The sum of the squares of the two digits
within the box.
Step 4: The difference between the two numbers
of both boxes. We have,
Step 5: The sum of the two digits of the number.  F sits third to the right of B, who is facing
the centre of the table.
 One person sits between F and K.
 Two persons sit between J and the one
who faces K.
From the above conditions, we have two
possibilities,

Directions (11-15):
11. Answer: A

Click Here For Bundle PDF Course | support@guidely.in Page 7 of 11


Bank Po Mains PDF Course 2024
Reasoning Day -25 (Eng)

Again we have,
 H sits opposite to J.
 L sits second to the right of the one who Again we have,
faces J.  I sits to the immediate left of E.
 The number of persons sitting between L  The number of persons sitting between E
and F is two less than the number of and C is two more than the number of
persons sitting between J and O, who persons sitting between B and A.
does not face K.  M does not face the centre of the table.
 C is an immediate neighbour of both O We cannot place G in case 1. Hence it is
and P. eliminated. Thus, case 2 gives the final
 N sits second to the left of the one who arrangement.
faces P.

Click Here For Bundle PDF Course | support@guidely.in Page 8 of 11


Bank Po Mains PDF Course 2024
Reasoning Day -25 (Eng)

We have,
 F delivered the food three floors above I,
where neither of them uses the same lift
nor uses lift II.
 F did not deliver above floor number 6.
 Both N and H deliver below I.
 J does not use lift II and delivered two
floors above H, where both of them use

Directions (16-20): the same lift.

16. Answer: D After applying the above conditions, there are

17. Answer: C two possibilities.

18. Answer: E
19. Answer: B
20. Answer: A (All the persons use lift I except
option a)
Final arrangement:

Click Here For Bundle PDF Course | support@guidely.in Page 9 of 11


Bank Po Mains PDF Course 2024
Reasoning Day -25 (Eng)

 Less than three floors are between the


floors on which C and G delivered the
food.
 L delivered immediately above the floor
on which G delivered the food. G uses lift
II.

Again, we have
 J and D delivered on the same floor.
 The number of persons delivered the food
below D is three more than the number of
persons delivered the food above C, who
did not use the same lift as N and J.
Again, we have

Click Here For Bundle PDF Course | support@guidely.in Page 10 of 11


Bank Po Mains PDF Course 2024
Reasoning Day -25 (Eng)

 More than two floors are between the


floors on which L and K delivered the
food.
 The number of floors between the floors
on which K and I delivered the food is one
less than the number of floors between
the floors on which F and M delivered the
food.
 E delivers below A but above B.
 E does not use lift III.
After applying the above conditions, case 1 gets
eliminated because E does not use lift III, hence
case 2 shows the final arrangement.

Click Here For Bundle PDF Course | support@guidely.in Page 11 of 11


Bank Po Mains PDF Course 2024
Quantitative Aptitude Day -25 (Eng)

Quantitative Aptitude

Directions (1-5): Study the following information carefully and answer the questions given below.
The given below bar graph shows the number of mobile phones manufactured by five different
companies [A, B, C, D and E] in the month of June and also given the number of mobile phones
manufactured in the month of July is how much more or less then the number of mobile phones
manufactured by the same companies in June.

The table given below shows the ratio of 4G to 5G mobile manufactured by the company in each month
out of total number of mobile phones
manufactured by the company.

1) Out of the total number of 4G mobile phones mobile phones has 64GB memory and out of the
manufactured by company A in the month June, total number of 4G mobile phones manufactured
40% has 128GB memory and the remaining by company A in the month July, 60% has

Click Here For Bundle PDF Course | support@guidely.in Page 1 of 12


Bank Po Mains PDF Course 2024
Quantitative Aptitude Day -25 (Eng)

128GB memory and the remaining mobile B.450


phones has 64GB memory. Find the total C.460
number of 64GB memory 4G mobile phones D.440
manufactured by company A in the months of E.None of these
June and July together.
A.920 4) Manufacturing cost of each 4G phone of
B.940 company C and D is Rs.4800 and Rs.6400
C.930 respectively and manufacturing cost of each 5G
D.960 phone of company C and D is Rs.5800 and
E.990 Rs.7000 respectively in both the months. Find
the difference (in thousand) of total
2) Ratio of snapdragon and Exynos processor in manufacturing cost in both months of company C
the 4G manufacturing phone of company B in the and D?
month of June is 3:2 respectively and Ratio of A.10100
snapdragon and Exynos processor in the 4G B.14200
manufacturing phone of company B in the month C.15200
of July is 1:4 respectively. Find the difference D.13500
between the total number of snapdragon E.None of these
processor 4G phone and total number of exynos
processor 4G phone manufactured by company 5) Number of defective mobile manufactured in
B in both months together?(Consider only month of June and July in company E is 20%
snapdragon and exynos processor is and 10% of total mobile manufactured in the
manufactured) company respectively and Number of defective
A.20 mobile manufactured in month of June and July
B.30 in company F is 10% and 10% of total mobile
C.40 manufactured in the company respectively. Find
D.0 the total defective mobile manufactured by
E.10 company E and F?
A.870
3) Find the difference between the total 5G B.820
phone manufactured in the month of June by all C.840
companies together and the total 5G phone D.850
manufactured in the month of July? E.None of these
A.420

Click Here For Bundle PDF Course | support@guidely.in Page 2 of 12


Bank Po Mains PDF Course 2024
Quantitative Aptitude Day -25 (Eng)

6) A 720ml solution of sugar and water is E.None of these


prepared in the ratio 7:2 respectively. Another
800ml solution of sugar and water in the ratio of 8) Certain distance is covered by a boat in
1:7, the two solutions are mixed to form a new downstream in 9 hours and in upstream in 11
solution. Now 20% solution is taken out from the hours. Find the percentage of distance covered
new solution. Then some amount of sugar is the boat in 27 hours rowing downstream over the
added which equals to 20% of the sugar content distance coveredin55 hours while rowing
in the first solution initially and 32ml water is upstream?
added. Water in the final mixture is how much A.45%
more or less than the sugar content? B.60%
A.80 more C.50%
B.50 more D.40%
C.20 less E.None of these
D.60 more
E.None of these 9) Half of the sum of the ages of Rohit, his wife
and his daughter after 6 years will be 41 years.
7) Three friends A, B and C invested in a firm After 8 years, the age of his daughter will be half
6:9:8, respectively. One year later, A increased the present age of his wife. Rohit’s age after 18
his investment by 25%. After 2 years, years will be equal to his wife’s age 10 years
investments of B and C become twice and 1.5 later. What will be his daughter’s age after 3
times of their initial investments respectively. years?
Profit of Rs. 7650 is distributed among them after A.11 years
3 years. Find the profit amount received by B? B.13 years
A.Rs. 3245 C.21 years
B.Rs. 3540 D.31 years
C.Rs. 3240 E.None of these
D.Rs. 4240

Directions (10-13): Study the following data carefully and answer the given questions:
Data given below is related to the number of boys and girls studying in under-graduation (UG) and post-
graduation (PG) in 5 different colleges A, B, C, D and E.
Table given below shows the following information:

Click Here For Bundle PDF Course | support@guidely.in Page 3 of 12


Bank Po Mains PDF Course 2024
Quantitative Aptitude Day -25 (Eng)

Note:
1: Total number of boys studying UG in all the 5 colleges together = 800
2: Total number of boys studying PG in all the 5 colleges together = 500
3: Some data in the table are missing, which needs to be calculated in the questions.
10) In college B, if the number of girls studying E.112
PG is 56 less than those in
UG, then find the average of number of boys 12) If the number of girls studying UG in college
studying PG in colleges A, B and E? E is equal to the number of boys studying UG in
A.100 college B, and ratio of boys to girls studying UG
B.80 in college E is 5: 6, then find the difference
C.75 between the total number of boys studying UG
D.120 and PG together in college E and the total
E.90 number of girls studying UG and PG together in
college E?
11) If the number of boys studying UG in college A.2
C is 168 less than those studying UG in college B.8
D and number of girls studying UG in college C C.4
is 16(2/3)% more than those studying UG in D.10
college A, then find the average number of boys E.6
and girls studying UG in college A?
A.102 13) If the average of number of girls studying PG
B.110 in colleges A and C is 115, then find the ratio of
C.108 the total number of students studying UG in
D.104 college C to those studying PG in college C?

Click Here For Bundle PDF Course | support@guidely.in Page 4 of 12


Bank Po Mains PDF Course 2024
Quantitative Aptitude Day -25 (Eng)

A.24: 25 C.2834
B.4: 5 D.2323
C.16: 17 E.2258
D.8: 9
E.14: 15 15) In Village C, the number of people in the age
of below 50 years is 80 more than the number of
Directions (14-17): Study the following people in the age of above 50 years. What is the
information carefully and answer the given percentage of the number of people in the age of
questions. above 50 years in total population of Village C?
There are four different Villages – A,B,C and D. A.38.38%
The following facts are known about number B.54.54%
male & female population and number of people C.45.45%
below & above the age of 50 years. In Village B, D.64.56%
the number of male population is 100 more than E.68.87%
female population. Male population of Village B is
20% more than that of in Village A and female 16) Find the difference between the female
population of Village B is 20% less than that of in population of Villages A & C together and the
Village A. Total population in Village A is 1000. In male population of Villages B & D together?
Village C, Male population is 50 less than female A.70
population in Village A and female population is B.90
70 less than male population of Village A. In C.80
Village D, male population is 4% less than that of D.75
in Village C and female population is 10% more E.65
than male population of Village C. The ratio of
number of people in the age of below 50 years 17) Total number of people in the age of below
and above 50 years in Village A and D is same 50 years in villages A and D together is how
and its ratio is 3:2. much more than total number of people in the
14) If the ratio between the number of people in age of above 50 years in Villages A&D together?
the age of below 50 years and above 50 years in A.406
Villages B&C is 4:3 & 6:5 respectively. Find the B.687
total number of people in the age of below 50 C.577
years for all given Villages together? D.605
A.3232 E.543
B.3032

Click Here For Bundle PDF Course | support@guidely.in Page 5 of 12


Bank Po Mains PDF Course 2024
Quantitative Aptitude Day -25 (Eng)

18) In the question two equations numbered I One of the roots of equations I and II are
and II are given. You have to solve both the common while value of A is a positive number.
equations and mark the answer. A.x < y
I: 2x2 – 3x – A = 0 B.x ≤ y
II: y2 – 20y + B = 0 C.x ≥ y
One root of equation I is the largest single digit D.x = y or relationship cannot be determined.
prime number and ratio of A to B is 7: 9. E.x > y
A.x < y
B.x ≤ y 20) In the question two equations numbered I
C.x ≥ y and II are given. You have to solve both the
D.x = y or relationship cannot be determined. equations and mark the answer.
E.x > y I: 2x – 525/x = 29
II: y2/ (36 – y) = 6
19) In the question two equations numbered I A.x < y
and II are given. You have to solve both the B.x ≤ y
equations and mark the answer. C.x ≥ y
I: 2x2 + 9x – 18 = 0 D.x = y or relationship cannot be determined.
II: 2 (y2 + 9) = Ay E.x > y
Click Here to Get the Detailed Video Solution for the above given Questions
Or Scan the QR Code to Get the Detailed Video Solutions

Answer Key with Explanation


Directions (1-5): Total number of mobile phones manufactured by
Total number of mobile phones manufactured by company A in the month of July is 1200 + 800 =
company A in the month of June is 1200. 2000.

Click Here For Bundle PDF Course | support@guidely.in Page 6 of 12


Bank Po Mains PDF Course 2024
Quantitative Aptitude Day -25 (Eng)

Number of 4G mobile phones manufactured by 3) Answer: B


company A in the month of June is 1200 * 5/8 = Required difference = [(450 + 400 + 1000 + 800
750 + 800 + 600) - (800 + 300 + 400 + 500 + 1000 +
Number of 5G mobile phones manufactured by 600)] = 450
company A in the month of June is 1200 - 750 =
450 4) Answer: A
Number of 4G mobile phones manufactured by Total cost of company C = 900 * 4800 + 1400 *
company A in the month of July is 2000 * 3/5 = 5800 = 12440000
1200 Total cost of company D = 2100 * 6400 + 1300 *
Number of 5G mobile phones manufactured by 7000 = 22540000
company A in the month of July is 2000 - 1200 = So, difference = 22540000 – 12440000 =
800 10100000 = 10100 thousands
Similarly, we can calculate the others value also.
5) Answer: A
Total defective mobile
= 1800 * 20/100 + 2500 * 10/100 + 1500 *
10/100 + 1100 * 10/100
= 870

6) Answer: A
According to question,
Total quantity of solution I = 720ml
Quantity of sugar: water in the mixture = 7:2
Sugar content in solution I = 560ml
1) Answer: C
Water content in solution I = 160ml
Total number of 64GB memory 4G phone
Total quantity of solution II = 800ml
manufactured by company A in the month June
Quantity of sugar: water in the mixture = 1:7
and July together = 750 * 60/100 + 1200 *
Sugar content in solution II = 100ml
40/100 = 930
Water content in solution II = 700ml
After combining the solutions,
2) Answer: D
Total sugar content = 660ml
Required difference = [600 * 3/5 + 200 * 1/5] -
Total water content = 860ml
[600 * 2/5 + 200 * 4/5] = 0
20% solution is taken out

Click Here For Bundle PDF Course | support@guidely.in Page 7 of 12


Bank Po Mains PDF Course 2024
Quantitative Aptitude Day -25 (Eng)

Sugar added = 20% x sugar content in the first Certain distance is covered by a boat in
solution initially downstream in 9 hours and in upstream in 11
So, 112ml sugar is added hours
32ml water is added Downstream speed of boat = D/9 kmph
Thus, final sugar content = 660 – 132 + 112 = Upstream speed of boat = D/11 kmph
640ml Distance covered in 27 hours rowing
Final water content = 860 – 172 + 32 = 720ml downstream = D/9 x 27 = 3D km
Required difference = 720 - 640= 80 more Distance covered in 55 hours rowing upstream =
Hence, answer is option A D/11 x 55 = 5D km
Percentage of distance covered by the boat in
7) Answer: C 27 hours rowing downstream over the distance
According to question, covered in55 hours rowing upstream = 3D/5D x
Let the initial amount invested by A, B and C = 100 = 60%
6x:9x:8x Hence, answer is option B
One year later, A increased his investment by
25% 9) Answer: A
After 2 years, investment of B becomes twice Let the present age of Rohit, daughter and wife =
After 2 years, investment of C becomes 1.5 R, D and W years
times According to question,
Total profit after 3 years = Rs. 7650 After 6 years, [1/2 x (Rohit + wife + daughter +
So, total time period= 3 years 18) = 41 years]
Ratio of profit shares = [(6x * 1) + (125% * 6x * 1/2 x (R + W + D + 18) = 41
2)]: [(9x * 2) + (2 * 9x * 1)]: [(8x * 2) + (1.5x * 8 * R + W + D = 82 – 18
1)] = 21:36:28 R + W + D = 64 ……………….. (1)
Profit amount received by B = Rs. 36/85 x 7650 After 8 years the age of his daughter = 1/2 x
= Rs. 3240 present age of his wife
Hence, answer is option C D + 8 = 1/2 x W
W – 2D = 16 ……………….. (2)
8) Answer: B Rohit’s age after 18 years =Rohit’s wife’s age 10
Let the distance covered = ‘D’ km years later
Let the speed of boat = ‘B’ kmph R + 18 = W + 10
Let the speed of stream = ‘S’ kmph W – R = 8 ……………….. (3)
According to question, From three equations we get,

Click Here For Bundle PDF Course | support@guidely.in Page 8 of 12


Bank Po Mains PDF Course 2024
Quantitative Aptitude Day -25 (Eng)

W = 32 years 12) Answer: A


R = 24 years Number of girls studying UG in college E =
D = 8 years Number of boys in UG in college B = 24% of 800
Rohit’s daughter age after 3 years = 11 years = 192
Hence, answer is option A Number of boys studying UG in college E = 192
* (5/6) = 160
10) Answer: A Number of boys studying PG in college E = 30%
Number of boys studying UG in college B = 24% of 500 = 150
of 800 = 192 Number of girls studying PG in college E = (2 *
So, number of girls studying UG in college B = (2 135) – 150 = 120
* 168) – 192 = 144 Required difference = (192 + 120) – (160 + 150)
Number of girls studying PG in college B = 144 – =2
56 = 88
And number of boys studying PG in college B = 13) Answer: B
(2 * 74) – 88 = 60 Number of boys studying PG in college A = 18%
Number of boys studying PG in college A = 18% of 500 = 90
of 500 = 90 Number of girls studying PG in college A = (2 *
Number of boys studying PG in college E = 30% 105) – 90 = 120
of 500 = 150 Number of girls studying PG in college C = (2 *
Required average = (60 + 90 + 150)/3 = 100 115) – 120 = 110
Number of boys studying PG in college C = 26%
11) Answer: C of 500 = 130
Number of boys studying UG in college D = 31% Total number of students studying PG in college
of 800 = 248 C = 130 + 110 = 240
So, number of boys studying UG in college C = Since, average of boys and girls studying UG in
248 – 168 = 80 college C = 96
Number of girls studying UG in college C = (2 * So, total number of students studying UG in
96) – 80 = 112 college C = 2 * 96 = 192
And number of girls studying UG in college A = Required ratio = 192: 240 = 4: 5
112 * (300/350) = 96
Since, number of boys studying UG in college A Directions (14-17):
= 15% of 800 = 120 Let the male population of village A is ‘x’ and
So, required average = (120 + 96)/2 = 108 female population is ‘y’

Click Here For Bundle PDF Course | support@guidely.in Page 9 of 12


Bank Po Mains PDF Course 2024
Quantitative Aptitude Day -25 (Eng)

x + y = 1000 -------(1) The total number of people in the age of below


In village B, 50 years for all given Villages= 600 + 618 + 560
120*x/100 – 80*y/100 = 100 ---------(2) + 480 = 2258
By solving (1) and (2),
x = 450 15) Answer: C
y = 550 Let number of people in the age of above 50
In village B, years is ‘x’
Male population = 540 The number of people in the age of below 50
Female population = 440 years = 80 + x
In Village C, x + x + 80 = 880
Male population = 550 – 50 = 500 x = 400
Female population = 450 – 70 = 380 Required percentage = 400*100 / 880 = 45.45%
In Village D,
Male population = 500*96/100 = 480 16) Answer: B
Female population = 500*110/100 = 550 Female population of Villages A& C = 550+380 =
Number of people in the age of below 50 years 930
in Village A = 1000*3/5 = 600 Male population of Villages B&D = 540+480 =
Number of people in the age of above 50 years 1020
in Village A = 1000*2/5 = 400 Required difference = 1020 – 930 = 90
Number of people in the age of below 50 years
in Village D = 1030*3/5 = 618 17) Answer: A
Number of people in the age of above 50 years Total number of people in the age of below 50
in Village D = 1030*2/5 = 412 year in Villages A and D = 600 + 618 = 1218
Total number of people in the age of above 50
years in Villages A and D = 400 + 412 = 812
Required difference = 1218 – 812 = 406

14) Answer: E 18) Answer: A


The number of people in the age of below 50 I: 2x2 – 3x – A = 0
years in B = 980*4/7 = 560 We know that: Largest single digit prime number
The number of people in the age of below 50 =7
years in C = 880*6/11 = 480 2 * 72 – 3 * 7 – A = 0
98 – 21 = A

Click Here For Bundle PDF Course | support@guidely.in Page 10 of 12


Bank Po Mains PDF Course 2024
Quantitative Aptitude Day -25 (Eng)

A = 77 2 * [(-6)2 + 9] = A * (-6)
2x2 – 3x – 77 = 0 90 = - 6A
2x2 – 14x + 11x – 77 = 0 A = -15 (Invalid)
2x * (x – 7) + 11 * (x – 7) = 0 Hence, 2 (y2 + 9) = 15y
(x – 7) * (2x + 11) = 0 2y2 + 18 – 15y = 0
x = 7 and -11/2 2y2 – 12y – 3y + 18 = 0
II: y2 – 20y + B = 0 2y * (y – 6) – 3 * (y – 6) = 0
A: B = 7: 9 (y – 6) * (2y – 3) = 0
77: B = 7: 9 y = 6 and 3/2
B = 99 When x = 3/2, y = 6; x < y
y2 – 20y + 99 = 0 When x = 3/2, y = 3/2; x = y
y2 – 11y – 9y + 99 = 0 When x = -6, y = 6; x < y
y (y – 11) – 9 (y – 11) = 0 When x = -6, y = 3/2; x < y
(y – 11) (y – 9) = 0 Hence, x ≤ y.
y = 11 and 9
When x = 7, y = 11; x < y 20) Answer: D
When x = 7, y = 9; x < y I: 2x – 525/x = 29
When x = -11/2, y = 11; x < y 2x2 – 525 = 29x
When x = -11/2, y = 9; x < y 2x2 – 29x – 525 = 0
Hence, x < y. 2x2 – 50x + 21x – 525 = 0
2x * (x – 25) + 21 * (x – 25) = 0
19) Answer: B (x – 25) * (2x + 21) = 0
I: 2x2 + 9x – 18 = 0 x = 25 and -21/2
2x2 + 12x – 3x – 18 = 0 II: y2/ (36 – y) = 6
2x (x + 6) – 3 (x + 6) = 0 y2 = 216 – 6y
(x + 6) (2x – 3) = 0 y2 + 6y – 216 = 0
x = 3/2 and -6 y2 – 12y + 18y – 216 = 0
II: 2 (y2 + 9) = Ay y (y – 12) + 18 (y – 12) = 0
Case 1: When ‘3/2’ is the common root. (y – 12) (y + 18) = 0
2 [(3/2)2 + 9] = A * (3/2) y = 12 and -18
45/2 = 3A/2 When x = 25, y = 12; x > y
A = 15 When x = 25, y = -18; x > y
Case 2: When ‘-6’ is the common root. When x = -21/2, y = 12; x < y

Click Here For Bundle PDF Course | support@guidely.in Page 11 of 12


Bank Po Mains PDF Course 2024
Quantitative Aptitude Day -25 (Eng)

When x = -21/2, y = -18; x > y Hence, relationship between ‘x’ and ‘y’ cannot be
determined.

Click Here For Bundle PDF Course | support@guidely.in Page 12 of 12


Bank Po Mains PDF Course 2024
English Day - 25

English Language
Directions (1-7): Read the following passage and standard of living is measured by real income.
answer the questions given below. When actual incomes rise, the standard of living
Giving people the resources and ability to learn rises with them, and vice versa.
about how things are going in our economy is an In actuality, prices fluctuate at various rates.
extremely important thing. One issue that is Some, such as the prices of traded commodities,
especially prevalent in today’s economy is fluctuate on a daily basis; others, such as
inflation and how it affects the overall well-being contract-based pay, require longer to adjust. In
of the people in our country and around the an inflationary environment, unevenly growing
globe. This economic issue affects every prices lower some customers’ purchasing power,
member of our society, and it is especially and this erosion of real income is the single most
important that our citizens keep themselves well- significant cost of inflation.
educated on this topic. In a sound economy, Inflation can also affect the purchasing power of
costs will in general increase – this is known as fixed-interest rate receivers and payers over
inflation. While you probably won’t care for that time. Take, for example, retirees who are
as a buyer, price growth rising moderately is an guaranteed a 5% annual rise in their pension.
indication of a solid, developing economy. When inflation exceeds 5%, a retiree’s
The U.S. Federal Reserve currently considers a purchasing power decreases. A borrower paying
2% inflation rate to act as the best growth rate for a 5% fixed-rate mortgage, on the other hand,
the economy, which is about its present level. Be would gain from 5% inflation because the real
that as it may, a few financial analysts, including interest rate (fixed rate of mortgage minus
those at the Federal Reserve, stress the inflation rate) would be zero; servicing this debt
economy is debilitating, which would make would be even easier if inflation were greater, as
inflation dip under its objective, which is long as the borrower’s income kept up with
something that needs to be avoided at all costs. inflation. Of course, the lender’s real income
The most recent information, which was released drops as a result. When nominal interest rates
to the public on June 12, implied that this might aren’t adjusted for inflation, some people gain
be occurring. and others lose purchasing power.
Households are worse off if their nominal A moderate measure of inflation is commonly
income, which they receive in current money, viewed as an indication of a solid economy, in
does not rise at the same rate as prices because light of the fact that as the economy develops,
they can afford to buy fewer things. To put it the demand for goods and services grows. This
another way, their purchasing power or real growth of demand pushes costs somewhat
(inflation-adjusted) income decreases. The higher as providers attempt to make a greater

Click Here For Bundle PDF Course | support@guidely.in Page 1 of 15


Bank Po Mains PDF Course 2024
English Day - 25

number of what buyers and organizations need e. Only option b


to purchase. When inflation becomes excessively
low, or vice versa, a horrendous cycle can wreak 2. Which of the following topics best suit the
havoc upon our economy. High inflation has a context of the passage?
wide scope of negative ramifications for a. Overcoming the negative effects inflation using
economies. At the point when work the right economic strategies.
compensation is unable to stay in line with the b. Moderate inflation – An indication of a
inflation of retail costs, the power of purchasing declining economy
of the pay checks that workers receive quickly c. The varying effects of inflation
diminishes. This causes a huge problem for d. Exploring household economic issues
households that receive low income, because e. A guide to improving the purchasing power of
any increase in the price of any goods or a common man
services can have very serious negative effects.
Laborers’ requests for growth in their wages can 3. According to the passage, why do
prompt an expansion in costs of labor, bringing organizations suffer when the inflation rate of a
about lower benefits for organizations. These country increases?
impacts of expansion can make a high level of a. When inflation increases, the cost of raw
vulnerability in an economy, prompting materials increases which eventually results in
diminished venture from those who aim to start increased expenditures for the organizations.
their own businesses. b. When inflation increases, the government
1. Which of the following statement/statements tends to increase the taxes imposed on
is/are true according to the passage given production and distribution of products and this
above? could deplete the capital funds of the
a. When the rate of inflation falls below the organization.
recommended growth rate, the economy of a c. When inflation increases, customers tend to
country can begin to lose strength and vitality. prevent buying products due to a surge in the
b. Households can suffer when their income cost of goods and this could lead to losses for
does not change in sync with the moving inflation organizations that produce commodities for the
rates. public.
c. The purchasing power of every person d. When inflation increases, the laborers tend to
increases when the inflation rates increase even request wage compensations to make up for the
when the income of every person remains the increase in the price of basic commodities and
same. this could bring in increased expenses for the
d. Only options a and b organization.

Click Here For Bundle PDF Course | support@guidely.in Page 2 of 15


Bank Po Mains PDF Course 2024
English Day - 25

e. None of the above reasons is right. Directions (8-12): Given below are a set of
sentences in which only one of them is
4. According to the passage, what grammatically and contextually correct. Identity
section/sections of the society lose/loses the sentence that is error-free.
purchasing power after an increase in the rates 8.
of inflation? a. Both Rita and Sita are not considering taking
a. Borrowers who pay fixed rate mortgage the IAS exams this month since they were held
b. People with contract-based income options up at work for the last few months and eventually
c. Individuals who receive retirement benefits had no time for the exam preparations.
d. Only option c b. Arjun has been planning to marry whoever his
e. Options b and c mother chooses since he dislikes going against
his mother's words.
5. What is the antonym of the word ‘debilitating’? c. If Reena takes up the upcoming exams
a. invigorating seriously, she could pass the exams with flying
b. exhausting colors.
c. draining d. Doctors advise us to drink at least 8 glassfuls
d. enfeebling of water every day to keep ourselves hydrated
e. None of the above during the summers.
e. Deepthi was confused about how some of her
6. What is synonym of the word ‘horrendous’? money were missing, but later she realized that
a. calming she had lent 5000 rupees to her sister this
b. pleasant month.
c. non-threatening
d. formidable 9.
e. None of the above a. Meena offered to pay for the watch she had
chosen using two five hundred rupees notes, but
7. What is the right antonym for the word the shopkeeper wanted her to transfer the
‘ramifications’? amount through mobile banking apps since he
a. outcome had no change.
b. consequence b. My friend is okay with me borrowing her car for
c. implication a day or two since I am yet to purchase my
d. cause vehicle.
e. effect

Click Here For Bundle PDF Course | support@guidely.in Page 3 of 15


Bank Po Mains PDF Course 2024
English Day - 25

c. Rita exclaimed that she loved watching movies b. Eagerly waiting to get a much-needed break
more than me, but I had been an ardent movie from her everyday routine, Roopa was
buff since childhood. heartbroken heavily when her car broke down
d. Neither the students nor the teacher is happy minutes before she started to drive to the mall.
about how the school canceled the science c. She called me several times in the last four
exhibition at the last minute since the class weeks until I had agreed to set an appointment.
collectively put in a lot of hard work to create the d. Every student was assigned a part in the play
models. that was to be staged on the school's annual day
e. The new building constructed at the heart of to ensure that all students got equal opportunities
the city is three times taller than the LIC building to showcase their acting talents.
at Chennai's Mount Road. e. All sentences are incorrect

10. 12.
a. Teju had no doubt whether she would pass the a. He could have passed the English literature
upcoming exams, but I doubted that I would clear test and secured a job offer in the United States
mine. if his father had been an English professor.
b. Renu said that she performed yoga every day b. Many a student from the Science department
to maintain her body fitness and improve her is contesting in the elections conducted to select
mental health. the members of the Student Union.
c. Meera had been planning to meet up with c. Not only the newly recruited interns speak
Rahul since the time she had asked him to pair French, but they are also fluent in English.
up with her for the upcoming project. d. While addressing the gathering, the minister
d. I wish I can meet Reena today for old time's stated that he believed that the youth of his state
sake, but I am uncertain about my schedule after would become powerful and effective with the
today's classes. required training and exposure.
e. The road to the airport was blocked due to the e. None of the sentences are incorrect.
arrival of the President for the inauguration of the
new world-class multi-specialty hospital in Directions (13-17): The following sentence has
Chennai. one part that is highlighted in bold. This
highlighted part may or may not be
11. grammatically and contextually correct. Choose
a. Tanya was living in Chennai before she the best alternatives among the four options
received an offer to study and work in one of the given below such that it rightly replaces the
most prestigious colleges in the United Kingdom. highlighted part grammatically and contextually.

Click Here For Bundle PDF Course | support@guidely.in Page 4 of 15


Bank Po Mains PDF Course 2024
English Day - 25

If the part given in bold is grammatically and 16. Karan does not deserve to take a claim on
contextually right, then choose the option ‘No the profits owned by his family business since he
replacement required’. hardly turned up at the office and engaged in
13. The police department has clamped down of work in the last few months.
piracy to prevent the illegal streaming of newly a. deserve to make a claim to the profits
released films on various online platforms. b. deserve to stake a claim to the profits
a. have clamped down of piracy to prevent the c. deserve to stake a claim on the profits
b. has clamped down on piracy to prevent the d. deserve to make a claim on the profits
c. has clamped down against piracy to prevent e. No changes required
the
d. has clamped down on piracy for the prevention 17. Tanya was wet behind the ears since she
e. No change required had just graduated but was excited about
venturing into new job roles in the company of
14. He, I, and you are collectively at fault for not her choice.
submitting the project within the specified a. waiting behind in years since she did
deadline since we lost track of what we should b. wet behind in years since she had
do midway through the project. c. waiting behind in years since she had
a. I, he, and you are collectively at fault d. wet beyond in ears since she had
b. He, you, and I are collectively at fault e. No changes required
c. I, you, and he are collectively at fault
d. He, you, and I are collectively in fault Directions (18-22): The following set of questions
e. No corrections required are not arranged in the right order. These
sentences when arranged in the right order gives
15. Rahul felt ashamed when he got his working right to meaningful passage. Identify the right
papers after being found guilty of leaking the pattern of arrangement and answer the
company's information to the competitors in questions that follow.
return for a ransom. A. Such kind of activism positively affect the
a. He got his walking papers after being society, as they attract the public’s attention to a
b. He had gotten his walking papers after being topical issue.
c. He got his working papers after being B. Women account for one of the groups most
d. He got his working papers after been affected by the disparities in the legal system.
e. No changes required C. Subsequently, various standpoints are
expressed through different means of

Click Here For Bundle PDF Course | support@guidely.in Page 5 of 15


Bank Po Mains PDF Course 2024
English Day - 25

communication, calling for an equal approach to e. None of the above


justice for both sexes.
D. Therefore, this sphere has been an area of 20. Which sentence comes second after
intense interest for feminist researchers and arranging the set of sentences correctly to form a
activists. meaningful passage?
F. Accordingly, it can be concluded that the a. A
country is currently experiencing the b. B
phenomenon of mass female incarceration, c. C
which _________ the unfair side of the legal d. D
system. e. E
G. Reports suggest that the incarceration rate of
women in a particular country has grown at a 21. Which sentence comes fourth after arranging
quicker pace than a similar number of men. the set of sentences correctly to form a
18. Which of the following words is the best meaningful passage?
antonym for the word highlighted in the passage a. A
– disparities? b. B
a. inequality c. C
b. distinction d. D
c. indifference e. E
d. imparity
e. None of the above 22. Which sentence comes sixth after arranging
the set of sentences correctly to form a
19. What word best suits the blank in the given meaningful passage?
passage? a. A
a. matches b. C
b. reflects c. D
c. resonates d. E
d. reproduces e. F

Click Here For Bundle PDF Course | support@guidely.in Page 6 of 15


Bank Po Mains PDF Course 2024
English Day - 25

Click Here to Get the Detailed Video Solution for the above given Questions
Or Scan the QR Code to Get the Detailed Video Solutions

Answer Key with Explanation


1. Answer: E some people (not every person) increases when
The context of option A is clearly stated in the inflation rates increase. Thus, option c is
second paragraph of that passage. But the incorrect.
financial analysts state that the inflation rate
could fall below the recommended growth rate 2. Answer: C
when economy tends to become weak and lose Option a is incorrect. Because the passage does
vitality. But option ‘a’ conveys describes the said not focus on disclosing or proposing economic
course of actions in the other way. Option a says strategies to overcome the negative effects of
that the economy of the country becomes weak inflation.
when the inflation rate dips below the Option b is incorrect. Because according to the
recommended growth rate which does not match passage, moderate inflation is described as an
the message conveyed through the passage. indication of a developing economy and not a
Thus, option a is incorrect. falling economy.
Option b talks about how households can suffer Option c is correct. Because the passage briefly
when their income does not change with the describes about how inflation can positively and
moving inflation rates. This statement is clearly negatively affect the people of a country.
stated in the third paragraph of the passage. Option d is incorrect. Because the passage does
Option c is incorrect. Since according to the fifth not solely focus on household economic issues
paragraph of the passage, people can gain and since topic is just one among the many
lose their purchasing power when the inflation discussed in the passage. For instance, the last
rates rise. But option c normalizes this statement paragraph of the passage also talks about the
to every person. The purchasing power of only issues organizations face as a result of laborers

Click Here For Bundle PDF Course | support@guidely.in Page 7 of 15


Bank Po Mains PDF Course 2024
English Day - 25

requesting wage compensations to tackle the get increased only after periodic intervals. Thus,
effects of inflation. the retirees would face financial challenges until
Option e is incorrect. Because the passage does the pension amounts gets renewed. Thus, option
not focus on improving the purchasing power of c is correct.
a common man but only explains how the Option b speaks about individuals with contract-
purchasing power of a common man gets affects based income options. Retirees are people with
when inflation rates change drastically. Also, the contract-based income options since they
passage does not explicitly state alternatives to receive contracted income options that increase
improve the purchasing power of a common only once in every year. Thus, option b is also
man. correct.
Thus, options b and c are correct.
3. Answer: D
According to the last paragraph of the passage, 5. Answer: A
it is clear that when the price of the commodities The word debilitating is an adjective which
increases, laborers tend to request wage means to impair the strength and vitality.
compensations and this could expand the costs Invigorating (adjective) – to impart strength and
of labor of the organization. Thus, option d is vitality
correct. Also, the passage does not explicitly talk Exhausting (adjective) – to have a debilitating
about how organizations get affected as a result effect
of increase in taxes, increase in the price of raw Draining (adjective) – to have a debilitating effect
materials, and less sales. Thus, options a, b, and Enfeebling (adjective) – to weaken
c are incorrect. The options b, c, and d are synonymous with
each other. The right answer is option a.
4. Answer: E
In the fifth paragraph of the passage, it is clearly 6. Answer: D
stated the borrowers who pay fixed rate The word ‘horrendous’ (adjective) means to
mortgage gain purchasing power when there is cause fear, dread, or terror.
an increase in the rate of inflation since in these Calming (adjective) – to be in a state of peace.
cases, their actual interest rates become zero. Pleasant (adjective) – calming and peaceful
Thus, option a is incorrect. Non-threatening (adjective) – not making feeling
In the same paragraph, it is stated that retirees nervous
lose purchasing power after an increase in the Formidable (adjective) – inspiring fear
rate of inflation because their pension amounts Thus, the right answer is option d.

Click Here For Bundle PDF Course | support@guidely.in Page 8 of 15


Bank Po Mains PDF Course 2024
English Day - 25

7. Answer: D ‘whomever’ is always used as an objective


The word ‘ramifications’ (noun) means the result pronoun. It is used as an object of a
of an event. verb/preposition in a sentence. Thus, whoever
The words outcome, consequence, implication, should be replaced with whomever in sentence
and effect are all nouns and they denote the B. The right sentence is:
result of an event. Thus, they are synonymous to Arjun has been planning to marry whomever his
the word ‘ramifications’. mother chooses since he dislikes going against
The word ‘cause’ (noun) indicates something his mother's words.
that encourages an event to occur. In simple The sentence C is a conditional sentence. The
contexts, the cause of an event is something that conditional part of the sentence is written in
happens before it and the outcome of an event simple present sentence. Thus, the main part of
happens after it. Thus, the word ‘cause’ is the the sentence should also be written in simple
right antonym for ramifications. present tense to follow uniformity. Thus, the use
of the word ‘could’ in the main part of the
8. Answer: D sentence is wrong since it is a simple past model
The sentence A uses the word ‘Both’ and ‘Not’ verb. The right choice of model verb (simple
together. The words ‘both’ and ‘not’ should not present) for this sentence is may/can. The right
be used together. In this case, it is advised to sentence is:
use ‘neither – nor’ with a singular or plural verb If Reena takes up the upcoming exams
according to the subject-verb agreement. The seriously, she can/may pass the exams with
right sentence is: flying colors.
Neither Rita nor Sita is considering taking the The sentence D is grammatically and
IAS exams this month since they were held up at contextually right.
work for the last few months and eventually had The sentence E uses the phrase ‘some of her
no time for the exam preparations. money’. The word money always takes singular
The sentence B has the word ‘whoever’. The form and thus, the verb that is used to describe
pronoun ‘whoever’ is always used as a money also takes singular form. Thus, the right
subjective pronoun. It is replaced as the subject sentence is:
of the verb in a sentence. But in this case, the Deepthi was confused about how some of her
pronoun is position to describe the object of the money was missing, but later she realized that
verb in the sentence, (i.e) the pronoun should be she had lent 5000 rupees to her sister this
positioned as an object of the verb (verb - month.
indicates who his mother chooses). The pronoun

Click Here For Bundle PDF Course | support@guidely.in Page 9 of 15


Bank Po Mains PDF Course 2024
English Day - 25

9. Answer: D the right first person subjective pronoun. The


The sentence A includes the phrase ‘two five right sentence is:
hundred rupees notes’ which is grammatically Rita exclaimed that she loved watching movies
incorrect. If the phrase is framed in the form – more than I, but I had been an ardent movie buff
‘number + unit + noun’ the unit mentioned here since childhood.
should take singular form. Here, the number is The sentence D is correct. Note that when there
‘five hundred’, unit is ‘rupees’ and noun is ‘notes. are two subjects separated by neither or nor,
The unit should take singular form. Thus, the then the verb should be inserted according to the
right sentence is: nearest subject like in the case of this sentence.
Meena offered to pay for the watch she had The sentence E speaks about comparisons.
chosen using two five hundred rupee notes, but When the word ‘times’ is used in the sentence, it
the shopkeeper wanted her to transfer the should not be followed by adjectives/nouns of
amount through mobile banking apps since he the comparative degree. Thus, ‘three times taller
had no change. than’ is grammatically incorrect. The comparison
The sentence B consists of a phrase ‘me is made using the phrase ‘as + positive degree
borrowing’ which is grammatically incorrect. If a adjective/verb + as’ to make it grammatically
pronoun is used before a gerund, the pronoun correct. The right sentence is:
should mandatorily take possessive form. Thus, The new building constructed at the heart of the
the word ‘me’ is an objective pronoun and hence city is three times as tall as the LIC building at
it is incorrect. The right word should be ‘my’ – Chennai's Mount Road.
possessive pronoun. The right sentence is:
My friend is okay with my borrowing her car for a 10. Answer: E
day or two since I am yet to purchase my The sentence A talks about exams. Sentences
vehicle. with the word ‘doubt/doubtful’ (unsure of
The sentence C compares the liking of two something) should be followed by the connectors
individuals. The comparison should be made ‘if/whether’ and the sentences with the word ‘no
between two similar entities, (i.e) here the doubt/not doubtful’ should be used with the
comparison is made between ‘she’ which a third connector ‘that’. Thus, the right sentence is:
person subjective pronoun and the speaker. Teju had no doubt that she would pass the
Thus, the speaker, begin the first person, should upcoming exams, but I doubted whether I would
be represented by a subjected pronoun as well. clear mine.
But ‘me’ is a first person objective pronoun. The sentence B speaks about the daily habit of
Thus, the word ‘me’ is incorrect. The word ‘I’ is Renu. The tense of a verb when describing a

Click Here For Bundle PDF Course | support@guidely.in Page 10 of 15


Bank Po Mains PDF Course 2024
English Day - 25

habitual event of a person does not change even I wish I could meet Reena today for old time's
in the case of reported speech. The tense of the sake, but I am uncertain about my schedule after
verb should be in simple present tense in indirect today's classes.
speech when describing about a habitual event The sentence E is grammatically and
of a person. Thus, ‘she performed yoga’ is wrong contextually correct.
and it must be changed to ‘she performs yoga’.
The right sentence is: 11. Answer: D
Renu said that she performs yoga every day to The sentence A speaks about two events. If
maintain her body fitness and improve her there are two events wherein one of the two
mental health. events was in progress in the past until the other
The sentence C consists of the word ‘since’. event occurred. In this case, the event that
Sentences with ‘since’ should be framed in such happened first should be written in past perfect
a way that the part of the sentence before the continuous tense. Thus, ‘Tanya was living in
word ‘since’ should be written perfect/perfect Chennai’ (lived-simple past) is incorrect. The
continuous tense, while the part of the sentence right use of verb should be in past perfect
after the word ‘since’ should be written in simple continuous tense ‘has been living’. The right
tense. In this sentence, the part of the sentence sentence is:
before ‘since’ is correctly framed with ‘had been Tanya had been living in Chennai before she
planning’ – past perfect continuous tense. But received an offer to study and work in one of the
the sentence after the word ‘since’ is not written most prestigious colleges in the United Kingdom.
in simple past. The correct form of the verb here In sentence B, the adverb ‘heavily’ is placed
should be ‘she asked’ and not ‘she had asked’. after the verb ‘heartbroken’. The adverb is
The right sentence is: normally placed before the verb to bring in
Meera had been planning to meet up with Rahul greater emphasis of the verb. Thus, the right
since the time she asked him to pair up with her sentence is:
for the upcoming project. Eagerly waiting to get a much-needed break
The sentence D includes the word ‘wish’. The from her everyday routine, Roopa was heavily
verb after the word ‘wish’ should always be heartbroken when her car broke down minutes
included in simple past form. Thus, ‘I wish I can’ before she started to drive to the mall.
is grammatically incorrect since the verb ‘can’ is In the sentence C, there are two actions. When
in simple present form. The right use of the verb there are two actions taking place, the one that
should be ‘could’. The right sentence is: had occurred first should be written in past
perfect tense and the action that follows that

Click Here For Bundle PDF Course | support@guidely.in Page 11 of 15


Bank Po Mains PDF Course 2024
English Day - 25

should be written in simple past tense. Thus, the students from the science department are’. The
use of verb ‘called’ is incorrect (action that right sentence is:
happened first) and it should be changed to past Many a student from the Science department is
perfect tense – ‘had called’. Also, for the action contesting in the elections conducted to select
that follow, the use of the verb ‘had agreed’ is the members of the Student Union.
incorrect and it should be replaced by ‘agreed’ The sentence D uses the phrases ‘not only – but
(simple past). Thus, the right sentence is: also’. This set of phrases makes use of the rule
She had called me several times in the last four of inversion. In this case, the sentence
weeks until I agreed to set an appointment. arrangement should follow this pattern : ‘not only
The sentence D is grammatically correct. + auxiliary verb +subject’. The right use of
phrase would be ‘Not only do the newly’. The
12. Answer: D right sentence is:
The sentence A talks about how the speaker Not only do the newly recruited interns speak
would have passed the English test if his father French, but they are also fluent in English.
was an English professor. But profession of his The sentence D is grammatically correct.
father is something that could not be changed. It
is impossible for his father to become an English 13. Answer: B
professor. Thus, the latter part of the sentence The sentence includes a phrasal verb that
talks about something unreal or impossible. should necessarily take the form ‘clamped down
Thus, the verb used here should be ‘were’ and on’. So the right use of preposition is ‘on’ and not
not ‘had been’. Thus, the right sentence is: ‘of’ or ‘against’. Thus, the options ‘a’ and ‘c’ are
He could have passed the English literature test incorrect. In option d, the phrase ‘for the
and secured a job offer in the United States if his prevention’ grammatically incorrect because the
father were an English professor. sentence after the highlighted part does not have
The sentence B starts with the word ‘many’ the preposition ‘of’ to rightly completely the
which should be immediately followed by a plural sentence. Thus, the right answer is option ‘b’.
noun in general cases. If the word ‘many’ is The right sentence is:
followed by ‘a’ then the noun that follows ‘Many The police department has clamped down on
a’ should be singular and the verb which follows piracy to prevent the illegal streaming of newly
the subject of the sentence should be plural. released films on various online platforms.
Thus, the right phrase should be ‘Many a student
from the science department is’ and not ‘Many a 14. Answer: C

Click Here For Bundle PDF Course | support@guidely.in Page 12 of 15


Bank Po Mains PDF Course 2024
English Day - 25

When the sentence speaks about the The sentence speaks about how a person
acceptance of guilt or grief, the order of the beings to claim the profits that he do not deserve
pronouns would be ‘first person, second person, in the first place since he had not worked for it.
and third person’. Thus, options a, b, and d are But the right use of preposition after claim in this
incorrect. The right order of pronouns is ‘I, you, context is ‘on’ and not ‘to’. The word claim
and he’. The right sentence is: follows the preposition ‘on’ when it speaks about
I, you, and he are collectively at fault for not something that the subject is not deserving of or
submitting the project within the specified have not rights on. Also, the right word that
deadline since we lost track of what we should should be used to denote the act of claiming
do midway through the project. properties or wealth is ‘stake’. Thus, the right
sentence is:
15. Answer: A Karan does not deserve to stake a claim on the
The sentence talks about how Rahul was fired profits owned by his family business since he
from his job after being found to be at fault for hardly turned up at the office and engaged in
selling company’s information. The sentence work in the last few months.
conveys this better with the use of the idiom ‘Got
your walking papers’ which means to ‘get fired 17. Answer: E
from the job’. Thus, the right use of word is The given sentence is grammatically and
‘walking’ and not ‘working’. Thus, options a and contextually right. The phrase or idiom ‘wet
d are incorrect. Also, the verb ‘had gotten’ is behind her ears’ means ‘young and lacking in
grammatically incorrect. Since the sentence is experience’.
written is simple past sentence but ‘had gotten’
is past participle. The use of ‘had gotten’ is 18. Answer: C
incorrect since these leads to non-uniformity Disparity is a noun which denotes inequality or
while considering about the tense of the difference in some respect.
sentence. Thus, option b is incorrect. Inequality (noun) – lack of equality
The right sentence is: Distinction (noun) – discrimination between
Rahul felt ashamed when he got his walking entities
papers after being found guilty of leaking the Imparity (noun) – lack of equality
company's information to the competitors in Thus, options a, b, and d are synonymous to the
return for a ransom. given word.
Indifference (noun) – the state of being
16. Answer: C unbiased, impartial, equal

Click Here For Bundle PDF Course | support@guidely.in Page 13 of 15


Bank Po Mains PDF Course 2024
English Day - 25

Thus, option c is the right answer. feminist researchers. Thus, sentence D should
come second in the arrangement. Since
19. Answer: B researchers have been brought into the picture
Matches (verb) – to coincide in characteristics they are bound to deliver viewpoints and
Reflects (verb) – to give evidence for a certain opinions on this issue. Thus, sentence C should
behavior or pattern come next since it contains the word
Resonates (verb) – to be similar to what ‘standpoints’ (noun) which means ‘viewpoints’.
someone believes of thinks Sentence C talks about the need for an equal
Reproduces (verb) – to replicate something approach to justice for both genders. An equal
Here, the sentence speaks about how a country approach to justice for both genders can bring
is experiencing mass female incarceration. The about a much required social equality. Thus, the
best choice of verb for this blank is ‘reflects’ sentence A should come next because it
since the mass female incarceration shows includes the phrase ‘such kinds of activism’ in
evidence for the unfairness in the legal system. the beginning wherein the word activism (noun)
Thus, the right sentence is: means a policy of actively campaigning to
Accordingly, it can be concluded that the country achieve a political or social goal. Now, there are
is currently experiencing the phenomenon of two sentences left, one of which speaks about,
mass female incarceration, which reflects the the incarceration rate of women has grown at a
unfair side of the legal system. quicker pace when compared to a similar
number of men and the other speaks about how
20. Answer: D something reflects the unfair side of the legal
The passage talks about the unfairness system. Also, the sentence F refers to the mass
observed against women. The sentence B incarceration of women that is introduced into
comes first because it introduces the topic to the the passage using the statistics of a report that is
readers. The sentence D talks about a certain mentioned in sentence G. Thus, sentence F
sphere sparks the interest of feminist follows sentence G.
researchers. The word sphere (noun) denotes a Thus, the right pattern of arrangement is:
particular aspect of an activity. The sentence A B-D-C-A-G-F
which talks about disparities in the legal system Thus, the second sentence after rearrangement
that work against women. Feminist researchers is D. The right answer is option d.
believe women should equal rights. Thus, the
disparities in the legal system that work against 21. Answer: A
women is that sphere that sparks the interest of

Click Here For Bundle PDF Course | support@guidely.in Page 14 of 15


Bank Po Mains PDF Course 2024
English Day - 25

The right pattern of arrangement is B-D-C-A-G- 22. Answer: E


F. Refer to the explanation given for question The right pattern of arrangement is B-D-C-A-G-
number 20 for a detailed solution for the F. Refer to the explanation given for question
rearrangement. Thus, the right answer is option number 20 for a detailed solution for the
a. rearrangement. Thus, the right answer is option
e.

Click Here For Bundle PDF Course | support@guidely.in Page 15 of 15

You might also like